Quick viewing(Text Mode)

Mcqs and Emqs in Surgery

Mcqs and Emqs in Surgery

1 The metabolic response to injury

Multiple choice questions ➜ Homeostasis B Every endocrine gland plays an equal 1. Which of the following statements part. about homeostasis are false? C They produce a model of several phases. A It is defined as a stable state of the D The phases occur over several days. normal body. E They help in the process of repair. B The central nervous system, heart, lungs, ➜ kidneys and spleen are the essential The recovery process organs that maintain homeostasis at a 4. With regard to the recovery process, normal level. identify the statements that are true. C Elective should cause little A All tissues are catabolic, resulting in repair disturbance to homeostasis. at an equal pace. D Emergency surgery should cause little B Catabolism results in muscle wasting. disturbance to homeostasis. C There is alteration in muscle protein E Return to normal homeostasis after breakdown. an operation would depend upon the D Hyperalimentation helps in recovery. presence of co-morbid conditions. E There is insulin resistance. ➜ Stress response ➜ Optimal perioperative care 2. In stress response, which of the 5. Which of the following statements are following statements are false? true for optimal perioperative care? A It is graded. A Volume loss should be promptly treated B Metabolism and nitrogen excretion are by large intravenous (IV) infusions of related to the degree of stress. fluid. C In such a situation there are B Hypothermia and pain are to be avoided. physiological, metabolic and C Starvation needs to be combated. immunological changes. D Avoid immobility. D The changes cannot be modified. E Helpful measures can be taken. E The mediators to the integrated response are initiated by the pituitary. ➜ Mediators 3. Which of the following statements about mediators are true? A They are neural, endocrine and inflammatory.

3 PRINCIPLES ➜ ➜ Figure 1 2. D 1. D Answers: Multiplechoicequestions 4

Stress response Homeostasis normal physiology occurs very soon. occursvery normal physiology and tachypnoea.Thebody’sinnatedefencemechanismscancombatmildstress,returnto tachycardia –pyrexia, immunological andmetabolicchangeswhicharereflectedinthephysiology totheinjury. Thereare indirectproportion is anincreaseinmetabolismandnitrogenexcretion (seeFig.1.1).injured patientof70 willelicitamajorresponse,requiringcareintheICU There stress responsefromwhichthepatientrecoversquitequickly. Ontheotherhand,aseverely ina30-year-oldfemale,willelicitminortransient such asalaparoscopiccholecystectomy inflicted.Anelectiveoperation inafitpatient, The stressresponseisgradedaccordingtotheinjury internal environment(the The normalphysiologicalstateofthehumanbodyisreferredtoashomeostasis–a syndrome. insultssuch asischaemia,infection,hypoxiaandcompartment ofsecondary and prevention i.e.attentiontonutritionandanticipation thembyjudiciousmanagement inanICU, preventing co-morbidity. In such cases, care in a high-dependency or intensive care unit (ICU) isessential. co-morbidity. orintensivecareunit (ICU) Insuchcases,careinahigh-dependency postoperatively thereturntonormalhomeostasiswouldtakelongerthaninthosewithno presenceofsepsisandanyongoinginsults.Ifthepatienthasco-morbidconditions, of injury, thisdependsupontheextent surgery; the homeostasistosomedegreeoccursinemergency Disturbancein alsoplaysapart. ofsurgery minimising thehomeostaticdisturbance.Theextent acid–base balance. maintenance. Theseorgansareinterdependentandthushelptomaintainanormalfluid role inits thespleen–playanimportant and,toalesserextent, lungs,kidneys brain, heart, In severe injury, thestressresponsecan bemodifiedbyanticipatingcomplicationsand injury, In severe In theelectivesituation,patientisalwaysoptimisedpriortoanyoperation,thereby Ongoing trauma,e.g.compartmentsyndrome(abdominal/limb) Inadequateoxygenation(hypoxia) Infection Secondary insults–3Is: Ischaemia .1

Metabolic response tosevere trauma. Severity ofinjury To getspeedyresolutionavoid(‘SO’) milieu intérieur Stress responsedependsupon ofClaudeBernard).Allthevitalorgans– Type ofinjury ➜ Figure 1.2 3.A,B,E Adrenal medulla

Adrenaline Mediators necrosis factor-alpha(TNF interleukins(IL-1, andtumour andhypermetabolism., IL-6) pyrexia protein breakdown, neuroendocrineresponseresultsinlipolysis,hepaticgluconeogenesis, minor role.Thisconcerted response. Thepancreasincreasesglucagonsecretion.Otherendocrines,thyroidandgonadsplaya Thiscreatesthe‘flightorfight’ hormone(GH). andgrowth hormone(ACTH) adrenocorticotrophic tosecrete thatactsonthepituitary factor(CRF) which secretesthecorticotrophin-releasing travelsalongafferentpathwaysofthespinalcordtohypothalamus Stress frominjury concert withtheendocrinesisreferredtoasneuroendocrineresponsetrauma. concert responses.Theneuralresponsethatinitiatesandactsin play neural,endocrineandinflammatory 1931),Langdon-Brown, setsinmotiontheentiresynchronousresponse.Thebodywillbringinto dysfunction syndromeoccurs( isinadequate,multipleorgan Iftheresponse toSIRS responsesyndrome(SIRS). inflammatory response,commonlycalledthesystemic antagonists,whichcontrols theproinflammatory andanyongoinginsults. Themediatorsdohelpintherepairprocessbyendogenous initial injury of weeks.Thetimefactordepends upontheextent andrepairphaselastingseveral a recovery by days,followed ofdamage.Itcanbedividedintoacatabolicphaselasting several the extent 12 phaselastsmuch longer dependingupon hours(clinicallymanifestingasshock).Theflow in1930.Cuthbertson Theebb,orearly,phasehelps initiatea‘holdingpattern’withinthefirst A modeloftwophases,‘ebb iscreated.Thetermwascoined bySirDavid andflow’, gland,rightlyreferredtoas‘theleaderintheendocrineorchestra’(SirWalter The pituitary response Neurohormonal Catecholamine-mediated ‘fight orflight’response Neuroendocrine response totrauma. nerves from peripheral Noradrenaline ␣ ) aresimultaneouslyreleased(seeFig.1.2). MODS), whichis Hypothalamus Stress responsetoinjury Cortisol andglucocorticoids Cortisol-releasing hormone (CRH) Adrenocorticotrophic hormone just astepawayfromdeath. Anterior pituitary Inflammatory response Cytokines 5

1: THE METABOLIC RESPONSE TO INJURY PRINCIPLES ➜ ➜ Figure 1 5.B,C,D,E 4.B,C,E 6

Optimal perioperativecare The recover Posterior pituitary Catabolism is an important aspectofrecovery.thebody’sstressresponsehas However, isanimportant Catabolism as itwillresultinoedema,peripheralandvisceral,thelattercausingdelayedgastricemptying. help insodiumandwaterretention.Thereforelargevolumesoffluidinfusionshouldnotbeused, apparatus(seeFig.1.3).angiotensin systemactivatedbythejuxtaglomerular AldosteroneandADH Aldosterone isalsoreleasedbytherenin– release aldosteroneandantidiuretichormone(ADH). archandleftatriumactto aortic aresult ofhypovolaemia,receptorsinthecarotidartery, As toreducemorbidityandmortality. settingusingaslidingscalehasbeenshown ICU complications: ,renalimpairmentandpolyneuropathy. Intravenousinsulin infusioninthe hyperglycaemic state–stress-induceddiabetes.Thepatientisthereforeatincreasedriskofdiabetic ofthestressdeterminesduration effectofstress.Theseverity resistance, atemporary to increasedglucoseproductionanddecreaseduptakeinperipheraltissuesasaresultofinsulin Hyperglycaemiaisanormalresponsetostress.Thisdue should thereforebeatamodestlevel. with increasedriskofhospital-acquiredinfections. ileus.Therefore,clinically,patientsareweakwithmalaiseandfunctionsuboptimally paralytic problems;gutmusclemaybeaffectedtoproduce hypoventilation withresultantpulmonary musclesareaffected,resultingin change isperipheralskeletalmuscle;sometimesrespiratory andadecreaseinmuscleproteinsynthesis.Themajorsiteforsuch protein breakdown immune systemandthewound.Duringcatabolism,musclewastingoccursfrom –theliver, parts peripheries, suchasmusclefatandskin,tothemoreimportant a capacitytotriagethecataboliceffect.Theeffectconcentratesawayfrom stressed patient. heatinthe mustbemadetoconserve Thereforeall efforts greater riskofcardiacarrythmias. CNS (hypothalamus) Hypothermia, duetoincreasedproduction ofadrenalsteroidsandcatecholamines,causes Hyperalimentation is not advisable as it enhances the metabolic stress. Nutritional support asitenhancesthemetabolic stress.Nutritionalsupport Hyperalimentation isnotadvisable ADH .3 ↑ Conservation ofsodiumandwater atrenaltubule H y y povolaemia intrauma. process Hypovolaemia fromhaemorrhage Fluid andelectrolyteconservation Aortic andcarotidbody Atrial natriureticpeptide Atrial stretchreceptors (’The three-linewhip’) pressure receptors (ANP) ↓ Renin-angiotensin Aldosterone system ↑ survival after recovery fromamajorstressresponse. afterrecovery survival stressresponse.Beta-blockersandstatinshavearoleinimprovinglong-term blocking thecortisol Epidural analgesianotonlyreducesstressfrompainbutalsotheinsulinresistance,by embolism. minimising thedreadedcomplicationsofdeepveinthrombosisandpulmonary musclewastingbesides thereforehelpsinpreventing impairs proteinsynthesis.Earlymobilisation appropriate routewillavoidlossofbodymass. 100 gofglucoseaday;thishasprotein-sparingeffect.Earlyinstitutionnutritionbythemost intravenouslyprovides bylossofleantissue.Atleast2L5percentdextrose tissue followed is donebymobilisingglycogenstoreshepaticgluconeogenesis.Fat ismobilisedfromadipose Perioperative care can be optimised by attention to feeding and preventing fluidoverload. Perioperative carecanbeoptimisedbyattentiontofeedingandpreventing Immobility shouldbeavoidedasitinducesmusclewasting.Inactivityofskeletal thebodyneedstoproduceglucosesustaincerebralfunction.This aresultofstarvation, As 7

1: THE METABOLIC RESPONSE TO INJURY 2 and blood transfusion

Multiple choice questions ➜ Cell metabolism D This is influenced by the duration and 1. Which of the following statements are extent of tissue hypoperfusion. true? E It usually does not cause death. A Cells change from aerobic to anaerobic ➜ metabolism when perfusion to tissues is Responses in shock reduced. 4. In which of the following cases might B The product of aerobic respiration is tachycardia accompany shock? lactic acid. A Hypovolaemia due to gastrointestinal C The product of anaerobic respiration is (GI) bleeds carbon dioxide. B Patients on alpha-blockers D The accumulation of lactic acid in the C Patients with implanted pacemakers blood produces systemic respiratory D Fit young adults with normal rate acidosis. of 50/min E Lack of oxygen and glucose in the cell E Cardiogenic shock. will eventually lead to failure of sodium/ 5. Which of the following regarding blood potassium pumps in the cell membrane pressure in shock are false? and intracellular organelles. A Elderly patients who are normally ➜ hypertensive may present with a ‘normal’ Hypovolaemic shock . 2. Which of the following statements B Children and fit young adults are able to regarding hypovolaemic shock are maintain blood pressure until the final true? stages of shock. A It is associated with high cardiac output. C Hypotension is one of the first signs of B The vascular resistance is high. shock. C The venous pressure is low. D Beta-blockers may prevent a tachycardic D The mixed venous saturation is high. response. E The base deficit is low. E Blood pressure is increased by reduction ➜ in volume and peripheral Ischaemia-reperfusion vasoconstriction. syndrome 3. Which of the following statements ➜ Compensated shock about ischaemia-reperfusion syndrome 6. Which of the following statements is correct? about compensated shock are false? A This refers to the cellular injury because A The preload is preserved by the of the direct effects of tissue hypoxia. cardiovascular and endocrinal B It is seen after the normal circulation compensatory responses. is restored to the tissues following an B Tachycardia and cool peripheries may be episode of hypoperfusion. the only clinical signs. C The increased sodium load can lead to C The perfusion to the skin, muscle and GI myocardial depression. tract is increased.

8 ➜ ➜ ➜ . Whichofthesestatementsabout 9. aretrue Whichofthefollowing 8. statementsare Whichofthefollowing 7.

B A Mixed venoussaturation E D C B A Inotropic supportinshock E D C B A Resuscitation inshock E D levels are 30–40 percent. are30–40 levels saturation The normalmixedoxygen bythetissues. extraction and delivery a measureoftheoxygen fromthebody is returning totheheart ofoxygen The percentagesaturation may beharmful. Use intheabsenceofadequatepreload catecholamines. the vasodilatationisresistantto Vasopressin maybeusedwhen cardiacoutput. complicated bylow in cardiogenicshockorseptic Dobutamine istheagentofchoice indicated indistributiveshockstates. Phenylephrine andnoradrenalineare hypovolaemic shock. This isthefirst-linetherapyin loss orsodiumoverload. inconditionsoffreewater shock except andshouldnotbeusedin expanders Hypotonic solutionsarepoorvolume iszero. colloids andcrystalloids capacityofboth The oxygen-carrying infusion offluids. rapid Long, wide-borecathetersallow addressed first. hypovolaemia andpreloadmustbe In allcases,regardlessofclassification, perfusion. filling andcoronary willhelptoincreasediastolic empty heart Administration ofinotropicagentstoan rate. higher mortality more than12 hourshaveasignificantly for Patients withocculthypoperfusion acidosisisseen. Systemic respiratory mixed venoussaturation arefalse? regarding inotropicsupportinshock? false? ➜ ➜ 11. Which of the following aboutblood Whichofthefollowing 11. about Whichofthefollowing 10. 12. Which of the following isa Which ofthefollowing 12.

Blood transfusion Reactionar C B A E D C B A E D C E D C B A E D reactionary haemorrhagearefalse? reactionary transfusion arefalse? This isdelayedhaemorrhageoccurring seen insepsis. are levels High mixedvenoussaturation with hypovolaemicshock. consistent delivery inadequate oxygen percentindicate 50 below Levels the superiorvenacava(SVC). of blooddrawnfromalineplacedin Accurate measurementsareviaanalysis Hypothermia. Hypokalaemia Hyperkalaemia Hypercalcaemia Coagulopathy transfusions? ofmassiveblood complication transfusion. forautologous 3 weeksbeforesurgery Patients canpre-donatebloodupto Platelets haveashelflifeof3weeks. deficiency. fibrinogen statesandinfactorVIII isusefulinlow- Cryoprecipitate haemorrhage. as thefirst-linetherapyincoagulopathic isconsidered Fresh frozenplasma(FFP) consideredatypicalindication. is now of10A haemoglobinlevel g/dLorless It isusuallyvenous. re-exploration. It canbesignificant,requiring It isassociatedwithinfection. slippage ofligature. ofbloodpressureor clot, normalisation It isusuallycausedbydislodgementof within 24hafteroperation. y haemorrhage 9

2: SHOCK AND BLOOD TRANSFUSION PRINCIPLES ➜ ➜ Extended matchingquestions Extended Choose and match the correct intervention witheachofthe scenariosgivenbelow: Choose and matchthecorrectintervention Chooseandmatchthecorrectdiagnosiswitheachofscenariosgivenbelow: 10

5 4 3 2 1 E D C B A 2. Vasopressor andinotropic supportinshock 7 6 5 4 3 2 1 G F E D C B A 1. T Distributive shock duetospinalcordinjury. Hypovolaemic shock duetosplenicinjury. Vasodilatation resistantto catecholaminesduetorelativeorabsolutesteroiddeficiency. Distributive shockduetosepsis. shockwhenmyocardial depressioncomplicatesshockstate. Cardiogenic Vasopressin Dobutamine Phenylephrine No roleforvasopressororinotropicagent Noradrenaline area. Hispulseis140/min is100/82 andhisBP mmHg. rugby. Heiscomplainingofleftupperabdominalpainandhas somebruisingoverthesame A 19-year-old whileplaying maleisbroughttothehospitalaftersustaining anabdominalinjury His pulseis122/min 100/62 andhisBP mmHg. limbs. deficitsinbothlower tohisbackandhasmotorsensory sustained anisolatedinjury A 28-year-oldmotoristisbroughttotheA&Eafteraroadtrafficaccident(RTA). Hehas peritonitis. Hispulseis130/min mmHg. 84/50 andhisBP unwellwithwarmperipheries.Hehassignsofgeneralised helooksvery week. Onexamination abdominalpainforthepast malehasbeencomplainingofincreasinglower An 86-year-old mmHg. is96/70 hypothermic. HerpulseisirregularandherBP A 55-year-oldwomanwithpoorlycontrolledhypothyroidismisfoundcomatose.She dehydrated.Hispulseis128/minhe isvery mmHg. is88/52 andhisBP tired.Onexamination andfeelsvery past 4days.Hehasbeenunabletokeephisfooddown A 76-year-old maleisbroughttothehospitalwithpersistentdiarrhoeaandvomitingfor myocardial infarction(MI). is74/48collapses. HispulseisirregularandBP featuresofananterolateral shows mmHg.ECG complainsofchestpainand disease(IHD) A 78-year-old ischaemicheart manwithknown mmHg. is60/38 HisBP andbreathingsupport. requires airway stridorandcollapsesaftereatingasnack.He develops A 7-year-oldboywithnutallergy Hypovolaemic shock–non-haemorrhagic Endocrinal shock Anaphylactic shock Neurogenic shock Hypovolaemic shock–haemorrhagic shock Cardiogenic Septic shock y pes ofshock ➜ ➜ ➜ 2.B,C(see Table 2.1) 1. A,E Answers: Multiple choicequestions Chooseandmatchthecorrectdiagnosiswitheachofscenariosgivenbelow:

H Cell metabolism 5 4 3 2 1 E D C B A 3. Complicationsofbloodtransfusion potassium, arereleasedintothebloodstream. andcelllysisensues.Intracellularcontents,including the lysosomes releaseautodigestiveenzymes anaerobic respirationceasesandthereisafailureofthesodium/potassium pump.Intracellular glucosewithinthecellsisexhausted, increasestissuehypoxia.As The resultantoedemafurther walls. oftheepithelialandendothelialcells,whichleadstolossintegrity and‘leaky’ injury Thisleadsto radicalsandcytokines. neutrophil primingwiththegenerationofoxygen-free the immuneandcomplementsystemsareactivated.Thisalsoresultsin thecomplementand tissueischaemia progresses, of lacticacidinthebloodproducessystemicmetabolicacidosis.As theaccumulation of anaerobicrespirationislacticacid.Whenenoughtissueunderperfused, aerobic respirationiscarbondioxide.Thiseliminatedefficientlythrough thelungs.Theproduct Theproductof Cells switchfromaerobictoanaerobicmetabolismwhendeprivedofoxygen. y He collapsesandbecomesanuric.isalsofoundtobejaundiced. minutesthepatientisunwellandhisurinehaemorrhagic. Withinafew transfusion isstarted. themassheisjustgoingoffforherbreak.Theblood tostart on thetableandasksHCA surnames)needingbloodtransfusions.Thestaffnursepointstotheunits the same staffhavephonedinsick.Therearetwopatients(with busyandquiteafew The wardisvery uncontrolled andthebloodfailstoclot. thatthebleedingisstill shockandrequiresmultiplebloodtransfusions.Itisobserved in severe sustainsmultipleinjuriesafteranRTA.A 34-year-oldmotorcyclist Heisbroughttothehospital tetanyandcomplainsofcicumoraltingling. as aresultoffall.Hedevelops unitsofbloodtransfusionduetomultipleinjuriessustained A 38-year-oldmanrequiresseveral parasite. day. Peripheral and chillswithrigorsthenext bloodsmeardemonstratesmalarial Africa. Hehaslostalotofbloodandishencegiven2unitstransfusion.develops in A 28-year-oldmaleistakentoanearbyhospitalaftersustaininginjurieswhileonsafari oedema. Chest X-rayconfirmspulmonary finecrepsbilaterally. reveals is foundtobehavingdifficultyinbreathing.Chestexamination 4 unitsofblood.Thesearetransfusedoveraperiod6h.Four hourslaterthepatient prescribes womanisadmittedwithahaemoglobin(Hb)of5.6g/dL.TheHO A 86-year-old Infection Hypocalcaemia Disseminated intravascularcoagulation(DIC) Fluid overload transfusionreactionduetoincompatibility Haemolytic povolaemic shock 11

2: SHOCK AND BLOOD TRANSFUSION PRINCIPLES ➜ ➜ ➜ ➜ 7. A,C 6.C,D 5.C,E 4.A,B 3.B,D 12

Table 2.2 atcaioi ++ +++ Increased Severehypotension Anuric Laboured Mild hypotension ++ Increased Normal Increased Reduced Comatose Increased Severe Increased Mildincrease ++ Normal Normal Moderate Normal Drowsy Mildanxiety Normal Blood pressure Pulse rate Mild + Compensated Normal Level ofconsciousness Urine output Lactic acidosis Parameter aedfictHg ihHg High Low High High High Low High High Low Low High High Low High Low High Distributive Obstructive Low Cardiogenic Low Hypovolaemia Low High Base defi cit Mixed venoussaturation Venous pressure Vascular resistance Cardiac output Parameter Table 2.1 Resuscitation inshock Compensated shock Responses inshock Ischaemia-reperfusion s wide-bore catheters that allow rapidinfusion offluids.Hypotonicfluidsare poorvolume wide-bore catheters thatallow state. Thefirst-linetherapyisintravenous (IV)accessadministrationofIVfluidsusingshort, Resuscitation shouldnotbedelayed inordertodefinitivelydiagnosethecauseofshocked Systemic metabolicacidosisisseeninshock(alsoseeTable 2.2) to maintaincardiacoutputbyincreasingboththerateandstrokevolume. initially compensates Hypotension maynotbeseenuntiltheshockiswellestablished.Theheart in patientsonbeta-blockers. Tachycardia isanearlywarningsigninhypovolaemicshock.Thetachycardicresponsenotseen hypoperfusion. and durationoftissue canbereducedbylimitingtheextent injury death. Ischaemia-reperfusion and andorgandamage.Thiscanleadtomultiplefailure (MOF) endothelialinjury further hypotension.Thecellularandhumoralcomponentsflushedbackintocirculationcause further potassium loadthathasbuiltupcanleadtodirectmyocardialdepression,vasculardilatationand syndrome.Theacidand normal circulationisrestored.Thistermedischaemia-reperfusion occursoncethe injury effects oftissuehypoxiaandlocalactivationinflammation.Further cellularandorgandamageprogressesasaresultofdirect During theperiodofreperfusion, Responses indifferent degrees ofshock Response indifferent t y ndrome y pes ofshock Degree ofshock Degree Type ofshock ➜ ➜ ➜ ➜ 12. A,C,D,E 11. A,D 10. C,E 9.B,C 8.B,C,D,E

Blood transfusion Reactionar Mixed venoussaturation Inotropic supportinshock perfusion. reduce diastolicfillingandthereforecoronary storesanddramatically willrapidlyandpermanentlydepletethemyocardiumofoxygen heart adequately resuscitatedbeforeundergoingsurgery. Administrationofinotropicagentstoanempty with surgery. obstructionandhypovolaemicshockshouldbe Conversely,apatientwithbowel fluid therapywithoutcontrollingthesiteofbleeding.Resuscitationshouldproceedinparallel bleed),itiscounterproductivetoinstitutehigh-volume GI aneurysm, ruptured abdominalaortic byanassessmentoftheresponse.Inpatientswhoareactivelybleeding(majortrauma, followed toassumethatitishypovolaemiaandbeginwithfluidresuscitation, cause ofshock,itissafer in diabetesinsipidusandsodiumoverload,e.g.cirrhosis).Ifthereisaninitialdoubtaboutthe isfreewaterloss,as andshouldnotbeusedinthetreatmentofshock(anexception expanders transfusion-related acute lung injury (usuallyfromFFP). transfusion-related acutelunginjury febrile transfusionreaction,allergic reaction, infection,airembolism,thrombophlebitisand ironoverload. transfusions overlongperiodsoftimedevelop Hypocalcaemia isanothercomplicationofmassivetransfusion.People whoreceiverepeated surgery. between6and8g Levels /dL areselectivelytransfused.AlsoseeTable 2.3. is acceptableinpatientswhoarenotbleeding,symptomaticand abouttoundergomajor The transfusiontriggerwashistorically10 g of6g thatalevel however, believed, /dL. It isnow /dL haemorrhage. Itisnotassociatedwithinfection,unlikesecondary The bleedingisusuallyarterial. distributive shock. (>70 levels shock. Highmixedvenoussaturation percent)areseeninsepsisandsomeformsof bythecells.Thisisconsistentwithhypovolaemicorcardiogenic extraction and increasedoxygen delivery percentindicateinadequateoxygen 50 below percent.Levels are50–70 levels saturation fromtheSVCgiveslightlyhighervalues.Normalmixedvenousoxygen the rightatrium.Samples bythetissues.Accuratemeasurementisalonglineplacedin andextraction delivery oxygen fromthebodyisameasureof returningtotheheart ofoxygen The percentagesaturation vasopressinmaybeused. agents (e.g.absoluteorrelativesteroiddeficiency), systemic vascularresistanceandahighcardiacoutput.Ifthevasodilatationisresistanttothese sepsis andneurogeniccauses.Thesestatesarecharacterisedbyperipheralvasodilatation,alow Phenylephrine andnoradrenalinearehelpfulindistributiveshockstates,suchasthosedueto The first-linetherapyinhypovolaemicshockisIVaccessandadministrationoffluids. Complications from a single transfusion include incompatibility haemolytic transfusionreaction, Complications fromasingletransfusionincludeincompatibilityhaemolytic y haemorrhage 13

2: SHOCK AND BLOOD TRANSFUSION PRINCIPLES ➜ ➜ ➜ 4C 3D 2E 1B 1D,2A(or5),3E,4B,5C2) 1E,2B,3G,4F,5A,6D,7C matchingquestions Answers: Extended 14

• • • 3. Complicationsofbloodtransfusion 2. Vasopressor and inotropic supportinshock 1. T aggressively treatedand,betterstill,avoided.Thestandardguidelinesare: complicationaftermassivemultipletransfusions.This shouldbe Coagulopathy isaknown complicationaftermassivetransfusions. Hypocalcaemia isaknown Disease(CJD). Creutzfeldt–Jakob andnew-variant bacterial infections,HIV The otherinfectionsthatcanbetransmittedthroughbloodtransfusion arehepatitisBandC, transfusionofunitsmaybeneeded. overload. Administrationoffrusemidefollowing cells arepreferabletowholebloodandthepatientmustbecloselymonitored forsignsoffluid inprescribingtransfusions/fluidstoelderlypatients.Packed cautionshouldbeexercised Extreme opnn osiunsIdctosSoaeShelflife Storage Indications Redbloodcellsspun Constituents Packed redcells Component Table 2.3 plasma (FFP) Fresh frozen rorcptt Supernatantprecipitate Cryoprecipitate concentrates Thrombocytopenia, complex Prothrombin Plateletconcentrates Platelets platelets, if platelet count < 50 × 10platelets, ifplateletcount<50 iffibrinogen<0.8 g/L cryoprecipitate, >1.5 thromboplastintime(PTT) timesnormalvalues andpartial ifprothrombintime(PT) FFP, y pes ofshock Components usedinbloodtransfusions oglto atr Coagulopathic factors down andconcentrated VIII and fiVIII brinogen –richinfactor of FFP included/separate isalso factor VII IX,X; of factorsII, Purificoncentrates ed 9 /L. hoi nei SAG-M solution Chronic anaemia haemorrhage haemorrhage in uncontrolled (Warfarin) anticoagulant Emergency reversalof platelet dysfunction defifactor VIII ciency Low fi brinogen states, Parameter 2–6ºC 4 o–0C2years –40 to–50ºC agitator) (special 2years 20–24ºC –30ºC –– 5 weeks 5 days 5A checked againsttheissueslipforthatpatient. prescription andthelabelofdonorblood.Additionallybloodserialnumbershouldbe Twoto ensurecorrectlabellingofsamples. individualsshouldcheckthepatient’sdetailsagainst shouldbetaken blood, itisessentialthatthecorrectpatientreceivestransfusion.Care happen.Whileprescribingandadministering This iscompletelyavoidableandhenceshouldnever 15

2: SHOCK AND BLOOD TRANSFUSION 3 Wounds, tissue repair and scars

Multiple choice questions ➜ Wound healing ➜ Management of wounds 1. Which of these factors influences 4. Which of the following statements healing of a wound? regarding management of the acute A Vascular insufficiency wound are correct? B Diabetes mellitus A A bleeding wound should be elevated C Malnutrition and a pressure pad applied. D Site of wound B Clamps may sometimes need to be put E Smoking. on bleeding vessels blindly. C Anaesthesia is usually not required in the 2. Which of the following statements assessment of wounds. about the process of wound healing D A thorough debridement is essential. are true? E Repair of all damaged structures may be A The inflammatory phase begins 2–3 days attempted in a tidy wound. after the injury. B The proliferative phase lasts from 3 days 5. Which of the following statements to 3 weeks following the injury. regarding the management of specific C The remodelling phase involves fibroblast wounds are true? activity and production of collagen and A A haematomata should never require ground substance. release. D Fibroblasts require vitamin C to produce B Anaerobic and aerobic organism collagen. prophylaxis is needed in bite wounds. E The white cells stick to the damaged C Puncture wounds should always be endothelium and release adenosine explored. diphosphate (ADP) and cytokines. D Degloving injuries will require serial excision until viable tissue is confirmed. 3. Which of the following statements are E Compartment syndrome typically occurs true? in a closed lower limb injury. A Healing by primary intention results in minimum and the best ➜ Chronic wounds scar. 6. Which of the following statements are B Granulation, contraction and true? epithelialisation are seen in healing by A The most common chronic wound in secondary intention. developed countries is the leg ulcer. C Tertiary intention involves immediate B Pressure sores occur in approximately closure of the wound. 5 per cent of all hospitalised patients. D A crushed and contaminated wound C Surgical treatment is usually required in is best suited for healing by primary the treatment of leg ulcers. intention. D Bed-bound patients should be turned E Primary repair of all structures should be every 4 h to prevent pressure sores. attempted in an untidy wound.

16 ➜ ➜ ➜ Extended matchingquestions Extended statements Whichofthefollowing 8. occuroverwhichof Pressuresorescan 7. Chooseandmatchthecorrect diagnosiswitheachofthescenariosgivenbelow:

6 5 4 3 2 1 F E D C B A 1. T D C B A infections Necrotising soft-tissue E D C B A Pressure sores E These arecleanwounds andusuallyincised.Thetissues arehealthywithseldomtissue loss. disturbanceand,finally,absentpulses. passive movement,distalsensory pain,painon limb injuriesandarecharacterisedbysevere These typicallyoccurinclosedlower should beregardedaspreventable. These woundsaredefinedastissue necrosiswithulcerationduetoprolongedpressure.They retained foreignbody. These arewoundscausedbysharpobjectssuchasneedles.X-raymay beneededtoruleouta tissues. invariably contaminatedandhasdevitalised This iscausedbycrushoravulsionforcesandusuallyhasvariableamount oftissueloss.Itis avulsion, leavingtheneurovascularstructures,tendonorboneexposed. This occurswhentheskinandsubcutaneousfatarestrippedfrom underlying fasciaby Pressure sore syndrome Compartment Untidy Degloving Puncture Tidy syndrome. Clostridial speciescausetoxicshock ‘Dishwater pus’isacharacteristicfeature. a chroniccourse. run The onsetisusuallygradualandthey are usuallypolymicrobialinfections. They Ischium. Greater trochanter Occiput Heel Sacrum occlusive pressure(30mmHg). thecapillary pressureexceeds external Risk oftissuenecrosisincreasesifthe are true? about necrotisingsoft-tissueinfections areas? the following y pes ofwound ➜ ➜ 10. Which of the following areusefulin Whichofthefollowing 10. statementsare Whichofthefollowing 9.

Keloids Scars E D C B A E D C B A E true? the treatmentofkeloids? A hypertrophied scar extends beyond beyond scarextends A hypertrophied reaches thatofthenormalskin. ofthescarnever The tensilestrength sutures. polyfilament Suture markscanbereducedbyusing collagen. oftypeB Keloids containanexcess Scars continuematuringfor3months. antibiotics. Treatment isessentiallymedicalusing Vitamin Dpreparations. andradiotherapy Excision andsteroidinjection Excision Silicone gelsheeting Elasticated garments incision. the boundariesofprevious 17

3: WOUNDS, TISSUE REPAIR AND SCARS PRINCIPLES ➜ ➜ ➜ Chooseandmatchthecorrect diagnosiswitheachofthescenariosbelow: Chooseandmatchthecorrectdiagnosiswitheachofscenariosbelow: 18

E D C B A 4. Wound dressings 5 4 3 2 1 E D C B A 3. Phasesofwoundhealing 8 7 6 5 4 3 2 1 H G F E D C B A 2. Managementofwoundsandrelated conditions Polymeric film Foam Hydrocolloid Hydrogel Debriding agent during thisphaseistype3. This phaseconsistsmainlyoffibroblastactivityandcollagenproduction. Thecollagenproduced This phasehasincreasedvascularitywithplentyofneutrophilsandlymphocytes. ceases. complete.Thevascularityhasreducedand growth The contractionofthescarisnow Platelet-enriched bloodclotanddilatedvesselsareafeatureofthisphase. wound contractionarealsoseeninthisphase. Realignmentofcollagenfibresalongthelinestension,decreased vascularityand achieved. This phaseischaracterisedbyreplacementoftype3collagen 1 untilaratioof4:1is Mature scar Remodelling phase Proliferative phase phase Late inflammatory phase Early inflammatory skin graft. bysplit isusuallydoneuntilpunctuatedermalbleedingseen.Thisfollowed excision ofallnon-bleedingskin.Serial withradicalexcision underanaesthesia(EUA) Examination withtissuebiopsiessentforculture.Skingraftmaybeneededlateron. Surgical excision Debridement andvacuum-assistedclosure(VAC). Fasciotomy. Intralesional steroidinjectionandintralesionalexcision. repairofallstructures–,tendon,vesselandnerve. Primary Multiple Z-plastiesoruseofskingrafts/flaps. bydefinitiveclosure/repair.Multiple debridementsfollowed Contractures Necrotising infections Keloids Pressure sore Untidy wounds syndrome Compartment Degloving wounds Tidy wounds ➜ ➜ 4.A,D,E 3.A,B 2.B,D 1. A,B,C,D,E Answers: Multiplechoicequestions Chooseandmatchthecorrectdescriptionwitheachofwounddressingsbelow:

Management ofwounds Wound healing 7 6 5 4 3 2 1 G F preferred in children. In limb injuries, particularly those ofthehand,atourniquetshould beused. preferred inchildren. Inlimbinjuries,particularly analgesia and/oranaesthesia (local,regionalorgeneral) arerequired.Generalanaesthesia is adequate likely andvascularanastomosisisrendered impossible.Inordertofacilitateexamination, damage is site andpossiblestructuresdamaged. Clampsshouldnotbeappliedblindlyasnerve (ATLS)Life Support takingintoconsiderationthe guidelines.Thewounditself shouldbeexamined, thewholepatientaccordingtoAdvancedTraumaThe surgeonshouldremembertoexamine repairofallstructuresshouldbeattemptedinatidywound. be carriedout.Primary occasionsbefore definitiverepaircan wound isbestmanagedbydebridementononeorseveral and theedgeslateropposedwhenhealingconditionsarefavourable. A crushedandcontaminated intention and inthisthewoundisinitiallyleftopen Tertiary intentionisalsocalleddelayedprimary vascularity andwoundcontractionduetofibroblastmyofibroblast activity. There isrealignmentofcollagenfibresalongthelinetension,decreasedwound 4:1 isachieved. phase ischaracterisedbymaturationofcollagen,withtype1 Theremodelling ofthewoundsurface. loopsandre-epithelialisation blood vesselsascapillary ofnew fibroblast activitywiththeproductionofcollagenandgroundsubstance,growth andmacrophages. cellssuchaspolymorphonuclearlymphocytes These attractinflammatory factor(TGF)-beta. plateletfactor4andtransforminggrowth factor(PDGF), platelet-derived growth suchas andcytokines stick tothedamagedendothelialliningofvesselsandreleaseADP phasebeginsimmediatelyafterthewoundingandlasts2–3days. Platelets The inflammatory other causeofimmunodeficiency. andany pressure, vitaminandmineraldeficiencies,(steroids,chemotherapy),HIV radiation, wounding (incision,crushoravulsion),contamination,lossoftissue,previous Wound healingisalsoinfluencedbyotherfactors,includingstructuresinvolved,mechanismof Silastic (Elastomer). Opsite, Tegaderm Granuflex acid Benoxyl-benzoic Porcine skin,amnion Intrasite, Lyofoam Kaltostat, Sorbsan Biological membranes Fibrous polymer The proliferativephaselastsfromthethirddaytoweek,consistingmainlyof replacing type3untilaratioof 19

3: WOUNDS, TISSUE REPAIR AND SCARS PRINCIPLES ➜ ➜ ➜ ➜ 9.B,D 8.A,C 7. A,B,C,D,E 6.A,B,E 5.B,C,D,E 20

(streptococcal gangreneandnecrotisingfasciitis). progression. Therearetwomaintypesofnecrotisinginfections–clostridial andnon-clostridial haemolytic sent forculture.Widerawareasare oftenleftbehindwhichmayrequireskingraft. syndrome andisoftencalled‘flesh-eating bug’.Treatment withtissuebeing issurgicalexcision of VAC healing. andskinflapsmaybehelpfulinachieving for 10 10 severy min. 2h,withthewheelchair-boundpatientbeingtaughttoliftthemselvesofftheirseat turned every diversioninappropriatecases.Thebed-boundpatient shouldbe fluidised bedsandurinary/faecal air-lossandair- with goodskincare,specialpressure,dispersioncushionsorfoams,theuseoflow isthebesttreatment the skinceases,causingtissueanoxia,necrosisandulceration.Prevention Scars Necrotising soft-tissueinfections • • • • Pressure sores Chronic wounds aspiration. If large,painfulorcausingneuraldeficit,ahaematomamayrequirereleasebyincision period ofayearor more.Atfirst,itispink,hard,raised anditchy. thescarmatures, itbecomes As The maturationphase ofhealingresultsinscarring.The maturescarbecomesover a S. pyogenes arepolymicrobialinfectionsinvolvingGram-positiveaerobes ( contamination. They withwound These arerarebutoftenfatalinfectionsthatusuallyseenaftertrauma orsurgery illpatients.Thestagesofpressuresoreareasfollows: the elderlyandseverely in about5percentofallhospitalisedpatientsandtheincidenceishigherparaplegicpatents, occur A pressuresoreisdefinedastissuenecrosiswithulcerationduetoprolongedpressure.They failed andthepatienthasintractablepain.Meshedskingraftsaremoresuccessfulthansheetgrafts. and diagnosisishencevital.Surgicaltreatmentonlyindicatedifnon-operativemethodshave malignant change(Marjolin’sulcer).Effectivetreatmentofanyulcerdependsontreatingthecause, and simpletreatments.Anulcernotresponsivetothistreatmentshouldbebiopsiedruleout Necrotic tissue,oftenattheulcercentre,iscalledslough.Mostulcersaremanagedbydressings ofgranulationtissue,andattemptstohealbyscarringleavesafibroticmargin. to anovergrowth An ulcerisdefinedasabreakintheepithelialcontinuity. phaseleads Aprolongedinflammatory involving muscle,tendon,boneorjoint. tissuedestruction,possibly stage 4:full-thicknessskinlossthroughfasciawithextensive underlying fascia intothesubcutaneoustissuebutnotthrough stage 3:full-thicknessskinlossextending skinlossinvolvingtheepidermisanddermis stage 2:partial-thickness withoutabreachintheepidermis stage 1:non-blanchableerythema Care of pressure sores follows theprinciplesofacutewoundmanagement. Debridement,use ofpressuresoresfollows Care to occlusionpressure(over30mmHg),bloodflow thecapillary pressureexceeds If external Streptococcus ), Gram-negativeaerobes( . Theinfectionsarecharacterisedbysuddenpresentationandrapid E. coli , P. aeruginosa Streptococcus pyogenes ), Clostridia , Bacteroides causes toxicshock S. aureus andbeta- , ➜ ➜ ➜ ➜ ➜ 1F,2B,3G,4A,5C,6E,7D 1D,2A,3E,4B,5C 1D,2H,3A,4F,5C,6E,7G,8B 1C,2D,3B,4F,5E,6A matchingquestions Answers: Extended 10. A,B,C,D

4. Wound dressings 3. Phasesofwoundhealing 2. Managementofwoundsandrelated conditions 1. T Keloids phaseofwoundhealingandfromunfavourablesitingthescar.a prolongedinflammatory oftheoriginalincisionorwound.Itresultsfrom theboundary beyond tissue thatdoesnotextend scar scarisdefinedasexcessive can bereducedbyusingmonofilamentsutures.Ahypertrophic reachthatofnormalskin.Suturemarks graduallyincreasesbutwillnever 1–2 years.Thestrength soft. Mostofthechangestakeplaceoverfirst3monthsbutascarwillcontinuetomaturefor almost acellularasthefibroblastsandbloodvesselsreduce.Thescarthenbecomespaler,flat hypervascularity andincreasedBtypeofcollagen. hypervascularity collagenwith excess shows whose pointsarethexiphisternumandeachshouldertip).Histology areasofthebody (atriangle andcertain factor, deeplypigmentedskin,aninheritedtendency ofgrowth levels butitisassociatedwithelevated isunknown incision orwound.Itsaetiology theboundariesoforiginal beyond scartissuethatextends A keloidisdefinedasexcessive y pes ofwound 21

3: WOUNDS, TISSUE REPAIR AND SCARS 4 Surgical infection

Multiple choice questions ➜ Koch’s postulates D Not using prophylactic antibiotics 1. Which of the following are part of E Uraemia. Koch’s postulates? ➜ A It must be found in considerable Secondary infections numbers in the septic focus. 5. Which of the following is a cause of B A reduction in the organisms should be secondary (or exogenous) infection? achieved by using appropriate antibiotics. A Poor hand-washing technique C It should be possible to culture it in a B Community pure form from the septic focus. C Perforated diverticular disease D Healing of a wound is possible without D Anastomotic leak pus formation. E Inadequate air filtration in the theatre. E It should be able to produce similar lesions when injected into another host. ➜ Surgical site infections 6. Which of the following statements ➜ Natural barriers to infection about surgical site infections (SSIs) are 2. Which of the following is a natural true? barrier to infection? A Infection in the musculofascial tissues is A Intact epithelial surface known as deep SSI. B High gastric pH B The patient may have systemic signs in a C Antibodies minor SSI. D Antibiotics C Infection causing delay in hospital E . discharge is a major SSI. D The differentiation between major and ➜ Host resistance to infection minor SSIs is not important. 3. Which of the following is a cause E Surveillance for surgical site infection of reduced host resistance to should be done for a year after infection? implanted surgery. A Malnutrition ➜ B Heart failure Abscesses C Cancer 7. Which of the following statements D AIDS regarding abscesses are true? E Systemic sclerosis. A Staphylococcus aureus is one of the most common causative organisms. ➜ Risk factors for wound B The abscess wall is composed of infection epithelium. 4. Which of the following is a risk factor C Most wound-site abscesses occur before for wound infection? the patient is discharged from the hospital. A Poor perfusion D Antibiotics are indicated if there is B Use of skin clips for wound closure. evidence of cellulitis. C Poor surgical technique E Actinomyces can cause a chronic abscess.

22 ➜ ➜ ➜ ➜ 11. Which of the following statements Whichofthefollowing 11. statements Whichofthefollowing 10. beseenin can Whichifthefollowing 9. statements Whichofthefollowing 8.

infections Clostridial wound s Severe sepsis(sepsis response s S Cellulitis B A E D C B A E D C B A E D C B A y y ndrome) stemic inflammator is seeningasgangrene. andsweet-smellingexudate Thin, brown spore-bearing cocci. Clostridia areGram-positiveaerobic tumour necrosisfactor(TNF)-alpha. and suchasinterleukins(IL-6) cytokines ismediatedbyreleased MSOF MODS. is theendstageofuncontrolled Multiple systemorganfailure(MSOF) infection. isthesystemiceffectof (MODS) Multiple organdysfunctionsyndrome infection. There isabsenceofdocumented iscommon. (ARDS) distresssyndrome Acute respiratory Tachypnoea ( >20/min). Tachycardia ( >90/min) No documentedinfection <4 White cellcount(WCC) C) ( <36º Hypothermia Blood cultureisusuallypositive. Systemic signsarenotpresent. perfringens It iscommonlycausedby It ispoorlylocalised. infection oftissues. This isnon-suppurativeinvasive are true? regarding clostridialwoundinfections syndrome) aretrue? about severesepsis(sepsis SIRS? regarding cellulitisaretrue? . y ndrome (SIRS) Clostridium ϫ y 1000/dL

➜ ➜ ➜ 13. Which of the following affectsthe Whichofthefollowing 13. statementsare Whichofthefollowing 12. 15. Which of the following measuresis Which ofthefollowing 15. 14. Which of the following mayrequire Whichofthefollowing 14.

Proph infections Treatment ofsurgical Surgical woundinfection D C B A E D C B A E D C B A E D C B A E true? infection? wound useful inreducingsurgical choice ofprophylacticantibiotic? should be done before starting should bedonebeforestarting organism Identification ofthecausative endotoxins. The signsandsymptomsareduetothe manure. The sporesarewidelyspreadinsoiland increase risk. Necrotic andforeignmaterialinwounds Shaving ofarea Antiseptic skinpreparation contamination. Unexpected ofprosthesis Insertion surgery Gastrointestinal bloodloss Excessive Prolonged operations antibiotic? more thanonedoseofprophylactic Local resistancestrains. Hospital policies Personal preference Cost likely tobeencountered spectrumoforganisms The expected route. locally ismoreeffectivethantheoral Administration ofantibioticpreparations wounds. Polymeric filmscanbeusefulininfected infection. decreases theincidenceofwound Subcuticular continuousskinclosure closure. orsecondary primary Wounds arebestmanagedbydelayed antibiotics. y lactic antibiotics 23

4: SURGICAL INFECTION PRINCIPLES ➜ ➜ ➜ 18. Which of the following statements Whichofthefollowing 18. statements Which ofthefollowing 17. statements Whichofthefollowing 16. 24

treatment Principles ofantimicrobial Infection causingbacteria Different t E D C B A E D C B A E D C B A surgical infectionsaretrue? surgical regarding antimicrobialtreatmentof infection aretrue? regarding bacteriainsurgical ofwoundsaretrue? about types catheterisation. cause ofinfectionrelatedtourethral Gram-negative bacilliareamajor infection. anddoes notcauseclinical commensal Staphylococcal epidermidisisa the population. ofupto15the nasopharynx percentof Staphylococci arenormallyresidentin and erythromycin. All streptococciaresensitivetopenicillin associated withinfection. Beta-haemolytic controversial. is in non-prostheticcleansurgery The roleofprophylacticantibiotics in‘dirty wounds’. mandatory Antibiotic prophylaxiswouldbe infection rateoflessthan10 percent. A ‘clean-contaminated wound’hasan classified as‘contaminated’. is surgery The woundafterabiliary between 1and2percent. The infectionrateina‘clean wound’is room. inrecovery Supplemental oxygen infections Increasing hospitalstaytodetectmore Avoid hypothermiaperioperatively sensitivity. organismand thecausative knowing before Antibiotics shouldnotbestarted inallSSIs. Antibiotics aremandatory y pes ofwounds Streptococcus isalways 20. Which of the following statements Which ofthefollowing 20. ➜ statements Whichofthefollowing 19. ➜

AIDS infections Antibiotics insurgical B A E D C B A E D C E D C regarding antibiotics in surgical regarding antibioticsinsurgical regarding AIDS aretrue? regarding AIDS infections aretrue? the treatmentof‘residentopportunists’. Rotating antibioticsmayberequiredin replacement forsurgicaldrainage. Antibiotics canbeusedasa infections. Staphylococcus aureus while treatingmethicillin-resistant A ‘broad-spectrum’approachisused 25–35 percentofpatients. isseenin infectiontoAIDS HIV Within 2years,progressionof protease synthesis. transcriptaseand inhibiting reverse actby Most antiviraldrugs(HAART) the stageofseroconversion. during transmissionriskislow The HIV (GALT) isnotaffected. The gut-associatedlymphoidtissue receptors. thevirusbindstoCD4 After exposure, and MRSA Vancomycin iseffectiveagainstboth more than1week. aminoglycoside therapyiscontinuedfor shouldbemonitoredif Serum levels Streptococcus faecalis Cephalosporins arenoteffectiveagainst infections. community-acquired staphylococcal isusefulintreating Flucloxacillin Tetracycline isabactericidalantibiotic. Clostridium difficile . (MRSA) (MRSA) . ➜ ➜ ➜ Extended matchingquestions Extended Chooseandmatchthecorrectorganismwitheachofdescriptionsbelow: Chooseandmatchthecorrectsystemswitheachofantibioticsbelow:

D C B A 3. Specificinfections 7 6 5 4 3 2 1 G F E D C B A 2. Bacteriainvolvedinsurgicalinfection 4 3 2 1 D C B A 1. Proph Tetanus Pseudomembranous colitis Necrotising fasciitis gangrene Gas cancauseepidemics. infection. Resistantstrains(MRSA) Gram-positive aerobiccoccus,whichformsgrape-likeclumps;causeswound andprosthesis gangrene, tetanusandpseudomembranouscolitis. Gram-positive, obligateanaerobeswhichproducespores;causesserious infectionssuchasgas surgery.cause ofintra-abdominalinfectionafterbowel Non-lactose-fermenting Gram-negativebacilluswhichisanormalresident ofthecolonandisa release ofenzymes. Gram-positive cocciwhichformchains;causescellulitisandspreading tissue destructionby Non-spore-bearing anaerobesthatcolonisethecolon,vaginaandoropharynx. Hospitalstrainscanacquireresistancetransferredthroughplasmids. also caseUTI. Gram-negative bacillus,whichtendstocoloniseburnsandtracheostomywounds.Thesecan Lactose-fermenting Gram-negativebacillus,whichisthemostcommoncauseofUTI. Bacteroides. Pseudomonas Proteus E. coli Clostridium Staphylococcus Streptococcus Second-generation cephalosporin. isarisk withorwithoutgentamicin,vancomycinrifampicin,ifMRSA Flucloxacillin Broad-spectrum cephalosporin(withanti-staphylococcalaction)orgentamicinbeads Second-generation cephalosporin(orgentamicin)andmetronidazole Colorectal. Biliary Orthopaedic Vascular y lactic antibioticsregimen 25

4: SURGICAL INFECTION PRINCIPLES ➜ Choose the correct operation type foreachofthescenariosbelow: Choosethecorrectoperation type Chooseandmatchthecorrectdiagnosiswitheachofscenariosbelow: 26

8 7 6 5 4 3 2 1 D C B A 4. Different t 7 6 5 4 3 2 1 G F E Wound infectionrate<10 percent. percent Wound infectionrate<40 Wound infectionrate15–20 percent disease bowel orgrossspillage orinflammatory Open viscussurgery Wound infectionrate1–2 percent surgery andbiliary Gastric No viscusopened Drainage ofanabscess Dirty. Contaminated Clean-contaminated Clean over thepastcoupleofdays. The woundappearsredwithsomeseropurulentdischargeatoneend.Shehasbeenfebrile anappendicectomyforinflamedappendix4daysago. A 30-year-oldfemaleunderwent failure. youfindopisthotonusandrespiratory finding itdifficulttobreathe.Onexamination bygeneralisedmotorspasms.Heis andjawmovementsfollowed difficulty inswallowing 3weeksagowhilegardeningpresentswith A 45-year-oldmalewhosustainedminorinjury growth. mixedaerobicandanaerobic cellulitiswithcrepitus.Cultureswabreveals extensive reveals pain.Examination wall afteralaparotomyforperitonitis.Hecomplainsofsevere rapidlyspreadinginfectionoftheabdominal An immunocompromisedpatientdevelops withoedemaandcrepitus. sweet-smellingexudate thin,brown, reveals painoverhislegwound.Examination severe A youngsoldierinjuredincombatdevelops vaguely palpabletendermass. witha unwellandhassignsofperitonitisovertheLIF heisvery irritation. Onexamination andurinary increased significantlyoverthelastcoupleofdaysandisassociatedwithfever pain.Thishas (LIF) ofleftiliacfossa malepresentswithaweek-longhistory An 80-year-old abdominalpain. andlower presents withdiarrhoea,fever A 16-year-old boywhohadanappendicectomyforagangrenousappendix1weekago generaliseddistensionandtenderness. reveals unwellandinshock. heisvery diarrhoeaforthepast3days.Onexamination, with severe A 78-year-old nursinghomeresidentwhohasfinishedacourseofantibioticsrecentlypresents abscess. Diverticular Pelvic abscess Surgical woundinfection y pes ofoperationsandinfectionrates ➜ ➜ ➜ ➜ ➜ ➜ 6.A,C,E 5.A,D,E 4.A,C,E 3.A,C,D 2.A,C,E 1. A,C,E Answers: Multiplechoicequestions

Surgical siteinfections Secondar Risk factorsforwoundinfection Host resistance toinfection Natural barrierstoinfection Koch’s postulates wound infection which are particularly useful in surveillance and research(seeTable usefulinsurveillance wound infectionwhich areparticularly 4.1) of fortheseverity antibiotic prophylaxis. Therearescoringsystems(Southampton andASEPSIS) inauditortrialsof isimportant between majorandminorinfections andthedefinitionofSSI systemicsignsordelayinreturnhome.Thedifferentiation discomfort, associated withexcessive Itresultsindelayedreturnhome. anda raisedwhitecount(SIRS). such astachycardia,pyrexia proceduretodrainit.Thepatientmayhavesystemicsigns spontaneously orneedsasecondary isdefinedasawoundthateitherdischargessignificantquantitiesofpus A majorSSI (mostly associatedwithindwellingvascularcatheters)andSSIs. bacteraemia tractinfection (UTI), infections, includingventilator-associatedpneumonia,urinary is termedhealthcare-associatedinfection(HAI).Therearefourmaingroups ofHAI:respiratory or admissiontohospital surgery (e.g.anastomoticleak).Infectionthatfollows at oraftersurgery inadequate airfiltration)ortheward(e.g.poorhand-washingcompliance) orfromcontamination sourcesincludeinfectionsacquiredfromtheoperationtheatre(suchas or exogenous Secondary pepticulcer). include infectionacquiredfromacommunityorendogenoussource(e.g. perforated sources Theprimary (exogenous). andsecondary The sourcesofinfectionareclassifiedasprimary as deadspaceandhaematoma. due tosystemicshockorlocalischaemia,foreignbodymaterialandpoorsurgicaltechnique,such andtranslocationinthegut,poorperfusion steroids, chemotherapyandradiotherapy,colonisation metabolic conditionssuchasdiabetesandjaundice,immunosuppressionduetocancer,AIDS, The factorsthatincreasetheriskofwoundinfectionincludemalnutrition(obesityandweightloss), radiotherapy, chemotherapyandsteroids. The othercausesofreducedhostresistanceincludediabetes,uraemia,jaundice,obesity, polymorphonuclear cellsandkillerlymphocytes. cells, gastricpH,complements,opsonins,phagocytic infection. Theothernaturalbarriersarelow andtraumaincreasestheriskof bysurgery oftheintactepithelialsurface The breakdown the firstdefinitionofinfectiousdisease. heralded recovery. MicrobeshadbeenseenunderthemicroscopebutitwasKochwholaiddown ofinfection recognisedthatlocalisation such aswineandvinegar,toirrigatewounds.Galen years.TheHippocraticteachingsdescribedtheuseofantimicrobials, documented for4000–5000 andtraumahasbeen Surgical infectionhasalwaysbeenamajorcomplicationofsurgery A minorwoundinfectionmaydischarge pusorinfectedserousfluidbutshouldnotbe y infections 27

4: SURGICAL INFECTION PRINCIPLES ➜ ➜ ➜ ➜ ➜ 11. B,C,D 10. A,C,D,E 9.A,B,C,D,E 8.A,B,C,E 7. A,D,E 28

and rigors,arecommon.Bloodculturesoftennegative. chills,fever whicharecausedbyreleaseofproteases.Systemiculceration mayfollow, signs–SIRS, Clostridial woundinfections Severe sepsis(sepsiss S Cellulitis Daily0–5 Daily0–10 Points Daily0–5 Daily0–10 10 Abscesses 5 10/5/10 Stay asin-patientprolongedover14 daysasaresultofwoundinfection. Isolation ofbacteriafromwound Separation ofdeeptissues Purulent Erythema Serous discharge Additional treatment–antibiotics/drainage/debridement Criterion Table 4.1 widespread insoil andmanure.Thesignssymptoms aremediatedbythereleaseof the Clostridia areanaerobic,terminal,spore bearing,Gram-positivebacteria.Thesporesare ofoneormoreorganfailures. sepsisorsyndromeis withevidence Severe (> arehyperthermia components ofSIRS > 38°C), WCC 12 withadocumented infection.Theother Sepsis isdefinedasthesystemicmanifestationofSIRS haemolytic Spreading infectionpresentinginsurgicalpracticeistypicallycausedbyorganismssuchasbeta- intentionisencouraged. ofcellulitis.Healingbysecondary there isevidence guidedaspiration.Antibioticsareindicatediftheabscessisnotlocalised,i.e. techniques mayallow hospital. 7–10 presentafterthepatienthasleft daystoformaftersurgery. manyas75percentofSSIs As in fluid.Thisincreasespressureandcausespain.Mostabscessesrelatingtosurgicalwoundstake oedema andthecellsofacuteinflammation.Abscessescontainhyperosmolarmaterialthatdraws and responseandapyogenicmembranecomposedoffibrinousexudate acute inflammatory freeradicalsandothermolecules.Anabscessissurroundedbyan oxygen damaging cytokines, cause suppurationandnecrosis.Pusiscomposedofdeaddyingwhitebloodcellsthatrelease (pain) andtumour(swelling).To (lossoffunction).Pyogenicorganisms thisisaddedfunctiolaesa An abscesspresentsallfeaturesofacuteinflammation–calor(heat),rubor(redness),dolour implanted hipandkneesurgery. and1yearafter fora30-daypostoperativeperiodnon-prostheticsurgery include surveillance y Accurate surveillance can only be achieved usingtrained,unbiasedandblindedassessors.Most canonlybeachieved Accurate surveillance Abscess cavitiesneedcleaningout/curettageafterincisionanddrainage.Modernimaging stemic inflammator Streptococcus ASEPSIS woundscore , Staphylococcus y y response s ndrome) and C. perfringens y ndrome (SIRS) ϫ . Tissuedestruction,gangreneand 1000/dL. ➜ ➜ ➜ ➜ ➜ 17. B,C,E 16. A,C, E 15. A,C,E 14. A,B,D,E 13. A,B,D,E 12. B

Infection causingbacteria Different t Surgical woundinfection Proph Treatment ofsurgicalinfections considered forbothat-riskwoundsandestablishedinfection. Theuseofantitoxinusinghumanimmunoglobulinoughttobe treatment withbenzylpenicillin. established infection,minordebridementofthewoundmaybeneededalongwithantibiotic treatment,but,inan anterior horncells.Prophylaxiswithtetanustoxoidisthebestpreventive tetanospasmin,whichaffectsthemyoneuraljunctionsandmotorneuronsof exotoxin in prosthetic (vascular and orthopaedic) surgery andinindwellingvascularcatheters. surgery in prosthetic(vascularandorthopaedic) recognisedasamajorthreat butisnow Staphylococcal epidermidiswasregarded asacommensal Beta-haemolytic already associatedwithinfectionandhenceprophylaxisisnottheissue. woundare15–20in acontaminatedanddirty and< percent,respectively. woundis 40 Adirty antibiotics areeffectiveinclean-contaminatedandcontaminatedoperations. Theinfectionrates that prophylactic orincisionthroughanabscess).Thereisundisputedevidence perforation (pusor disease) anddirty spillage), contaminated(openviscuswithspillageorinflammatory Wounds areclassifiedintoclean(noviscusopened),clean-contaminated (viscusopened,minimal colonisation. wound enhancessuperficial thenightbefore,becauseminorskininjury doubled ifitisperformed maybe rateaftercleanwoundsurgery astheSSI should bedoneimmediatelybeforesurgery shaving adherenceofdressings.Ifitistobeundertaken, foraestheticreasonsortoprevent except Medical staffshouldalwayswashhandsbetweenpatients.Preoperativeshavingbeavoided andacquisitionsofHAI. theriskofacquiringMRSA preoperativehospitalstaylowers A short Intravenous administrationatinductionofanaesthesiaisoptimal. be presentatthetimewhichfirstincisionismadeandbeforecontaminationoccurs. should When usingantibioticsforprophylaxis,ideally,maximumbloodandtissuelevels andresistance. because oftherisksallergy but arenotindicatedforuseinwoundinfections.Theoftopicalantibioticsshouldbeavoided the woundopen.Polymeric filmsareusedasincisiondrapesandalsotocoversuturedwounds pustodrainadequately. toallow contaminatedwounds,itislogicaltoleave necessary, Inseverely ofsuppuration,suturesorclipsneedtoberemoved,withcurettageif there isclearevidence Ifaninfectedwoundisundertension,or until theorganismsandtheirsensitivitiesareknown. The choiceofantibioticsmayneedtobeempiricalinitiallyasitisillogicalwithhold y lactic antibiotics y Streptococcus pes ofwounds resides in the pharynx of5–10 residesinthepharynx percentofthepopulation. 29

4: SURGICAL INFECTION PRINCIPLES ➜ ➜ ➜ ➜ ➜ ➜ ➜ 1D,2A,3B,4A,5C,6C,7D,8B 1C,2F,3G,4A,5B,6D,7E 1D,2F,3G,4A,5E,6C,7B 1D,2B,3A,4C matchingquestions Answers: Extended 20. A,D,E 19. B,C,E 18. E 30

several bacteria, acting in synergy, mayberesponsiblefortheinfection. bacteria,actinginsynergy, several orwhenitissuspectedthat spectrum antibioticcanbeusedwhentheorganismisnotknown radiologicaldrainageoropenanddebridement. interventional is do notreplacesurgicaldrainageofinfection.TheappropriatetreatmentlocalisedSSIs choice beingempirical,andlatermodifieddependingonthemicrobiologicalfindings.Antibiotics 4. Different t 3. Specificinfections 2. Bacteriainvolvedinsurgicalinfection 1. Proph AIDS Antibiotics insurgicalinfections Principles ofantimicrobial treatment Antibiotics are needed to treat SSIs if there is evidence ofspreadinginfection,bacteraemiaor ifthereisevidence Antibiotics areneededtotreatSSIs transmission duringthephaseofseroconversion. early weeksofinfectiontheremaybeaflu-likeillnessandthepatientspresentgreatestrisk antibody productionanddelayedhypersensitivity. MacrophagesandGALT arealsoaffected.Inthe subsequent lossofCD4+cells,T-helper cellsandotherinvolvedincell-mediatedimmunity, thevirusbindstoCD4receptors.Thereisa IV drugaddictionandinfectedblood.Afterexposure, transmission, throughsexual virusisaretrovirusthathasbecomeincreasinglyprevalent The HIV ofaminoglycosidetherapy.taken 48hafterthestart immediatelybeforeand1hafterintramuscularadministrationmustbe bacteriostatic. Serumlevels are anderythromycin Penicillins andaminoglycosidesarebactericidal,whilsttetracycline or systemic complications(SIRS A narrow-spectrum antibiotic may be used to treat a known infection, e.g. MRSA. Abroad- infection,e.g.MRSA. antibioticmaybeusedtotreataknown Anarrow-spectrum y lactic antibioticsregimen y pes ofoperationsandinfectionrates MODS). Antibiotics should notbeheldbackifindicated,the 5 Tropical surgery

Multiple choice questions ➜ Ascariasis D The larvae are released in the lungs. 1. Which of the following are caused by The developing larvae are swallowed in Ascaris lumbricoides? sputum and complete their maturation in A Intestinal symptoms as a larva the intestine. B Pulmonary symptoms as a larva E Eggs are released into the environment C Intestinal symptoms as an adult worm in stool as cysts which survive. D Pulmonary symptoms primarily as an 5. How is Loeffler’s syndrome adult worm characterised? E Biliary disease as a larva. A Chest pain with productive cough the 2. What percentage of the world’s presence of friction rub and a chest X-ray population is affected by roundworms showing pleural effusion (Ascaris lumbricoides)? B Retrosternal burning, discomfort on A 2 per cent ingestion of food, fever and mediastinal B 10 per cent widening on chest X-ray C 25 per cent C Epigastric pain radiating to the right, a D 50 per cent tender palpable liver and an elevated E 75 per cent. right dome of the diaphragm D Chest pain, dry cough, dyspnoea and 3. How is roundworm disease fever with fluffy on chest transmitted? X-ray. A Inadvertent ingestion of the larva in soil B Inadvertent ingestion of the fertilised egg 6. Which of the following are caused by in soil roundworm infestation? C The larva penetrates unbroken skin A Malnutrition and failure to thrive D Inadvertent ingestion of fertilised eggs in B Mechanical obstruction of the intestine contaminated meat C Perforation of the intestine E The larva is transmitted to the lung as a D Ascending cholangitis, obstructive droplet infection. jaundice and acute pancreatitis E Chest symptoms 4. Which of the following are true of the F All of the above. life cycle of the Ascaris lumbricoides? A Eggs release larva in the lumen of the 7. Which of the following may be found intestine which then develop into adult in roundworm infestations? worms. A A high eosinophil count B Eggs release larva into the portal blood B Charcot–Leyden crystals in stool and thereafter into the liver and then the C Larvae in sputum or bronchoscopic lungs. lavage C Eggs penetrate the intestinal wall and D Fluffy exudates on chest X-rays travel to the liver where they develop E Barium meal may show the worm lying into adult worms and are released into freely in the lumen. the intestinal lumen via the biliary tract.

31 PRINCIPLES ➜ 10. Which of the following aretruewith Which ofthefollowing 10. arevalid Whichofthefollowing 8. . Apatientpresentswithsuddensevere 9. 32

H E D C B A bile duct. Figure 5.1 B A y high polymorphsandanobstructive elevated whitecellcount(WCC) with thathehasan His bloodtestsshows tender overtherightupperabdomen. rigors. Hehasicterusandisdiffusedly upper abdominalpainwithchillsand wormload. the presenceofaheavy in mayrequireexteriorisation perforation A long-standingsmallintestinal Gastrojejunostomy enemas saline subsequently treatedwithhypertonic large intestineatlaparotomy,tobe Kneading awormbolusintothe enemas saline andhypertonic saline Nasogastric suction,intravenous(IV) Urgent laparotomy intestinal obstruction? roundworm uncomplicated for thetreatmentofadiagnosed datid disease Dogs arethedefinitive host. Man isthedefinitive host. Echinococcus granulosus by regard tohydatiddiseasecaused Stones inthegallbladderandalinearshadowdilatedcommon ? E D C B A E D C The conditionshouldbetreatedwith worm intheCBD. The conditionislikelytobeduealive stones intheCBD. The mostlikelydiagnosisisalineof cholecystectomy. by of theroundwormfollowed may betreatedbyendoscopicremoval ananthelmintic,thecondition Following albendazole. The conditionshouldbetreatedwith kills wormsbycausingtetanicconvulsion. pyrantel palmoate,ananthelminticwhich following statementsaretrue? following to changeitsposition.Whichofthe which inrealtimeisfound the CBD in alinearshadow shows (MRCP) resonance cholangiopancreatography Themagnetic bile duct(CBD). inadilatedcommon a linearshadow stonesinthegallbladderand shows (Fig.5.1)jaundice. Theultrasound scan liver withoutanydefinite margin. The diseasecauses aninfiltrationofthe in theliverbutmayaffectanyorgan. lesions mainly The diseasecausescystic Sheep andcattleareintermediatehosts. 11. How ishydatiddiseasetransmitted? How 11. 13. Which of the following featuresmay Whichofthefollowing 13. statements Whichofthefollowing 12. Figure 5.2 E D C B A E D C B A E D C B A Features ofobstructivejaundice. worms instool Patients oftencomplainofpassinglive collapse aftertrivialtrauma Abdominal pain,anaphylacticshockand abdomen A dullachingpainintheupperright calledLoeffler’ssyndrome fluffy exudates coughwith Chest symptomsofadry hydatid diseasepresentwith? the intestinallymphatics. It spreadstotheliverbytravellingalong daughter cysts. The germinalenvelopemaygiveriseto parasites. a germinalmembranecontainingviable whichhas It containsaninnerendocyst which isnon-infective. It containsanintermediateectocyst spreading livingparasites. madeupof It containsanouterpericyst true? hydatiddiseaseare regarding cystic It spreadsbydropletinfection. It maybevector-borne unshod feetbylarvae Through penetrationoftheskin ingestion ofeggs By thefaeco-oralroutethrough affects onlynon-vegetarians By eatinginfectedmeatandittherefore CT scanofschoolgirl withadullpaininherabdomen. 5 Ayounggirlwasdancinginschool 15. features Whichultrasound scan 14. E D C B A E D C B A diffuse skinrash andstartedherona In theeveningmothernoticeda and feltadullpainintheabdomen. withsplitwalls. A cyst rim ofoedema A solidlesionwithasurrounding wallwithcalcifications A cyst outlines andirregular wallwithshaggy A cyst withmultipleseptations A cyst characterise ahepatichydatidcyst? laparotomy. Anaphylaxis canoccurduring albendazole. laparotomy undercoverof Treatment iswithexploratory Treatment iswithmasterlyinactivity. hydatid cyst. The featuresuggestsaruptured oftheliver.ruptured simplecyst The featureiscompatiblewitha scenario? this clinical aretrueof Which ofthefollowing (Fig.5.2). isshown and aCTscan ahigheosinophilcount tests showed lesionwithasplitwall.Blood cystic a whichshowed an ultrasound scan, intensity. Aweeklatersheunderwent responded andthepainreducedin course ofantihistaminestowhichshe 33

5: TROPICAL SURGERY PRINCIPLES 16. A young woman presents with Ayoungwomanpresentswith 16. 34 with echogenicmaterialandaseptatedc Figure 5.3 Figure 5.4 echogenic material and a septated cyst echogenic materialandaseptatedcyst with adilatedCBD bladder calculi, gall (Fig.5.3)shows ultrasound scan tender overtheupperabdomen.An coloured urine.Sheisjaundicedand byhigh- chills andrigorsfollowed pain inherupperabdomenwith Ultrasound showinggallbladdercalculi,adilatedcommonbileduct CT scanshowingaseptatedc y st intheleftlobeofliver. y st oftheleftlobeliver. following aretrueofthiscondition? following sphincterotomy. Whichofthe structures weredeliveredafter multiplemembranous (ERCP), retrograde cholangiopancreatography of theliver(Fig.5.4).Atendoscopic oftheleftlobe aseptatedcyst shows in theleftlobeofliver. ACTscan ➜ 18. Which of the following aretrueof Whichofthefollowing 18. 17. statementsare Whichofthefollowing

Poliom E D C B A E D C B A E D C B A flaccid paralysis. motorneurontypeof It causesalower cranially. losswhichspreads It causessensory spinal cord. It targetstheanteriorhorncellsof faeco-oral route. It isarotavirusthatspreadsbythe by inhalationoringestion. It isanenteroviralinfectionthatspreads dissemination ofthedisease. maycause Rupture ofacyst mandatory. wallis Surgical curettageofthecyst watched andtreatedwithalbendazole. calcification intheirwallsmaybe whichshow Small deepcysts into multiloculatedhydatidcysts. injectionofscolicidalsolutions following Sclerosing cholangitisismorecommon cysts. than inthetreatmentofmultiloculated treatment ofunilocularhydatidcyst Anaphylactic shockiscommonerinthe true ofhydatidcysts? communications. presence ofbiliary sclerosing cholangitisbecauseofthe scolicidal solutionshasariskofcausing Treatment byinjectionof ofthecyst intothecyst. of sclerosants byinjection followed cholecystectomy shouldbewitha clearance oftheCBD Treatment endoscopic following require acholecystectomy. shouldonly clearance oftheCBD Treatment endoscopic following sand. and gallbladdercalculi,duetohydatid intheCBD causing daughtercysts communication, withbiliary hydatid cyst The likelydiagnosisisamultiseptated liver cyst. choledocholithiasis withanincidental and The likelydiagnosisischolecystitis poliomyelitis? y elitis ➜ 19. Which of the following types of types Whichofthefollowing 19. 1 A24-year-old femalefromapoor 21. Whatpresentingfeaturesdopatients 20.

intestine ofthesmall E D C B A E D C B A E D C B A tuberculosis infection with byintestinal infection maybecaused terminal ileum Hyperplasia andthickeningofthe Apple-core lesionsofthecolon intestine Tubercles aspectofthe ontheserosal edges intheileum Transverse ulcerswithundermined a pulledupsubhepatic caecum. oftheterminalileumwith narrowing a The bariummealX-rayshows a normallyplacedcaecum oftheterminalileumwith narrowing a The bariummealX-rayshows Skip lesionsoftheintestine lungs Presence ofactivetuberculosisthe a poorsocioeconomicbackground from country Residence inadeveloping tuberculosis? make youconsideradiagnosisof should Which ofthefollowing out(Fig.5.5). meal X-ray iscarried with abdominaldistension.Abarium repeated attacksofabdominalpain presents with developing country socioeconomic backgroundina onhistology. Characteristically, anon-caseating discharge undermined edgesandawatery A watering-canperineumwith A massintherightiliacfossa area oflocalisedascites A doughyfeeloftheabdomenfrom riseoftemperature evening Weight loss,vagueabdominalpainand intestine show? with tuberculosisofthesmall propensity forfistulaformation. Transmural inflammationwitha ? Mycobacterium 35

5: TROPICAL SURGERY PRINCIPLES 22. Surgery for ileal and ileocaecal forilealandileocaecal Surgery 22. 23. Which of the following aretruein Whichofthefollowing 23. 36 attacks ofabdominalpainwithdistension. Figure 5.5 E D C B A E D C B A conservatively. It shouldalwaysbe treated chemotherapy. after completionofantituberculous continuity by restorationofbowel asafirststep,followed exteriorisation It maybetreatedbyresectionand perforation. bystricturoplastythroughthe followed It maybetreatedbyresuscitation, resection oftheaffectedsegment. by It istreatedbyresuscitationfollowed present. Free gasunderthediaphragmisalways tuberculous perforation? All oftheabove. Ileo-transverse anastomosis Right hemicolectomy Limited ileocolicresection Stricturoplasty tuberculosis mayinvolve: Barium mealX-ra y ina24- y ear-old femalewhopresents withrepeated ➜ 24. Which of the following aretrueof Which ofthefollowing 24. 25.

Amoebiasis A E D C B amoebiasis? occurs? water orfood.Which ofthefollowing contaminating the bodyascysts organism behindamoebiasis,enters Entamoeba histolytica contaminated foodandwater. The organismentersthegutvia hygiene orsanitation ofstandards there isabreakdown oral route,thediseaseoccurringwhere The modeofinfectionisviathefaeco- affected. percentofpeople in morethan50 manifestation,occurs extraintestinal Amoebic liverabscess,thecommonest some kind. The majorityproducesymptomsof and SouthAmerica. ofCentral subcontinent, Africaandparts The diseaseiscommonintheIndian , the causative , thecausative 27. Which of the following arepresenting Whichofthefollowing 27. 6 Analcoholic,middle-agedman 26. right abdomen. Figure 5.6 C B A E D C B A seen inthecaecum granuloma. Thisis mostcommonly due totheformation ofachronic An apple-corelesiononbariumenema cough painful dry right shoulder-tippain,hiccoughsand a Pain intheupperrightabdomenwith abdominalpain with colicky Chronic diarrhoea,oftenbloodstained, He hasatenderhepatomegalywith toxic, anaemicandmildlyjaundiced. and weightloss.Heisfoundtobe sweats, anorexia,malaise,cough abdomen. Hehashadfever,night presents withpainintheupperright percentandtherestaremultiple. 80 in 10 percentofcases,theleftin The rightlobeoftheliverisaffected faeces asaninfectiveform. Cysts mayformwhicharepassedinthe and liquefactivenecrosisintheliver. circulation,causingfocalinfarction portal the colonandpasstolivervia The trophozoitesmayinvadethewallof the colon. causeflask-shapedulcersin The cysts colonise thecolon. and hatchinthelargebowel The cysts features ofamoebiasis? CT scanofanalcoholic,middle-agedmanwithpainintheupper 8 Whatisthetreatment foranamoebic 28. E D C B A C B A E D jelly-like fluid. The abscesscavitycontainsblackcurrant amoebic liverabscess. scaniscompatiblewithan The CT diarrhoea mayalsohavebeengiven. ofbloody endemic areaandahistory suggestive ofamoebiasis,traveltoan While theclinicalfeaturesareall furoate issufficient Medical treatmentwithdiloxanide or tinidazole Medical treatmentwithmetronidazole Urgent laparotomyupondiagnosis abscess? All oftheabove. Features ofperitonitiswithshock cases. of theabscesscavityinaminority Trophozoites maybefoundinthewall infected. sterile butbecomessmellyifsecondarily The collectionisnormallyodourlessand the following statementsaretrue? the following (Fig.5.6).Whichof isshown CT scan in therightlobeofliverandhis ahypoechoiclesion hadshown scan intercostal spaces.Anultrasound tenderness overtherightlower 37

5: TROPICAL SURGERY PRINCIPLES ➜ 29. Asiatic cholangiohepatitis, also called cholangiohepatitis,alsocalled Asiatic 29. 30. What is the treatment of Asiatic What isthetreatmentofAsiatic 30. 38

Asiatic cholangiohepatitis E D C B A E D C B A E D to thedisease? pertain fluke. Whichofthefollowing All oftheabove. and treatedinitssubclinicalform. The diseaseshouldideallybediagnosed performed. with orwithoutanaccessloopmay be A Roux-en-Y choledochojejunostomy may bedone. duct andcholedochoduodenostomy ofthebile exploration Cholecystectomy, drugs ofchoice. Praziquantel andalbendazolearethe cholangitis? dilatation ofthecommonductsystem. intrahepatic ductswithonlyminimal uniform dilatationofsmallperipheral Ultrasound scancharacteristicallyshows may leadtocholangiocarcinoma. channels oftencausedysplasia,which Changes aroundtheintrahepaticbiliary system. nidus forstoneformationinthebiliary The eggsordeadwormsmayformthe periductal fibrosis. causing epithelialhyperplasiaand channel, worm intheintrahepaticbiliary The parasitematuresintoanadult who arethedefinitivehost. ingestion ofinfectedfishandsnails, act astheintermediatehost,by The diseaseoccursinhumans,who carcinoma cannotbeexcluded. responding totreatmentorwhere suspected amoeboma,whichisnot and toxicmegacolonfora haemorrhage for patientswithsevere indicated forarupturedliverabscess, Surgical treatmentforamoebiasisis of imminentrupture abscesses whichpresentwithfeatures maybecarriedoutfor Aspiration system by from infestationofthehepatobiliary ‘Oriental cholangiohepatitis’,results Clonorchis sinensis , aliver ➜ ➜ 31. Which of the following statements Whichofthefollowing 31. 33. A 33-year-old femalecomestothe 33. 32. Which of the following statements Which ofthefollowing 32.

Filariasis Lepros C B A E D C B A E D about filariasisaretrue? oculi causing exposurekeratitis and oculi causing nose, paralysis of theleftorbicularis bridge andliftingofthetip collapseofthenasal of eyebrows, facialfeatures:loss with thefollowing outpatients departmentofahospital blockage, resultinginmassivelimb blockage, The adultwormscauselymphatic intoadultworms. hatch andgrow matured eggsenterthecirculationto Once inoculatedbyamosquitobite,the causes thedisease. parasite some 10 percentofsufferers,the Brugia malayi It ismainlycausedbytheparasite limb. operations toreducethesizeof Elephantiasis canbeeasilytreatedwith skinmayneedtrimming. but excess Hydroceles aretreatedintheusualway when hugeelephantiasisoccurs. diethylcarbamazepine iseffectiveeven Medical treatmentwith nocturnal peripheralbloodsmear. Immature wormscanbeseenin count. lymphocyte anelevated Blood testsoftenreveal about filariasisaretrue? eosinophilia. coughiscalledtropicalpulmonary dry tractandpresentswith the respiratory A mildformofthediseasewhichaffects seen infilariasis. cancers blockinglymphaticsarenever Chyluria andchylousascitesseenin to elephantiasis. leading lymphatic channelsandfurther leading toadditionalfibrosisofthe infection, by streptococcalsecondary oedema. Thisisoftencompounded y Wuchereria bancrofti and Brugia timori also . In 34. Which of the following statements Which ofthefollowing 34. Figure 5.7 E D C B A C B A orthopaedic surgeon. orthopaedic ophthalm disease specialist,plasticsurgeon, teamofinfection multidisciplinary The patientistreatedbya demonstrating thebacilliinurine. The conditionisdiagnosedby ‘leonine facies’(lookinglikealion). as facies thusproducedisknown lepromatous variety. Thecharacteristic patient’s faceintheacutephaseof There areoftennodularlesionsonthe roughsurface. edges andadry maculeswithelevated erythematous anaesthetic, hypopigmentedor which areasymmetrical,well-defined, Patients maypresentwith‘leprids’, whicharetender.nerves, characterised bythickeningofthe Patients haveneuralinvolvement into twogroups,lepromatousand The diseaseisbroadlyclassified contact. The diseaseistransmittedbysexual bacillus, The diseaseiscausedbytheacid-fast disease? featuresarerelevanttothe following of leprosyismade.Whichthe blindness (Fig.5.7).Adiagnosis disease) aretrue? Hansen’s about leprosy(alsocalled Mycobacterium tuberculosis ologist, handand A 33- y ear-old womanpresenting withlepros . 35. Tropical chronicpancreatitisisa 35. ➜

Tropical chronic pancreatitis E D D C B A condition aretrue? statements aboutthe of thefollowing strata indevelopingcountries.Which generation frompoorsocioeconomic disease whichaffectstheyounger anaesthesia ofthehandsandfeet. resulting fromeffectssuchas leprosy oritsreactions,andsecondary, whicharecausedby into primary, The deformitiesproducedaredivided and peripheralnerves. tract affects theskin,upperrespiratory progressiveand The diseaseisslowly the formofdiscrete stonesonstraight pancreaticcalcificationin Patients show diabetes mellitus. calcium carbonatestonesandtype I pancreatic periductalfibrosis,intraductal Patients presentwithextensive It iscausedbyalcoholism. toxicityandthedisease. cyanogen detoxified intheliver,leadingto frombeing thecyanide prevents containing aminoacidinthediet concurrent absenceofsulphur- The contains derivativesofcyanide. (tapioca), arootvegetable,which It iscausedbyingestionofcassava response ofthepatienttodisease. tuberculoid, dependingontheimmune y . 39

5: TROPICAL SURGERY PRINCIPLES ➜ ➜ matching questions Extended Typhoid by feveriscaused 36. Choose and matchthecorrectdiagnosiswitheachofscenarios below: 37. How is typhoid diagnosed? istyphoid How 37. 40

1 E D C B A 1. Upperabdominalpain T B A E D C B A E y phoid severe upperabdominal painradiatingtothebackfor last 5months.Hehasalsonoticed severe A 35-year-oldmalepatientresident ofsouthernIndiapresentswithcomplaintsepisodic Myocardial infarction Tropical chronicpancreatitis Lobar pneumonia Typhoid fever Amoebiasis blood cells. looks forthepresenceofbacteriainred Thetest become obsoleteintheUK. the Indiansubcontinent,althoughithas The Widaltestisoftencarriedoutin stool cultures. It isdiagnosedbypositivebloodand with featuresofperitonitisandshock. due tomelaenaandhypovolaemiaor Patients maypresentasanemergency ileus. distension duetoparalytic to anendemicarea,withabdominal for2–3weeksafteravisit high fever A typicalpatientmaypresentwith the ulcersmaybleedorperforate. If leftuntreatedorinadequatelytreated, and ulceration. causing hyperplasiaandlaternecrosis patches intheterminalileum,initially The organismcolonisesthePeter’s infected bloodorcontaminatedneedles. The bacteriaenterthebodythrough Some requiresurgery. andtreatmentforpain. insufficiency andendocrinepancreatic exocrine for Patients needmedicalsupport disease. scanning ofthepancreasconfirm abdominal X-ray. UltrasoundandCT following aretrueofthedisease? following negative bacillus.Whichofthe Salmonella typhi , whichisaGram- 38. What is the treatment for typhoid What isthetreatment fortyphoid 38. E D C B A E D C in thesecondorthirdweekof Abdominal distensionintyphoiddisease unless provedotherwise. be consideredtohaveaperforation accompanied byabdominalpainshould signsofdeterioration who shows In thesecondorthirdweek,anypatient detect bothIgGandIgM. immunoglobulin MandTyphiDot to immunoglobulin G,Tubex todetect such asMulti-Test Dip-S-Ticks todetect often negative,specialkitsmaybeused, out treatment,wherebloodculturesare In thescenarioofinadequatelycarried immunological statusisbetter. as thepatient’snutritionaland hasabetterprognosis of typhoidfever occurringinthefirstweek Perforation leading towounddehiscence. happens, as woundinfectioninevitably often keptopenfordelayedclosure, The skinandsubcutaneoustissueare peritoneal soiling. andtheamountof of perforations of thepatient,siteandnumber out, dependingonthecondition surgicaloptionscanbecarried Several Chloramphenicol isthedrugofchoice. antibiotics inanintensivecareunit. resuscitation withIVfluidsand Treatment involvesvigorous perforation? washout togetridofthetoxins. should betreatedbyhigh-bowel fever ➜ Chooseandmatchthecorrectdiagnosiswith eachofthescenariosbelow:

2 1 E D C B A 2. Lumpintheabdomen 5 4 3 2 abdomen, whichis tendertotouch.Thepainisaggravated withmovementandcoughing. general malaiseforthelast5days. He hasalsonoticedthepresenceofaswellinginupper withnightsweatsand pain intherightupperabdomenassociated withhigh-gradefever, A 51-year-old graduallyincreasing malepatient,residing inBangladesh,presentswithanorexia, count(DLC) eosinophilsconstituting12 showing withadifferentialleucocyte percent. 8600 count(TLC) of atotalleucocyte A bloodcountreveals finding intheabdominalexamination. tenderonpalpation.Thereisno othersignificant enlargement oftheliverwhichissomewhat healthy. firm reveals abdomen overthelast3months.Heisotherwise A 45-year-oldsheepfarmercomplainsofagraduallyenlargingpainful mass intherightupper Acute cholecystitis Hydatidosis Hyperplastic ileocaecaltuberculosis Acute typhoidfever Amoebic liverabscess normal breathsounds. thepresenceof reveals examination soft onpalpationandthereisnotenderness.Respiratory is110/60 andregular,bloodpressure(BP) the pulserateis80/min mmHg.Abdomenis ofoccasionalheavinessalongtheinnersideleftarm.Onexamination, and hasahistory also associatedsweatingandafeelingofweakness.Thepatienthasbeensmokerfor20years ofbreath.Thereis 3 hours,associatedwithheavinessintheleftsideofchestandshortness A 55-year-oldmalepresentswithcomplaintsofpainintheuppercentralabdomenforlast reaching theumbilicus. thepresenceofafirmsplenomegalyalmost reveals and febrile.Abdominalexamination sheisdehydrated ontakingrest.Onexamination, the upperleftabdomenthatgetsrelieved ofhercolleagueshavealsobeenunwell.Shecomplainsadraggingpainin that several diarrhoea forthelast10 days.Sheisaresidentofworkingwomen’shostelandcomplains withchillsforthelast2weeks,and ofhigh-gradefever A 25-year-oldfemalegivesahistory each respiration.Theabdomenisnormalonexamination. zoneandacrunchingsoundwith with thepresenceofbronchialbreathingoverleftlower rateof18/min, arespiratory upper abdomenandhigh-gradefever.reveals Theexamination ofbreath,painovertheleft sputum,shortness coughproductiveofyellow-coloured reports smokingcomplainsoffeelingunwellforaweek.He ofheavy malewithahistory A 50-year-old in theright5th,6thand7thspaces.Theliverspanisincreasedto20cmonpercussion. thecostalmargininrightmidclavicularline,andthereisintercostaltenderness 5 cmbelow bloody diarrhoeaabout3–4weekspreviously. thereistenderhepatomegaly Onexamination ofhavingsufferedfrom day andisaggravatedwithmovementcoughing.Hegivesahistory 5 days.Thepainhasbeenconstantandmainlyintherightupperabdomensinceprevious withnightsweatsandgeneralmalaiseforthelast abdomen associatedwithhigh-gradefever graduallyincreasingpainintheupper A 45-year-oldmalepatientpresentswithanorexia, acrossthespine. upper abdomenextending is180blood sugarlevel linearcalcificationinthe mg/dLandplainX-rayoftheabdomenshows withnopalpableabnormality. theabdomenisunremarkable diet. Onexamination, Hisfasting palestools.Heconsumestapiocaasastaple ofbulky increased thirstwithpolyuriaandpassage 41

5: TROPICAL SURGERY PRINCIPLES ➜ Chooseandmatchthecorrectdiagnosiswith eachofthescenariosbelow: 42

3 2 1 D C B A 3. Vomiting 5 4 3 diarrhoea forthe past 10 days.Inthe last2daysthecolourofstools hasbecomedarker, A 17-year-old withchillsforthepast2 weeks,and of high-gradefever boygivesahistory of centralabdominalairfluidlevels. thepresence freefluidintheabdomen. AnerectX-rayoftheabdomenreveals clinically evident sounds aremarkedlydiminishedandthereisno loops.Thebowel fullness withvisiblebowel thepatient ispaleanddehydrated,theabdomenhascentral bout ofcolic.Onexamination, ofprojectilebiliousvomitingassociatedwitha thelast2weekstherehasbeenahistory Over ofdiarrhoeaalternatingwithconstipation duringthisperiod. documented. Thereisahistory central abdominalpainoverthepast12 months,associatedwith weightlossthatisnot ofoff-and-on colicky presentswithhistory A 25-year-oldmalepatient,residinginSriLanka, of asuccussionsplashonauscultation. 100/60 mmHg.Thereisdistensionintheepigastrium andumbilicalregionswithpresence thepatientisdehydratedandhasapulserateof100/minOn examination, of andaBP ofepisodesupperabdominalgnawing paininthepast. after vomiting.Hegiveshistory painthat arerelieved upperabdominalcolicky The vomitingisassociatedwithboutsofsevere andisfoul-smelling. past week.Thevomituscontainsresidueoffooditemseatenpreviously ofprojectilenon-bilious vomitingforthe malesmokerpresentswithhistory A 40-year-old outletobstruction–chronicduodenalulcer Gastric Typhoid enteritis Ascariasis Subacute intestinalobstruction–Koch’s terminal ileum. regionthatistender.palpable inthesame stringsignof reveals Abariummealfollow-through andonpalpation,afirmlumpis and theabdomenhasaslightfullnessinrightiliacfossa, thepatientispale, ailment.Onexamination, ofaprolongedrespiratory 2 yearspreviously this period.Shehasalsohadirregularmenstruationoverthepast4–5months.Herfatherdied ofdiarrhoeaalternatingwithconstipationover pain forthepast6months.Thereisahistory A 19-year-old centralabdominal ofoff-and-oncolicky femalepatientpresentswithahistory reaching theumbilicus. thepresenceofafirmsplenomegaly,almost reveals and febrile.Abdominalexamination sheisdehydrated byrest.Onexamination, intheupperleftabdomenthatisrelieved sensation ofhercolleagueshavealsobeenunwell.Shecomplainsadragging that several diarrhoea overthepast10 days.Sheisaresidentofworkingwomen’shostelandcomplains withchillsforthepast2weeks,and ofahigh-gradefever A 25-year-oldfemalegivesahistory The restoftheabdomenisnormal. region. inthemidclavicularlineonrightside,andalumpispalpablesame thereistendernessonpalpationatthetipofninthcostal upper abdomen.Onexamination intheright ofintolerancetofattyfoodsinthepast,associatedwithslightdiscomfort a history radiating tothebackonrightsideandisassociatedwithnauseavomiting.Shegives ontakinganalgesics.Thepainisalso relieved inintensityandonlypartially which issevere night, A 45-year-oldfemalecomplainsofpainintherightupperabdomensinceprevious liver spanisincreasedto20cmonpercussion. midclavicular line,andthereisintercostaltendernessintheright5th,6th7thspaces.The thecostalmargininright thereistenderhepatomegaly5cmbelow On examination ➜ ➜ Chooseandmatchthecorrectdiagnosiswitheachofscenariosbelow:

C B A 5. Acuteabdomen 4 3 2 1 E D C B A 4. Abdominaldistension 4 Typhoid perforation Ruptured liverabscess Appendicular perforation 12 percenteosinophilsindifferential count. HisHbis10.2 soundsareexaggerated. loops.Thebowel palpable bowel g/dLandthereare forhisage.Theabdomen isdistendedwith the boyisfoundtobepaleandunderweight ofwormsperrectuminthepast. Onexamination, of failuretothriveandalsothepassage episodesofvomiting.Thereisahistory abdominalpainassociatedwithseveral colicky severe A 10-year-old boyisbroughttothehospitalbyhismother withcomplaintsofasuddenonset distributedmainlyinthecentralabdomen. multiple air-fluidlevels thepresence of andhigh-pitched.AbdominalX-raysshow soundsareexaggerated The bowel has centraldistensionthatisresonantonpercussion.Theabdomen tense butnon-tender. theabdomen anappendicectomyasachild.Onexamination the onsetofpain.Heunderwent since is associatedwithvomitingandabdominaldistension.Hehasalsonot movedhisbowels painforthepast 8hours.It centralabdominalcolicky A 35-year-oldmanpresentswithsevere filled structuresspreadallovertheabdominalviscera. constituting 12 thepresenceofmultiplefluid- shows percent.Anabdominalultrasound(US) withaDLC aTLC Abloodcountreveals eosinophils showing of8600 abdominal examination. tender onpalpationanddullpercussion.Thereisnoothersignificantfindinginthe firmdistensionoftheabdomenwhichissomewhat reveals complaints. Physicalexamination healthyandhasnobowel the last6monthsthathasbecomepainfuloflate.Heisotherwise ofadog-breedingfarmcomplainsgradualabdominaldistensionover A 45-year-oldowner arewithinnormallimits. levels electrolyte andtheserum ofthesmallbowel alloverthelength the presenceofmultipleair-fluidlevels sounds.AbdominalX-raysshow is worriedastheabdomenseemstobefull,withnobowel 2dayspreviously.malignancy Thehousesurgeon,afterhavingseenthepatientinward, retroperitoneallymphnodedissectionfortesticular A 30-year-oldmanhadanextensive Adhesive intestinalobstruction ileus Paralytic Ascariasis Multiple hydatidosis Acute intestinalobstruction haemoglobin (Hb)is10.2 g /dL andthereare10 percenteosinophilsindifferentialcount. isnormal.His forhisage.Theabdominalexamination is foundtobepaleandunderweight theboy Onexamination, ontheirown. abdominalpainthatarerelieved episodes ofcolicky of thrive. Theboy’sheightismuchlessthanthatofhisclassmates.Therealsoahistory A 13-year-old boyisbroughttothehospitalbyhismotherwithcomplaintsoffailure abdominal tenderness. thecostalmargin.Thereisassociatedcentral the presenceofapalpablespleenjustbelow reveals heisdehydratedandfebrile.Abdominalexamination weeks.Onexamination, last few anddiarrhoeainthe ofhisfriendshavebeensufferingfromfever and complainsthatseveral and ontwooccasionstherewasfrankbloodinthestools.Hestudiesaboardingschool 43

5: TROPICAL SURGERY PRINCIPLES ➜ Chooseandmatchthecorrectdiagnosiswitheachofscenariosbelow: 44

E D C B A 6. Swollenleg 5 4 3 2 1 E D Cellulitis lymphoedema Primary Pelvic tumour– vein congestion Deep veinthrombosis(DVT) Filariasis presenceoffreefluidintheabdomen. reveals X-rays arenormal.AbdominalUS soundsareabsent.Total abdomen.Thebowel so inthelower countsareraised andabdominal has tachycardiaandtachypnoea.Thereisrigiditytendernessallover theabdomen,more thepatientisfebrile, dehydrated and Onexamination, isunremarkable. The menstrualhistory practitioner. inthelast2days. Thereisassociatedvomiting,loosestoolsandhigh-gradefever onsome drugsbyaprivate the painwasbetterforacoupleofdaysaftershestarted that andgeneralizedinthelast24h.She says 4–5 daysthathas,oflate,becomesevere abdomen duringthepast A 20-year-oldmarriedfemalepresentswithpainintherightlower essentially normal. offreefluidintheperitonealcavity.. Thereisevidence The abdominalX-raysare thecostalmargin.Theliverspanisincreasedto20cmon is tenderhepatomegaly5cmbelow ofhavingsufferedfromdiarrhoeaabout3–4weekspreviously.there history Onexamination 5 days.Thepainsuddenlyworsenedandbecamegeneralizedoverthelastday. Hegivesa withnightsweatsandgeneralmalaiseforthepast abdomen associatedwithhigh-gradefever, graduallyincreasingpainintheupper A 45-year-oldmalepatientpresentswithanorexia, the diaphragm. freegasunder abdomen withthepresenceoffreefluid.AnerectX-rayreveals sounds.Thereisrigidityalloverthe and theabdomenisdistendedwithabsenceofbowel thepatientispaleanddehydrated, tuberculosisintherecentpast.Onexamination, pulmonary ofabsoluteconstipationoverthelast2days.Herhusbandsufferedfrom is alsoahistory progressively.the lastweekpainhasbeencontinuousandisincreasinginseverity There ofdiarrhoeaalternatingwithconstipationduringthisperiod.In 3 months.Thereisahistory pain forthepast10 months,associatedwithweightlossandamenorrhoeaoverthelast centralabdominal ofoff-and-oncolicky A 30-year-oldfemalepatientpresentswithahistory under thediaphragm. freegas rigidity intheabdomenandreboundtenderness.Anerectfilmofreveals the patientisdehydrated,hasapulserateof100/min of100/60 andaBP mmHg.Thereis ofepisodesupperabdominalgnawingpaininthepast.Onexamination, and givesahistory the upperabdomenandlaterbecamegeneralized.Hehasbeenachronicsmokerfor15 years in abdominalpainthatstarted ofsudden-onsetsevere A 30-year-oldmalepresentswithhistory under thediaphragmonerectabdominalradiograph. sounds.Thereisfreegas aboard-likerigiditywithabsentbowel reveals Abdominal examination dehydratedandfebrile. sheisseverely aswellgeneralized.Onexamination, became severe abdomenandlater inthelower ofabdominalpainforthepast4–5daysthatstarted history 3 weeks,anddiarrhoea10 whichlastedforabout1week.Shealsogivesa dayspreviously, withchillsoverthepast ofhigh-gradefever A 25-year-oldfemalepatientgivesahistory Tubercular peritonitis Duodenal ulcerperforation ➜ Chooseandmatchthecorrectdiagnosiswitheachofscenariosbelow: Chooseandmatchthecorrectdiagnosiswith eachofthescenariosbelow:

3 2 1 E D C B A 7. Scrotal swelling 5 4 3 2 1 over thepast6months. Itisassociatedwithpainless enlargement ofhisscrotumoverthe past limbs A 55-year-oldmale residentofBangladeshcomplains ofprogressiveswellinghislower the testisandreduceswithagurgle whenmanipulatedbythepatient. aninguinoscrotalswelling isseentobepresentwhichseparatefrom Onexamination, down. over thepast10 years. Thesizeoftheswellingincreasesonstraininganddecreases lying A 70-year-old malepatient complainsofprogressivelyincreasingswellinginhisrightscrotum the testisisnotfeltseparatelyfrom it. theswellingislimitedtoscrotum,transilluminant and scrotal trauma.Onexamination, of 2 years,whichhasbeenprogressivelyincreasinginsize.Itispainlessand thereisnohistory A 30-year-oldmalepatientcomplainsofswellingintherightsidehis scrotumoverthelast Varicocele Hernia Hydrocele failure Congestive heart Filariasis peripheral pulsesarepalpableandnormal. swollenandtender. Allthe pain andfever.thelimbiswarm,erythematous, Onexamination, progressiveswellingoverthefootandlegassociatedwith whichhedeveloped following ofasmallprickonhisrightfoot1weekago, A 45-year-olddiabeticpresentswithhistory raised andthedrainingnodesarenotenlarged. theknee.Thelocaltemperatureisnot swelling isnon-pittingandlimitedtothelegbelow the ortrauma.Onexamination, ofanyfever attained menarche1yearago.Thereisnohistory limbswellingof1year’sdurationthatisprogressiveinitscourse.Thegirl unilateral lower A 13-year-old girlisbroughtbyhermotherwithcomplaintsofinsidiousonsetpainless well ascompressionofthethigh. ofthefootas involving mainlythecalfregion.Thepainisaggravatedbypassivedorsiflexion thelocaltemperatureoflimbisraisedandswelling past 2weeks.Onexamination, pain associatedwithswellingintheoppositelegfor2days.Shehasbeenonbedrest forfracturedneckoffemurcomplains ofrecentsurgery A 70-year-old femalewithahistory presence ofmarkedeosinophilia. the reveals and rough.Theswellingofthelimbdoesnotpitwithpressure.Bloodexamination thick oftheaffectedlimbisfoundtobeincreasedandskinvery thegirth examination, 4 months,duringwhichthelegbecomesredandpainful,swellingalsoincreases.On overthelast over thelast6months.Hehasalsobeenhavingepisodesofhigh-gradefever maleresidentofBihar,India,complainsprogressiveswellinghisrightleg A 40-year-old tothefoot. fromthethighdown extending thepresenceofpittingoedema limbisswollenandshows Theleftlower is unremarkable. limitofwhichcannotbereached.Theresttheabdomen abdomen,thelower the leftlower thereisafirmtohardmassin oranytraumatothelimb.Onexamination, offever history limboverthelast2months.Thereisno also noticedheavinessandswellingoftheleftlower over thelast3months,associatedwithoccasionalinstancesofspottingpervaginum.Shehas abdomen inthelower womancomplainsofvaguediscomfort postmenopausal A 60-year-old 45

5: TROPICAL SURGERY PRINCIPLES ➜ Chooseandmatchthecorrectdiagnosiswitheachofscenariosbelow: 46

4 3 2 1 E D C B A 8. Clawhands 5 4 metacarpophalangeal, proximalanddistalinterphalangeal jointsoftheringfinger deformityinvolvingthe glands 2monthspreviously. thereisfixedflexion On examination left handoverthepast3months.He alsohadapainlessenlargementofboththeparotid deformityoftheringfingerhis malecomplainsof progressive flexion A 50-year-old lost hereyebrows. anaesthetic areasinvolvingboththe forearmsandhands.Incidentally,sheisalsofoundtohave ofthedigits.Therearemultiplehypoaestheticand non-healing ulcersoverthetipsofseveral thereisbilaterallysymmetricalclawingofallthedigitsbothhandswith On examination, ofanytrauma. ofthem.Thereisnohistory fingers andnon-healingulcersinvolvingseveral both herhandsoverthepast3–4years.Shealsocomplainsofnumbness ofthetipsher femaleresidentofMyanmar(Burma)complainsprogressivedeformityaffecting A 40-year-old over thepalmaraspectoflateralthreedigits. movements.There is alsohypoasthaesia ofthelateralthreedigitsisdecreasedinflexion power of thelateralthreedigitsrighthand.Thethenareminenceseems tobeflatandthe oftheinterphalangealjoints ofmetacarpophalangealjointsandflexion there ishyperextension glassjust abovethewristjoint.Therearenoothercomplaints.Onexamination, from window hesustained acutinjury three digitsoftherighthandoverpastmonth.Amonthpreviously ofweaknessanddeformitythelateral A 20-year-oldmalepatientpresentswithahistory deformity affectingthesetwodigits. inthemedialtwofingers.Thereisalsomild flexion is somenumbnessandweaknessofflexion there tingling andweaknessaffectingtheringlittlefingersofherlefthand.Onexamination 2 weeksaftertheoperationshecomplainstooperatingsurgeonthathasdeveloped A 47-year-old womanisadmittedtohospitalforamajorabdominalsurgery. Ondischarge contracture Dupuytren’s Poliomyelitis palsy Median nerve palsy Ulnar nerve Leprosy side. worms feeltothecordandtestisisalsosmallerflabbyonsame thantherightone.Thereisbagof theleftscrotumisfoundtobelower On examination, past year. oftrauma.Hehasbeenmarriedfor2yearsbutnochildren. Thereisnohistory inlefthemiscrotumoverthe recruitcomplainsofadraggingsensation A 23-year-oldmilitary ofthe scrotum. transilluminant andtheswellingdecreasesonelevation There istenderenlargementoftheliverwithfreefluidinperitonealcavity. Thescrotumis at thelungbases,jugularvenouspressure(JVP)israisedandhedyspnoeicrest. hehasbilateralfinecrepitations swelling ofbilateralscrotuminthelastweek.Onexamination, 2 weeks.Itisassociatedwithprogressivedistensionoftheabdomenoverpastweeksand complains ofsuddenonsetincreasedbreathlessnessandeasyfatiguabilityoverthepast patient disease(COPD) chronicobstructivepulmonary malewhoisaknown A 50-year-old fluctuant andthetestiscannotbepalpatedseparatefromit. thickandrough.Thescrotalswellingistransilluminant, to beincreasedandtheskinisvery oftheaffectedlimbisfound thegirth oftraumatothelegsorscrotum.Onexamination, history for3–4months.Thereisno 3 months.Hehasalsobeenhavingepisodesofhigh-gradefever ➜ ➜ Chooseandmatchthecorrectdiagnosiswitheachofscenariosbelow:

E D C B A 10. Paral 4 3 2 1 D C B A 9. Limbdeformit 5 Acute emboliclimb ischaemia Cerebrovascular accident Traumatic paraplegia Pott’s spine Poliomyelitis region. paravertebral a gibbusdeformityoftheupperlumbarspinewhichistenderwithfullness alongtheleft There isalso offullnessintheabdomenoverleftiliacfossa. left hipjointwithevidence deformity of the thereisflexion overthepast4–5months.Onexamination fever evening oflow-grade for 6–7monthswhichshedidnottakeanytreatment.Shealsohas history ofbackache at thehipjointforlast2weeks.Thedeformitywasprecededbya history limb A 30-year-oldfemalepresentscomplainingofaninabilitytostraighten herleftlower thepresenceofmarked eosinophilia. reveals structure ontheX-rayisnormal.Bloodexamination limb doesnotpitwithpressure.Theaffectedappearstobegreatly deformed,butthebony thickandrough. Theswellingofthe affected limbisfoundtobeincreasedandtheskinvery ofthe thegirth the past4months,duringwhichlegswellingincreases.Onexamination, over his rightlegoverthepast6months.Healsohasbeenhavingepisodesofhigh-gradefever maleresidentofBihar,India,complainsprogressiveswellinganddeformity A 40-year-old she isalsofoundtohavelosthereyebrows. hypoaesthetic andanaestheticareasinvolvingboththeforearmshands.Incidentally ofthedigits.Therearemultiple the handswithnon-healingulcersovertipsofseveral thereisbilaterallysymmetricalclawingofallthedigitsboth any trauma.Onexamination, of ofthem.Thereisnohistory tips ofherfingersandnon-healingulcersinvolvingseveral affecting bothherhandsoverthepast3–4years.Shealsocomplainsofnumbness femaleresidentofMyanmar(Burma)complainsprogressivedeformity A 40-year-old isnormal. examination areunelicitable.Thesensory talipes equines.Thedeeptendonreflexes limbwithflailknee,footdropand There isflaccidparalysisofthemusclesleftlower theboyismalnourished,withnoneckrigidity.and hasnotbeenimmunised.Onexamination, limb.Helivesinaremotevillage headacheandweaknessdeformityoftheleftlower severe with A 6-year-oldboyisbroughtbyhisparentswiththecomplaintsofsudden-onsetfever, Psoas abscess Leprosy Poliomyelitis Filariasis isnormal. examination apposition ofthethumb.Thesensory ofthefingersand accompanying flatteningofthethenareminencewithweaknessflexion andwrist,moreontheleftside.Thereisalso is weaknessofthemusclesshoulders, theboyismalnourished,withnoneckrigidity. Onexamination, There small villageinSriLanka. headacheandweaknessdeformitiesofboththeupperlimbs.Heisaresident severe with A 6-year-oldboyisbroughtbytheparentswithcomplaintsofsudden-onsetfever, arenormal. andmotorexaminations The sensory affected hand.Theskinofthepalmalsoappearstobethickerwithsubcutaneousnodules. y sis y 47

5: TROPICAL SURGERY PRINCIPLES ➜ 2. C 1. B,C Answers: Multiplechoicequestions Chooseandmatchthecorrectdiagnosiswitheachofscenariosbelow: 48

Ascariasis 5 4 3 2 1 symptoms. ascariasis intheirintestines.Thevast majorityareasymptomaticorhavetransientill-defined world,harbour Some 25percentoftheworld’spopulation, principallyinthedeveloping mature and causeintestinalsymptoms. maturation intheintestine.Inintestine they insputumandcompletetheir areswallowed chest X-raycalledLoeffler’ssyndrome. Thelarvae on cough,chest painandfluffyexudates to thebronchioles.Thiscausesinflammationwithadry penetrate intestinal wallandgetcarriedviathebloodstreamtolungs.In lung, thelarvae which penetratethe Theingestedeggsreleaselarva, ofascariasisisasfollows. The lifecycle there is complete loss of all sensations below T10 below there iscompletelossofallsensations level. ofT12, thelevel thoracolumbar spine.Thereisflaccidparalysisofallthemusclesbelow and upper halfofhisbodynormally. thereisastepdeformityatthejunctionof Onexamination, day.a heighttheprevious oflossconsciousness.Heisabletomovethe Thereisnohistory limbsafterfallingonhisbackfrom halfofhisbodyandthelower inability tomovethelower and ofsudden-onsetlosssensation A 25-year-oldmalepatientisbroughtwithahistory isnormal. examination areunelicitable.Thesensory deep tendonreflexes is flaccidparalysisofthemusclesbilateralhips,thighsandlegswithfootdrop.The thegirlismalnourished,withnoneckrigidity.received anyvaccines.Onexamination, There limbsoverthepast2weeks.Shehasnot headache andinabilitytomoveboththelower withsevere A 5-year-oldgirlisbroughtinbyherfatherwithcomplaintsofsudden-onsetfever, normal. facultiesare the left-sidedlimbsis3/5comparedwith5/5onrightside.Thesensory 14/15. Thereisnoneckrigidity. of arenormalandthemotorpower Allthecranialnerves is210/160 thepatientisconscious,BP trauma. Onexamination, ComaScoreis andGlasgow of over thepasthour. similarepisodesandthereisnohistory ofprevious Thereisnohistory limbs of sudden-onsetslurringspeechandaninabilitytomovehisleftupperlower isbroughttothehospitalwithahistory hypertensive, A 65-year-oldsmoker,whoisaknown and isbounding. isalso1/5The motorpower Inthislimbonlythefemoralpulseispalpable intheaffectedpart. is130/90.BP theknee. below Theaffectedlimbiscold,mottledandtherealossofsensation and ofmyocardialinfarction6monthspreviously.thepulseis60/min history Onexamination, and inabilitytomovehisleftlegfootforhalfanhour. cardiacpatientwith Heisaknown A 55-year-oldmalesmokerisadmittedwithcomplaintsofsudden-onsetpain,numbness withlossoflumbarlordosis. destruction oftheL2vertebra complete ofL2–L3.AnX-raythelumbarspineshows thelevel sensorimotor lossbelow regions.Thereisbilateral lumbar spinewhichistenderwithfullnessalongbothparavertebral thereisagibbusdeformityofthe overthepast4–5months.Onexamination fever evening oflow-grade 6–7 monthsforwhichshedidnottakeanytreatment.Shealsohasahistory for backache ofsevere limbsoverthepast2months.Thiswasprecededbyahistory lower progressiveweaknessofbothher A 30-year-oldfemalepresentswithcomplaintsofslowly ➜ 10. B,C,D 9.B,D,E 8.B,C,E 7. A,C,D,E 6. F 5. D 4.B,D 3. B

H fertilised eggsarepresentinthesoilcontaminatedwithinfectedfaeces.Faeco-oral contamination fertilised The conditions. Hotandhumidconditionsareideallysuitedfortheeggstoturnintoembryos. The eggofthe than bydirectinvasion. rather andspreadbyexpanding site ofinfection,but mayinfectanyorgan.Thelesions arecystic system.Therefore, theliveriscommonest worms spreadintothebloodstream viatheportal arethedefinitivehost. Theeggsinfectsheep,cattleandhumans. intheirintestine.They grow echinococcaldisease,infectsdogsand Echinococcus granulosus,theparasite whichcausescystic by anendoscope. theuseofalbendazole,ananthelmintic, thewormmaybeextricated in thiscondition.Following difficult.Therefore thisshouldnotbeused convulsion andmakessubsequentremovalextremely worm inthecommonbileduct.Pyrantelpalmoate,ananthelmintic,kills wormsbycausingtetanic The imaginginrealtimeisfoundtochangeitsposition.Itmorelikely tobeduearound- ofthebowel. inamalnourishedpatientmaysometimesideallyrequire exteriorisation perforation enemasissometimesawayoftreatingthecondition. Along-standing saline with hypertonic andsubsequent treatment not indicated.Kneadingthebolusofwormsintolargebowel andthereforeagastrojejunostomy is asituationofanactualpyloricnarrowing there isnever enemas.Whenlaparotomyisindicated, saline with nasogastricsuction,IVfluidsandhypertonic An uncomplicatedroundwormobstructionoftheintestineshouldalwaysbetreatedconservatively ingested bytheworm. oralinearstreaklyingparalleltotheintestine.Thisrepresentsbarium by anegativeshadow Abariummealoftendemonstratesroundwormsintheintestineeither fluffyexudates. X-ray shows InLoeffler’ssyndrome,chest crystals. orCharcot–Leyden larvae Bronchoscopic lavagemayshow Worm infestationisoftenassociatedwithahigheosinophilcountasinmostparasiticinfestations. Vater, acutepancreatitis. causingascendingcholangitis,obstructivejaundiceandeven Thewormsmaytravelupthepapillaof perforation. may causeintestinalobstructionandeven compete fornutrients.Thismayleadtomalnutritionandfailurethrive.Awormbolus heavy, colonisetheintestineandifinfestationisvery circulating blood.Additionally,they arereleasedintothebronchiolesfrom Roundworms causeprobleminthelungswhenlarvae combined withcharacteristicfeaturesonchestX-rayiscalledLoeffler’ssyndrome. onchestX-ray. withfluffyexudates chest pain,dyspnoeaandfever Thiscombinationofsymptoms cough, ofascariscausesaninflammationthelungcharacterisedbydry The releaseoflarva matureandcauseintestinalsymptoms. their maturationintheintestine.Inintestine,they insputumandcomplete areswallowed onchestX-raycalledLoeffler’ssyndrome.Thelarva exudates cough,chestpainandfluffy penetratetothebronchioles.Thiscausesinflammationwithadry larva circulationtotheliverandthenlungs.Inlung wall andgetcarriedviatheportal whichpenetratetheintestinal Theingestedeggsreleaselarva, ofascariasisisasfollows. The lifecycle causes humaninfection. y datid disease Ascariasis lumbricoides survives in the external environment even underhostile environmenteven intheexternal survives 49

5: TROPICAL SURGERY PRINCIPLES ➜ 18. A,C,E 17. B,C,E 16. B,E 15. B,D,E 14. A,C,E 13. B,C,E 12. B,C,D 11. B 50

Poliom intestine to reach and spread through the portal system. intestine toreachandspreadthroughtheportal theparasitepenetrates The ovumgainsaccesstohumansbybeingingested.Onexcystation, In thedog,adultwormreachessmallintestineandeggsarepassedinfaeces. disease targets the anterior horn cells, causing lower motor neuronparalysis.Itdoesnotcause disease targetsthe anteriorhorncells,causinglower Poliomyelitis isanenteroviraldisease whichentersthebodybyingestion orinhalation.The need onlybetreatedwithalbendazole. danger. calcificationsignifythattheparasitemaybedyingordeadand which show Deepcysts wallisoftenfraughtwith such astheintrahepaticinferiorvenacava.Surgicalcurettageofcyst oftenhasathinlayerofcompressedtissue spreadovervitalstructures are largeandtheectocyst Someofthesecysts anaphylactic reactionsaremorecommoninmultiloculatedhydatidcysts. to hydatidantigen,and cholangitis. Moreover,thehostismorelikelytohavebeenexposed channelsandthusinjectionofscolicidalsolutionsismore likelytocausesclerosing with thebiliary aremorelikely tocommunicate oftenoccurafterinsignificanttrauma.They Multiloculated cysts channelandcausingsclerosingcholangitis. solution escapingintothebiliary colonised byhydatiddisease.Injectingscolicidalsolutionsintothispatient runsarealriskofthe systemhasbecome channel.Therefore,thelikelydiagnosisisthatentirebiliary the biliary oftencausedbytrivialtrauma,ismore likelytocommunicatewith Aseptatedcyst, hydatid cyst. aseptated Theimagingrevealed multiplemembranousstructuresintheCBD. revealed ERCP measures initiated. initiate aviolentanaphylacticattack.Thisneedstobeanticipatedandappropriateprophylactic could ofthecyst duringexploration tothehydatidantigen,arepeatexposure is alreadyexposed bylaparotomy.ruptured, needsimmediatetreatmentwithalbendazolefollowed thepatient As havingalready withasplitwall.Thecyst, acyst eosinophilcountandimagingshows elevated an Thebloodtestsreveal responded toantihistamines,issuggestiveofarupturedhydatidcyst. byskinrashwhich The clinicalscenarioofabdominalpainafteratrivialtrauma,followed sometimesgivesrisetothecharacteristic‘cartwheel’ appearance. multiple daughtercysts splitwalls.Inaddition,thepresenceof when thelamellarmembraneseparatesfromexocyst, arecharacterisedbythepresenceofmultipleseptations,calcificationinwalland, Hydatid cysts jaundice. tract,causingobstructive maycommunicatewiththebiliary and additionallydaughtercysts pressureontheCBD, mayexert Alargecyst as trivialtraumasometimescausesaruptureofthecyst. antigen meansthatpatientsoftenpresentwithanaphylacticshockandcollapseinadditiontopain pain intheupperrightabdomenduetoenlargementofliver. tothehydatid Exposure Hydatid diseasecommonlyaffectstheliver. Hence,patientscommonlypresentwithadullaching germinal membraneandcontainsviableparasiteswhichcanseparateformingdaughtercysts. thatisthe whichisnon-infectiveandaninnerendocyst tissues, anintermediatehyalineectocyst derivedfromcompressedhostorgan ischaracterisedbythreelayers,anouterpericyst The cyst y elitis ➜ ➜ 25. B,C,D 24. A,D,E 23. B,D 22. E 21. A,B,E 20. A,B,C,D 19. A,B,D

Amoebiasis Tuberculosis ofthesmallintestine andmuscleweakness. with fever loss,afactthatdistinguishesitfromtheGuillain–Barrésyndrome,whichalsopresents sensory carried to the colon. Here they attach themselves to the mucosa andmaypenetratetocause attachthemselvestothemucosa carried tothecolon. Herethey hatch in thesmallintestineandlargenumbers oftrophozoitesarereleasedand The cysts contaminated foodorwater. of patientsaffectedwithamoebiasis. Themodeofinfectionisviathefaeco-oralroutethrough The majorityproducenosymptoms andamoebicliverabscessoccursinlessthan10 per cent ofCentralandSouthAmerica. The diseaseiscommonintheIndian subcontinent,Africaandparts continuityaftercompletionofantituberculouschemotherapy.restoration ofbowel by asafirststep isfollowed adhesions.Intheseindividuals,resectionandexteriorisation severe indicated, resectionandanastomosismaynotbefeasibleinaseptic,undernourished patientwith late.Whilelaparotomy isalmostalways is oftenabsent.Thesepatientspresentvery duetoadhesions, gasunderthediaphragm of peritonitisand,becausealocalisedperforation patientspresentwithfeatures needstobemaintained.Rarely, suspicion fortubercularperforation of countriesandwithpoorsocioeconomic background,ahighindex For patientsfromdeveloping surgical proceduresmaybecarriedoutforapatientwithintestinaltuberculosis. Tuberculosis affectsvarioussitesandamountsoftheintestine,thereforeallnoted tuberculosis oftengetspulledupintothesubhepaticposition. because ofintensefibrosistheintestineandarounddraininglymphnodes,caecumin the terminalileumonabariumstudyispresentinCrohn’sdiseaseastuberculosis.However, of multiple sitesoftheintestine,‘skiplesions’arecharacteristicCrohn’sdisease.Narrowing tuberculosis.Whiletuberculosismayaffect pulmonary background. Thereisoftenaco-existent Tuberculosis countrieswithpoorsocio-economic isoftenendemicinpeoplefromdeveloping isthe onhistology Caseation also presentasmultipleperianalfistulae,sometimescausingatypicalwatering-canperineum. Thediseasemay obstructionorperitonitisfromperforation. withdistalsmall-bowel emergency Thepatientmayalsopresentasan the areaoflocalisedascitesoramassinrightiliacfossa. mayalsopresentwithadoughyfeeloftheabdomenfrom riseoftemperature.They and evening A patientwithtuberculosisoftheintestinemaypresentweightloss,vagueabdominalpain formation isafeatureofCrohn’sdisease. of theabundancelymphoidfollicles.Transmural inflammationwithapropensityforfistula reaction,causinghyperplasiaandthickeningoftheterminalileum because marked inflammatory virulence oftheorganism.Itisoftencausedbydrinkinginfectedunpasteurisedmilk.Therea The othervariety,calledthehyperplastictype,occurswhenhostresistanceisstrongerthan colonises thelymphaticsofterminalileum,causingtransverseulcerswithunderminededges. Typically infectedsputum,theorganism tuberculosisswallows whenapatientwithpulmonary sine quanon of tubercularinfection. 51

5: TROPICAL SURGERY PRINCIPLES 27. E 26. A,B,D,E 28. B,D,E ➜ ➜ 32. B,D 31. B,C,E 30. E 29. B,C, D,E 52

Filariasis Asiatic cholangiohepatitis sickpatients. principles ofvigorousresuscitationmustbeappliedtothesevery thegeneral iscarriedoutforcomplicationsofamoebiasis.However, amoebicidal drugs.Surgery rupture,butalsopromotesthepenetrationof notonlyprevents intestinal amoeba.Aspiration furoate, whichisnoteffectiveagainsthepaticinfestation,usedfor10 daystodestroyany and tinidazolearetheeffectivedrugs.Aftertreatmentwithmetronidazoleortinidazole,diloxanide effectivetreatmentforanamoebicliverabscessismedical.Metronidazole andvery The primary biopsies andrepeatthecolonoscopyafteradequatetreatment. totakerepeated withacarcinomaofthecolonwhichiswhyitmandatory often co-exists Thecondition a rightsidedlesionissuggestiveofanamoeboma,carcinomamustbeexcluded. habitin ofalteredbowel ofblood-stainedmucoiddiarrhoeaandahistory background, ahistory inadequate self-.Thisconditionmaymimicacarcinoma.Whilebeingfromanendemic ofrepeatedindiscriminateand right sideofthecolon,wherepatienthashadalonghistory peritonitis andshock.Achronicgranuloma,calledanamoebomamayform,commonlyonthe abscess oftenpresentswithupperabdominalandchestsymptoms.Ruptureofthecauses A largemajorityofpatientswillpresentwithbloodydiarrhoeaduetocolonicinfestation.liver symptoms. Untreatedabscessesarelikelytorupture. oftherightlobe.Thismaycausepulmonary abscess isusuallyhighinthediaphragmaticsurface of bloodandnecroticlivertissue.Thisfluidbecomessmellywhensecondarilyinfected.The fluid,amixture sauce’-like an abscess.Thefluidischocolate-coloured,odourlessand‘anchovy peripheralenhancementarecharacteristicof hypodense lesionwithirregularwallsshowing scanfeaturesofa The clinicalfeaturesareallthoseofanamoebicliverabscess.CT whicharepassedinstoolthatcaninfectotherhumansaswell. in theintestineformcysts percentofcases,theleftlobein10lobe in80 percentandtherestaremultiple.Alargenumber veintobecarriedtheliver.flask-shaped ulcers,orinvadetheportal affecttheright Herethey condition is rarely treated with surgery as none of the operations described are universally successful. asnoneofthe operationsdescribedareuniversally condition israrely treated withsurgery the earlystagesbefore thegrossdeformityofelephantiasis setsin.Onceelephantiasis in,the aneosinophilia.TreatmentBlood testsreveal effectivebutonlyin withdiethylcarbamazepineisvery Brugia malayi The diseaseismainlycausedby thediseaseofcholangiocarcinoma iseliminated. subclinical form.Therefore,theriskofdeveloping foreggsorwormsisdiagnostic.Thediseasecanbecured whentreatedinits Stool examination system. Patients fromendemicareasshouldbeofferedscreeningbyultrasonography ofthebiliary acutepancreatitis. even symptoms butcanalsopresentwithfeaturesofascendingcholangitis, obstructive jaundiceand eating mammals.Thediseasemayremaindormantforyears.Itoftenpresents withnon-specific cookedcausesinfectioninhumans andotherfish- fish andsnailswheneatenraworpartly Clonorchis sinensis and infectssnailsandfish,whichactastheintermediatehost.Ingestionof infected Brugia timori . Chyluriaandchylousascitesmaybe seeninfilariasis. Wuchereria bancrofti and in10 percent iscausedbytheparasite ➜ ➜ ➜ 38. A,C,D,E 37. A,C, D 36. B,C,D,E 35. A,C,D,E 34. A,B,C,E 33. C,D,E

T Tropical chronic pancreatitis Lepros The diseaseiscausedbya of rampantinfection. woundinfectionisanaccepted procedure,asiscreatingalaparotomyinthepresence to prevent withside-to-sideileo-transverseanastomosis.Delayedwound closure closure oftheperforation ofileumorcolonand withanastomosisorexteriorisation segment,resectionof bowel perforated afterfresheningtheedges,wedgeresectionof including,closureofthe perforation can vary, sparingly, asitcancauseaplasticanaemia. Surgicaloptionsfortreatmentoftyphoidperforation specificfortyphoidinfection,isusedvery Chloramphenicol, thoughvery typhoid perforation. Metronidazole, cephalosporinsandgentamycinareusedincombination inthetreatmentof freegasintheabdomen. exclude sick,to using anerectchestX-rayorabdominalinlateraldecubitus,if thepatientisvery signsofdeteriorationinthesecondorthirdweekshouldbeinvestigated appropriately, shows high incasesofuntreatedtyphoidfever. Therefore,apatientwhohas abdominalsymptomsand antigens ofthe testsforthepresenceof agglutininstoOandH The Widaltest,whichisobsoleteintheUK, suspicions aboutthedisease. ofavisittoanendemicareashouldraise Theclinicalfeaturesandahistory inadequate sanitation. The bacillusisingestedincontaminatedfoodanddrinkaresultofpoorhygiene forintractablepain. pancreaticojejunostomy. isusedasalastresort Resectionalsurgery forstonesinadilatedpancreaticduct.Thechoiceofprocedureisusuallylateral pain, particularly forintractable isnecessary isusedasatherapeuticprocedure.Surgery pancreatic insufficiency. ERCP present withabdominalpain,thirst,polyuria,weightlossduetomalnutritionandfeaturesofgross (tapioca), whichiseatenbythepoorasastaplediet.Thediseasethuspresentinfamilies.Patients incassava The diseaseisnotcausedbyalcoholingestionbutderivativesofcyanide approachtotheirsolution. a multidisciplinary guidelineswithrifampicin,dapsoneandclofazimine.Deformitiesformedneed according toWHO thecharacteristichistology.bacillus, andbyaskinbiopsy,whichshows Treatment iscarriedout The diseaseisdiagnosedbyobtainingaskinsmear,whichoftendemonstratestheacid-fast lymphocytes. dissemination ofabundantbacilliinthetissueswithmacrophagesandfew granuloma formation,tolepromatous,wherethereisapoorimmuneresponse,withwidespread between tuberculoid,wherethereisastrongimmuneresponsewithscantybacteriaandepithelioid tothehost’simmuneresponse,withaplethoraofgrades feet. Thetissuedamageisproportional anaesthesia ofhandsandfeetcausingrepeatedunrecognisedtraumadamagetothe resultingfromeffectssuchas therefore adirectcauseofthediseaseoritsreactions,andsecondary, orneurologicalmanifestations.Thedeformitiesare disease presentswithskinandupperrespiratory years,the being contractedinchildhoodorearlyadolescence.Afteranincubationofseveral Mycobacterium tuberculosis y phoid y Salmonella typhi Mycobacterium leprae . It is transmitted by the nasal secretions of a patient, the infection secretionsofapatient,theinfection . Itistransmittedbythenasal and paratyphi in the patients’ serum. The risk of a perforation is inthepatients’serum.Theriskofaperforation , which is weakly acid-fast compared with , whichisweaklyacid-fastcomparedwith 53

5: TROPICAL SURGERY PRINCIPLES 3C ➜ 2A 1D matchingquestions Answers: Extended 54

endemic areas,ismuchmorecommonthanpyogenicabscess. manifestation,occursinlessthan10extraintestinal percentoftheinfectedpopulationand,in is infected.Themajorityremainasymptomaticcarriers.Amoebicliverabscess,thecommonest ofCentralandSouthAmericawherealmosthalfthepopulation subcontinent, Africaandparts abdominal pain. headacheand pleurisy,haemoptysis,fatigue,myalgias,anorexia, chills, chestdiscomfort, without sputumproduction,anddyspnoea.Othercommonsymptoms includerigors,sweats, coughwithor anacuteorsubacuteonset offever, Most patientswithpneumoniaexperience due totheeffectsofruptureintoperitoneal,pleuralorpericardialcavity.emergency the patientmaypresentasan Occasionally, atingeofjaundiceorascitesmaybepresent.Rarely, pneumonitis–thelastsymptomisusuallyalatemanifestation. a pleuraleffusionandbasal rigidity, tenderhepatomegaly,andbulgingintercostalspaces,overlyingskinoedema, ofsuspicion. bloody diarrhoeaortraveltoanendemicarearaisestheindex of upper abdomen,shouldertippain,hiccoughsandanon-productivecough.Apasthistory cough andweightloss,whichgraduallyprogresstomorespecificsymptomsofpainintheright nightsweats,malaise, fever, and insidiousonsetofnon-specificsymptomssuchasanorexia, fever examiner ashavingan abdominalpathology. examiner symptoms attributable totheupperabdomenonlyand canbemisdiagnosedbythecallous parapneumonic pleuraleffusionispresent. foralteredbreathsoundsand rales.Dullnesstopercussionmaybepresentifa remarkable isoften Manypatients willappearacutelyill.Chestexamination desaturation. oxygen arterial • • • 1. Upperabdominalpain Amoebiasis iscausedby scanofthepancreasconfirmdiagnosis. Ultrasound andCT typicalpancreaticcalcificationintheformofdiscretestonesduct. abdominal X-rayshows causingsteatorrhoeaandmalnutrition.Helooksillemaciated. insufficiency, The clinicalpresentationisabdominalpain,thirst,polyuriaandfeaturesofgrosspancreatic food. cause,becausefamilymemberseatthesame toxicity isanimportant thatcassava thetheory tosufferfromthiscondition;strengthens family havebeenknown membersofthesame consumed asastapledietbypeoplefrompoorbackground.Several familialandgeneticfactorsbeingpossiblecauses. remains obscure,withmalnutrition,dietary, countries,seenmostlyinsouthernIndia.Theaetiology socioeconomic strataindeveloping Tropical chronicpancreatitisisadiseaseaffectingtheyoungergenerationfrompoor Parenchymal infiltrateonchestradiograph. findings. Bronchial breathsoundsorralesarefrequentauscultatory sweatsorrigors. sputum, dyspnoea,chestdiscomfort, Symptoms orhypothermia,coughwithwithout andsignsofanacutelunginfectionarefever Examination reveals apatientwhoistoxicandanaemic.Thewillhaveupperabdominal reveals Examination ofpainand The typicalpatientwithamoebicliverabscessisayoungadultmalehistory Occasionally, a patient with involvement of lower zonesof thelungsmaypresentwith Occasionally, apatient withinvolvementoflower or hypothermia,tachypnoea,tachycardiaandmild Common physicalfindingsinclude fever Initial routinebloodandurinetestsconfirmthatthepatienthastype1diabetesmellitus.Plain yearsandfromapoorbackground. The patient,usuallymale,isalmostalwaysyoungerthan40 andistherefore (tapioca)isarootvegetablethatreadilyavailableandinexpensive Cassava Entamoeba histolytica . ThediseaseiscommonintheIndian 5E ➜ 1D 4B 2A

the cardiovascular and respiratory systems. the cardiovascularandrespiratory including andcarefulexamination, todiagnosisliesinameticuloushistory may benone.Thekey mayincorrectlyattributethefindingstoanintra-abdominalcondition,whereasthere zone, maypresentwithsignsandsymptomsreferredtotheupperabdomen.Anunwary lower thoracicconditionslikeacutemyocardialinfarctionandlobarpneumonia,especiallyofthe Primary untilprovenotherwise. considered tohaveatyphoidperforation asuddendeteriorationaccompaniedbyabdominalsignsshouldbe whoshows typhoid fever freegasintheperitonealcavity. usually are)willshow they Infact,anypatientbeingtreatedfor ill,as classical featuresofperitonitis.AnerectchestX-rayoralateraldecubitusfilm(inthevery with typhoidulcer. ill,deterioratesfurther heralds aperforated Thepatient,whoisalreadyvery ulcer; thiscanleadtohypovolaemia. ileus.Thepatientmayhavemelaenaduetohaemorrhagefromatyphoid distension fromparalytic and suffersfromahightemperaturefor2–3weeks.Thepatientmaybetoxicwithabdominal of atyphoidulcer. becauseofperforation but thesurgeoniscalledupontotreatpatientwithtyphoidfever Itisadiseasenormallymanagedbyphysicians, of poorhygieneandinadequatesanitation. intothehumangastrointestinaltractasaresult negative organism.Theorganismgainsentry in ordernottomissthesepotentiallylethalbuttreatableconditions. upper abdominalsymptoms,especiallyinapatientwhocanhaveanyoftheseconditionsaswell 2. Lumpintheabdomen Typhoid iscausedby fever Commonly called dog tapeworm, hydatiddiseaseiscausedby Commonly calleddogtapeworm, is muchmorecommon thanpyogenicabscess. manifestation, occurs inlessthan10 percentoftheinfectedpopulation and,inendemicareas, majority remainasymptomatic carriers.Amoebicliver abscess,themostcommonextraintestinal ofCentralandSouth Americawherealmosthalfthepopulationisinfected.The Africa andparts Amoebiasis iscausedby of anaphylacticshockwithoutanyobvious cause. withfeatures apatientmaypresentasanemergency scanmaybediagnostic. Rarely, when theCT lesion,iflargeenough,causesdyspnoea. liver capsule,andapulmonary swelling causingpressureeffects.Thus,ahepaticlesioncausesdullpain fromstretchingofthe scanisdoneforsomeothercondition.Symptomatic diseasepresentswitha ultrasound orCT patient. organs in thesame mostcommon.Theparasitecanaffectanyorganorseveral next hydatidisthe shouldbeconsidered.Theliveristheorganmostoftenaffected. Thepulmonary cyst painful massintherightupperquadrantwithphysicalfindingsofa liverswelling,ahydatid healthy,complains ofagraduallyenlarging presentation. Whenasheepfarmer,whoisotherwise transmitted totheintermediatehosts:humans,sheepandcattle. community. Thedogisthedefinitivehostand,asapet,commonestsource ofinfection theoccasionalpatientmaycomefromaruralsheepfarming is commoninthetropics,UK Thus acutemyocardialinfarctionandlobarpneumoniamustbekeptindifferentialdiagnosisof generalisedabdominalpain,this In thesecondorthirdweekofillness,ifthereissevere A typicalpatientisfromanendemicareaorsomeonewhohasrecentlyvisitedsuchacountry The patient may present as an emergency with severe abdominal pain following minortrauma abdominalpainfollowing withsevere The patientmaypresentasanemergency orwhenan The diseasemaybeasymptomaticanddiscoveredcoincidentallyatpostmortem organinthebody,conditioncan beproteaninits every theparasitecancolonisevirtually As Entamoeba histolytica Salmonella typhi , alsocalledthetyphoidbacillus.ThisisaGram- . ThediseaseiscommonintheIndian sub continent, Echinococcus granulosus . Whileit 55

5: TROPICAL SURGERY PRINCIPLES 5C 4B 3E 56 with sweating,vagueabdominalpain withdistensionandalternatingconstipationdiarrhoea. habit,shouldarousethesuspicionofintestinaltuberculosis. altered bowel who hasrecentlyarrivedfromanendemicareaandfeatures of generalisedillhealthand Infection by untilprovenotherwise. considered tohaveatyphoidperforation asuddendeteriorationaccompaniedbyabdominalsigns shouldbe whoshows typhoid fever freegasintheperitonealcavity. usuallyare)willshow they Infact,anypatient beingtreatedfor ill,as classical featuresofperitonitis.AnerectchestX-rayoralateraldecubitus film(inthevery with typhoidulcer. ill,deterioratesfurther heralds aperforated Thepatient,whoisalready very ulcer; thiscanleadtohypovolaemia. ileus.Thepatientmayhavemelaenaduetohaemorrhagefromatyphoid distension fromparalytic and suffersfromahightemperaturefor2–3weeks.Thepatientmaybetoxicwithabdominal ofatyphoidulcer. becauseofperforation upon totreatthepatientwithtyphoidfever Itisadiseasenormallymanagedbyphysicians,butthesurgeoncalled and inadequatesanitation. intothehumangastrointestinaltractasaresultofpoorhygiene organism. Theorganismgainsentry In thehyperplastic type,amassmaybefeltintheright iliacfossa. chronically illpatientwitha‘doughy’ feeltotheabdomenfromareasoflocalisedascites. a shows Examination ofatuberculous ulcerinthesmallbowel. peritonitis fromperforation apatientmaypresentwithfeaturesof theterminal ileum.Rarely, particularly the smallbowel, obstructionfromstricturesof presentwithfeatures ofdistalsmall-bowel they anemergency, As bladder alongwithadherentomentum. regionwhichisconstitutedbytheinflamedgall infrequently apalpabletenderlumpinthesame ontherightside(Murphy’ssign),andnot specifically atthetipofninthcostalcartilage of therightshoulderandmayalsoradiatetoinferioranglescapula. systemicsymptoms.Thepainmaybereferredtothetip than adayandisassociatedwithsevere elaborate manifestation.Herethepatienthasamorepronouncedpainwhichmaylastfor isamore recurringepisodes,acutecholecystitis associated withnausea,andisseenasshort colicreferstoaself-limitingpaininrightupperabdomen most common.Whereasacutebiliary arethe colicandacutecholecystitis Of thevariouspresentationsofthiscondition,acutebiliary hasbeendescribedasapatientofgallstonedisease. flatulentfemaleof40 Classically afat,fertile, duetotheeffectsofruptureintoperitoneal,pleuralorpericardialcavity.emergency thepatientmaypresentasan Occasionally, atingeofjaundiceorascitesmaybepresent.Rarely, pneumonitis–thelastsymptomisusuallyalatemanifestation. pleural effusionandbasal rigidity, tenderhepatomegaly,andbulgingintercostalspaces,overlyingskinoedema,a ofsuspicion. bloody diarrhoeaortraveltoanendemicarearaisestheindex of upper abdomen,shouldertippain,hiccoughsandanon-productivecough.Apasthistory cough andweightloss,whichgraduallyprogresstomorespecificsymptomsofpainintheright nightsweats,malaise, fever, andinsidiousonsetofnon-specificsymptomssuchasanorexia, fever Typhoid iscausedby fever Patients present electively with weight loss, chronic cough, malaise, evening rise intemperature Patients presentelectivelywithweightloss,chroniccough,malaise,evening generalisedabdominal pain,this In thesecondorthirdweekofillness,ifthereissevere A typicalpatientisfromanendemicareaorsomeonewhohasrecentlyvisitedsuchacountry The classical findings on abdominal examination aretendernessintherighthypochondrium, The classicalfindingsonabdominalexamination apatientwhoistoxicandanaemic.Thewillhaveupperabdominal reveals Examination ofpainand The typicalpatientwithamoebicliverabscessisayoungadultmalehistory Mycobacterium tuberculosis Salmonella typhi is common in the tropics. Any patient, particularly one iscommoninthetropics.Anypatient,particularly , also called the typhoid bacillus. This is a Gram-negative , alsocalledthetyphoidbacillus.ThisisaGram-negative 2A ➜ 1D

is obtained much earlier) should be done. A barium meal and follow-through (or small bowel (orsmallbowel is obtainedmuch earlier) shouldbedone.Abarium meal andfollow-through weeks)andstainingbytheZiehl–Neelsen methodforacid-fastbacilli(theresult take several significant inapatientfromanendemic area.Sputumforcultureandsensitivity(theresultmay faeculent vomitingaretypicalofsuch apatientwhoisinextremis. fromintestinalobstruction.Abdominal painanddistension,constipation,bilious emergency In thehyperplastictype,amassmaybefeltinrightiliacfossa. chronically illpatientwitha‘doughy’ feeltotheabdomenfrom areasoflocalisedascites. a shows Examination ofatuberculousulcerinthesmallbowel. peritonitis fromperforation apatientmaypresent withfeaturesof theterminalileum.Rarely, particularly the smallbowel, obstructionfromstrictures of presentwithfeaturesofdistalsmall-bowel they anemergency, As with sweating,vagueabdominalpaindistensionandalternatingconstipation anddiarrhoea. habit,shouldarousethesuspicionofintestinaltuberculosis. altered bowel who hasrecentlyarrivedfromanendemicareaandfeatures of generalisedillhealthand Infection by patient’s abdomen. possible toseethedistendedstomach,andasuccussionsplashmaybeaudibleonshaking thepatientitmaybe complains oflosingweight,andappearsunwelldehydrated.Examining dayspreviously.often itispossibletorecognisefoodstufftakenseveral Thepatientcommonly disappear. innatureandistotallylackingbile.Very Thevomitusischaracteristicallyunpleasant ulcer disease.Insomepatientsthepainmaybecomeunremittingandinothercaseslargely 3. Vomiting the first part oftheduodenum,mostcommonsiteforapepticulcer.the firstpart Commonly, whentheconditionisduetounderlyingpepticulcerdisease,stenosisfoundin atleastintheWest. obstruction shouldbeconsideredmalignantuntilprovenotherwise, in theincidenceofpepticulcerationandadventpotentmedicaltreatments,gastricoutlet withthedecrease thelatterwasmorecommon.Now, topepticulceration.Previously secondary The twocommoncausesofgastricoutletobstructionarecancerandpyloricstenosis of dilated jejunum and featureless ileum with evidence offluidbetweentheloops. of dilatedjejunumandfeaturelessileumwithevidence obstruction–valvulaeconniventes typicalsmallbowel dehydration. PlainabdominalX-rayshows straight lineratherthanatanangle. upwardsina ileumenteringthecaecumdirectlyfrombelow subhepatic caecumwiththenarrow theileum,typicallywithahigh particularly stricturesofthesmallbowel, enema)willshow bowel (orsmall result isobtainedmuchearlier)shouldbedone.Abariummealandfollow-through weeks)andstainingbytheZiehl–Neelsenmethodforacid-fastbacilli(the may takeseveral not significantinapatientfromanendemicarea.Sputumforcultureandsensitivity(theresult haemoglobinandapositiveMantouxtestareusual,althoughthelastis (C-reactive protein),low and faeculentvomitingaretypicalofsuchapatientwhoisinextremis. fromintestinalobstruction.Abdominalpainanddistension,constipation,bilious emergency Raised ESR and CRP, low Hbandapositive Mantouxtestareusual,althoughthelastisnot low andCRP, ESR Raised countries, patientsmaypresentlateasan thisisadiseasemainlyseenindeveloping As rise intemperature Patients presentelectivelywithweightloss,chroniccough,malaise,evening In the patient presenting as an emergency, urea and electrolytes show evidence ofgross evidence show ureaandelectrolytes In thepatientpresentingasanemergency, sedimentationrate)andC-reactiveprotein sedimentationrate(erythrocyte erythrocyte Raised countries,patientsmaypresentlateasan thisisadiseasemainlyseenindeveloping As The termpyloricstenosisisnormallyamisnomer. Thestenosisisseldomatthepylorus. Clinical features. Mycobacterium tuberculosis In benign gastric outlet obstruction there is usually a long history ofpeptic Inbenigngastricoutletobstructionthereisusuallyalonghistory is common in the tropics. Any patient, particularly one iscommoninthetropics.Anypatient,particularly 57

5: TROPICAL SURGERY PRINCIPLES 4B 3C ➜ 1D 58

crystals orthelarvae. crystals Charcot–Leyden ova.Sputumorbronchoscopicwashingsmayshow mayshow Stool examination should behighonthelistofpossiblediagnoses. andpancreaticsymptoms,ascariasisinfestation gastrointestinal,hepatobiliary with pulmonary, oronewhohasrecentlyreturnedfromanendemic area,presents country, tropical developing necrosis fromthebolusofworms. mayoccurfromischaemicpressure ofthesmallbowel perforation a surgicalemergency. Rarely, inchildren,duetoabolusofadultwormsincarceratedtheterminalileum.Thisis particularly malnutrition, failuretothriveandabdominalpain.Smallintestinalobstructioncanoccur, andintestinalsymptomsasanadultworm. symptoms asalarva oftheworld’spopulation.Theparasitecausespulmonary infect thehumanandaffectsaquarter Ascaris lumbricoides untilprovenotherwise. considered tohaveatyphoidperforation asuddendeteriorationaccompaniedbyabdominalsignsshouldbe whoshows typhoid fever freegasintheperitonealcavity. usually are)willshow they Infact,anypatientbeingtreatedfor ill,as classical featuresofperitonitis.AnerectchestX-rayoralateraldecubitusfilm(inthevery with typhoidulcer. ill,deterioratesfurther heralds aperforated Thepatient,whoisalreadyvery ulcer; thiscanleadtohypovolaemia. ileus.Thepatientmayhavemelaenaduetohaemorrhagefromatyphoid distension fromparalytic and suffersfromahightemperaturefor2–3weeks.Thepatientmaybetoxicwithabdominal ofatyphoidulcer. becauseofperforation upon totreatthepatientwithtyphoidfever Itisadiseasenormallymanagedbyphysicians,butthesurgeoncalled and inadequatesanitation. intothehumangastrointestinaltractasaresultofpoorhygiene organism. Theorganismgainsentry • • • • 4. Abdominaldistension Typhoid iscausedby fever offluidbetweentheloops. of dilatedjejunumandfeaturelessileumwithevidence obstruction–valvulaeconniventes typicalsmall-bowel dehydration. PlainabdominalX-rayshows straight lineratherthanatanangle. upwardsina ileumenteringthecaecumdirectlyfrombelow subhepatic caecumwiththenarrow theileum,typicallywithahigh particularly stricturesofthesmallbowel, enema) willshow bowel, with associated distension, vomiting, absence of bowel soundsandabsoluteconstipation. withassociateddistension,vomiting, absenceofbowel bowel, to neuromuscularfailure.Theresultantstasisleadsaccumulationof fluid andgaswithinthe ileusisastateinwhichtherefailureoftransmissionperistaltic wavessecondary Paralytic metabolic. ileus reflex infection postoperative In the second or third week of the illness, if there is severe generalisedabdominalpain,this In thesecondorthirdweekofillness,ifthereissevere A typicalpatientisfromanendemicareaorsomeonewhohasrecentlyvisitedsuchacountry Increase intheeosinophilcountiscommon,keepingwithmostother parasiticinfestations. ifoneisnottomissthediagnosis.Ifapersonfrom ofsuspicionisnecessary A highindex upto45cmlong.Itspresenceinthesmallintestinecauses The adultwormcangrow In the patient presenting as an emergency, urea and electrolytes show evidence ofgross evidence show ureaandelectrolytes In thepatientpresentingasanemergency, Varieties of paralytic ileus are: Varieties ofparalytic , commonlycalledtheroundworm,iscommonestintestinalnematodeto Salmonella typhi , also called the typhoid bacillus. This is a Gram-negative , alsocalledthetyphoidbacillus.ThisisaGram-negative 2B 3A them tobedistinguishedradiologically, asfollows: the jejunum,ileum,caecumandremainingcolonhaveacharacteristic appearancethatallows There arefourcardinalfeaturesofacuteintestinalobstruction: of anaphylacticshockwithoutanyobviouscause. withfeatures apatientmaypresentasanemergency scanmaybediagnostic.Rarely, when theCT lesion,iflargeenough,causesdyspnoea. liver capsule,andapulmonary swelling causingpressureeffects.Thus,ahepaticlesioncausesdullpainfromstretchingofthe scanisdoneforsomeothercondition.Symptomatic diseasepresentswitha ultrasound orCT patient. organsinthesame mostcommon.Theparasitecanaffectanyorganorseveral is thenext hydatid shouldbeconsidered.Theliveristheorganmostoftenaffected.pulmonary liver cyst painful massintherightupperquadrantwithphysicalfindingsofaliverswelling,hydatid healthycomplainsofagraduallyenlarging its presentation.Whenasheepfarmerwhoisotherwise transmitted totheintermediatehosts,humans,sheepandcattle. community. Thedogisthedefinitivehostand,asapet,commonestsourceofinfection theoccasionalpatientmaycomefromaruralsheepfarming is commoninthetropics,UK hydatiddiseaseiscausedby Commonly calleddogtapeworm, with multipleair-fluidlevels. gas-filledloopsofintestine theabdomenshows tympanitic. Painisnotafeature.Radiologically, offlatus.Abdominaldistensionbecomesmoremarkedand soundsandnopassage of bowel intestinal obstruction,allhernialorificesmustbeexamined. clinicalperitonitis.) Inallcasesofsuspected leadingtoovert perforation stages, therecanbebowel late ischaemia, whenintestinalobstructioncanleadtostrangulationandgangrene. Inthevery duetobowel refers tothesignsofreboundtendernessinabdomenonclinicalexamination, embarrassmentandperitonism.(Notethatperitonism septicaemia,respiratory shock, pyrexia, • • • • • • • • • • • Ileum –thedistal ileum hasbeenpiquantlydescribed byWangensteenasfeatureless. andareregularlyspacedgiving a‘concertina’of thebowel orladdereffect. The jejunumischaracterizedbyits valvulae conniventesthatcompletelypassacrossthewidth lie transversely. Nogasisseeninthecolon. ischaracterized bystraightsegmentsthataregenerallycentraland The obstructedsmallbowel presence orabsenceofintestinalischaemia. underlying age oftheobstruction location oftheobstruction constipation. distension vomiting pain The patient may present as an emergency with severe abdominal pain following minortrauma abdominalpainfollowing withsevere The patientmaypresentasanemergency orwhenan The diseasemaybeasymptomaticanddiscoveredcoincidentallyatpostmortem organinthebody,conditioncanbeprotean every the parasitecancolonisevirtually As Radiological diagnosis. Late manifestationsthatmaybeencounteredincludedehydration,oliguria, hypovolaemic accordingto: These featuresvary Clinical features. Ittakesclinicalsignificanceif72hafterlaparotomytherehasbeennoreturn Thisisbasedonasupineabdominalfilm.Whendistendedwithgas, Echinococcus granulosus . Whileit 59

5: TROPICAL SURGERY PRINCIPLES ➜ 1C 4C 3E 60

they usually are) will show free gasintheperitonealcavity. usuallyare)willshow they Infact,anypatientbeingtreated for ill,as classical featuresof peritonitis.AnerectchestX-rayor alateraldecubitusfilm(inthevery with typhoidulcer. ill,deterioratesfurther heralds aperforated Thepatient,whoisalreadyvery ulcer; thiscanleadtohypovolaemia. ileus.The patientmayhavemelaenaduetohaemorrhagefromatyphoid distension fromparalytic and suffersfromahightemperature for2–3weeks.Thepatientmaybetoxicwithabdominal ofatyphoid ulcer. becauseofperforation upon totreatthepatientwithtyphoidfever Itisadiseasenormallymanagedbyphysicians,but thesurgeoniscalled and inadequatesanitation. intothehumangastrointestinaltract asaresultofpoorhygiene organism. Theorganismgainsentry 5. Acuteabdomen • • • • • Typhoid iscausedby fever bronchoscopic washings may show Charcot–Leyden crystals orthelarvae. crystals Charcot–Leyden bronchoscopic washingsmayshow ova. Sputumor mayshow in keepingwithmostotherparasiticinfestations.Stoolexamination should behighonthelistofpossiblediagnoses.Increaseineosinophil countiscommon, andpancreaticsymptoms,ascariasisinfestation gastrointestinal,hepatobiliary with pulmonary, oronewhohasrecentlyreturnedfromanendemicarea,presents country, tropical developing pressure necrosisfromthebolusofworms. mayoccurasaresultofischaemic ofthesmallbowel perforation is asurgicalemergency. Rarely, inchildren,duetoabolusofadultwormsincarceratedtheterminalileum.This particularly malnutrition, failuretothriveandabdominalpain.Smallintestinalobstructioncanoccur, andintestinalsymptomsasanadultworm. symptoms asalarva oftheworld’spopulation.Theparasitecausespulmonary infect thehumanandaffectsaquarter Ascaris lumbricoides precursors andareanindicationforearlyintervention. proceduresarethemostcommon Operationsforappendicitisandgynaecological bowel. ‘easy’ flimsyonesand‘difficult’ denseones. late (fibrous)orbytheunderlyingaetiology. From apracticalperspective,thereareonlytwotypes: fibrous tissue. maybecomevascularisedandreplacedbymature whenthecauseisremovedorthey disappear Earlyfibrinousadhesionsmay production, whichproducesadhesionsbetweenopposedsurfaces. commonest causeofintestinalobstruction.Anysourceperitonealirritationresultsinlocalfibrin In Western countrieswhereabdominaloperationsarecommon,adhesionsandbandsthe a portion ofgreateromentumusuallyadherenttotheparietes. a portion bacterialperitonitis previous a stringbandfollowing congenital, e.g.obliteratedvitellointestinalduct are spacedirregularlyandtheindentationsnotplacedoppositeoneanother. haustralfoldswhich,unlikevalvulaeconniventes, forthecaecum,itshows –except Large bowel intherightiliacfossa. byaroundedgasshadow –adistendedcaecumisshown Caecum In the second or third week of the illness, if there is severe generalisedabdominalpain,this In thesecondorthirdweekofillness, ifthereissevere A typicalpatientisfromanendemic areaorsomeonewhohasrecentlyvisitedsuchacountry A high index of suspicion is necessary ifoneisnottomissthediagnosis.Ifapersonfrom ofsuspicionisnecessary A highindex upto45cmlong.Itspresenceinthesmallintestinecauses The adultwormcangrow Usually onlyonebandisculpable.Thismaybe: small Postoperative adhesionsgivingrisetointestinalobstructionusuallyinvolvethelower areearly(fibrinous)or ofwhetherthey Adhesions mayheclassifiedintovarioustypesbyvirtue , commonlycalledtheroundworm,iscommonestintestinalnematodeto Salmonella typhi , also called the typhoid bacillus. This is a Gram-negative , alsocalledthetyphoidbacillus.ThisisaGram-negative 4B 3E 2D endemic areas,is much morecommonthanpyogenic abscess. manifestation,occurs inlessthan10extraintestinal percentoftheinfectedpopulation and,in is infected.Themajorityremainasymptomatic carriers.Amoebicliverabscess,thecommonest sub ofCentralandSouthAmericawherealmosthalfthepopulation continent, Africaandparts altered bowel habitshouldarousethesuspicionofintestinaltuberculosis. altered bowel who hasrecentlyarrivedfromanendemicarea,featuresofgeneralisedillhealthand Amoebiasis iscausedby offluidbetweenthe loops. of dilatedjejunumandfeaturelessileumwithevidence obstruction–valvulae conniventes typicalsmall-bowel dehydration. PlainabdominalX-rayshows straight lineratherthanatanangle. upwardsina ileumenteringthecaecumdirectlyfrom below subhepatic caecumwiththenarrow theileum,typically with ahigh particularly stricturesofthesmallbowel, enema)willshow bowel (orsmall result isobtainedmuchearlier)shouldbedone.Abariummealandfollow-through weeks)andstainingbytheZiehl–Neelsenmethodfor acid-fastbacilli(the result maytakeseveral last isnotsignificantinapatientfromanendemicarea.Sputumforculture andsensitivity(the faeculent vomitingaretypicalofsuchapatientwhoisinextremis. fromintestinalobstruction.Abdominalpainanddistension,constipation, biliousand emergency hyperplastic type,amassmaybefeltintherightiliacfossa. chronically illpatientwitha‘doughy’ feeltotheabdomenfromareasoflocalisedascites.In a shows Examination ofatuberculousulcerinthesmallbowel. of peritonitisfromperforation apatientmaypresentwithfeatures theterminalileum.Rarely, particularly of thesmallbowel, obstructionfromstrictures presentwithfeaturesofdistalsmall-bowel they an emergency, As with sweating,vagueabdominalpaindistensionandalternatingconstipationdiarrhoea. 50 per cent of cases with perforated pepticulcer. percentofcaseswithperforated 50 peritonitis. of presentationneedanoperationwithoutwhichthepatientwilldeterioratewithaseptic because ofthepain.Theabdomendoesnotmovewithrespiration.Patientsthisform aboard-likerigidityandthepatientisdisinclinedtomove condition. Theabdomenexhibits hoursusuallyaccompaniedbyadeteriorationinthepatient’s overafew peritonitis supervenes inbacterialload, Although thecontentsofanacid-producingstomacharerelativelylow generalisedabdominalpainduetotheirritanteffectofgastricacidonperitoneum. severe sudden-onset ofpepticulceration,develops Thepatient,whomayhaveahistory recognisable. gastroduodenal artery. contrast, posteriorduodenalulcerstendtobleed,sometimesbyerodingalargevesselsuchas and,by pepticulcerhaschangedlittle.Anteriorlyplacedulcerstendtoperforate of perforated agentsanderadicationtherapy,theincidence Despite thewidespreaduseofgastricantisecretory untilprovenotherwise. considered tohaveatyphoidperforation asuddendeteriorationaccompaniedbyabdominalsignsshouldbe whoshows typhoid fever In the patient presenting as an emergency, urea and electrolytes show evidence ofgross evidence show ureaandelectrolytes In thepatientpresentingasanemergency, haemoglobinandapositiveMantouxtestareusual, althoughthe low andCRP, ESR Raised countries,patientsmaypresentlateasan this isadiseasemainlyseenindeveloping As riseintemperature Patients presentelectivelywithweightloss,chroniccough,malaise,evening Infection by of freegasunderthediaphragminexcess An erectplainchestradiographwillreveal Clinical features. Mycobacterium The classical presentation of perforated duodenalulcerisinstantly Theclassicalpresentationofperforated Entamoeba histolytica

tuberculosis is common in the tropics. Any patient, particularly one iscommoninthetropics.Anypatient,particularly . ThediseaseiscommonintheIndian 61

5: TROPICAL SURGERY PRINCIPLES ➜ 1C 2A 5A 62

• • • • • 6. Swollenleg compression; compartment syndrome,tumours). compression; compartment compression(veinsbeingthin-walled arepronetoextrinsic veins (duetothrombosis)orextrinsic the normalvenousreturnoutoflimb.Thisinturncouldbebecauseanintrinsicblock with Venous causeoflimbswelling.Itcanariseduetoaninterference congestionisanimportant of theparasitecalled Filariasis ismainlycausedbytheparasite felt subcutaneously. withlymphadenitis andlymphangitis.Occasionally,adultwormsmaybe episodic attacksoffever bodies, thusmakingthemlessprone tomosquitobites.Intheacutepresentation,thereare two-thirds ofwhomliveinIndia,China andIndonesia. about 10 millionpeopleworldwide, percent ofsufferers.Theconditionaffectsmorethan90 on which the pain seems to be improving, and then suddenly experiences a worsening of the pain aworseningofthepain on whichthepainseemstobeimproving,andthensuddenlyexperiences pain,receivessomemedicaltreat a patientwhoinitiallypresentswithtypicalrightiliacfossa ment clini hand, generaliseddisseminationcausesafulminantbacterialperitonitis.Thisisevident cally as under pressure,aredisseminatedinthegeneralperitonealcavity. cellswhich are appendix givesway,andthecontents,loadedwithbacteriainflammatory transmurally, andthepressureinsidelumenofappendixrises.Ultimately,wall can beresolutionoritmayprogresstoafulminantinflammation.Theinflammationspreads appendicitis isappendicectomyandnotantibiotics. point(parietal).Thetreatmentofanestablishedcase atMcBurney’s shifts torightiliacfossa The painofclassicalappendicitisisinitiallyperiumbilical(visceral:derivativemidgut)andlater duetotheeffectsofruptureintoperitoneal,pleuralorpericardialcavity.emergency Occa thepatientmaypresentasan sionally, atingeofjaundiceorascitesmaybepresent.Rarely, pneumonitis–thelastsymptomisusuallyalatemanifestation. oedema, apleuraleffusionandbasal abdominal rigidity, tenderhepatomegaly,andbulgingintercostalspaces,overlyingskin ofsuspicion. bloody diarrhoeaortraveltoanendemicarearaisestheindex of upper abdomen,shouldertippain,hiccoughsandanon-productivecough.Apasthistory cough andweightloss,whichgraduallyprogresstomorespecificsymptomsofpainintheright nightsweats,malaise, fever, andinsidiousonsetofnon-specificsymptomssuchasanorexia, fever to the lower abdomen,beforebecominggeneralized. to thelower becomesgeneralizedalongwithdeteriorationintheclinicalstatusofpatient. which now normal pulsations and neurologicalexamination. normal pulsations veins engorged superficial warmth incolour dusky pitting oedema On examination, therewillbeclassicalsignsofperitonitis,whichintheinitialstagesarelimited On examination, theomentumhaswalledoffappendix,itresultsinalocalizedabscess.Onother If bynow toprogressasaresultofincorrectdiagnosis, The naturalcourseofappendicitis,ifallowed apatientwhoistoxicandanaemic.Thewillhaveupper reveals Examination ofpainand The typicalpatientwithamoebicliverabscessisayoungadultmalehistory It is mainly males who are affected because females in general cover a greater part oftheir It ismainlymaleswhoareaffected because femalesingeneralcoveragreaterpart The findingsinalimbwithvenouscongestionare: Brugia malayi and Brugia timori Wuchereria bancrofti is responsibleforcausingthedisease in carried bythemosquito.Avariant 4D 3B Use oforalcontraceptivedrugs,especiallybywomenover30andthosewhosmoke,may of bedrest,suchascardiacfailureorstroke,arealsoassociatedwithahighincidenceDVT. appreciably higherincidencesofthromboemboliccomplications.Illnessesthatinvolveperiods operations,suchastotalhipreplacement,areassociatedwith effective prophylaxis).Certain about 3percentofpatientsundergoingmajorgeneralsurgicalprocedures(intheabsence deep veinsofthecalf,althoughitcanariseinfemoraloriliacveins. percentofthetimein are mostfrequentlyinvolved.Theprocessbeginsapproximately80 andpelvis extremities post-thrombotic limbandvenousulceration.Thedeepveinsofthelower ofa termortolong-termmorbidityduethedevelopment to suddendeathintheshort Venous thrombosisofthedeepveinsisaseriouslife-threateningconditionwhichmaylead fluid. forms ormicrofilariae.Theparasitemayalsobeseeninchylousurine,ascitesandhydrocele there maybeahydroceleunderlyingscrotalfilariasis. swollen limbwiththickenedskin,producingtheconditionofelephantiasis. Recurrent attacksoflymphangitiscausefibrosisthelymphchannels,resultinginagrossly streptococcalinfectioniscommon. thelimbswelling.Secondary breast cancer,thusexacerbating the cutaneouslymphaticscausesskinthickening,notunlike‘peaud’orange’appearancein limboedema.Obstructionto worms causelymphaticobstruction,resultinginmassivelower primarily inyoungwomen;andpitting oedema,whichrarelybecomesbrawnyandnon-pitting. involvement ofregionalnodesbymalignant diseaseorfilariasis. lymphatic obstructionfromtrauma, regionallymphnoderesectionorirradiation,extensive ormechanical formoflymphoedema involvesinflammatory form.Thesecondary the primary hypoplastic orhyperplasticinvolvement oftheproximalordistallymphatics,itisreferredtoas abnormalities consistingof When lymphoedemaistheresultofcongenitaldevelopmental fromanextremity. The underlyingmechanisminlymphoedemaisimpairmentofthelymph flow spasmissuperimposed. arterial orareflex restriction ofbloodflow orpaleandcoolwith massive swellingand ifvenousobstructionissevere skin maybecyanotic andtachycardia.Thesesymptomsareoftenabsentormayoccurinthe absenceofDVT.fever The venouscollaterals,slight include aslightswellingintheinvolvedcalf,distensionofsuperficial cases,thewholeleg,especiallywhen walking.Typicalin thecalfor,moreextensive findings without symptomsordemonstrableabnormalitiesintheextremities. embolism,presumably fromthelegveins, early stages.Thepatientmaysufferapulmonary associated withvenoushypercoagulability. nocturnalhaemoglobinuriaarealso andparoxysmal venous thrombosis.Homocystinuria shouldbeconsideredinyoungpatientswithpositivefamilyhistoriesand recurrent deficiency factorssuchasfactorVLeiden,proteinCandSdeficiencies,antithrombinIII Finally, hereditary adenocarcinoma,andespeciallyintumoursofthepancreas,prostate,breastovary. particularly ofDVT.be prescribedforwomenwithahistory incancer, Hypercoagulabilityisalsoobserved be associatedwithhypercoagulability,resultinginDVTsomewomen.Thesedrugsshouldnot The essentials of diagnosis are: painless persistent oedema of one or both lower extremities, extremities, The essentialsofdiagnosisare:painless persistentoedemaofoneorbothlower Symptoms andsigns. inthe Approximately halfofpatientswithDVThavenosymptomsorsignsintheextremity upto2weekspostoperatively,occurin Clinical manifestationsofDVT,whichmaydevelop theimmature Eosinophilia iscommon,andanocturnalperipheralbloodsmearmayshow limbfilariasisisoftenassociatedwithscrotalandpenileelephantiasis.Earlyon, Bilateral lower years.Theadult Chronic manifestationsappearafterrepeatedacuteattacksoverseveral The patientmaycomplainofadullache,tightfeelingorfrankpain 63

5: TROPICAL SURGERY PRINCIPLES 5E ➜ 1C 64

developing cellulitis. developing thesesignsmaybe swelling andpainintheinvolvedtissues.Anyskinwoundorulcerthatexhibits swollen lymphnodesneartheareaofinfectedskin.Thesignscellulitisincluderedness,warmth, –sometimeswithchillsandsweats afever begins toenlarge,thepersonmaydevelop whether minorfootlesionscontribute. and obesity,whichcanaffectcirculation,aswellburnsboils,althoughthereisdebateto diabetes injectingdrugs(especiallysubcutaneousorintramuscularinjection),pregnancy, eczema, skin, athlete’sfoot,dry insect bite,blistering,animaltattoos,pruriticskinrash,recentsurgery, Predisposingconditionsforcellulitisinclude actual infectionwhileontheskin’soutersurface. ofthenormalfloraskinbut causeno the mostcommonofthesebacteria,whicharepart the skin.Thisbreakdoesnotneedtobevisible.GroupA ofthebody.part legsismostcommonlyaffectedbythisinfection,althoughcellulitiscanoccuronany face orlower Skinonthe blisters, burns,insectbites,surgicalwoundsorsitesofintravenouscatheterinsertion. beenbroken:cracksintheskin,cuts, bacteria, andoftenoccurswheretheskinhaspreviously subcutaneous layersoftheskin.Cellulitiscanbecausedbynormalskinfloraorexogenous inflammationofdermaland Cellulitis isadiffuseinflammationofconnectivetissuewithsevere prolongedpressure. following except Lymphoedema pitting willpiteasilyatfirstbut,withtime,fibrosisanddermalthickeningprevent Lymphoedema andlesscommonlyaffectsthewholeleg. usuallyspreadsproximallytokneelevel one-third ofallcases. lymphoedema.ThefamilialformisreferredtoasMeige’sdiseaseandrepresentsabout primary tothekneeandaccountsforabout20percentof be unilateralthanbilateral,usuallyonlyextends aftermenarche,isthreetimesmorelikelyto females thaninmales,hasapeakincidenceshortly to bemadeinmostcases. lymphoedema fromsecondary differentiationofprimary willallow pathology, orcoexistent history and may occasionally contain crystals oftyrosine and mayoccasionally containcrystals damaged tissue.Inlong-standingcases, hydrocelefluidissometimesopalescentwithcholesterol asmallquantityofbloodthathasbeenincontactwith the fluidcoagulatesifitismixedwith even todrain intoacollectingvessel,theliquiddoesnotclot,but contents ofahydroceleareallowed • • • • 7. Scrotal swelling usually thetunica. of theprocessusvaginalis, A hydroceleisanabnormalcollectionofserousfluidinsomepart by connectionwithaherniaofthe peritoneal cavityinthecongenitalvariety. withlymphaticdrainageofscrotalstructures by interference although thereasonwhyfluidisnotabsorbedobscure hydroceles for mostprimary by defectiveabsorptionoffluid.Thisappearstobetheexplanation hydrocele e.g.secondary productionoffluidwithinthesac, by excessive Cellulitis usually begins as a small area of tenderness, swelling and redness. As thisredarea Cellulitis usuallybeginsasasmallareaoftenderness,swellingandredness.As Cellulitis iscausedbybacteriaenteringtheskin,usuallywayofacut,abrasionorbreakin Signs. Lymphoedema praecox(onsetfrom1to35yearsofage)isthreetimesmorecommonin Hydrocele fluidisamber-colouredand sterile,andcontainsalbuminfibrinogen.Ifthe History. Aetiology. Unlike othertypesofoedema,lymphoedemacharacteristicallyinvolvesthefoot. The ageofonsetpainlessswelling,togetherwiththepresenceorabsenceafamily Ahydrocelecanbeproducedinfourways: Streptococcus and Staphylococcus are 2D 3A be commodious. arereducible,theinguinalcanalwillbefoundto produce epigastricpain.Ifthecontentsofsac may ofweight,anddraggingonthemesentery stands up.Inlargeherniasthereisasensation assoonthepatient timegoeson,theherniacomesdown and persistsuntilreduced.As abdominal wallwhilestandingbehindtherespectiveshoulderofpatient. the lookingdown theinguinalregionfromsideoreven may bebetterseenbyobserving isstilllimitedtotheinguinalcanal,bulge impulse.Whenthesac together withanexpansile Whenaskedtocough,asmalltransientbulgingmaybeseenandfelt taking strenuousexercise. workor heavy complains ofpaininthegroinorreferredtotesticlewhenperforming Males are20timesmorecommonlyaffectedbyinguinalherniasthanfemales.Thepatient ofthescrotum. the swelling’onexamination between aswellingofinguinaloriginandonewhichisentirelyintrascrotal. • • • • • • • • cancer, thus exacerbating the limb swelling. Secondary streptococcalinfectioniscommon. thelimbswelling. Secondary cancer, thusexacerbating cutaneous lymphaticscausesskinthickening,notunlikethe‘peaud’orange’ appearanceinbreast limboedema. Obstructiontothe worms causelymphaticobstruction,resultinginmassivelower felt subcutaneously. withlymphadenitisandlymphangitis.Occasionally, adult wormsmaybe episodic attacksoffever bodies, thusmakingthemlesspronetomosquitobites.Intheacutepresentation, thereare thirds ofwhomliveinIndia,ChinaandIndonesia. about 10 millionpeopleworldwide,two- percentofsufferers.Theconditionaffects more than90 of theparasitecalled Filariasis ismainlycausedbytheparasite forms ormicrofilariae. Theparasitemayalsobeseen inchylousurine,ascitesandhydrocele fluid. there maybeahydroceleunderlying scrotalfilariasis. swollen limbwiththickenedskin,producing theconditionofelephantiasis. Recurrent attacksoflymphangitiscause fibrosisofthelymphchannels,resultinginagrossly femoral hernia. hydrocele ofthecanalNuckismostcommondifferentialdiagnosticproblem aredisplayedbyoperation. until theparts diagnosis.Itisusuallynotsettled lipoma ofthecord–thisisoftenadifficult,butunimportant, this condition incompletely descendedtestisintheinguinalcanal–anherniaisoftenassociatedwith femoral hernia spermatocele hydroceleofthecord encysted vaginal hydrocele Differential diagnosisinthemale an indirect inguinalherniaincreasesinsize,itbecomesapparentwhenthepatientcoughs As Differential diagnosisinthefemale usingfingerandthumbacrosstheneckofscrotumwillhelptodistinguish Examination Clinical features. Eosinophilia is common, and a nocturnal peripheral blood smear may show theimmature Eosinophilia iscommon,andanocturnal peripheralbloodsmearmayshow limbfilariasisisoftenassociated withscrotalandpenileelephantiasis.Earlyon, Bilateral lower years. Theadult Chronic manifestationsappearafterrepeatedacuteattacksoverseveral oftheir It ismainlymaleswhoareaffectedbecausefemalesingeneralcovera greaterpart Hydroceles arealmostinvariablytranslucentanditispossibleto‘getabove Brugia malayi and isasfollows: isasfollows: Brugia timori Wuchereria bancrofti isresponsibleforcausingthediseasein carriedbythemosquito.Avariant 65

5: TROPICAL SURGERY PRINCIPLES 5E 4B 66 Palpating a varicocele can be likened to feeling a bag of worms. When lying down, gravitymay Palpating avaricocele canbelikenedtofeelingabag ofworms.Whenlyingdown, The repairedtesticlewillreturntoits normalsizeinmanycases. the affectedtesticleissmallerthan the otherone,repairofvaricoceleisoftenrecommended. When physicalexamination. The conditionisoftendiagnosedin adolescentboysduringasports haveundergonevaricocele repair.production afterthey problemswillhavea significant improvementinthequalityand/orquantityofsperm with fertility percentofmen that50–70 of thetesticlesandaffectsspermproduction.Studieshaveshown isthatthecondition raisesthetemperature contribute totheseproblems,butacommontheory varicoceles how forcertain and anincreaseinthenumberofdeformedsperm.Itisnotknown signs ofvaricocelescanbeadecreasedspermcount;motility, ormovement,ofsperm; werefound tohaveavaricocele.Other percentofmenwhoweresubfertile study, asmany40 problemsinanyonecase.In ifavaricoceleisthecauseoffertility it isdifficulttobecertain oranyoftheotherproblemslistedbelow.varicocele iscausingdiscomfort symptoms aremildandmanydonotrequiretreatment.Treatment ifthe maybenecessary tends tooccurinyoungmen,usuallyduringthesecondorthirddecadeoflife.Typical varicocele Varicocele isarelativelycommoncondition(affectingapproximately10 percentofmen)that intothescrotum,thighsandabdominalwall. failureandmayextend patients withrightheart mayalsobepresent.Peripheral pittingoedemaisacommonsignin tricuspid regurgitation.Ascites enlargement –tenderornon-tenderduetopassivecongestion.Systolic maybefeltin pulsations failuremayhavehepatic rightheart transudation offluidintothealveoli.Patientswithsevere inthejugularvenoussystem.Inlungs,cracklesatlungbasesreflect of thepulsations Rightatrialpressuremaybeestimatedthroughtheheight lungs, abdomenandtheextremities. tachycardia, hypotensionandreducedpulsepressuremaybepresent. Thevitalsignsmaybenormal,but failuremayappearcachecticorcyanotic. heart standing severe orminoractivity,andthosewithlong- at rest.Otherswillbedyspnoeicduringconversation • • • • • • • predominant feature. Signs of fluid retention predominate in right heart failure,withthepatient predominant feature.Signsoffluidretentionpredominateinrightheart venouspressure;dyspnoeaisthe pulmonary cardiacoutputandelevated symptoms oflow failurehave failuremayberight-sidedorleft-sided(orboth).Patientswithleftheart Heart cause of right ventricular (RV) failure. cause ofrightventricular(RV) or signsofbothright-andleft-sidedfailure,leftventricular(LV) dysfunctionistheprimary symptoms oedema,hepaticcongestionand,onoccasion,ascites.Mostpatientsexhibit exhibiting infertility. visible orpalpable(abletobefelt)enlargedvein,likenedfeelingabag ofworms atrophy (shrinking)ofthetesticle(s) feeling ofheavinessinthetesticle(s) dragging-like orachingpainwithinthescrotum LV failure. venouspressure,hepatomegaly,dependentoedema,usuallydueto failure –elevated RV venouscongestion andpulmonary cardiac enlargement,rales,galloprhythm nocturnaldyspnoea, LV paroxysmal dyspnoea,cough,fatigue,orthopnoea, failure–exertional Upon palpationof thescrotum,anon-tender,twisted massalongthespermaticcordis felt. Testicular atrophy. problems. Fertility Symptoms oftheneck, failurecanbedetectedbyexamination peripheralsignsofheart Important symptoms,appearcomfortable failure,includingsomewithsevere Many patientswithheart Essentials ofdiagnosis. ofavaricocelemayinclude: There is an association between varicoceles and infertility or subfertility, but orsubfertility, Thereisanassociationbetweenvaricocelesandinfertility Atrophy,orshrinking,ofthetesticles isanotherofthesignsvaricocele. These areasfollows: 3A ➜ 1B 2C

bacillus, Leprosy, alsocalled Hansen’sdisease,isachronicinfectious diseasecausedbytheacid-fast fingers. onthepalmaraspectofradialthreeandahalf of thethenarmusclesandlosssensation fractures ofthedistalradiusorcompression inthecarpalcanal.Clinicalfeaturesincludeparalysis • • • • 8. Clawhands the sensitivityofexamination. directioninavaricocelewithValsalva,that hasaDopplermodecanseebloodreverse increasing inavessel.Anultrasoundmachine technique ofmeasuringthespeedatwhichbloodisflowing intra-abdominal venouspressureandincreasethedilatationofveins.Dopplerultrasoundisa bothofwhicharedesignedtoincrease movement) orstandingupduringtheexamination, tohaveabowel provocative manoeuvre,suchasValsalva’s manoeuvre(straining,likeheistrying togreaterthan2mm.Thepatientbeingstudiedshouldundergoa of thepampiniformplexus of thevaricocelemayornotbesmallerthanthatonotherside. varicocele,andabsencemaybeasignforclinicalconcern.Thetesticleontheside true inprimary suchthatthemassisnotobvious.Thisespecially thedrainageofpampiniformplexus allow transposition maybeindicated. passes behindthemedialepicondyleofhumerus,inwhichcasedecompressionoranterior ismostcommonlydamagedbylacerationsintheforearmorentrapmentasit The ulnarnerve fractures of the distal humerus or dislocation of the joint. fractures ofthedistalhumerusordislocationelbow aredueto orwrist. Injuriesattheelbow isclassicallyinjuredattheelbow The mediannerve half of the ring fingers, as well as part ofthepalm. half oftheringfingers,aswellpart middle andtheradial islostoverthepalmaraspectofthumb,index, –sensation Sensory the thumb Thethenarmusclesareparalysedwithresultinglossofabduction andoppositionof ulnar nerve. digitorumprofundus whichissuppliedbythe oftheflexor bytheportion fingers isperformed oftheother attheinterphalangealjoints, butflexion fingerandthumbcannotbeflexed index digitorumprofundus.The oftheflexor carpiulnarisandthemedialpart oftheflexor exception ofthewrist andfingers,withthe Motor –paralysisofthepronatorsforearmandflexors islostonthemedialoneandahalffingers. –sensation Sensory profundus digitorum carpiulnarisandmedialhalfoftheflexor alsocauseparalysisoftheflexor elbow the interosseousspacesandalongmedialborderofhand.Lesionsproximalto in pollicis longus(Froment’s sign).Inlonger-standingcases,musclewastingwillbeevident position,asweaknessoftheadductorpollicispermitsover-actionflexor assumes aflexed finger,thedistalphalanxofthumb a pieceofpaperbetweenthethumbandindex oftheinterphalangealjointsresultinaclaw-typedeformity. Ifthepatientpinches and extension ofthemetacarpophalangealjoints grip apieceofpaperbetweenthem.Weakness offlexion and lateraltwolumbricals.Thepatientisunabletoabductadductthefingers,orindeed ofthethenarmuscles Motor –paralysisofthesmallmuscleshand,withexception Damage to the median nerve atthe wristiscomparativelycommonasaresultoflacerations, Damage tothemediannerve Varicocele dilatationofthevessels canbereliablydiagnosedwithultrasound,whichwillshow Clinical features Clinical features Mycobacterium leprae include: include: , that is widely prevalent inthetropics.Globally,India,Brazil, Nepal, , thatiswidelyprevalent 67

5: TROPICAL SURGERY PRINCIPLES 4E 68 • thedistributionisabouteven. more oftenthanwomen.Afterthe age of80 atthisagemenareaffected with right-orleft-handedness.Incidence increasesaftertheageof40; in bothhandsandhasnoconnection todominantornon-dominanthands,noranycorrelation ofthepalm andmayappearsimilartoasmallcallus.Itcommonlydevelops visible onthesurface bands, whicharethesourceofreducedmobilitycommonlyassociated withthecondition,are These associated withtheconditiontendstogoaway,buttoughbandsoftissue maydevelop. contracture. the fingerscannotmovefreely. Thepalmaraponeurosisbecomes hyperplasticandundergoes sothat the tendonsconnectedto under theskinonpalmofhandthickenandshorten andisusuallypainless.Inpatientswiththiscondition, thetissues contracture progressesslowly fingerandthethumbarenearlyalwaysspared.Dupuytren’s in advancedcases,buttheindex ring andlittlefingersarethosemostcommonlyaffected.Themiddlefinger maybeaffected the affliction. thesurgeonwhodescribedanoperationtocorrect It isnamedafterBaronGuillaumeDupuytren, contractureofthehandwherefingerscannotbefullyextended. fibromatosis) isafixedflexion diseaseorpalmar Dupuytren’s asMorbusDupuytren, contracture(alsoknown Dupuytren’s causes atrophy,whichinturnresultsgynaecomastia. trophic ulceration,chronicinfection,contractionandauto-amputation.Involvementofthetestes be felttothickenedbehindtheupperendoffibula.Anaesthesiafeetpredisposes can isaffected,itleadstofootdrop,andthenerve Whenthelateralpoplitealnerve tibial nerve. burns andinjuries.Similarly,clawingofthetoesoccursasaresultinvolvementposterior atthewrist.Anaesthesiaofhandsmakesthesepatientsvulnerabletofrequent median nerve branch. inthebonycanalorofzygomatic branches ofthefacialnerve bridgeandliftingofthetipnose. Theremaybeparalysisofthe with collapseofthenasal resultingfromeffectssuchasanaesthesiaofthehandsandfeet. secondary, whicharecausedbyleprosyoritsreactions,and deformities producedaredividedintoprimary, palemaculesthatformplaquesandnodulescalledlepromas.The occurs intheformofseveral Cutaneousinvolvement leprids. Inlepromatousleprosy,thediseaseissymmetricalandextensive. –lesionscalled androughsurface edgesandadry maculeswithelevated or erythematous whicharetender.the nerves, Theremaybeasymmetricwell-definedanaesthetichypopigmented the bodywithlimiteddeformityofthatorgan.Neuralinvolvementischaracterisedbythickening of Intuberculoidleprosy,thedamagetotissuesoccursearlyandislocalisedonepart nerves. tothehost’simmuneresponse. and giantcells.Thetissuedamageisproportional immune responsewithscantbacilliinthetissues,epithelioidgranulomas,numerouslymphocytes astrong resistance fromthepatient.Intuberculoidleprosy,onotherhand,patientshows Thisisareflectionofthepoorimmuneresponse,resultingindepletedhost lymphocytes. a few leprosy, thereiswidespreaddisseminationofabundantbacilliinthetissueswithmacrophagesand accounts for78 percentoftheworld’sdisease. Mozambique, AngolaandMyanmar(Burma)accountfor91 percentofallthecases;Indiaalone Japan although itisalso widespreadinsomeMediterranean countries(e.g.SpainandBosnia) and in ithasbeencalledthe‘Viking disease’, Europeanancestry; people ofScandinavianornorthern The diseaseisbroadlyclassifiedintotwogroups,lepromatousandtuberculoid.In Dupuytren’s disease is a very specific affliction,andprimarilyaffects: diseaseisavery Dupuytren’s time,pain It usuallyhasagradualonset,oftenbeginningastenderlumpinthe palm.Over contractureiscausedbyunderlyingcontracturesofthepalmar fascia.The Dupuytren’s andthe attheelbow The handsaretypicallyclawedbecauseofinvolvementtheulnarnerve andseptumofthenose anddestructionofthelateralcartilages There islossofeyebrows tractandperipheral progressiveandaffectstheskin,upperrespiratory The diseaseisslowly 5D ➜ 1B

claw hands. hipdislocation. of theglutealmusclesandparalytic andflailknee. recurvatum talipes equines,equinovarus,cavovarus,equinovalgusandcalcaneus. talipes equines, equinovarus,talipescavovarus, talipesequinovalgusandcalcaneus. should helptodifferentiatethetwoconditions. analysis symptomsandsigns,cerebrospinalfluid(CSF) Thelatterhassensory be excluded. withmuscleweakness,theGuillain–Barrésyndromeneedsto fever When apatientdevelops causesmuchmisery. dooccur,thedisability symptomsbut,whenthey paralytic sufferers develop limbareaffectedtwiceasfrequentlythoseoftheupperlimb.Only1–2percent the lower muscles. andpharyngeal of involvementtherespiratory paralysisofthebulbarformthatmaynotbecompatiblewithlifebecause consisting ofextensive variety andheadachetotheextreme daysofmildfever spectrum ofsymptoms,fromafew enters thebodybyinhalationoringestion.Clinically,diseasemanifestsitselfinawide countries.Thevirus Poliomyelitis isanenteroviralinfectionthataffectschildrenindeveloping claim. isnotconnectedwithhandednesscastssomedoubtonthis thefactthatDupuytren’s However, ofthehands. may betriggered,byphysicaltrauma,suchasmanuallabourorotherover-exertion maybecaused,oratleasttheonset disease, althoughthereissomespeculationthatDupuytren’s Dupuytren’s leadtoahigherriskofdeveloping hand injuriesorspecificoccupationalexposures that andliverdisease.Thereisnoprovenevidence alcoholism, epilepsytherapywithphenytoin 9. Limbdeformit • • • • the twoconditions. analysisshouldhelptodifferentiate symptomsandsigns,CSF Thelatterhassensory excluded. withmuscleweakness, theGuillain–Barrésyndromeneedstobe fever When apatientdevelops causesmuchmisery. dooccur,thedisability symptomsbut, whenthey paralytic sufferers develop limbareaffectedtwiceasfrequently asthoseoftheupperlimb.Only1–2percent the lower muscles. andpharyngeal of involvementtherespiratory paralysisofthebulbarformthatmaynotbe compatiblewithlifebecause consisting ofextensive variety andheadachetotheextreme daysofmildfever spectrum ofsymptoms–fromafew enters thebodybyinhalationoringestion.Clinically,diseasemanifests itselfinawide countries.Thevirus Poliomyelitis isanenteroviralinfectionthataffectschildren indeveloping people withlivercirrhosis. contracture) Dupuytren’s percentofthoseafflictedhaveageneticpredispositionto (60–70 people withafamilyhistory people overtheageof40 men ratherthanwomen(menare10 thecondition) timesaslikelytodevelop Shoulder, elbow,wristandhand. Trunk. . Knee. Foot andankle. motorneuronparalysis.Musclesof The diseasetargetstheanteriorhorncells,causinglower contractureincludetrauma,diabetes, Some suspected,butunproven,causesofDupuytren’s Foot andankle. motorneuronparalysis. Musclesof The diseasetargetstheanteriorhorncellscausinglower Common problems are flexion andabductioncontractures,hipinstabilityduetoparalysis Commonproblemsareflexion The common knee deformities are flexion contracture,quadricepsparalysis,genu Thecommonkneedeformitiesareflexion Unbalancedparalysiscausesscoliosisalongwithpelvicobliquity. The most common deformities are claw toes, cavovarus foot, dorsal bunion, Themostcommon deformitiesareclawtoes,cavovarus foot,dorsal The most common deformities are claw toes, cavovarus foot, dorsal bunion, The mostcommondeformitiesareclawtoes,cavovarusfoot,dorsal y Commonproblemsareshoulderweakness,wristdropand 69

5: TROPICAL SURGERY PRINCIPLES 3A 2C 70 thirds ofwhomliveinIndia,China and Indonesia. about 10 millionpeopleworldwide,two- percentofsufferers.Theconditionaffects more than90 the cutaneouslymphatics causesskinthickening,not unlikethe‘peaud’orange’appearance in limboedema.Obstructionto worms causelymphatic obstruction,resultinginmassive lower felt subcutaneously. withlymphadenitis andlymphangitis.Occasionally,adultwormsmaybe episodic attacksoffever bodies, thusmakingthemlessprone tomosquitobites.Intheacutepresentation,thereare the nerves, whicharetender.the nerves, Theremaybeasymmetricwell-definedanaesthetichypopigmented the bodywithlimiteddeformityofthatorgan.Neuralinvolvementischaracterisedbythickening of Intuberculoidleprosy,thedamagetotissuesoccursearlyandislocalisedonepart nerves. response. tothehost’simmune andgiantcells.Thetissuedamageisproportional numerous lymphocytes astrongimmuneresponsewithscantbacilliinthetissues,epithelioidgranulomas, patient shows in depletedhostresistancefromthepatient.Intuberculoidleprosy,onotherhand, Thisisareflectionofthepoorimmuneresponse,resulting lymphocytes. macrophages andafew lepromatous leprosy,thereiswidespreaddisseminationofabundantbacilliinthetissueswith accounts for78 percentoftheworld’sdisease. Mozambique, AngolaandMyanmar(Burma)accountfor91 percentofallthecases;Indiaalone bacillus, Leprosy, alsocalledHansen’sdisease,isachronicinfectiousdiseasecausedbytheacid-fast claw hands. of theparasitecalled Filariasis ismainlycausedbytheparasite causes atrophy,whichinturnresultsgynaecomastia. trophic ulceration,chronicinfection,contractionandauto-amputation. Involvement ofthetestes be felttothickenedbehindtheupperendoffibula.Anaesthesia ofthefeetpredisposesto can isaffected,itleadstofootdrop,andthenerve Whenthelateralpoplitealnerve tibial nerve. burns andinjuries.Similarly,clawingofthetoesoccursasaresultinvolvement oftheposterior atthewrist.Anaesthesiaofhandsmakesthesepatients vulnerabletofrequent median nerve branch. inthebonycanalorofzygomatic branches ofthefacialnerve bridgeandliftingofthetipnose. Theremaybeparalysisofthe with collapseofthenasal resultingfromeffectssuchasanaesthesiaofthehandsandfeet. secondary, –whicharecausedbyleprosyoritsreactions –and deformities producedaredividedintoprimary palemaculesthatformplaquesandnodulescalledlepromas.The occurs intheformofseveral Cutaneousinvolvement leprids. Inlepromatousleprosy,thediseaseissymmetricalandextensive. –lesionscalled androughsurface edgesandadry maculeswithelevated or erythematous of the gluteal muscles and paralytic hipdislocation. of theglutealmusclesandparalytic andflailknee. recurvatum Chronic manifestations appear after repeated acute attacks over several years.Theadult Chronic manifestationsappearafter repeatedacuteattacksoverseveral oftheir It ismainlymaleswhoareaffected because femalesingeneralcoveragreaterpart The hands are typically clawed because of involvement of the ulnar nerve at the elbow andthe attheelbow The handsaretypicallyclawedbecauseofinvolvementtheulnarnerve andseptumofthenose, anddestructionofthelateralcartilages There islossofeyebrows tractandperipheral progressiveandaffectstheskin,upperrespiratory The diseaseisslowly The diseaseisbroadlyclassifiedintotwogroups–lepromatousandtuberculoid.In Shoulder, elbow,wristandhand. Trunk. Hip. Knee. Common problems are flexion andabductioncontractures,hipinstabilityduetoparalysis Commonproblemsareflexion Mycobacterium leprae The common knee deformities are flexion contracture,quadricepsparalysis,genu Thecommonkneedeformitiesareflexion Unbalancedparalysiscausesscoliosisalongwithpelvicobliquity. Brugia malayi , that is widely prevalent inthetropics.Globally,India,Brazil,Nepal, , thatiswidelyprevalent and Common problemsareshoulderweakness,wristdropand Brugia timori Wuchereria bancrofti is responsibleforcausingthediseasein carriedbythemosquito.Avariant 4D ➜ 1B 2E

10. Paral scanmayhelpinarrivingatadiagnosis. orCT an US deformity ofthehip,andlumponuppermedialthigh.AnX-raydorsolumbarspine, flexion theaffectedside,massiniliacfossa, space andscoliosiswithconcavitytowards joint. muscle onthefemur. deformityofthehip Theaffectedmusclemaygointospasm,causingflexion ofthe theabscessmaypointonmedialsideofthighatinsertion Rarely, fossa. toformaniliopsoasabscess.Thismaybepalpableasalumpintheiliac for acommoninsertion, infection tracksalongthemusclesheathandmayinvolveiliacusmuscle,whichjoinspsoas orneurologicalcomplaints.The may havesymptomsattributabletothespinesuchasbackache ormalaise, At thistimethepatientmayhavesystemicfeaturesofinfectionsuchasfever common), haematogenous,orfromtheoverlyingperitonealcavity. bodies(tubercularspineisthemost The sourceofinfectionmaybefromtheadjacentvertebral inthefascialsheathofpsoasmajor.Psoas abscessisaconditioninwhichandevelops forms ormicrofilariae.Theparasitemayalsobeseeninchylousurine,ascitesandhydrocelefluid. there maybeahydroceleunderlyingscrotalfilariasis. swollen limbwiththickenedskin,producingtheconditionofelephantiasis. Recurrent attacksoflymphangitiscausefibrosisthelymphchannels,resultinginagrossly streptococcalinfectioniscommon. thelimbswelling.Secondary breast cancer,thusexacerbating Gram-negative bacilli, puncture, epiduralanaesthesiaandspinalsurgery. Mostcasesaredueto osteomyelitis,decubitus ulcers,lumbar oflocalpredisposingconditionsarevertebral examples ofalocal infectiontothesubduralspace; endocarditis). Theremainderarisefromdirectextension ordentalabscesses) deepviscera(bacterial the skin(furunculosis),softtissue(pharyngeal weakness andothersignsofmyelopathyappear,progressionmaybe rapid. spinalcorddamageresultsfromvenouscongestion andthrombosis.Once further expands, theabscess whitebloodcellcountandsedimentationrate. As usual, accompaniedbyelevated monthsor longer.presentation isgenerally2weeksbutmayonoccasionbeseveral is Fever always present,eitheroverthespineorinaradicularpattern.Thedurationofpainpriorto permanentsequelae.Achingpainisalmost of thisdistinctiveprocesswillinmostcasesprevent andprogressivelimbweakness.Promptrecognition pain,fever clinical triadofmidlinedorsal compressive myelopathy. Itresultsintheformationofanepiduralabscessand presentsasa Pott’s spine,amanifestationofbonytuberculosis,isonethecausesnon-neoplastic usually inadequate unless there has been pre-existing occlusivedisease. Ontheotherhand, usually inadequate unlesstherehasbeenpre-existing occlusion dependonthestateofcollateralsupply.arterial Thecollateralsupply inthelegis tissuenecrosiswithin6h.Theeffectsofsudden complete acuteischaemialeadsto extensive segment,oranembolus.Withoutsurgicalrevascularisation, occluded,thrombosedarterial partially Acute limbischaemiaismostoften due toeitheranacutethromboticocclusionofapreviously world. underdeveloped Tuberculosis causeinthe source,Pott’s disease,remainsanimportant fromanadjacentvertebral Bilateral lower limbfilariasisisoftenassociatedwithscrotalandpenileelephantiasis.Earlyon, Bilateral lower The clinical signs may include tenderness of the affected vertebrae, fullness of the paravertebral fullnessoftheparavertebral The clinicalsignsmayincludetendernessoftheaffectedvertebrae, The abscessremainssilentforalongtimeduetothedeep-seatedlocationofmuscle. theimmature Eosinophilia iscommon,andanocturnalperipheralbloodsmearmayshow Two-thirds ofepiduralinfectionsarearesulthaematogenous spread ofbacteriafrom y sis Streptococcus , anaerobesandfungicanalsocauseepiduralabscesses. Staphylococcus aureus 71 ;

5: TROPICAL SURGERY PRINCIPLES 3D 72 • • • • • • • • • • • • • • • makealimbnon-viable. necessarily doesnot hasmanycollateralvesselssothatocclusionofamajorartery the subclavianartery diabetes, hyperlipidaemia, carotid artery stenosisandatrialfibrillation. diabetes, hyperlipidaemia,carotidartery the elderlypopulationgrows. cerebrovascular diseasesincreaseswithageandthenumberofstrokes isprojectedtoincreaseas malformations(AVMs). areamajorcauseof disability. They arteriovenous Theincidenceof and stroke, haemorrhagicstrokeandcerebrovascularanomaliessuchasintracranial aneurysms disorders:ischaemic Cerebrovascular diseasesincludesomeofthemostcommonanddevastating longer than several seconds. Neurologicalsymptomsaremanifest withinsecondsbecauseneurons longer thanseveral thatlasts the brainanditsvasculature. Cerebralischaemiaiscaused byareductioninbloodflow thediagnosis. including brainimagingareusedto support studies attributable toafocalvascularcause. Thusthedefinitionofstrokeisclinical,andlaboratory A stroke,orcerebrovascularaccident, isdefinedbythisabruptonsetofaneurologicaldeficitthat arteries. popliteal andfemoral echocardiogram;ultrasoundofaorta, Identify sourceofembolus–ECG, urgentarteriography If diagnosisisindoubt,perform thrombophilia screen. otherconnectivetissuedisorders),glucose(diabetes),lipidsand disease, e.g.giantcellarteritis, (inflammatory Blood tests–forfullbloodcount(ischaemiaisaggravatedbyanaemia),ESR flow Hand-held Dopplerultrasound–mayhelptodemonstrateanyresidualarterial cellulitis. The limbmayberedwhendependent,leadingtoamisdiagnosisofinflammation,e.g.goutor changes. The onsetoffixedmottlingtheskinimpliesirreversible (‘the 6Ps’). becomespale,pulseless,painful,paralysed,paraestheticandpoikilothermic The affectedpart limb. disease,potentialsourceofembolusandthestatepulsesincontralateral chronic arterial featurestodifferentiateincluderapidityofonsetsymptoms,pre-existing Important embolic orthrombotic. ofischaemiaandwhetheritislikelytobe shouldidentifytheseverity andexamination History hypoplasia. congenital causesofearly-onsetlegischaemia,e.g.aortic closed anatomicalspace tissuepressureina pressurefallsbelow syndrome–occurswhenperfusion compartment syndrome Raynaud’s trauma atherosclerotic narrowing thrombosis –mostcasesoflegischaemiaresultfromthepresencethrombusatsites proximal atheromatousstenosis,malignanttumourorforeignbody femoralorpopliteal), (aorta, valves,aneurysm infarction, prostheticandabnormalheart embolism –e.g.leftatriuminpatientsatrialfibrillation,muralthrombusaftermyocardial Investigations Presentation ofacutelimbischaemia Causes ofacutelimbischaemia Most cerebrovasculardiseasesaremanifest bytheabruptonsetofafocalneurologicaldeficit. smoking, The variousriskfactorsassociatedwithcerebrovasculardiseasesarehypertension, The clinical manifestations of stroke are highly variable because of the complex anatomyof The clinicalmanifestationsofstroke arehighlyvariablebecauseofthecomplex are: 5C 4A reflexes andBabinski signs(theuppermotorneuron syndrome).Suchlesionsalsotypically reflexes overtime ofincreasedmuscletone,heighteneddeeptendon or quadriplegia,withtheevolution andothermotortractscauseparaplegia side. Lesionsthattransectthedescending corticospinal theneckoneach trunk andsequentiallymoveduptoward applied totheproximallegsandlower tuningforkafterimmersionincoldwater) patient toidentifyapinprickorcold stimulus(e.g.adry issoughtbyaskingthe level function isimpairedahallmarkof spinalcorddisease.Thissensory and treatedatanearlystage. ifrecognized inputofthetrunkandlimbs.Manyspinalcorddiseases are reversible and sensory because thespinalcordcontains,inasmallcross-sectionalarea,almost theentiremotoroutput system inthenervous wouldinflictelsewhere thedamagethey deficitsfarbeyond and sensory producequadriplegia, paraplegia They Diseases ofthespinalcordarefrequentlydevastating. claw hands. hipdislocation. of theglutealmusclesandparalytic andflailknee. recurvatum talipes equines,equinovarus,cavovarus,equinovalgusandcalcaneus. the twoconditions. analysisshouldhelptodifferentiate symptomsandsigns,CSF Thelatterhassensory excluded. withmuscleweakness,theGuillain–Barrésyndromeneedstobe fever When apatientdevelops causesmuchmisery. dooccur,thedisability symptomsbut,whenthey paralytic sufferers develop limbareaffectedtwiceasfrequentlythoseoftheupperlimb.Only1–2percent the lower muscles. andpharyngeal of involvementtherespiratory paralysisofthebulbarformthatmaynotbecompatiblewithlifebecause consisting ofextensive variety andheadachetotheextreme daysofmildfever spectrum ofsymptoms–fromafew enters thebodybyinhalationoringestion.Clinically,diseasemanifestsitselfinawide countries.Thevirus Poliomyelitis isanenteroviralinfectionthataffectschildrenindeveloping headache. hemiparesis); changeinvision,gait,orabilitytospeakunderstand;asudden,severe motor functionononesideofthebody(nearly85percentischaemicstrokepatientshave structures, fromthetoxiceffectsofblooditselforbyincreasingintracranialpressure. into oraroundthebrain;itproducesneurologicalsymptomsbyproducingamasseffectonneural Intracranialhaemorrhageiscausedbybleedingdirectly sourceortheheart. a proximalarterial or infarctionisusuallycausedbythrombosisofthecerebralvesselsthemselvesembolifrom strokehasoccurrediftheneurologicalsignsandsymptomslastfor>24h.Focal ischaemia injury; permanentbrain ofnew resolve within24hregardlessofwhetherthereisimagingevidence requiresthatallneurologicalsignsandsymptoms ThestandarddefinitionofTIA attack (TIA). recover fullyandthepatient’ssymptomsareonlytransient:thisiscalledatransientischaemic isquicklyrestored,braintissuecan infarction ordeathofbraintissueresults.Whenbloodflow minutes, lastsformorethanafew offlow failureisrapid.Ifthecessation lack glycogen,soenergy The presence of a horizontally defined level below which sensory, motorandautonomic whichsensory, below The presenceofahorizontallydefinedlevel Shoulder, elbow,wristandhand. Trunk. Hip. Knee. Foot andankle. motorneuronparalysis.Musclesof The diseasetargetstheanteriorhorncells,causinglower and/or lossofsensory thesuddenonsetoffollowing: Most ofthepatientsexperience Common problems are flexion andabductioncontractures,hipinstabilityduetoparalysis Commonproblemsareflexion The common knee deformities are flexion contracture,quadricepsparalysis,genu Thecommonkneedeformitiesareflexion Unbalancedparalysiscausesscoliosisalongwithpelvicobliquity. The most common deformities are claw toes, cavovarus foot, dorsal bunion, Themostcommondeformitiesareclawtoes,cavovarusfoot,dorsal Commonproblemsareshoulderweakness,wristdropand 73

5: TROPICAL SURGERY PRINCIPLES 74 cauda equinaandconusmedullarissyndromescoexist. spinalcanaloftenproduceamixedclinicalpictureinwhich elementsofboth lesions inthelower and bladder function.Mass andrelativesparingofbowel extremities, inthelower variable areflexia loss, backandradicularpain,asymmetriclegweakness andsensory cord, arecharacterizedbylow preserved. islargely areabsent.Musclestrength The bulbocavernosus(S2–S4)andanal(S4–S5)reflexes retentionandincontinencewithlaxanaltone)impotence. dysfunction(urinary and bowel anaesthesia(S3–S5),prominentbladder syndrome isdistinctive,consistingofbilateralsaddle segments.Theconus andsinglecoccygeal sacral of thespinalcord,comprisinglower the anklejerks(S1). ofthethigh,andabolish atthekneeandextension only movementsofthefootandankle,flexion LesionsatL5–S1paralyse atthekneeandabolishpatellarreflex. thigh, weakenlegextension (Beevor’s sign). (Beevor’s muscles, resultinginupwardmovementoftheumbilicuswhenabdominalwallcontracts accompany theparalysis.LesionsatT9–T10 –butnottheupperabdominal paralysethelower nipples (T4)andumbilicus(T10). function Legweaknessanddisturbancesofbladderbowel of midlinebackpainifitaccompaniesthesyndrome.Usefulmarkersforlocalizationare cordlesionatanylevel. may accompanyacervical isimpaired.Horner’ssyndrome(miosis,ptosisandfacialhypohidrosis) finger andwristflexion andtriceps;atC8, in thebiceps;atC7weaknessisfoundonlyfingerandwristextensors andreflexes diaphragm. LesionsatC4–C5producequadriplegia;C5–C6,thereislossofpower acute polyneuropathy. damagetomanysegmentsofthecordorforan weeks, andshouldnotbemistakenforextensive days,rarelyfor initially beflaccidratherthanspastic.Thisstateof‘spinalshock’lastsforseveral andacutetransverselesions,thelimbsmay together withsignsoflongtractdamage.Withsevere occur disorders;thus,segmentalsignsaremostusefulwhenthey focal rootorperipheralnerve Thesesignsalsooccurwith maybenotedatthislevel. muted orabsentdeeptendonreflex segments,ora byoneorseveral disturbance, fasciculationsoratrophyinmusclesinnervated (hyperalgesiaorhyperpathia)attheupperendofsensory A bandofalteredsensation byanindividualcordsegment. innervation signs correspondingtodisturbedmotororsensory dysfunction. andsexual and bladder,bowel theimplicatedcordlevel, produce autonomicdisturbancesconsistingofabsentsweatingbelow By contrast, lesions of the cauda equina, the cluster of nerve roots derived from the lower rootsderived fromthelower By contrast,lesionsofthecaudaequina,clusternerve Sacral cord/conusmedullaris. Thoracic cord. Lumbar cord. The uppermost level ofaspinalcordlesioncanalsobelocalizedbyattentiontothesegmental The uppermostlevel Cervical cord. Cervical ofthespinalcordareoutlinedbelow.The mainfeaturesoftransversedamageateachlevel Lesions at the L2–L4 spinal cord levels paralyse flexion andadductionofthe paralyseflexion LesionsattheL2–L4spinalcordlevels Upper cervical cordlesionsproducequadriplegiaandweaknessofthe Uppercervical Lesions here are localized by the sensory level onthetrunkandbysite level Lesionsherearelocalizedbythesensory Theconusmedullarisisthetaperedcaudaltermination 6 Paediatric surgery

Multiple choice questions ➜ General surgical E Consideration must be given to non- management accidental injury (NAI). 1. Which of the following statements are ➜ Inguinoscrotal swellings false? A A neonate is a baby up to 4 weeks old. 4. Which of the following statements B The urinary bladder is an intra-abdominal about inguinoscrotal swellings are organ in infants and children. false? C An infant’s head accounts for 20 per cent A A hydrocele is a patent processus of the body surface area. vaginalis as is a hernia. D Gluconeogenesis in infants is as good as B Hernia is always indirect. in adults. C Hernia can be direct or indirect. E The infant’s immune system is immature. D In incarcerated hernia, reduction should be attempted by taxis followed by 2. Which of the following statements are operation 24 h later. true? E A hydrocele always needs an operation. A Children are regarded as small adults and therefore their surgical management ➜ Undescended testis is adjusted according to their size. 5. Which of the following are true for B Attention to thermoregulation is very undescended testis? important. A Orchidopexy in a subdartos pouch is the C Postoperative management requires treatment of choice. extreme care because of inadequate B The operation is recommended at the stress response. age of 2 years. D Postoperatively children do not recover C When a testis is impalpable and therefore as quickly as adults. intra-abdominal, laparotomy should be E Minimal access surgery (MAS) can be done. used at all ages. D Laparoscopy is the gold standard procedure for an intra-abdominal testis. ➜ Paediatric trauma E Orchidopexy reduces the chance of 3. Which of the following statements malignancy. regarding paediatric trauma are ➜ true? Acute scrotal pain A Follow usual Advanced Trauma Life 6. Which of the following statements Support (ATLS) rules as in an adult. are true with regard to acute scrotal B Usual rules of resuscitation apply. pain? C Blunt trauma is more common than A Acute testicular pain can be from torsion penetrating trauma. of the testis, torsion of the hydatid of D Splenic injury in the majority should be Morgagni or acute epididymitis. treated by splenectomy.

75 PRINCIPLES ➜ ➜ Extended matchingquestions Extended Chooseandmatchthecorrectdiagnosiswitheachofscenariosgivenbelow: Chooseandmatchthecorrectdiagnosiswitheachofscenariosgivenbelow: 76

1 E D C B A 2. Acuteabdomen 3 2 1 E D C B A 1. Vomiting E D C B tenderness, andthe abdominalwalldoesnotmovewith respiration. abdominaltenderness,rigidityandrebound tachycardia, lookstoxicandhasmarked lower of39°C, for thelast24horso.Hehasbeen offhisfoodforacoupleofdays.Hehaspyrexia A 10-year-old anddiarrhoea boy hasbeenbroughtinwithabdominalpain,vomiting,pyrexia pneumonia Right lower-lobe tractinfection Urinary Constipation Acute appendicitis Non-specific abdominalpain shelooksillanddehydratedtherightsideofabdomen feelsempty.examination noticedthatthenappyhasbloodstainedmucus.On denoting spasms.They and elbows, herknees isinpainintermittentlybecauseshescreams,flexing feelthatMary-Ann They days. Theparentsnoticedthatthevomitusissometimesgreenandat other timesbrownish. a10-month-oldMary-Ann, girl,hasbeenbroughtinwithoccasionalvomitingforacoupleof was beingfedbyhismother. The babyisdehydrated.paediatricianfeltalumpintheupperabdomenwhenJonathan while.Themothernoticedsometwitchingofmuscles. feed onlytobringitupwithinashort andtakesits forthelast24h.Thevomitusisclearfluid.infantseemshungry incessant Jonathan, a5-week-oldboy,hasbeenvomitingintermittentlyfor2weeks.Thisbecome abdomen withared,irreducibleswellingintheleftgroinnotnoticedbyparents. depressedfontanelles,lossofskinturgor.is dehydrated–sunkeneyes, Thereisatympanitic thechildlookstoxicand action.Onexamination fluid.Therehasbeennobowel brownish dirty hourshasconsistedof withbutoverthelastfew parents thevomituswasgreenishtostart withvomitingfor24h.Accordingtothe A 2-year-oldboyhasbeensentinasanemergency Intestinal malrotation+/–volvulus Duodenal atresia Incarcerated inguinalhernia Intussusception pyloricstenosis Congenital hypertrophic scrotum mustbecarriedout. ofthe In caseofanydoubt,exploration symptoms. Incarcerated herniamaycausesimilar suspected testiculartorsion. Doppler ultrasoundshouldbedonein the groinorsuprapubicarea. Pain oftesticulartorsionmayoriginatein . Whatistheoperation forcongenital 7. E D C B A Ivor-Lewis. Ramstedt Heller Whipple Hartmann hypertrophic pyloricstenosiscalled? ➜ Chooseandmatchthecorrectdiagnosiswitheachofscenariosgivenbelow:

7 6 5 4 3 2 1 G F E D C B A 3. Congenitalmalformations 3 2 bilious vomiting.Thebabyistoxicwith septicshock. abdominaldistension,bloodstainedstoolsand aneonatehasdeveloped daysafterbirth, A few skinbruises. Therearesomesuperficial sincebirth. discoloration ofskinandconjunctivaever increasing yellowish A 2-week-oldneonatewhowasbornwithjaundicehasexhibited mostlyontherightsidewithasubhepaticcaecum. thebowel and acontrastmealshows sick ofbloodstainedstools.The babyisvery A neonatehasbile-stainedvomitingwithpassage appearance andtheconditionwassuspectedonprenatalultrasound. a‘double-bubble’syndrome. Thebabyhasbiliousvomiting.plainabdominalX-rayshows toababy whohasDown’s A motherwhosufferedfrompolyhydramnioshasgivenbirth ultrasound. up inthechestandthereisgasabdomen.Theconditionwassuspected onprenatal that thetubeiscurled symptoms worse.Afineorogastrictubeisarrested.plainX-rayshows Anyattempttofeedmakesthe andepisodesofcyanosis. A neonateisbornwithfrothysaliva intheneonatalICU. support compromise andisonventilatory respiratory the abdomenandchest.Theprematureneonatehasbeenbornwithsevere thepaediatricianstoacongenitalabnormalityaffecting A prenatalultrasoundscanalerted normally, withahugelydistendedabdomenandgrossdehydration. ababythathasnotbeenthriving shows that thebabyisundulyconstipated.Examination ofmeconium.Theparentsfeel ofdelayedpassage intermittent biliousvomitingwithahistory syndromebabyhasbeenbroughtinwithgradualabdominaldistension, A 2-month-oldDown’s Necrotising enterocolitis Hirschsprung’s disease atresia Biliary Intestinal malrotation Intestinal atresia Congenital diaphragmatichernia Tracheo-oesophageal fistula bilateral lointenderness. temperature of100°C, lookstoxic,listlessanddehydratedhasgeneralisedabdominal shehasa frequency. Onexamination andhasbeenvomiting.Shemarkedurinary anorexic which hasbeenrecurrentover6monthsorso.Onthisoccasionshelooksill,outofsorts, a5-year-oldgirl,hasbeenbroughtbyherparentswithgeneralisedabdominalpain Kerry, respiration. without anyrigidityorreboundtenderness,withtheabdominalwallmovingfreely awell-looking,introspectivechildwithgeneralisedabdominaltenderness revealed Examination within24h.Onthisoccasionithaspersisted. pain inthepast,whichsubsidedonitsown timesbecauseofsimilarattacksabdominal for2days.Shehasmissedschoolafew anorexia Millie, a10-year-old girl,hasbeenadmittedwithgeneralisedabdominalpain,vomitingand 77

6: PAEDIATRIC SURGERY PRINCIPLES ➜ ➜ 3.B,C,E 2.B,C,E 1. D Answers: Multiplechoicequestions 78

Paediatric trauma General surgicalmanagement repeated judiciously. ininfants. Interosseousaccessinto theuppertibiamaybenecessary 25 percentofblood volume)20mL/kgRinger’slactate solutionisusedasabolustobe cansuddenly deteriorate. as thechildwhoisbeingobserved orultrasound.Theteammustbe preparedtoanticipatetheneedforimmediateoperation CECT withcloseclinicalmonitoringsupplementedbyserial injuries areusuallymanagedconservatively isthe investigationofchoice.Liver,pancreatic,splenicandrenal computed tomography(CECT) canoccurwithoutanyfracturedribsorsternum. hepatic,pancreaticandsplenicinjury pulmonary, skeleton, underlyingsolidorganscanbedamagedwithoutoverlyingskeletal damage:cardiac, system.Because oftheelasticitychild’s vital organstotheskinandapoorthermoregulatory far moreseriousbecausethebodyhaslessfat,elasticconnective tissue, thereisproximityof of children.Thereforetraumaresultsinalargerforceappliedperunit of bodyarea.Theeffectsare deaths areavoidable.AdultATLS becauseof the smallerbodymass guidelinescannotbefollowed inchildhood;mostofthe isthemostcommoncauseofdeathand disability In theWest, injury and insufflationpressureshavetobetailored. access surgicaltechniqueshavealltheadvantagesasseeninadults;obviouslyinstruments balance.Minimal –helptomaintainoptimumfluidandelectrolyte orisotonicsaline dextrose with2.5percent than adultsundersimilarcircumstances.Intravenousfluids–0.45percentsaline problems. refluxmayresultinaspiration,causingpulmonary sodium needsarehigh.Gastro-oesophageal sodiumisimpaired;thereforefluidand vitamin K.Abilitytoconcentrateurineandconserve withintramuscular immune systems.Theeffectsofclottingdeficienciesneedtobeprevented core temperaturecloselymonitored. area),infusionsneedtobewarmed,awarmairblanketusedandthe a fifthofthebody’ssurface Therefore thetheatremustbewellheated,theirheadandneckcovered(theisalmost vasomotorcentre. have littlesubcutaneousfat(hencenonaturalinsulation)andanundeveloped systemisimmature. They are differentfromthoseinadults.Inchildrenthethermoregulatory beregardedassmalladults.Conditionsthatafflictchildrenbyandlarge Children shouldnever susceptible toinfection,whichmaymanifestwithnon-specificfeatures. easilyinthepostoperativeperiod.Animmatureimmunesystemrendersthemmore very of9percentuntiltheage14proportion years. andthisdoesnotequal anadult The headininfantsaccountsfor20percentofbodysurface and achildadolescentupto16 years. 5years; is calledaneonateupto4weeksinage;aninfant1year;pre-schoolchildbelow born before37 weeksiscalledapreterm;onebornbetween37 and42weeksisfullterm.Ababy tohaveaclearideaaboutthevariousagegroups.Ababy itisimportant In paediatricsurgery, Blunt ratherthanpenetratingtrauma ismorecommon.Ifthechildstable,contrast-enhanced With meticulousattentiontodetailalongwithgoodpainrelief,childrenrecovermorequickly The metabolicresponsetostressisinadequatebecauseoftheimmatureneurohormonaland The abilityforgluconeogenesisismuchimpairedininfants,whichrendersthemhypoglycaemic bladderisintra-abdominal. pelvis,theurinary infantsandsmallchildrenhaveashallow As A child’s blood volume is 80 mL/kg. Inshock(systolicbloodpressurefallsafterlossof A child’sbloodvolumeis80 ➜ ➜ ➜ 7. 7. D 6.A,B,D,E 5.A,B,D 4.C,E

Acute scrotal pain Undescended testis Inguinoscrotal swellings obstruction. torsion. Incarceratedherniawillpresent withvomitingandabdominaldistensionfromintestinal theseriousconditionoftesticular timeexcluding early cureoftheproblemwhileat same ispreferableasitresults in Excision of thetestis.Itcanbeleftaloneif diagnosisiscertain. boys.Sometimesthebluishappendagecanbeseenontop appendage occursinprepubertal be used,provideditdoesnotcompromisepromptnessoftreatment.Torsion ofthetesticular may reducedbloodflow Doppler ultrasound,aninvestigationnotusuallycarriedout,toshow Colour forthwith. asanemergency past. Onclinicalsuspicion,thescrotumshouldbeexplored Torsion ofintermittentpaininthe ofthetestiscanoccuratanyage.Theremaybeahistory chances ofearlydetection. butincreasesthe doesnotreducethechanceof malignancy more easilyidentifiable.Orchidopexy and toenableearlierdiagnosisofatumourwhenanyabnormality scrotaltestisismuch torsion,trauma, infertility toprevent Amaldescendedtestisshouldbebroughtdown orchidopexy. procedure ofchoice.Thetestiscanthenbelocalisedandmobilisedasastagedfor pouch. the testisisfixedinasubdartos operation.Thisisideallycarriedoutbeforetheage of 2yearsand child requiresanorchidopexy to theparents.Whenatestisispalpableinlineofdescentandnotretractiletestis, reassurance scrotum, theinfanthasaretractiletestis.Thisdoesnotrequireanytreatmentexcept testis. super ficial inguinalpouch,rootofthescrotumorfemoraltriangle,itisthenregardedasanectopic other hand,whenthetestisisfoundatasiteawayfromitsnormalpathofdescent,suchasin When atestisisarrestedinthenormalpathofdescent,itcalledanundescendedtestis.On of 2years,thepersistentprocessusisligatedthroughagroinincision. causing thehydrocele.Thisdoesnotrequireanysurgicaltreatment.Ifitispersistentafterage through;onlynormalperitonealfluidcomesintothescrotum, anybowel toallow is toonarrow operation. byemergency requires IVfluidresuscitationfollowed ill, dehydratedandtoxicwithadistendedabdomen,strangulationisimminentorpresent.This iftheinfantis However, theoedematosettledown. as anelectiveprocedure24hlatertoallow manual reductionunderanalgesia(taxis)canbeattemptedsothattheoperationdone it maybeincarcerated,resultinginvomitingandirreducibility. Intheearlystagesofobstruction, An inguinalherniainachildisindirectasitoccurspatentprocessusvaginalis.Sometimes should beinvolvedforthwith. betweenfamilymembers,NAIshouldbestronglysuspectedandpaediatricians inconsistent history trauma,whenthereisrepeatedtraumaand undue delayinseekingmedicaladvicefollowing When the severity of trauma is at variance with the degree of injury, whentherehasbeen oftraumaisatvariancewiththedegreeinjury, When theseverity When thetestisisnotpalpable,itmeansthatintra-abdominal.Laparoscopy intothe andthetestiscanbecoaxeddown When thescrotumisemptybutwelldeveloped, attheinternalring A congenitalhydroceleisapatentprocessusvaginaliswherethepatency 79

6: PAEDIATRIC SURGERY PRINCIPLES ➜ 2A 1C matchingquestions Answers: Extended 3B 80

pylorduodenal junction:Congenitalhypertrophic pyloricstenosis. F 1. Vomiting right upperquadrantincision. 1867–1963,German surgeonWilhelmConradRamstedt, in1911) isdonethroughatransverse by pyloromyotomy(anoperationfirstperformed After optimumresuscitation,aRamstedt’s (Fig.6.1).gastrograffin swallow maybeobtainedonultrasoundora epigastrium ontestfeed.Confirmation,ifnecessary, tetany. and,inlatecases,mayhavemusclespasmsfromalkalotic is hungry first 6weeksoflifeusuallyinafirst-bornmaleinfant.Typically thebabyhasnon-biliousvomiting, pyloricstenosis.Thisoccursduringthe Jonathan suffersfromcongenital(idiopathic)hypertrophic operationshouldbecarriedout. prophylactic antibiotics.Onceoptimised,anemergency a sinistersign.Theinfantshouldberesuscitatedwithintravenousfluids,nasogastricsuctionand orfaeculentvomitus is child istoxic,incarcerationmaybeproceedingtostrangulation.Brownish the As obstruction, whereasstrangulationmeanscompromiseofthebloodsupplytobowel. indicates anincarceratedinguinalherniaasthecause.Incarcerationintraluminal Bilious vomitinginaninfantisasignofintestinalobstruction.Anirreduciblelumpthegroin gastrograffin enema which shows atypicalcrabclawdeformity (Fig.6.2). gastrograffin enema whichshows theumbilicus. Confirmationisbyultrasound or a felt whichmightmigrate withconcavitytowards massmaybe andsometimesasausage-shaped (‘redcurrant jellystools’),anempty right iliacfossa invaginates intothecaecumandascending colon.Thereisbloodstainedmucusfromtheanus resulting instrangulatingintestinalobstruction.Usuallytheileum into theadjacentdistalpart, ofthebowel suffersfromintussusception. Thisistheinvaginationofaproximalpart Mary-Ann i gure 6 The biochemical abnormality of hypochloraemic, hypokalaemic, metabolic alkalosis iscorrected. metabolicalkalosis The biochemicalabnormalityofhypochloraemic,hypokalaemic, adehydratedbabywithhardmass(‘pylorictumour’)feltinthe shows Examination . 1 A gastrograffin swallowshowingnoflowofcontrastpastthe ➜ 3D 2A 1B

ileo-caeco-colic intussusception. F (a) i 2. Acuteabdomen cystogram and/orisotoperenogram. cystogram bymicturating andrenalultrasoundarethe initialinvestigations.Thisisfollowed examination anyabnormality. usuallydoesnotreveal from ascendingpyelonephritis.Clinical examination Urine long-termdeleteriouseffectsofrenalscarring promptly diagnosedandtreatedso astoprevent such aspelviuretericjunctionobstruction orvesicouretericreflux.Suchconditionsshouldbe tractabnormality tractinfection.Childrenusuallyhaveanunderlying urinary hasurinary Kerry admitted tohospitalwithacuteabdomen.Sometimesconstipationmay beacause. percentofchildren the latterdenotingpsychosocialproblems.Thisisadiagnosismadein 30–50 abdominal symptoms,nosignsofperitonismandmissingschoolonmore thanoneoccasion, Millie hasfeaturessuggestiveofnon-specificabdominalpain–recurrent attacks,generalised pneumonia. rightlower-lobe abdominal paininchildrentoexclude ofthechestshouldbedoneinall cases ofacute omentum.Completeexamination developed is commonbecauseoftheirinabilitytolocaliseintra-abdominalinfection asaresultofpoorly inchildren position oftheappendix.Theabdominalsignsaretypicalperitonitis. Perforation maybeatypical.Diarrhoeadenotesapelvic He hasacuteappendicitis.Inchildrenthehistory according tothefindings. resectionisdone andmanualreductionorbowel unsuccessful, laparotomyisundertaken gure 6 After fullresuscitation,radiologicalreductionwithairorcontrastenemaisattempted;if . 2 Barium enemashowingtypicalcrab-clawdeformityofacute (b) 81

6: PAEDIATRIC SURGERY PRINCIPLES ➜ 3A 2B 1F 4C 82

3. Congenitalmalformations Congenital megacolon,firstdescribedbyHaraldHirschsprungin1911, occursduetocongenital carried outwithinadayortwoofbirth. be present.Afterconfirmationofdiagnosis,referraltoapaediatricsurgeonismade.Operation andskeletalsystemmay kidneys anomaliesaffectingtheheart, oesophageal fistula.Associated is thediagnosis,commonestvarietybeingablindproximalpouchwithdistaltracheo- oesophagealatresia andcyanosis, whenaneonatepresentswithfrothysaliva Soon afterbirth hypoplasia. pulmonary failureduetosevere cases succumbtorespiratory isobtained.Almostathirdof initial management.Repairiscarriedoutifadequateoxygenation inthe isthekeynote support Respiratory soonafterbirth. therefore theteamisreadytointervene presentation withanemptyfeelingoftheabdomen.Thediagnosisisusuallymadeprenatally; embarrassmentisthekey This isacongenitaldiaphragmatichernia(Fig.6.3).Respiratory diseased segment;itisconfirmedbybiopsy. ofthe siteandextent theexact The diagnosisisestablishedbyacontrastenemawhichshows abnormality isrestrictedtotherectumandcolon.Enterocolitisadreadedcomplication. obstruction:in3outof4patientsthe absence ofintramuralganglioncellscausinglargebowel showing loopsofbowelinthelefthemi-thorax:Congenitaldiaphragmatic hernia. thorax: Congenitaldiaphragmatichernia. posterior viewofagastrograffin swallowshowingloopsofbowelinthelefthemi- and collapsedleftlungwithgasshadowsinthehemi-thorax. F duodenoduodenostomy. pyloricstenosis.Itiscaused byacompletemembrane.Theoperationis congenital hypertrophic appearance onplainX-rayisdiagnostic (Fig.6.4).Bile-stainedvomitingdifferentiatesitfrom ofintestinalatresia.The‘double-bubble’This isduodenalatresiawhichmay be apart (a) i gure 6 . 3(a) Plain X-rayofneonateshowingabsenceleftdomediaphragm (b) (c) Lateralviewofagastrograffin swallow (c) (b) Anterior 7G 6E 5D bilious vomiting and aspirate. A variable length oftheintestineisaffected. bilious vomitingandaspirate.Avariablelength The babypresentswithfeaturesoftoxicshock,abdominaldistension, bloodstained stoolsand diseaseoccurringintheprematureneonate. bowel Necrotising enterocolitisisaninflammatory radioisotope scanandliverbiopsyaretheinvestigationsofchoice. coagulopathy isaproblemwhichneedstobecombatedwithvitamin K. Anultrasoundscan, within 2weeks.Theincidenceis1in17 000.Inthepresence ofconjugatedhyperbilirubinaemia, doesnotsubside atresiashouldbesuspectedifthejaundiceinnewborn Congenital biliary basepredisposingtoamidgutvolvulus. has anarrow mesentery ontherightside.Thesmallbowel ahighcaecumwiththeduodenojejunalflexure meal shows The neonatehasbile-stainedvomiting,whichisasignofintestinalobstruction.Acontrast F (a) i gure 6 . 4(a) and(b) Double-bubble appearanceofduodenalatresia onplainX-ray. (b) 83

6: PAEDIATRIC SURGERY 7 Oncology

Multiple choice questions ➜ Malignant transformation ➜ Cancers associated with 1. Which of the following statements obesity about malignant transformation in 4. Which of the following cancers is cells are true? associated with obesity? A Cells become immortal. A Breast B Cells acquire angiogenic competence. B Kidney C Cells increase apoptosis. C Colon D Cells resist signals that inhibit growth. D Oesophagus E Cells evade detection/elimination. E Endometrium. ➜ Gompertzian growth ➜ Cancer screening 2. Which of the following statements 5. Which of the following are criteria for are true with regard to Gompertzian cancer screening? growth? A Sensitive and specific test A The majority of the growth of the tumour B Acceptable to the screened population occurs before it is clinically detectable. C Rare disorders B ‘Early tumours’ are genetically old. D Recognisable early stage C The rate of regression of a tumour E Treatment at an early stage to be as depends upon its age. effective as that at a later stage. D By the time of clinical detection, the window during which tumours are most ➜ Staging of colorectal cancer sensitive to antiproliferative drugs would 6. Which of the following concerning the have passed. staging of colorectal cancer are true? E The growth of the tumour has an A T3b refers to invasion of between 5 and exponential relationship. 15 mm beyond the muscularis propia. B N2 means involvement of four or more ➜ Causation of cancer regional lymph nodes. 3. Which of the following statements C V1 means intramural vascular invasion. about the causation of cancer are true? D T0 means tumour limited to mucosa. A Environmental factors have been E R0 means complete surgical resection implicated in more than 80 per cent of with adequate margins. cancers. B HPV infection is associated with cancer of ➜ Treatment of cancer the penis. 7. Which of the following statements C Familial breast cancer involving BRCA1 regarding surgery for cancer are and BRCA2 has an autosomal recessive true? inheritance. A The diagnosis of cancer should always be D Wood dust is associated with paranasal confirmed before surgery. sinus cancers. B Ultraradical surgery has a significant E Pituitary tumours are a part of MEN type role in reducing the incidence of distant 2A syndrome. metastases.

84 ➜ Extended matching questions Extended Choose and matchthecorrecttreatmentoptionwitheach ofthescenarios below: statements Whichofthefollowing 10. becured can Whichofthefollowing 9. statements Whichofthefollowing 8.

1 G F E D C B A 1. Treatment optionsin cancer E D C B A E D C B A E D C treatment ifindicated. has gradedittobe aDuke’sCtumourT3N1(2/20) M0 R0.Heisfitandkeenonanyfurther manhashadaright hemicolectomyforacaecalcarcinoma.Thepathologist A 60-year-old Palliative care forsecondaries Surgery Adjuvant chemotherapy Palliative surgery Chemoradiotherapy bysurgery Neoadjuvant chemoradiotherapyfollowed Curative surgery Medulloblastoma. Anal cancer Breast cancer Lymphoma Leukaemia without surgery? RT alone. cancerscanbecuredby Early laryngeal cycle. with thelateG2andMphasesofcell The fractionsshouldbetimedtocoincide beneficial thanshorter. longer overalltreatmenttimeismore Repopulation phenomenonsuggeststhat than others. Hypoxic cellsaremoreradiosensitive repair. Delivering RTinfractionsfacilitatescell are true? regarding radiotherapy (RT) forcancer colorectal cancer. in to beequallyeffectiveasopensurgery Laparoscopic approachhasbeenshown hasnoroleinpalliation. Surgery resection ofcolorectallivermetastases. aftersuccessful long-term survival Up toone-thirdofpatientscanexpect treatment aretrue? regarding chemotherapy incancer ➜ ➜ 12. Which of the following areend-of-life Whichofthefollowing 12. statements Whichofthefollowing 11.

End-of-life issues Palliative therap E D C B A E D C B A E D C B A issues? are true? regarding palliativetherapy incancer different modesofactions. referstouseofagentswith Synergy radiotherapy. combined useofchemotherapyand Spatial cooperationreferstothe stromal tumours(GIST). Imatinib isusefulingastrointestinal thymidylate synthase. Cis-platinum actsbyinhibitionof single agents. drugsareusuallyusedas Cytotoxic Bereavement. Spirituality Living wills Euthanasia withcurativeintent Active intervention Palliative careisessentiallypaincontrol. isoutsideitsremit. Spiritual support aspect. important Quality-of-life assessmentisan possible. hence shouldbedelayedasmuch An earlyreferralmaybedistressingand It mayinvolvesurgery. y incancer 85

7: ONCOLOGY PRINCIPLES ➜ ➜ Chooseandmatchthecorrect drugwitheachofthemechanismsactionbelow: 86

C B A 3. Appropriate chemotherapeuticagentselection 8 7 6 5 4 3 2 1 H G F E D C B A 2. Mechanismsofactiondrugsusedincancer 7 6 5 4 3 2 Imatinib Tamoxifen Goserelin Inhibition ofthymidylatesynthase. Inhibition ofdihydrofolatereductase replication fromunwindingandrepairing during DNA Inhibits topoisomerase1andhenceprevents with replication strandsandinterferes Forms adductsbetweenDNA cross-linkages This isaprodrugwhich,onconversion,causesDNA Blocks oestrogenreceptor withtheformationofmicrotubules–spindlepoisons Interferes Inhibition ofmutantc-KIT Tamoxifen Imatinib Irinotecan Cyclophosphamide Methotrexate 5-Fluorouracil (5-FU) Vincristine Cis-platinum problem. and keenonanytreatmentforherbowel fit suggestsalocalisedmasswhichiscausingtheobstruction.Sheotherwise obstruction. CT ovariancarcinomapresentswithfeaturesofintestinal femalewithknown A 46-year-old keentoavoidastoma. Heisvery a squamouscellcarcinoma(SCC). A 78-year-old malepresentswithalumpattheanalmargin.Biopsiesfromtheseconfirm spread. scanstagesthisasT3bN1. rectalcancerandMRI alow ofdistant reveal Thereisnoevidence Investigations malepresentswithrectalbleedingandalterationinhisbowels. A 68-year-old spread. extrahepatic of anyevidence scandoesnotshow lobe metastasis.Positron emissiontomography(PET) left- asmallsolitary scanshows CT levels. antigen(CEA) to haveraisedcarcinoembryonic forcoloniccarcinoma3yearsagoisobserved surgery fitmalewhounderwent A 56-year-old spread. oflocalordistant anyevidence scandoesnotshow confirmed onbiopsytobemalignant.CT asmallcaecaltumour femalepresentswithanaemia.Colonoscopyreveals A 50-year-old multiple liverandlungmetastases. disseminatedintra-abdominalcancerwith scanreveals symptoms ofintestinalobstruction.CT operationforrectalcancerpresentswith aprevious A 92-year-oldmanwhounderwent ➜ ➜ ➜ ➜ 4.A,B,C, D,E 3.A,B,D 2.A,B,C,D,E 1. A,B,D,E Answers: Multiple choicequestions Chooseandmatchthecorrectdrugwitheachofconditionsbelow:

invade, implantandformdistantmetastases. are detected, tumourswillhavehadplentyoftimeforindividualcellstodetach, By thetimethey clinical viability. implicationsforthediagnosis andtreatmentofcancer. important Thishasseveral Cancers associatedwith obesit Causation ofcancer Gompertzian growth Malignant transformation 6 5 4 3 2 1 F E D Maintaining ideal weight and regular exercise may be helpful in the prevention ofmanycancers. may be helpfulintheprevention Maintaining ideal weight andregularexercise cell tumours. type1 syndromewhichalsoconsistsofparathyroidandislet ofMEN tumoursareapart Pituitary hasanautosomaldominantinheritance. andBRCA2 Familial breastcancer involvingBRCA1 percentofcancers.Geneticfactorsactalongtheseenvironmental factorstocausecancer.80 cancer.or notanindividualdevelops Environmentalfactorshavebeen implicatedinmorethan determinantsofwhether areimportant Both environmentandinheritance(naturenurture) will containabout10 A tumour10 duringitsgrowth. cells producedmaybelost,mainlybyexfoliation, mmindiameter in competitionnotonlywiththehostcellsbutalsooneanother. Ninety-ninepercentofthe Thetumourcellsare rateisprobablybecauseofdifficultiesinnutritionandoxygenation. growth Thedecreasein rateslows. thegrowth butasthetumourgrows, isexponential, stages growth Initsearly growth. –so-calledGompertzian doubling timeofwhichincreasesexponentially relationship,the ofatypicalhumantumourcanbedescribedbyanexponential The growth communicationtoandfromtheenvironment/milieu. baggageandsubvert jettison excess acquire theabilitytoinvade,disseminateandimplant,genomicinstability, apoptosis, evade autonomous lineage,acquireindependencefromsignalsstimulatinggrowth, cellsestablishan The otheraspectsofmalignanttransformationincludethefollowing: Ovarian cancer.Ovarian Lymphomas Prostate cancer GIST Colorectal cancer Breast cancer Oxaliplatin Cis-platinum Vincristine 9 cells,whichimpliesitwouldtake30generationstoreachthethreshold of y 87

7: ONCOLOGY PRINCIPLES ➜ ➜ ➜ ➜ 11. A,C 10. C,D, E 9.A,B,D,E 8.A,D,E 7. C,E 6.B,E 5.A,B,D 88

Palliative therap Treatment ofcancer Staging ofcolorectal cancer Cancer screening The othercriteriainclude:treatmentatanearlystageismoreeffectivethanalaterstage;it pain control and includes several otherissuessuchassymptom relief,quality-of-lifeassessment, pain controlandincludes several a referralismadeearlyenoughinthe courseofthedisease.Palliativecareismuchmorethan factor inthesuccessfulpalliativemanagementofapatientisthat seamless. Themostimportant lastcancercell. Transition ofevery fromcurativecaretopalliativeshouldbe extirpation controland improvingqualityofliferatherthan The aimofpalliativetherapywillbe growth with replication. strands andinterferes overlapping toxicities.Cis-platinumactsbyformingadductsbetweenDNA are proventobeeffective.Agentsselectedwhichhavedifferentmodes ofactionandnon- areusuallyused incombinationswhich agentsarerarelyusedassingleandthey Cytotoxic medulloblastomaandskincancers. cancer, cancerofthecervix, andWilm’stumour, earlylaryngeal cancer, tumoursofchildhood,suchasrhabdomyosarcoma The othermalignancieswithpotentiallycurativenon-surgicaltreatment includesmall-celllung cell cycle. ItishencebeneficialtosynchronisetheRTwithmostvulnerablephasesof the cellcycle. aspossible.ThesensitivitiesofthecellstoRTvarieswithin if theoveralltreatmenttimeisasshort whereby whenRTkillscancercells,rapidproliferationofcellsisstimulated.Itthusbetter Repopulationisaphenomenon cells arehypoxic–thisisanobvioustherapeuticdisadvantage. andtumour cells.Normalcellsarewelloxygenated radio-resistant comparedwithwelloxygenated Hypoxiccellsarerelatively repair capacitybetweenthetumourandnormalcellscanbeexploited. Repair half-timesaretypically3–6h.Fractionation offersameanswherebyanydifferentialsin valuableforpalliationinsomecases. canbeextremely local recurrence.Surgery inobtainingacureandpreventing isoftheutmostimportance tumour atthetimeofexcision, takingcarenottodisrupttheprimary toappreciatethathigh-qualitymeticuloussurgery, however, ofmetastases.Itisimportant, probablyhaslittleeffectonthedevelopment ultraradical surgery recognisedthat but occasionallyasurgicalprocedureisrequiredtomakethediagnosis.Itnow iscarriedout, In mostcases,thediagnosisofcancerhasbeenmadebeforedefinitivesurgery tumour.5 and15 ofprimary vascularinvasion.T0meansnoevidence mm.V1referstoextramural T3b referstoslightinvasionofbetween1–5 themuscularispropia;T3cisbetween mm beyond insidious onset;andthebenefitmustoutweighphysicalpsychologicalharm. ifthediseaseisof screeningwillberepeatedatintervals to minimisemorbidityandmortality; high-qualitytreatmentforscreen-detecteddisease for thosewithapositivetest;thereexists adequatediagnosticfacilities inexpensive, sufficiently commontowarrantscreening;therearesafe, y incancer ➜ ➜ ➜ ➜ 1B,2F,3C,4A,5D,6E 1G,2B,3H,4E,5A,6F,7D,8C 1E,2G,3A,4F,5B,6C,7D matchingquestions Answers: Extended 12. B,C,D,E

3. Appropriate chemotherapeuticagentselection 2. Mechanismsofactiondrugsusedincancer 1. Treatment optionsincancer End-of-life issues andspiritualsupport. financialsupport socialsupport, support, nutritional informationandknowledge, physicalandpracticalsupport, psychosocial interventions, and the problem of medicalisation ofdeath. and theproblemofmedicalisation deathathome toallow physician-assistedsuicide,support appropriateness ofactiveintervention, the specificneedsofthosewhoarefacingbereavement.Theotherissuesatendlife tothefeelingsofapproachingdeathincludespiritualneed,profoundfearand more relevant of apatient’slife.Manyissuessuchassymptomcontrolarecommontobothareas.Problems months End-of-life careisdistinctfrompalliative–end-of-lifeconcernsthelastfew 89

7: ONCOLOGY 8 Surgical audit and research

Multiple choice questions ➜ Audit A Range is the value with the highest 1. Which of the following statements frequency observed. regarding audit are true? B Unpaired t-test is used to compare A It addresses clearly defined questions, two groups which are numerical and aims and objectives. normally distributed. B It measures against a standard. C A confidence interval which includes C It may involve randomisation. zero usually implies a lack of statistical D Re-audit is not necessary. significance. E There is no allocation to intervention D A P-value of <0.5 is commonly taken to group. imply a true difference. E Chi-squared tests are useful in comparing ➜ Research studies two groups that are categorical. 2. Which of the following statements ➜ regarding research are true? Research A A cross-sectional study is one where a 4. Which of the following questions series of patients with a particular disease should be answered before or condition are compared with matched undertaking research? control patients. A Why do the study? B Type 1 error is when benefit is perceived B Will it answer a useful question? when really there is none (false positive). C Will there be any financial incentives? C Randomised trials are essential for testing D Is it practical? new drugs. E What impact will it have? D It is a common practice to set the level of ➜ power for the study at 80 per cent with a Electronic information sites 5 per cent significance level. 5. Which of the following are electronic E A single-blind study is when the clinician information sites? is unaware of the treatment allocation. A Medline B EMBASE ➜ Statistical analysis in C ERIC research D NRES E Omni. 3. Which of the following statements regarding statistical analysis in research are true? Extended matching questions ➜ 1. Types of research study A Observational B Case–control

90 ➜ ➜ 1. B,E Answers: Multiplechoicequestions Chooseandmatchthecorrecttestwitheachofscenariosbelow: Chooseandmatchthecorrectstudywitheachofdescriptionsbelow:

Audit 5 4 3 2 1 E D C B A 2. Statisticaltests 6 5 4 3 2 1 F E D C interviews or questionnaires. Re-audit is an important part toclosetheloop. part orquestionnaires. Re-auditisanimportant interviews study. databutmayincludeadministrationofsimple Itusuallyinvolvesanalysis ofexisting group,whichisseeninaresearch orallocation tointervention does notinvolverandomisation reachapre-determinedstandard?This This isdesignedtoanswerthequestion: Doestheservice of bestcare. audit studyisdesignedandconducted toproduceinformationinformthedelivery criteria.An involves comparingaspectsofcare (structure, processandoutcome)againstexplicit Clinical auditisaprocessusedbyclinicians whoseektoimprovepatientcare.Theprocess To comparetwogroupswhicharenumericalandnormallydistributed. normally distributeddata To assesswhetheravariablehaschangedbetweentwotimepoints innumericalbutnot distributed data To assesswhetheravariablehaschangedbetweentwotimepointsinnumericalandnormally To comparetwogroupswhicharecategorical To comparetwogroupswhicharenumericalbutnotnormallydistributed Chi-squared test Wilcoxon’s signed-ranktest Mann-Whitney Unpaired Paired results. selecting asmallsimilargroupandexpanding Measurements aretakenoveraperiodoftime,notlookingatthewholepopulationbut results. small similargroupandexpanding Measurements madeonasingleoccasion,notlookingatthewholepopulationbutselecting This studyinvolvesacontrolgroupwhoreceivesstandardtreatment. prospective orretrospective. inadefinedpopulation.Thisstudycaneitherbe A conditionortreatmentisevaluated patients. diseaseorconditionarecomparedwithmatchedcontrol Series ofpatientswithaparticular Two randomlyallocatedtreatmentsarecompared. controlled Randomised Randomised Longitudinal Cross-sectional t -test t -test U -test 91

8: SURGICAL AUDIT AND RESEARCH PRINCIPLES ➜ ➜ ➜ ➜ 5.A,B,C,E 4.A,B,D,E 3.B,C,E 2.B,C,D 92

Electronic informationsites Research Statistical anal Research studies Research is designed to generate new knowledge and might involve testing a new treatmentor andmightinvolvetestinganew knowledge Research isdesignedtogeneratenew of allhealthandsocialcareresearch. ( National ResearchEthicsService available resources?’and‘Whatfindingsareexpected?’ itbeaccomplishedinthe available timeandwiththe The otherquestionstobeaskedare:‘Can value. The mean,medianandthemodealllieatsame parametric ornon-parametric.Normallydistributedresultshaveasymmetrical,bell-shapedcurve. isthelargesttosmallestvalue. collection. Range –usedfornominaldata observed ranked data.Modeisthevaluewithhighestfrequency –usedfornumericalor aboveandbelow middle valuewithanequalnumberofobservations Medianisthe Mean (average)istheresultofdividingtotalbynumberobservations. results. expanding single occasion,notlookingatthewholepopulationbutselectingasmallsimilargroupand matched controlpatients.Across-sectionalstudyisonewheremeasurementsaremadeona diseaseorconditionarecomparedwith study isonewhereaseriesofpatientswithparticular conceptual ortheoreticaljustifications. underpinnedby framework Thisusesaclearlydefinedsampling and relationshipsareexperienced. andusuallyinvolvesthewayinwhichinterventions establishedmethodology themes following groups.Qualitativeresearchidentifies/explores may involveallocatingpatientstointervention ones.Thestudydesign new particularly orcomparinginterventions, which mayinvolveevaluating broad types:quantitativeandqualitative.Quantitativeresearchisdesignedtotestahypothesis, regimen. Aresearchstudyaddressesclearlydefinedquestions,aimsandobjectives.Itisoftwo compare twogroups. variable haschangedbetweentwotimepoints. Whitney points. Whenanalysingnumericalbutnotnormallydistributeddata(e.g.tumoursize),aMann– to comparetwogroups,andapaired when therewasnorealdifference. only a1in20chancethatthedifferencebetweenvariableswould have happenedbychance The most important decisionforanalysisiswhetherthedistributionofresultsnormal,i.e. The mostimportant A single-blindstudyiswhenthepatientunawareoftreatmentallocation.case–control When analysingnumericalandnormallydistributeddata(e.g.bloodpressure), A achi-squaredtestisusedto When dealingwithcategoricaldata(e.g.admissiontoITU), P -value <0.05iscommonlytakentoimplyatruedifference.Thissimply meansthatthereis U -test isusedtocomparetwogroupsandWilcoxon’ssigned-ranktestassessifa y sis inresearch NRES) provides guidance regarding independent ethical review providesguidanceregarding independentethicalreview NRES) t -test toassessifavariablehaschangedbetweentwotime t -test isused ➜ ➜ 1C,2E,3A,4D,5B 1E,2B,3A,4F,5C,6D matchingquestions Answers: Extended

2. Statisticaltests 1. T y pes ofresearch stud y 93

8: SURGICAL AUDIT AND RESEARCH 9 Surgical ethics

Multiple choice questions ➜ informed consent B Children can unconditionally refuse 1. Which of the following statements are treatment. true with regard to Informed consent? C In patients who cannot give consent A Consent should be obtained by the because of their illness, e.g. they are person doing the operation. unconscious or there is psychiatric B The written communication material illness, their legal guardian can give must always be in English. consent. C Consent is necessary before physical D Therapy can proceed after consent examination of a patient. from a carer in an unconscious patient D Every possible hazard, however remote irrespective of any previous wishes of the the possibility, should be explained in patient. detail. E Legally, a signed consent from a patient ➜ Matters of life and death is proof that valid consent has been 3. In matters of life and death, which of properly obtained the following are true statements? ➜ A The surgeon is always obliged to provide Consent in difficult life-sustaining treatment. situations B Decision to withhold treatment should be 2. In difficult situations, which of the taken along with another senior clinician following statements are true? and recorded in detail. A There is no need to explain to children C In palliation for pain in advanced the procedures for which consent has malignancy, a potential lethal dose of already been given by their parent/ analgesia is appropriate. guardian. D Confidentiality is absolute.

Answers: Multiple choice questions ➜ Informed consent 1. A, C The surgeon who will carry out the operation should take the consent. The obtaining of consent should not be delegated to a junior member of the team who has not performed the procedure. Informed consent must be obtained before starting treatment. Informed consent denotes that patients understand what they are consenting to. Patients must be given information and choices so that they can make subsequent plans and decisions in future. They must be given appropriate and accurate information to agree to undergo surgical treatment and in a language that they can understand. Information should consist of: the condition and the reason for the operation; the type of operation; the prognosis with anticipated side-effects; unexpected complications; any alternative but successful treatment; and the outcome of not having the procedure carried out. Patients should be given the chance to ask questions and voice any misgivings.

94 ➜ ➜ 3.B,C 2. C

Matters oflifeanddeath Consent indifficult situations to prevent seriouscrimeorprotectindividuals whomaybeatrisk. to prevent order,orinanattempt ofothers,ifthereisacourt poses aseriousthreattothehealth andsafety havegivenimpliedconsent.Confidentiality cannotbestrictlyadheredtoifdoingso plan, they should thisinformationhelpinthe patient’s management.Whenpatientsconsenttoatreatment team is notabsolute.Surgeonsmaycommunicatewithothermembersofthe multidisciplinary this consent;todosowouldincurthewrathofGeneralMedical Council.Nevertheless, explicit premise of‘double effect’–painreliefanddeathmightfollow. depression,thushasteningdeath.Thisislegallyjustifiableonthe doses thatmaycauserespiratory increasingly difficult.Themanagementofpainundersuchcircumstances mayrequireanalgesiain shouldberecorded. details ofdecisionsandconversations A decisiontowithholdtreatmentshouldbetakeninconsultationwith a seniorcolleague.All thereispermanent braindamage. cases: ifdoingsoisfutile;deathimminentandirreversible; The surgeonisnotobligedtoprovideorcontinuelife-sustainingtreatment inthefollowing –‘alivingwill’. refusing specificintervention tothisruleiswhenthepatienthasalreadydrawnupalegallyvaliddocument The exception disability. lifeorprevent treatmentcanproceedtosave incompetent togiveconsentforsurgery, surgicaltreatmentcanproceed.Ifadultpatientsarepermanently provide consent,thenlife-saving areincompetentto surgical treatmentshouldbeassumedandhenceconsentsought.Ifthey psychiatriccare,theircompetencetoconsent to In patientswhoaredetainedforcompulsory management. this statement,thesurgeonmustrespectchild’sautonomywithregardtosurgical are18 cannotunconditionallyrefuseituntilthey care, althoughthey yearsold.’Notwithstanding consenttosurgical both adultsandchildren.‘UnderEnglishlaw,childrencanprovidetheirown shouldbesought.Thisisinkeepingwithpatientautonomy,for and wherepossibletheirviews tochildreninlayperson’stermstheproposedtreatment, The surgeonmusttakecaretoexplain record in the notes details of information given, particularly aboutcomplications. record inthenotesdetailsofinformationgiven,particularly weregivenappropriateinformation.Surgeonsshouldtherefore and dosometimesdenythatthey A signedconsentformisnoproofthatavalidhasbeenproperlyobtained.Patientscan should informpatientsofhazardsthatanyreasonablepersonintheirpositionwouldliketoknow. consent hasbeentaken. under thecareofsurgeon.Impliedconsentwillhavebeengivenbypatientonceaninitial timeapatientistobetouchedwhile every it constitutesbattery.thisisunnecessary However, mind. be abletomakeuptheirown interpreter, shouldbegivenwithadequatetimeforpatientstomullovertheadvicegiven,soas English. Written andverbalcommunicationinthepatient’spreferredlanguage,byuseofan It is not necessary to inform of every possible hazard, however remotethepossibility. possiblehazard,however Surgeons toinformofevery It isnotnecessary consentmustbeobtained;otherwise Prior totouchingapatientforphysicalexamination, The communicationneednotalwaysbeinEnglishasthepatientmayspeakorunderstand In thepresenceofpsychiatricillnessormentalhandicap,legalguardiancangiveconsent. A surgeonmustnotdiscussapatient’sclinicalconditionwithanyoneelse withoutthepatient’s There arecircumstanceswhenpalliationfordisseminatedandinoperable cancerisbecoming 95

9: SURGICAL ETHICS 10 Diagnostic imaging

Multiple choice questions ➜ Hazards of imaging and D MRI scan has no disadvantages. radiation E Radionuclide imaging allows function to be studied. 1. Which of the following statements are true? ➜ Orthopaedic imaging A Low-osmolality contrast media (LOCM) are safer than their higher-osmolality 4. In orthopaedic imaging, which of the counterparts. following statements are false? B Routine steroid prophylaxis is A Synovitis can be detected by plain X-ray. recommended before use of contrast in B MR arthrography is the ideal imaging for the high-risk patient. articular cartilage damage. C In the diabetic, metformin should be C MRI is the ideal method of staging a stopped before using contrast. malignant bone tumour. D The majority of ionising radiation D X-ray is the first investigation in comes from medical exposure from destructive bone lesions. investigations. E US is used to examine mass lesions of E Portable X-ray machines use much more soft tissues. radiation to achieve the same result. F Plain film of a joint is best for suspected acute joint infection. ➜ Diagnostic imaging ➜ Imaging in trauma 2. Which of the following statements is false? 5. In trauma imaging, which of the A Conventional X-rays will delineate following statements are false? different soft tissues reliably. A In a multiply injured patient, CT of head B Conventional X-rays can be manipulated. and spine should be the first line of C Dedicated transducers can help in imaging. endocavitary ultrasound (US). B Focused assessment with sonography D Change in the frequency of an US wave for trauma (FAST ) helps in detecting can be caused by red blood cells. intraperitoneal fluid and cardiac E The higher the frequency of the US wave, tamponade. the greater the resolution of the image. C CT should not be used when a patient is unstable. 3. Which of the following statements are D US is useful for diagnosing occult true about diagnostic imaging? pneumothorax. A US has no disadvantages. E CT is the main imaging method for B Computed tomography (CT) scan intracranial, intra-abdominal and has a higher resolution than plain vertebral injuries. radiographs. F Use of MRI in trauma is limited. C Magnetic resonance imaging (MRI) scans give excellent contrast resolution.

96 ➜ ➜ ➜ 2. A 1. A,E Answers: Multiple choicequestions statementsare Whichofthefollowing 6.

Diagnostic imaging Hazards ofimagingandradiation E D C B A abdomen Imaging intheacute cells. This change in frequency helpstomeasurethe speedanddirectionofmovement –the cells. Thischange in frequency wave,thegreaterisresolutionofimage. oftheUS Thehigherthefrequency gastric US. andthepancreasin the adjacentstructuressuchasmediastinal lymphnodesinoesophagealUS imagingofthewallviscusbutalso oesophagus andstomach).Thelatter notonlyallows imaging, suchastransvaginal,transrectalandendoscopic(of forintracavitary been developed obtained. systems andX-rayenergies,differentiationbetweenthedifferenttypes ofsofttissuescanbe of water.suchasmammography,by manipulatingtheX-ray circumstances,however, Incertain quantity Different softtissuescannotbereliablydistinguishedasall contain thesame result. Along withfluoroscopy,theseimagingequipmentsusemuchmoreradiation toobtainthesame accounting foronly12 X-ray machinesshouldbeavoidedasmuchpossible. percent.Portable reactions in1:10 000patients. diethyltriaminepentacetic acid(DTPA) canbeused.Mildreactionsoccurin1:200andsevere gadolinium ofiodineallergy, as allcontrastmediaarenephrotoxic.Inpatientswithahistory can begiveninthepresenceofnormalrenalfunction.Arecentserumcreatinineshoulddone afterinjection. occur shortly patient for30minafterinjectionisrecommendedwiththecannulastillinsitu.Mostreactions ofthe routine steroidprophylaxisforhigh-riskpatients.TheuseofLOCM andcloseobservation doesnotrecommend intheUK (RCR) higher-osmolality agents.TheRoyalCollegeofRadiologists scan,LOCMthanthe aremuchsafer For contrast-enhancedcomputedtomography(CECT) for suspected perforation orobstruction. for suspectedperforation Plain X-rayisthefirst-lineinvestigation obstruction. site ofbowel candiagnosethecauseand andCT US suspected uretericcolic. bladder) isthebestimagingfor ( Plain X-rayofKUB ureter, kidney, diverticulitis. isthebestinvestigationforacute CT isagoodfirst-lineinvestigation. US acute abdomen? false withregardtoimagingofthe A change in the frequency of the US wavecanbecausedbymovingobjectssuchasredblood ofthe US A changeinthefrequency Ultrasound isthesecondmostcommon methodofimaging.Specialtransducershave The majorityofionisingradiationinthehumancomesfromnaturalsources, medicalexposure recommendthatmetformincanbecontinuedandnomorethan100The RCR mLofLOCM ➜ 7. Which of the following statements Whichofthefollowing 7.

D C B A Imaging inoncolog resection. isusedinliver Intraoperative US andCT.detected byUS Liver andlungmetastasesarebest is notpossible. Accurate preoperativenodalinvolvement endoscopic US. Early diseaseisbeststagedby about oncological imagingaretrue? about oncological y 97

10: DIAGNOSTIC IMAGING PRINCIPLES 3.B,C,E ➜ ➜ 5. A 4.A,F 98

Imaging intrauma Orthopaedic imaging (PET/CT). whencombinedwithCT particularly tomog isasimilarimagingmethodwhichusefulinthedetectionofrecurrentcancer, raphy (PET) for ittobetrackedbyagammacamera,thusformingfunctionalimage.Positron emission gallium, thallium,iodine–iscoupledwithothercompoundsandadministeredintravenously magneticfields. becausetheinvestigationentailsuseofhigh-strength cannot beexamined needs tobemotionless,makingitdifficultinthosewithpain.Thosepatientsmetallicimplants itistime-consuming;andthepatient thereislimitedavailabilitybecauseofexpense; downsides: doeshavesome oftissueswithrelativelylittlenaturalcontrast.MRI lends itselftoimagingparticularly pictures forsolidrenallesions. invascularliverlesionsandthedelayed phase,forexample, of thebloodsupply,earlyarterial imagesatvariousphases with similarattenuationcharacteristics.Theinjectionofcontrastallows the processhasalonglearningcurve. pictures; itdoesnotgothroughairandbone;resolutiondependsuponthemachineused; bylookingatstatic during theactualscanningprocesssothatimagescannotbereliablyreviewed operator-dependent;theinformationismostlyuseful Ultrasound hasitsdrawbacks:itisvery solid organ–thusdiagnosingstenosiswithinavesselwall. hisprinciplein1842),articulated throughavesselor whichcanrecordthespeedofbloodflow physics, (ChristianJohannDoppler,aVienneseProfessorofexperimental principle ofDopplerUS pelvis and the cervical spine(C/S). pelvis andthecervical ofthe (intubated ornot), theinitialradiographsareX-raysof thechest,ananteroposteriorview andtheconditionofpatient areas tobeX-rayedwilldependupon themechanismofinjury will givearapidassessmentofmajorinjuries.Althoughthe Initially plainX-raysandnotCT guidance. aspirated underUS willdetectaneffusion whichcanthenbe film maybenormalunlessthereisboneerosion.US isused. solid,thenMRI an unidentifiablemassortheispartly onall palpablelesions;whenthereis Insoft-tissuelesions,theroutineshouldbeUS is performed. ifthelesionhasasolidelement toit,thenMRI imaging.However, it obviatestheneedforfurther toestablishafirmdiagnosis. are mandatory investigations issuspected,further distinguish malignantfrombenignlesions.Whenmalignancy interpretation isessentialto upsoft-tissuecalcificationinmuscle,tendonandfat.Careful shows whenmultiple lesionsaresuspected. necessary isparticularly in consultationwiththesurgeonisthencarriedout.Bonescanorwhole-bodyMRI needlebiopsy orMRI) CT staging ofboneorsoft-tissuemalignanttumours.Image-guided(US, gadolinium DTPA isthebestinvestigationfor isinjectedintothejoint,‘thegoldstandard’.MRI after whereascanisperformed arthrography, damage.MR is thebestwaytodetectarticular upsynovialthickening.MRI best detectedbygadoliniumDTPA-enhanced whichwillshow MRI, ofjoints,earlysynovitisis Although radiographsarethefirst-lineimagingusedforexamination The use of a radionuclide allows thestudyoffunction.Thechosenradionuclide–technetium, The useofaradionuclideallows contrastresolutionwithoutanyradiationhazard.It Magnetic resonanceimaginggivesexcellent theassessmentoftissues Computed tomographyscanhasahighcontrastresolution,allowing Ultrasound isthemostaccuratemethodofassessingacuteinflammation ofajoint,asplain Ultrasound isdiagnosticinthemajorityofmasslesionssofttissues. When thelesioniscystic, A plainX-rayisthefirstimagingtechniquewhenadestructivebonelesion issuspected.Italso ➜ ➜ 7. A,B,C,D 6. C

Imaging inoncolog Imaging intheacuteabdomen finding thatwillinfluence thedefinitivemanagement. smaller than1cm. Goodhigh-resolutionimagingwill detectsecondariesassmall0.5 cm –a are ifthey particularly andCT, secondaries maybemissedbythe conventional preoperativeUS may beoverlookedin10–30 percentofpatients. lungs andliver. C occult lesions deposits.However, T isthemostsensitivetechniqueforpulmonary reactive hyperplasia. astheenlargementcan beduetometastasisor involvement, onecannotbeabsolutelycertain (pararectal inrectalcancerormediastinaloesophagealcancer)gives anideaofnodal images. the US soasnottodistort thisshouldbedoneonlyaftertheUS time;however, same istrueofearlyoesophagealcancer.is theimagingofchoice.Thesame Biopsyiscarriedoutatthe endoluminal ultrasound(EUS) In thestagingofearlycarcinoma(T1andT2)distallargebowel, abdominal X-raytoincludethediaphragmaticdomesorlateraldecubitus film(ifthepatientistooill). closed-loopobstructionfromcarcinoma. confirmation ofacutelarge-bowel colonographyisincreasinglybeingusedinthe often identifythesiteandcauseofobstruction.CT isthemostsensitiveimagingprocedureforuretericcolic. ureteric stones.UnenhancedhelicalCT areaisoflimitedvalue.Faecoliths cannotbedistinguishedfrom organ), plainX-rayoftheKUB abscess. CT-guided drainageofanabscesscanalsobedone. wall,paracoliccollectionand/or thickeningofthebowel istheinvestigationofchoice–showing CT acuteappendicitis,pancreatitisandpelvicdiseases. cholecystitis, colic,acute Ultrasound isagoodinitialimagingformostacuteabdominalconditions–biliary detailed imagingofspinalinjuries. compatibility. Itismainlyconfinedto ventilated patientwithmonitoringequipmentandMRI onlyforthestablepatient. Hence thisimagingmodalityshouldbereserved body canbecompletedin5min–farlesstimethanittakestoorganisetheinvestigation. oftheentire fractures.Usingamultidetectorscannercomprehensiveexamination and vertebral motion ofthepleuraislostinapneumothorax;haemothoraxcanalsobediagnosed. probe willdetectthepleuraasanechogenicstripeanditsmovementcanbeassessed;sliding withadequateresuscitation. own whichshouldonlybeconsideredinapatientwhoisholdinghis should belostinarrangingaCT, used whentheinitialtesthasbeennegative. surgicalemphysema.ArepeatFAST gasorextensive be helpfulinthepresenceofbowel maybe efficientmethodtodetectintraperitonealfluidandcardiactamponade.FASTextremely maynot The value of emergency MRI islimitedbecauseofpracticalproblemsaccessibilitythe MRI The valueofemergency Computed tomographyistheidealmethodofimagingintracranialandintra-abdominalinjuries usingahigh-resolution Pneumothorax inasupinechestX-raycanbedifficulttosee.AnUS accordingtotheclinicalneedsandnotime The unstablepatientneedstobetreatedforthwith Focused assessmentwithsonographyfortrauma(FAST), althoughoperator-dependent,isan When patients present with left iliac fossa pain and a diagnosis of acute diverticulitis ismade, painandadiagnosisofacutediverticulitis When patientspresentwithleftiliacfossa Intraoperative US isroutineduring liverresectionformetastasis.Deep-seatedimpalpable Intraoperative US Ultrasound andC T aretheidealimagingmethodstodetecthaematogenous spreadtothe While thesizeofnodes Accurate preoperativenodalstagingcannotbeobtainedbyEUS. viscusisbestconfirmedbyanerectchestX-rayor ofahollow Clinical suspicionofperforation updilatedfluid-filledloopsandcan areusefulinshowing andCT In intestinalobstructionUS In suspecteduretericcolic(wronglycalledrenalbecauseonecannotgetinasolid y 99

10: DIAGNOSTIC IMAGING This page intentionally left blank PART 2 Investigation and diagnosis

11 Gastrointestinal endoscopy 103 12 Tissue diagnosis 107 This page intentionally left blank 11 Gastrointestinal endoscopy

Multiple choice questions ➜ Sedation in endoscopy C It is not necessary to stop clopidogrel 1. Regarding sedation during endoscopy, before a colonoscopic polypectomy. which of the following statements are D Verbal consent for an endoscopy is false? acceptable practice. A Sedation has no significant dangers and E Perforation and haemorrhage are can be used without restrictions. uncommon but significant complications B All sedated patients require secure of the procedure. intravenous access. ➜ C Co-administration of opiates and Post-ERCP pancreatitis benzodiazepines has a synergistic effect. 4. Which of the following are not risk D The use of supplementary oxygen is factors for post-ERCP (endoscopic essential in all sedated patients. retrograde cholangiopancreatography) E All sedated patients require pulse pancreatitis? oximetry to monitor oxygen saturations. A Young age B Difficult cannulation ➜ Antibiotic prophylaxis in C Increased bilirubin endoscopy D Pancreatic sphincterotomy E Balloon dilatation of biliary sphincter. 2. In which of the following should antibiotic prophylaxis not be ➜ Recent trends in endoscopy considered? 5. Which of the following about recent A Prosthetic heart valves developments in endoscopy are false? B Previous history of endocarditis A Chromoendoscopy involves the use of C Severe neutropenia stains or pigments to improve tissue D Chronic liver disease undergoing variceal localisation. sclerotherapy B Narrow-band imaging relies on an optical E Previous cholecystectomy. filter technology that radically improves the visibility of veins and capillaries. ➜ General issues in endoscopy C High-resolution magnifying endoscopy 3. Which of the following statements achieves near cellular definition of the regarding endoscopy are true? mucosa. A It is easy to get views beyond D Capsule endoscopy acquires video the ligament of Treitz during an images during natural propulsion oesophagogastroduodenoscopy (OGD) through the gut. with a standard endoscope. E Balloon enteroscopy permits visualisation B The current state-of-the-art endoscope is of the small bowel but is unable to the fibreoptic endoscope. perform therapeutic procedures.

103 INVESTIGATION AND DIAGNOSIS ➜ ➜ Extended matchingquestions Extended Choose and matchthecorrectdiagnosiswitheachofscenarios below: Chooseandmatchthecorrectdiagnosiswitheachofscenariosbelow: 104

5 4 3 2 1 E D C B A 2. Endoscopiccomplications 9 8 7 6 5 4 3 2 1 I H G F E D C B A 1. Endoscopicdiagnosis An 86-year-old female patient has severe bradycardia and low bradycardiaand low female patienthassevere An 86-year-old OGD. anuneventful female patienthaspainandbleeding frommouthfollowing An 86-year-old chestpainandchangeinvoice. oesophageal stricturecomplainingof severe 8hoursafteranendoscopicdilatationof An 70-year-old maleisrushed totheemergency Patient collapsesandisfoundtobe hypotensiveafteranendoscopicpolypectomy. room. intherecovery coughing andbreathingproblemsafter eatingasandwich afterlocalanaestheticsprayhassevere patientwhohasjust hadanOGD An 80-year-old Dental injury Sedation overdose Aspiration Perforation Bleeding oesophageal fluid. andincreased The gastro-oesophagealjunctionisat34cmwithprolapsingfundalmucosa for3cmintotheoesophagusabove. extending mucosa erythematous andalineartongueof ofgastro-oesophagealrefluxdisease(GORD) There isevidence obstruction. endoftheoesophagus,causing There isanulceratedandpolypoidalmassatthelower tobenormal. endoscopicbiopsiesconfirmthemucosa The superficial normal. isstretchedoverthislumpbutotherwise lumpisseen.Themucosa A submucosal lesionisseen. stretched butnomucosal capacity.The stomachisdifficulttodistendandappearshavelow appears Themucosa streaks oferythema. The pylorusisseentobewideopenwithplentyofbileinthestomach.antrumshows tearisseenatthecardiawithsomefreshblood. A mucosal inflammation andulceration. endoftheoesophagusalongwith A smooth,taperingstrictureisseenatthelower endoftheoesophaguswhicharedilatedand‘grape-like’. Abnormal veinsareseenatthelower stromaltumour Gastrointestinal Linitis plastica Oesophageal varices Reflux gastritis Hiatus hernia Peptic stricture tear Mallory–Weiss oftheoesophagus Cancer Barrett’s oesophagus P o 2 andisdifficultto arouse. ➜ ➜ ➜ ➜ ➜ 5. E 4. C 3. E 2. E 1. A Answers: Multiplechoicequestions

Recent trends inendoscop Post-ERCP pancreatitis General issuesinendoscop Antibiotic proph Sedation inendoscop small bowel andmaybe attemptedviatheoralorrectal route. small bowel for,theentire of,andtherapeuticintervention thedirectvisualisation Balloon enteroscopy allows performed. volumeofERCPs stoneandlow absent CBD related pancreatitisandpancreaticduct contrastinjection.Possible factorsalsoinclude female sex, normalbilirubin,priorERCP- complication includesuspectedsphincter ofOddidysfunction(SOD), pancreatitisisaround4.3percent.Theadditional riskfactorsforthis The incidenceofpost-ERCP endoscopic procedures. arerelatedto to theprocedure.Approximately1percentofmedicalnegligenceclaims intheUSA procedure andcomplicationsclearlytothepatienttakeafullyinformed signedconsentprior the toexplain andsedation-relatedproblems.Itishencemandatory as bleeding,perforation itisstillassociatedwithrarebutpotentiallylife-threatening complicationssuch endoscopy issafe, clopidogrel priortoacolonoscopicpolypectomyminimisetherisks of bleeding.Although monitor. Thepatientwillneedtostop which isprocessedbyacomputerandtransmittedtoTV inthe1960s. (CCD) coupled device thecreationofadigitalelectronicimage allows TheCCD endoscope wastheoriginalworkhorsebutbecameobsoleteafterintroductionofcharge- oftheduodenum.Thefibreoptic uptothesecondpart usuallyexamines A diagnosticOGD 5–10 IVifallergictopenicillin). minpriortotheprocedure(teicoplanin400mg depend onlocalpoliciesbutastandardprotocolis1gofamoxicillinand120 mgofgentamicinIV require antibioticsforprocedureswhichcausesignificantbacteraemia.Theantibioticregimenwill orregurgitation only moderate-risk conditionssuchasmitralvalveprolapsewithleafletpathology of infectiveendocarditisshouldhaveprophylaxisforallendoscopicprocedures.Patientswith history valvesoraprevious neutropenia,prostheticheart high-risk conditionssuchassevere percentafteranoesophagealdilatation.Patientswith of bacteraemiacanbebetween34and54 are2–4,4and11occluded commonbileduct(CBD) percent,respectively. Infact,theincidence foran andERCP The incidenceratesofbacteraemiaafteracolonoscopy,diagnosticOGD endoscopicproceduresareassociatedwithasignificantbacteraemia. certain prophylaxis. However, withouttheneedforroutineantibiotic safely The majorityofendoscopycanbeperformed agentsmustbereadilyavailable. equipment andsedationreversal assistant shouldbeavailableforpatientmonitoringthroughouttheprocedure.Resuscitation guidelines suggestthatthedoseofsedationishalvedinpatientsover70 yearsofage.Atrained isessentialinallsedatedpatients.Thecurrent andpulseoximetry use ofsupplementaloxygen anaesthesiamayincreasetherisk ofaspirationinsedatedpatients.The essential. Pharyngeal sedationpracticesare andhencesafe is thegreatestcauseofendoscopy-relatedmortality depressioninelderlypatientsorthosewithco-morbidities Medication-induced respiratory y laxis inendoscop y y y y 105

11: GASTROINTESTINAL ENDOSCOPY INVESTIGATION AND DIAGNOSIS ➜ ➜ 4F 3C 2D 1G matchingquestions Answers: Extended 9E 8A 7B 6I 5H 106

1C, 2A,3B,4E,5D 2. Endoscopiccomplications 1. Endoscopicdiagnosis H. pylori Increased duodenogastricrefluxresultsinbile-induceddamagetothepyloricarea.CLO testfor Endoscopyconfirmsthediagnosis,whichisself-limiting. oesophageal perforation. acutepancreatitis,myocardialinfarctionand differential diagnosesincludeduodenalperforation, epigastricpain,retchingandmildhaematemesis.Theclinical Patients presentwithsevere tapering andabsenceofshoulderingdifferentiatethisfrommalignancy. Barrett’soesophagusmaybepresent.Smooth This isacomplicationoflong-standingGORD. indesperatesituations. except indicatednowadays almost never is orvasopressinmaybeused.Surgery are usuallyinjectedorbanded.Morerarely,embolisation presentingwithhaematemesis.Thevarices hypertension This maybeseeninpatientswithportal perforation. oesophagitis. Thiscanrarelypresentwithcomplicationssuchasincarceration, necrosisand A smallhiatusherniamaybeanincidentalfinding.Someareassociated withfeaturesofreflux for surgery. Invasivecarcinomamaybefoundinalmosthalf ofthesecases. dysplasticchangeswould beastrongcandidate about itsmanagementbutmostagreethatsevere Thereisstillsomedebate dysplasticchangesneedsregularsurveillance. oesophagus showing endoftheoesophagusispresently increasing.Barrett’s incidence ofadenocarcinomaatthelower wasconfirmed.The afteritsassociationwithmalignancy This isbecomingincreasinglyimportant major resections.Prognosisstillremainspoor. beforeembarkingon computedtomographyscanandlaparoscopyareimportant endoscopic US, This causesprofoundweightlossandrapidlyprogressivedysphagia.Staginginvestigationssuchas respond welltoimatinib. attention. Prognosis,thoughvariable,isalotbetterthanadenocarcinoma.Evenadvancedcases This maybeanincidentalfindingbutcancauseuppergastrointestinalbleed,whichdraws poorprognosis. negative. Patientsusuallypresentlateandhencethiscarriesavery mayappeardeceptivelynormal(thoughabitstretched)andbiopsiesbe The mucosa

should bedone. 12 Tissue diagnosis

Multiple choice questions ➜ Specimens for histology ➜ Inflammation and cytology 1. Which of the following statements 4. Which of the following statements is regarding specimens for histology are untrue? false? A Inflammatory conditions are characterised A They must always be sent fixed in by the predominant cell type. formalin. B Cytology has some advantages over B They can be obtained by fine-needle histology. aspiration cytology (FNAC). C In an ulcerated tissue, ideally cytology C They are classified as biopsies and and biopsy should be taken from the resections. centre of the lesion. D Frozen section histology has many D Additional techniques may be necessary disadvantages. to elucidate the diagnosis. E Both macroscopic and microscopic E Special stains may sometimes be findings are reported. necessary. ➜ Cytology ➜ Immunohistochemistry 2. Which of the following statements are 5. Which of the following statements true with regard to cytology? are true with regard to A It gives as much information as immunohistochemistry? histology. A This is just a special staining method. B In some instances, an invasive method B It relies on the use of a specific antibody. has to be used to obtain material. C It helps to determine cell type and C A negative cytology has no value. differentiation. D There is risk to laboratory personnel D It has a role in the determination of associated with cytology and histology. treatment and prognosis. ➜ ➜ Principles of microscopic Special techniques diagnosis 6. Which of the following statements is false? 3. With regard to microscopic diagnosis, A Electron microscopy is routinely used in which of the following statements are histology. true? B Polymerase chain reaction (PCR) A Malignancy is diagnosed histologically is a useful investigation to detect by invasion, architectural changes and microorganisms. cytological features. C Study of chromosomes (cytogenetics) B Dysplasia indicates microscopic features can be done using fluorescence in-situ of cancer. hybridisation (FISH). C False-positive diagnosis can occur. D Autopsy can only be done with the D Tissue assessment helps in prognosis. coroner’s permission.

107 INVESTIGATION AND DIAGNOSIS ➜ ➜ ➜ 3.A,B,C,D 2.B.C,D 1. A,B Answers: Multiplechoicequestions 108

Principles ofmicroscopic diagnosis C Specimens forhistolog must bedisclosed tothepathologist,asradiotherapy changesmaymimiccancer. radiotherapy ofprevious interpretation which arebestavoidedbygoodclinical details.Ahistory intraepithelial neoplasia. oesophagus orcervical disease, oesophagealcarcinomainBarrett’s bowel e.g. colorectalcarcinomaininflammatory dysplasiaisregarded bymostasindicatingcarcinomainsitu, Severe moderate andsevere. abnormalities ofhyperchromatismand pleomorphism. blood vesselsandlymphatics;architecturalchanges;atypicalmitoticfigures; andnuclear areinvasionofneighbouringtissue, Features confirmingahistologicaldiagnosisofmalignancy maymeanthattheincorrecttissue hasbeenneedled. Absence ofcancercellsfromanFNAC hasnovalue. masses orfine-needleaspirationofliver,pancreasandkidney. Anegative cytology guidance formediastinallymphnodes,transbronchialfine-needleaspiration formediastinal whenfreshtissueisbeingsent. particularly infection,suchashepatitisBor tuberculosis, status. Thereisalwaysariskfortransmissable presence ofperivascularandlymphovascularinvasion,givesanideatheoestrogenreceptor ATru-Cut the biopsygiveshistologicaltissuewhichhelpstogradethetumour,mayshow FNAC. about thepresenceofmalignancy. Therefore,inabreastlump,cancercan be establishedby doesnotgiveasmuchinformationhistology. Itisastudyofcellsandgivesanidea Cytology resection margins,tumour,lymphnodalstatusandneighbouringnon-neoplastictissue. fromamalignantspecimenwillinclude Report arethenreported. for microscopicexamination Slices formalin, andafterabout24hadescriptionofthemacroscopicappearanceisreported. theprocedureistime-consuming. quality isinferior,therebycompromisingdiagnosticaccuracy; staff;the definitive treatment;thetissueisnotfixedandsothereariskofinfectiontolaboratory will nothaveapreoperativediagnosisandsocannotmakeaninformedchoicewithregardtothe small ulceratedskinlesions. aresectionisusuallyalsotherapeuticasin of breastlumporaprostaticnodule.Biopsyfollowing resections. Types ofbiopsyincludepunchbiopsy,asinskinlesions,andcoreTru-Cut orultrasoundguidance. out underCT needstobecarried breast andthyroidlumpsinlymphadenopathy. situations,FNAC Incertain isaninvasivemethodandhelpfulin givesanideaaboutthecelltype.FNAC value. Cytology lymph nodeissentfreshandtheremainderfixedinformalin. ofanexcised asinsuspectedtuberculosis.Insuchasituation,part assessment isnecessary aresentforfrozensection,whenmicrobiological aresentfixed.Fresh tissuesamples histology All specimensneednotbesentfixedinformalin(10 forroutine percentformaldehyde).Samples y Any specimen sent for histology is first sliced up into parts dependinguponthesize, fixedin isfirstslicedupintoparts Any specimensentforhistology thepatient The speedofadiagnosisafterfrozensectionisoutweighedbymanydisadvantages: specimensareclassifiedasbiopsiesand histology While biopsymeansanytissuesample, ithasalimited comparedwithhistology, onlygivescytology; Fine-needle aspirationcytology False-positive diagnosis canoccurfromcontaminationorinterchangingoftissue,pitfalls in Dysplasia isatermusedtoindicate microscopicfeaturesofcancer. Itisgradedasmild, canbeobtainedonlybyinvasive techniques,e.g.CT In someinstances,cellsforcytology tolog y y ➜ ➜ ➜ 6.A,D 5.A,B,C,D 4. C

Special techniques Immunohistochemistr Inflammation andc with the consent of the next of kin. with theconsentofnext require thecoroner’spermission;itcanbedoneforpurposeofmedical educationandaudit technique. Autopsydoesnotalways isdoneusingtheFISH chromosomes, calledcytogenetics, The techniqueisusedtodetectchromosomalabnormalitiesandmicroorganisms. Thestudyof isdetectedusingtechniquessuchaselectrophoresis. TheamplifiedDNA amplifiesDNA. PCR andisusedselectively.Electron microscopyistime-consuming,labour-intensiveandexpensive herpes virusandhepatitisB. Epstein–Barrvirus(EBV), (CMV), are antibodiestomanyinfectiveagentssuchascytomegalovirus selection oftreatmentandinthepredictionprognosis.Italsohasa role ininfections.There It determinescelltypeanddifferentiationsiteoforigin.Themethodhasaroleinthe which islabelledwithadyeand,whenboundtoitstargetantigen,seenascolouredstain. This techniqueisaspecialstainingmethod.Itdetectsspecificantigenusingantibody upamyloidosis. upbyreticulinstainwhileCongoredshows Prussian bluestain.Fibrosisisshown mucin inadenocarcinoma.Ironaccumulation,ashaemochromatosis,isdemonstratedbyPerls’ acid-Schiff (PAS) staindemonstratesglycogenandmucin;adiastasePAS up stainshows (D-PAS) additional techniquesavailable. are blocks,asinlinitisplastica.Specialstainsandimmunohistochemistry with extra levels non-viable tissue. necrosisand fromthecentreofanulcermayonlyshow and architecturalchanges.Asample withsomeapparentlynormaltissue.Thisenablesinterpretationofinvasiveness periphery invasive, fastandcheap;non-medicalstaffcanbetrainedinitsinterpretation. as aforeignbodyreaction. areseeninmycobacterialinfection(tuberculosisandleprosy),fungal and histiocytes, inflammation. Eosinophiliaindicatesparasiticinfestation.,acollectionofepithelioid andplasmacellsareseeninchronic indicates acuteinflammation.Presenceoflymphocytes The typeofcelldoesdeterminetheinflammation.Polymorphonuclear leucocytosis margins carry apoorprognosis. margins carry poorly differen tiated, high-gradetumours.Vascular andperineuralinvasionwithpositiveresection microscopi well-differentiatedtumourshaveagoodprognosisasopposedto cally: low-grade, Gradeisdetermined (UnionInternationalecontreleCancer). factor accordingtotheUICC Prognosis can be determined by tissue assessment. Stage is the important prognostic Prognosis canbedeterminedbytissueassessment.Stageistheimportant Special stainsareusedwhenroutineonesdonotprovidetheanswer. theperiodic For example, specimensmayneedtobeobtainedfromdeeper In aminorityofdifficultsituations,further When takingapieceoftissueforbiopsy(incisionbiopsy),itshouldbetakenfromthe itisless awiderareamaybesampled; doeshavesomeadvantagesoverhistology: Cytology y tolog y y 109

12: TISSUE DIAGNOSIS This page intentionally left blank PART 3 Perioperative care

13 Preoperative preparation 113 14 Anaesthesia and pain relief 118 15 Care in the operating room 122 16 The high-risk surgical patient 124 17 Nutrition and fluid therapy 127 18 Basic surgical skills and anastomosis 135 19 Laparoscopic and robotic surgery 137 20 Postoperative care 144 This page intentionally left blank 13 Preoperative preparation

Multiple choice questions ➜ Preoperative patient C There is no need to control preparation tachyarrhythmias preoperatively. D Preoperative transfusion should be 1. Which of the following are true of considered if the Hb level <10 g/dL. preoperative patient preparation? E In patients with malnutrition, A It includes a thorough history-taking and preoperative nutrition therapy should be medical examination. started 2 weeks prior to surgery. B The patient’s medical state is optimised C It is to anticipate and plan for ➜ Surgery in obese patients management of perioperative problems. D Good communication is required. 4. Which of the following is a problem E It involves taking informed consent. associated with surgery in obese patients? ➜ Preoperative investigations A Myocardial infarction B Aspiration 2. Which of the following statements C Deep vein thrombosis (DVT)/embolism regarding preoperative investigations D Pressure sore are true? E Pain control. A Chest X-ray is routinely requested in all patients over 60 years old. ➜ Surgery in the jaundiced B A ventricular ejection fraction of less than 35 per cent indicates a high risk of patient cardiac complications. 5. Which of the following is a problem C A body mass index (BMI) <15 is associated with surgery in the associated with significant hospital jaundiced patient? mortality. A Clotting disorders D ECG is usually required in patients above B Hepatorenal syndrome 65 years. C Infection E HIV testing requires patient consent. D Poor wound healing E Myocardial infarction. ➜ Preoperative management ➜ 3. Which of the following statements Surgery in the diabetic regarding preoperative management of patient specific medical problems are true? 6. Which of the following is a surgical risk A Patients with a diastolic pressure above in a diabetic patient? 95 mmHg should have their elective A Infection operations postponed. B Myocardial infarction B Elective surgery should be delayed C Pressure sore until at least 1 year after a myocardial D Poor wound healing infarction (MI). E Pain control.

113 PERIOPERATIVE CARE ➜ ➜ ➜ ➜ Extended matchingquestions Extended which Regardingconsentforsurgery, 8. isariskfactor Whichofthefollowing 7. Chooseandmatchthecorrectdiagnosiswitheachofscenariosbelow: 114

E D C B A 2. Perioperativemedications/interventions 7 6 5 4 3 2 1 G F E D C B A 1. Potentialoperativeandpostoperativeriskfactors Consent forsurger E D C B A DVT risks Smoking forAF Warfarin Diabetic medications–minorsurgery Steroids Antihypertensives obstruction,forwhichheisdue tohaveanoperation. small-bowel tohaveahiatusherniapresentswithsymptoms andsignsof A 45-year-oldobesemaleknown malnourished andjaundiced. carcinoma. Heis maleisscheduledtohaveanoperationforhisbowel An 86-year-old DVTandhasnotbeenmobileforthelast2months. previous of obesefemaleisscheduledtohaveanabdominaloperation. Shehasahistory A 60-year-old pepticulcer. laparotomyforaperforated needs tohaveanemergency spine andanunstablecervical A 70-year-old womanwithlong-standingrheumatoidarthritis of thecaecumwhichrequiresurgentsurgery. beendiagnosedtohavecancer 2monthsago.Hehasnow A 72-year-oldmalehashadanMI for thepast15 yearsandthecontrolhasbeenerratic. maleneedstohaveanurgentabdominaloperation.Hehashadtype2diabetes A 66-year-old disease(COPD). andlong-standingchronicobstructivepulmonary have severe A 76-year-oldto Sheisknown femaleisscheduledtohaveanelectiveoperationonherbowels. MI. embolism(PE) DVT/pulmonary Increased needforpostoperativeadmissiontoanintensivetreatmentunit(ITU) Postoperative infection Poor woundhealing Aspiration Difficult intubation Malignancy. Trauma Smoking Pregnancy Young age of the following aretrue? of thefollowing for thrombosis? y E D C B A consent. if anadultisdeemednotcompetentto reasonsfortheactions form explaining Two seniordoctorsneedtosignthe inacompetentpatient. surgery Consent isnotrequiredforlife-saving above 1percentshouldbediscussed. All minorcomplicationswithanincidence child underacareorder. A socialworkercangiveconsentfora cannot giveconsent. theageof16Children below years ➜ ➜ 2.B,C,D, E 1. A,B,C,D,E Answers: Multiplechoicequestions Choose and match the correct condition with each of the interventions below: Chooseandmatchthecorrectconditionwitheachofinterventions

Preoperative investigations Preoperative patientpreparation 8 7 6 5 4 3 2 1 H G F to guidepreoperative investigationrequests.Therequest formsneedtobefilledoutlegibly and Investigations should onlybeorderedwhenclinically indicated.Mosthospitalshaveprotocols Directive andduringhandoverbefore weekends/totheon-callteam. intheincreasing ‘shiftwork’cultureimposedbytheEuropeanWorking Time especially important implementation. plans to bemadeforimmediate contingency –comprehensiveplanningwillallow out asexpected caused bydelayagainstbenefitsoftreatingmedicalproblemspriorto surgery. setting,thereshouldbeabalance betweenpotentialharm elective operation.Inanemergency maximises benefits.Allmedicalproblemsshouldbeidentified,assessed andaddressedbeforean of caredeliveredbytheteam. admission todischarge.Itisalegallyadmissibledocumentandwould be areflectionofthequality findings, investigations,diagnosis,treatmentandtheprogress/problems encounteredfrom examination medical fileisalong-lastingrecordofallaspectspatientcare,includingthehistory, patientjourney.various responsibilitiesanddutiesthatareessentialforasafe memberofthatteam.Thetraineehencehas Atraineeisakey individual operatingexperience. seeingapatientthroughtheir A consultantsurgeonleadsalargeteamofpeopleinvolvedinsafely 2 months)willrequireincreaseddosestoavoidacrisis. Patients whohavebeenreceivingthisonaregularbasis(andreceivedwithinthepast eating. the patienthasstarted shouldbeomittedandmedicationsrecommencedonce The doseonthemorningofsurgery This shouldbestoppedatleastamonthpriortoelectivesurgery. This shouldbegivenforatleast2weekspreoperativelytohaveanyrealbenefit. patient recoversfromsurgery. dosewillneedtoberecommencedoncethe immediate postoperativeperiod.Theprevious andthenselectivelyasrequiredinthe The medicationscanbegivenuptothetimeofsurgery afterthesurgery. ontheevening andrestarted This isstopped3–4dayspriortosurgery isprolonged. Should becontinuedperioperativelyandchangedtoIVformsifstarvation Sliding scaleandIVfluids. Anticonvulsants. Insulin –majorsurgery Preoperative nutritionalsupport Good communicationwiththepatientandamongstmembersof teamisvital.This thingsdonot turn manyofthemtobeavoided.Inevitably, allows Anticipating adverseevents minimisesriskand Good preoperativeplanningtooptimisemedicalproblemsbeforesurgery The Recording informationaccurately,completelyandlegiblyisofparamountimportance. 115

13: PREOPERATIVE PREPARATION PERIOPERATIVE CARE ➜ ➜ ➜ ➜ 6.A,B,C,D 5.A,B,C,D 4.A,B,C,D 3.A,E 116

Surger Surger Surger Preoperative management infection,biliuria,glycosuriaandinappropriateosmolality.detect urinary problems.Dipstickanalysisofurineisusuallycarriedoutroutinelypreoperatively.respiratory Itcan significant bloodlossesareexpected. oranaestheticproblemsandbeforemajoroperationswhere ofcardiovascular,pulmonary history isindicatedinallpatientsover65,thosewitha conditions.Electrocardiogram(ECG) respiratory of bleedingdiathesis. screen isindicatedinanypatientonanticoagulants,withcompromisedliverfunctionorevidence orclottingproblems.Clotting poornutritionalreserves hypertension, portal hepatitis, malignancy, problems andinpatientsonregulardiuretics. orrenal ofcardiac,pulmonary lose asignificantamountofbloodintheatre,thosewithhistory patients. chronicbloodlossandelderly inmenorrhagiaandthecaseofanyotherknown is expected, appropriate actionsinstituted. clinicaldetails.Theresultsshouldbecheckedpromptly,recordedand completely, givingrelevant fluid and electrolyte balance. fluid andelectrolyte complications thatoccurmoreoften indiabeticsincludeCVA, renalproblemsand with cardiovascular, peripheralvascularand neurologicalstatusshouldalwaysbemade.Other Diabetic patientsareathighriskof complications. Acarefulpreoperativeassessmentoftheir fluids. tobody Infective causesofjaundicemayposearisktomembersstaffpotentially exposed lifting, transferringandoperatingtableweightlimits. (CVA), compromise,poorwoundhealing, infectionandmechanicalproblemssuchas respiratory intheobeseincludedifficultintubation, cardiovascularaccident The otherproblemsofsurgery whenitislife-threatening. except tractinfectionsshouldbetreatedbeforesurgery respiratory lower cardiacvalves.Significant inpatientswithprostheticorleaky antibiotic coverisusuallynecessary third-degree blocksmayrequirepacingpreoperatively. 6 monthshaveelapsed.Fast atrialfibrillation(AF)mustbecontrolledbeforesurgery. Second-and shouldideallybedelayeduntilatleast anaesthesia within3monthsofinfarction.Electivesurgery mayrequireinvestigationstolookforanunderlyingcause. diagnosedhypertension Newly Patients withsystolicpressuresof160 deferred. mmHgoraboveshouldhaveelectivesurgery A preoperative full blood count (FBC) isneededinalloperationswheresignificantbloodloss A preoperativefullbloodcount(FBC) Chest X-ray is not usually required unless the patient has a significant cardiac history or Chest X-rayisnotusuallyrequiredunlessthepatienthasasignificantcardiachistory areoccasionallyrequiredfordetailedassessmentofchronic bloodgases(ABGs) Arterial Liver functiontests(LFTs)orsuspected areindicatedinpatientswithjaundice,cirrhosis,known (U&E)arenormallyrequiredinallpatientsover65,whomay Urea andelectrolytes Preoperative transfusionshouldbeconsideredifthepreoperativeHb<8g/dL. from isastrongcontraindicationtoelectiveanaesthesia.Theresignificantmortality Recent MI y y y inthediabeticpatient inthejaundicedpatient inobesepatients ➜ ➜ ➜ ➜ matchingquestions Answers: Extended 8.B,C,E 7. B,C,D,E

1G, 2H,3D,4A,5F,6E,7C,8B 2. Perioperativemedications/interventions 1E, 2D,3G,4A,5F,6C,7B 1. Potentialoperativeandpostoperativeriskfactors Consent forsurger DVT risks There are well defined risk factors for DVT including age > 40 years, burns, major surgery, hip years,burns,majorsurgery, There arewelldefinedriskfactorsforDVTincludingage>40 discussed andhasnomorequestions,beforebeingaskedtosign. toensurethatthepatienthasunderstoodwhatbeen be discussed.Finally,itisimportant complications thataresignificantorhaveanincidenceofatleast1percentmoreshould operation shouldbeoutlinedandconfirmedwiththepatient.Alternativetreatmentsall consent. Firstly,thepatient’sdemographicdetailsshouldbechecked.Secondly,planned possible complicationsandalternatives.Thereshouldbeastructuredapproachtoobtaining including withplannedsurgery, The personobtainingtheconsentmustbefullyconversant history. personal orprevious limbamputationandfamilyhistory paraplegia,lower and otherjointsurgery, y 117

13: PREOPERATIVE PREPARATION 14 Anaesthesia and pain relief

Multiple choice questions For the following questions, choose the single ➜ Suxamethonium best answer. (succinylcholine) ➜ The anaesthetic triad 4. What is the most frequent complaint made by patients in whom 1. Which of the following is not part suxamethonium (succinylcholine) has of the anaesthetic triad used during been used? surgery? A Pain at the site of injection A Unconsciousness B Prolonged action in those with B Pain relief pseudocholinesterase deficiency C Amnesia C Diplopia D Muscle relaxation. D An increase in body temperature E Diffuse muscle pains. ➜ Laryngeal mask airway 2. What is the most significant ➜ Lidocaine vs bupivacaine disadvantage of the laryngeal mask 5. Lidocaine can be injected airway (LMA) over an endotracheal intravenously, but what is the main tube? reason why bupivacaine should not A Failure to provide a competent airway be injected into a vein during local B Risk of pulmonary aspiration anaesthesia? C Unreliable placement A It lasts longer. D Enhanced risk of tube obstruction B It is often used with adrenaline. E Failure to allow tracheal suction. C It can cause methaemoglobinaemia. D It may cause convulsions. ➜ Correct placement of an E It is cardiotoxic. endotracheal tube 3. What is the most reliable way to ➜ Bier’s block ascertain correct placement of an 6. A Bier’s block is a useful technique endotracheal tube? to provide anaesthesia for upper A Detection of a pressure waveform on limb surgery. Which is the best local inflation anaesthetic to use? B Direct visualization A Prilocaine C Detection of breath sounds on B Lidocaine C Levobupivacaine D Measurement of end-tidal carbon dioxide D Bupivacaine concentration E Amethocaine. E Movement of the chest wall on manual inflation.

118 ➜ ➜ ➜ ➜ 3. D 2. B 1. C Answers: Multiple choicequestions greatlyintheir Patients vary 8. Whatistheextra riskinvolvedwhen 7. 9. Which of the following isassociated Whichofthefollowing 9.

Correct placementofanendotrachealtube Lar The anaesthetictriad E D C B A Assessment ofpain E D C B A on thecapnograph–carbondioxide monitor. beingabletohearbreathsounds, Unfortunately, becomeastandardwithinWesternnow practicetoverifycorrecttubeplacementbyseeingatrace often afailuretoacceptthepossibility thatthetubeisnotintracheaacauseofdelay. Ithas consequences,and Unrecognised incorrectplacementof anendotrachealtubecanhavedisastrous used correctly. endotracheal tube.Unreliableplacementandtubeobstructionshould be nodifferentifbothare withriskofaspirationisthemainproblemLMAoveran An unprotectedairway donotrememberit. as they aslong thatitdoesnotmatterwhatthepatientexperiences components sincesomemaybelieve debating pointastowhetheritwouldbereasonableofferamnesia without theother degrees forallprocedures.Amnesiamayoccurbutitisnotactuallysought.Itaninteresting invarying The anaesthetictriadconsistsofunconsciousness,painreliefandmusclerelaxation Examine forwoundsplinting. Examine Look fortachypnoea. ofhypertension. thelevel Assess the patienttomeasurethepain. Ask Measure thedegreeoftachycardia. That toxicityismorelikely. frequent That failureislikelytobemore Because adrenalinecannotbeused The cuffismorelikelytodeflate. thelimb exsanguinating The difficultyinadequately assess adequacy ofpainrelief? assess adequacy analgesia. Whatisthebestwayto requirement forpostoperative surgery? a Bier’sblockisusedforlower-limb with toomuchanalgesia? y ngeal maskairwa y ➜ 10. Which of the following shouldbe Whichofthefollowing 10. 11. Which of the following isnotan Whichofthefollowing 11.

Chronic paincontrol E D C B A E D C B A E D C B A conditions? avoided forpaincontrolinmalignant Codeine Pethidine Fentanyl Remifentanil Small tidalvolumes. Deep veinthrombosis Depression ofconsciouslevel Agitation Hypocarbia Tramadol. Antidepressants Corticosteroids Radiotherapy Tamoxifen of chronicpain? forthetreatment alternative strategy Methadone. 119

14: ANAESTHESIA AND PAIN RELIEF PERIOPERATIVE CARE ➜ ➜ ➜ ➜ ➜ 10. A 9. C 8. B 7. E 6. A 5. E 4. E 120

Chronic paincontrol Assessment ofpain Bier’s block Lidocaine vsbupivacaine Suxamethonium (succin arenotsufficientlyreliable. Mackintosh laryngoscope seeing thechestmovingandactuallyvisualizingtubegoingthroughvocalcordswitha a syrup,subcutaneous orintravenous. preparations–skinpatches, oralas the otheragentscould beusedandareoftenin varying andisnotusedinchronicpaincircles.Any of short-acting anaesthetic (TIVA) regimen. Itisvery ofatotalintravenous opioidwhichisoftenusedaspart Remifentanil isanultra-short-acting rate, oftenwithlargetidalvolumes,andadepressionofconsciouslevel. Too respiratory muchpainrelief,usuallyassociatedwithopioiduse,willresult inhypercarbia,slow 10 attheother. cmline,with‘nopain’atoneendand‘theworstpain ever’ relief, or,moreformally,toaskthemcompleteananaloguescorewhere amarkisplacedon ofpainreliefistoaskthepatient aboutthequalityofpain The bestwayofassessingtheadequacy aresubjectivemeasures. Although alltheotheranswersarefeaturesofinadequatepainrelief,they statedabove,bupivacaineshouldnotbeused. and bestagenttouse.As Although otherlocalanaestheticscanbeusedforBier’sblock,prilocaineisadvocatedasthesafest block). Bupivacaineisconsiderablymorecardiotoxicthanotherlocalanaesthetics. spinal andepiduralblocks.BupivacaineshouldnotbeusedforIVregionalanaesthesia(Bier’s Lidocaine andbupivacainearebothlocalanaestheticsusedforinfiltration,fieldblocks muscle fasciculations. in theposturalmusclesandmostoftenyoungfitmales.Thisisthoughttoberelated surprisingly largenumberofpatientsdocomplaingeneralizedmuscleachesandpains,usually atropine.A drugs usedduringanaesthesiacanalsoincreasebodytemperature,mostimportantly increase inbodytemperature,butagainthepatientwouldnotcomplainofthis.Interestingly,other haswornoff. willstillbeanaesthetisedwhentheeffectofsuxamethonium becausethey however, areunlikelytocomplainofdiplopia, Patientswhohavehadsuxamethonium muscles oftheeye. becauseofthesensitivityorbital returned. Diplopiacanoccurwithanymusclerelaxant has tokeepthemanaesthetiseduntilnormalmusclepower time becauseitwouldbenecessary andliverdiseases.Patientswouldnotcomplainofbeingparalysed foralong include pregnancy ofacquireddeficiency Examples inthisenzyme. in thosewithaninheritedoracquireddeficiency ismetabolisedbypseudocholinesteraseanditseffectprolonged most cases.Suxamethonium occurs.Muscletonereturnsafter3–4minin beforerelaxation with musclefasciculationsshortly ofthedepolarisingclass.Itsuseisassociated musclerelaxant isashort-acting Suxamethonium Suxamethonium is a known trigger for malignant hyperpyrexia whichcanbeassociatedwithan triggerformalignanthyperpyrexia isaknown Suxamethonium y lcholine) 11. 11. E the WHO pain ladder – it is an intermediate-strength opioidand,assuch,wouldhavebeentried pain ladder–itisanintermediate-strength the WHO whenrelatedtocancer.chronic pain–particularly Tramadol isanagentonthesecondrungof analgesicsareusedasadjunctivetherapyinpatientswith Many agentsotherthanwellknown before adjunctivetherapy. 121

14: ANAESTHESIA AND PAIN RELIEF 15 Care in the operating room

Multiple choice questions ➜ Diathermy C Eyelids should be taped to protect the 1. Which of the following statements with corneas. regard to diathermy are false? D Extra precautions are needed if the A Shave the patient’s hair over the site for patient is to be held in a lateral position. the diathermy plate the day before the E All of the above. operation. ➜ B Ensure good contact between the patient Scrubbing and the plate. 4. In scrubbing, which of the following C Check the plate if the patient is moved statements are false? during surgery. A If the surgeon has a suspected infected D Place the plate as close to the operative lesion, it is sprayed with iodine and site as possible. covered with a sterile dressing before E Make sure that the patient is not gloving. touching any earthed metal objects. B The first scrub of the day should take about 5 min from start to drying. ➜ Use of a tourniquet C A sterile scrubbing brush and nail cleaner 2. Which of the following statements are used for 1–2 min at the first scrub regarding use of a tourniquet are true? provided the surgeon stays within the A The theatre charge nurse has overall theatre suite in between cases. responsibility in its use. D After applying disinfectant, the arms B Distal neurovascular status must be are washed from distal to proximal with checked before and after its use. hands up and elbows flexed. C The tourniquet must be placed as E Drying, using a towel for each side, proximally as possible. should start with the fingers and work D The tourniquet must be placed snugly across the hand and up the arm. enough so as not to slide during the ➜ operation. Skin preparation E Always note the time of inflation and 5. In skin preparation prior to operation, deflation. which of the following statements are true? ➜ Set-up on the operating A In preparing open wounds, aqueous table solutions are used. B For intact skin, alcohol-based solutions 3. In a transfer and patient set-up on the may be used. operating table, which of the following C Prepare the skin from the incision site statements are true? outwards. A Although the operating table is padded, D Heavily contaminated areas are prepared make sure that pressure areas have last, with the swab being discarded. additional padding. E All of the above. B Limbs not involved in surgery should be especially protected to prevent nerve damage.

122 ➜ ➜ ➜ ➜ ➜ 5. E 4. A 3. E 2.B,C,D,E 1. A Answers: Multiplechoicequestions

Skin preparation Scrubbing Set-up ontheoperatingtable Use ofatourniquet Diatherm A surgeonshouldnotscrubifthereisanopenwoundorasuspectedinfectedlesion. deflation. havereturnedafter andthatcirculationsensation sure thatallprecautionshavebeenfollowed mustmake Surgeons alwayshaveoverallresponsibilityregardingtheuseoftourniquet.They organisms. by anaesthetised. Earliershavingshouldnotbedonebecauseitencouragesskincolonisation The sitewherethediathermyplateistobeappliedshouldshavedafterpatienthasbeen y 123

15: CARE IN THE OPERATING ROOM 16 The high-risk surgical patient

Multiple choice questions ➜ High-risk patient B Stopping smoking prior to surgery is of 1. Which of the following groups little benefit. constitute ‘high-risk’ patients? C Oral medications can still be given with A Elderly a little water in patients who are nil-by- B Ethnic minority mouth. C Significant co-morbidities D A course of steroids prior to surgery may D Emergency surgery be necessary in patients with chronic E Complex major surgery. obstructive airways disease. E Preoperative physiotherapy has no role. ➜ Preventable factors of mortality in high-risk patients ➜ Goal-directed therapy 2. Which of the following are preventable 5. Regarding goal-directed therapy (GDT), factors of mortality in high-risk patients? which of the following statements are A Pain true? B Insufficient patient monitoring A It is a term used to describe the C Lack of early intervention as perioperative administration of complications develop intravenous (IV) fluids and inotropic D Advanced age agents to achieve a predefined ‘optimal’ E Inadequate critical care facilities. goal for oxygen delivery to the tissues. B This aims to reduce cardiac and renal ➜ Risk scoring systems output. 3. Regarding risk scoring systems, which C This has no effect on patient survival if of the following statements are true? used for short periods after surgery. A ASA does not provide an objective D Observations have shown that patients assessment of the risk of death. who survive have higher cardiac output, B POSSUM works best for individuals, not oxygen delivery and oxygen consumption population. than those who did not. C MET measures exercise tolerance. E GDT uses a minimally invasive cardiac D PARSONNET is used in neurosurgery. monitoring technique. E Anaerobic threshold is time-consuming ➜ and needs measuring several days prior Management strategies in to surgery. high-risk patients ➜ 6. Which of the following statements Preoperative management are true about specific management of high-risk patients strategies for high-risk patients? 4. Regarding preoperative management A Prophylactic beta-receptor antagonist of the high-risk patient, which of the agents have been used to decrease following statements are true? perioperative myocardial infarction. A A course of antibiotics should always be B Oesophageal Doppler-guided fluid given in patients with chronic sputum therapy is an accurate estimation of the production. patient’s fluid needs.

124 ➜ ➜ ➜ 2.A,B,C, E 1. A,C,D,E Answers: Multiplechoicequestions matchingquestions Extended Choose and match the correct intervention witheachofthescenariosgivenbelow: Chooseandmatchthecorrectintervention

Preventable factorsofmortalit High-risk patient 6 5 4 3 2 1 F E D C B A 1. Interventions/assessmentinthehigh-risksurgicalpatient D C Dehydration and hypothermia are also preventable factors thatincreaseperioperativerisk. Dehydration andhypothermia arealsopreventable largeanticipatedbloodloss and largeinsensiblefluidloss. body cavitysurgery, surgery, majororcomplex surgery, nutritional tolerance.Thesurgicalfactors includeemergency toleranceandpoor disease, diabetes,advancingage,poorexercise chronic obstructivepulmonary for complicationsordeath. undergo generalsurgicalprocedureseachyear,ofwhom166 000canbeidentifiedasathighrisk 1.3more frequent.Thisrepresentsthehigh-riskgroup.InUK millionhospitalin-patients 10–15 percentofin-patientsurgicalprocedures, inwhichseriouscomplicationsanddeathare thereisasubgroupcomprising 1percent.However In mostcases,thisriskiswellbelow surgicalprocedureinvolvessomeriskofsignificantpostoperative complicationsordeath. Every The mainbenefitsaretheabsenceofneedforageneralanaestheticandintubation. management. This isdeliveredviaatight-fittingmaskandhelpfulinpostoperativerespiratory bloodgases. forarterial patient, andenablesrepeatedbloodsampling This facilitatesimmediaterecognitionofhaemodynamicchanges,especiallyinanunstable improves cardiacoutput,renalcomplicationratesandpatientsurvival. totissues.This delivery ofoxygen predefinedlevels This referstotreatmentaimedatachieving This isusedtomeasurecardiacoutputguideIVfluidadministration. status. infusions tostabilisetheircirculatory This isinvaluableincriticallyillpatientswhorequireIVfluidreplacementsorvasoactive This isusedinpatientsconsideredtobeatriskofperioperativemyocardialischaemia. Oesophageal Doppler Prophylactic perioperativebeta-blockade Non-invasive ventilation Goal-directed therapy outputmonitoring Cardiac pressuremonitoring Arterial operative ileusafterabdominalsurgery. Better fluidmanagementcanreducepost- beneficial. patients withestablishedcriticalillnessis Sustained useofgoal-directedtherapyin Some of the patient-related factors which make them high risk include ischaemic heart disease, Some ofthepatient-relatedfactorswhichmakethemhighriskinclude ischaemicheart y inhigh-riskpatients E associated withreducedhospitalstay. Early returntoentericfeedingisnot 125

16: THE HIGH-RISK SURGICAL PATIENT PERIOPERATIVE CARE ➜ ➜ ➜ ➜ ➜ 1E,2B,3F,4C,5A,6D matchingquestions Answers: Extended 6.A,B,D 5.A,D,E 4.C,D 3.A,C,E 126

indicators, asarealcoholandtobaccointakeinformationonmedicationsregularlytaken. areuseful ageandBMI tolerancegivesagoodguidetocardiacreserve, problems. Exercise reliable whenappliedtoindividuals.PARSO 1. Interventions/assessmentinthehigh-risksurgical patient Management strategiesinhigh-riskpatients Goal-directed therap Preoperative managementofhigh-riskpatients Risk scorings POSSUM andPARSOPOSSUM the Enhanced Recovery Programme(ERP). the EnhancedRecovery of be harmful.Earlyreturntoentericfeedingisassociatedwithreducedhospitalstayandpart Sustained useofGDT inpatientswithestablishedcriticalillnessisnotbeneficialand,fact,may in fact,maybeharmful. to notethatsustaineduseofGDT inpatientswithestablishedcriticalillnessisnotbeneficialand, postoperative GDT usingaminimallyinvasivecardiacoutputmonitoringtechnique.Itisimportant withupto8hof Significantreductionsincomplicationratecanbeachieved patient survival. Goal-directed therapyaimstoimprovecardiacoutput,renalcomplicationratesand helpful forpatientswithchronicsputumproduction. Preoperativephysiotherapyis possible withtheofferofcounsellingandotherpracticalsupport. shouldbeencouragedwherever treatment ofanysubsequentpneumonia.Smokingcessation selection ofresistantbacteriawithoutanytherapeuticbenefit,and,worsestill,maycomplicatethe should begivencarefulconsideration.Indiscriminateantibioticusemaysimplyresultinthe A courseofpreoperativeantibioticsistemptinginpatientswithchronicsputumproductionbut Preoperative assessment of risk should include a history that focuses on cardiac and respiratory thatfocusesoncardiacandrespiratory Preoperative assessmentofriskshouldincludeahistory y NNET stems systems weredesignedforpopulationsofpatientsandaremuchless y NNET is used incardiacsurgery. 17 Nutrition and fluid therapy

Multiple choice questions ➜ Starvation B Diabetic ketoacidosis (DKA) 1. Which of the following statements C Inborn errors of metabolism regarding starvation are true? D Starvation A Cerebral energy metabolism E Insulin therapy. requirement is 100 g/day of glucose. ➜ B Glycogenolysis comes into play in the Fluid compartments first 2–3 days. 5. Which of the following statements C Mobilisation of fat is dependent on a regarding water homeostasis are fall in circulating insulin levels. true? D Intravenous dextrose has a protein- A The total body water (TBW) content in sparing effect. an adult male is 70 per cent of body E Ketone bodies cannot substitute for weight. glucose for cerebral energy metabolism. B Two-thirds of TBW is intracellular. C An average adult has approx. 3 L of ➜ Total parenteral nutrition plasma. 2. A 67-year-old female is referred for D TBW is highest in elderly women. consideration of total parenteral E Water moves freely across cell nutrition (TPN) following a total membranes. colostomy. Which of the following ➜ biochemical and clinical markers are Malabsorption suggestive of malnutrition? 6. Which of the following statements are A Albumin untrue? B Urea A The small bowel receives approximately C Transthyretin 7–8 L of fluid/day. D Skin fold thickness B Malabsorption is inevitable if 30 per E Weight loss. cent of the small intestine is removed. C The jejunum is vital in the absorption of ➜ Serum proteins and disease nutrients. 3. Which of the following is a cause of D Vitamin B12 and bile salts are absorbed hypoproteinaemia? in the ileum. A Burns E The enterohepatic circulation of bile B Syndrome of inappropriate antidiuretic salts is essential to maintain the bile salt hormone hypersecretion (SIADH) pool. C Liver disease D Analbuminaemia ➜ Starvation E AIDS. 7. Which of the following statements regarding starvation are true? ➜ Ketoacidosis A Fats are initially mobilized to meet 4. Which of the following is a major energy requirements. cause of ketoacidosis? B Muscle glycogen can be directly utilized. A Alcoholism C After a short fast, insulin level falls.

127 PERIOPERATIVE CARE ➜ ➜ ➜ ➜ 11. The following resultswereobtained Thefollowing 11. statements Whichofthefollowing 10. statements Whichofthefollowing 9. statements Whichofthefollowing 8. 128

Pre-anal Bowel resections Gastrointestinal secretions Intravenous fluids E D C B A E D C B A E D C B A E D bowel resection. bowel stonesarecommonerfollowing Oxalate syndrome. short-bowel Peptic ulcerationisacomplicationof of water. output fromjejunostomytodrinkplenty It isappropriateforpatientswithhigh mmol/L. fistulas isabout90 The sodiumcontentofhighoutput resection. ileal Diarrhoea isunusual,following andpotassium. normal saline Most intestinallossesarereplacedwith with hypochloraemicalkalosis. outletobstructionisassociated Gastric potassium. fluidhasthehighestquantityof Gastric around 110 mmol/L. The chloridecontentofgastricfluidis Pancreatic fluidisrichinbicarbonate. hypovolaemia. It isthebestfluidtobeusedin It doesnotcontaindextrose. inpotassium. It islow sodium andchloride. It hasequimolarconcentrationof as plasma. sodiumconcentration It hasthesame liver. Gluconeogenesis onlytakesplaceinthe occurs. fast,adaptiveketogenesis After ashort with melaena:Na, 133 mmol/L;K, on anelderlygentleman seeninA&E resectionsaretrue? regarding bowel secretions? are trueofgastrointestinal(GI) are true? regarding 0.9percentnormalsaline y tical errors ➜ ➜ ➜ 3 A15-year-old girlwasadmittedwith 13. areassociated Whichofthefollowing 12. 14. A 6-year-old child has been unwell A6-year-oldchildhasbeenunwell 14. What conditionarethesefeatures

imbalances Diarrhoea andelectrol H Acidosis E D C B A E D C B A E D C B A • • y povolaemia K, 3.0 mmol/L; Cl, 90 mmol/L; K, 3.0mmol/L;Cl, 90mmol/L; results wereobtained: Na,143 mmol/L; irritable anddehydrated. Thefollowing of milk.Onexamination,thechild is any foodandhasbeendrinkingsips 2 days.Hehasbeenunabletotake with abdominalpainandvomiting for follows: acidosis. Herinvestigationsareas She ishypokalaemicwithametabolic after returnfromholidayinKenya. abdominal painandseverediarrhoea with hypovolaemia? cause? (ALP), undetectable.Whatisthe (ALT), alkalinephosphatase 40IU; alaninetransaminase (AST), 32 IU; 0.82 mmol/L;aspartatetransaminase urea, 12.7 mmol/L;correctedCa, 10 mmol/L;creatinine,153 μmol/L; All oftheabove. Liver failure Contaminated blood Renal failure K Coeliac disease. diarrhoea Secretory Zollinger–Ellison syndrome Tropical sprue syndrome Carcinoid consistent with? 248; K,50osmolality, mmol/L Diarrhoeal fluid:Na,77 mmol/L; Cl, 115 mmol/L;HCO3, 13 mmol/L Blood: Na,147 mmol/L;K,2.0 Increased urineoutput. Hypotension sodiumexcretion Decreased urinary Uraemia Tachycardia + overdosage y te ➜ ➜ ➜ Extended matchingquestions Extended obstruction ofsmall-bowel Incase 15. scenarios(theaboveoptions clinical Choose andmatchtheprobabilitieswithfollowing

2. Fluidandelectrol 5 4 3 2 1 1. Riskofrefeeding s vomiting Metabolic consequencesof can be usedmorethanonce): can D C B A C B A B A E D C B A A 64-year-old male with coeliac disease and BMI of19 malewithcoeliacdiseaseandBMI A 64-year-old kg/m admission withdysphagia. malewithoesophagealcancerhaslost12A 46-year-old kginthepreceding 4weekspriorto of26kg/m A chronicalcoholicwithaBMI is 14.7 kg/m alaparotomyforbleeding duodenalulcer. underwent HerBMI A youngwomanwithanorexia 25 kg/m order. is his deportation Hehasonlyconsumedsipsofsweet tea duringthistime.HisBMI A 27-year-oldasylumseekerwhoisfitandwellonahungerstrikefor2daysprotesting weakness andhypokalaemia. Multiple myeloma Fluid overload SIADH Pancreatitis probability ofrefeedingsyndrome Low Intermediate probabilityofrefeedingsyndrome High probabilityofrefeedingsyndrome There isariseinurineosmolality. The ureaandcreatininecanrise. Malnutrition. sepsis Severe Lactic acidosis ketosis Starvation DKA this metabolicpicturesuggest? 6 mmol/L(<0.3).Whatconditiondoes 119 μmol/L;beta-OH-butyrate, 7.5 mmol/L;urea,7.6 mmol/L;creatinine, 16bicarbonate, mmol/L;glucose, statements aretrue? and vomiting,whichofthefollowing 2 . 2 . y te abnormalities y ndrome 2 isadmittedwithbleedingvarices. ➜ 16. Which of the following statements Whichofthefollowing 16.

Acid–base balance E D C B A E D C regarding acid–basebalancearetrue? A raisedaniongapcanoccurinethanol A raisedaniongapcanoccurinDKA. myeloma. A decreasedaniongapcanoccurin Bicarbonate isthemajoranion. Na andKarethemajorcations. rises. ADH Hyperchloraemia isacommonfeature. excretion. Na There isincreasedurinary intoxication. 2 presentswithgeneralized 129

17: NUTRITION AND FLUID THERAPY PERIOPERATIVE CARE ➜ ➜ ➜ 2.C,E 1. A,C,D Answers: Multiplechoicequestions scenariosdescribedbelow: Chooseandmatchtheconditionsabovewithclinical Choose and match the conditions above with the clinical scenariosdescribedbelow: Chooseandmatchtheconditionsabovewithclinical 130

Total parenteral nutrition Starvation 4 3 2 1 3. Metabolicpresentations in 4 3 2 1 is also prone to dilutional effect, type of protein consumed and presence of GI bleed. is alsopronetodilutional effect,typeofproteinconsumed andpresenceofGI couldstillbenormal despitenutritionalinadequacy.long half-lifeofabout 20daysandlevels Urea ithasa Albumin ispronetovariationwithdilutional state(e.g.IVfluids),liverdiseaseetc.Further arenothelpfulinanin-patientsetting. indirect measuresandaresubjectto hugevariation–they disease. Anthropometricmeasurements suchasskin-foldthicknessandmidarmcircumferenceare recentweightlossandpresenceorabsenceofacute (BMI), consideration bodymassindex ScreeningToolThe MalnutritionUniversal foradultsiswidelyusedandtakesinto (MUST) The bestwayofassessingnutritional supplementationisanestimationofweightloss. metabolism. asasubstituteforglucosecerebral energy lactate. Ketonebodiescanserve fromaminoacids,glyceroland mobilizing glycogenstores.Thereafter,gluconeogenesisisachieved inthefirst24hby thebodyfacesaneedtogenerateglucose.Thisisachieved During starvation, D C B A blood results are as follows: glucose,7.2blood resultsareasfollows: mmol/L;K A 56-year-old man presents with visual disturbance and low Naof110. manpresentswithvisualdisturbanceandlow A 56-year-old hypothalamichormones. T4 andlow T3, acraniotomyforsubduralhaematoma.Hehaslow following manisinITU A 68-year-old forjaundice.HeisonslidingscaleinsulinandK cholangiopancreatography (ERCP) A youngdiabeticisrecoveringafterundergoinganendoscopicretrograde nmol/L. glucose, 2.2mmol/L;ammonia256 circumcision.Hisbloodresultsareasfollows: seizuresfollowing A maleinfantdeveloped K, 3.3mmol/L;urineNa,30mmol/L. results:Na.130An 82-year-oldmanwithabronchialtumourhasthefollowing mmol/L; Na, 124 mmol/L;urineNa,<5.0mmol/L. mmol/L;K,4.5serumosmolality,250 profile: thefollowing alaparotomyforpancreatitisshows A 62-year-oldmanwhounderwent AST/ALT/amylase, unabletobeanalysed. mmol/L; totalcholesterol,6.0mmol/L;triglycerides,66 symptoms. Hisbloodtestsareasfollows: A 25-year-oldmanundercareoflipidologists,presentswithvomitingandnon-specificabdominal mmol/L; totalprotein,102 g/L;albumin,30creatinine134 μ mol/L. Na,123 operativerepairtostabilisethefractureareasfollows: following mmol/L;K,3.9 fracture.Bloodtests An 87-year-old avertebral womanisadmittedtohospitalfollowing Response toillness tumour Pituitary Inborn errorofmetabolism Transcellular shift y oung andoldpatients + , 3.5mmol/L;PO 4 ,

0.30 mmol/L. + IVI. His IVI. ➜ ➜ ➜ ➜ ➜ ➜ 8.B,D 7. C,D 6.B,C 5.B,C,E 4.A,B,C,D 3.A,B,C,D,E

load (Na Na/K-ATPase pump. Intravenous fluids Starvation Malabsorption Fluid compartments Ketoacidosis Serum proteins anddisease Normal saline has 154 mmol/L of sodium and chloride. It contains no dextrose orK has154Normal saline mmol/Lof sodiumandchloride.Itcontainsnodextrose Subsequently, fatsaremobilized. ofaminoacids. Continued fasting(>24h)resultsin gluconeogenesisfrombreakdown tolactatewhichismetabolisedinthe livertousefulglucose. well. Glycogenhereisconverted ofliverglycogen). Additionalstoresofglycogenarepresentinthemuscleas (breakdown glycogenolysisoccursintheliver begintorise.Inearlystarvation, as glucagonsandcortisol hormonessuch declinesandcounter-regulatory anovernightfast,insulinlevel Following and thusreducedabsorptionofthefat-solublevitaminsA,D,EK. poolresultsinfatmalabsorption Depletionofthebilesalt offluidandelectrolytes. conservation intestinaltransittimeandthehighestabsorptivityofnutrients.It is thuscriticalforthe the slowest couldberesectedwithoutpermanent effects.Theileumhas percentofthesmallbowel up to50 Only 200–300cmofsmallintestineisneededtomeetthenutritionalneedsanindividual.Thus, NaandK response tochangesintonicityofadjoiningcompartments. is 3L.Theremainderof16 Lisinterstitialfluid.Watermovesfreelyacrosscellmembranesin is42L,intracellularwater is23Landplasmavolume age. Inanaveragemaleof70 kg,TBW and Thereisvariationwithsex proportion. whileelderlywomenhavealower ofTBW proportion Total percentoftotalbodyweight.Infantshavearelativelyhigher bodywaterisapproximately60 increased aniongap. ketogenesis, thereisconsumptionofbicarbonateions,whichresultsinmetabolicacidosiswithan glucose. Thistriggersreleaseoffreefattyacidsasanalternativesourceenergy.low During insulin:glucagonratiowhenthereis Ketoacidosis isametabolicresponseofthebodytolow increaseinotherproteins. condition inwhichpatientslackalbumin.Thereisoftenacompensatory lesionsandinprotein-losingenteropathy.in burnsandlargeexudative Analbuminaemiaisabenign Losscanalsooccurviatheskin states suchaswateroverload,specimenfromdriparmorSIADH. the mostabundantproteinandisproducedinliver. Hypoproteinaemiacanoccurindilutional globulin, acutephaseproteins,Apolipoproteins,immunoglobulinsandclottingfactors.Albuminis largenumberofdifferentproteins,suchasalbumin, The termplasmaproteinsdescribeavery closely, butforahigher quantityoflactate,andisthe preferredreplacementfluidinhypovolaemia. the assay isnotroutinelyavailable. the assay Transthyretin half-life,makingitagoodmarker,but isalsocalledprealbuminandhasashorter + ) is excessive compared with plasma. Hartmann’s solutionresemblesplasmamore compared withplasma.Hartmann’s ) isexcessive + areinfluencedbythe + . Thesolute 131

17: NUTRITION AND FLUID THERAPY PERIOPERATIVE CARE ➜ ➜ ➜ ➜ ➜ 13. D 12. A,B, C,D 11. C 10. B,D,E 9.A,B,D,E 132

polypeptides could causeasimilarpicturebutarenot consistentwithclinicaldetailsprovided. cholerae diarrhoea.Itiscausedbytoxinssuchas (such asglucose)whichareseenin secretory <10 sodium. mmol/Lofurinary Diarrhoea andelectrol H Pre-anal Bowel resections Gastrointestinal secretions load, e.g.inhypernatraemicdehydration. solutionsareconsideredwhenpurewaterreplacementiswithoutelectrolyte Dextrose a common association with old, stale or haemolysed blood samples. K a commonassociationwithold,staleorhaemolysedbloodsamples. calculated osmolalityis2(Na) mechanismtothelossofbicarbonate. compensatory hyperchloraemia istypicalofthekidney’s asaresultofdehydration.The metabolites viadiarrhoealfluid.TheNaismildlyelevated bicarbonate),whichisduetolossofthese andmetabolicacidosis(low suggest hypokalaemia diarrhoea(possiblyduetoparasiticinfestation). Theelectrolytes This isconsistentwithsecretory correction byretainingNa toattempt for healthykidneys reduced urineoutputaretypical.Biochemicallythereisatendency refilltimeand Clinically tachycardia,hypotension,reducedskinturgor,increasedcapillary Intravascular volumedepletionisassociatedwithcharacteristicclinical and biochemicalfeatures. calcium andMg.EDTA isoftenfoundasK clues.EDTA serumcalciumandALPactivityarefurther Thelow cause ofhyperkalaemia. chelates Ethylenediaminetetra-acetic acid(EDTA, usedforfullbloodcount)contaminationofisa absorptionfromthecolonarenotuncommon. stones duetoincreasedoxalate gastricacidsecretionandrenal could worsenthesituation.Peptic ulcerationduetoexcessive often requireparenteralsupplementation,asdrinkinglargeamountsoforalhypotonicsolutions ofintactjejunumandlosewaterthroughtheirstoma.They portion secretors onlyhaveashort absorbers haveover100 Net cmofintactjejunumandabsorbmostthefluidelectrolytes. isresected,ajejunostomybecomesessential.Net to diarrhoea.When>200cmofthebowel fluidreachthecolon,predisposing ilealresection,largevolumesofbilesalt-laden Following hasahighK content andlossoftheseresultsinmetabolicacidosis.Saliva and chloridecontent(140 and100 mmol/L,respectively),pancreaticfluidhasahighbicarbonate secretionsaresimilarintheirsodium faeces andinsensiblelosses.Duodenal,pancreatic,biliary tract(ifappropriate)urine, Fluid replacementtherapyisbestmanagedbyassessingdeficitfromGI and gastricjuicehasahighchloridecontent(around110 mmol/L). or drugs)willnotcausechangesincalciumALPlevels. y This differenceinosmolalityof42mmol/L isduetounmeasuredosmoticallyactivesubstances The stool shows excretion of electrolytes. Thestoolosmolalityismeasuredat248but ofelectrolytes. excretion The stoolshows povolaemia and y E. coli. tical errors Hormonalcauses suchasZollinger–Ellisonandvasoactive intestinal + sothatintravascularvolumeincrease.Thisresultsineliminationof ϩ Kandis204. y te imbalances + salt in full blood count bottles. Hyperkalaemia isalso in fullbloodcountbottles.Hyperkalaemia salt + overload(bysupplements + content(25mmol/L) Vibrio ➜ ➜ ➜ 3B ➜ 2A 1C matchingquestions Answers: Extended 16. A,C,D,E 15. A,B,E 14. B 4A

Ca andMgaretheothercations. Ca when thereareparaproteinaemiastates. andethanolintoxication.Itcanbereduced lacticacidosis,renalfailure,salicylate such asDKA, are amongtheotheranions.Normalaniongapisaround8–16. inmetabolicacidosis, Itiselevated 1. Riskofrefeeding s Acid–base balance Metabolic consequencesofvomiting Acidosis with BMI <18.5with BMI kg/m inpatients withalcohol-inducedliverdisease,especiallyifassociated mustbeexercised Caution refeeding syndrome. nutritional intakeformorethan10 of daysare,individually, high-riskfeaturesforthedevelopment Starvation ketosis is commoner in children due to low glycogenstores.Thechildisableto ketosisiscommonerinchildrenduetolow Starvation BMI <16BMI kg/m increase risk.Theseareabsentinthecaseabove. abnormalities abnormalitiesandelectrolyte pre-existing BMI, route. Underlyingmalnutrition,low complications inmalnourishedpatientsundergoingfeedingbytheoral, enteralorparenteral shiftsandtheir associated fluidandelectrolyte Refeeding syndromeoccursduetosevere (Na Anion gapisameasureofanionsotherthanchlorideandbicarbonatecalculatedas: is seenindiarrhoea. can giveacluetothecauseofloss.Hypochloraemiaiscommoninvomitingandhyperchloraemia ofNaisreducedto<1percent.Theplasmachloride reabsorption ofNa,andfractionalexcretion osmolality. whichincreasesurinary creatinine. ThisalsotriggersreleaseofADH, Thereisrenal theureaand Water andNalossresultsinareductionintravascularvolume.Thiswillelevate thus otherketonebodiessuchasacetoacetatemayberesponsible. and mmol/L(athirdofthisisaccountedforbyglucoseandbeta-OH-butyrate) anion gapof40 muscles areabletoutilizethisasasourceofenergy. an Theassociatedmetabolicacidosisshows released duringstress.Mountingaketoneresponseisappropriateinthissituationasthebrainand andglucagon,thatare hormones,suchascortisol be duetotheeffectofcounter-regulatory maintain normoglycaemiaduetoeffectivegluconeogenesis.Theslightlyhighglucosecouldalso Cachexia and underlying malnutritioncontribute to highrisk. Cachexia areonchemotherapeutic agents,insulinordiuretics. 3–6 monthsorifthey The majoranionsarechloride(100 mmol/L)andHCO ϩ K) Ϫ (Cl 2 , unintentionalweightlossgreaterthan15 percentinrecentmonths andno ϩ HCO 2 orunintentionalweightlossof10 percentoftheirbodyweightinthelast 3 ). ThemajorcationsinthebodyareNa(140 mmol/L)andK(4mmol/L). y ndrome 3 (27mmol/L).Proteinsandphosphate 133

17: NUTRITION AND FLUID THERAPY PERIOPERATIVE CARE ➜ ➜ 4C 3D 2A 1B 4B 3C 2A 1D 5A 134

3. Metabolicpresentations in 2. Fluidandelectrol can besupplementedduringfeeding. imbalancebeforefeeding.Electrolytes tocorrecttheelectrolyte it isnotalwaysnecessary However, of potassium,magnesiumandphosphatepriortofeedingputpatientsathighrisk. levels Low hyponatraemia. tumourcancausechiasmalcompression,leadingtovisual disturbancesandassociated A pituitary down. axisslows In thechronicphaseofcriticalillness,hypothalamic-pituitary-target paralysis. Very phosphatecancausemuscleweakness,includingrespiratory low Itmovespotassiumand phosphateintothecells. Insulin causestranscellularshiftsinelectrolytes. disorder. defect,aminoaciddisorderorafatty of aninbornerrormetabolismsuchasaureacycle stress(circumcision).Onemustconsideradifferentialdiagnosis hyperammonaemia following inthisinfantitisassociatedwith Hypoglycaemia isacommonerfeatureinneonates,however ofNa excretion productionbythebronchialtumour,resultinginexcessive couldbeduetoectopicADH SIADH sodiumisanappropriateresponseinthiscase(<10urinary mmol/L). fluidreplacementisthecommonestcauseofpostoperativehyponatraemia.Alow Overzealous oftheplasma,displacingwater. occupy alargepart they causesspurioushyponatraemialiketheabnormalproteins.Thisisbecause Hypertriglyceridaemia serum osmolalitywill,infact,benormalthesecases. Abnormal proteinsinmelanomaorparaproteinaemiacancausespurioushyponatraemia.The In starvation: move intracellarly ↑ In refeeding balance ↑ Availability ofglucose Glucogenolysis + : via the kidneys despitethepresenceofhyponatraemia. viathekidneys ⇒ ⇒

clinicalsymptomsofrefeedingsyndrome ↑ gluconeogenisis ⇒ y insulinrelease te abnormalities ⇒ y oung andoldpatients proteincatabolism ⇒ stimulationofATPase pump ⇒ negativenitrogen ⇒ K + andPO 4

18 Basic surgical skills and anastomosis

Multiple choice questions ➜ Tissue apposition ➜ Needles 1. Which of the following statements are 3. Which of the following statements are true? true? A Skin wound edges should not be closed A Cutting needles are used for skin and tightly. aponeurosis. B Glues have a place in the approximation B Hand needles are ideal for skin closure. of tissues. C Round-bodied needles are used for C Bowel anastomosis is always done in closure of laparotomy wounds. single layer. D Needles with a loop-suture should be D Vascular anastomosis is always done with used for laparotomy closure. non-absorbable sutures. E In arterial suturing, double-ended needle E All arteriotomies must be closed with a sutures are used. vein patch. ➜ Anastomosis ➜ Sutures 4. Which of the following statements are 2. Which of the following statements are false? false? A Large-bowel anastomosis must be done A Polymeric synthetic suture materials only by one-layer technique. cause minimal inflammatory reaction. B The bowel ends being anastomosed B Monofilament non-absorbable sutures must be well mobilised so as not to are easy to use in tying secure knots. create tension in the anastomosis. C The integrity of polypropylene sutures C Synthetic polymers are to be used for in holding tissues together can last intestinal anastomosis. indefinitely. D In vascular anastomosis the needle must D Braided suture material can be a nidus pass from within outwards. for infection. E Polypropylene-like sutures with indefinite E Absorption is more predictable and integrity must be used for vascular complete with absorbable sutures. anastomosis.

Answers: Multiple choice questions ➜ Tissue apposition 1. A, B, D Wounds should be closed with no tension, with a minimal gap between the edges to allow for the swelling that inevitably occurs as a result of the inflammatory process of normal healing. Tissue glues, the cyanoacrylates, can be used for skin closure. Their use requires perfect haemostasis and they are ideally used in children for a laceration on the forehead. They are relatively expensive. Fibrin tissue glues work on the conversion of fibrinogen by to fibrin. They are extensively used: for haemostasis in the liver and spleen, for dural tears, in ear, nose and throat

135 PERIOPERATIVE CARE ➜ ➜ ➜ 4. A 3.A,C,D,E 2. B 136

Anastomosis Needles Sutures isalwaysclosedwithasyntheticpatch. thearteriotomy the lumen;incarotidendarterectomy, of narrowing longitudinal incisionisused,itpreferabletoclosewithaveinpatchprevent ifa if atransverseincisionismadeitcanbeclosedinthedirectionofincision.However, the calibreofvesselandclosurehastobeprecisewatertight. material needstobepermanent.Itshouldmonofilament.Thesizeofthesuturedependsupon on theoutside. theseromuscular,secondlayerofsutures continuous all-layersutureissupplementedbyinverting Whendoneintwolayers,theinner This causestheleasttissuenecrosisorluminalnarrowing. anastomosis. individual surgeon.Onemethodisthesingle-layerinterruptedextramucosal method ofstoppingbleedingendoscopically. In uppergastrointestinalhaemorrhage,fibringluesincombinationwithcollagenareaneffective postoperativeadhesionsincardiacandgeneralsurgery. andtoprevent and ophthalmicsurgery indefinite integritymustalwaysbeused. an intimalflapandtofixanyatheroscleroticplaque.Polypropylene-like sutureswhichhavean sutures. out. Thesuturematerialis2/0–3/0syntheticpolymerusinginverting interrupted anastomosisiscarried seromuscular suture.Whendoneinasinglelayer,extramucosal two layers,theinnerlayerisanall-layercontinuoussutureandouter isaninterrupted anastomosiscanbedonebyeitherone-ortwo-layertechnique.Whenin Large-bowel technique. Moreover,handneedlesincreasethedangerofneedlestickinjuries. shouldbecarriedoutusingtheno-touch Hand needlesshouldnotbeusedbecausesurgery anastomosis. areusedinvascular polypropylene suturesinholdingtissuestogethercanlastindefinitely,they theintegrityof resultinginasutureknotsinus.As the materialcancausebacterialcolonisation, requiremoreskillintyingknots.Thebraidedformof Intheirmonofilamentform,they strength. reactionandareofpredictable Polymeric syntheticsuturematerialscauseminimalinflammatory Bowel anastomosiscanbedoneinoneortwolayersdependinguponthechoiceof Bowel All arteriotomies neednotbeclosedwithaveinpatch.Whiledoingfemoralembolectomy, All arteriotomies All vascularanastomosismustbedoneusingnon-absorbablesuturesastheintegrityof In vascularsuturing,theneedlemustalwayspassfromwithinoutwards toavoidcreating 19 Laparoscopic and robotic surgery

Multiple choice questions ➜ Minimal access surgery D Gas embolism 1. Which of the following are advantages E Reduced cardiac return. of minimal access surgery? 6. Which of the following statements A Decrease in wound size regarding pneumoperitoneum are true? B Decreased postoperative pain A A Tuohy needle can be used to achieve C Shorter operating time access to the peritoneal cavity. D Improved vision B The usual intraperitoneal pressure E Reduced operating theatre costs. maintained during surgery is 25 mmHg. 2. Which of the following are limitations C The temperature of the gas used is of minimal access surgery? maintained at normal body temperature. A Technically more demanding D Nitrous oxide is preferable to carbon B Loss of tactile feedback dioxide in patients with cardiac disease. C Extraction of large specimens E The gases used for pneumoperitoneum D Poor vision have high water content. E Difficulty with haemostasis. 7. Which of the following are ➜ Pneumoperitoneum complications associated with creating pneumoperitoneum? 3. What are the desired characteristics A Bleeding of the gas used to provide B Bowel injuries pneumoperitoneum in laparoscopic C Gas dissection within the abdominal wall surgery? D Puncture of blood vessels A It should be a supporter of combustion. E Omental tears. B It should be highly soluble in blood. C It should be rapidly excreted from the ➜ Laparoscopic surgery body. 8. Which of the following are energy There should be a low risk of embolism. D sources used in laparoscopic surgery? E It should have a low diffusion coefficient. A Monopolar diathermy 4. Which of the following are gases used B Bipolar diathermy/combinations (Ligasure) to provide pneumoperitoneum? C Laser A Methane D Ultrasonic energy (harmonic scalpel) B Carbon dioxide E Vapour pulse coagulation (VPC). Helium C 9. What parameters are taken into Nitrous oxide D account while using laparoscopic Argon. E simulators to assess laparoscopic skills 5. Which of the following are competencies? complications associated with A Time taken to do the task pneumoperitoneum? B Age A Hyperthermia C Left/right side dominance B Acidosis D Number of errors C Cardiac arrhythmias E Depth (3D) perception.

137 PERIOPERATIVE CARE ➜ ➜ 14. How can bleedingfrom atrocharsite can How 14. are Whichofthefollowing 10. 3 What arethecurrentdrawbacks of 13. Whatarethebenefitsofrobotic 12. statements Which ofthefollowing 11. 138

Trochar sitebleeding Robotic surger A E D C B A E D C B A E D C B A E D C B A pressure withthetrocharitself By applyingupwardsandlateral Increased operatingtime. Socioeconomic implications Haemostasis Prolonged learningcurve Increased costs robotic surgery? learningcurve. Shorter operatingtime Shorter theatre not physicallypresentintheoperating surgeons Guidance fromexperienced Reduces theneedforassistants Better ergonomicoperatingpositions overlaparoscopicsurgery? surgery learningcurve. It hasashort benefit. particularly will Specialities thatusemicrosurgery telebiotics andtelementoringsystems. isacombinationof SOCRATES manipulators. andtheDaVinciaretele-robotic ZEUS isaroboticcamerasystem. AESOP aretrue? regarding roboticsurgery Reduced costs. Ergonomic environment degreesoffreedom wrist’ withseven Improved manoeuvringdueto‘robotic hand tremor Greater precisionduetoeliminationof magnification) andstereoscopicviews (higher Better visualisation be controlled? surgeon? tothe advantages ofroboticsurgery y ➜ ➜ 15. Which of the following statements Whichofthefollowing 15. 17. Which of the following can beused can Whichofthefollowing 17. 16. What are the main causes of Whatarethemaincauses 16.

Natural orificetransluminal laparoscopic surger Electrosurger surger E D C B A E D C B A E D C B E D C B A surgery? inlaparoscopic of electrosurgery are trueinrelationtotherisks transluminal surgery (NOTES)? transluminal surgery pointin natural orifice as theentry Diathermy. balloon catheter Applying pressurefromaFoley By suturing suture loop By usingapercutaneousmonofilament Retroperitoneum. Anus Umbilicus Vagina Mouth coagulated tissue. fromrecently tobowel Current passage Direct sparking Insulation breaks Direct coupling tissue duringcurrentapplication touchingorgraspingof Inadvertent surgery? injuriesinlaparoscopic electrosurgical Bipolar diathermyisequallydangerous. The injuriesareusuallyminor. recognised atthetimeofoccurrence. The electricalinjuriesareusually per 1000 operations. The incidenceisabout1to2cases of monopolardiathermy. theuse The majorityoccurfollowing y y in y ➜ ➜ Extended matchingquestions Extended Choose and match the correct intervention witheachofthe scenariosgiven below: Choose and matchthecorrectintervention Chooseandmatchthecorrectdiagnosiswitheachofscenariosgivenbelow:

1 F E D C B A 2. Laparoscopic/robotic nomenclatures 7 6 5 4 3 2 1 G F E D C B A 1. Laparoscop for thebestimage possible. imagequality.This helpstoprovide excellent ofbrightness andadjusts Itdetectsdifferentlevels Verres VATS NOTES CCD Hasson Master-slave unit repeated cleaning. abdomen wasdistendedandtheendoftelescopecoveredwith blooddespite hada‘red-out’. The could notvisualiseanystructuresintheperitonealcavityasview pelvic pain.Thewomancollapsedsoonaftertheintroductionoftrochar. Thelaparoscopist A 34-year-oldslimfemalewasscheduledtohaveadiagnosticlaparoscopy forherunexplained angiogram confirmsthediagnosis. pulmonary ofbreath.ACT with chestpainandshortness aweekagoisreadmitted A 70-year-old obesefemalewhohadalaparoscopiccholecystectomy the area.Itisreducibleandhasacoughimpulse. a2cmsoftlumpover reveals Clinicalexamination coughing. Itiscausinghimlocaldiscomfort. increasinglumpoverthesubsternalscar.a slowly Thisgetsmoreprominentonsittingand 2yearsagopresentswith alaparoscopiccholecystectomy A 55-year-oldmalewhounderwent non-tender. complains ofpainoverherrightshoulder. stableandherabdomenissoft Sheisotherwise day theprevious alaparoscopiccholecystectomy femalewhounderwent A 40-year-old okay.woman istachycardicbutotherwise Theabdomenissoftandnotdistended. thatthedressingsaregettingsoakedwithblooddespiterepeatedchanging.The observe diagnosticlaparoscopy4hpreviously.A 32-year-oldfemalehadanuneventful Thenurses signsofperitonitis. abdomen shows unwellandthe sheisvery with increasingabdominalpainanddistension.Onexamination 2daysagoisbroughttoA&E A 36-year-oldfemalewhohadalaparoscopiccholecystectomy no coughimpulseanditisnotreducible. scars.Thisispainlessandhard. Thereis lump onhisabdominalwalloveroneoftheport-site malewhohashadalaparoscopicresectionofhiscoloniccancerpresentswith A 64-year-old recurrence Port-site hernia Port-site Referred pain embolism(PE) Deep veinthrombosis(DVT)/pulmonary bleeding Port-site Major vesselinjury perforation Bowel y complications 139

19: LAPAROSCOPIC AND ROBOTIC SURGERY PERIOPERATIVE CARE ➜ ➜ 7. A,B,C,D,E 6.A,D 5.B,C,D,E 4.B,C,D,E 3.B,C,D 2.A,B,C,E 1. A,B,D Answers: Multiplechoicequestions 140

Pneumoperitoneum Minimal accesssurger 6 5 4 3 2 hence utmostcare needstobeemployed.Riskscan be reducedbyavoidingablindpuncture and The procedureforcreatingapneumoperitoneum canbeassociatedwithpotentialmajorrisks,and hypothermia. The temperatureofthegasusedto providepneumoperitoneumis21ºC andhencecancause syndrome. Increased pressuresriskaffectingtissue microcirculationsimilartocompartment intraperitoneal pressureemployedisbetween12 and14 15 mmHgandrarelyexceeds mmHg. Hasson trocharorasimilarblunt-tipisemployedintheopentechnique. Theusual wall areincisedunderdirectvision.Thisavoidsthemorbidityrelatedto ablindpuncture.A method employedbygeneralsurgeonsisthe‘open method’, wherethelayersofabdominal A Verres needleisusedintheclosedmethodofcreatingpneumoperitoneum. Themostcommon The othercomplicationsincludehypothermiaandreferredshoulder-tip pain. The mostcommongasusediscarbondioxide.Methanecannotbeused, asitiscombustible. watercontent. shouldhavelow coefficient tominimiserisksofembolism.They combustion, asthiswillcausefirewiththeuseofdiathermy. Itshouldhaveahighdiffusion of The gasusedtoprovidepneumoperitoneumshouldnotbecombustibleorasupporter ofhospitalstay.higher, althoughsomeofthisisrecoveredbyreducedlength techniques.Theset-upcostsandsomeoperatingcanalsobe feedback andrelianceonnew The otherlimitationsarerelianceonremotevisionandoperating,dependencehand-eye popularity toreducepostoperativehospitalstay. whichisgaining Programme(ERP), componentsofanEnhancedRecovery one oftheimportant to lesstissuetrauma.Italsoaidsearlypostoperativemobilityanddecreasedheatloss.ishence entrapment.Itreducespostoperativeadhesions due dehiscence, bleeding,herniationandnerve The otheradvantagesarereductioninwoundpainandwound-relatedproblems,suchas via anaturalorificesuchasthemouth,rectumorvagina. wherebytheperitonealcavityisentered This maybethefutureofminimallyinvasivesurgery, abdominal operations. This trocharisusefulwhencreatingapneumoperitoneuminpatientswhohavehadprevious arms controlledbyasurgeonviaremoteconsoleandbinocularviewer. suchastheDaVinciroboticsystemwithpreset ofroboticsurgery These arethetwoparts tobedoneundervideoassistance. This enablesthoracicsurgery This needleisusedtocreateapneumoperitoneumbytheclosedmethod. y ➜ ➜ ➜ ➜ 15. A,B 14. A,B, C,D,E 13. A,B, C,D,E 12. A,B, C 11. A,B,C,D 10. A,B,C,D 9.A,C,D,E 8.A,B,C,D,E

Electrosurger Trochar sitebleeding Robotic surger Laparoscopic surger to repairthedamage. helpsought,ifneeded, be promptlyrecognised.Theabdomenisopenedwithoutdelayandexpert tomajorvesselsislife-threateningandshould generated andthevolumesofgasinsufflated.Injury correlationbetweenpressures insufflationandcloselyobserving peritoneal cavitybeforestarting within theabdominalwallcanbeavoidedbyconfirming,withoutdoubt,placementin dissection operations.Gas careindifficultcases,suchasthosewhohavehadprevious taking extra patients may present 3–7 days after injury with complaints of fever andabdominalpain.Bipolar withcomplaints offever patients maypresent3–7daysafter injury areoftennotrecognisedatthetimeofoperationand The injuriescanbepotentiallyserious. They Controlofthebleeding should alwaysbeensured. insertion. isinjured.Thisbestavoidedbytaking careduringtrochar or ifoneoftheepigastricarteries Considerable bleedingcanoccurifthefalciformligamentisimpaledwith thesubsternaltrochar widely usedinfuture. tobemore stillhaspotentialadvantagesand isexpected Despite drawbacks,roboticsurgery limited bycostandavailability. willbemorewidelyusedinfuturebuttheir useiscurrently thatroboticsurgery It isbelieved long. notsurprisingly,is surgeons notphysicallypresentintheoperatingtheatre.Thelearningcurve, guidancetobeprovidedby alsoenableexperienced better ergonomicoperatingpositions.They These systemsofferadvantagestothesurgeonsbyreducingneedforassistantsandproviding frombeingexpensive. still notwidelyavailableapart Thesesystemsare,however, tele-manipulator systems,thuscreatingahuman-machineinterface. techniques. Thishasbeenprimarilyemployedintheformofautomatedcamerasystemsand intelligence apredefinedprogrammeorsetofgeneralguidelinesusingartificial supervision, automatedphysicaltasksaccordingtodirecthuman thatperforms A robotisamechanicaldevice during theactivities. The otherparametersincludecompletingthetasksuccessfullyandpathstakenbyinstruments become themainstayinfuture. sources,arebeingmorewidelyusedandlikelyto bipolar diathermyandultrasonicenergy methods,suchas butpresentlymoreexpensive to beusedwiththeutmostcaution.Itissafer The monopolardiathermyisthemostcommonsource.Thishaspotentialrisksandhenceneeds y inlaparoscopic surger y y y 141

19: LAPAROSCOPIC AND ROBOTIC SURGERY PERIOPERATIVE CARE 16. A,B,C,D,E ➜ ➜ 7B 6D 5F 4E 3C 2A 1G matchingquestions Answers: Extended 17. A,B,D 142

1. Laparoscop Natural orificetransluminalsurger sources. energy safer currentapplication,meticulousattentiontoinsulationandusingalternative avoiding excessive visualimage, measuresincludeattainmentofaperfect safety minimise injuries.Theimportant Electrosurgical injuriescanbecausedbyvariousmechanisms.Awareness ofthiswillhelpto areas. thanmonopolardiathermyandispreferred,especiallyinanatomicallycrowded diathermy issafer utmost care should hence be exercised duringtheir use. utmost careshould hencebeexercised canbecausedbyboththeVerresup oftheabdomen. Theinjury needleandthetrochar. The This isamajorcomplicationand life-threatening. Itrequiresimmediaterecognitionandopening appropriate DVTprophylaxis. arestillpotentialrisks afterlaparoscopicsurgery. Henceallpatientsshouldreceive DVT andPE riskofhernia formation. muscle splitting,andnotcutting,maybeassociatedwithalower madeusingbladeless trochars,whichare incisions of1cmormorearesutured.Lateralports thatallport tooccurin1–2percentofpatients. Itishenceimportant herniaisknown Port-site the diaphragmandshouldsubsidewithinacoupleofdays. Referred paintotheshouldersisnotuncommonafterlaparoscopicsurgery. Thisisreferredfrom significant bleeding. orthevesselsinfalciform ligamentcancause bleed. Thebleedingfromepigastricarteries wound bleedingorsignsofshockduetointernal bleedingcaneitherpresentwithovert Port-site daysaftertheoperation. presenting withsignsofperitonitisafew Thisshouldhencebesuspectedinanypatient withsubsequentnecrosisandperforation. bowel whichisfrequentlyoutofsightandnotrecognised.Grasperscanalsocrushthe diathermy injury, isapotentialcomplicationafterlaparoscopicsurgery. Thiscouldbedueto perforation Bowel wound protectorsandendobagsforspecimenretrieval. duetoincreasedawarenessandtakingprecautionssuchasusing low incidence isfortunately formalignancies.The recurrenceisapotentialproblemafterlaparoscopicsurgery Port-site sitearethemaintechnicalchallenges. closureoftheperitonealentry outasafe carry successfully carriedout.Minimisingpotentialcontaminationoftheperitoneumandabilityto andappendicectomyhavebeen cholecystectomy andtechniques. NOTES endoscopic technology carriedoutusingspecialised cavity isenteredendoscopicallyviaanaturalorificeandthesurgery This isalsocalled‘scarless’or‘incisionless’surgery. Itisatechniquewherebytheperitoneal y complications y ➜ 6D 5B 4A 3E 2F 1C

2. Laparoscopic/robotic nomenclatures Advances in optic technology includinghigh-definitionimageshavegreatlycontributedto Advances inoptictechnology the peritoneal entry sitearethemaintechnicalchallenges. the peritonealentry closureof outasafe Minimising potentialcontaminationoftheperitoneumandabilitytocarry makethissafer.The blunttipanddirectvisualisation movementsinalimitedspace,andbetterergonomicenvironment. external andlarge outcomplex degreesoffreedom,abilitytocarry seven the ‘roboticwrist’,whichallows eliminationofhandtremors,greaterprecision, improvedmanoeuvringdueto stereoscopic views, benefitsforthesurgeon,suchashighermagnificationwithbetter ‘slave’. Thisprovidesseveral withtheoperatorbeing‘master’androbot This createsahuman-machineinterface, video guidance. toenablesomethoracicoperationsbedoneunder This referstovideo-assistedthoracicsurgery to‘openThis ispresentlylesswidelyusedbygeneralsurgeons,asmostareresorting technique’. inlaparoscopicsurgery.developments 143

19: LAPAROSCOPIC AND ROBOTIC SURGERY 20 Postoperative care

Multiple choice questions ➜ The immediate ➜ Deep vein thrombosis postoperative period 4. Which of the following statements 1. Which of the following statements are are true with regard to deep vein false? thrombosis (DVT)? A Infusion and certain monitoring systems A Obese patients are more prone to DVT can cause complications. B Clinical diagnosis is very obvious B Abdominal surgical wounds may C Hip and surgery are compromise postoperative respiratory high risk function. D Confirmation is by venography and/or C The commonest cause of postoperative duplex Doppler ultrasound (US) hypotension is bleeding or insufficient E Optimum hydration is essential to fluid administration. prevent DVT. D Postoperative deep vein thrombosis ➜ Postoperative vomiting (DVT) is classically diagnosed by Homan’s sign. 5. Which of the following statements with E Oliguria is defined as urinary output of regard to postoperative vomiting are less than 0.5 mL/kg per h. false? A Inadequate analgesia can be a cause of ➜ Postoperative shortness of postoperative vomiting. breath B All abdominal operations must routinely have a nasogastric tube inserted 2. Which of the following is not a cause preoperatively. of acute shortness of breath on the C Metoclopramide and cyclazine can help. first postoperative day? D Pulmonary aspiration may inadvertently A Atelectasis occur. B Pulmonary embolism E Wound dehiscence is a distinct possibility. C Myocardial infarction D Chest infection ➜ Postoperative oliguria E Pneumothorax. 6. Which of the following statements with regard to postoperative oliguria are ➜ Postoperative hypotension false? 3. After an anterior resection, which of A The commonest cause is inadequate the following conditions are causes of fluid replacement. postoperative hypotension within the B Oliguria is defined as <1 mL urine/kg of first 12–24 h? body weight per h. A Postoperative bleeding C Patients undergoing an operation for B Myocardial infarction obstructive jaundice are particularly C Epidural anaesthesia or excessive susceptible. morphine D Renal US is carried out to look for D Inadequate fluid replacement hydronephrosis from blocked ureters. E Leakage of bowel anastomosis. E Inotropic support may be necessary.

144 ➜ ➜ Extended matchingquestions Extended Chooseandmatchthecorrect diagnosiswitheachofthescenariosbelow: Chooseandmatchthecorrectdiagnosiswitheachofscenariosbelow:

4 3 2 1 D C B A 2. Postoperativeconfusion 4 3 2 1 D C B A 1. Postoperativep He underwent a left hemicolectomy with end colostomy and a Hartmann’s closureofthe alefthemicolectomy withendcolostomyandaHartmann’s He underwent fromischaemiccolitis. laparotomyforintestinalobstruction for gangrenousbowel emergency hadan disease(COAD), man,whosuffers from chronicobstructiveairways An 80-year-old hyperdynamic circulation. of16a temperatureof39ºC,whitecell count (WCC) anda IU 000,aserumamylaseof3500 home onthesecondpostoperative daywhenhebecameconfusedanddisorientated.Hehas He wasduetogo and endoscopicpapillotomyforastoneinthecommonbileduct(CBD). anendoscopicretrograde cholangiopancreatography(ERCP) A 70-year-old patientunderwent werenormal,aswashischest X-ray.breathing wasnormal.Hiselectrolytes of . Threedaysintothepostoperativeperiod,hebecame incoherentandgarrulous.His for afracturedneck patientofchronicpancreatitisaged75hadahemiarthroplasty A known 120 mmol/L. Hisserum sodiumis has oliguria.Thereissomepittingoedemaofhisanklesandsacrum. withdifficulty. androusable drowsy somelighthaematuria andhe very Hiscatheterstillshows atransurethralresection ofhisprostate2daysago.Heisnow A 70-year-old patientunderwent Alcohol withdrawal Septic shock Pneumonia disorder Electrolyte of micturitionandburningpainduringvoiding. due togohome,10 withrigors.Hecomplainedoffrequency pyrexia dayslater,hedeveloped After anabdominoperinealresection,a70-year-old manrecoveredwell.Onthedayhewas plained ofpaininthewound.Shehadatemperatureandwoundlookedredinflamed. patient recoveredafterastormypostoperativeperiod.Onthe10th postoperativedayshecom- a70-year-old female diverticulitis, operationforperforated Hartmann’s anemergency Following right lungbase. inthe He ishypoxic.Thereoedemaoftheskininrightupperquadrantandnoairentry withrigorsandpaininhisrightshouldertip. of40ºC, 5 daysago.Heisunwellwithafever duodenalulcerlaparoscopically closureofaperforated patientunderwent A 50-year-old reduced breathsoundsonthatside. attemptatbreathing.Thechestwallmovementwasrestrictedontherightwith with every withhisalanasimoving of39ºC.Hewastachypnoeic,centrallycyanosed, apyrexia developed arighthemicolectomy.A 75-year-oldpatientunderwent Onthesecondpostoperativedayhe Wound infection Subphrenic abscess tractinfection Urinary Atelectasis y rexia 145

20: POSTOPERATIVE CARE PERIOPERATIVE CARE ➜ ➜ 1. 1. D Answers: Multiplechoicequestions Chooseandmatchthecorrectdiagnosiswitheachofscenariosbelow: 146

The immediatepostoperativeperiod 4 3 2 1 D C B A 3. Postoperativerenal failure oxygen andregular physiotherapy,shouldminimise thechancesofthiscomplication. oxygen postoperative atelectasis.Adequate relief ofpain,includingepiduralanalgesia,administration breathe ineffectivelybecauseofpain. Thisresultsinretentionofsecretionsandencourages must becloselyinspectedforfinger necrosis. line by pain,rednessandindurationaround thesiteofneedleentry. Thosewhohaveanarterial pressure. Thrombophlebitisarisingfromthenatureoffluidinfusedmay occur. Thisismanifested of airisaccidentallyintroduced,causinghypotension,tachycardiaand distended jugularvenous equipment hasnotcausedanycomplications.Airembolismmayoccur whenmorethan15 mL tomake surethatthemonitoring Monitoring equipmentcancausecomplications.Itisimportant 130/60 mmHg.Sheisacidoticwithbiochemicalfeaturesofrenal failure. blood pressureis130/80 araisedureaandcreatinine. show mmHg.Hiselectrolytes output hasbeen20mL/h.Hercentralvenouspressure(CVP)is10 cmH onthesecondpostoperativeday,herurinary withfaecalperitonitis.IntheITU diverticulitis operationforperforated Hartmann’s anemergency womanunderwent A 60-year-old outputduringthelast24hhasbeen700 mL.HisCVPis8cmH Hisurinary HDU. Onthethirdpostoperativeday,heisbackonwardfrom operation wasstraightforward. carcinoma.The aWhipple’soperationforperiampullary A 55-year-oldmalepatientunderwent renal failure. featuresofearly show outputhasdropped,heisacidoticandhisserumelectrolytes urinary Whileontheward,over2dayshis day hehasbeentransferredtothewardfromITU. was putonantibiotics–cefuroxime,metronidazoleandgentamicin.Onthefifthpostoperative managementofsofttissueinjuries.He injuriesandconservative oftheorthopaedic and fixation and bluntabdominaltrauma.Theinjurieshavebeenappropriatelytreated–openreduction renalcontusion Hesustainedfractureshaftoffemur,fracture-dislocationelbow, polytrauma. accidentresultingin A 25-year-oldmalepatientwasinvolvedinaseriousmotorcycle anuria. intothebladdershows A catheterinserted featuresofincipientrenalfailure. show the immediatepostoperativeperiod.Serumelectrolytes mLofurinein accident.Heputoutonly50 left nephrectomyfortraumafromamotorcycle a Inthepast,atageof30,heunderwent anduneventful. operation wasstraightforward arighthemicolectomyforcarcinomaofthecaecum.The manunderwent A 60-year-old Postrenal iatrogeniccause Septic shock Nephrotoxin Hepatorenal syndrome inbothlungs. reducedairentry moving, withseverely percentonthepulseoximeter. of89 saturation Hischestwallisnot breathing andanoxygen confused,withlaboured distal stump.From thesecondpostoperativeday,hehasbeenvery Abdominal incisionssplintthediaphragm postoperatively,therebycausingthepatientto 2 O and her BP is O andherBP 2 O andhis ➜ ➜ ➜ ➜ ➜ 6. B 5. B 4.A,C,D,E 3.A,B,C,D 2. B

Postoperative Oliguria Postoperative vomiting Deep veinthrombosis Postoperative h Postoperative shortnessofbreath fluid infused in 1 h. The urine output, and CVP are monitored very closely.fluid infusedin1 h. Theurineoutput,jugularvenous pressureandCVParemonitoredvery mLof inadequate fluidreplacement.Hence theinitialtreatmentshouldbeafluidchallengeof250 Oliguria isdefinedasurineoutputof <0.5mL/kgbodyweightperh.Thecommonestcauseis aspirationpneumonia. tokeepthestomachdecompressedandprevent is inserted on theoesophagusorstomach.Ifacutedilatationofstomachissuspected, anasogastrictube Abdominal operationsdonotroutinelyrequireanasogastrictubeunless theoperationhasbeen ontheslightestsuspicionarehallmarkofanearlydiagnosis. Doppler US Clinical awarenessandduplex classically arecalfpain,swelling,warmth,rednessandpyrexia. noclinical signs,which The clinicaldiagnosisinDVTisoftennotobvious.Mostpatientsshow postoperative day. occursusuallyaroundthefifth infarction mustbeborneinmind.Anastomoticleakage causes. Ifthepatientsufferedfrommyocardialischaemiapreoperatively,then inadequate fluidreplacement.Anepiduralanaestheticortoomuchmorphinecanbeother bloodpressureintheearlypostoperativeperiodisbleedingor The commonestcauseoflow preoperatively. pathology with cardiacandrespiratory infarction, chestinfectionandpneumothoraxaretheothercausesthatpronetooccurinthose physiotherapy. embolismisalatecausearoundthe10th Pulmonary postoperativeday. Myocardial andachestX-rayconfirmthediagnosis.TreatmentClinical examination isprompt,vigorous musclesofrespirationworkingtocombatthehypoxia. withtheaccessory usually cyanosed, ofbreathisatelectasis.Thepatient The commonestcauseofearlypostoperativeshortness appropriately treated. by anormalCVPandbloodpressure,variousothercausesofrenalfailureshouldbesought asseen Oncethepatientisadequatelyperfused, hypotension causingreducedrenalperfusion. ultrasound orvenogram. Doppler, 1803–1853, physics,Vienna,enunciatedtheprinciplein1842) professorofexperimental aretypicalofDVT.with pyrexia Doppler(ChristianJohann Thediagnosisisconfirmedbyduplex such acomplication.Pain,swelling,tenderness,warmthandshinyskininthecalfpatient unreliableinthediagnosisofDVT.is extremely Beingawareoftheriskfactorswillhelptoidentify epidural analgesia.MonitoringoftheCVPwouldhelpinfindingcausehypotension. ofanalgesics,particularly fluid replacement;othercausesaremyocardialinfarctionandexcess Oliguria is defined as urinary outputoflessthan0.5mL/kgperh.Thisisadirectresult Oliguria isdefinedasurinary Homan’s sign(JohnHoman,1877–1954, MedicalSchool) Harvard professorofclinicalsurgery, The mostcommoncauseofpostoperativehypotensionisbleedingorinadequate y potension 147

20: POSTOPERATIVE CARE PERIOPERATIVE CARE ➜ ➜ 2D 1A 4B 3D 2C 1A matchingquestions Answers: Extended 148

2. Postoperativeconfusion 1.Postoperative p (antabuse), anantioxidant, maybeused. Advice fromthephysicianshouldbe soughtwithregardtothebestformoftreatment.Disulfiram ofalcoholismshouldbeobtainedfromhimorhisrelatives. of whichisalcoholabuse.Ahistory tosufferfromchronic pancreatitis,thecommonestcause biochemical parameters.Heisknown Having beentreatedforhisfractured neckoffemur,thispatientisconfused.Hehasnormal thiscomplication. postoperatively mayhelptoprevent for12above theoperatingtable,usinganirrigatingresectoscopeandIVnormal saline h 20cm as somefeelthatfluidrestrictionisallneeded.Keepingtheirrigating fluidbelow the CVPline(infusionthroughaperipheralwillcausethrombophlebitis). Thisiscontroversial, mL)isinfusedthrough (250–500 saline frusemide. Incaseofseizures,3–5percenthypertonic cases,seizuresandcollapse. in extreme isconfused,restless, has visualdisturbancesand, during resection.Thepatienthashypertension, fluid (1.5 percentglycine)isabsorbedthroughthelargevenous channelsthatareopenedup overaprolonged operatingtime.Theirrigating whenlargeresectionsareundertaken particularly (transurethralresectionoftheprostate)syndrome. Thisoccurs from aconditioncalledTURP Patients undergoingtransurethralresectionofprostatemaysufferfromdilutionalhyponatraemia areavailable. culture reports andheisputonanappropriateantibiotic,whichmightneedtobechangedafterthe satisfactorily are sentforculture,anultrasoundisdonetomakesurethatthepatientemptyinghisbladder Urineandblood tractinfectionfromcatheterisation. go home.Thesymptomsaretypicalofurinary abdominoperinealresection,thecatheterhasbeenremovedandpatientisreadyto Following with thewoundleftopentogranulate.Thepatientshouldbeonanantibiotic. latesepsis.Thefeaturesarethoseofawoundinfection.Thisneedstobedrained has developed septic complications.Herethepatienthascomethroughimmediatepostoperativeperiodand withfaecalperitonitishasahighchanceofdeveloping diverticulitis An operationafterperforated drainagemaybenecessary. ofacatheter. openextraperitoneal insertion Rarely, byCT-guided scanfollowed drainageoftheabscessand Confirmation isdonebyultrasoundorCT thorough. laparoscopicclosure,peritoneallavagemightnothavebeenvery right side.Following theendoffirstweekandsignspleuraleffusionon towards skin oedema,pyrexia This patienthasthetypicalfeaturesofasubphrenicabscesswithpaininshouldertip,overlying bronchoscopicaspirationbyananaesthetistwillbenecessary. not improve,bedsideflexible vigorousphysiotherapyandantibiotics.Ifthepatientdoes done. Thepatientistreatedbyoxygen, auscultation, bronchialbreathingmaybeheard.AchestX-rayhelpful.Bloodgasesare intherightlung.On musclesofrespirationandreducedairentry useofaccessory cyanosis, This patienthastheclassicalfeaturesofatelectasis–hypoxiawithreducedchestwallmovement, The patient needs ITU monitoring.TreatmentThe patientneedsITU intheform offluidrestrictionand issupportive y rexia ➜ 3A 2B 1D 4B 3C

3. Postoperativerenal failure addition tohavingacutepancreatitiswitharaisedamylase,whichmaybedirectcomplicationof thatheisseptic,in shows andendoscopicpapillotomyhehasbeenconfused.HisWCC ERCP Following This patientwouldhavebeenjaundicedandsepticbecauseofhisstoneintheCBD. precipitated byhypovolaemia, sepsisandnephrotoxins. andcausingrenalfailure.Itmaybe for intrarenalvasodilatation),resulting inpoorrenalperfusion causes adecreaseintheproduction ofintrarenalprostaglandins(whichisnormallyresponsible of vasoconstrictors,catecholamines andangiotensin-producingintrarenalvasoconstriction.This hepatorenal syndrome.Liverfailure causessystemicvasodilatation,resultinginthesecretion and incipientliverfailurepreoperatively. Suchpatientsaresusceptibletorenalfailurefrom carcinomawould havebeensufferingfromobstructivejaundice This patientwithperiampullary microbiologist. aremeasuredtoensurethereisnodrugtoxicityandadvice issoughtfromthe gentamicin levels myoglobinuria. Treatment diuresis.Inaddition, istomaintainhydrationandencourage alkaline causing acutetubularnecrosis.Diagnosisisconfirmedbyraisedplasma creatinekinaseand releasing myoglobinfromdamagedmuscle.Myoglobinprecipitatesin the renaltubules, myoglobinuria, bothofwhichactasnephrotoxins.Inmultipletrauma, rhabdomyolysis occurs, This youngpatientissusceptibletorenalfailurefortworeasons:useof gentamicinand Textbook ofIntensiveCare P,eds. Withington Stuart or reimplantation(seeSkinnerR,WatsonD.Renalfailure.In:GoldhillDR, ofuretericrepair byaurologistwhowouldconsidersomesort patient needsurgentexploration kidney. todeterminerenalfunctionfromhissolitary an urgentIVU Oncethismishapisproven,the ureter thatisindangerduringthisoperation. would havebeentheurineproducedbeforerightureterwastiedoff.Classicallyitis mLofurinepassed structures atoperationoccurmostlyduring‘relativelyeasy’operations.The50 factthatiatrogenicdamageto proven.Itisasad failure shouldbeconsideredunlessotherwise iatrogenicdamagetotherightureterresultinginpostrenalrenal anduneventful’, ‘straightforward anuriaimmediatelyaftertheoperation.Althoughoperationwas virtual has developed ontherightside,has had arighthemicolectomy.This patient,whohasonlyonekidney He result. may distresssyndrome (ARDS) cases,acuterespiratory Inextreme support. require respiratory Prompt andaggressivetreatmentisinstitutedwiththehelpofananaesthetist.Thepatientmay aretheclinicalsigns.ThediagnosisisconfirmedbyachestX-ray.percussion andlackofairentry confusion arethehallmarksofthiscondition.Diminutionchestwallmovements,dullnesson arepronetopostoperativebronchopneumonia.Tachypnoea,such asCOAD, and cyanosis thosewithco-morbiddisease laparotomy,particularly Elderly patientsundergoingemergency appropriately treated. hepatorenalsyndrome.Hisacutepancreatitisshouldbeproperlystratifiedand to prevent (bounding pulse,raisedpulsepressure)denotessepticshock. ERCP. Itisalsopossiblethathestilljaundicedaftertheprocedure.Hishyperdynamiccirculation He needs to be treated in the ITU withclosemonitoring,antibioticsandmannitolintravenously He needstobetreatedintheITU The patient needs an urgent ultrasound of his kidney to show any hydronephrosis, followed by anyhydronephrosis,followed toshow The patientneedsanurgentultrasoundofhiskidney , ChapmanandHall,1997, Ch.52). 149

20: POSTOPERATIVE CARE PERIOPERATIVE CARE 4C 150 and useofdiuretics.Thedopamineiscontroversial. avoidance ofnephrotoxicdrugs,maintenanceoptimumhydrationcloselymonitoredbyCVP nephrotoxic. Treatment useofappropriateantibiotics, careintheITU, ismainlysupportive: This patienthassepticshock,causingrenalfailure.isoforiginasinfection anastomosis moredifficult. makesthecholedochojejunal oftheCBD, may introduceinfectionand,byreducingthegirth othersurgeonsprefernottodosobecausestentinsertion ofserumbilirubin.However, the level days,thusreducing stentpreoperativelyandleavingitinforafew aCBD jaundice byinserting mannitol. To the minimisethechancesofhepatorenalsyndrome,somesurgeonsalleviate infection withprophylacticantibioticsandmaintainingdiuresisperi-postoperative This problem is best prevented by good preoperative and intraoperative hydration, preventing bygoodpreoperativeandintraoperativehydration,preventing This problemisbestprevented PART 4 Trauma

21 Introduction to trauma 153 22 Trauma epidemiology 156 23 Head injury 160 24 Neck and spine 166 25 Trauma to the face and mouth 171 26 Trauma to the chest and abdomen 176 27 Extremity trauma 185 28 Burns 195 29 Plastic and reconstructive surgery 205 30 Disaster surgery 212 This page intentionally left blank 21 Introduction to trauma

Multiple choice questions ➜ Trauma D A temporary cavity is one that lasts 1. In trauma, which of the following momentarily and is not apparent during statements are true? clinical examination. A It is the third most common cause of E A permanent cavity gives an idea of the death overall. extent of damage. B It is the leading cause of death and ➜ disability below 40 years of age. Blunt injury C In children, one must always be alert 4. Which of the following statements to the potential of non-accidental injury regarding blunt injury are false? (NAI). A The mechanisms are direct or indirect. D The ‘imperative of time’ dictates the B In indirect injury, associated injuries may priority of treatment. be present and should be sought. E The pressure of time shapes trauma C Overt injury should lead the clinician to management. look for a covert injury as well. D Proper exposure is essential so as not to ➜ Assessment of trauma miss other injuries. E In chest injuries damage to abdominal 2. Which of the following statements are organs is rare. false? A The mechanism of injury and the ➜ Covert injury injury produced are the keystones in 5. Regarding covert injuries, which of the management. following statements are true? B It is as essential to identify overt A Adopt a deductive approach. (obvious) injuries as it is to identify the B Adopt a look-everywhere approach. covert (hidden) injury. C Adopt a focused approach. C Penetrating injuries usually involve the D Positively exclude a critical diagnosis. use of weapons. E Screen patients where clinical signs are D Blunt injuries are the outcome of obvious. acceleration/deceleration, such as falls or road traffic accidents (RTAs). ➜ Non-accidental injury E Knife injury over a limb is easy to 6. In relation to NAI in children, which of evaluate. the following statements are false? ➜ A History is inconsistent with the injury Firearm injury sustained. 3. In firearm injuries, which of the B There is a changing history. following statements are true? C There are likely to be injuries of differing A Low-velocity bullet wounds behave like ages or duration. knife injuries. D There is likely to be aggressive behaviour B High-velocity bullets cause cavitation. from the carers. C A permanent cavity is one that remains E In multiple fractures, osteogenesis after the initial impact. imperfecta may be first diagnosis.

153 TRAUMA ➜ ➜ ➜ ➜ ➜ ➜ 5.A,B,C, D 4. E 3.A,B,C,D 2. E 1. A,B,C,D,E Answers: Multiplechoicequestions statements Whichofthefollowing 7. 154

Covert injur Blunt injur Firearm injur Assessment oftrauma Trauma B A Pol posterior dislocation ofhipinadashboardinjury. istolookforaposteriorcruciateligamenttearand/or in ahead-oncollision.Anotherexample to thepancreasandduodenumin a childwithlapseatbeltinthebackseatofcarinvolved anddamage spinalinjury can bedonebythedeductiveapproach, suchaslookingforaflexion Ideally screeningofat-riskpatients should becarriedoutbeforeclinicalsignsareobvious.This ofadislocatedupperendheadradius(Monteggia fracturedislocation). injury covert theremaybea injury), ulna causedbyindirecttraumafromfallonanoutstretchedhand(overt infractureoftheshaft shouldbesought.For example damage awayfromthesiteofinjury at thesiteofimpactandeffectsonsofttissuesareinjured site.Inindirectinjury, isconcentrated chest, liverandsplenicinjuriesarecommon.Indirectblunttrauma,the injury intothechest.Therefore, inblunttraumatothelower Anatomically theabdominalcontentsextend may requireradicalwoundexcision. and ofunderlyingdamage.Thisnecessitateswidewoundexploration no ideaabouttheextent thesiteofinjury. wellbeyond Apermanentcavitygives cavity thatisnotapparentandcanextend cavity canbeproduced:apermanentthatremainsaftertheinitialimpactandtemporary crushesthetissuesinitspathwayandproducesacavity.High-velocity bulletinjury Two typesof penetration ofaneighbouringjointmaynotalwaysbeobvious. because firearms.Knifeinjuriesoveralimbarenotalwayseasytoevaluate knives orlow-velocity orinRTAs,Blunt woundsarecausedbyfalls,atsport whereaspenetratingwoundsarecausedby arebluntandpenetrating. In civilianpracticethetwocommonestgroupsofmechanismsinjury situationwhentimeisatapremium. protocolpreciselyhelpsinapolytrauma Life Support) enunciatedbytheATLS breathing,circulationanddisability) ABCD (airways, (AdvancedTrauma haematoma oranischaemiclimbcanbedealtwithbyvariousgradesofurgency. Theorderof tensionpneumothorax,anextradural anobstructedairway, and thosethatcanwait.For example, The term‘imperativeoftime’meansdeterminingwhichinjuriesneedtobedealtwithimmediately There isageneralisedimmuneresponse. A dropinbodytemperatureoccurs. patient? are truewithregardtoapolytrauma y trauma y y y E D C be transgressed. A protocol-drivensystemshouldnever The earlymanagementisprotocol-driven. mechanisms tobloodloss. The patienthascompensatory ➜ ➜ 7. A,B,C,D 6. E

Pol Non-accidental injur orthopaedic surgeons will tailor the need of that particular individualpatient. surgeonswilltailortheneedofthatparticular orthopaedic deliberationsbetweenthevascularand ofthepoplitealfossa, inabulletinjury For example, leader maynotalwaysdecidetoadhereslavishlyaprotocolinthebestinterestofpatient. outlining theproposedoverallmanagement.Whilstprotocolsdenotegoodpractice,team the patient’snotes,butagoodmethodistorecordalldetailsonwhiteboardintheatre, protects thepatientanddoctor,latterfromlitigation.Thesurgicalplanwillberecordedin foreasierandquickerdecision-making.Moreover,theiruse usually protocol-driven,whichallows vital organs;hypotensioninitselfisthereforenotaproblem.Inmostcases,earlymanagement of mechanismstobloodlossmaintainperfusion Thepatienthascompensatory be prevented. inactivity,hypovolaemiaandlossofvasomotorcontrol.Heatshould may beduetoexposure, ofthepatient’smetabolicresponsetotrauma,thereisadropinbodytemperature.This apart As andcareasateam. thoroughevaluation clinician shouldadmitthepatientforfurther, asthecauseofmultiplefractures.Thisshouldnotbeacceptedand imperfecta) The carersofanallegedlyabusedchildwouldusuallycitebrittle-bonedisease(osteogenesis echocardiography incardiactamponade. haematomaorafocusedabdominalsonographyintrauma(FAST)scan insuspectedextradural or intheATLS survey’ by theterm‘secondary protocol. injuriesandruptureddiaphragm.Thisapproachissummarised scaphoid fracture,peripheralnerve A focused exclusion approach is mandatory in life-threatening injuries, such as performing a CT aCT inlife-threateninginjuries,suchasperforming approachismandatory A focusedexclusion anklefracture, approachisessentialinanunconsciouspatienttoexclude A look-everywhere y trauma y 155

21: INTRODUCTION TO TRAUMA 22 Trauma epidemiology

Multiple choice questions ➜ Epidemiology C ABC (airway, breathing, circulation) is the 1. Which of the following statements primary survey. regarding trauma are true? D Airway management should be A Trauma is the second commonest cause accompanied by cervical spine control. of death worldwide in the population E Oxygen should be administered with aged 1–40 years. great care in polytraumatised patients B Road traffic accident injuries are the who might suffer from chronic bronchitis. commonest cause of injury-related death F The physical signs of a tension worldwide. pneumothorax are difficult to pick up in C Falls are the commonest cause of injury- the resuscitation room and so it is best related death worldwide. diagnosed by an immediate chest X-ray. D A 10 per cent increase in vehicle speed 3. A 25-year-old motorcyclist comes in leads to a 20 per cent rise in case fatality on a spine board with cervical spine risk. control. He was found at the roadside E Ejection from the vehicle at the time of unconscious following a collision with accident saves a significant number of an oncoming car. The airway is clear people from more serious injury. but his breathing is clearly laboured F Seatbelts and airbags can actually cause with a respiratory rate of 40/min. injuries. His pulse is 110/min and his blood G The Advanced Trauma Life Support pressure is 90/60. It appears that the initiative (ATLS) required that a primary wing mirror of the car has penetrated survey is performed before resuscitation his chest and there is a wound on the is started. right side which is bubbling. He is not H The secondary survey is designed responding to verbal command but to ensure that no new injuries have is muttering incoherently. He will not developed in what is an emergent open his eyes. situation. (a) List the following tasks in their order of ➜ Assessment and priority: management of the A Insert a chest drain. polytraumatised patient B Give 100 per cent oxygen. C Put a flap valve dressing over the 2. Which of the following statements bubbling wound. regarding the immediate management D Insert two wide-bore cannulae. of a polytraumatised patient are true? E Take an arterial blood gas (ABG) sample. A When the patient enters the resuscitation F Start checking for other injuries. room the first priority is to listen to the G Give intravenous (IV) fluids. history given by the rescue team. H Start a blood transfusion. B The patient should be asked a simple question such as ‘What is your name?’

156 ➜ 1. B,F Answers: Multiplechoicequestions List inordertheprioritiesofyournext (b) you survey, During thesecondary 4. A child of3yearsisbroughtintothe 4.

Epidemiolog F E D C B A stable, butthereisawoundoverthe is silentanddistending.Thepelvisfeels discover bloodinoneear. Theabdomen increase inmortality. to thesquareofvelocity,anincrease inspeedofaslittle10 percent percentleadstoa40 involvedinanaccidentisproportional ofinjury. inpredictingseverity theenergy As is important group, roadtrafficaccidentsarethe commonestcauseofdeathwithfallsrunningsecond.Energy Trauma Withinthat isthecommonestcause ofdeathworldwideinthepopulationaged1–40. refill. in thefootandthereisnocapillary befelt is dusky-coloured.Nopulsescan open hiseyestocommand.Theleftfoot has improvedtotheextentthathewill of blood.Thepatient’sconsciousstate swinging normallyandhasdrained 100 mL pressure ismaintained.Thechestdrain is improvement, asdoesathird.Theblood 110). Asecondbolusproducesafurther 90/0) andthepulsefallsto105/min (from pressure risesto100/70 mmHg(from After thefirstbolusoffluid,blood left tibiawhichisobviouslyangulated. pupils areequalandrespondingto a bruiseontheforeheadandthat reveals survey normal limits.Primary blood pressureandpulsearewithin and breathingrapidly (60/min)but first floorbalcony. Heisunconscious A&E departmenthavingfallenfroma catheter.Put inaurinary oftheleftleg. for anarteriogram Ask debridement ofthetibialwound. Take thepatienttotheatrefor head. ofthechestandabdomen Get aCT pelvis. spine,chestand Get X-raysofthecervical for alaparotomy. Take thepatienttotheatreimmediately actions: y Choosethebestoption: A 70-year-old femalefallsandsustains 5. actionsintoorderof Put thefollowing B A F E D C B A D C priority: Blood shouldbesentforcross-match. Blood gasesshouldbetaken. Intraosseous needlesshouldbeinserted. observation. ward andputonhourlyneurological The childshouldbemovedtothe however longyouhavetowait. however Insist thatyouoperatenight, ‘outis now ofhours’. Leave allplanninguntilthemorningasit onthe morninglist. surgery anaesthetist planningtooptimiseherfor and thendiscussthesituationwith Wait untilthebloodresultsareback possible thatevening. Set herupforoperationassoon injuries. alert andorientatedhasnoother and herpulseis70/min. Sheisfully her bloodpressureis160/100 mmHg diuretics andlivesalone.You find hypertensivewhoison is aknown admitted tohospitalat7.30pm. She a fractured neckoffemur. Sheis operating theatreforalaparotomy. The childshouldbetakenstraighttothe abdomen shouldbeobtained. scanofthe An ultrasoundscanorCT gain IVaccess. appear normal.Itisnotpossibleto is silent.Thepelvisstable.legs The abdomenappearsdistendedand isgoodintobothlungs. light. Airentry 157

22: TRAUMA EPIDEMIOLOGY TRAUMA 3.(b)B ➜ 3.(a)B 2.A,B,C,D 158

necessary, more than one surgical team can work on him under the same anaesthetic. morethanonesurgicalteamcanworkonhim underthesame necessary, oftheabdominalandthoracicdamagesothat,if abdomen andchesttodeterminethe fullextent ofthehead, itwouldbewisetogetaCT However, hasbeenperformed. once thearteriogram The lefttibiaisanopenfractureandsoneedscleaningintheatre damage withanarteriogram. ofthat andextent priorityisdefiningthelevel ofvasculardamage sothenext has gotsomesort catheter isputin,justincasethere is apelvicfracturewhichhastorntheurethra.Theleftleg beforetheurinary spine,chestandpelvis)theseshouldbeperformed set ofX-rays(cervical outsomeinvestigations. Heneedsthestandard blood.Thereisthereforetimetocarry oxygenated to theatreimmediately. Hisconsciousstateisalsoimproving, suggesting thatheisperfusing it doesnotappearthatheiscurrentlybleedinghard.Theretherefore noneedtotakehim The patienthasrespondedwelltoafluidbolusso,althoughhemayhave lostalotofblood, secondary survey. secondary should youproceedontothe chest drainwillbeneeded.OnlywhenABandChavebeensorted will beneededassoonpossible,andthen,oncethehypovolaemic shock hasbeenstabilised,a hispulseandbloodpressurerespondtotheinitial fluids.Bloodgases blood yetuntilweseehow as thecannulaeareinplace.Bloodshouldbesentforcross-matchbut thereisnoplaceforgiving pulse suggestthatheisalreadyinhypovolaemicshockandsoIVfluidsshouldbegivenassoon priorityisthereforecirculation.Heneedstwowide-borecannulae,buthisbloodpressureand next crisis.The priority,asyouhavedealtwiththecriticalrespiratory be neededbutthisisnotthenext Achest drainwill effort. put overthisassoonpossible,willimprovehisventilatory 100 Thepatientclearlyhasasuckingwoundsoflapvalvedressingshouldbe percentoxygen. priorityisbreathing,sothefirstactionheretogive it shouldstillbecheckedanyway. Thenext isclear,although The factthatthepatientismuttering,albeitincoherently,suggestshisairway Assessment andmanagementofthepol patient maybedeadbeforethepictureisavailable. bemadebyX-rayastheconditioncanprogresssoquicklythat but thediagnosisshouldnever The signsofatensionpneumothoraxcanbedifficulttoseeandhearintheresuscitationroom, (without fail)shouldreceive100 athighrate(15 percentoxygen L/min)viaarebreathingmask. ischecked.Alltraumapatients timeastheairway atthesame and socontrolshouldbestarted spineneedsprotectingassoonresuscitationstarts, ofthebrain.Thecervical adequate perfusion beabletobreatheandhave order tospeakcoherently,apatientmusthaveclearairway, ABCsurvey. In canrespond,thenyouhaveeffectivelycompletedtheprimary because ifthey information. Thepatientshouldthenbeaskedasimplequestionsuchas‘Whatisyourname?’ accident, thetimesinceaccidentoccurredandlikelymajorinjuries,allcrucialpiecesof involvedinthe whocangivevitalinformationontheenergy from theambulance(rescue)crew thatthehandoveristaken When thepatientfirstentersresuscitationroomitisimportant systems once the primary survey hasidentifiedimmediatelife-threateningproblems. survey systems oncetheprimary ofthebodyand isaimedatcheckingallotherparts survey the initialresuscitation.Thesecondary timeas iscarriedoutatthesame survey Theprimary any patientwhohassustainedpolytrauma. the abdomenandthorax. number),characteristicallyin alsocauseinjuries(afarfewer they of deathsandinjuries.However, largenumbers rate.Bothseatbeltsandairbagsprevent associated withamuchhighermortality The ATLS andquickestpossiblediagnosistreatmentfor isasystemforprovidingthesafest ejection,whichis percentandalsoprevent bynearly50 Seatbelts reducetheriskofinjury → → C F → → E D → → C G → → D E → → A A → F y traumatised patient 5. B 4. B therefore beplannedforfirstthinginthemorning. 10pm should fromthatinvolvinglifeandlimb.Thesurgery shouldceaseapart whenallsurgery whenitiscomplete.Itunlikelythatthework-upwillbecompletedbefore only beundertaken should immediately,butsurgery to minimisetheanaestheticrisk.Thisplanningneedsstart isundertaken, it isbestpracticetoensurethatthepatient’sconditionoptimisedbeforesurgery elderlypatientsusuallyhavemorethanonethingwrongwiththemand independence. However, canmaintain aspossiblesothatthey their painandsecondlytokeepstayinhospitalasshort Patients withafracturedneckoffemurshouldbeoperatedonassoonpossible,firstlytorelieve cardinal signs( in thisagegroup)ofhypovolaemicshock. the stageoftreatment.Thisislikelytobealaparotomyasthischildshowing planning thenext ultrasound ( FAST ofthehead)isvitalfor scanoftheabdomen(combinedwithaCT scan)oraCT Blood shouldalsobesentimmediatelyforcross-matchasitmayneededimminently. Either of accesstogivefluids.ThiswillbeviaintraosseousneedleifIVcannotobtained. hypovolaemicshock.Thefirstpriorityisthereforesomeform severe that thischildisdeveloping 30percent).Intheabsenceofanyabnormalfindingsinchest,itmustbeassumed exceeds latechangesinchildren(whenbloodloss rate asfallinbloodpressureandrisepulsearevery be. Inyoungchildrentheonlysignofhypovolaemicshockmaybeanincreaseinrespiration rateishigherthanitshould ratethanadultsbutthisrespiratory Children haveahigherrespiratory → D → E → F → C → A 159

22: TRAUMA EPIDEMIOLOGY 23 Head injury

Multiple choice questions ➜ Cerebral physiology ➜ Head injury 1. What is normal cerebral blood flow? 5. Which of the following statements A 10mL /100 g per min regarding Glasgow Coma Scale (GCS) B 25 mL /100 g per min are true? C 55 mL /100 g per min A The maximum score is 15. D 100 mL /100 g per min B The minimum score is 0. E 5.5 mL /100 g per min. C A GCS of 10 means the patient is in coma. D Eye opening to command/speech is 2. Cerebral autoregulation maintains scored as 2. normal cerebral blood flow in E A GCS of 6 means the patient is in coma. what range of mean arterial blood pressures? 6. When does primary brain injury occur? A 50–150 mmHg A At the moment of impact B 50–100mmHg B In the first hour after injury C 25–50 mmHg C In the first 4 h after injury D 80–200 mmHg D Only if the patient is in coma E 100–250 mmHg. E In the first 24 h after injury. 3. How is cerebral perfusion pressure 7. Which of the following can cause defined? secondary brain injury? A Systolic arterial blood pressure minus A Hypoxia diastolic blood pressure B Hypotension B Systolic arterial blood pressure minus C Raised ICP venous pressure D Reduced cerebral perfusion C Mean arterial blood pressure minus E Pyrexia. venous pressure 8. What is the incidence of traumatic D Mean arterial blood pressure minus brain injury in the UK? intracranial pressure (ICP) A 450 cases per 100 000 population E Venous pressure minus ICP. B 100 cases per 100 000 population 4. Which of the following compensatory C 20–40 cases per 100 000 population mechanisms maintain normal ICP in D 20–40 cases per 1 000 000 population the face of an increasing intracranial E 5 cases per 100 000 population. mass lesion in adults? A Reduction in venous blood volume ➜ Clinical management of B Increase in skull volume head injury C Sunsetting eyes 9. Which of the following clinical signs D Reduction in cerebrospinal fluid (CSF) are evidence of a base of skull volume fracture? E Increase in CSF production. A CSF rhinorrhoea B CSF otorrhoea

160 ➜ Extended matching questions Extended Choose and match the conditions above with the clinical scenariosdescribedbelow: Chooseandmatchtheconditions abovewiththeclinical 12. Patients with severe head injury Patients withsevereheadinjury 12. statements Whichofthefollowing 11. statements Whichofthefollowing 10.

3 2 1 C B A 1. Headinjur A E D C B A E D C B A E D C his friends. When the ambulance arrived he was GCS 15.his friends.When the ambulancearrivedhewasGCS InA&Ehewasnotedtohave aright A 25-year-oldmanfell15 feet fromatree.Therewasbrieflossofconsciousnesswitnessedby aBattle’ssign. shows blood andclearfluidleakingfromhis rightear. Examination head onthekerb.Heisbrieflyknocked outbutrecoversquickly. 15. InA&EheisGCS Hehas A 30-year-oldmanisinvolvedina fight. Heispunchedandfallstotheground,strikinghis to A&E. 7withafixeddilatedrightpupil.Heisintubatedfortransfer himtobeGCS shows Examination A 45-year-oldmanisinvolvedinaroadtrafficaccident.Hedeeplyunconscious atthescene. Petrous temporalskullfracture Subdural haematoma haematoma Extradural secretion ofantidiuretichormone(ADH) The syndromeofinappropriate arecauses? following level (hyponatraemia). Whichofthe bloodsodium often developalow scans. shapeonCT It hasabiconvex It hasagoodprognosis. blood vessel. It canbeduetodisruptionofacortical brain. It canbecausedbylacerationofthe dura andarachnoidmembranes. It isacollectionofbloodbetweenthe are true? regarding acutesubdural haematoma Prognosis ispoor. Treatment iswithaburrhole. There isoftenalucidinterval. It commonlyoccursatthepterion. artery. with lacerationofthemiddlemeningeal It isoftencausedbyaskullfracture true? regarding extradural haematomaare Haemotympanum. Periorbital bruising Battle’s sign y 14. Which of the following statements Whichofthefollowing 14. Inwhatpercentageofpatientswith 13. E D C B A E D C B A E D C B Never, unless they hadanearlyseizure. unlessthey Never, percent 90 percent 50 5 percent 20 percent occur? week afterheadinjury) dolateseizures(i.e.>1 head injury productionofcortisol. Excess Uncontrolled diabetesmellitus Secretion ofatrialnatriureticfactor Uncontrolled diabetesinsipidus It is never bilateral. It isnever the haematoma. It usuallyoccursontheoppositesideto byaherniateduncus. nerve It iscausedbypressureonthethird upwards. todeviate It maycausetheeye It causesafixed,dilatedpupil. aretrue? injury palsyinhead regarding thirdnerve 161

23: HEAD INJURY TRAUMA 4.A,D 3. D ➜ 2. A 1. C Answers: Multiplechoicequestions 162

why it is so important to take seriously small deteriorations in function or level ofconsciousness. totakeseriouslysmalldeteriorations infunctionorlevel why itissoimportant asapressure–volumecurve. shown rise. Atthispoint,smallincreasesin volumewillcauselargeincreasesinpressure.Thiscanbe to willstart mechanismsandICP pointthemasslesionwill overcome thecompensatory certain volumeismaintaineduntilthelatestages.Ata increases, itmaycompressthebrain. Arterial themasslesion volumeandtherefore pressurewillremainconstant.As blood volumeandCSF contains brain,bloodandCSF. masslesion willinitiallycausereductioninvenous Anexpanding It This isthedoctrineofMonro–Kellie.Itstatesthatheadaclosed box thatcannotexpand. volume. mass lesioninadultsincludereductionvenousbloodvolumeand inCSF inthefaceofanincreasing intracranial mechanismsthatmaintainnormalICP Compensatory cerebral venouspressurebutiseasiertomeasure. minusICP. BP pressureisdefinedasmeanarterial cerebral perfusion as isusuallythesame ICP pressureminusvenousbut pressuregenerallyequatestomeanarterial Perfusion ischaemia soitshouldbeusedwithcaution. therefore reducesICP. thecerebralvasoconstrictionthuscausedcanresultinbrain Unfortunately, vasoconstriction (ifautoregulationisstillpresent)andreducescerebralbloodvolume increases andfallsasthebloodpressurefalls. increasesasthebloodpressure maybe‘pressure-passive’,i.e.cerebralbloodflow blood flow • • • Cerebral ph mechanisms areinvolved: bloodpressure.Threemain ismaintainedinthefaceofvariationsarterial blood flow and150pressures of50 mmHg.Cerebralautoregulationisthemechanismwherebycerebral blood betweenmeanarterial Cerebral autoregulationmaintainsnormalcerebralbloodflow normal. blood flow. 20percentof fallsbelow Brainfunctionwillquicklyceaseifcerebralbloodflow is55mL/100normal cerebralbloodflow gpermin,equatingtoapproximately770 mL/minof isthereforetightlyregulated.A todelivertheseessentialsubstrates.Cerebralbloodflow flow andisdependentoncerebralblood The braindoesnothavelargestoresofglucoseoroxygen Increased concentrationsofthesemetabolitescausevasodilatationandincreasedbloodflow. Concentrations ofmetabolitessuchasK The concentrationofO increases flow. and alterthediameterofbloodvesselaccordingly. anddilatation Constrictionreducesflow inthebrainrespondtostretchvesselwall Smooth muscleinthewallsofarterioles arterioles, e.g.increasedCO arterioles, This iswhyneurosurgical patientscanrapidlydeteriorate havingbeenapparentlystable and Reducing CO braininjury.Autoregulation isoftenabnormalorlostcompletelyinsevere Inthiscase,cerebral was noted to be GCS 8withafixeddilatedrightpupil. was notedtobeGCS scoredroppedto13.temporal scalplaceration.OnehourlaterhisGCS minuteslaterhe Thirty 2 levels (hyperventilation) canbeusedtherapeutically. (hyperventilation) levels Itcausescerebral y siolog 2 andCO y 2 causesdilatationandincreasedbloodflow. 2 (butprincipallyCO + andH + causevariationsinthebloodvesseldiameter. 2 ) causesalterationinthediameterof ➜ ➜ 11. A,B,C 10. A,B,C 9.A,B,C,D,E 8. C 7. A,B,C,D,E 6. A 5.A,E

Clinical managementofheadinjur Head injur The GCS is a three-point scale used to describe the level ofconsciousnessinpatientswithhead isathree-pointscaleusedtodescribethelevel The GCS usually associated withunderlyingbraininjury. Thecauseofthehaematoma is oftenlaceration haematoma because itis Acute subduralhaematoma hasapoorerprognosisthan anextradural throughasmallburrhole. controlled andthebloodclotisoften solidandcannotbeevacuated choice iscraniotomy. isnot recommendedasthebleedingvesselcannotbe Burrholeevacuation promptly.can beevacuated theinjury. deteriorateinthehoursfollowing then slowly Theprognosis isgoodifthehaematoma but andthesepatientsclassically havea‘lucidinterval’ There isoftennounderlyingbraininjury dura. Theduraisstrippedofftheinternalaspectofskullashaematoma increasesinsize. whichthenbleedsintothepotentialspace betweentheskulland middle meningealartery, runs.Theedges ofthefracturerupture pterion, asthisiswherethemiddlemeningealartery haematomasareusuallyassociatedwithaskullfracture,often intheregionof Extradural ifmeningitisoccurs. resistant organismsintheCSF toincreasetheincidence ofmultiply havebeenshown reduce theriskofmeningitis,butthey anterior skullbase. subconjunctival haemorrhageswithnoobviousposteriormargin,isoftenseeninfracturesofthe whichmaybeassociatedwith asracooneyes), temporal bone.Periorbital bruising(oftenknown back ofthethroatandfromtheremaybeseenasrhinorrhoea. theEustachiantubeinto maytrackdown eardrum. Ifthetympanicmembraneisintact,CSF maybeseenbehindthebulging ruptured. Ifthetympanicmembraneisintact,bloodandCSF otorrhoeaifthetympanicmembraneis orCSF to lacerationorcompressionofthefacialnerve, motorneuronfacialweaknessdue ipsilaterallower ear duetodamagethecochleaoritsnerve, fractures inthefrontalorethmoidsinuses.Thiscanpredisposetomeningitisbrainabscess. throughthenosevia ofCSF maycauseleakage A baseofskullfractureintheanteriorcranialfossa inyoungmen. disability isasignificantcause ofpermanent population. Manyofthesecasesareinyoungmen.Headinjury casesper100 iswelldocumentedat20–40 intheUK The incidenceoftraumaticbraininjury 000 masslesions,seizuresandincreasedtemperature. expanding hypoxia, hypotension,raisedICP, andisoften causedby injury canoccuratanytimeaftertheprimary braininjury Secondary thathappensatthemomentofimpactorinjury. describestheinjury braininjury Primary opening)orlessisconsideredtobeincoma. eye 8(withno ApatientofGCS motorresponsesandvocalisation. assessed onabilitytoopeneyes, injury. Thehighestscoreis15 scoreis3(deepcoma).Thepatient (normal)andthelowest Base ofskullfracturesaffectingthepetroustemporalbonecancausedeafnessinipsilateral CT scan shows a biconvex haematoma whichdoesnotcrosssuturelines.Thetreatmentof abiconvex scanshows CT to leaksastheirusehasnot beenshown Prophylactic antibioticsarenotindicatedforCSF Battle’s signisapatchofbruisingbehindtheear,indicatingfracturepetrous y y 163

23: HEAD INJURY TRAUMA 12. A,C 14. A,C 13. B ➜ 2C 1B matchingquestions Answers: Extended 164

1. Headinjur tolieina‘down palsycausestheeye A thirdnerve andout’position. willbeaffectedandbothpupilsbecomefixeddilated. increase, theoppositethirdnerve thepressure andshift sideasthelesion.As will thenbecomefixed.Thisisusuallyonthesame sideasthelesion.Thiscausesa dilatedpupilwhich onthesame it compressesthethirdnerve cause theuncusoftemporallobetobepushedacrossandthroughtentorialhiatuswhere haematomas Expanding leavesthemidbrainininterpeduncularfossa. The thirdcranialnerve Seizurescanusuallybecontrolledwithanticonvulsants. inform theDVLA. todriveforsometimeafterinjury. arenotallowed Patientsmustbetoldto headinjury severe andforthatreasonpatientswith Late seizuresoccurin5percentofpatientswithheadinjury butwillnotcausehyponatraemia. response toinjury ofthestress secretionwilloccuraspart but isnotacauseofhyponatraemia.Increasedcortisol injuredpatients (hypernatraemia). Diabetesmellitus(raisedbloodglucose)iscommoninseverely areunabletoconcentratethe urine.Thesodiumwillrise and dehydrationasthekidneys replacement. Treatment bycollecting24hurinesamples. canbeshown by thekidneys andwater issalt CVP. alow urine. ThepatientisdehydratedandCVPmeasurementswillshow excretion salt Excess thesodiuminto aswaterfollows andtotalbodywaterwillalsobelow sodium willbelow dilutional effectandwaterrestrictioncanbeusedtotreatthiscondition. duetoa The totalbodysodiumisnormalbutwaterincreased.Sodiumlow ontherenaltubules. ADH holdingontowaterduetheactionofexcess caused bythekidneys is todistinguishbetweenthe two.SIADH Itisimportant secretion ofatrialnatriureticfactor(ANF). orexcess secretion(SIADH) Hyponatraemia isoftencausedbysyndromeofinappropriateADH shape. have abiconvex some casesthearachnoidmembraneisalsobreached. The bloodcollectsunderthedura,usuallybetweenduraandarachnoidmembranes,althoughin sinus.Multiplebleedingpointsarenotunusual. bridgingveinsaroundthesuperiorsagittal cortical the frontalortemporallobes(‘burstlobe’).Insomecasesbloodcomesfromtorn contusions.Thisoftenaffectsthetipsof cortical vesselinassociationwithsevere of acortical prophylactic antibiotics as these have not been shown toreducetheriskof meningitis. prophylactic antibiotics asthesehavenotbeenshown behind theear. on Thepatientwill haveanincreasedriskofmeningitis butshouldnotbestarted side.Battle’ssignisanareaofbruising palsyordeafness onthesame also haveafacialnerve Base ofskullfracturesaffectingthe petrous temporalbonecancauseotorrhoea.Thepatientmay patient toavoidhypoxiaorhypercarbia. Hypotensionmustalsobetreated. management(i.e.intubationandventilation)isessentialinanunconscious andairway injury, brain toavoidsecondary The patientisusuallyunconsciousfromthetimeofinjury. Itisimportant underlying braininjury.Subdural haematomasareseriousinjuriesoftenassociatedwithsevere Diabetes insipidus is caused by lack of secretion of ADH andcausesahighurineoutput Diabetes insipidusiscausedbylackofsecretionADH andthereforetotalbody causessodiumtobelostbythekidneys, secretionofANF Excess haematomaanddoesnot scanthehaematomaislesswelldefinedthananextradural On CT y 3A middle meningeal artery or one of its branches. The extradural collectsovertime.Initiallythe oroneofitsbranches.Theextradural middle meningealartery well.Thepatientmusthavehadanunderlyingskullfracturewhichlaceratedthe can appearvery whenthepatient theinjury following haematomasoftenpresentwithalucidinterval Extradural Mannitol maybegiven.Urgentreferraltotheneurosurgeonisessential. deteriorationcanhappenquickly.further tohappen, todeteriorate.Whenthisstarts andthepatientstarts brain shiftoccurswithraisedICP buteventually mechanismsdescribedintheMonro–KelliedoctrinemaintainICP, compensatory The patient should be intubated and ventilated and an urgent CT scan should be performed. scanshouldbeperformed. The patientshouldbeintubatedandventilatedanurgentCT 165

23: HEAD INJURY 24 Neck and spine

Multiple choice questions ➜ Anatomy of spine injury (b) What is the safest and most reliable 1. Which of the following statements way to clear the cervical spine? regarding spine injury are true? A Cervical spine series of plain X-rays A Cervical spine cord injuries are common. B Flexion and extension views B Pre-hospital care has slightly reduced C Magnetic resonance imaging (MRI) their incidence. D Keep him on a spine board until he C A factor which reduces length of stay and recovers consciousness. improves outcome is referral to specialist (c) What fluids need to be given? spinal centres. A No fluids are needed at this stage. D The size of the spinal canal makes the B Two litres of saline should be given cervical spine especially susceptible to stat and then further litres until the injury. systolic pressure comes above E The stability of the cervical spine is 110 mmHg. mainly provided by the bony anatomy. C Put a central venous pressure line, and a F The cervicothoracic junction is especially urinary catheter. Fluids should be titrated susceptible to injury because it is a against preload pressure and urine transition zone from the mobile to the output. rigid segment of the spinal cord. G All three columns of the spinal column The patient wakes up and cannot move must be injured for the spine to be his legs. Examination reveals a step unstable. between T10 and T11 which is painful. H The spinal cord ends at L5/S1. (d) Which of the following tests give the I The cervical roots exit above the best idea of prognosis? vertebral body of the same name, while A Perianal sensation the thoracic and lumbar roots exit below. B Bulbocavernosus reflex J The secondary spinal injury is usually a C A loss of power proprioception on one result of the unstable spine moving during side with loss of temperature and pain rescue and treatment of the patient. sensation on the other side. ➜ Case study: injury after fall Examination of the lower limbs of the patient reveals sensation present in the 2. A patient is admitted unconscious with lower limbs but no motor power. a head injury following a 4-metre fall from the roof of a building. His blood (e) What Frankel Grade is this patient? pressure is 80/60 mmHg, pulse 45/min, A A and he has well-perfused extremities. B B (a) What is the likelihood of him having a C C significant spinal injury? D D A < 20 per cent E E. B > 20 per cent C > 50 per cent.

166 ➜ ➜ ➜ Extended matchingquestions Extended Apatientisfoundtohavean 4. statements Whichofthefollowing 3. foreach ofthescenarios Choose andmatchthecorrectfracture/dislocation/subluxation

3 2 1 Q P O N M L K J I H G F E D C B A 1. Fractures, dislocationsandsubluxations Cervical spineinjur C B A spine injur inneckand Visualisation given below: subluxation isthis? subluxation ratioofmorethanone. Whatfracture/dislocation/ A patientisfoundtohaveaPower expanded. isfracturedand A patientpresentshavingfallenontheirheadfrom aheight.TheC4vertebra expanded. A patientpresentshavingfallenon their headfromaheight.Theatlasisfracturedand Burst fracture Jefferson's Type 3odontoidpegfracture Type 2odontoidpegfracture Chance fracture Facet dislocation dislocation Posterior craniocervical Type 1odontoidpegfracture Hangman's fracture SCIWORA Type Cthoracicfracture Wedge fracture Type Bthoracicfracture Type Athoracicfracture Teardrop fracture. Atlantoaxial instability dislocation Anterior craniocervical injury. help understandthenatureofbone isbestforreconstructionswhich MRI tissue haematomainthecord. scanisbestatdemonstratingsoft- CT spinal injuries. identify almost100 percentofsignificant is visualised,plainX-raysareadequateto junction Provided thatthecervicothoracic neurological deficit(Frankelneurological C). Hehasapartial bifacet dislocation. with spineinjury, unstable cervical regarding visualisationarefalse? y y (c) The now startsto now The neurology (c) What istheoptimumtreatment? (b) (a) What investigationisneeded? (a) C B A C B A C B A Stiff collar bone graftplatesandscrews. with Open reductionandinternalfixation Traction viaahalo Stiff collar MRI. CT Tomograms bone graftplatesandscrews. with Open reductionandinternalfixation Traction viaahalo plan now? deteriorate. Whatisyourmanagement 167

24: NECK AND SPINE TRAUMA ➜ 1. C,F,I Answers: Multiplechoicequestions 168

Anatom conus medullaris(the endofthespinalcord)stopsat T12/L1. From then onthespinalroots usually stablebutnotalways. instability, butdisruptionofallthree does.Wherethereisdisruptionoftwocolumns,thespine ligaments andthefacetjointswiththeir pedicles).Disruptionofonecolumndoesnotproduce and theanteriorlongitudinalligaments) andtheposterior(thespineswiththeirinterspinous body discs),themiddle(thebackofvertebral bodyandintervertebral front ofthevertebral areamostlikelytobefracturedordislocated. to visualise)isalsothevery junction(theareamostdifficult loads,andsothecervicothoracic area mostvulnerabletoextreme relatively rigid.Thetransitionbetweenmobileandstiffsegmentsinany mechanicalstructureisthe is providedbytheligamentsconnectingmotionsegments.Incontrast, thethoracicspineis littlebonystability. planesandsothereisvery mobileinseveral Instead,thestability spine isvery spinecanoccurwithoutcompromisetothe spinalcorditself.Thecervical ofthecervical vertebra specialist spinalinjuriesunit. outcome, isreferraltoa ofstayinhospital,andindeedtoeventual largest differencetolength tomake the thesinglefactorwhichhasbeenshown does seemtohaveimproved.However, not changeddespitetheintroductionofstringentprotocolsforpre-hospital care,buttheoutcome permillionannum).Theirincidencehas rare(<50 spinecordinjuriesarereallyvery Cervical 14 14 13 12 11 10 8 7 6 5 4 9 At birth the spinal cord extends the length ofthespinalcanal,butbyadulthood the thelength thespinalcordextends At birth The anatomyofthespinalcolumn provides stabilitythroughthreecolumns:theanterior(the spacious,sorelatively largedisplacementsofthe regionisvery The spinalcanalinthecervical visible onX-ray. injury. Thereisaneurologicaldeficitbutnoabnormality A childsustainsahigh-speedflexion atT6. pain andakyphus thoracicspine hassevere An elderlyfemalepatientslipsandlandsonherbottom.Shenow werewearingaseatbelt. speed roadtrafficaccidentwherethey ahigh- atthethoracolumbarjunctionfollowing distractioninjury A patientsustainsaflexion onC4. forward C3subluxed combinedwithaxialloading.X-rayshows injury A patienthasahyperflexion tothediscspace. bodynext vertebral injury. asmallchipofboneoffthefront X-rayshows A patienthasahyperextension 50 percentlessatthefrontthanback. 50 is heightofC4vertebra injury. thatthevertical TheX-rayshows A patienthasahyperflexion A patienthasafracture,whichiseffectivelyspondylolisthesisofC2onC3. A patientsustainsarotationalunstablefractureofthethoracicspine. odontoid peg. atransversefracturethroughtheneckof spinereveals ofthepatient'scervical CT tohaveanisolatedruptureofthetransverseligament. A patientisnotedonMRI neckwithacockrobinappearance. awry A childspontaneouslydevelops y ofspineinjur y ➜ ➜ 3.A,B,C 2.(e)B 2.(d)A,C 2.(c)C 2.(b)C 2.(a)B

Visualisation inneckandspineinjur Visualisation Case stud injury. secondary its bloodsupply. Failure toproperlyimmobilisethespinalcordisprobablynotacommoncauseof ischaemiacausedbypressureonthecordor reduced assoonpossibletominimiseanyfurther involvesdisplacementthroughfractureordislocationthenthistooshouldbe If thespineinjury andoxygenated. can bestbeminimisedbymakingsurethatthepatientremainswellperfused ischaemiatothespinalcord.It is usuallyaresultofhaemorrhageandoedema,causingfurther injury disruption oftheanatomyspinalcanalwithdamagetocord.Thesecondary T1vertebra. below andtheT1rootexits C7vertebra below soC8exits vertebrae cervical seven number. bodywiththesame rootsandonly abovethevertebral Butthereareeightcervical exit spine,thespinalroots bodies.Inthecervical theirvertebral below toexit (cauda equina)passdown injuries, so will show upoedemaandhaematoma ofthecord. injuries, sowillshow isalwaysmuchbetter forsoft-tissue usingreconstructions. MRI oftheinjury nature andextent inthespineand canalsoprovevaluableinunderstandingthe is bestfordiagnosingbonyinjury cannot bevisualisedwithconventional filmsthena‘swimmer’sview ’ mayneed tobetaken.CT junction junction.Ifthecervicothoracic ofthe cervicothoracic provided thereisgoodvisualisation thatisonlytrue Plain X-rayswillonlydiagnose85per centofsignificantspinalinjuriesandeven presentwithusefulbutnotnormalmotorfunction. GradeEisnormalfunction. Grade Dissensory gradelessthan3). present withmotorfunctionwhichisnotgoodenoughtobeuseful(MRC andmotor. present,motorabsent (thispatient).GradeCissensory absent sensory Bissensory ineachdermatome.Thesearethencombinedintothe Frankel grade.Ais (0–5), andsensation grading system usingtheMRC measuresmusclepower Association The AmericanSpinalInjury syndromeandalsocarriesagoodprognosis. is theBrown-Sequard on theotherside proprioceptionononesidewithlossoftemperatureandpainsensation power indicatesagoodprognosis. Alossof Returnofperianalsensation bereliablyperformed. can now prognosis. Itmerelyindicatesthattheperiodofspinalshockhasendedandtestsprognosis returnsin24–48 The bulbocavernosusreflex h afterspinaltransection.Itisnoindicatorof of overloadingacirculationwhichmaylacksympathetictonewouldbebest. whileminimisingtherisk flooding thepatient.Thustitrationoffluidstoensureadequateperfusion periphery. Fluidsshouldbegivenwithgreatcareherebecauseoftherisk and awell-perfused bloodpressure,abradycardia The vitalsignsarehighlysuggestiveofneurogenicshockwithalow ruling outanunstablespineinjury. andmostreliablewayof wouldbethesafest because oftheriskcausingbedsores.AnMRI notbeleftonaspineboardanylongerthanabsolutelynecessary The patientshouldcertainly damage. anunquantifiedriskofcausingfurther carry views andextension adequate andflexion spine.PlainX-rayswillnotbe the patientisunconscious,itcanbedifficultto'clear'thecervical As 30 percent. isaround accompanyingaheadorfacialinjury injury The likelihoodofasignificantcervical Primary spinal injury is the neurological deficit apparent immediately after the injury causedby istheneurologicaldeficitapparentimmediatelyafterinjury spinalinjury Primary y : injur y afterfall y 169

24: NECK AND SPINE TRAUMA ➜ ➜ 1P,2Q,3A,4B,5B,6N,7M,8I,9F,10C, 11L, 12M, 13F, 14H matchingquestions Answers: Extended 4.(a)C;(b)(c)C 170

1. Fractures, dislocationsandsubluxations Cervical spineinjur An unstable cervical spinefracturecaneitherbetreatedintractionviaahaloorbyopen An unstablecervical haematoma of the cord (visible on MRI) withoutanyapparentmusculoskeletaldamage. haematoma ofthecord(visibleon MRI) young childrenwithahyperelasticspine wherethedeformityattimeoftraumaproduces image intensifiercontrol. under performed are usuallystablebutthepainanddeformitycanbehelpedwithvertebroplasty called theChancefracture. atthislevel injury has producedacharacteristicflexion/distraction disc. tomakesurethereisnotaprolapsed intervertebral shouldbeperformed an MRI discandalsoneurologicaldamage.Before reductionisundertaken, damage totheintervertebral lockingintoposition.Thereisoften facets maydislocateandlockoverthefrontoffacetbelow, iscombinedwithaxialcompressionthen eitherone(uni-)orboth(bi-) fracture. Ifthehyperflexion body. andunstable asateardropfracture.Itsmildappearancebelies asevere Thisisknown pullsoffasmallfragmentofbone onthefrontofvertebral Hyperextension hyperextension. unstableindeed. fracture,whichisvery the rotatory stable; typeBalsohasdisruptionoftheposteriorelementsandismoreunstable;whileC bodyoftheaxis. intothevertebral down case here)isthroughtheneck;whiletype3extends atlantoaxial instabilitymayproduceanisolatedruptureofthetransverseligament. spontaneous onsetofatlantoaxialinstability. Haltertractionshouldleadtoreduction.Traumatic axis (frontofspinalcanal)tothefrontbackedgeforamenmagnum. the frontedgeofbackatlasoverdistancebetween posterior one.Theratioisthedistancefromfrontofforamenmagnum(occiputedge)to A ratioofmorethan1indicatesanteriortranslation,andaless0.75 oftheocciputonaxis. ratiomeasuresthedegreeofsubluxation is usuallyfatal.ThePower dislocation Craniocervical fragments willneedtoberemovedatopenreductionandstabilisation. fracture. Thefragmentscanbedisplacedintothespinalcanal,causingcorddamage.These itissimplya‘burst’ as aJefferson'sfracture,whichcanbestableorunstable.Atanyotherlevel known The classicfracturefromlandingonthetopofheadisC2burstfracture,otherwise offerstheonlyoption. surgery isabsolute– isdeterioratingthentheindicationforsurgery isstable,butiftheneurology neurology arerelativeifthe Theindicationsforsurgery other complications,suchasbedsores,developing. minimisingtherisksof treatment isopenreductionasthepatientcanthenbemobilisedsafely, Thebest discwhichmightcompressthecordwhenreductionisachieved. prolapsed intervertebral tomakesurethatthereisnota EitherwayitisbesttogetanMRI reduction andinternalfixation. SCIWORA is a spinal cord injury without objectiveradiologicalabnormality. isaspinalcordinjury Thisoccursin SCIWORA minortrauma.They wedgefracturesarecommonintheelderlyfollowing Osteoporotic flexion andthe introductionofseatbelts The thoracolumbarjunctionisespeciallysusceptibletoinjury, The Hangman’sfractureiseffectivelyaspondylolisthesisofC2onC3caused bytraumatic Thoracic spinefracturesarealsoclassifiedintothreetypes:typeAisananteriorcrushand There arethreetypesofodontoidpegfracture:type1isthroughthetippeg;2(the A childwithacockrobinneckmayjusthavesternomastoidspasmbutcanalso y 25 Trauma to the face and mouth

Multiple choice questions ➜ Le Fort II maxillary fractures B The condyle is the most commonly 1. What are the common findings in fractured site in the mandible. patients with Le Fort II fractures? C The orbital roof is the weakest area of A Malocclusion the orbital cavity. B Inferior alveolar nerve paraesthesia D Nasal fractures are best treated at C Infraorbital nerve paraesthesia 3 weeks post-injury. D Palatal mobility E Retrobulbar haemorrhage is a potential E Exophthalmos. complication of fractures of the zygoma. ➜ Zygomatic fractures ➜ Epidemiology of facial 2. What are the common signs and fractures symptoms of a fractured zygoma? 5. What is the most commonly fractured A Battle’s sign facial bone? B Infraorbital paraesthesia A Mandible C VII nerve palsy B Zygoma D Supraorbital paraesthesia C Nasal E Subconjunctival haemorrhage. D Maxilla E Orbital floor. ➜ Mandibular fractures 3. The action of which of the following ➜ Epistaxis muscles can displace bilateral fractures 6. Which of the following may be of the mandible in the canine region associated with epistaxis? posteriorly? A Fractured zygomatic arch A Thyrohyoid, genioglossus and geniohyoid B Fractured anterior wall of frontal sinus muscles C Orbital blow-out injury B Anterior belly of digastric, geniohyoid D Le Fort II fracture of maxilla and genioglossus muscles E Fractured mandible. C Mylohyoid, genioglossus and styloglossus muscles ➜ Avulsed teeth D Mylohyoid, masseter and geniohyoid 7. Which of the following has an impact muscles on successful reimplantation of avulsed E Masseter, medial pterygoid and permanent teeth? styloglossus muscles. A The aetiological cause of the injury B The transport medium used ➜ Maxillofacial injuries C The use of steroids in the peri-implant 4. Which of the following statements period regarding maxillofacial injuries are D The presence of dental caries in the true? crown of the tooth A Orthopantomogram (OPT) radiographs E The length of time between avulsion and are ideal to image middle-third fractures. reimplantation.

171 TRAUMA ➜ ➜ ➜ Extended matchingquestions Extended Inapatientwhohassustained 8. scenariosgiven below: Choose and matchthecorrectdiagnosis/signswitheach oftheclinical scenariosgiven Choose andmatchthecorrectdiagnosis/symptomswitheachofclinical 172

Immediate management 1 E D C B A 2. Maxillofacialtrauma 5 4 3 2 1 E D C B A 1. Maxillofacialtrauma B A below: complains of eye painand decreasedvisionontheaffected side. complains ofeye aftertheaccidenthe totherightcheekbone. Shortly male sustainsablow A 40-year-old Palatal mobility. oftheconsensuallight reflex preservation with Increasing proptosis,markedsubconjunctival oedema,lossofdirectlightreflex Mental paraesthesia Sialocele oftheparotidgland andforeheadparaesthesia Ptosis oftheuppereyelid onthatside. of thezygoma the leftcheekandoccasionalalteredbloodfromnostril.There is noclinicalflattening totheface.Hecomplainsof numbness of ablow A 45-year-oldmalepresents7daysfollowing contacts ofherposteriorteeth. her maxillaisstable,butshedemonstratesamarkedanterioropenbite, withpremature An 18-year-old tothepointofherchin afterfallingfromahorse.Clinically femalesuffersablow scanssuggestafractureoftheanteriorwallfrontal sinus. the glabella.CT demonstrates bilateralperiorbitalecchymosis,epistaxisandcosmeticflatteningintheregionof tothefacewithabluntobject.Clinicalexamination malesustainsablow A 46-year-old bones. maxilla andnasal demonstrate significantdisruptionofthemedialandinferiororbitalrims,frontalprocess scan andCT A 30-year-oldfemalesuffersafractureoftheleftnasorbitalregion.Radiographs tissue facialinjuries. side.Therearenosoft- right, andanassociatedcerebrospinalfluidotorrhoeaonthesame 12 demonstratesapositiveBattle’ssignonthe improves overthenext h.Clinicalexamination ComaScaleof13/15 withaGlasgow he sustainsaheadinjury onadmissiontohospital.This roadtrafficaccident(RTA). maleisinvolvedinasevere A 40-year-old aresultoftheaccident As Bilateral mandibularcondylarfracture palsy nerve motorneuronVII Lower Supraorbital paraesthesia Epiphora injury Orbital blow-out obstruction. airway Inhaled bloodanddebriscancause position. The patientshouldbenursedinasupine following statementsaretrue? following whichofthe severe facialinjury, E D C considered. shouldalwaysbe spineinjury Cervical toestablishanairway. necessary Surgical cricothyroidotomymaybe blood loss. An immediatedangertolifemaybe ➜ ➜ ➜ ➜ 4.B,E 3. B 2.B,E 1. A,C,D Answers: Multiplechoicequestions

Maxillofacial injuries Mandibular fractures Z Le FortIImaxillar 5 4 3 2 mobile andmore problematic totreat. the fracturesarebesttreatedat7–14 days;afterthisperiodthebonyfragmentsbecome less soft-tissue swellinghassubsided,so thatanydegreeofdeformitycanbebetterassessed.Ideally, fracturesshouldbetreatedaftertheassociated with associatedfrontalbonefractures. Nasal cavity. Isolatedorbitalroof fracturesarerare.Injuriesoftheorbitalroofoccurmorecommonly bythe medialorbitalwall,istheweakestareaof mandible. Theorbitalfloor,followed radiographisbetterforimagingthe AnOPT andlateralfacialboneviews. occipitomental views, The mostappropriatewaytoimagemiddle-thirdfractureswithplainradiographs arewith genioglossus. bilateral parasymphysealmandibularfracturearetheanteriorbellyofdigastric, geniohyoidand The musclesattachedtotheanteriormandiblewhichcancauseposterior displacementina impinges onthecoronoidprocessofmandible. arch addition, limitationofmandibularlateralmovementscanbeseeifadisplacedzygomatic upward gaze,oculartethering,infraorbitalparaesthesiaandsubconjunctivalhaemorrhage.In arecosmeticflattening,diplopiaon The commonsignsandsymptomsofafracturedzygoma fractures, unlessthefractureisimpacted. unlessthefractureisundisplaced.Palatalmobilityseeninallmaxillary malocclusion isevident, plates.Infraorbitalparaesthesiaisacommon sign,and antrumtothepterygoid the maxillary rim, andoftenthroughtheinfraorbitalforamen.Itcontinuesposteriorlylateralof oftheinfraorbital through thebridgeofnoseandethmoids.Itcontinuestomedialpart fracturesofthemaxillaarepyramidalinshape.Thefractureinvolvesorbit,running II Le Fort y of reductioninthedimensionssuperiororbitalfissure. suggestsmedialdisplacementofthegreaterwingsphenoid,withevidence examination tothebonyorbitonleftside.Radiographic injury femalesuffersasevere A 60-year-old An 18-year-old malesuffersafracturedmandibleastheresultofanallegedcivilassault. are usedforthedeeptissueplanes,andmonofilamentnylonskinclosure. borderofthemandible.Resorbablesutures archtothelower thezygomatic from justbelow penetratingcheeklaceration,extending A 28-year-oldmaleundergoesrepairofa7cmvertical, fracture ofthemandible. ofa significant oedema.Thedentalocclusionisderanged,andtherenoclinicalevidence RTA. demonstratesbilateralepistaxis,infraorbitalparaesthesiaand Clinicalexamination An unrestrained55-year-oldfemalepassengerstrikesherfaceonthedashboardofacarin gomatic fractures y fractures 173

25: TRAUMA TO THE FACE AND MOUTH TRAUMA ➜ ➜ ➜ ➜ ➜ 3C 2B 1D matchingquestions Answers: Extended 8.B,C,D,E 7. B,E 6.B,C,D 5. C 174

1. Maxillofacialtrauma Immediate management Avulsed teeth Epistaxis Epidemiolog The nasal bones are the most commonly fractured facial bones. This is then followed bythe bonesarethemostcommonlyfracturedfacialbones.Thisisthenfollowed The nasal paraesthesia isacharacteristic sign. fromthesuperioraspectof orbit overtheorbitalrim.Ifinvolved, supraorbital exit these nerves Fractures ofthefrontalsinusmay involve thesupraorbital(andsupratrochlear) foramina,where necessitatingadacrocystorhinostomy.the lacrimalsac, aredamaged.This can resultinepiphoraorsometimesamucoceleof the lacrimalpassages regionwhere injury.eyelid Lessfrequentlyit canresultfrombonyfractureinthenasomaxillary tothecanalicularapparatus,fromsoft-tissue Epiphora mayresultfromanysoft-tissue injury parotid region. facialinjuries,unlessassociatedwithdeep,penetratinginjuries ofthe inthemoresevere even relativelyrare, palsyisotherwise through thefacialcanalbybonyfracture/disruption.Facial nerve can beinjuredasitpasses fromtheearconfirmsabaseofskullfracture.Thefacialnerve leakage CSF Battle’s signindicatesapossiblebaseofskullfractureinvolvingthemiddle cranialfossa. and headinjuriesinpatientswhohavesustainedfacialinjuries. spine Dueconsiderationmustalwaysbegiventothepossibilityofbothcervical tongue forward. lyingontheirback. ontheirbentarm,never semi-prone position,withtheirheadsupported position. To avoidcomplicationsduringtransfer,patientsshouldalwaysbenursedinthe inthesupine betransported facialinjuriesshouldnever Patients whohavesustainedsevere reimplanted toothshouldbesplintedbythepatient’sdentalsurgeonassoonpossible. Successful reimplantationalsorequires7–14 andthereforeany days’rigiddentalfixation, ormilk,areacceptablealternatives. saliva, media, thepatient’sown of anymedicaltransport medium.Intheabsence insomeformofclean,physiologicaltransport avulsed toothtransported minpost-avulsionisassociatedwithapoorprognosis.Anyperiodofdelayshouldseethe 60 tooth shouldbeadequatelyreimplantedassoonpossibleaftertheinjury. Anydelayafter factors.Ideallythe The successfulreimplantationofpermanentteethisdependentonseveral arch),maxillaandanteriorwalloffrontalsinuscanallresultinepistaxis. zygomatic (butnotthe or antracanresultinepistaxis.Inthismannerfracturesoftheorbitalfloor,zygoma cavity Epistaxis isacommonclinicalsigninfacialinjuries.Anyfracturethatinvolvesthenasal fractured thanthemandible,butthisdifferenceisprobablyinsignificant. ismarginallymorecommonly bonesandthemandible.Instudies,zygoma zygomatic Damaged teeth,bloodandsecretionscanthenfalloutofthemouth,gravitypulls y offacialfractures ➜ 5A 4C 3B 2E 1D 5A 4E

2. Maxillofacialtrauma signs of a maxillary fracture, this acquired deformity points toward abilateralfractureofthe fracture,thisacquireddeformitypointstoward signs ofamaxillary commonly bybilateral,displacedfracturesofthemandibularcondyles.Inabsenceclinical Anterior open-bitedeformitycancharacteristicallybecausedbyfracturesofthemaxilla,ormore ophthalmic division of Vth nerve, are usually evident inthissyndrome. areusuallyevident ophthalmic divisionofVthnerve, tothefrontalbranchof palsyandsupraorbital/supratrochlear palsy,duetoinjury nerve IIIrd Ptosis,dueto oculomotorandabducentnerves. involved –lacrimal,frontal,trochlear, nasociliary, passingthroughthefissure maybe structures areinvolved.Inthecomplete syndrome,allnerves the superiororbitalfissure.Therecan bevariationinclinicalpresentation,accordingtowhich Superior orbitalfissuresyndromeresults fromcompressionofthestructurespassingthrough displacedfractures,acompleteneurotmesis. more severe, maybeasimpleneuropraxia, anaxontmesisor,inthe mental foramen).Theinsulttothenerve the angleofmandibleandthoseinbody(posterior to,orinvolving,the iscommonlyinvolvedbyfracturesofthemandible, especiallythosein The inferioralveolarnerve formation. drainage, andsubsequentcyst/sialocele ductismorelikelytoresultintotalobstruction oftheduct sialocele. Tightclosureofthesevered and fistulas, parotidcysts recognize thisresultatthetimeofinitialrepaircaninsalivary inpenetratingsoft-tissueinjuriesofthecheek, andfailureto The parotidductmaybesevered fromtheparotidgland. ofsaliva A parotidsialoceleiscausedbyobstructiontotheoutflow relies onotherclinicalsignsandradiographicfeatures. fracture,which orIII I,II mobility alonedoesnothelpinclinicallydifferentiatingbetweenaLeFort a fracturedmandibleanddentoalveolarinjuries,ispathognomonicofmaxilla.Palatal Palatal mobilityistheclassicsignofafracturedmaxilla.Derangedocclusion,inabsence treatment. decompression isthenecessary retinalischaemia,buturgentsurgical Inprogressingcases,medicalmeasuresmayslow reflex. oftheconsensuallight withpreservation ophthalmoplegia andalossofthedirectlightreflex proptosis, markedsubconjunctivaloedemaandhaemorrhage,atenseglobe,dilatingpupil, thesurgeontopossibilityofretrobulbarhaemorrhage.Clinicalsignsinclude should alert ischaemia andsubsequentblindness.Symptoms ofpainanddecreasing visualacuity/blindness injury.surgical treatmentofazygomatico-orbital Ifprogressiveanduntreated,itwillleadtoretinal orsometimesasaresultof and orbitaldisruption.Itmayoccurasaresultoftheactualinjury, bodyfractures complicationofzygomatic Retrobulbar haemorrhageisarare,butimportant, fromoedemaaccompanyingtheinjury. masked intheinitialstagespost-injury antrumcanresultinenophthalmos,althoughthissignoftenbe soft tissueintothemaxillary antrum.Inaddition,theherniationofperiorbital resultsinhaemorrhageintothemaxillary injury ofepistaxis,asthe are infraorbitalparaesthesia,diplopiaonupwardgazeandoftenahistory injuriesclassicallyresultfromblunttotheglobe.Thesignsandsymptoms Orbital blow-out mandibular condyles. 175

25: TRAUMA TO THE FACE AND MOUTH 26 Trauma to the chest and abdomen

Multiple choice questions ➜ Chest and abdominal injury 4. Which of the following are not 1. Which of the following statements are potentially life-threatening injuries? false? A Aortic injuries A 40 per cent of deaths from trauma are B Tracheobronchial injuries due to torso injury. C Myocardial contusion B Good history and understanding of the D Rupture of the diaphragm mechanism of injury will help predict the E Oesophageal injuries type of injury. F Pulmonary contusion C Junctional zones help in the overall G Hollow viscus injury. management. ➜ D Multiorgan failure is the commonest Investigations cause of death in trauma. 5. Which of the following statements E The vast majority of chest injuries can be regarding focused abdominal managed closed. sonography in trauma (FAST) are false? F Early preventable deaths due to trauma A It is accurate when there is >100 mL of are often the result of lack of delay in free blood in a cavity. airway control. B The technique focuses on pericardial, splenic, hepatic and pelvic areas. 2. Which of the following statements are C It is operator-dependent. false? D It is a useful tool to diagnose hollow A 80 per cent of chest injuries can be viscus injury. managed non-operatively. E It can be used in penetrating injury. B In the unstable patient, chest X-ray is the first investigation of choice. 6. Which of the following statements are C A chest drain can be both diagnostic and untrue? therapeutic. A All patients with multiple injuries must D A penetrating chest injury always requires have a contrast-enhanced computerised a thoracotomy. scan (CECT) of the abdomen for accurate E In a chest injury, auscultation from the evaluation. front only can be misleading. B In the stable patient with intra-abdominal injury, CECT is the ‘gold standard’ of ➜ Life-threatening injury investigations. 3. Which of the following are not C Diagnostic laparoscopy has a screening immediately life-threatening injuries? role in the stable patient. A Tension pneumothorax D Video-assisted thoracoscopy (VATS) B Cardiac tamponade is an accurate method of evaluating C Flail chest diaphragmatic injury. D Open pneumothorax E Diagnostic peritoneal lavage (DPL) as an E Massive haemothorax investigation has largely been superseded F Liver injury by FAST. G Airway obstruction. F A negative DPL is often more helpful.

176 ➜ ➜ matchingquestions Extended Chooseandmatchthecorrectdiagnosiswitheachofscenariosbelow: Chooseandmatchthecorrectdiagnosiswitheachofscenariosbelow:

1 D C B A 2. Investigations 5 4 3 2 1 E D C B A 1. Chesttrauma investigation should bedoneatthisstageintheA&E department? chestandthere isaclinicalsuspicionofanintra-abdominal bleed.Whatimaging and lower 110/min andbloodpressureof130/80 mmHg.Hehasconsiderable bruisingoverhisabdomen He hasbeenresuscitatedaccording totheATLS protocolandisstablewithapulserateof chesttrauma. bluntabdominalandlower A 30-year-oldpatienthasbeenadmitted following Diagnostic laparoscopy(DL) CECT DPL FAST a suckingsoundcanbeheard. percent.Thereisanopenwoundintheregionoffifthleftinterspace throughwhich 90 of asaturation left sideofhisaxilla.Heisgaspingforbreathandpulseoximetershows A 15-year-old having beenstabbedonthe boyhasbeenbroughtintotheA&Edepartment badly bruisedandthechestwallisunstablewhenhecoughsortriestotakeadeepbreath. ofhislefthemithorax.Theskinovertheribslooks tenderoverthecentralpart and extremely tachypnoeic sustainedinamatchabout2hago.Heisvery bluntinjury chest painfollowing left-sided withsevere A 22-year-oldrugbyplayerhasbeenbroughttotheA&Edepartment mmHg, theleftchestisdullonpercussionandtherenoairentry. hisbloodpressureis80/50 marked bruisingofthechestwallonleft.Onexamination paininhischestand withsevere 35-year-old manhasbeensenttotheA&Edepartment fellonhischest,a masonry atabuildingsitewherepieceofheavy aninjury Following breath soundsdifficult. makeslisteningto hyperresonance overtheleftchestwall.ThenoiseinA&Edepartment The tracheaisshiftedtotheright,lefthemithoraxdoesnotmoveandthere percent). of90 saturation he isunabletobreatheproperlyandintenselyhypoxic(oxygen stateas inapanicky hasbeenbroughttotheA&Edepartment A 25-year-oldmotorcyclist of92percent. saturation anoxygen The pulseoximetershows but breathsoundsaredifficulttodiscernbecausethereisalotofnoiseintheA&Edepartment. mmHgandpulserateof130/min.with asystolicpressureof80 Thetracheaisinthemidline, ofthebrokensteeringwheel.Hisneckveinsare distended causedbyapart penetrating injury was ahead-oncollisionwhilenotwearingseatbelt.Thereisbruisingoverhissternumand ofbreath.Hewasinvolvedinaroadtrafficaccident(RTA) shortness with severe whenthere (A&E)department vandriverhasbeenbroughttotheaccidentandemergency A 40-year-old Massive haemothorax Open pneumothorax Flail chest tamponade Cardiac Tension pneumothorax 177

26: TRAUMA TO THE CHEST AND ABDOMEN TRAUMA ➜ Chooseandmatchthecorrectdiagnosiswitheachofscenariosbelow: 178

4 3 2 1 F E D C B A 3. Abdominalinjur 4 3 2 shows the tip of the nasogastric tube next to the heart intheleftchest. totheheart thetipofnasogastrictubenext shows andblood,anasogastric tube.AchestX-ray catheter, twoIVcannulaewithcrystalloids of92percent. Hehasanindwelling saturation ofbreathwithanoxygen continues tobeshort pressure comesup to120/90 mmHgandhis pulserateis100/min. HisCVPis5cmH resuscitationbytheATLSand bruisinginthe rightupperquadrant.Following protocol,hisblood Cullen’ssignwithmarkedtenderness shows his rightfifthtoninthribs.Abdominal examination breath withtachycardia.Hispulseis 110/min, mmHg andclinicallyhehasfractureof is90/70 BP of pain,short severe rightchest.Heisinvery being kickedinhisupperabdomen andlower by havingbeenviciouslyassaulted manwasbroughtto theA&Edepartment A 50-year-old lumbar regionandrightloinwithsome obliterationoftheloincurve. andbloodpressureof110/8090/min hehadbruisingovertheright mmHg.Onexamination He wasstablewithapulseof urine itwashaematuria.HebroughttotheA&Edepartment. rugby. Afterabriefperiodofrest,hefinishedhisgame.Inthe changingroomwhenhepassed A 17-year-old boysustainedakickintherightsideofhis abdomenandloinwhileplaying stable after4h–pulseof100/min, bloodpressureof 130/90 mmHgandCVPof4cmH mmHg.Hehasbeenresuscitatedby theATLSand abloodpressureof80/60 protocolandis abdomen.Heistachypnoeicandinhypovolaemicshock,with a pulseof130/minon hislower fell pieceofmasonry atabuildingsitewhenheavy A 30-year-oldmansustainedaninjury failureandgrossabdominaldistension. compromise,type2respiratory cardiorespiratory for tornliver. hebecameoliguric,had Onthethirdpostoperativeday,whilestillinITU, hepatectomy abdominal trauma.Hehadasplenectomyforrupturedspleenandpartial bluntupper A fit35-year-oldmanwhowasworkingonabuildingsitesustainedsevere syndrome Abdominal compartment Diaphragmatic injury Renal injury Splenic injury Pancreatic injury Liver injury you consider? FAST needstobeexcluded. intraperitoneal injury isnormal.Whatscreeningproceduremight painaroundtheinjuredsite,heisstablehaemodynamically.Although insevere Atthisstagean tohiscentralabdomen. A 25-year-oldmalepatienthasbeenbroughtinwithastabinjury isnotimmediatelyavailable.Whatinvestigationwouldyouconsider? not availableandCECT FAST ofhisco-morbid disease.Unfortunately confirmation oftheclinicalsuspicioninview is Although clinicallyhaemoperitoneumissuspected,alaparotomywouldbeconsideredonlyon andangina. disease(COAD) ofchronicobstructiveairways his abdomen.Hehasahistory pulse is110/min andbloodpressureis110/80 mmHg.Helookspaleandistenderallover abdomen.AfterresuscitationaccordingtotheATLSin hisupperandlower protocol,his havingbeenbrutallyassaulted A 70-year-old patienthasbeenbroughtintotheA&Edepartment toconfirmthediagnosis? the subhepaticpouch.Whichimagingtechniqueshouldbeusednext umbilicus (Cullen’ssign).Clinicallyliverlacerationissuspected.AFAST fluid(blood)in shows right shouldertip.Heisstablehaemodynamicallyandtachypnoeicwithbruisingaroundhis complainingofright-sidedabdominalpainwhichisradiatingtothe into theA&Edepartment bluntabdominaltrauma,afteraboxingmatch,22-year-oldmalehasbeenbrought Following y 2 O. 2 O. He ➜ 1. 1. D Answers: Multiplechoicequestions

Chest andabdominalinjur 6 5 protocol takesthe highestpriority. Foreign bodiessuchasdentures, teeth,bloodandsecretions control–hence the‘A’attention toairway ofABCDintheAdvanced Trauma (ATLS) LifeSupport has beeninserted. and intercostaldrainage.Anopenpneumothorax shouldnotbecloseduntilanintercostaldrain be managednon-operativelybythe closedmethod.Mostpatientsrequireappropriateresuscitation syndrome. response combination ofhypothermia,acidosis,coagulopathyandsystemicinflammatory entertained. ofsuspicionmustbe and theurologist.Retroperitonealinjuriesmaybemaskedahigh index surgeon,theneurosurgeon,cardiothoracic thegeneralsurgeon the orthopaedic approachby challengingandrequireamultidisciplinary the pelvis.Suchinjuriesareextremely neck andthechest,betweenchestabdomen, theabdomenand zones, referredtoasjunctionalmaybedamaged.The zonesare:betweenthe ofwhichisessentialforpropermanagement.Eachsidethetraditional anatomical knowledge column. injuries resultfromthecompressionbetweensteeringwheeloraseatbeltandvertebral duodenalandpancreatic injury; overthebeltwithvertebral injury belts) arepronetoacuteflexion to impactwiththeground;thosewearingonlylapseatbelts(asopposedfullanddiagonal legsandpelvisthentheheaddue tothelower face, chestandknees;pedestrianssufferinjury to unrestrainedvehiclepassengerssufferinjury those atthesceneofaccident.For example, isoftenobtainedfromtheparamedicalstaffandsometimes damage inthepatient.Thishistory in theearlyin-hospitalsituation. percentofdeathsoccur injuredpatientsarereachingthehospitalsothatalmost50 more severely hours andtheremaining20percentarelatedeaths.Withimprovementsinpre-hospitalcare, Half thedeathsfromtraumaoccurinpre-hospitalsituation,30percentduringfirstfew ofthebleeding. haemorrhage, mostofthesebeingtheresultfailuretoappreciateextent Torso percentofdeathsfromtrauma.Thereasonforthisismajor accountsfor40 injury when shefelloffherbicycle. thatitoccured10the skinisbruised.Whenaskedaboutbruising,shesaid daysago upper abdomenwherethereisasuggestionofsmoothfirmmassintheepigastrium shehasadistended and afeelingoflistlessnessshehasbeenoffschool.Onexamination pain of10 days’duration.Shehasnotbeenfeelingwell,withnauseaandoccasionalvomiting, A 10-year-old complainingofupperabdominal girlhasbeenbroughttotheA&Edepartment shoulder injury. Shehasfractureofherleft9th,10th and11th ribs. is 100/60 tenderovertheleftupperquadrantandclinicallythereisno mmHg.Sheisvery shoulder whichhadimpactedagainstthefrontseat.Herpulseis110/min andbloodpressure seatbelt. Sheisbreathlessandcomplainingofpainintheleftupperquadrant Shehasbeenbroughtinwithgeneralisedabdominalpainacrossthesiteof extricated. RTA, asevere Following a35-year-oldfemalepassengerinthebackseatofcarhadtobe Multiorgan failureaccountsforaminorityof7percentdeaths.This isusuallyfroma usuallytraversesanatomical zonesinthebody,a In apatientwithmultipletrauma,theinjury helptopredicttheactual andunderstandingthemechanismofinjury A goodhistory Immediately following trauma, the commonest cause of preventable deathis due tofailureof trauma,thecommonestcauseof preventable Immediately following percentofchestinjuriescan Thoracic injuriesaccountfor25per centofalltrauma.About80 y 179

26: TRAUMA TO THE CHEST AND ABDOMEN TRAUMA 2. D ➜ ➜ ➜ 3E 2A 1B matchingquestions Answers: Extended 5.D,E 6. A 4. 3. G F 180

1. Chesttrauma Investigations Life-threatening injur spine. controlwithintubationandofcervical airway callforexpert to thyroidandcricoidcartilages oedemaandinjury factors.Maxillofacialtraumawithdelayednasopharyngeal are thecontributory than 200mLofblood perhover3–4h.Urgentvigorous resuscitationforthehaemorrhagic with collapsedneckveins,absenceof breathsoundsanddullnessonpercussiontheleftside. Thepatientsuffersfromhaemorrhagic shock artery. from intercostalvesselsortheinternal mammary typically willproduceamassivehaemothorax,thebleedingoccurring This patient’stypeofinjury engorgement ofneckveins.Thedifferential diagnosisiscardiactamponade. route ofescape,causingcompletelungcollapse.Thereisadecreasein thevenousreturn,causing air leaksthroughaone-wayvalvefromthelung.Airisforcedinto chest cavitywithoutany aCVPlinethroughthesubclavian route.Thisoccurswhen iatrogenic lungpuncturewhileinserting line iscarriedout. ofanintercostaldraininthefifthspace intheanterioraxillary procedure ofinsertion onetobuytimeforsome20minorsoduring whichthedefinitive midclavicular line.Thisallows alarge-boreneedleintothesecond intercostalspaceinthe be dealtwithimmediatelybyinserting whichneedsto Thisisanabsoluteemergency difficult tohearinabusyA&Edepartment. trachea,hyperresonantpercussionnoteandprobablynobreathsounds, whichmaybe deviated hasalefttensionpneumothorax.Heintensehypoxia,istachypnoeic, This motorcyclist theJVPandCVPmaynotbeelevated. the patienthaslostalargeamountofbloodelsewhere, if measurebeforeaformalthoracotomyiscarriedoutbycardiacsurgeon. However, emergency Withdrawal of10–15 mLofbloodwillresultinmarkedimprovementandisalife-saving treatment. controlistheemergency if thediagnosisisindoubt.Pericardiocentesis underECG bleeding elsewhere. Theneckveinsmaynotbeengorgedifthereisserious surroundings ofanA&Edepartment. The lastsignmaynotbereliableasitdifficulttointerpretbecauseofnoiseinthebusy sounds(Beck’striad). systolic pressure,raisedjugularvenouspressure(JVP)andmuffledheart proven.Theclassictriadconsistsoflow deemed tohavepericardialtamponadeunlessotherwise whoisshocked neartheheart penetrating chestinjury. Anypatientwithapenetratinginjury directbluntand This vandriverisashockedpatientwithdistendedneckveinsfollowing The usual causes are penetrating or blunt chest trauma with lung injury, causingairleakor The usualcausesarepenetratingorbluntchesttraumawithlunginjury, A chestX-ray,FAST, echocardiographyandarisingcentralvenouspressure(CVP)mayhelp Massive haemothorax isdefinedasaninitialdrainage ofmorethan1.5 Lofblood ormore y 5D 4C ➜ 3B 2C 1A

2. Investigations needed. asathoracotomymightbe suction. Anearlyreferraltoacardiothoracicsurgeonismandatory Thedrainmaybeattachedtoalow-pressure (using aslargeadrainpossible)isinserted. that theaircanescapeoutandnotcauseatensionpneumothorax.Aformalintercostaldrain immediately closedwithaplasticOpsitedressingtapedonlyonthreesidestocreatevalveso This youngboyhasanopenpneumothoraxorasuckingchestwound.Thewoundshouldbe chest. inselectedcasestostabilisethe internaloperativefixation to opiates.Somesurgeonsperform painisagoodalternative blocktorelieve help internallysplinttheflailsegment.Intercostalnerve inthehypoxia. due topain;paradoxicalrespirationcontributesaminorpart underlyinglungcontusionandlackofchestwallmovement primarily causedbytheinevitable whenthepatientbreathesorcoughs.Hypoxiais is aclinicaldiagnosismadeonobservation itmovesoutwards–paradoxicalrespiration.This chest wallmovesinwardsandonexpiration there aremorethan3ribsfracturedattwosites.Oninspirationtheloosesegmentof segment ofthechestwallthatisdiscontinuouswithrestthoraciccage.Itoccurswhen segment ofchestwallontheleftside.Thisistypicalaflailchest.Adefinedas totheleftsideofhischest,hasanunstable This rugbyplayer,havingsustainedabluntinjury thoracotomy byacardiothoracicsurgeon. byanurgent ofanintercostaldrainiscarriedout.Thisfollowed byinsertion shock followed particular patient, DPL would be very useful. wouldbevery patient, DPL particular presence ofblood intheperitonealcavitydoesnotalways requirealaparotomy. Hence,inthis isoftenmorehelpfulbecausethe indicates diaphragmaticinjury. Ingeneral,anegativeDPL whohasanintercostal drain,egressoflavagefluidthroughthechestdrain with chestinjury percentsensitivity rateforbloodanda1percentcomplicationrate.Inpatient has97–98 DPL isused.Ingoodhands, FAST isnotreadilyavailable,aDPL diagnosis. As is unavailableandCECT itisbestnottodosowithoutafirm haemoperitoneum andlaparotomy maybenecessary, hasco-morbid disease.Althoughhemayhave This elderlypatient,whohasbeenassaulted, istheinvestigation ofchoice. asthepatientisstableatpresent.ACECT particularly damageforfuturereference, theexact toascertain managed conservatively. Butitisimportant of bloodtrackingalongtheligamentumterestoumbilicus.Themajority ofliverinjuriesare withpainintherightshouldertipandpositive Cullen’ssignbecause right-sided abdominalinjury from bluntabdominaltrauma: This youngboxerclinicallyhastheclassicalfeaturesofliverinjury viscus. tohollow isfullofgas.Itwillnotdetect injury unreliable inanobesepatientorifthebowel carried out. It isarapid,reproduciblebedsidetechniqueandcaneasilybedonewhileresuscitationbeing presence ofbloodorfluid.Thetechniquelooksatthecardiac,splenic,hepaticandpelvicareas. cardiactamponade)andabdomenforthe any radiation,usedtoassessthechest(particularly haemoperitoneum. FAST investigationofchoice.Itisanon-invasivetechnique,without isthenext this patientisstable,oneneedstofindoutifthereanyhaemopericardiumor As Pneumothorax orhaemothoraxisacomplication.Thetreatmentmechanicalventilationto FAST of100 willdetectfreefluidorbloodwhenitisinexcess mL.Itisoperator-dependentand 181

26: TRAUMA TO THE CHEST AND ABDOMEN TRAUMA ➜ 4A 3D 2E 1F 4D 182

3. Abdominalinjur prevent an unnecessary laparotomyinpenetratingabdominaltraumaalthoughtheinvestigation anunnecessary prevent requiringalaparotomy. tothediaphragmoranyinjury Itisusefulto intraperitoneal trauma,injury isausefulmethodtodetectany In stablepatientswithpenetratingtraumatotheabdomen,DL carried out to determine the exact degree of liver injury and exclude othersolidorgandamage. andexclude degreeofliverinjury carried outtodetermine theexact scan is patient isresuscitated accordingtotheATLS reasonablystable.ACECT protocol. Heisnow umbilicus throughtheligamentum teres. Liverisdamagedtypicallyasaresultofblunttrauma.This becausebloodfromthetornlivertracksto Cullen’s signwhichisatelltalefinding ofliverinjury chestandabdomen.Hehas patient’sinjuriesare mostlyontherightlower This badlyassaulted hydronephrosis orhorseshoekidney. suchasacongenital toapathological kidney, thesurgeontopossibilityofinjury should alert minor trauma Haematuriafollowing embolisation. torenalartery renal angiogramwithaview if thehaematuriadoesnotimprovewhilepatientisstillstable,there isaplaceforselective noted (seeFig.26.3).Thiswouldgiveanideaifthehaematuriaisgetting better. Inselectedcases, inatransparentjarandthetimeof passing urineis patient passesurine,someofitissaved patient is17 yearsold. asthe radiationis avoided particularly scan,unnecessary information canbeobtainedfromanUS scan. Butasthesame days’timetomakesurethatitisreducinginsize.SomemaydoaCECT few ina ultrasound isdonetogaugethesizeofperirenalhaematomaand repeated ifnecessary out (seeFig.26.1). isnormal.Arenal Thisismainlydonetomakesurethattheotherkidney After institutingATLSiscarried intravenousurogram(IVU) anemergency resuscitationpolicy, asaresultofbluntabdominalandlointrauma. Heisstable. This youngmanhasrightrenalinjury andrepairedifnecessary. abdominal organsisexcluded to injury. Surgicalrepairiscarriedoutinallcases.Theabdominalroutepreferredsothatinjury stable, VATS intra-abdominal isalsousefulifonlytoexclude istheinvestigationofchoice.ADL confirmstheinjury. thepatientis As totheheart chest X-ray,thetipofnasogastrictubenext onetothepossibilityofadiaphragmaticinjury. Onthe shouldalert force. Thisnatureofinjury tohisabdomenandpelvis,resultinginacompression This patienthassufferedabluntinjury percent. is carriedoutlater. ofabout60 Theconditionhasahighmortality abdominal traumawouldleavetheabdomenopen,coveredwithaplasticmesh.Definitivesurgery aftersevere majorsurgery immediate abdominaldecompression.Somesurgeonsfollowing indwelling catheterwillhelpinmakingadiagnosis. accountforthiscomplication. reduction invisceralperfusion embarrassmentfromraisingofthediaphragm and in preloadandincreaseafterload,respiratory deterioration oftheirindividualfunctions.Renalfailure,decreasedcardiacoutputfromreduction from raisedintra-abdominalpressure,whichcompressestheindividualorgans,therebycausing the patientwithmultipleinjuries.Quiteoftenitisnotrecognised.Allclinicalfeaturesresult andmorbidityin Thisisamajorcauseofmortality syndrome(ACS). abdominal compartment traumaarepronetothecomplicationof following Patients whohaveundergonemajorsurgery should notbeusedasasubstituteforopenlaparotomy. The vast majority of renal injuries are treated conservatively (see Fig. 26.2). Every timethe (seeFig. 26.2).Every The vastmajorityofrenalinjuriesaretreatedconservatively An intra-abdominalpressureofmorethan25mmHgissinisterandthepatientrequires Measurement ofintra-abdominalpressurebymeasuringtheintravesicalthrough y side ofhisabdomenandloinwhilepla both kidne Figure 26.2 concavit Figure 26.1 These findingsare t extravasation ofcontrastontheleftside;andnormalexcretion from rightkidne systems. The principles of surgery in liver injury canbesummarisedbythefourPs–push, inliverinjury systems. Theprinciplesofsurgery canbegradedbyvarious Liverinjury the patientbecomesunstable,laparotomyisundertaken. If the patient remains stable, liver injuries are treated conservatively, asmostofthemare.If If thepatientremainsstable,liverinjuriesaretreatedconservatively, y totheright;elevatedleftdomeofdiaphragm;losspsoasshadow; y s perfectl Emergenc Intravenous urogram ofthesamepatient6monthslater showing y pical ofaleftrenal injur y functioningafterconservative managementoftheinjur y intravenousurogram in17- y ing rugb y withaperi-renal haematoma. y , showing:markedscoliosiswith y ear-old bo y kickedintheright y . 183 y .

26: TRAUMA TO THE CHEST AND ABDOMEN TRAUMA 6B 5C 184 route. isdonebytheendoscopic largerthan6cmwillneedtobedrained.Acystogastrostomy A cyst Depending uponthesizeandpatient’ssymptoms,itshouldbemanaged accordingly. pseudocyst. theensuing10hospital help.Over amass intheepigastriumtypicalofa daysshehasdeveloped suffered fromtraumaticacutepancreatitiswhichobviouslywasnotclinicallybadenoughtoseek 10 handlebarinjury. Thisgirlsustainedsuchaninjury from acycle daysago.Atthetimeshe The commonestcauseofacutepancreatitisinachildisfromblunttraumatotheabdomen This 10-year-old ofthepancreas. girlhastheclassicalmanifestationofatraumaticpseudocyst in ameshbagisattempted. byplacingit laparotomy isdoneatwhichasplenectomyorsplenorrhaphysplenicpreservation a ifshecontinuestobestable.Otherwise will confirmthediagnosis.Thepatientisobserved scan diaphragmatic irritationfrombloodundertheleftdomearupturedspleen.ACECT ribsandreferredpaintohershouldertipdue and abdomen.Shehasfracturedleftlower chest This femaleback-seatpassengerinacarhashadblunttraumaovertheleftlower unit. hepatobiliary should betransferredtoatertiary injuries,afterpackingthedamagedliver,patient severe Pringle’s method,plug,pack.Invery during conservativemanagementshowingthattheurineisbecomingclear. Figure 26.3 This should be confirmed by ultrasound scan, which would show a cyst behindthestomach. acyst This shouldbeconfirmedbyultrasoundscan,whichwouldshow Serial specimensofurinecollectedfrom apatientwithrenal injur y

27 Extremity trauma

Multiple choice questions ➜ Commonly missed injuries D It takes account of whether or not there 1. Which of the following important is soft-tissue cover of fractured bone. injuries are commonly missed, E It takes account of contamination. because some X-rays may look F It takes account of the body part normal? involved. A Fractured neck of femur ➜ B Fractured patella Growth plate injuries C Posterior dislocation of the shoulder 4. Which of the following statements D Perilunate dislocation about growth plate injuries are true? E Colles’ fracture A Fractures which pass along the line of the F Broken ribs physis (Salter–Harris type 1) commonly G Compartment syndrome cause growth arrest. H Slipped upper femoral epiphysis B Fractures which pass along the I Talar neck fracture epiphyseal plate and then deviate off into the metaphysis (Salter–Harris type 2) are ➜ Compartment syndrome the commonest and rarely cause growth 2. Which of the following statements arrest. regarding compartment syndrome are C Fractures which track along the true? epiphyseal plate and then deviate into A It is detected by a loss of distal the joint (Salter–Harris type 3) may cause and sensation. arthritis secondary to joint incongruity. B It produces pain out of all proportion to D Fractures which pass across the the injury. epiphyseal plate from the metaphysis to C It is more common in closed fractures the epiphysis (Salter–Harris type 4) are than open ones. unlikely to cause problems. D It only occurs close to a fracture. E Fractures which compress the growth E It is a result of raised pressure in a plate (Salter–Harris type 5) are difficult to compartment collapsing the veins. diagnose and cause growth arrest. F It is a clinical diagnosis. ➜ G It is treated by a fasciotomy. Treatment of fractures 5. Which of the following statements ➜ Classification of fractures regarding the treatment of fractures 3. Which of the following statements are true? regarding the Gustillo and Anderson A All fractures should be reduced. classification of fractures is true? B Reduction of a fracture means jamming A It applies only to the soft tissues. the fragments together. B It relies primarily on the length of any C All fractures should be stabilised. laceration. D Relative stability means that some C It is influenced primarily by the energy movement at the fracture site is going involved. to occur.

185 TRAUMA ➜ Extended matchingquestions Extended Choose and match the optimal imaging modality witheachofthescenariosgivenbelow: Chooseandmatchtheoptimalimagingmodality statements Whichofthefollowing 6. 186

6 5 4 3 2 1 F E D C B A 1. Imagingintrauma A G F E weight onthatlegitisliabletogivewayunderhim,andalsolocks occasions. time heturnswithhis A rugbyplayeristackledandfeelsacrackinhisknee.Sincethen,every lateral malleolus. tender overthemedialand A footballerhastwistedhisankleinatackle.Theisvery needsreconstruction. which now A roadtrafficaccidentvictimpresentswithamulti-fragmentanddisplaced fractureofthepelvis, pathological fractureofthefemur. You ifthere areanyothersecondaries. wanttoknow presentswith weightlossanda 5yearspreviously A patientwhohadabreastcancerexcised held withK-wiresintroducedpercutaneously. distalradialfractureisbeingreducedundergeneralanaestheticand A displacedintra-articular onset andaccompaniedbyasharpcrackingnoise. A patientpresentsafteragameofsquashwithpaininthebackcalfwhichwassudden scan Computed tomography(CT) Bone scan Ultrasound Magnetic resonanceimaging(MRI) Fluoroscopy Plain X-rays does notdamagecirculation. Plaster ofParishastheadvantagethatit true? regarding theholdingoffractures is compromised. blood supplytothebonewillbe it,the attempted if,inachieving Absolute stabilityshouldnotbe bone healing. Absolute stabilityleadstosecondary fracture together. compressing thefragmentsof reductionandthen getting exact Absolute stabilityisobtainedby F E D C B a fracture. shouldnotbeusedtotreat where thereislossofsofttissue. areespeciallyuseful fixators External bones. used ingrowing nailsshouldalwaysbe Intramedullary to beobtained. absolutestability allow Plates andscrews delayed healing. Treating afractureintractionleadsto ➜ ➜ Choose andmatchthediagrams offractures inFig.27.1 witheachofthedescriptionsgiven Chooseandmatchthecorrectdiagnosiswitheachofscenariosgivenbelow:

Figure 27.1 1 D C B A 3. T I H G F E D C B A 2. T below. Decidewhethereachisstableorunstable. he canwalkonit. prominent lumpontheoutsideof thigh.Four monthsafterthefractureitispainless, and refuses surgery. aperiod of8weeksthefemurbendsinto15º Over ofvarus,producinga A patientbreakshisfemur. Itistreatedintractionandthenwithaplasterbrace,becausehe Delayed union Infected non-union Non-union Malunion Spiral Oblique Transverse Wedge Segmental Angulated Translated Shortened Rotated y y pes offracture pes offracture union 1345 6 2 7 8 9 187

27: EXTREMITY TRAUMA TRAUMA In each of the following scenarios,choosetheoptimummethodofstabilisation: Ineachofthefollowing ➜ 188

11 11 10 9 8 7 6 5 4 3 2 1 J I H G F E D C B A 4. Methodsofholdingfractures 4 3 2 External External fixator Hemiarthroplasty A youngmanfallsandsustainsthree brokenribs. fracturedneckoffemur.An elderlypatientfallsandsustains adisplacedpertrochanteric An elderlypatientfallsandsustains adisplacedsubcapitalfracturedneckoffemur. joint. The talushasshiftedlaterallyinthemortice hasafractureofboththetibia andthefibula. A footballerbreakshisankleinatackleandnow femur. A childisinvolvedinaroadtrafficaccidentandsustainsfractureacross themidshaftof The womaninScenario5hasthefracturereducedbutitwillnotstay position. reduced toabetterposition.Thereductionisgoodandtheposition fairlystable. angulatedandsois An elderlywomanfallsandsustainsadistalradialfracturewhichisvery ulna. A teenagerfallsandsuffersatransversedisplacedmidshaftfractureof both theradiusand ripped offoveradistanceof10 cm. sustainsafractureofthetibiawhereskinandmusclehavebeen A youngmotorcyclist through theupperthirdoffemur. hewasreasonablywell. Upuntilnow An elderlymanwithcarcinomaofthelungandmetastasespresentsapathologicalfracture An elderlywomanfallsandsustainsanobliquefractureofthemidshafthumerus. nail Intramedullary Analgesia Kirschner (K)wires–theKapandjitechnique Plaster ofParis Plates andscrews Collar-and-cuff traction Balanced traction Dynamic hipscrew although somecallusisvisibleonX-ray. in aplaster. At6weeks,theplasterisremovedbutitclearthatfracturestillmobile, A childsuffersasupracondylarfractureofthehumerusafterfallingouttree.Itistreated pain andnosignofunionthefracture. 3monthslater,thefracturesiteisredandhot.Therealotof nail.Atreview intramedullary A youngmanhasafractureofthetibiainfootballaccident.Itisfixedwithlocking some monthslater. plateisputinbutthisalsobreaks Anew that theplatebreaksandfracturefallsapart. 3monthsaftertheaccident,fractureisstillvisibleonX-ray.but atreview Someweeksafter endoftheirfemur.An elderlypatientfallsandbreaksthelower Itisfixedwithabladeplate ➜ ➜ ➜ 3.A,D, E 2.B,C,E,F,G 1. C,D,G,H,I Answers: Multiplechoicequestions

Classification offractures Compartment s Commonl concern is the energy that has been imparted to the softtissuesandhencetheirdisruption. thathasbeenimparted concern istheenergy ofanylacerations,asitsprimary state ofthesofttissues. Ittakeslittleaccountofthelength The GustilloandAndersonclassification isdesignedforopenfracturesandappliesonlytothe surgically (fasciotomy)andthenleft openuntiltheswellinghasreduced. involvedneedtobeopened If thereisnotimmediatereliefofsymptomsthenthecompartments factorwhichmightbecausingcompression, suchasaplasterorbandage. to releaseanyexternal needs tobereleasedwithinhoursifpermanentdamageisavoided. Theinitialtreatmentis pain.The pressureinthecompartment extreme thepatientexperiences (passive extension), and sothediagnosisisaclinicalone. pressure difficulttomeasuretheintracompartmental andischaemiabegins.Itisvery compartment bedofthe pressure, allcirculationceasesbecausebloodcannotenterorleavethe capillary has reachedmeansystolic be grosslynormal.Oncethepressureinaffectedcompartment inthefootwill willbeaffectedandsosensation neither theanteriornorposteriortibialnerve itwillbeunaffected. not passthroughthatcompartment does stillpumpblood,andifthedistalnerve maybeunimpaired,becausethearteries sensation thebloodfromdrainingout.Thedistalpulsesandeven continues torise,andthisprevents isthereforesetupwherethepressureincompartment Aviciouscycle the compartment. continuetopumpbloodinto butthehigh-pressure,stiff-walledarteries from thecompartment, pressurecollapseinonthemselvesandnolongerdrainblood veins containingbloodatlow Thethin-walled fascialcompartment. bruising) increasingthepressureinsideaninextensible andsmallmusclesofthehandfoot.Itresultsfromswelling (usually tibial compartments e.g.peronealandanterior fascialsac, thatthemusclesareheldinaninextensible anywhere oftheforearmbut canoccur compartment ofthecalfandinflexor compartment flexor it ismorecommoninclosedfracturesthanopenones.Itoccursmostcommonlythedeep fracture, itcanresultfromanyblunttraumaandthebonedoesnotneedtobefractured.Infact, syndromeisasurgicalemergency. Althoughitisnormallyassociatedwithaclosed Compartment to avascularnecrosisofthetalarbody. have direconsequences,asthefracturemaydividebloodsupplyfromtalarheadandlead oftheslip. normal onanX-ray. thetrueextent shows Onlyafrog’s-leglateralview its presence.Itcanonlyreliablybediagnosedbyhavingahighcoefficientofsuspicion. tolookforthechangeinrelationshipofbonescarpus how X-ray unlessyouknow it looksalmostnormalontheanteroposterior(AP)X-ray. dislocation oftheshoulderismuchrarerthanananteriorandeasilymissedonX-rayas All theconditionswhicharecommonlymissedrareandalsodifficulttodiagnose.Posterior Talar neckfractureislikethescaphoidfracture.ItcanbedifficulttoseeonX-rayandyet Slipped upperfemoralepiphysisislikeaposteriordislocationoftheshoulderandlooksfairly rareandhasnoreliableinvestigationforprovingorexcluding syndromeisvery Compartment difficulttoseeon Perilunate dislocationsaremuchrarerthanascaphoidfractureandvery The patient experiences pain out of all proportion to the injury, and ifthemusclesarestretched totheinjury, painoutofallproportion The patientexperiences ofthelegthen inthedeepposteriorcompartment syndromedevelops If acompartment y missedinjuries y ndrome 189

27: EXTREMITY TRAUMA TRAUMA ➜ ➜ 6.C,E 5.D,E,G 4.B,C,E 190

Treatment offractures • • • • • Growth plateinjuries ofthebody. greatlyindifferentparts supply andsoft-tissuecovervary involved,despitethefactthatblood outcome. Itdoesnottakeanyaccountofthebodypart indeterminingtreatmentandpredicting contamination, asboththesefactorsareimportant It alsotakesaccountofwhetherthereissoft-tissuecoverthefractureand mobilise, leavehospital andreturntomanyactivitiesof dailylivinglongbeforethefracture hasunited. enables, onoccasions, absolutestabilitytobeobtained. Inthesecircumstancesthe patient can (callus formation)occurs.Thisisthe fastestformoffracturehealing. bonehealing created inthesofttissuestostabilise it.Thisstabilityisrelativeandsosecondary compromise ofthelimb.Thisiswhy allinitialplastersshouldbesplit. ifaclosedplasterisputon,swelling in theearlystagesmayleadtocirculatory However, afracturetobestabilised(relatively)withouttheneed foropensurgery.Plaster ofParisallows the fragmentsofbonemaygetstrippedfromtheirsupplyandso failtounite. fractureisthat remodelling. Theriskofattemptingtoobtainabsolutebonestabilityin acomplex then compressedtogether. asbone bonehealing, whichisineffectthesame Itleadstoprimary boneunionwithcallusformation. fragments missing.Relativelystablefractureshealbysecondary be Thismaybebecausethefractureisinmultiplefragmentsor theremayeven site isinevitable. willheal.Relativestabilitymeansthatsomemovement atthefracture they stabilised. Nevertheless arequitestablealready.impacted thatthey Others,likebrokenribs cannoteasilybereducedor needed. necessary. Ifthefractureisundisplacedordisplacementdoesnotmatter,reduction back togethersothattheshapeandalignmentofbonearecorrect.Thisisnotalways There aretwostagesintreatingafracture.Thefirstoftheseisreduction,orputtingthefragments classification: Salter—Harris plateareclassifiedaccordingtoshapeandprognosisusingthe Fractures involvingthegrowth is a rare fracture. But growth arrest is common as the growth platehasbeendestroyed. arrestiscommonasthegrowth is ararefracture.Butgrowth 5isacrushfractureoftheepiphysealplate.ThismaynotbeobviousonX-rayand Salter-Harris arrest. metaphysis totheepiphysis.Thiswillcausegrowth then, whenhealingoccurs,abarofbonebridgesacrosstheepiphysealplatefrom 4fracturescrossfromthemetaphysistoepiphysis.Ifthereisanydisplacement Salter–Harris sothisfractureneedscarefulmanagement. premature traumaticosteoarthritis, then onintothejointline.Ifitcreatesastepinjoint,thereissignificantriskofonset 3issimilarto2buttheangulationoffracturelineintoepiphysisand Salter–Harris (metaphyseal)sideoftheepiphysealplate. again, itistrackingalongthesafe metaphysis. Thisfractureisbyfarthemostcommonandluckilyhasagoodprognosisas,once 2issimilarto1,Salter–Harris andbreaksoffafragmentofthe butthefracturelineexits potentialoftheplate. from theepiphysis,sothisfracturedoesnotdamagegrowth along theepiphysealplate.Themetaphysisisweakestsideandbloodsupplyenters slip)whichpassesalltheway 1israreandatranslationalfracture(sideways Salter–Harris Fixation with plates and screws is one form of open reduction and (ORIF). It (ORIF). isoneformofopenreduction andinternalfixation withplatesandscrews Fixation The alternative,non-operativeway of treatingafractureistoapplytractionandusethetension and Absolute stabilityisonlypossiblewhenthebonefragmentscanbereduced perfectly Someareso stabilisation. Notallfracturesneedextra The secondstageis‘stabilisation’. ➜ ➜ 3G 2H 1I 6C 5A 4F 3E 2B 1D matchingquestions Answers: Extended

2. T 1. Imagingintrauma axial loadingmerely compressesit. reductionisobtained, as player landsontheir leg.Itcanbearelativelystable fracture, ifaperfect occursinthetibiaoffootballerswhenanother point, whilethetwoendsarefixed). Thisinjury This isatransversefracture,caused bythree-pointloading(aforceisappliedacrossthefracture out ofpositionifweightistakenon thisbrokenbone. This isanobliquefractureprobably causedbyabendingforce.It,too,isunstableandwillslide femur ofyoungchildren,itiscommonlytheresultnon-accidentalinjury. force.Itisanunstablefracture. Ifitisseeninthe This isaspiralfracturecausedbyrotatory injury. isthebestimagingmodality forthistypeofsoft-tissue ligament oratornmeniscus,both.MRI The rugbyplayerhasaninternalderangementoftheknee,eitheraruptured anteriorcruciate best wayofimagingthistypeinjury. A footballerwhohasgoneoveronhisanklemayhavefracturedthemalleolus.PlainX-raysare this. scanallows the reconstruction.CT fractures,athree-dimensionalreconstructionofthefracturewillbehelpfulinplanning In complex activity.for increasedlytic thewholeskeletontobescreened If metastasesaresuspected,aradioisotopebonescanallows the radiationdosetoaminimum. thiswhilekeeping the bonefragments.Fluoroscopy(imageintensifier)isbestwayofachieving togetasetofimages When fracturesarebeingreducedunderanaesthetic,itmaybenecessary Soft tissuesareusuallybestvisualisedwithultrasound,especiallyifdynamicimagingisneeded. Sudden onsetofpaininthecalfduringagamesquashmayberuptureAchillestendon. loss ofbloodsupplytofragmentsetc.). thepossibilityofreconstruction(through inafracturewhichhasdamagedjointbeyond recovery infection. is soft-tissuedamageorlossandtheuseofaplatenailwouldgiveanunacceptablerisk plate. damagethegrowth therefore notbeusedinchildrencasethey should willhavetocrossepiphysealplates.They inevitably through theendsofbones,they Arthroplasty is an important toolinthearmamentariumofatraumasurgeonforgivingquick isanimportant Arthroplasty canbridgeacrossdamagedtissueandsoareespeciallyusefulwhenthere fixators External havetobeintroduced butasthey nailsareanotherformofinternalfixation, Intramedullary y pes offracture 191

27: EXTREMITY TRAUMA TRAUMA ➜ 4D 3C 2B 1A 9A 8B 7C 6D 5E 4F 192

3. T seen inapedestrianhitonthetibiabybumperofcar. outontheotherside.Thisisclassicallyinjury The bonebendsandthewedgefragmentblows blow. fragmentfractureoccurswhenthebonereceivesasharpandheavy A wedgeorbutterfly fracture shouldhaveunitedin4–6 weeks, sothisisdelayedunion. There issomecallusontheX-raybut thefractureisnotyetunited.Inachild,anupperlimb non-union. no signofunion.Thelikelydiagnosisherethenisinfected rapid bonehealing,buttheX-rayshows The factthatthefractureisredandhotsuggeststhereinflammation. Thiscouldbearesultof at thistime. something morethansimplyre-fixingthefracturewillbeneeded.Itis normal toputbonegraftin unless somethingisdone.Itmaybethatthefracturesitehascompletely lostitsbloodsupply,but non-union anysignsofhealingatall,soisprobablyheading towards This fractureisnotshowing isapplied. fixation unless new against time’ofthebonehealingbeforeimplantfailswillbelostand thefracturewillbend thenthe‘race thebloodsupplyhasbeendamagedduringinternalfixation, reason, forexample healingisdelayed forany bonehealing(callusformation)isnotstimulated.Ifprimary secondary mechanics ofthekneeandankle. withthe producing anunsightlybumponthelateralside.Thedeformityislikelytointerfere If afracturehealsinpoorposition,thisiscalledmalunion.Thisfemurhasshiftedintovarus, match asthebonesareoftenovalincross-section. maynot OnX-raythediameterofcortices fragments areusuallydisplacedandshortened. This fracturehasrotated.Normallythisisaccompaniedbyaspiralfracture,inwhichcasethe thanbefore. the fracturewillstillheal,butlimbhealmuchshorter This fractureissodisplacedthatthetwofragmentshaveslidpasteachother. Ifthisisnotreduced, amalunionwillresult. otherwise to needreductionbeforefixation, This fractureisdisplaced.Onefragmenthasmovedinrelationtotheother. Onceagainitislikely will result. amalunion otherwise This fractureisangulated,andsolikelytoneedreductionbeforefixation, a problem. unstable.Thecentralfragmentscansometimeslosetheirbloodsupply,sonon-unionmaybe very andis injury This isasegmentalorcomminutedfracture.Itinmanypieces.high-energy If fracturesarefixedinternally,thefragmentsheldrigidlytogether. Theresultisthat y pes offracture union ➜ 8D 7B 6F 5E 4D 3J 2H 1C

4. Methodsofholdingfractures The obliquefractureofthehumerusisanunstablepatternbutupperlimbcanbe reduction. for obtaininganaccurateandstable offersthebestopportunity withplatesandscrews fixation Incases liketheanklefractureinfootballer,openreductionand to leadprematurearthritis. islikely Fractures which enter ajointposespecialproblemsasanyincongruityinthesurface routinely usedtomanagefemoralfracturesinchildren. ofthebone,sobalanced tractionisstill inthesecases,asitmaydamagegrowth disadvantages hassignificant fast,andinternalfixation fracturesinchildrenhealvery has healed.However, resource-intensiveinhospitalbeds,asthepatientisconfinedto abeduntilthefracture very wayoftreating a fracturebutis safe with thepatient’sweightbytiltingbed.Itisavery Balanced tractionisthemethodofpullingonafracturewithweightswhile balancingthatforce by impalingitwithouttheneedtoopenfracture. unstablefractureeitherby providing abuttressor willhelptocontrolavery plaster ofParis,they percutaneously(theKapandjitechnique).Whenusedwith Kirschner (K)wirescanbeinserted result. complication that,ifthereissignificantswellingwithinaclosedplaster,seriousischaemiamay andcheapmethodofstabilisingafracturebutcarriesthe as asimpleColles’fracture.Itissafe can providerelativestability,whichmaybeadequateforafractureisnottoounstable,such Plaster ofParisprovidesgoodprotectiontoaninjuredlimb,andifitisappliedcarefully,then in thebest-equippedandorganisedoperatingtheatre. even requirescarefulplanning,andinfectionisarisk Optimalfixation supination aretobeachieved. reductionisneedediffullpronationand isidealforforearmfractureswhereaperfect and screws Theanatomicalfixobtainablewithplates (including weight-bearing)immediatelyafterfixation. to obtaincompressionand‘absolutestability’sothatthepatientcanmakefulluseoflimb with maximumstabilityandminimumdamagetosofttissues.Insimplefractures,itispossible offractures toenablefixation The AOsystemprovidesasetofinterchangeableplatesandscrews canbecomeinfected. and thetracksofpinsorwiresusedtofixbonesfixator iscumbersome Thefixator toapplyinternalfixation. totry tissue damagethatitwouldnotbesafe soft- providesanoptimumwayofstabilisingopenfracturewithsuchsevere fixator An external weight-bearing, whileminimisingvascularcompromise. immediate itcanprovideadequate‘relative’stabilitytoallow Combined withlockingscrews nailisvaluableintreatinglongbonefractures,includingpathologicalfractures. The intramedullary oraffectfunction. position, thisdoesnotmattertoomuchinthehumerus,asitshow, the patient’sarmtoprovidetractionusingacollarandcuff.Ifresultisnotunioninperfect limbwhichmusttakeweight).Itisthereforepossibletousetheweightof rested (unlikealower 193

27: EXTREMITY TRAUMA TRAUMA 11G 10A 9I 194 parlous blood supply) or because of actual bone or cartilage loss.Theintracapsularfractureof parlous bloodsupply)orbecauseofactualbonecartilage fractureshavea Reconstruction maybeimpossibleeitherbecauseofbonenecrosis(articular fractures. isthetreatmentofchoiceinpatientswithunreconstructableintra-articular Arthroplasty impairing breathing.Theonlyassistancethatcanbegiventothepatientisanalgesia. which tendstobesplintedbytheneighbouringribsandcannotstabilisedwithout way. isaribfracture, Some fracturessimplycannotbestabilisedinanymeaningful Oneexample together. Thisimprovesstabilityandstimulatesbonehealing. nail intheneckisabletoslideintoaslotsothatfractureendscanbecompressed tothesideoffemur. platescrewed fitted upthefemoralneckwhichconnectstoaheavy The nail iseffectivelyacombinationofanintramedullary early weight-bearing.Thedynamichipscrew toallow fracturehasnoproblemwithbloodsupplybutneedsgreatstrength The pertrochanteric hemi-arthroplasty. the neckoffemurdestroysbloodsupplytofemoralhead,soitneedsreplacingwitha 28 Burns

Multiple choice questions ➜ Burns: general D Haemoglobin combines with carbon 1. Which of the following statements monoxide less easily than with oxygen. regarding burns in children in the UK E Hydrogen cyanide interferes with are true? mitochondrial respiration. A The majority are electrical or chemical. ➜ B The majority are most commonly scalds. Smoke inhalation C Hot water thermostat setting at 60ºC 4. Which of the following statements helps to improve safety in homes. regarding smoke inhalation are true? D Intravenous (IV) resuscitation in children A Inhaled smoke particles can cause a is required for burns greater than 5 per chemical alveolitis and subsequent cent of total body surface area (TBSA) increased gaseous and less than 10 per cent. exchange. E Non-accidental injury is common in B Inhaled smoke particles may be children’s burns. suspected with a specific situation in an enclosed space. 2. Regarding burn injury in adults in the C Early elective intubation is UK, which of the following statements contraindicated. are true? D Symptoms can take 24 h or up to 5 days A Electrical and chemical burns are to develop. common. E The result of carbon monoxide poisoning B Scalds in the home are more common is a metabolic alkalosis best treated by than flame burns. low inspired oxygen. C Alcohol problems are rare in relation to burn injury. ➜ Extent of burns D Effective care requires multidisciplinary 5. Which of the following statements are input. true in relation to burns and TBSA? E Intravenous fluids are required for burns A Epidermal destruction can occur when a of 15 per cent TBSA or more. surface temperature of 70ºC is applied ➜ for 1 s. Respiratory problems in B A child’s head comprises a smaller burns percentage of TBSA than that of an 3. Which of the following statements adult. regarding respiratory problems in C According to the Lund and Browder burns are true? chart, an adult with burns involving both A Burn injury to this function may be sides of one upper limb as well as the lethal. hand has been burned on 15 per cent of B Injury can be due to inhalation of hot or TBSA. poisonous gases. D The ‘rule of nines’ is an accurate guide C Burn injury is more common in the to the size of a burn outside the hospital supraglottic than in the lower airway. environment.

195 TRAUMA ➜ 7. Which of the following statements Whichofthefollowing 7. 8. Which of the following statements Which ofthefollowing 8. statements Whichofthefollowing 6. 196

E D C B A E D C B A Consequences ofburns E D C B A E assessment andmanagementofaburn. parametersinthe inhalation arekey sign inaburnedpatient. clinical A changeinvoice isanimportant limb canresultinischaemia. Circumferential full-thicknessburns of a complicationinaburnedpatient. known Malabsorption fromgutdamageis a in largeburns. Infections withbacteriaandfungiarerare major burns. Cell-mediated immunityisincreasedin true? are regarding burncomplications totheareaofburn. directly proportional shock andthevolumelostasfluidsis TBSA in In burnsaffectingmorethan15 percent space. from intravasculartotheextravascular aburn,wateronlymoves Following vascular permeabilityoccurs. aresult ofaburn,anincreasein As cytokines. Mast cellsdonotreleaseprimary subsequently, neutrophils. causes degranulationofmastcellsand, aresult ofaburn,complement As 2 weekstohealwithoutsurgery. Deep dermalburnstakeamaximumof burns. superficial fillingisnotpresentin Capillary contact burns. Fat burnsaredeeperthanelectrical burns. in superficial includingcement,usuallyresult Alkalis, percentage of The depthofaburntogetherwith regarding burndeptharetrue? guide toassessaburn. is 1percentofTBSAandauseful In smallburnsthepatient’swholehand are true? regarding theconsequencesofburns an adult,fluidlossresultsin TBSA and smoke ➜ ➜ ➜ 10. Which of the following statementsare Whichofthefollowing 10. statements Whichofthefollowing 9. 11. Which of the following statementsare Whichofthefollowing 11.

Burn depth Burn E D C B A Treatment ofburns Fluid management E D C B A D C B A presence ofanyinhalationinjury. are percentageof Major determinantsofburnoutcome a burn. with Other non-burninjuriesmaycoexist burn. associatedwithafresh theinjury slowing min isofnovalueingivinganalgesiaor Cooling ofascaldforminimum10 true? regarding thetreatmentofburnsare be managedasanoutpatient. admitted toaburnsunitandcaneasily A significanthandburnshouldnotbe orareunnecessary. unit donotexist Criteria foracuteadmissiontoaburns true? true? adequacy offluidreplacement. adequacy Urine outputgives amajorclueasto solution suchasDioralite. fluid managementbyrehydratingwith a Hyponatraemia canbeavoidedinoral standard formula. Fluids requiredcanbecalculatedfrom a replacement inburns. are essentialwhengivenasfluid Oral fluidscontainingnosalt burns. partial-thickness diagnostic andmanagementtoolin Tangential shavingmaybeauseful thickness burn. istotallyabsentinafull- Sensation destruction ofthewholedermis. burnsinvolve Deep, partial-thickness full-thickness. certainly Electric contactburnsarealmost agent. nature ofthecausative temperature, timeofapplicationand be assessedfromtheoffending The depthofaburncaninitially TBSA, depth and the ➜ 14. Which of the following statementsare Whichofthefollowing 14. 13. Which of the following statementsare Whichofthefollowing 13. statementsare Whichofthefollowing 12.

Escharotom B A E D C B A E D C B A E contemplated. to beconsidered when escharotomyis Significant bloodlossisnotafeature burns. circumferential superficial Escharotomy isassociatedwithnon- infection risk. thoughthereisincreased required even byacentrallinemaybe perfusion In largeburns,monitoringtissue haematocrit. weight perhisassociatedwithalow of2mL/kgbody Urine outputinexcess recognised bycoolextremities is In resuscitation,hypoperfusion the fluidrateinfusionistoolow 1 mL/kgbodyweightperhindicates In resuscitation,aurineoutputof be altered mean thattherateofinfusionshould 0.5 mL/kgbodyweightperhdoesnot In resuscitation,aurineoutputof true? mL. 4800 upper limbs,includingthehands,is of 70 kgwithaburninvolvingboth requirement inthefirst24hforaman Using theParklandformula,fluid first 24h. calculates thefluidreplacementin and widely usedformulaintheUK The Parklandformulaisthemost thereby helpingtoreduceoedema. which reducesproteinleakoutofcells, Human albuminsolutionisacolloid space. extracellular of intracellularwatershiftingtothe producesanexcess saline Hypertonic isRinger’slactate. crystalloid The simplestandmostcommonlyused true? orcolloids. saline lactate, hypertonic IV fluidreplacementinburns:Ringer’s Three typesoffluidcanbeusedfor true? y ➜ 15. Which of the following statementsare Whichofthefollowing 15. 16. Which of the following statementsare Whichofthefollowing 16.

Dressings E D C B A E D C D C B A true? Superficial burnscanbetreatedbya Superficial medially. incision shouldbeanteriortotheankle limb,forescharotomy,the In thelower theatre. best doneoutsideofamainoperating Escharotomy ofthehandandfingersis incisions. the resultofincorrectescharotomy willnotbe Damage tomajornerves but canbeeffective. burns unusual dressings forsuperficial orboiledpotatopeelare Honey and isnon-permeable. burns Mepitel canbeusedinsuperficial A fenestratedsiliconesheetsuchas burns. burnsbutnotfordeep of superficial dressings forone-stopmanagement Amniotic membranesareideal mixed-depth burns. changed andareusefulindeep such asBiobranedonotneedtobe Biological dressingsandsyntheticones true? borderline-depth burns. make adifferencetotheoutcomein An optimalhealingenvironmentcan Staphylococcus aureus antibiotic butnotformethicillin-resistant used effectivelyasabroadspectrum- Silver sulphadiazine(1percent)canbe Duoderm canbeleftonfor14 days. Hydrocolloid dressingssuchas operations. treat infection,reducescarring,and in ordertoreducepain,or nearly deepdermal,requiredressings Deep dermalburns,orthosethatare available. isreadily intensive nursingsupport the face,whenclimateishotand forsmallburnsof method, particularly Vaseline gauze,orbytheexposure variety ofsimpledressings,suchas (MRSA). 197

28: BURNS TRAUMA ➜ ➜ ➜ 19. Which of the following statements Whichofthefollowing 19. statementsare Whichofthefollowing 18. 17. statementsare Whichofthefollowing 198

Allied therapies Infection control management Burn B A E D C B A E D C B A E 2–3 weeks. Physiotherapy canbebestdoneafter physiotherapy management. is dependentonintensivenursingand psychological careoftheburnpatient Success orfailureofbothphysicaland an infection. tipsareapossiblesourceof Catheter with amicrobiologist. based onculturesanddiscussion Antibiotics givenshouldbeideally the patient’sflora. are ofnouseinbuildingapicture Swabs takenfromtheburnandsputum infection. decreasing clinicalstatusaresignsof A riseorfallinwhitecellcountanda prevention. handwashing andcross-contamination Infection controlrequiresattentionto occurs between20and25days. The greatestnitrogenlossesinburns required. cent TBSA In adultswithburnscovering15 per effective instoppingthecatabolicdrive. healing reducepainandarealso Removing theburntissueandachieving are best. injectionsofopiates intramuscular (IM) For largeburnsover management. ofburn Analgesia isavitalpart sulphadiazine cream. not treatableby1percentsilver Pseudomonas aeruginosa patients aretrue? regarding alliedtherapy inburn true? true? or more, extra feedingis or more,extra 10 percent TBSA, is ➜ ➜ 21. Which of the following statements Whichofthefollowing 21. statementsare Whichofthefollowing 20.

Surgical management depth burn Treatment ofblistersand A E D C B A E D C E D C B true? burns aretrue? managementof regarding surgical them intactorremoving– The managementofblisters–leaving burns isnotindicated. ofhandsthathavesustained Elevation relatives oftheburnedpatient. Psychological helpmayberequiredfor as aresultofburns. Post traumaticstressdisordercanoccur burns. oflarger excision stable coverfollowing homografts, mayprovidetemporary Synthetic dermis,including Integraor affect bloodclotting. 36ºCmay A coretemperaturebelow solution ofadrenaline. graft ortopicalsubcutaneousdiluted a tourniquetorbyapplicationofskin Blood lossmaybereducedbyuseof major burns. in Blood lossisnotafeatureofsurgery major burn. and essentialinthemanagementofa The anaesthetistisofgreatassistance possible. andgraftingif need surgicalexcision but thesmallestfullthicknessburns shaving andsplit-skingraftingwhileall Deep dermalburnsneedtangential be reassessedafter2weeks. Any burnofindeterminatedepthshould skin grafting avoidingdebridementand it isworth If aburnhasnothealedwithin3weeks, contraindicated. solutionis with chlorhexidine Initial cleaningofaburnwound remains debatable. ➜ ➜ ➜ 24. Which of the following statements Whichofthefollowing 24. statementsare Whichofthefollowing 23. statementsare Whichofthefollowing 22.

escharotom Scar management Surgical management Role ofph A E D C B A E D C B A worn foramonth. treated bytheuseofpressuregarments ofaburnscarcanbe Hypertrophy available inthetreatedburnscar. when goodvascularisedtissueis inburn scarmanagement unnecessary tissue, asinafreeflap,aregenerally Full-thickness graftsorvascularised scarring causingrestrictedmovement. reconstruction ofbroadareasburn A Z-plastyisausefultechniquefor alopecia causedbyaburn. isusefulintreating Tissue expansion skin grafts. Contractures arebesttreatedwithsplit- and axilla. isindicatedinthehands Early surgery are burned. Early caremustbetakenwheneyelids function. respiratory burn oftheuppertrunkshouldhelp Escharotomy ofthecircumferential about 2weeks. any affectedjointsshouldbeginafter physiotherapists underdirectvisionof movementby Supervised after skingrafting. tosplintthehand It isnotnecessary contracture of movementandreducingjoint inmaintainingrange important Physiotherapy andsplintageare are true? managementofburns regarding scar true? true? y siotherap y ande y elid care y , ➜ ➜ 26. Which of the following statementsare Whichofthefollowing 26. statementsare Whichofthefollowing 25.

burns Chemical andradiation Electrical burns E D C B A E D C B E D C B A true? true? mothers. a topicalagentinpregnantornursing Flamazine creamshouldnotbeusedas quality. thickness burnscarcanimprove ahealedfull- Use ofIntegratoresurface scars isnotimportant. Pharmacological treatmentofitchyburn burn scarhypertrophy. patches maybeusefulinsmallareasof Intralesional steroidinjectionorsilicon need excision andsplit-skingraftrepair need excision Local radiationburnscausingulceration associated withhypercalcaemia. Hydrofluoric acidburnscanbe with acids. is usuallylessthan Damage fromalkalis treated bywaterlavage. Elemental sodiumburnsshouldnotbe measure forphosphorusburns. Copious waterlavageisthebestfirst-aid electrical burns. iscommoninlarge alkalosis Severe burns. of low-tension Myoglobinuria isaseriouscomplication burns. associated withhigh-tensionelectrical subcutaneous tissuesandmuscleare Large amountsofdamageto injuries. likely inlow-tension muscledamageis Underlying heart the home. most likelytobefoundinaccidents is electricalburninjury Low-tension 199

28: BURNS TRAUMA 6. A ➜ ➜ ➜ ➜ 5.A,E 4.B,D 3.A,B,E 2.A,D,E 1. B,C Answers: Multiplechoicequestions 200

take longer. Sensation and capillary filling are features of a superficial burn. take longer. fillingarefeaturesofasuperficial andcapillary Sensation deepdermalburns will burns. Anyburnthat healsspontaneouslywithin3weeks issuperficial; such ascementusuallyproducedeepdermalorfull-thickness injuries. Theburnscausedbyalkalis inelectriccontact fat usuallycausesdeepdermalburns, whilefull-thicknessburnsaremorecertain agent.Thus the stressedsituation.Thelikelydepth ofanyburncanbederivedfromthecausative percentage of factorsintheassessmentandmanagement ofaburnaresmokeinhalation,depthand The key hand–palm as1per Extent ofburns Smoke inhalation Respirator general Burns: hand (bothsides)as8percent burnunitsandmeasures awholearmand iswidelyusedinUK chart The LundandBrowder largerthaninanadult. It shouldberememberedthatthesizeofachild’sheadisproportionally ‘rule ofnines’offersaroughguidetoareabutisprobablyusefuloutside thehospitalenvironment. The at thattemperature.Burnareacanbemeasuredbyaseriesofformulae plottedonachart. oftemperature andtimeofapplication The firstindicationofburndepthcomesfromthehistory monoxide poisoningcausesametabolicacidosisandistreatedbyinhalationofpureoxygen. Carbon Symptoms andcantakeupto5daysdevelop. maynotbeimmediatelyevident andisdefinitelynotcontraindicated. Earlyelectiveintubationisimportant with gaseousexchange. cancauseachemicalirritationoralveolitis.Thisresultsininterference Inhaled smokeparticles withmitochondrialrespiration. house fires,whichinterferes isametabolictoxinproducedin percentislikelytobelethal.Hydrogencyanide dangerous; 60 can bedonetoconfirmthediagnosis.Aconcentrationofcarbonmonoxideabove10 percentis Bloodgasmeasurement ofoxygen. combining withhaemoglobinandthusblocksthetransport for times greater thanoxygen monoxidehasanaffinity240 in thesupraglotticairway. Carbon than airway ismorecommoninthelower gases orpoisonousvapoursareinhaled.Burninjury the faceorneckareburned,whenfirecausingburnisinanenclosedspace,hot andlungswithlife-threateningconsequences.Thiscanoccurwhen Burns candamagetheairway care. IVfluidsinadultsisrequiredwhen15 those whohavesufferedburninjury. Amultidisciplinedapproachmustbeavailableforeffective burns. Thepresenceofalcohol,drugabuse,epilepsyandmentaldisorderisnotuncommonin Most electricalandchemicalburnsoccurinadults,whilescaldsarelesscommonthanflame fluids inchildrenisrequiredwhentheTBSA10 percentormore. havereducedtheincidenceofburns.IV smoke alarmsandhotwaterthermostatskeptat60ºC, Legislation, healthpromotionandappliancedesign,togetherwitheducationofpatientsregarding The majorityofburnsinchildrenarescaldswithkettles,pans,hotdrinksandbathwater. TBSA y problems inburns affected. Iforalfluidsaretobeused, saltmustbeaddedtocounterlossin cent of TBSA. of TBSA.Insmall percentormore burns itmaybeusefultousethepatient’swhole of TBSA is affected. ➜ ➜ ➜ ➜ 13. C,D,E 12. A,C,D 11. B,C, D,E 10. A,B,D,E 9.B,C 8.C,D,E 7. A,C,E

Fluid management depth Burn Treatment ofburns Consequences ofburns As a result of a burn, complement is produced which degranulates mast cells and leucocytes. Mast aresultofburn,complementisproducedwhichdegranulatesmastcellsandleucocytes. As clinical status.For adults,aurineoutput of0.5–1.0 mL/kgbody weightperhisnormal.Ifit offluidreplacement can beassessedfrommeasuringurine output,haematocritand Adequacy mL. fluids requiredwouldbe4480 weight (kg) replaced inthefirst24h,withhalf of thisvolume space.TheParklandformulacalculatesthefluidtobe of intracellularwatertotheextracellular also reducesoedemabyproducinghyperosmolarityandhypernatraemia –thisreducestheshift saline proteinleakfromthetissuecells. Hypertonic colloid thatcanreduceoedemabypreventing fluid,whilealbuminisa Ringer’s lactateisthemostcommonlyused,andcheap,burncrystalloid orcolloid. saline requirements whichmaybegivenasRinger’slactate,hypertonic gives agoodindicationoffluidrequirements.Various formulae areavailabletocalculatefluid loss.Dioraliteisusefulinthetreatmentofhyponatraemia. Theoutputofurine take accountofsalt andwaterinoralfluidsgivenforresuscitationinsteadofalone,to togivesalt It isimportant bleeding afteroneortwoshavesconfirmsthistypeofinjury. thicknessburns–thepresenceofpunctate management anddiagnosticoperationinpartial endingshavebeendestroyed.Tangentialall nerve shavingusingaskingraftingknifeisuseful isabsentinfullthicknessburnsas burnssomedermaltissueremains.Sensation partial-thickness agent.Electricalcontactusuallyresultsinfull-thicknessburns.Indeep, nature ofthecausative thetemperature,timeofapplication,and Most burnscanbeassessedfromthehistory, significant handburnshouldalwaysbeadmittedeventhough andadviceonthisanyburnisreadilyavailable.A Criteria foradmissiontoeachburnsunitexist withburnsandshouldnotbeoverlooked. burn.Non-burninjuriesmaycoexist but notforevery –thismaysometimesbedonebyusingcoldwater the localtissueswillhelptoreduceburninjury Cooling ofafreshburnfor10 thetemperatureofagentcausingburnand minbylowering systemincludestridororchangeinvoice. respiratory impairment.Warningsignsof burnstothe full-thickness burnsofthechestmaycauserespiratory reason,circumferential the limbsasaresultofswellingandtourniqueteffect.For thesame malabsorption. Ischaemicchangesareadefiniteriskforfull-thicknesscircumferentialburnsof fungi.Damageinthegutliningtissuesmaycause lookout forinfectionswithbacteriaandeven In theburnedpatientthereisareductionincellularimmunity,andoneshouldbeon is 15 percentormore. resulting shockisrelatedtothisfluidlossand duetoanincreaseinvascularpermeability. compartment The from theintra-toextravascular Thereisamovementofwaterandsalt cytokines. andproduceprimary cells actonleucocytes ϫ 4 ϭ volume(mL).For a70 kgpatient in anadultbecomes to begiveninthe with twotimes8per the percentageofTBSAmaybesmall. significant when first 8h: cent TBSA TBSA percentage TBSA burn, the affected 201 ϫ

28: BURNS TRAUMA ➜ ➜ ➜ ➜ 18. A,B,D,E 17. A,C,D 16. B,D 15. A,B,E 14. Noneoftheanswersiscorrect 202

provided byproteins.Thegreatestnitrogen lossesoccurbetween5and10 days. Infection control management Burn Dressings Escharotom infection. thoughtheremaybeanincreasedriskofintroducing of acentrallineshouldbeconsideredeven shouldbetakeninpatientswithacuteorchroniccardiacproblemsandtheuse be reduced.Care of2mL/kgperhmeanstherateinfusionshould overresuscitated andaurineoutputinexcess thatpatientsarenot Itisimportant may besignsofrestlessness,tachycardiaandcoolextremities. thanthisandthehaematocritishighernormal,morefluidshouldbegiven–there lower catabolism arewarnings ofinfection.Swabstakenfrom theburnwoundandsputumcan helpin andanincreasein measures.A riseinwhitebloodcellcount,thrombocytosis prevention Control ofinfectionbeginswithpolicies onhandwashingandothercross-contamination nasogastric tubeinburnscovering15 feedingmustbegivenand this shouldbeby this.Extra the mostsignificantfactorsinreversing oftheburnandstable woundcoverageare burn woundremainsunhealed,andrapidexcision ketamine ormidazolammayrequireananaesthetist.Therearecatabolic changesaslongthe uncontrolled anddangerous.Administrationofthisotheragentssuch asgeneralanaesthesia, asabsorptionis Intravenousopiatesareindicatedforlargerburns–not IM aresuperficial. if they ofburnmanagement.Oralparacetamolisuseful insmallburns,especially Analgesia isavitalpart Pseudomonas fenestrated siliconesheetthatcanalsobeused.Silversulphadiazine1percentiseffectiveagainst orbananahasalsobeenusedaroundtheworld.Mepitelisapermeable burns. Honey butnotdeepdermalor Biobrane andamnioncanbeusedfordressingssuperficial when thedepthismixedoruncertain. optimal healingenvironmentwillprovidethebestchanceofquicklyandwell,especially torememberthatthe Itisimportant sulphadiazine (1percent)iseffectiveincasesofMRSA. 3–5days.Silver colloid dressing,isusefulinmixeddepthburnsbutshouldbechangedevery helpful ifitisheavilycontaminated.Vaseline gauzeisacommonlyuseddressing.Duoderm, anaesthetic isprudentandapplicationofsilversulphadiazinecreamfor2–3daysmaybe are contaminatedorbecomeinfected.Cleaningthewoundundergeneral they Mosthealirrespectiveofthedressingunless burnscanbedressedorleftexposed. Superficial vein. saphenous tomaketheincisionposteriorankleavoiddamage limb itisalsoimportant lower Inthe anescharotomytoavoiddamagemajorstructures,includingnerves. in performing shouldbetaken significant bloodloss.Adequateshouldbeavailablefortransfusion.Care limbischaemiaandcanbeassociatedwitha should bedoneearlytoaidrespirationandprevent A tourniqueteffectoccursduetotheincreasingpressureandswellingthatresult.Thisprocedure Escharotomy isindicatedforcircumferentialburnsoflimbsortrunk–deepdermalgreater. infection. y per cent TBSA or more; 20percentofkilocaloriesshouldbe ➜ ➜ ➜ ➜ ➜ ➜ 24. B,D,E 23. A,C, E 22. A,D,E 21. A,C,D,E 20. A,E 19. A,C,D

Scar management Surgical management Role ofph Surgical management Treatment depth ofblistersandburn Allied therapies tipscanbeasourceofinfection. decision onantibiotics.Catheter inthe establishing theflorapicture.Theroleofabacteriologistandadvicearegreatimportance scratching maycause distressandincreasethechance ofintroducinginfectionanddelayed totreatanyitchyareaswithappropriate pharmacologicalagents, as patient areas. Itisimportant injection ofsteroidorapplication asiliconpatchorsheetmaybeusefulforsmallerhypertrophic and wornforatleast6months wouldbeineffectiveifonlyusedfor1month.Intralesional butrequiretobefitted Pressure garmentsmaybeeffective inreducingburnscarhypertrophy absent inthelatter. are usefulinsituationswherebloodsupplyisgood;andfreeflaps useful whereitispooror maybeusefulintreatingalopeciaofthescalpcausedbyburns. Full-thickness grafts expansion notbroad,scarcontractures.Tissue best treatedbygraftswithZ-plastiesbeingusedfornarrow, andgraftingareindicatedinburnsoftheaxillahands. Contracturesareusually Early excision areburned. ifeyelids ifnecessary, toprovideearlycare,includingsurgery important dermal circumferentialburnsoftheuppertrunk,escharotomywillhelp toimproverespiration.Itis canbeaided.Infull-thicknessburnsordeep shouldoccuronday1sothatearlyrecovery physiotherapyforanyaffected splint isusedinhandstreatedbysplit-skingrafts.Supervised jointcontractures.Ahand Physiotherapy andsplintagebothhelpconsiderablyinpreventing burn thathasbeenexcised. wayofdressingalarge In largeburnstheuseofIntegraorhomograftcanbeausefultemporary 36ºC;thustheoperatingroommustbekeptwarm. the coretemperatureofpatientfallsbelow 000–willhelptocontrolorreducebleeding.Blood-clottingirregularitiescanoccurwhen 1:500 available. Theapplicationoflimbtourniquet,topicalorsubcutaneousadrenalinesolution–diluted can beafeatureoflargeburndebridement,facilitiesforbloodtransfusionandmust bloodloss An anaesthetistmustbeavailablefordressingordebridementofamajorburn.As Z-plasties maybeindicated. keratitisarises–full-thicknessgrafts,transpositionflapsor Eyelids mustbetreatedbeforeexposure outcome. reinforced daily. andsplintagewillimprovethe Allburnsofthehandscauseswellingandelevation earlyandinthecaseofhandburnsthisshouldbeonday1 Physiotherapy shouldbestarted posttraumaticstress. mayhelptherelativesandprevent Psychologicalsupport importance. Intensive nursing,physiotherapy,andpsychologicalmanagementofaburnedpatientare y siotherap y , escharotom y ande y elid care 203

28: BURNS TRAUMA ➜ ➜ 26. B 25. A,C 204

Chemical andradiationburns Electrical burns useful, shouldnotbeusedinnursingorpregnantwomen. healing. TheuseofIntegramayimprovethequalityburnscars.Flamazinecream,although will nottakeonavasculartissuesovascularisedflapsorfreeflapreconstructionsarerequired. burnscausetissuedamagethatisassociatedwithbloodvesselnecrosis–split-skingrafts Radiation burns areassociatedwithhypocalcaemia–hencetreatmentis10 percentcalciumgluconate. burns aregenerallydeeperthanacidandactforalongertime.Hydrofluoric Alkali With phosphorusandelementalsodiumburns,washingwithcopiouswateriscontraindicated. treatment withbolusesofbicarbonate. resulting inmyoglobinuriaandrenaldamage.Acidosisisfoundlargeburnsmayrequire high-tension burnswhichareassociatedwithlargeamountsofsubcutaneousandmuscledamage, burnsbutcanbein damageisnotafeatureoflow-tension tension’.Heart appliances are‘low electricalburnsareassociatedwithinthehomeduetofactthat Low-tension 29 Plastic and reconstructive surgery

Multiple choice questions ➜ Plastic surgery – general C A graft has a separate blood supply 1. Which of the following statements are which enables it to survive on an true? avascular wound. A The word ‘plastic’ is derived from the D A full-thickness graft has the whole French word ‘plastique’. dermis attached with fat trimmed off. B Plastic surgery is a new speciality. E A composite graft is a full-thickness graft C Plastic surgery involves techniques to to which other structures such as hair restore form and function in tissues may be added by suturing on. damaged by injury, cancer or congenital 4. Which of the following statements abnormalities. regarding grafts are true? D Sir Harold Gillies did his reconstructive A Imbibition is not a process associated work after the Second World War. with survival of split-skin grafts in the E Some of the work of Gillies was first 48 h. portrayed by Harry Monks. B Gentle handling and the best ➜ Skin characteristics postoperative care help to ensure the successful take of a full-thickness graft. 2. Which of the following statements are C Grafts will take on exposed tendons and true? cortical bone. A The surface of the skin is an important D Contraction occurs in all grafts used in biological layer for homeostasis. tissue repair but is dependent on amount B Epidermis regenerates from deeper of dermis taken with the graft. follicular elements. E The more dermis in the graft, the more is C Epidermal keratinocytes cannot be the contraction. cultured and thus are of no value in wound management. 5. Which of the following statements D The depth of skin varies in different parts regarding grafts are true? of the body. A Split-skin grafts are sometimes known as E In the absence of skin, a wound heals Thiersch grafts. by secondary intention with fibrosis and B Full-thickness grafts are useful in small contracture. areas such as fingers, eyelids, or on the face. ➜ Skin grafts C John Wolfe, an Aberdeen orthopaedic 3. Which of the following statements surgeon, described a composite graft in regarding blood supply and grafts are 1902. true? D Split-skin grafts produce a superior A Skin blood supply comes from muscle cosmetic result compared with full- and fascial perforating vessels. thickness grafts. B Direct cutaneous vessels can also E Scars placed in ‘the lines of election’ or contribute to the blood supply of the lines of minimal tension produce the skin. best cosmetic results.

205 6. Which of the following statements B Inclusion of fascial tissue in a skin flap regarding split-skin grafts are true? does not make for greater safety. A Split-skin grafts can be cut at varying C The design of a transposition flap TRAUMA thicknesses using handheld or electrical demands knowledge of a pivot point. dermatomes. D The transposition flap length equals the B The best donor site for getting a split-skin length of the defect to be covered – graft in children or females is the thigh. assuming the breadth:length ratio is no C Other useful donor sites for split-skin greater than permitted. grafts are the buttocks and the scalp. E Following the use of a transposition D The size and number of bleeding points flap, skin grafting of the donor defect is in the donor tissue for split-skin grafts a likely necessity though direct closure do not identify the thickness of the graft. may be just possible. E The take of a split-skin graft is affected 10. Which of the following statements by a number of factors, including the regarding flaps are true? presence of group A beta-haemolytic A Z-plasties are triangular transposition Streptococcus. flaps. B Z-plasties are able to lengthen very 7. Which of the following statements broad contracture scars. regarding mesh grafts are true? C A bilobed flap is useful to close a small A Mesh grafting enables expansion of a convex defect in the nose tip. split-skin graft to be done. D Rhomboid flaps are useful in the repair B Mesh grafting prevents release of of defects of the fingertips. exudates from under a split-skin graft. E Rotation flaps are used for buttock or C The possible donor site for a full- scalp defect repairs. thickness graft is from behind the ear. D Condition for take of a full-thickness graft 11. Which of the following statements are not as critical as for a split-skin graft. regarding flaps are true? E Large full-thickness grafts when used in A Burrow’s triangles are not of significance the face and over good facial muscle do in the use of bipedicle flaps. not produce a satisfactory cosmetic B Multiple Y to V releases are one of the result. most effective means of managing moderate isolated burn scars over ➜ Flaps flexion creases. 8. Which of the following statements C V to Y flaps are ineffective in the regarding flaps are true? management of fingertip injuries. A Flaps introduce blood supply into an area D The key to successful random flap use for reconstruction. is to pull available local spare lax skin B Classification of flaps can be made on into the defect so that the closed scar the basis of their blood supply. lies in a good line of election. C In a random pattern flap, the maximum E A disadvantage of using local flaps may length:breadth ratio that is safe is 2:1. be a poor cosmetic result. D Delay is a technique that can further 12. Which of the following statements lengthen a random flap. regarding myocutaneous and E Axial flaps, based on known blood fasciocutaneous flaps are true? vessels, enable longer flaps to be moved A Myocutaneous and fasciocutaneous over longer distances. flaps are unreliable in plastic surgery 9. Which of the following statements repairs. regarding flaps are true? B The above flaps require complex A Islanding of a random flap can safely and equipment to be available. usefully be done.

206 15. Which of the following statements Whichofthefollowing 15. statements Whichofthefollowing 14. statements Whichofthefollowing 13. B A E D C B A E D C B A E D C repaired usinga‘bucket handle’flap. may be eyelid A defectofthelower the useofZ-plasties. trianglesareassociatedwith Burrow’s regarding flapsaretrue? transfers. in themanagementofmajortissue catecholamine outputisgoodadvice Well-controlled analgesiatoreduce input problem. inflaps whichhaveanarterial last resort Medicinal leechescanbeusefulasa cause offlapfailure. blood supplytoflaptissuewillbea ofanatomyandthe Poor knowledge including freeflaps. typeofflap, cause flapfailureinevery Too muchtensionofflapinsetcan venous supplyiscompromised. A paleandcoldflapisasignthatthe true? withflapsare regarding complications surgeon isexperienced. microsurgical procedureunlessthe associatedwitha the surgery It usuallytakesmoretimetoperform infree-flapsurgery.disadvantage Major donorsitemorbidityisapossible of freeflaps. fortheuse reconstruction isnecessary Debridement ofthearea free flaps. areessentialfortheuseof microsurgery andequipmentin Surgical expertise tissue. reconstructing majorcompositelossof Free flapsarethebestwayof regarding freeflapsaretrue? vessels. small perforating flaps asskinislanddependson oftheskinwhenusedinthese Survival required. These flapscanbeusedwithoutskinif used. of useisessentialwhentheseflapsare ofbloodsupplyinthearea Knowledge ➜ ➜ ➜ 17. statementsare Whichofthefollowing statementsare Whichofthefollowing 16. 18. Which of the following statementsare Which ofthefollowing 18.

T Reconstructive options Major tissuerepair options B A E D C B A E D C A E D C y pes offlaps true? true? true? 75 percentinareaandinvolvingskull asmuch A largescalpdefect–say compromise toexcision. maybe flaps usedintumoursurgery oftheuselocal A disadvantage pattern flap. ofanaxial A bilobedflapisanexample glabellar area. using atranspositionflapfromthe An innercanthaldefectcanberepaired meticulous planning. defects requiresa teamapproachand Success inrepairofmajortissue flap repairforoesophagealdefects. The jejunumisausefulmeansoffree reconstruction. free flapsorpedicledinbreast rectus abdominisflapscanbeusedas The latissimusdorsiandtransverse musculocutaneous flap. The radialforearmflapisa reconstruction asafreeflap. The fibulacanbeusedinjaw grafts. transferred asfree,non-vascularised andtendonscannotbe Nerves identified usingaDopplerapparatus. vesselscanbe Position ofperforating fasciocutaneous flap. skin canbeaccomplishedbyusinga Repair ofanankledefectinvolvingthe skin graft. Achilles tendonscarproblemisasplit- The definitiverepaircoveringforan using anislandedpedicledinstepflap. A defectintheheelmayberepaired rhomboid flap. –canbebestrepairedusinga excision 207

29: PLASTIC AND RECONSTRUCTIVE SURGERY TRAUMA ➜ ➜ ➜ 5.A,B,E 4.B,D 3.A,B,D 2.A,B,D,E 1. C Answers: Multiplechoicequestions 208

Skin grafts Skin characteristics Plastic surger reconstruct an eyelid. Full-thickness grafts are useful in the repair of face and eyelids andproduce Full-thickness grafts areusefulintherepairoffaceand eyelids reconstruct aneyelid. Full-thickness graftsweredescribed byJohnWolfe, ophthalmic surgeonin1875, aGlasgow to inLeipzig,Germany,whodescribedfreeskingrafting1874.Thiersch wasaprofessorofsurgery and isthusgreatestinsplit-skingrafts andleastinfull-thicknessgrafts. Graft contractiondependsontheamountofdermisingraft granulations orvascularsupport. bonebecausethesedonotproduce thickness graft.Graftsdonottakeon baretendonorcortical tocreatethebestconditions fortakeofafull- its recipientsite.Gentlehandlingisimportant Imbibition isthemeanswherebyasplit-skingraftnourishedduring first48hoflifein rim. deficient nasal ofthecompletegraftandnotsecondarilysuturedon,e.g.hairtransplants orreconstructionof part tissues deliberatelytakenas orotheradnexal thickness graftwhichcontainshairfollicles,cartilage epidermis andthewholeofdermisfromwhichfathasbeenremoved; thelatterisafull- for successfultakeofeitherafull-thicknessgraftorcompositegraft.The formerconsistsof it.Asimilar situationexists that ingressofcapillariesintothegraftcanoccurandrevascularise has beendetachedasa‘shaving’–itrequiresthebedorreceivingarea tobevascularisedso ofepidermis anddermiswhich and, wherepresent,fromunderlyingmuscle.Askingraftispart fromunderlyingfascia Skin bloodsupplycomesfromdirectcutaneousvesselsandperforators intentionfromthesides. base, healingoccursbysecondary canbeculturedandusedinwoundcare.Ifwoundshaveno dermis intheir Keratinocytes ofthebodyandregeneratesfromfollicularelementsdermis. depth variesindifferentparts homeostatictissue.Its andtemperaturecontrol–itisanimportant forsensation Skin isimportant Tonks.was illustratedinpaintingsbyHenry was apioneeringplasticsurgeonwhobecamefamousaftertheFirstWorld War. Muchofhiswork SirHaroldGillies oftumours,orthroughaccidentsbirth. defects causedbytrauma,aftersurgery dealswith therepairof Sushruta describedtheforeheadflaptoreconstructnoses.Plasticsurgery where hieroglyphicsandinIndiathe6thcentury speciality withdescriptionsinancientEgypt The word‘plastic’isderivedfromtheGreek C B tissue induration. recipient site,andnoinfectionorlocal and venousconnectingvesselsinthe availability ofasuitableflap,goodartery Free flapsuccessisdependentonthe a goodoption. canstillbe equipment, freeflapsurgery In theabsenceoftrainedstaffor y –general plassein , meaningtoshapeormould.Itisanold E D free flapsis10 h. transferof The ischaemictimeforsafe loupes. increased magnificationprovidedby isbestdoneusingthe Microsurgery 7. A,C 6.A,C,E ➜ 11. A,B,D 10. A,C,E 9.C,E 8.A,B,D,E

Flaps not worsen,theresult. graft andthepresenceofactivemuscleunderneathafull-thicknessfacewillimprove, to bevascularised.Iftheconditionsaregood,cosmeticresultssuperiorforafull-thickness of split-skingraftsiseasierthanforfull-thicknessbecausetherelesstissuedepthrequiring full-thickness grafts–othersitesincludethesupraclavicularneckorhairlessgroinskin.Thetake Theretroauriculartissueprovidesausefuldonorsitefor graft alsoenableescapeofexudates. asmallergraft.Theholesinthe Mesh graftingofsplit-skingraftsisausefultechniquetoexpand will nottakeonavasculartissue. streptococci,shearingforcesandagoodbloodsupplyintherecipient area.Grafts beta-haemolytic graft. Grafttakedependsonanumberoffactors,includingpresenceinfection,notablygroupA the bleedingnatureofdonorsite–larger-spacedpunctatepointsindicateathicker are thethighsandshavedscalp.Itispossibletodeterminedepthofsplitskintakenfrom where anyproblemsinhealingandtheriskofpoorscarscanbehidden.Otherusefuldonorsites electricity. Thebestdonorsitefortakingasplit-skingraftinchildorfemalesisthebuttock, by dermatomeoronepowered Split-skin graftscanbecutbyusingahandheld,hand-powered of minimaltensiontogetthebestcosmeticresultbutdonotalwayscorrespondLanger’slines. a bettercosmeticresultthansplit-skingrafts.Incisionsandresultingscarsarebestplacedinlines fingertip defects. Agoodcosmeticresultinrandom flapscanbeobtainedwhenattention indesign fingertip creases,andVto Yflapsareusefulinrepairof flaps areusefulin treatingburnscarsoverflexure inthedesignoruseofbipedicleflaps.MultipleYtoV trianglesdonotplayanypart Burrow’s scalp defectsorinthebuttocks. butcanbeinthetemple orback.Rotationflapsaremostlyusedinmoderatelysized fingertips good alternativetoaretroauricular full-thickness graft.Therhomboidflapisnotaforusein contracture bands.For tip-of-the-nosedefectsofabout 1cmindiameter,abilobedflapis notbroad, narrow, Z-plasties aretriangulartranspositionflapswhichusefulinlengthening laxareaofskin. be possibleifthisisinavery thoughinsome casesadirectclosuremay Usually thedonordefectwillhavetobegraftedinpart, ofthedefect. ofthistypeflapwillbelongerthanthelength defect tobecovered.Thelength awayfromthe the flaptobeused.Thispointissituatedatbaseofon sidefurthest of totakenoteofthepivotpointasthis determinesthelength transposition flap,itisimportant areincluded. Inthedesignofa with askinflapincreasesthebloodsupplyifperforators the caseforanaxialpatternflap,whichcanbeislanded.Inclusionofunderlying muscleorfascia precisely; thisisnot A randomflapcannotbeislandedbecausethebloodsupplyisnotknown ofanaxialpatternflapsuchasthegroinflap. an example alongerflapcanalsobeusedatgreaterdistance.Thisis vascular supplyisconfidentlyknown, areusedinthereconstruction.Whenmain weeks beforethewholeflapanddelayedportion is‘delayed’ portion ortemporarilyraisedandreplacedforafew more canbedoneiftheextra or poor,vascularity. ratiois1:1.5; breadth:length For arandomflap,themaximumsafe extending bloodsupplytotherecipientarea.Flapscanthusbeusedreconstructareaswithno, their own Flaps canbeclassifiedaccordingtothetypesofbloodsupplyand,incontrastgrafts,introduce 209

29: PLASTIC AND RECONSTRUCTIVE SURGERY TRAUMA 14. B,C,E 13. A,B,C,D 12. C,D,E 15. B,C,E ➜ ➜ 17. B,D,E 16. B,D, E 210

T Reconstructive options created andnotviceversa. always bethefirstpriority,withrepairofresultingdefectbyaflapdesignedtofit ofanytumourshould to fitthedesignofalocalflapshouldnotbedonebutitisrisk.Excision itisamodifiedrotationortranspositionflap.Compromisingtumourexcision vessels –ifanything, of theinnercanthus.Thebilobedflapisnotanaxialpatternasitbasedonknown Aglabellartranspositionflapcanbeusedtorepairadefect eyelid. reconstruction ofthelower triangles.Thebuckethandleflapisusedin Z-plasties donotneedtomakeuseofBurrow’s tissue transferssothatcatecholamineproductionisreduced. thatappropriateanalgesiaisgiveninmajor be usedonceonanindividualpatient.Itisimportant problem.Eachleechcanonly has beencompromisedbutareofnovalueifthereisanarterial supply totheflapbeingused.Medicinalleechesareusefulinsituationswherevenousoutput Tension boththeanatomyandblood canaffectalltypesofflapadversely,asfailuretoknow inputproblemwhileabluedistendedflaphasvenousproblem. A pale,coldflaphasarterial oftheoperatorandassistants. types ofreconstructionbutdependsontheexperience not aproblem.Theoperativetimeformicrosurgicalproceduresisusuallylongerthanother ofateammanagement is success butmajordonorsitemorbiditywhenchosencarefullyaspart debridementoftheareaforreconstructionisessential microsurgical instruments.Careful and Free flapreconstructionisthebestmethodforcompositetissuelossbutrequiresexpertise overlying skin. especiallyifislanded,butthefasciaandmusclecanbeusedasflapswithout perforators, dependsonthe ofanatomyandbloodsupplyfortheseflaps.Skinsurvival good knowledge tohavea itisimportant equipment andhighlytrainedsurgeonsarenotrequired.However, reliablebloodsupply,andcomplex Myocutaneous andfasciocutaneousflapshavevery better. graft becauseitisthicker,hasabetterbloodsupplyandretainscolourtexture is madetothelinesofelectioninlocatedarea.Thecosmeticresultaflapbetterthan oesophageal defects, freejejunumgraftsofferagood wayforreconstruction. be usedasfreeflaps orpedicledmusclemusculocutaneous flapsinbreastreconstruction. For vessels.Latissimus dorsiortransverserectusabdominisflapscan is designedaroundwell-known ofanaxialpatternflapasit bone toreconstructthejawTheradial forearmflapisagoodexample longus tendonsareusefulsourcesof donortissue.Thefibulaisausefulsourceoffreeflapfor and,whenavailable,thepalmaris andtendonscanbeusedas freegrafts–thesuralnerve Nerve vessels ontheskinsurface. fasciocutaneous orfreeflaps.Dopplerapparatusisaneasyandgoodway toidentifyperforating flap repairisbetter. Wounds third ofthelegcanberepairedusing oftheankleandlower repaired byusingsplit-skingraftsbecauseofdurability,vascularityand mobilityproblems–a by employingapedicledinstepflap.Achillestendonwoundsarenotpermanently andproperly require reconstructionwithafreeflap.Heelulcersaredifficultduetotheir sitebutcanbetreated A largescalpdefectwithbonetissueremovedcannotberepairedbya rhomboidflapbutwould y pes offlaps ➜ 18. A,C

Major tissuerepair options skin or/andfasciaflaps. formuscle-containingfreeflaps–longertimesofupto6harepermissibleonlyin period issafe presence orabsenceofmuscletissueinthefreeflap–itislesscaseformer. A1–2h forsuccessfulrepair.reconstruction arealsoimportant Theischaemictimeisdependentonthe recipient area,thelackoftissueinduration,tensionandinfectioninarea best resultsareobtainedusingproperstaffandapparatus.Goodvesselsinbothdonorflap andthe this istobedoneusingamicrovascularprocedure,theuseofloupesnotsatisfactory For majortissuereconstructions,meticulousplanningandteamworkisessentialforsuccess.If 211

29: PLASTIC AND RECONSTRUCTIVE SURGERY 30 Disaster surgery

Multiple choice questions ➜ Natural disasters E Triage is best recorded with an 1. Which of the following are appropriately coloured label attached to characteristics of a natural disaster? the patient. A Civil administration is usually disrupted. ➜ B The armed services will be needed to Transport of patients restore order and reconstruct. 4. Which of the following statements C Shelter for large numbers will be one of regarding transport of patients is true? the first priorities. A It should be delayed until the patient is D They occur over a short period of the stabilised. time. B It should be carried out using the fastest E Most countries have organisations form of transport. prepared to handle these disasters. C Casualties must be accompanied by trained staff. 2. Which of the following statements D Patients should not be sent out with drip regarding action priorities in a natural sets and fluids if these are needed at the disaster are true? scene of the accident. A Assessment of the extent of damage is undertaken once rescue operations are ➜ Emergency care in the field underway. hospital B Experienced senior staff should stay back from the disaster area to carry out 5. Which of the following statements planning. regarding emergency care in the field C Local volunteers should not be involved, hospital are true? only trained staff. A Major surgery should only be considered D The first priority is to prevent further if the patient will not survive transfer to a damage or harm to staff from major centre. occurring. B Measures to control haemorrhage should be undertaken in the field. ➜ Triage C Amputation of devitalised limbs and for gas gangrene should be undertaken in 3. Which of the following statements the field. regarding triage are true? D Replantation of limbs should be A Triage means treating the most seriously attempted. injured first. E Open fractures should be cleaned in the B Triage is carried out where the casualties field. are found. F Repair of damaged major vessels should C Triage is carried out at the same time be attempted, if this is needed to save a as simple emergency life-saving limb. procedures. G Repair of damaged nerves should be D Triage does not mean that a patient’s attempted. triage category cannot be reviewed.

212 ➜ ➜ ➜ 8. Which of the following statements Whichofthefollowing 8. statements Whichofthefollowing 7. statements Whichofthefollowing 6.

G F E D C B A Necrotising fasciitis G F E D C B A Tetanus F E D C B A contaminated wounds Debridement of Hyperbaric oxygen reducesthemortality. Hyperbaric oxygen the patientistoxic. while shouldnotbeundertaken Surgery percent. rateisover50 The mortality not found. It spreadssteadilyandskiplesionsare the microvasculature. Necrosis isassociatedwiththrombosisof differentorganisms. several It canalsobecausedbyinfectionwith haemolytic This isprimarilycausedbybeta- require paralysingandventilating. managed usingsedationanddonot tetanuscanbe Patients developing organism. Penicillin Visineffectiveagainstthe toxoid. anti-tetanus globulinaswelltetanus Heavily contaminatedwoundsrequire The sporesarefoundinsoil. It thrivesinanaerobicconditions. It causesdamagebyreleasinganexotoxin. perfringens It iscausedbytheorganism It doesnotinvolvedefinitivetreatment. open. It frequentlyinvolvesleavingthewound It mayrequirerepeatedexploration. All doubtfultissueshouldberemoved. incision. It canbedonethroughashort meanslettingoutpus. The termnow regarding necrotisingfasciitisaretrue? regarding tetanusaretrue? regarding debridementaretrue? . Streptococcus . Clostridium ➜ ➜ ➜ 10. Which of the following statements Whichofthefollowing 10. statements Whichofthefollowing 9. (a) Which two of the following arelikely ofthefollowing Whichtwo (a) Apatientistrapped underacollapsed 11.

s Case stud Blast injuries Gas gangrene E D C B A G F E D C B A B A y ndrome andh regarding gasgangrenearetrue? causes ofhisconfusion? causes of thepatientis27ºC. either ofthem.Thecoretemperature not appeartobeablefeelormove are cold,paleandpulseless.Hedoes but regularandhisrightarmleg freed, heisconfused,hispulsefaint building for18 h.Whenheisfinally regarding blastinjuriesaretrue? The absence of crepitus excludes the The absenceofcrepitusexcludes hypertension. The toxinreleasedcausesmalignant haemolysed redcells. from The gasproducedisoxygen left open. It ismorelikelytooccurifwoundsare tissues. It thrivesonlyinwell-perfused A varietyoftoxinsareproduced. It maybecausedbycoliforms. Head injury Hypothermia communication isaproblem. Patients areusuallydeafso fragments. an issueastheheatsterilisesany Contamination ofawoundisnot deep andtheirbordersdifficulttodefine. Penetrating woundsfromfragmentsare the body. Blasts mainlyaffectfluid-filledcavitiesin injury. obstructions areprotectedfromblast hiddenbehindwallsorother Casualties diagnosis. y –crush y pothermia 213

30: DISASTER SURGERY TRAUMA ➜ ➜ 3.B,C,D, E 2.B,D 1. A,B,C Answers: Multiplechoicequestions (b) Which three of the following Whichthreeofthefollowing (b) 214

Triage Natural disasters improve duringthe minutesandhoursafterthedisaster. ofapatient isbest Thetriagecategory arereassessedatalater time.Somepatientsmaydeteriorateandothers changed whenthey apatienthavingtheirtriagecategory to delaytheprocessoftriage.There isnothingtoprevent procedureswhichshouldnotbeallowed is carriedoutsimultaneouslywithsimple life-saving couldbe loss oflifeamongstthemanyforwhom quickandsimpleinterventions Triagelife-saving. allyourresources toasmallnumberofthemmightresultinfargreater to die,becausedevoting Triage mayhavetobeleft meantreatingthemostseriouslyinjuredfirst. They doesnotnecessarily inthelongterm. tosustainablerecovery key andinitiative aregoingtobe maynothavearrivedandlocalknowledge early hoursanddaysthey butinthe trainedspecialiststafftodealwith adisaster Itistemptingonlytoallow rescue effort. andlogistics ofthe otherseniorstaffwillneedtostaybackorganisethestrategy However, can tellwhatisneeded. staffwillneedtogointothefieldasonlythey Experienced not afterwards. ofdamagemusttakeplacebeforeandduring the initialrescueefforts, oftheextent Assessment thebest-laidplans. overwhelm major disasters of usuallyfindthatthesizeandcomplexity those countrieswhoshould,andcan,planfordisaster areoftenthoseleastpreparedforthemintermsofplanningandinfrastructure.Even disasters havebeenrepaired.Thecountriesmostsusceptibletonatural consequences ofthedisaster within themediaandpublic,whichfrequentlyresultsininterestbeinglostlongbefore termbutthereisanaturalfatigue intheshort attractattentionandsupport Disasters single event. periodandarenotalwaysa overanextended suchasfloodingcandevelop Natural disasters offood,water,shelterandmedicines.Thesearegoingtobetheearlypriorities. for transport The armedforceswillbeneededtomaintainlawandorderprovidetheskilledmanpower andsowillbecrippled. order andbringhelptothoseinneedareequallyaffectedbythedisaster neededtomaintaincivil services arethatthevery The specificcharacteristicsofanaturaldisaster E D C B A J I H G F Blankets andawarmenvironment Hot oralfluids his rewarming? techniques aremostappropriatefor Alcohol. Liver failure Uraemia Hypoxia Atrial fibrillation Social isolation Septicaemia syndrome tocrush Hypovolaemia secondary c Whatorganismostatriskasthe (c) E D C B A E D C Heart –duetosludgingofbloodand Heart degradation products –duetoreleaseofmuscle Kidney Lung –duetomultiplemicroemboli products Liver –duetobloodbreakdown crushed limbs Brain –duetohypoxicshuntingthe high ‘end load’. limbs starttoreperfuse? bypass. Extracorporeal Large volumesofwarmedIVfluids patient tobreathe forthe Heated humidifiedoxygen ➜ ➜ ➜ ➜ 7. B,C,D,E 6.C,D,E,F 5.A,B,C,E,F 4.A,B,C,D

Tetanus Debridement ofcontaminatedwounds Emergenc Transport ofpatients havebeentriagedandwhatitisfeltneedstobedone. medical workercanseeatoncethatthey recorded onacolouredlabelattachedtotheirwristorhungaroundneck,sothatanyother they willneedtobeparalysed andventilated. they willhavedifficultybreathing forthemselves,andifspasmsbecomesevere, tetanusthey blown full- to penicillinVbutantibioticscannot substituteforgooddebridement.Oncepatientsdevelop sensitive to giveanti-globulinaswelltetanus toxoidtoprotectthepatient.Theorganismisvery and itthrivesinanaerobicordead tissue. Ifawoundisheavilycontaminatedthenitadvisable (tetanospasmin).Thesporesarefoundinsoil, Gram-positive cocciwhichproduces anexotoxin The organismis be donebyspecialistteamsinanappropriatelyequippedunit. been removedandthatthewoundisclean.Thisnotdefinitivetreatment –thatshouldbeleftto thatallnon-viabletissuehas untilitiscertain The processshouldberepeatedat24–48hintervals tissue ofdoubtfulviabilityshouldberemoved,andthewound be leftopenandpacked. tetanusandgangrene.All wound frombecominginfected,andmostespeciallyforpreventing acontaminated toolforpreventing seen andadequatelydealtwith.Itisthesinglemostpowerful non-viable tissue.Itrequiresalongincisiontobesurethatalloftheaffected tissuecanbeclearly the processofcleaningandtidyingawound,removingforeignmaterial,contaminated The ‘old’ cometobeuseddescribe meaningofthiswordwastoreleasepus,butithasnow laterinadefinitiveenvironment. can betterbeundertaken They repairsbeattempted. resources, whichcannotthenbeusedformaximumgain,norshouldnerve tieup asthey operationsshouldnotbeundertaken Replantation andotherlongcomplex alimb,thistooshouldbeattempted. andthiswillsave of amajorvesselcanbeundertaken contaminationwillrapidlyturntorampantinfection.Ifarapid repair in thefieldasotherwise andpotentiallygangrenouslimb.Openfracturesalsoneedcleaning or amputationofadevitalised (damagelimitation).Thismightbetocontrolcatastrophic haemorrhage bysurgery can beaverted inthefield,unlessthereisathreattolife oflimbwhich shouldnotbeundertaken Major surgery tobecompletedsafely.their evacuation zone.Eachpatientshouldgowithenoughfluidsandmedicationtoenable skills fromthedisaster will needtobeaccompaniedbyatrainedmemberofstaff,despitethefactthatthisremove thejourney.in starting Ifthepatientisbeingmonitoredandlinesareused,then thereislittlepoint delayed;otherwise however adequate suppliesprovidedtocoverthejourney, sothepatientshouldbepreparedforworstpossiblescenario,and take longerthanexpected always need.Journeys Thepatientwillalsoneedtogowithalltheequipmentthatthey starts. and lines(IVaccess,chestdrainsetc.)shouldbesetupcarefullytiedinbeforethejourney difficultindeed.Alltubes during transfer(whetherinahelicopteroranambulance)isvery aretransferred,asassessmentandtreatmentofapatient Patients shouldbestabilisedbeforethey y Clostridium tetani care inthefieldhospital ( C. perfringens producesgasgangrene).Theorganismisa 215

30: DISASTER SURGERY TRAUMA ➜ ➜ ➜ ➜ 11. (c)D 11. (b)B, C,D 11. (a)A, C 10. C,E 9.A,B 8.A,B,C,E,G 216

Case stud Blast injuries Gas gangrene Necrotising fasciitis The main causative agent is beta-haemolytic agentisbeta-haemolytic The maincausative possible. No other organ is primarily at risk in the same way aftercrushsyndrome. possible. Noother organisprimarilyatriskinthesame assoon unlessarapiddiuresisisstarted from damagedmuscle)whichwillblock thekidneys thereisarealriskofmyoglobin(released to reperfuse, andthelimbsstart thepatientrewarms As hypothermicpatients, butisunlikelytobeappropriatehere. severely helpful invery tobe difficulttousebutissaid bypassisvery as wellraisingcoretemperature.Extracorporeal of warmedIVfluidscanbeusedtotreatthehypovolaemicshockcaused bythecrushsyndrome, techniqueforwarmingthecore(whenitisavailable). Largevolumes humidified airisapowerful andsteadily. thecoretemperaturetoriseslowly environment andblanketswillallow Heated withcare.Warmedfluidsorallyshouldnot beused.Awarm shouldbeundertaken Rewarming other morelikelycauseshavebeenexplored. should notbeinvokeduntilthe orthatalcoholcouldbeinvolved,sotheseexplanations injury his pulseisregularsoitunlikelythatheinatrialfibrillation.There is nosuggestionofahead to becausedbyinfection,uraemiaorliverfailure.Socialisolationdoes notcauseconfusionand confusion arethereforehypothermiaandhypovolaemia.Itwillbetoo soon forthisconfusion This patientislikelytohavea‘crush andtobehypothermic.Thelikelycausesofhis injury’ notsterile. borders maybeill-defined.Thefragmentsheavilycontaminatedandarecertainly fast andsocanpenetratedeeply,leavingalargetrackofdamagedsofttissuebehindit,whose arefrequentlydeaf).Theblastmaybeaccompaniedbyfragmentswhichmovingvery (so they and,indeed,eardrums Damage withinthebodyisfocusedonair-filledcavitiessuchaslungs,bowel Obstructions donotprotectvictimsastheblastspreadsaroundfixedobjectsitisasoundwave. thediagnosis. crepitus doesnotexclude the conditioniscalled‘gas’gangreneandgasinsofttissuesdoesproducecrepitus,absenceof variety oftoxins,somewhichcauseshock,andsothepatient’sbloodpressurefalls.Although iscompletelyabsent.Theorganismproducesa produced ismainlyhydrogensulphide.Oxygen Thegas If awoundisleftopen,itmoredifficultforananaerobicenvironmenttodevelop. tissueswhichcreateananaerobicenvironment. poorlyperfused It onlythrivesindeadorvery gangrene isusuallycausedby Gas early aggressivesurgery. spreadoftheinfectionbutisnosubstitutefor mayhelptoprevent Hyperbaricoxygen of surviving. toremovedeadtissueifthepatientishaveachance and thereshouldbenodelayinsurgery rateisover70 percent, remotefromtheoriginalsite.Themortality areasdeveloping with new toxins causesthrombosisofthemicrovasculature,butsurprisinglytheremayalsobeskiplesions differentspeciesoforganism.Necrosisspreadsrapidlybecauserelease infection withseveral y –crushs y ndrome andh Clostridium perfringens Clostridium perfringens Streptococcus y pothermia , buttheconditioncanalsoresultfrom but canalsobecausedbycoliforms. PART 5 Elective orthopaedics

31 Elective orthopaedics: musculoskeletal examination 219 32 Sports 224 33 The spine 233 34 The upper limb 238 35 The hip and knee 246 36 The foot and ankle 252 37 Infection and tumours 257 38 Paediatric orthopaedics 264 This page intentionally left blank 31 Elective orthopaedics: musculoskeletal examination

Multiple choice questions ➜ Musculoskeletal history E Examiners should not watch their hands 1. Introducing yourself to a patient when feeling. includes: 4. Which of the following statements A Giving your own name regarding the ‘move’ stage of the Apley B Checking the patient’s name system of examination are true? C Explaining what is to happen A ‘Move’ is the third stage of the Apley D Obtaining the patient’s consent to system of examination. proceed. B You should move the limb first. E Washing your hands C Stability includes checking power of F Obtaining adequate exposure muscles, stability of joints and special G Checking for tenderness. tests. ➜ D Abduction is a movement away from the Musculoskeletal examination midline. 2. Which of the following statements E Muscle power is measured on a 6-level about the Apley system of examination scale. are true? A It is a four-stage system. ➜ Spine examination B The ‘look’ stage only starts once the 5. Which of the following statements are patient has been undressed. true? C It requires exposure only of the affected A Asking the patient to bend forward limb and one joint above and below. accentuates the rib hump of idiopathic D It starts with checking the skin for scars, scoliosis. wounds and redness. B The thoracic spine normally has a E It then moves to bone-checking for lordosis. deformity. C A hairy tuft at the base of the spine is F It requires that the distal neurovascular diagnostic of Down’s syndrome. status is checked in all cases. D The Lasègue straight-leg raise test 3. Which of the following statements specifically tests the range of movement regarding the ‘feel’ stage of the Apley in the hip joint. system of examination are true? E of the spinous processes allows A The ‘feel’ stage is the second stage of the you to feel a spondylolisthesis. Apley system. ➜ B It is wise to ask the patient if anywhere is Hand and wrist examination tender before starting ‘feeling’. 6. Which of the following statements are C When feeling, the same triad is used as true? in looking. A Dupuytren’s contracture is seen as tight D Distal neurovascular status must bands in the skin, usually pulling down be checked in all musculoskeletal the fingers on the ulnar side of the examinations. hand

219 ELECTIVE ORTHOPAEDICS ➜ ➜ ➜ 1. A,B,C,D Answers: Multiple choicequestions statementsare Whichofthefollowing 8. statementsare Whichofthefollowing 7. 220

Musculoskeletal histor A Hip examination F E D C B A examination Elbow andshoulder F E D C B explain whatyou areproposingtodobeforeobtaining thepatient’spermissiontoproceed. explain nameandchecking thepatient’sname.YouIntroduction consists ofgivingyourown should then of osteoarthritis ofthehip. of osteoarthritis Fixed abductiondeformityischaracteristic joint. diagnostic oftheshoulderbeingout The shoulderapprehensiontestis rotator cuffimpingement. Jobe’s emptycantestisspecificfor humeral headusuallymovesanteriorly. In dislocationoftheshoulder under thetipofacromion. The rotatorcuffintheshoulderruns origin isdiagnosticoftenniselbow. Tenderness overthecommonflexor finds mostcomfortable. apatient isthedegreeofflexion elbow angleofthe The physiologicalcarrying fingers. if thepatientfeelslightningpainsin retinaculumandsee tap overtheflexor compression inthecarpaltunnelisto A usefultestformediannerve alone. thedistalinterphalangealjoint flex patient’s fingerfirmlyandaskingthemto canbetestedbyholdingthe superficialis Transection digitorum offlexor inthehand. surgery before performing Allen’s testshouldalwaysbeperformed the hand. lossin accurate wayofdetectingsensory Two pointdiscriminationprovidesan palsy.diagnostic ofanulnarnerve Wasting ofthethenareminenceis true? true? y ➜ ➜ 10. Which of the following statementsare Whichofthefollowing 10. statementsare Whichofthefollowing 9.

The footandankle B A E D C B A E D C B E D C true? true? flexed to90º. flexed to45ºandtheknee the hipflexed Rotation ofthehipisbesttestedwith called theTrendelenburg test. with theirhandsrestingonyoursis thepatienttostandononeleg Asking walk. asthey bob upanddown Patients withweakabductormuscles into trueandapparent. canbedivided discrepancy Leg-length can stillstandontheirtoes. Patients witharupturedtendoAchilles physiological fromspasticflatfoot. The windlasstestdistinguishes ankle joint. occuratthe Inversion andeversion diabetes. stocking distributionisassociatedwith inaglove-and- Loss ofsensation syndrome. Pes cavusisassociatedwithMarfan’s apprehension test. their patellawillhaveapositive dislocated A patientwhohaspreviously draw test. ligament disruptionwillhaveapositive A patientwithaposteriorcruciate is testedwiththekneeinfullextension. The integrityofthecollateralligaments in thekneejoint. The lagtestchecksforlossofextension stroke testisperformed. easily seenonthemedialsidewhena A smalleffusioninthekneeismost ➜ ➜ ➜ 6.A,C,D, F 5.A,E 4.A,C,D,E 3.A,B,C,D,E 2.D,F

Hand andwristexamination Spine examination Musculoskeletal examination The Apley system is a triad (three parts) comprising ‘look’, ‘feel’, ‘move’. Looking actually starts comprising‘look’,‘feel’,‘move’.Lookingactuallystarts systemisatriad(threeparts) The Apley on the hand as it is crucial to know about the blood supply and any reserve if an artery hastobe ifanartery aboutthe bloodsupplyandanyreserve on thehandasit is crucialtoknow tocheckthisbefore operating Itisalways important supplied byboth the radialandulnarnerves. discrimination needstobetestedwith apaperclip.Allen’stestensuresthatthepalmararchis loss,two-point ofsensory andseverity for comparison,butaccuratedelineation ofextent usingthestroketest,oppositehand inthehandcanbequicklyexcluded sensation Abnormal tissues, wastingofthethenareminence isdiagnosticofdamagetothemediannerve. Inthesoft intoflexion. contracted bytightbandsandthelittle andringfingerarepulleddown contractureisimmediately visiblewhenfirstinspectingthehand.Theskinis Dupuytren’s roots(sciatica). from tighthamstringsfocusingonpainproducedbytuggingthesciatic nerve painarising fromthehipjoint,and scoliosis. TheLasèguestraight-legtestspecificallyexcludes accentuatestheribhumpcausedby rotationofidiopathic thepatienttobendforward Asking (spondylolisthesis). bodyontheonebelow slipofonevertebral and stepcausedbytheforward youtofeelthe characteristicgap again hasalordosis.Palpationofthespinousprocessesallows andthelumbarspine (kyphosis), spineisconcave(lordosis),thethoracicconvex the cervical lookedatfromthe side, plane.However, condition). Thespineshouldbestraightinthesagittal A tuftatthebaseofspineisdiagnosticspinabifidaocculta(the mildest formofthis systemaselsewhere. thesame thoracicandlumbar–follows ofthespine–cervical, Examination which goesfromzeroto5(6grades). scale, isusuallymeasuredontheMRC ligamentstabilityandspecialtests.Musclepower power, midline) andabduction(awayfromthemidline).The‘stability’checksincludecheckingmuscle the (backwards),adduction(towards extension (forward), the fullrangeofmovement:flexion thelimitsofactivemovement.Eachjointshouldbeputthrough especially whenmovingbeyond with‘passive’movement,watchingthepatient’sfacecarefullyforsignsofdiscomfort then follows limb–be‘active’asthiswilldemonstrateanylimitsofpain.Theexaminer to movetheirown Patientsshouldfirstbeinvited systemoftheexamination. Move isthethirdstageofApley willbefound. patient’s facethatthefirstsignsofdiscomfort Itisbesttowatchthepatient’sface,notyourhands,whenpalpatingasiton examinations. carefully.last andthenworkvery Distalneurovascularstatusmustbetestedandrecordedinall for ‘looking’.Itiswisetoaskapatientifthereanyareathatespeciallytender;leavethisuntil systemasused systemof‘look’,‘feel’,‘move’,thesame Feeling isthesecondstageofApley for bothswellingandwasting.Finallythebonesarecheckeddeformity. tobechecked,looking (especiallytheback)ismissed.Softtissuesarenext sure thatnoview at theskin.Thisisoftenbestdonewithpatientstanding,walkingaroundtomake withlooking systemstarts comparison canbemadebetweennormalandabnormal.TheApley theaffectedareaaswelloppositeside,sothat a from onejointaboveandbelow consistsofremovingcoveringclothes Exposure the patientobviouslyhasrheumatoidarthritis). gentleif foraHorner’ssyndrome)andshakehands(bevery introduce yourself(checktheeyes as thepatientcomesthroughdoor(checkingforlimp,painandwalkingaids)whenyou 221

31: ELECTIVE ORTHOPAEDICS: MUSCULOSKELETAL EXAMINATION ELECTIVE ORTHOPAEDICS ➜ ➜ ➜ 9.A,D,E 8.B,D,E 7. C,D,E 222

Knee examination Hip examination Elbow andshoulderexamination compression. arediagnosticofnerve distribution ofthemediannerve retinaculum.Shootingorlightningpainsrunningintothefingersin used to‘tap’overtheflexor profundus. Whentestingforcarpaltunnelsyndrome,Tinel’stestisuseful.Thetipofafinger by the distalendofmiddlephalanx.Theinterphalangealjointisthereforeonlyflexed into digitorumprofundustoinsert splitsandcrossesflexor digitorumsuperficialis divided. Flexor the knee forward toanabnormalpositionfromthere. the kneeforward disrupted anterior cruciatehasthetibiainanormalposition atrest,butitisthenpossible todraw andmovesto a normalone,whilethepatientwith a fromanabnormal position‘sagged’ starts ligament willproduceapositivedraw sign.Thedifferenceisthataposteriorligamentdraw posterior structuresofthekneelock itandmaskanyinstability. Adisruptionofeithercruciate positive lagtest. manoeuvreactively.same Adifference betweenpassiverangeofmovementandactiveindicatesa thekneefully. the arethenaskedtoperform They made thatthepatientcanpassivelyextend mechanism(quadriceps) oftheknee.Acheckisfirst a sensitivetestforweaknessintheextensor inflatingthedimpleonmedialsideof knee. Thelagtestis fluid isthensqueezeddown, side ofthekneeisstrokedoutandupintolargesynovialpouchunder thequadriceps.This When testingforakneeeffusion,themostsensitivetestis‘stroke test’. Anyfluidinthemedial rotation. measure therangeofinternalandexternal legcanbe usedasaprotractorto sothatthelower (45º)andthentoputthekneeat90º flexion themselves fromtopplingover. onyourhandtheoppositesidetoprevent willbeforcedtopressdown weight onthatlegthey take on eachleginturnwhilerestingtheirhandsyours.Iftheabductorsareweak,thenasthey The testforweakabductorsistheTrendelenburg. Onewayofdoingitistoaskpatientsstand swaybutdonotbob. takeweightthroughit,sothey overthathipwhenthey their bodysideways onthehip)havetothrow Patientswithweakabductormuscles(afterpolioorsurgery lengths. asmeasuredbycheckingthebone when thereisnodifferenceinthe‘real’leglengths even leg(because ofthedeformity) andadducted.Thiscreatesanapparently‘short’ flexed of comfort, ofthehipiscommonestconditionaffectinghip,whichmovesintoaposition Osteoarthritis dislocation. Itcannotbeusedwhentheshoulderisoutofjoint. The apprehensiontestisusefulfordiagnosingapatientwhohas,inthepast,hadananterior tobeinflameduponthebeakofbonecausingproblem. presses themostlikelypart is specificforimpingementasitplacesthesupraspinatustendonunderacromionandthen bone onthetipofacromionwhichcanleadtopainfulrotatorcuffsyndrome.Jobe’stest intotherimofglenoidanditisformationabeak under theacromiontoinsert origin.Intheshoulder,rotatorcuffpasses istherarerequivalentincommonflexor elbow originonthelateralsideofelbow. Golfer’s painful inflammationofthecommonextensor andmaybealteredifthereismalunionofasupracondylarfracture.Tennisforwards, is elbow straight,armsatthesidesandhandspointing arm whenthepatientstandswithelbow angleistheofvalgus whichtheforearmmakeswithupper The physiologicalcarrying When theshoulderdislocates,itisparadoxicalthathumeralheadusuallymovesanteriorly. When testing the collateral ligaments, it is necessary to flex the knee slightly, as otherwise the thekneeslightly,asotherwise toflex When testingthecollateralligaments, itisnecessary When testingrangeofrotationthehip,itisbesttoputintomiddle ofitsrange ➜ 10. B,D,E

The footandankle still stand on their toes using their toe flexors as plantarflexors oftheankle. asplantarflexors still standontheirtoesusingtoeflexors coalition). ItisaparadoxthatpatientswithcompleteruptureofthetendoAchillescanusually not appear/increasethenthepatienthasaperonealspasticflatfoot(oftencausedbytarsal to standontiptoes,thearchoffootshouldincrease.Thisiswindlasstest.Ifdoes at theanklejoint,whileinversionandoccursubtalarjoint.Whenyouaskapatient occur andplantarflexion loss.Dorsiflexion produces aglove-and-stockingdistributionofsensory rootentrapmentwillbeinthedistribution ofadermatome,butdiabetes resulting fromnerve inthefoot syndrome produceshyperlaxityandsotendstoproduceaflatfoot.Lossofsensation Pes cavusisahigharchedfootandassociatedwithspinalneurologicaldisorders.Marfan’s apprehension inthepatient. from thenonanymanualattempttodislocatethepatellalaterallywillcause However, Dislocation ofthepatellacanbedifficulttodiagnoseasitnormallyrelocatesspontaneously. 223

31: ELECTIVE ORTHOPAEDICS: MUSCULOSKELETAL EXAMINATION 32 Sports medicine

Multiple choice questions ➜ Case study – shin injury ➜ Tendon injuries 1. A cricketer complains of pain in his 3. Which of the following statements shin which has been present for regarding tendon repair are true? several months but which got worse A The strength of a damaged tendon today after a prolonged bout of fast decreases for some time after injury bowling. The shin is slightly red, a little and only returns back to normal after 6 swollen and tender to touch. months. B Tendons heal by degeneration of the (a) What class of injury is this likely to distal portion followed by regrowth from belong to? the proximal end. A Acute extrinsic C Paratendinitis has a poor prognosis. B Acute intrinsic D Tendinosis can be painless. C Chronic. E Tendons consist of type 1 fibres. (b) What is the physical treatment plan? A Immediate intensive physiotherapy ➜ Case study – ligament injury B Plaster of Paris immobilisation 4. A sportsman presents having sprained C Continue sport, by controlling symptoms his thumb. There is slight painless with analgesics laxity in the ulna collateral ligament. D Steroid injection What grade of ligament injury is this? E Find alternative exercise plan which does A Grade 0 not load the tibia. B Grade 1 (c) Give two reasons why non-steroidal C Grade 2 anti-inflammatories might be helpful. D You cannot tell until you have compared A To reduce pain it with the other side. B To reduce oedema C To enable immediate return to sport ➜ Bursae before the injury is healed 5. What are the characteristics of bursae? D To prevent the problem spreading to the A They are normal structures designed to other leg. reduce friction. B They are lined with synovial membrane ➜ Soft-tissue haematoma and connected to the joint beneath. 2. Which of the following statements C They are able to become inflamed and regarding soft-tissue haematomas are infected. true? D They have no nerve supply. A Most resolve spontaneously. B If a cyst develops, surgical excision may ➜ Bone healing be needed. 6. Which of the following statements are C The changes can become malignant. true? D The bruised muscle may sometimes be A The first phase of bone healing is the replaced with cartilage. laying down of bone in the fracture cleft.

224 ➜ ➜ ➜ ➜ 0 A20-year-old mansustains adislo- 10. statements Whichofthefollowing 9. reductionofan Whyistheanatomical 8. 7. Stress fractures:

injur Case studies–shoulder D C B A mechanism oftheankle Injuries totheplantarflexor E D C B A Ankle injur E D C B A Stress fractures D C B causing nerve damage. causing nerve Achilles carriesasignificantriskof Surgical repairoftherupturedtendo calf andlookingtoseeifthefootmoves. Simmond’s testinvolvessqueezingthe aretreated. they cent ofcaseshowever occursinlessthan101 yearofinjury per Re-rupture oftheAchillestendonwithin Achilles tendondonotrequiresurgery. Musculotendinous junctiontearsofthe osteoporosis. To avoidtheonsetofpremature osteoarthritis To avoidprematureonsetof To improveproprioception To intheankle fullstrength allow movement To enablereturnofafullrange Heal quickly. imaging (MRI) up wellonmagneticresonance Show Are difficulttoseeonX-ray Produce highlylocalisedpain load sports Are mostcommoninhighintensity be laiddown. Stress onbonestimulatesmoreto many monthsafterthefracture. Remodelling ofthebonegoesonfor the fracturearea. around ismaturebonelaiddown Callus injury aretrue? injury regarding plantarflexormechanism ankle fracture soimportant? cation oftheshoulder playingrugby.cation y y 2 Arugbyplayerfellontohisshoulder 12. A25-year-old suffersatearofthe 11. Themanaskswhatisthechanceof (b) ➜ a Whatwouldyoutellhimisthe (a) 3 Asportsmancomplainsofpainover 13. 4 Aweight-lifter complainsof 14.

Case studies–elbowinjur E D C B A C B A C B A C B A E D C B A E D C B A again? chance ofthisshoulderdislocating answer wouldyougivehim? What the othershoulderdislocating. What hasheinjured? diameter onthetopofhisshoulder. with apainfullumpabout4cmin presents some weeksago.Henow successful? as thechancesofrepairbeing repair it.Whatwouldyouofferhim rotator cuffinabadfall.You offerto the sourceofproblem? the lateral sideofhiselbow. Whatis Ͼ Ͼ Ͻ Ͼ Ͼ Ͻ The brachialplexus. The mediannerve The ulnarnerve Acromioclavicular joint origin Common extensor What hasheinjured? numbness andtinglinginhisfingers. The ulnarnerve. The radioulnarjoint The radiohumeraljoint Acromioclavicular joint origin Common extensor The rotatorcuff. The sternoclavicularjoint The headofthehumerus The acromioclavicularjoint origin Common extensor Ͼ Ͼ Ͻ 80 percent. 80 percent 50 percent 50 percent. 80 percent 50 percent 50 80 percent. 80 percent 50 percent 50 225 y

32: SPORTS MEDICINE ELECTIVE ORTHOPAEDICS ➜ Extended matchingquestions Extended (b) Choose a test or investigation from the following that ismostappropriatetoconfirmeach Choose atestorinvestigationfromthefollowing (b) Choose andmatcheachoftheabovescenarioswithstructurethatislikelytobe (a) 226

F E D C B A L K J I H G F E D C B A 8 7 6 5 4 3 2 1 1. Kneeinjur MRI oftheknee MRI Standard X-raysof theknee Tenderness onpalpationofthe synoviumaroundthepatella MacMurray’s test test Posterior sag Lachmann’s test Chondromalacia patellae Fractured patella Torn anteriorcruciateligament Snapping pesanserinus Femoral condylefracture Torn posteriorcruciateligament Medial collateralligamenttear Hoffa’s syndrome Dislocated patella Torn meniscuswithbuckethandletear Tibial plateaufracture Osteochondritis dissecans place. itwasclicksbackinto tostandup,whatever asherollsovertotry place inthisknee.However, painandnoticessomethingoutof A rugbyplayeristackledandhiskneetwists.Hefeelssevere knee. Hefeelssomethinggivewayandiscarriedoffthepitch. tacklewhereanotherplayerfallsacrosstheoutsideofhis A footballerisinvolvedinaheavy knee pain,especiallywhenhetriestostraightenit. haschronic Henow A teenagelongjumperinjureshiskneebylandingwithithyperextended. stairs. togodown oftimeandwhentrying length atrivialinjury. kneepainfollowing A teenagegirldevelops Itisatitsworstwhenshesitsforany not straightenandhehastojiggleituntilfreesup. cleanerwhoplaysfootballatweekendscomplainsthatfromtimetohislegwill A window painful andswollenshecannotwalk. tochurchtripsonthestepsandfallsherleftknee. Itisnow An elderlywomanhurrying from thesteeringwheel.Hisrightkneeisalsopainfulandswollen. and crashesintoawall.Hehasfacialinjuriesfromthewindscreenfracturedsternum A joyrider,whoisnotwearingaseatbelt,finallyarrestedwhenhelosescontrolofthecar offthemountainonastretcher.immediately andhehastobebroughtdown fails toreleaseandhefeelsacrackinhisknee.Nothingseemsbeoutofplace,butitswells A skiertwistswithhisbodyturningoutwards(thetibiarotatesinwards)ashefalls.Thebinding of thediagnosesin(a): likelytobesustained: damaged ortheinjury y

➜ ➜ Choose and matchasuitablediagnosisforeachofthescenarios givenbelow: Choose themostappropriatetreatmentforeachofdiagnoses: (b) thatislikelytobe Choose andmatcheachoftheabovescenarioswithinjury (a)

1 F E D C B A 3. Footinjur E D C B A E D C B A 5 4 3 2 1 2. Ankleinjur M L K J I H G fast bowling. fast bowling. painaroundthemetatarsophalangeal jointofhisbigtoeafterabout A cricketerdevelops Heel bumps Freiberg’s disease Turf toe Metatarsalgia March fracture neuroma Morton’s Serial plastersplints Heel cups±localsteroidinjection Modified footwearwithinsoles Surgical repairofthetendonsheath Non-steroidal anti-inflammatoriesandrest Plantar fasciitis Rupture oftheAchillestendon oftheperonealtendons Subluxation Tendinitis ofthetibialisanterior Rupture ofthetibialisposteriortendon oftime. sitting stillforanylength unilateralheelpainwithoutanytrauma.Itisworstafter A middle-agedactivepatientdevelops him. inthebackofankle,althoughitappearsthatnothinghit A squashplayerfeelsasharpblow third ofthetibia. lower pain,swelling,tendernessandcrepitusoverthefrontof A long-distancerunnerdevelops falling intovalgus. An elderlywomannoticesasuddencollapseofthelongitudinalarchherfootwithheel in theoutsideofherankle. A patientwithahigh-archedfootcomplainsofthespontaneousonsetsnappingsensation Tunnel oftheknee views Anterior drawtest Patella apprehensiontest grindtest Apley Injection oflocalanaesthetic Stressing thekneeintovalgusinslightflexion Tenderness topalpationovertheanteriorfatpad sustained: y y 227

32: SPORTS MEDICINE ELECTIVE ORTHOPAEDICS ➜ ➜ ➜ 3.A,D,E 2.A,B,D 1. (c)A,B 1. (b)E 1. (a)C Answers: Multiplechoicequestions 228

Tendon injuries Soft-tissue haematoma Case stud 6 5 4 3 2 a partial tear,there isaperiodwhenitactuallyweaker thanbefore(for7–10a partial days)beforeit atendonhealsfrom thathealbydistal degeneration(Wallerian).As collagen.Itisnerves new Tendons aremadeupoftightly packedtype1collagen.Tendons healbyfibroblastslayingdown on X-ray.which forms(myositisossificans),canlookquitelikeamalignantsarcoma bone andeven Thechangesdonotbecomemalignantbutthe cartilage, a courseoflastresort. Thisis,however, mayneed excising. mayform,andinrarecasesthiscyst the thigh)thenacyst large(usuallyin arevery Most haematomatainsofttissuesresolvespontaneously,butifthey has healed. beforetheproblem canbere-started not normallybeusedtosuppresssymptomssothatsport should answerthey Non-steroidals willreducepainandswelling,butasdiscussedintheprevious machine. orarowing which reststhetibiaandmusclesaroundit.Thisislikelytobeswimming,cycling tried. Thebestmanagementplanistokeeptheathleteactivebutadvisethemchooseasport Equallyasteroidinjectionisnotindicateduntilothermethodshavebeen can continuesport. it allthetime,thereisnoplaceforusingmedicationtosuppresssymptomssothatanathlete thisproblem.Althoughprofessionalathletesdo physiotherapy, whichmayactuallyexacerbate Thereisalsonovaluein be astressfracture),sothereisnoneedforplasterimmobilisation. fracturewithpotentialinstability(althoughtheremay ofafull-blown There isnoevidence The diagnosisislikelytobesomeformofshinsplint,eitheratendinitisorstressfracture. (acute intrinsic),andithascontinuedforsometime. orofasuddenfailure (acuteextrinsic) ofadirectblow This isachronicinjury. Thereisnohistory the 2nd metatarsal. the 2ndmetatarsal. apainfulcrack(undisplaced)acrosstheneckof A long-distancerunnerspontaneouslydevelops calcaneum. intothe platewheretheAchillestendoninserts associated withtheapohysealgrowth ahotandtenderareaovertheheelapparently fastdevelops An adolescentwhoisgrowing head. An adolescentwithpainintheforefootisfoundtohaveavascularnecrosisof2ndmetatarsal the frontofherfootatbase2nd,3rdand4thtoes. painunder todevelop A long-distancerunnerdoesmuchofhertrainingbarefooted.Shestarts pain andnumbnessin2toes. buthedevelops andinjureshisforefoot.Thereisnoboneinjury fallsfromhisbicycle A cyclist y –shininjur y ➜ ➜ ➜ ➜ ➜ ➜ 9.A,B,C,D 8. D 7. C,D 6.C,D 5.A,C 4. D

Injuries totheplantarflexormechanismof ankle Ankle injur Stress fractures Bone healing Bursae Case stud completely symptomlessbeforeitpresentswithacatastrophicfailure. has agoodprognosiswithperiodofrest.Tendinosis (degenerationofthetendonitself)canbe Paratendinitis(inflammationofthemembranearoundtendon) tobuildupstrength. starts intact. ThisisSimmond's test. thenthetendonis rupture ofthetendoAchillesistosqueeze thecalf.Iffootplantarflexes therepair. damage whileperforming Theclassic testfora small butsignificantriskofnerve returntonormalactivitiesmaybeslightly quickeraftersurgery.However, Againstthatthereisa should beusedtoassistserialplastering. Eitherway,there-rupturerateislessthan10 per cent. tendon itselfhasapoorhealingpotential, andopinionisdividedastowhethersurgicalrepair of thetendon),andsoitwillhealquickly anddoesnotrequireanyassistancefromsurgery. The The musculotendinousjunctionofthetendoAchilleshasagoodblood supply(unlikethebody willresult. thatprematurearthritis cartilage fragments healmorethan2mmoutofplace,therewillbesuchhigh peak loadsonthearticular The properreductionofananklefractureiscrucialbecausethe isintra-articular. Ifthe toheal. are depressinglyslow They hasprovedusefulforvisualisingthefracture. difficulttoseeonX-ray.can bevery MRI However, fractures repetitions, e.g.long-distancerunning.Thepainisnotatallwelllocalised,andthepartial largenumbersof whichinvolvevery sports low-load Stress fracturesarecommoninlow-intensity, loading afracturedoesstimulatehealing,providedthattheloadisnotexcessive. healing. TheHeuter–Volkmann inresponsetostress,so principlestatesthatboneislaiddown isimmaturebone,whichthenreplacedbymatureboneinthefinalstagesof by bone.Callus so noboneisinvolvedinitially. whichinturnisreplaced Thisscartissuemaythenformcartilage, offibrin, ofthehaematomaandlayingdown The firstphaseofbonehealingisorganisation supply. obviouslyhaveanerve can becomeinflamedandpainful,sothey each other. arelinedinsynoviumbutdonotnormallyconnectwiththejointbeneath.They They structuresunderloadarelikelytorubon arenormalstructureswhichfoundwherever Bursae There isusuallytendernessbuttherealwayssignificantlaxitywithnofirmend-point. disruption,painandincreasedlaxitywithanend-point.Grade3isacompletedisruption. partial Grade 0isnopainorlaxity. Grade1ispainonstressingwithoutincreasedlaxity. Grade2isa comparingthelaxitywithother(normal)side. ligament asadisruption,itisalwaysworth variablebetweendifferentpeople,sobeforediagnosingalax The laxityofligamentsisvery y y –ligamentinjur y 229

32: SPORTS MEDICINE ELECTIVE ORTHOPAEDICS ➜ ➜ ➜ Scenario 3 Scenario 2 (b)Test/investigation Scenario –1A,2B,3E,4F,5D,6G,7H,8K –1J,2G,3K,4C,5L,6E,7F,8D (a)Structure/injury 1 matchingquestions Answers: Extended 14. D 13. A 12. B 11. C 10. (a)B;(b)A 230

1. Kneeinjur Case studies–elbowinjur Case studies–shoulderinjur There is a 60 per cent chance of a further dislocationanda10 percentchanceofafurther There isa60 percentchanceoftheother and so the most likely injury isafractureofthepatella. ItcanbediagnosedbyX-ray.and sothemostlikely injury injury. onthe frontoftheknee Shehastakenadirectblow isaclassic acuteextrinsic This injury whichisdiagnostic. position.Itisthereforetheposterior sag posterior sag froma thedrawisforward test (justlikeapatientwithruptured anteriorcruciate).However, ligament. Patientswitharupturedposterior cruciateligamenthaveapositiveanteriordraw tibiaonthefemurrupturesposteriorcruciate This posteriordisplacementoftheflexed When ajoyridercrasheswithoutseatbelt,thekneeisdrivenbackby impactwiththedashboard. femur. onthe ofthe tibiaforward positive Lachmann’stest,bothofwhichtestforabnormalsubluxation anterior cruciateligamentismadebyfindingthatthepatienthasapositive anteriordrawtest,ora toplayonandmaynotswelluntilsomehourslater.a sportsperson Thediagnosisofadisrupted on.Atornmeniscus sometimesallows with blood.Theinjuredpersonisusuallyunabletocarry cruciate ligamenthasabloodsupply(unlikethemeniscus)andso kneeswellsimmediately and themedialcollateralligament(theso-calledunhappytriadofO’Donoghue). Theanterior cruciate ligament.Thisisanacuteintrinsicinjury. Itmaythen goontotearthemedialmeniscus kneewhenthefootislockedontogroundtearsanterior oftheflexed A twistinginjury operative measuresdonotwork,thensurgicalreleasewillbeneeded. atthewrist(carpaltunnelsyndrome).Ifnon- Weight-lifters commonlyinjurethemediannerve difficult totreatbutmaybehelpedbylocalsteroidinjection. origin.Theconditionis irritabilityofthecommonextensor This islikelytobe‘tenniselbow’ joint, butthismaynotcurethepain. pressontotheshoulder.shoulder strapsofarucksack the Theonlypossibletreatmentistoexcise whichproducesapainfullumpjustwherethe is inflammationthenearlyaggressivearthritis The acromioclavicularjointistypicallyinjuredbyafallontothepointofshoulder. Theresult theprognosisispoor.elderly, however, In theyoungpatient,cuffstillhassomeabilitytorepairsoprognosisisgood. shoulder dislocating. y

y y ➜ Scenario 4 Scenario 3 Scenario 2 Scenario (b) Treatment –1B,2C,3A,4E,5D 1 –1C,2A,3B,4D,5E (a)Injury Scenario 8 Scenario 7 Scenario 6 Scenario 5 Scenario 4

2. Ankleinjur then has to wiggle the knee to relocate the torn fragment before they canmovethekneeagain. then hastowigglethekneerelocatetornfragmentbeforethey in shapethensometimesthebuckethandlewillfoldoverandlockkneejoint.Thepatient isclassiclocking.Theremostlikelyatornmeniscus.Ifthetearbucket-handled This history sport. There is often a sharp crack and patients feel as if they havebeenhitonthebackof Thereisoften asharpcrackandpatientsfeel asifthey sport. strenuous Rupture ofthetendo Achillesoccursquitesuddenlyin middle-agedmenperforming anti-inflammatories accompaniedby resttoletthetendinitissettle. and tendernessoverthetibialisanterior. There mayalsobepalpablecrepitus.Thetreatmentis tendinitis ofthetibialisanterior.Long-distance runnersmaydevelop Therewillbepain,swelling thefallenarch. and manyaremanagedusingmodifiedfootwearwithinsolestosupport This isatypicalpresentationofrupturedtibialisposteriortendon.Surgical repairisalastresort, that holdsthetendonsinplace.Theonlyusefultreatmentissurgicalrepair ofthesheath. which hasrupturedthesheath This ismostlikelyifthepatienthassufferedsomekindofinjury This problemcommonlyarisesfromtheperonealtendonssubluxingover thelateralmalleolus. apprehension). thekneewillberesistedbypatient,whohaveasenseofimpendingdoom(the flex be leftwithapatellaapprehensionsign.Anyattempttopushthelaterallyasyoupassively butoncetheinitialinflammationhassettled,patientwill and sowillbedifficulttoexamine, then spontaneouslyrelocated(asitsooftendoes).Thekneewillbepainfulandswolleninitially This isoneofthecommoneracuteintrinsicinjuriestoknee.Thepatellahasdislocatedbut stabilisers) willbediagnostic. (toreleasetheposterior valgus force.Stressingthekneeintowithslightlyflexed This isaclassicacuteintrinsicinjury. Themedialcollateralligamentwillhavebeentornbythe tendernessofthefatpad. reveals resectionofthefatpad.Examination may requirearthroscopic becomes swollenandpainful.ThisiscalledHoffa’ssyndromeusuallysettlesspontaneouslybut ofthekneecrushesfatpadinfrontknee.Thethen injury A hyperextension tender. is notestwhichhelpswiththediagnosis,butpalpationofsynoviumaroundpatella andisquiteresistanttoanalgesia.Italmostalwaysresolvesspontaneously. disabling be very There after atrivialinjury. Itismostpainfulafterlongperiodsofsittingorondescendingstairs.can inadolescents(usuallywomen)often Anterior kneepainorchondromalaciapatellaedevelops diagnosed byMRI. (MacMurray’stestisnotreliable)anditbest There isnoreliabletestonphysicalexamination y 231

32: SPORTS MEDICINE ELECTIVE ORTHOPAEDICS Scenario 5 ➜ 6B 5F 4E 3C 2A 1D 232

3. Footinjur tender area. somecliniciansinjectsteroidsintothe condition butmaybehelpedwithheelcupsupports; afterlocaltraumaoritmayarisespontaneously.Plantar fasciitismaydevelop Itisaself-limiting to whichtreatmentisbest. heel. Treatment as isserialplastersorsurgicalrepairwithplasters.Therenoclearevidence sport thatcausedit. sport The Marchfractureisanotherstressandbestmanagedbyprolongedrestfromthe These areheelbumps,causedbyshoesrubbing.Changingfootwearcanimprovethings. best treatedsymptomatically,butthenecroticheadandremainingfragmentscanbeexcised. asFreiberg’s disease.Itis headisknown Spontaneous avascularnecrosisofthesecondmetatarsal heads. the tendermetatarsal Itisbesttreatedwithmodifiedfootweartoreducetheloadon patients withrheumatoidarthritis. Thiscanalsooccurin headswillproducemetatarsalgia. Repetitive bruisingofthemetatarsal distribution. the patientwithpermanentnumbnessinthatnerve leave oftheneuromabutthiswillalmost certainly totrauma.Thetreatmentisexcision secondary neuroma),inthiscaseprobably (Morton’s This isaneuromaformingintheinterdigitalnerve toe.Treatmenttoe. Itscommonnameisturf isrestandanalgesia ofthebig This isatearofthemetatarsophalangealjointcapsuleduetoforciblehyperextension y 33 The spine

Multiple choice questions ➜ Epidemiology, anatomy and (b) What physical tests (when present) physiology of back pain would help to exclude this diagnosis? A Testing for weakness of extensor hallucis 1. Which of the following statements longus regarding back pain are true? B Unilateral loss of ankle reflex A The chances of anyone having back pain C Saddle anaesthesia at some time in their life is around 50 D Loss of anal tone per cent. E Increase in pain. B Over 80 per cent of episodes of back pain settle within 6 weeks. (c) What action needs to be taken? C The radicular artery of Adamkiewicz A Analgesia and bed rest is the main blood supply to the lower B Immediate lumbar spine X-ray spinal cord. C Lumbar puncture D Onset of back pain after the age of 55 is D Urgent referral to a neurologist a ‘red flag’ sign. E Immediate referral to a spine surgeon. E ‘Yellow flags’ are worrying but less critical than ‘red flags’. ➜ Imaging in back disorders 3. Which of the following statements ➜ Case study – sudden onset about imaging are true? of urinary incontinence A Plain X-rays of the spine should only be taken if there is a history of trauma. 2. A 30-year-old nurse lifting in the B Magnetic resonance imaging (MRI) is the ward experiences sudden pain in her best way of visualising the disc and nerve back. She goes to Sister’s office to sit roots. down. The pain eases but now she C CT scan is best for looking for patients notices that she has difficulty standing with multiple vertebral body collapses and walking (her legs feel weak and secondary to myeloma. wobbly, and she has been incontinent D Bone densitometry is needed to of urine). diagnose osteoporosis. (a) What is the rare but very dangerous E Bone scintigraphy is a simpler method diagnosis which needs excluding? for measuring bone density. A Collapsed vertebral body secondary to ➜ occult myeloma Case study – collapsed B Central disc prolapse vertebrae C Transverse myelitis 4. A 70-year-old man who has smoked D Dissecting aneurysm blocking off the all his life now presents feeling unwell blood supply to the spinal cord. and with weight loss. He also has

233 ELECTIVE ORTHOPAEDICS ➜ ➜ . A40-year-oldlabourerpresentswith 6. A65-year-oldmanwhohaslivedall 5. (b) The man’slesionisthoughttobean (a)Whatinvestigationislikelytobemost 234

Case stud wakes atnight Case stud E D C B A E D C B A E D C B A Embolisation oftumours Embolisation Steroids rodstothespine Harrington Chemotherapy Radiotherapy options areavailable? Whattreatment (probably bowel). origin ofunknown adenocarcinoma Intravenous pyelogram. Barium mealandfollow-through oflung CT Barium enema biopsy Computed tomography(CT)-guided collapses? ofthe thecause useful foridentifying been sent. vertebrae. Bloodtestshavealready several collapsed of thespineshows Chest X-ray isnormalbutplainX-ray backache whichwakeshimatnight. on needlebiopsy? bodies. Whatwouldyouexpecttofind destruction intotheadjacentvertebral disc spaceatL2/L3withinvasionofthe destructionofthe sweats. X-rays show wakes himatnightandhehas thelegs.It does notradiate down and constantbackpainatL3,which his lifeinBritainpresentswithsevere diminished sensation overthedorsum extensor hallucis longus.Thereisalso on examinationhehasweakness in to thefoot.Hisleftfootdrags and theleftleg backache radiating down Sterile avascularnecrosis. Escherichia coli Mycobacterium tuberculosis cells chondrosarcoma Primary Tumour cells(metastases) y y –sciatica –painwhich ➜ ➜ ➜ 8. A young rower complainsofsudden Ayoungrower 8. spinalclaudication can How 7. (a)What isthelikelydiagnosis? A13-year-old femalepresents witha 9.

Case stud E D C B A Claudication E D C B A teenager Case stud D C B A B A calf. Whatisthelikelydiagnosis? calf. of thefootandlateral sideofthe claudication? be distinguishedfromvascular root. Neuroma ofanerve Infective discitis Facet jointarthritis root the L5nerve Prolapse oftheL5/S1discpressingon root the S1nerve Prolapse oftheL5/S1discpressingon equal andnormal. previously. Abdominalreflexes are rib hump.Shehashadnoproblems Muscle strain. Collapsed vertebra Spondylolisthesis disc Prolapsed intervertebral diagnosis? at L5/S1. Whatisthemostlikely lumbar spinewithasteppalpable There isconsiderable lordosisofthe deficit. and thereisnoneurological lifting. Thereisnoradiation ofthepain backpainwhenweight- onset oflow spinal decompression. Spinal claudicationcanbetreatedby percentofcases. more than50 percent;vascularprogressesin than 50 Spinal claudicationprogressesinless claudication. Distal pulsesarepresentonlyinspinal vascular inflexion. Spinal claudicationisworseinextension exercise. Only vascularclaudicationisinitiatedby Lipoma Neuromuscular scoliosis y y –ribhumpin –lowbackpain ➜ ➜ ➜ 3.B,D 2.(c)E 2.(b)C,D 2.(a)B 1. B,C,D Answers: Multiplechoicequestions (b) beusedtomeasurethe What testcan

Imaging inbackdisorders Case stud Epidemiolog E D C B A densitometry shouldbeused. densitometry method. Osteoporosis cannotbediagnosedreliablyon plainX-rayorbyscintigraphy,and sobone multiple myelomausingX-rayswould belargeandbonescintigraphywouldabetterscreening to imagebonecontoursiswithCT. theradiationdoseneededtosearchforlesionsof However, but thebestway roots,isbyMRI, tissues ofthespine,includingdisc, thethecaandnerve and/orweightloss.Thebestwaytoimagethesoft ofnight pain,fever and anyonewithahistory ofcancer, includes trauma,butalsopatients under20orover55,thosewithahistory thereisbackpainwith‘redflag’signs.This Plain X-raysofthespineareneeded whenever isimportant. decompression surgery by (toconfirm thediagnosis)followed and soanimmediatereferraltoaspinesurgeonforMRI thatthesoonercompressionisreleased,better chanceofrecovery There isevidence protrusion). roottrappedinthe lateralforamen(a unilateral signsareusuallyrelatedtoasinglenerve acentraldisc,and anal tone(whenpresent)arereliablesigns.Absenceofpaindoesnot exclude anaesthesia andlossof There arenocompletelyreliablesignsofcentraldiscprolapsebutsaddle transverse myelitisisnotusuallypainful. asthepatientis tooyoungand orananeurysm syndrome). Itisunlikelytobeoccultmalignancy The scenariodescribedcouldbethepresentationofacentraldiscprolapse(caudaequina treatment butismorelikelytobepsychologicalinorigin. signs arethosewhichwarnthatthebackpainisnotlikelytobeamenableconventionalsurgical aftertheageof55isonethese.‘Yellow onsetofbackache a non-benignprognosis.New flag’ ‘Redflag’signsarethosewhichassociatedwith ofAdamkiewicz. filled fromtheradicularartery whichismainly spinalcordissuppliedbytheanteriorartery butthelower radicular arteries, for anyaggressiveinvestigationortreatment.Thebloodsupplytothespinalcordismainlyby percentofepisodessettlespontaneouslywithin6weeksonsetwithouttheneed but over80 percent, and80 The chanceofsomeonehavingbackpainatsometimeintheirlifeisbetween60 E D C Coronal section MRI chest. Coronal sectionMRI Self-esteem questionnaire Rib X-rayswithintercostalmeasurement Cobb’s angleonaspineX-ray Lung functiontest severity oftheproblem? severity Undescended scapular. Idiopathic scoliosis Steroid-induced ribhump y –suddenonsetofurinar y , anatom y andph y (c) What method can beusedtopredict What methodcan (c) siolog E D C B A y Plasma calcium. Degree ofcurvature from parentalheight potentialascalculated growth Further Risser’s sign Age ofthepatient curve)? the prognosis(likelyprogressionof incontinence y ofbackpain 235

33: THE SPINE ELECTIVE ORTHOPAEDICS ➜ ➜ ➜ ➜ ➜ ➜ 9.(b)B 9.(a)D 8.(a)B 7. B,C,D,E 6. B 5. D 4.(b)B,D,E 4.(a)A 236

Case stud Case stud Claudication Case stud Case stud Case stud A CT-guided biopsywillgiveatissuediagnosisandthisinturnpointtopossiblesource. the maximum angle of the curvature ofthespineonananteroposteriorX-ray.the maximumangleofcurvature Lungfunctionis ismeasuredwithCobb’sangle.Thisameasureof ofanidiopathicscoliosiscurve The severity group. Anundescendedscapulawould havebeenpickeduplongbefore13. neuromuscular scoliosisunlikely. Shehasnotbeenonsteroidsandlipomasarerareinthisage make This isthenormalageandgenderforidiopathicscoliosis.The abdominal reflexes theleg(sciatica).Amusclestrainwouldnotproducea step inthespine. pain radiatingdown majortrauma.Heorshecouldhaveprolapsedadiscbut thentherewouldlikelybe without very the spineaswellalordosis.Ayoung,fitpersonwouldbemostunlikely tocollapseavertebra The mostlikelydiagnosisinthisagegroupisspondylolisthesisand doesproduceastepin decompression canoffersignificantreliefofsymptoms. Spinal claudicationonlyprogressesinaround20percentofcasesandifitdoes,spinal inextension). (whenthespinalcanalnarrows spinal claudicantsgetworsepaingoingdownhill while vascular claudicantsgetpaingoinguphill(whenthelegmusclesrequiremoreoxygen) butasageneralrule In bothspinalandvascularclaudication,painisinitiatedbyexercise, of thecalf.SothisisL5radiculopathy. Aneuromaisrareandnotalwayspainful. longus ispureL5,andthedermatomaldistributionofL5dorsumfootlateralside hallucis anddiscitisareunlikelytobethecause.Themotorsupplyextensor facet jointarthritis tothefootsuggeststhatitisradicularratherthanreferred,so The factthatthepainradiatesdown likely diagnosisis sothemost tumourosteonecrosisorTB, The lesionisaroundthedisc.Thiseffectivelyexcludes isplannedtodecompressthespineorremovetumour.probably onlybeattemptedifsurgery should withrods.Steroidsmighthelpthepainandembolisation indication forinternalfixation Adenocarcinomas areradio-resistantsochemotherapywillofferthebestpalliation.Thereisno information isavailable. 10 testsuntilmore orkidney withlung,bowel percent),sothereislittlepointinstarting breast (20percent),lung(15 percent),prostate,renal,gastrointestinalandthyroid(allunder percent)andthecommonestsecondariesare Most tumoursinthespinearesecondaries(98 y y y y y –ribhumpinteenager –lowbackpain –sciatica –painwhichwakesatnight –collapsedvertebrae E. coli discitis. 9.(c)B deformity is likely to be when growth endsandthedeformitystopsprogressing. deformity islikelytobewhengrowth the thedeformitywhenpatientfirstpresents,moresevere Obviously themoresevere (anddeformity)potentialthereisintheskeleton. muchmoregrowth therefore predictshow platesinthepelvisand to determineskeletalmaturityfromtheclosureofepiphysealgrowth islikely. growth Risser’ssignisthemethod tall andthereforeaconsiderableamountoffurther especiallyiftheparentsarevery willbecomesevere, and skeletally,themorelikelythatcurve sotheyoungerpatientpresenting,bothchronologically deteriorationofthecurve, and severe witha‘longwaytogo’isassociatedlikelyrapid growth rate atwhichitisoccurring.Rapid isanticipatedandthe muchmoregrowth dependsonhow The likelyprogressionofthecurve Self-esteem mayindeedbedamagedbutthereisnovalidatedmeasureinthesecases. commonly affectedinneuromuscularscoliosisbutrarelyidiopathicadolescentscoliosis. 237

33: THE SPINE 34 The upper limb

Multiple choice questions ➜ C Physiotherapy first but if that fails then 1. Which muscles control the rotator cuff? reefing of the capsule. A Supraspinatus 4. A child develops a habit (party trick) of B Teres major subluxing her shoulder backwards. C Teres minor D Infraspinatus (a) What type of dislocation is this most E Pectoralis minor likely to be? F Subscapularis. A Habitual B Traumatic anterior ➜ C Traumatic posterior 2. What benefits are offered by a D Atraumatic subluxation. shoulder replacement? (b) What is the best treatment? A Pain relief is guaranteed. A Physiotherapy and re-education B Range of movement will improve B Physiotherapy first but if that fails then dramatically. surgical repair of the Bankart lesion C An absence of the rotator cuff makes no C Physiotherapy first but if that fails then difference. reefing of the capsule. D An would give a much worse range of movement. 5. A rugby player puts his shoulder out E A partial shoulder replacement is also an falling in a scrum. option. (a) What type of dislocation is this most ➜ Case studies – shoulder likely to be? A Habitual dislocation B Traumatic anterior 3. A young woman notices that her C Traumatic posterior shoulder comes out of joint sometimes D Atraumatic subluxation. when she is asleep. She is able to put it back by jiggling her shoulder (b) What is the best treatment? carefully. A Physiotherapy and re-education B Physiotherapy first but if that fails then (a) What type of dislocation is this most surgical repair of the Bankart lesion likely to be? C Physiotherapy first but if that fails then A Habitual reefing of the capsule. B Traumatic anterior C Traumatic posterior ➜ D Atraumatic subluxation. 6. Which of the following deformities (b) What is the best treatment? are seen in a patient with chronic and A Physiotherapy and re-education severe rheumatoid arthritis? B Physiotherapy first but if that fails then A Swan neck deformity surgical repair of the Bankart lesion B Boutonnière deformity

238 ➜ Extended matchingquestions Extended (a) Choose andmatcheachoftheabove scenarioswiththelikelydiagnosis: (a)

E D C B A 9 8 7 6 5 4 3 2 1 1. Shoulderinjur K J I H G F E D C Prominent ulnarhead ofthewrist Ulnar deviation Frozen shoulder Rotator cuffimpingement Dislocating shoulder Acute massiverotatorcufftear Chronic rotatorcufftear shoulder. complainsofincreasingpain,stiffness andweaknessinthe A patientwithrheumatoidarthritis in theshoulderwhichlimitsmovementandpower. agirderandwrencheshisshoulder.A buildertripswhilecarrying pain Hepresentswithsevere and painfulwithagrazeoverthedeltoid.Allmovementsarerestricted. An elderlywomanrunningforchurchtripsandlandsonhershoulder. Theshoulderisswollen calcificationfloatingbetweentheacromionandhumeralhead. shows withoutmuchpain,butothermovementsarepainful,especially activeones.X-ray externally thatshecannotsleep.Sheisableto rotatetheshoulder anterolaterally andthepainissosevere A middle-agedwomangetssuddenonsetofpaininhershoulderafter notrauma.Sheistender presents withincreasingpainstiffnessandweaknessoftheshoulder.now fractureoftherightshoulderasayoungman intra-articular A patientwhosustainedasevere apprehensiveandasksthemtostop. liftsthearmupandbackhebecomesvery examiner better.back inagain.Itwasswollenandpainfulforsomeweeksbutisnow whenthe However, A youngmaninjureshisshoulderplayingrugby. Itfeelsasifitcameoutofplacebutwent movements arerestricted.Shecannotsleepforthepain. andsuddenonsetofpainintheshoulderafteraminortrauma.All severe A patientdevelops ofabduction. littlepower be very He canonlyabductitwithatrickmovementhunchinghisshoulder. thereseemsto Otherwise An elderlypatientpresentswithapainlessbutweakshoulder. Ithasgotworseovertheyears. andoncethearmisrightup,itnolongerpainful. much lesssevere, thepainis ifthearmisliftedbyexaminer, she triestoliftherarmfromside.However, A patientpresentswithgradualonsetofpainintheshoulder. when Thisisespeciallysevere tenosynovitis. Extensor joints anddislocationoftheMCP Subluxation joints metacarpophalangeal (MCP) ofthe deviation Radial contracture Dupuytren’s Trigger finger tendonrupture Extensor Heberden’s nodes y 7. Which of the following treatmentsgive Whichofthefollowing 7. G F E D C B A Tendon transfer Tendon repair ofthewrist Arthrodesis interphalangeal joints and Prosthetic replacementoftheMCP Prosthetic replacementofthewrist ofthedistalulna Excision of thehand? reliable resultsinrheumatoidarthritis 239

34: THE UPPER LIMB ELECTIVE ORTHOPAEDICS ➜ (b) (b) For eachscenario,choosethetest orinvestigationthatislikelytobemostusefulfor Choose andmatch eachoftheabovescenarioswithlikelydiagnosis: (a) For eachscenario,choosethemostappropriatetreatment: (b) 240

D C B A G F E D C B A 8 7 6 5 4 3 2 1 2. Elbowinjur H G F E D C B A H G F I Aspiration andculture Aspiration X-ray Injection oflocalanaesthetic andpronation Forced palmarflexion diagnosis: Osteochondritis dissecans Tuberculosis entrapment Ulnar nerve Olecranon bursitis oftheelbow Arthritis oftheradiohumeraljoint Arthritis fever. andalow-grade A patienthasachronichotswollenandpainfulelbow A patientpresentswitharedhotlumpoverthebackofelbow. stiffness intheelbow. manuallabour. Hedoesheavy presentswithpain,weaknessand manyyearsagonow A patientwhobroketheirelbow the medialsideofforearmintolittlefinger. presentswithpainandnumbnessdown manyyearsagonow A patientwhobroketheirelbow joint. repeated lockingoftheelbow presentswithsuddenand problemsnow ofelbow A youngpatientwhogivesnohistory bone destruction. andsignificant attendswithpainandweaknessintheelbow A patientwithrheumatoidarthritis supination. mainlyonpronationand attendswithpainintheelbow A patientwithrheumatoidarthritis the house. aweekendredecorating following A patientattendswithpainoverthelateralsideofelbow Rotator cuffrepair Collar andcuff. Analgesia andgentlephysiotherapy Advice andcommiseration;noactivetreatment Subacromial decompression Shoulder replacement Physiotherapy Steroid injection Osteoarthritis tendinitis Calcifying Fracture headofhumerus oftheglenohumeraljoint Arthritis y ➜ ➜ scenarios: Choose andmatchthecorrectdiagnosiswitheachoffollowing activities: ofgripwitheach of thefollowing Choose andmatchthecorrecttype For eachscenario,choosethetreatmentmostlikelytobeofbenefit inthefirstinstance: (c)

5 4 3 2 1 F E D C B A 4. Handinjur 5 4 3 2 1 E D C B A 3. Handgrip H G F E D C B A G F E passive flexion is painful, and pressing on the palmar side of the finger is very sore. is painful,andpressingonthepalmar sideofthefingerisvery passive flexion thefingerisredandswollen.Even The patientgotathornintheirfinger whengardening.Now The patient,whoisahairdresser,has achronicsinusintheinterdigitalcleft. their handitdoesnot‘pinkup’. open isoccludedwithpressure.Whenthey The patientclenchestheirfistwhile theradialartery smoothly, whereasontheotherhanditsticks. noticesthatasapenisstrokedacrossthepalmofthispatient’s hand,itruns An examiner makeafistonefingercrossesandtangles withtherest. A patientcomplainsthatwhenthey damage Nerve Disjunction ofthepalmararchonulnarside tendonsheathinfection Flexor Pilonidal sinus Malunion ofanobliquefracturetheproximalphalanx hood Rupture oftheextensor Holding acreditcardasyouputitintocashmachine asuitcase Carrying Picking uploosechange Holding ahammer Holding apen Chuck Key Hook Power Pinch Appropriate antibiotics Intra-articular decompressionwithpossibletransposition Nerve Arthrodesis Time RICE Total replacement. elbow oftheradialhead Synovectomy andexcision Formal clinicalexamination Arthroscopy Electrical conductionstudies y 241

34: THE UPPER LIMB ELECTIVE ORTHOPAEDICS ➜ ➜ ➜ ➜ 3.(a)D;(b)A 2.A,E 1. A,C,D, F Answers: Multiplechoicequestions Choose and match the correct condition with each of the following descriptions: Chooseandmatchthecorrectconditionwitheachoffollowing 242

Case studies–shoulder dislocation Shoulder replacement Rotator cuff 7 6 5 4 3 2 1 G F E D C B A 5. Otherconditionsinthehand 6 and re-education appear togiveareasonablechance ofsuccess. shewouldnotfinditso easytogetbackintoposition.Physiotherapy dislocation), asotherwise (justapartial There isnotraumainvolvedandthis islikelytobeanatraumaticsubluxation results. (hemi)jointreplacement willalsogivegood apartial unlikely). Iftheglenoidiswellpreserved, time (whichis replacement muchlessreliableunlessthecuffcanberepairedatsame beused.Theabsenceoftherotatorcuffmakes shoulder the scapulothoracicjointcannow a betterrangeofmovementthanreplacement,asthefull availablein oftenresultsin movement. Indeed,thepainreliefprovidedbyaglenohumeralarthrodesis Shoulder replacementgivesreliablepainreliefbutoftendoesnotincrease shoulderrangeof infraspinatus, subscapularisandteresminor. intotherotatorcuffandcontrolit:supraspinatus, There arefourmuscleswhichinsert the skin. butnotto A lumponthepalmarsideofwristwhichisattachedtojointcapsulebelow Spontaneous avascularnecrosisofthelunatebonediagnosedonMRI. Wasting ofthehypothenareminence. outofbed;clumsinessinthefingers. Painful wakeningatnight;havingtoholdthehanddown pollicis. Inflammation ofthetendonsheathsabductorpollicis,andextensor A handwhichlookscompletelynormaluntilthepatientmakesafist. tools andalcoholiccirrhosis. butalsoassociatedwiththeuseofvibrating An autosomaldominantcommoninAnglo-Saxons tunnelsyndrome Carpal Trigger finger Keinbock’s disease contracture Dupuytren’s Ganglion compressioninGuyon’scanal Nerve tenosynovitis de Quervain’s ofthedistalinterphalangealjoint. interphalangeal jointandhyperextension oftheproximal hasapeculiarshapedfingerwithflexion A patientwithrheumatoidarthritis ➜ ➜ (b)Treatments –1D,2E,3F,4B,5C,6A,7G,8H,9C (a)Diagnoses–1D,2A,3E,4C,5I,6H,7G,8B,9F matchingquestions Answers: Extended 7. A,B,D,E,G 6.A,B,D,E,H,J,K 5.(a)B;(b)B,C 4.(a)A;(b)A

1. Shoulderinjur Rheumatoid arthritis re-education should be sufficient as the child will grow out of it if they stopdoingit. outofitifthey re-education shouldbesufficientasthechildwillgrow ofdislocation.Onceagain,physiotherapyand This isahabitualdislocation.Therenohistory to simply explain the diagnosis and commiserate with them. If, however, thetearispost-traumatic thediagnosisandcommiseratewith them.If,however, to simplyexplain a poorbloodsupplyandsoanyattempt atrepairisdoomedtofailure.Itmaythereforebebest gotamassivetearoftherotatorcuff.Thecuffhas overtheyearshasalmostcertainly developed too respondwelltoashoulderreplacement. andthey inpatientswithrheumatoidarthritis, intheshoulder alsodevelops replacement. Arthritis with gradualonsetofpain,stiffnessandweakness,thebesttreatment isprobablyashoulder intheyearsto come. Thiswillpresent isinevitable used tohappen),thentraumaticosteoarthritis fractureisleft(as maybetheonlyoption.Ifintra-articular is damaged),ahemiarthroplasty orgrosslydisplaced(sothatthebloodsupplyto humeral head itisintra-articular however, thenitcanbe managedinacollarandcuff.If, osteoporotic bone.Ifthefractureisextra-articular of thelabrumandcapsule. of themusclesaroundshoulder.the strength Repeated dislocations willneedasurgicalrepair shoulder thathasdislocatedanteriorly. Inthefirstinstance, treatmentisphysiotherapytobuild theclassicpatternofa rotated shows andexternally apprehensive whenitisabducted,extended symptoms (traumaticorgradual)whichgiveacluetothediagnosisinmostofconditions. It isthepatternofthesethree,ageandgenderpatient,causeonset ofthethree. intheshouldercanconsistofstiffness,painand/orweaknessoramixture Disability tendon transferusinghealthytendonsgivesbetterresults. andinterphalangealjointsisprovingvaluable.Tendonthe MCP repairstendnottoworkwellbut prostheticreplacementof However, as valuableintermsofpainreliefandfunctionarthrodesis. but mayproducesomewristinstability. Prostheticreplacementofthewristisnotcurrentlyproving ofthedistalulnaalsohelps Synovectomy givesgoodpainreliefandimprovesfunction.Excision isradial. deviation condition altogether. jointsisulnar,notradial,whereasinthewrist the oftheMCP Thedeviation contractureisaseparate Dupuytren’s Heberden’s nodesareassociatedwithosteoarthritis. However, There arealargenumberofdeformitiesassociatedwithrheumatoidarthritis. percentofcases. provide goodresultsinover90 lesionandreefingofthecapsule will torepairtheBankart not settleandrecurs,thensurgery The mostcommonformoftraumaticdislocationistheanteriordislocation.Ifdoes The patientpresentingwithapainless butweakshoulder(Scenario2)wheretheproblemhas The womanwhohasfallenontohershoulder(Scenario7)willhavesuffered afracturethrough The patient(Scenario4)whofeelshisshoulerhas‘come out’ andgonebackinwhois y 243

34: THE UPPER LIMB ELECTIVE ORTHOPAEDICS ➜ (c)1C,2A,3B,4G,5F,6E, 7D,8H (b)1A,2B,3C,4C,5E,6C,7G,8D (a)1A,2B,3C,4G,5E,6C,7D,8F 244

2. Elbowinjur theimpingementandthereforepain. should relieve tipoftheacromion(subacromialdecompression) trimmingofthedownward-pointing However, local anaestheticintotheimpingingarea,andsomeattempttreatmentwithasteroidinjection. cuff impingementundertheacromion.Thisconditioncanbediagnosedwithaninjectionof quite suddenlyabletoabductthefinalbitwithoutmuchpain.Thisischaracteristicofrotator are liftsthearmforpatientthroughthispainfularc,they examiner (Scenario 1).If,however, painful ofthepainfularc)becauseitbecomesvery point(thestart acertain their sidebeyond supraspinatus tendonbetweentheacromionandhumeralhead. rotation.OnX-rayacalcificcloudwillbevisibleinthe tendinitis usuallydoesnotaffectexternal rotation.Calcific shoulder producesglobalpainandlossofmovement,includingonexternal a frozenshoulderbutcouldalsobecalcifyingtendinitisofthesupraspinatustendon.Frozen months ofpainandstiffness,butmostclinicianswouldofferanalgesiagentlephysiotherapy. helpsreducethe a frozenshoulder. toresolveanditisdoubtfulwhetheranything slow Thisisvery isnotacceptabletothepatient. supply intheyoungerpatientanddisability and inayoungerpatient,thenrotatorcuffrepairshouldbeattempted,asthereissomeblood nerve entrapment will need release and transposition of the ulnar nerve. Tuberculosis entrapment willneedreleaseandtransposition oftheulnarnerve. nerve ofthe removal oftheloosefragmentin firstinstance.Patientswithsignsof ulnar by arthroscopic replacement.Osteochondritisdissecanscanbemanaged willlastbetterthanan elbow arthrodesis labour,an heavy replacement isindicated.Ifthepatientperforms joint isinvolved,atotalelbow ifthewholeelbow joint.However, pain reliefwithoutcreatingtoomuch instabilityintheelbow oftheradialheadshouldgivegood injection oflocalanaesthetic,thensynovectomy andexcision by as demonstratedbypainonpronationandsupinationrelieved isolated radiohumeralarthritis, has Localsteroidinjectionmayalsobeused.Ifapatientwithrheumatoid arthritis elevation). (rest,ice,compression, and andolecranonbursitisisRICE The treatmentfortenniselbow arthritis. Tuberculosisexamination. joint.Itproduces anindolentseptic hasapredilectionfortheelbow studies shouldconfirmthediagnosis.Olecranonbursitiscanbediagnosed bygoodclinical joint. Electricalconduction whereitpassesbehindtheelbow and compressionoftheulnarnerve canleadtoirritation inthe elbow aswellosteochondritisdissecans.Arthritis and osteoarthritis this jointanddemonstratingpainrelief.PlainX-rayisthemostusefuldiagnostic testinrheumatoid diagnostic oftenniselbow. paincanbediagnosed byinjectinglocalanaestheticinto Radiohumeral originandis withpronationputsspecific loadontothecommonextensor Forced palmarflexion tuberculosis mustbeinthedifferentialdiagnosis. ismorelikelytobeinfection,andintheelbow fever jointwithalow-grade and painfulelbow islikelytobeolecranonbursitis,whereasahot oftheelbow surface hot lumpovertheextensor Aredand ifitisnotpossibletogetanatomicalreductionofthejointsurfaces. traumatic arthritis willgoontoaggressive breakingoffspontaneously).Fractures intotheelbow surface articular oftraumaisthetypicalpresentationosteochondritisdissecans(afragment a history it thengoesontoaffecttherestofelbow. without Suddenonsetoflockingintheelbow isfrequentlyattackedfirstintheradiohumeraljoint.However, theelbow rheumatoid arthritis, Tennis activity. afteranyheavy In afterplayingtennisbutcanstart doesnotonlydevelop elbow The presentation of severe painandstiffnessafterminortrauma(Scenario3)ischaracteristicof The presentationofsevere Painful arcsyndromeisthefindingthatpatientcannotactivelyabducttheirarmfrom rapidlywithoutanytraumaatall(Scenario3)couldbe painandstiffnessdeveloping Severe y ➜ ➜ ➜ 1D,2F,3A,4G,5B,6E,7C 1B,2F,3E,4C,5D,6A 1E,2B,3A,4C,5D

5. Otherconditionsinthehand 4. Handinjur 3. Handgrip joint maybetheend-point. ofthe a considerabletimetoensurethattheinfectioniseradicated.Eventhen,fibrousankylosis jointwillneedtheappropriateantibiotics(checkorganismssensitivity)tobegivenfor elbow y 245

34: THE UPPER LIMB 35 The hip and knee

Multiple choice questions ➜ Anatomy and physiology of ➜ Radiological features in the hip osteoarthritis 1. Which of the following are involved 4. Which of the following are radiological in blood supply to the adult femoral features of osteoarthritis of the hip? head? A Subchondral sclerosis A Artery of the ligamentum teres B Multiple microfractures B Retinacular branches of the medial C Subchondral cysts circumflex femoral artery D Coarsening of the trabecular pattern C Capsular branches of superior gluteal E Narrowing of the joint space artery. F Osteophyte formation G Periarticular osteoporosis. 2. Which of the following structures are involved in static stability of the hip ➜ Total joint? 5. If you had to explain to a patient the A Capsule complications of total hip replacement, B Labrum which of the following would you C Gemelli mention? D Pectineus A Infection E Iliopsoas B Deep vein thrombosis F Ligamentum teres C Renal failure G Anterior inferior iliac spine D Urinary tract infection H Cup and socket shape of hip joint E Nerve damage I Abductor muscles F Synergistic gangrene J Hamstrings G Chest infection K Gluteals. H Stroke 3. Which of the following structures are I Dislocation involved in supporting the pelvis when J Fracture standing on one leg? K Severe bleeding requiring transfusion A Capsule L Death from any cause less than B Labrum 1 per cent C Gemelli M Leg length inequality more than 10 cm D Pectineus N Compartment syndrome. E Iliopsoas F Ligamentum teres ➜ Knee stability G Anterior inferior iliac spine 6. Which of the following are dynamic H Cup and socket shape of hip joint stabilisers of the knee? I Abductor muscles A Biceps femoris J Hamstrings B Anterior cruciate ligament K Gluteals. C Posterior cruciate ligament

246 ➜ ➜ Extended matchingquestions Extended scenarios: Choose andmatchthecorrecttreatmentwitheachoffollowing factors: Choose andmatchthecorrectconditionwitheachoffollowing

6 5 4 3 2 1 F E D C B A 2. Managementofhipconditions 4 3 2 1 C B A 1. Aetiolog J I H G F E D Adductor longus. Quadriceps femoris A 75-year-old patient has a severely displacedsubcapitalfractured neckoffemur.A 75-year-oldpatient hasaseverely sleep. ofthehip,limitingwalkingandpreventing osteoarthritis hassevere A patientof80 sleep. ofthehip,limitingwalkingandpreventing osteoarthritis hassevere A patientinherearly50s bearing areaofthefemoralhead. diseasehasasmallarea ofcollapseimmediatelyinthemainload- An adolescentwithPerthes’ fracture. toanacetabular A youngpatientwhoisamanuallabourerhaspainfulandstiffhipsecondary of deadbone. areas several shows in sclerosisbutnolossofjointspace.Magneticresonanceimaging(MRI) aslightincrease A patientpresentswithreductionofmovementthehip.X-raymerely shows Arthrodesis Core grafttohip Conventional stemmedhipreplacement Hemiarthroplasty hipreplacement Surface disease Perthes’ Slipped upperfemoralepiphysis Genetic predispositionorfemoro-acetabularimpingement Alcohol ortreatmentwithhigh-dosesteroids osteoarthritis Secondary Avascular necrosis Idiopathic osteoarthritis Lateral Medial meniscus Semitendinosus Gracilis Sartorius y ofhipproblems ➜ 7. Which of the following are indications areindications Whichofthefollowing 7.

G F E D C B A Arthroscop Relieve ajointeffusion. Relieve Decompress Osgood–Schlatterdisease Repair cruciateligamentrupture Remove loosebodies Repair rupturedpatellatendon Check foravascularnecrosis Diagnose andtreattornmeniscus for arthroscopyoftheknee? y oftheknee 247

35: THE HIP AND KNEE ELECTIVE ORTHOPAEDICS ➜ ➜ ➜ 2.A,B,F,H 1. B Answers: Multiplechoicequestions scenarios: Choose andmatchthecorrecttreatmentwitheachoffollowing scenarios: Choose andmatchthecorrectdiagnosiswitheachoffollowing 248

Anatom 4 3 2 1 D C B A 4. Managementofkneeconditions 4 3 2 1 D C B A 3. Hipreplacement teres alsoprovidessomestability. margin oftheacetabulum.Astrong fibrouscapsuleenclosesthewholehipjoint.Theligamentum labrum(lip)aroundthe stabiliser,asisthecartilaginous of thehip(ballandsocketisanimportant ofthehipcannotincludemuscles (thesearedynamic).Theanatomicalshape The staticsupports fracturedneckoffemur.capsule andsoaredisruptedinan intra-articular arecloselyattachedtothe periosteumwithinthejoint factbecausethesearteries is animportant This artery. ligamentum teres,thebulkcomesinretinacularbranchesofmedial circumflex little comesthroughthe The bloodsupplytothefemoralheadisparlous.Inadult,very infected again. A 70-year-old andhasbecome hasaninfectedkneereplacementwhichbeen revised oftheknee. idiopathicarthritis A 65-year-oldpatienthastricompartmental osteoarthritis. A 65-year-oldpatienthasidiopathicunicompartmental pain,agoodrangeofmovementandsignificantvarusdeformity.has severe fracture.He afteranintra-articular osteoarthritis A 20-year-oldpatienthasmedialcompartment oftheknee Arthrodesis kneereplacement Unicompartmental High tibialosteotomy Total kneereplacement paininwhatwasapainlesship. has severe A patientswhohadahipreplacementsomeyearsbeforestumbledonstepsandnow toclunk. which isalsostarting A patientwhohadahipreplacement15 complainsofincreasingpaininthehip, yearsagonow the femoralcomponent. alucentlineappearingaroundthecementof reallysettled.X-rayshows pain inthehipnever some6monthsbeforecomplainsthatthe A patientwithatotalhipreplacementperformed hip. in hisnew pain feltasevere On gettingoutofbed,apatientwhohadhipreplacement4dayspreviously Infected implant Periprosthetic fracture Dislocation loosening Aseptic y andph y siolog y ofthehip ➜ ➜ ➜ ➜ ➜ 1B,2A,3C, 4C matchingquestions Answers: Extended 7. A,D,E 6.A,D,E,F,I 5.A,B,E,I,J,K,L 4.A,C,E,F 3.H,I

1. Aetiolog Arthroscop Knee stabilit Total hipreplacement Radiological features inosteoarthritis able tostandononelegeasily. the cupandsocketofhipjoint.Bothmustbepresentworkingnormallyforapatientto thepelvisagainstfulcrumof whichsupports power The abductormusclesprovidetheleverage necrosis. Both slipped upper femoral epiphysis (SUFE) and Perthes’ disease inchildhoodlead andPerthes’ necrosis. Bothslipped upperfemoralepiphysis(SUFE) Both alcoholand treatment withsteroidsareassociated withanincreasedincidenceofavascular not needanykindofsurgery. Osgood–Schlatter disease(swellingandinflammationofthetibialtubercle inadolescents)does problemssuchasarupturedpatellatendonare besttreatedopenwhile aspiration. Extra-articular point indrainingajointeffusionthroughthescope,asthiscanbedone moresimplybyneedle Thereislittle throughthe arthroscope. the knee.Evencruciateligamentrepairscanbeperformed torn meniscuscanbediagnosed,trimmedorrepaired,whileloosebodies canberemovedfrom So,a arthroscopist. bemanagedbyanexpert conditionsinthekneecannow Most intra-articular actontheposteromedialside.Quadricepsfemorisstabilisesfromfront. sartorius biceps stabilisetheposterolateralside,whilesemimembranosus,semitendinosis,gracilisand The dynamicstabilisersofajointarethemuscleswhichactacrossthatjoint.Inthiscase, more than2cm;10 cmwouldbemostunlikely. inequalityisacommonproblembutrarely 1percent.Leg-length the incidenceiswellbelow but theimplant(<1percent).Deathisapossibilityinanymajorsurgery complication ofinserting occurs inaround5percent,especiallytheearlystages,andfractureisararebutserious should thepossibilityofsignificantbloodlossrequiringtransfusion(5–10 percent).Dislocation occursin1–5percentofcasesandsoshouldbementioned,as especially thesciaticnerve, aroundthehip, are allpossiblebutrarecomplicationsofanymajorsurgery. Damagetonerves syndromeorstroke,althoughthese infection, synergisticgangrene,chestcompartment tract in around5percent.Renalfailureisnotassociatedwithhipreplacement,norareurinary of casesingoodunits.Deepveinthrombosis(withsignificantclinicalproblems)probablyoccurs than 1percentincidencewhenobtainingconsentfromapatient.Infectionoccursin1–2 complicationsandanyotherwithmorecommon It isnormaltomentionallsevere Microfractures cannotbeseenonX-ray. andcoarseningofthetrabeculaeisfoundin Paget’sdisease. arthritis usually seenininflammatory osteoporosisis formation.Periarticular ofthejointspaceandosteophyte narrowing and cysts, ofthehip:subchondralsclerosis There arefourcharacteristicfeaturesofidiopathicosteoarthritis y y oftheknee ofhipproblems y 249

35: THE HIP AND KNEE ELECTIVE ORTHOPAEDICS ➜ ➜ ➜ 1B,2C,3A, 4D 4C 3A 2D 1B 6B 5C 4A 3F 2E 1D 250

4. Managementofknee conditions 3. Hipreplacement 2. Managementofhipconditions predisposing factor. genetic componentbutithasalsobeensuggestedthatfemoro-acetabularimpingementisa ofthehipisthoughttohaveasignificant Idiopathicosteoarthritis congruity ofthejointsurfaces. laterinlifeprobablybecauseofdamagetothe osteoarthritis to anincreasedriskofsecondary to withstand the loads put on it for very long.Ifthere isagoodrangeofmovement,then ahigh to withstandtheloads putonitforvery young, high-activitypatient,akneereplacement shouldbeavoidedasitisunlikely In thevery around thejointreplacement(stress raisers)andthebonecanfracture. loadscanbeplacedonthebone If patientstriporfallwithatotalhip replacement,extreme natural lifespan.Thisisasepticloosening. clunkafter15 togivepainandeven If thehipisstarting yearsthen it hascometotheendofits investigating, especiallyifthereareradiologicalsignsofprematureloosening. infection andsoneeds Pain persistingfor6monthsafterahipreplacementmaybelow-grade trouble. yet re-formedaroundthehip.Ifhipisputstraightback,itmaynot causefurther tomobiliseandthecapsulehasnot firststart Dislocation ismostcommoninpatientswhenthey head andsowillrequireahemiarthroplasty. haveadeadfemoral An elderlypatientwithagrosslydisplacedsubcapitalfracturewillcertainly andwithreliableresultswhichwilllastthelifetimeofpatient. to perform An olderpatientmightbebetterofferedaconventionalstandardtotalhip,whichisrelativelyeasy inthefuture. more optionsforrevision replacementasthisremovestheminimumofboneandthereforemaygive offered asurface manuallabourer)couldbe demand(notaheavy andlow A youngpatientwithhiparthritis rotation osteotomy. Themethodofdoingthisisa out oftheload-bearingareaandreplacingwithhealthycartilage. diseaseneedsthatdamagedareamoving A patientwithalocalareaofcollapsefromPerthes’ the hip.Thiscanalwaysbeunpickedandreplacedwithtotalhipreplacementlaterinlife. not haveatotalhipreplacementasitwillfailrapidly. (fusion)of Thebestoptionisanarthrodesis Young manuallabourshould ofthehipwhoareinvolvedinheavy arthritis patientswithsevere is forage,drillingoutacoreofboneandreplacingitwithvascularisedgraft. The presentationischaracteristicofavascularnecrosisintheearlystages.treatmentchoice standard totalkneereplacementistheoperationofchoice. a osteoarthritis, total kneereplacement.For theelderlypatientwithconventionaltricompartmental comparedwithaconventional toperform knee replacement,althoughtheseoperationsaretricky (eitherthemedialorlateral)shouldbeconsideredforaunicompartmental one compartment uncontrollable infection). (suchasin cannotbeperformed amputation, ifatotalkneereplacementhasfailedandrevision from isalsotheonlyoption,apart wouldbeabetteroption.Arthrodesis then anarthrodesis thekneewaspainfulandrangeofmovementminimal the unaffectedside.If,however, It candothisbychangingthealignmentofkneesothatbulkloadgoesthrough sideofthejoint. tibial osteotomywillretainthatrangeofmovementwhileunloadingthearthritic A patient who has more than 20 years’ life expectancy and in whom there is only arthritis in andinwhomthereisonlyarthritis A patientwhohasmorethan20years’lifeexpectancy 251

35: THE HIP AND KNEE 36 The foot and ankle

Multiple choice questions ➜ Foot – general non-operative possibilities have been 1. Which of the following statements are exhausted. true? C is a good option for A The talus is narrower posteriorly so that arthritis of the tibiotalar joint in the low- as the foot comes into dorsiflexion the demand patient, providing there is not talus locks in the mortice between the severe deformity. medial malleolus on the tibia and the D Arthrodesis is the treatment of choice for lateral malleolus on the fibula. The foot an ankle joint which has severe arthritis then rotates externally. and which is painful in a high-demand B The subtalar joint is responsible for patient. inversion and eversion of the hindfoot. E Osteophyte removal is a good alternative C The third metatarsal head is recessed to in the painful ankle where it can be act as a ‘keystone’ in the transverse arch. shown that impingement is causing pain D The windlass test is performed to check and there is no significant osteoarthritis. the integrity of the flexor muscles in the F When only one joint of the hindfoot sole of the foot. complex is involved, it is nevertheless E The skin on the dorsum of the foot best to perform a triple fusion because is mainly supplied by the superficial all the joints are interrelated. peroneal nerve. G Rupture of tibialis posterior leads to a valgus flatfoot, which is painful. ➜ Midfoot and hindfoot H Pes cavus is a common presentation of a neurological condition such as Charcot– 2. Which of the following statements are Marie–Tooth or of diastomatomyelia. true? I Osteomyelitis in the diabetic foot does A Surgery to deal with deformity in the not usually spread and so can be treated knee should only be undertaken once with minimal local interference. problems of pain, stiffness and deformity Charcot joints tend to be painful, with in the foot have been resolved. J X-ray changes minimal compared with B Surgery in the foot should be undertaken the pain experienced by the patient. early and aggressively, not once all Extended matching questions ➜ 1. Diagnosis in foot problems 1 A child is born with one foot plantarflexed, inverted and supinated. The position can almost be corrected but reverts to its original position as soon as the foot is released 2 A teenager presents with pain in the foot. On examination there is a flatfoot which does not form an arch when he stands up on tiptoe.

252 (b) For eachscenario,choosetheinitialmanagement: (b) Choose andmatcheachofthesescenarioswiththelikelydiagnosis: (a) H G F E D C B A L K J I H G F E D C B A 9 8 7 6 5 4 3 Non-operative measures, then surgery iftheproblem doesnotsettle. Non-operative measures,thensurgery Serial splints Reassurance thatthiswillusuallyimprove withtime theforefoot,straighten thetoeandremoveprominentlump An osteotomytonarrow Reassurance thatthisisnotabnormalandnotreatmentneededorappropriate andfusionofthejoint Removal oftheprominentbone(osteophyte) toreleasetightstructuresandtransfertendons Soft-tissue surgery structures, ifappropriate limb,thenreleaseoftight byphysiotherapytoassessthewholelower A carefulreview Freiberg’s disease Tight tendoAchilles Tarsal coalition Hallux rigidus Physiological flatfoot Talipes equinovarus neuroma Morton’s Mallet toes Curly toes Hammer toes Claw toes Hallux valgus the cleft. toes,andespeciallyin numbness inherfootlocalisedaroundthebaseofthirdandfourth A youngwomanwholikeswearingtight,fashionableshoescomplainsofincreasingpainand withflatteningofitshead. head ofthesecondmetatarsal ahighlyabnormal A teenagerpresentswithpainatthebaseofsecondtoe.X-raysshow thetoescanstillbestraightenedout. rub onhisshoes.However, highandthetoesbentupsothatthey presents withproblemsinhisfeet.Thearchesarevery limbs ofaprogressiveneurologicaldisorderaffectingthelower A teenagerwithafamilyhistory painful movement.Thepatientwantstobeablecontinuerunning. limitedrangeof andhasavery years ago.Thefirstmetatarsophalangealjointisreddorsally A middle-agedrunnercomplainsofincreasingpaininhisrightbigtoewhichhebrokemany minutes. formorethanafew comfortably the unsightlinessofthisbumpandfactthatshecannolongerwearhigh-heeledshoes distressedby lump overthemedialsideofbigtoemetatarsophalangealjoint.Sheisvery dressedpresentswithapainfulred A middle-agedwomanwhoseworkinvolvesbeingsmartly on thesolesofherfeet(plantigrade). A childwithcerebralpalsyistoe-walking.Shereferredtoyouforhelpingettingherwalk flatfooted. Sheisnotinpainandwhenshestandsontiptoe,anarchforms. A childaged10 thatsheis isbroughtalongbyhermotherbecausedancingteachersays 253

36: THE FOOT AND ANKLE ELECTIVE ORTHOPAEDICS ➜ ➜ 2.C,D,E,G,H 1. A,B,E Answers: Multiplechoicequestions 254

Midfoot andhindfoot Foot –general amputations. treatmentwill merelyconsistofasequenceever-higher necrotic andinfected tissue,otherwise wouldsuggest.Surgicaltreatment needstobeaggressivefindandremoveall examination clinical thansuperficial indolent initsspread,whichmaybe muchfasterandmoreextensive always leadtofullrecovery. Thisiscalled adiastomatomyelia.Releaseofthebardoesnot equina astheteenagergrows. occulta mayleaveabarofboneorscartissueacrossthespinalcanalwhich trapsthecauda of aprogressiveneurologicaldisordersuchasCharcot–Marie–Tooth. Alternativelyaspinabifida reconstitute thearchshouldbeconsidered. sorepairofthetendonor atendontransferto can onlyofferlimitedsymptomaticsupport, and leadstoapainfulrapidlyprogressiveflatfootwithvalgushindfoot. Modifiedfootwear joints isaviableoptionforstiffandpainfulhindfoot. nottobethecase,sosinglefusion ofaffected known deterioration intheotherjoints.Thisisnow pain andrangeofmovement.Thiswillnotworkifthereissignificantarthritis. removalmayproduceasignificantimprovementin thepain),simpleosteophyte injection relieves significant demandsonthelimb(buildersetc.) remains theoperationofchoiceforpatientwithsignificantdeformityorwhoisgoingtomake deformity.put highloadontheiranklesandinwhomthereisnotsevere arthrodesis However, isconsidered. recover. beforesurgery Therefore,allnon-operativeoptionsshouldbeexplored knee beforeembarkingoncorrectionsinthefoot. outanyproblemsinthe tosort attempts toputthingsrightinthefoot.Itisthereforeimportant Deformities inthekneeproducehighlyabnormalforcesfoot,whichwillcompromiseany the foot. supplyingthemedialsideof nerve supplyingthelittletoeanditsbase,saphenous nerve becomes morepronounced. fascia (notthemuscles),andifthisarchoffootareintact,longitudinal arch. arch. occurinthesubtalarjoint.Themidfoothasalongitudinalandtransverse inversion andeversion locking itintoposition. rotatingthefibulaandfoot externally thetalusjamstightlyinmortice, dorsiflexion, foot.In mobilitytotheplantarflexed increasedrotatory posteriorly,allowing The talusisnarrower The skin on dorsum of the foot is supplied by the superficial peroneal nerve, withthesural peronealnerve, The skinondorsumofthefootissuppliedbysuperficial The ‘windlass’testconsistsofaskingthepatienttostandontiptoe.Thistightensplantar tothistransverse headisrecessedtoactasakeystone The second(notthethird)metatarsal intheankle(tibiotalar)joint,while occuralmostexclusively andplantarflexion Dorsiflexion Osteomyelitis inthediabeticfootmay beindolentinitspresentationbutthereisnothing ofpescavusinateenagerisanominoussignandusually thepresentation The development women Rupture ofthetibialisposteriortendorismostcommoninmiddle-aged overweight It usedtobethoughtthatfusionofonlyonejointinthehindfootmerely producedarapid canbeseen,whichisimpinging(localanaesthetic In ankleswhereaprominentosteophyte givinggoodlong-termreproducibleresultsinpatientswhodonot Ankle replacementisnow whenitdoes,takesalongtimeto andeven The footdoesnotrespondwelltosurgery ➜ Scenario 5 Scenario 4 Scenario 3 Scenario 2 (b)Treatments Scenario –1G,2H,3D,4A,5E,6C,7B,8F,9H (a)Diagnoses–1G,2J,3H,4K,5A,6I,7B,8L,9F 1 matchingquestions Answers: Extended

1. Diagnosisinfootproblems metatarsal headtoremovethehalluxvalgus. Finallythebunionitselfisremoved. metatarsal time thebigtoeisstraightenedon and correctthemetatarsus primusvarus.Atthesame theforefoot tonarrow isperformed Anosteotomy of thefirstmetatarsal behind mostisthesame. The numberofoperationsdescribed suggestthatnonearecompletelysuccessful.Buttheprinciple istheonlyoption. itisprogressiveandsurgery progress, butoncethedeformityhas developed, earlystages,then,carefulchoiceoffootwearmaydelay of thesecondandthirdtoes.In very over theunderlyingbone.Thebigtoe isforcedlaterally(halluxvalgus)andmaycauseoverlap andan osteophyte headcreatesredthickenedskin,abursa medial sideofthefirstmetatarsal shoe.Rubbing onthe Bunions resultfrompressurebetweenawideforefootandnarrow ‘nonnocere’. isundertaken physiotherapy assessmentofthechild’sgaitbeforeanysurgery thatthereisacareful stand plantigradebutthekneewillthencollapse.Itisthereforeimportant withoutthinkingaboutthis,thechildmaybeableto them. IfthetendoAchillesislengthened theknee fromcollapsingunder sostandingontiptoehelpsthemprevent knee andhipextensors, toremainstanding.Theirproblemsmaybecompoundedby weakoruncoordinated strength deformitiesofbothkneeandhip,so needconsiderable mayhavefixed-flexion walk. They Achilles, butalsobecausetheloadstransmittedthroughtotibiahelp themtostandand often haveatighttendo becausethey Children withcerebralpalsyfrequentlytoe-walk,partly appropriate. as thereisnothingwrongwiththechild,soreassurancethatthisanormalvariantall areofnovaluehere,especially he standsontiptoethearchre-forms(windlasstest).Orthotics This canbedifferentiatedfromapathologicalflatfootbecauseitispainless,andwhen havea‘physiological’flatfoot. areflatfootedsimplybecausethey Many childrenaretoldthatthey coalition. inthenon-unionoftarsal repeatedstressfracturesandarthritis needed toprevent ifitdoesnotsettle,aformalfusionmaybe is non-operativeassomeofthesewillsettle.However, coalitioninthefoot.Infirstinstance,treatment abnormalitysuchasatarsal that thereisasevere A flatfootwhichdoesnotformanarchwhenthepatientstandsontiptoes(windlasstest)indicates tosurgery. without theneedtoresort bepositionalintheuterus,andserialsplintingmaygiveagoodresult the deformitymay,inpart, thefootiscorrectablethen willbetheonlyoption.If,however, underlying problemandsurgery adductedandsupinated.Ifthedeformityisrigid,theremaybeasevere inverted, is plantarflexed, isclubfoot,ortalipesequinovarus.Herethefoot One ofthecommonerdeformitiesfoundatbirth differentiate the two. Charcot foot needs supportive treatmentwhilefusiontakesplace. differentiate thetwo.Charcotfootneedssupportive would suggest.Thedifferentialdiagnosisisinfectionandonlybiopsywillbeabletoreliably towhattheminimalpain jointdestructionoutofallproportion is swollenandtheX-rayshows The Charcot joint is now mostcommonlyseenindiabetics.Thepainisminimalbutthefoot The Charcotjointisnow 255

36: THE FOOT AND ANKLE ELECTIVE ORTHOPAEDICS Scenario 9 Scenario 8 Scenario 7 Scenario 6 256 who wishes to continue playing sport, afusionofthejointshouldbeconsidered. who wishestocontinueplayingsport, joint becomesstiffandpainfulespeciallyattoe-off.Modifiedfootwearcanhelp,butinapatient trauma.The inthefirstmetatarsophalangealjointiscommon,especiallyafterprevious Arthritis will obviouslyleaveanumbinterdigitalcleft. canimprove thesymptomsbut theswollenandtraumatisednerve toexcise improvement, surgery neuroma.Ifachangeoffootweardoesnotresultinan asMorton’s heads andisknown heads.Thisisespeciallycommonbetweenthe3rdand4thmetatarsal betweenthemetatarsal nerve Transverse pressureontheforefoot,especiallythatcausedbytightshoes,maytrapaninterdigital helpful. does notrequireanyactivetreatment,butifitfailstodosothendebridementofthejointmaybe in youngpeopleandmaybeasurprisingfindingonX-ray. Itusuallysettlesspontaneously,so head,whichdevelops Freiberg’s diseaseisaspontaneousosteonecrosisofthesecondmetatarsal offer anybenefit. islikelyto (suchasanarthrodesis) correction cannotbecorrectedpassively,onlybonesurgery theprogressionofdisorder.releases andtendontransferwillcorrectorslow ifthe However, (diastomatomyelia). Ifthedeformityispassivelycorrectable,thereapossibilitythatsoft-tissue such asCharcot–Marie–Tooth orofhavingsomekindentrapmentthespinalroots Teenagers aneurologicaldisorder pescavusmustbesuspectedofdeveloping developing 37 Infection and tumours

Multiple choice questions ➜ Rheumatoid arthritis G Hyperextension of the proximal 1. Which of the following statements interphalangeal joint with hyperflexion of regarding rheumatoid arthritis are the distal interphalangeal joint true? H Dupuytren’s contracture. A It is much commoner in women than in ➜ men. Gout and pseudogout B A negative rheumatoid factor excludes 4. Which of the following statements this diagnosis. regarding gout are true? C The disease appears to be based on a A Gout leads to pyrophosphate deposition. T-cell autoimmune response. B Gout and septic arthritis are similar and D The disease usually attacks large joints, can coexist in the same joint. such as the hip. C Gout is associated with renal calculi. E Joints are destroyed by an inflammatory D The key to the treatment of gout is to pannus. lower the plasma urate levels as quickly as possible. 2. Which of the following are extra- E Gout is the commonest cause of acute articular manifestations of rheumatoid monoarthropathy in the elderly. arthritis? F The metatarsophalangeal joint is the A Retinal detachment first joint to be attacked in most cases of B Subcutaneous nodules gout. C Myocardial infarction secondary to G Pyrophosphate crystals are positively atherosclerosis birefringent under polarised light. D Renal failure secondary to nephritis E Sarcoidosis ➜ Bone tumours Heberden’s nodes F 5. Which of the following statements Early malignancies in affected tissues G about bone tumours are true? H Asthma. A Patients with a past history of malignancy 3. Which of the following are who present with backache have characteristic features of rheumatoid metastases until otherwise proven. arthritis of the hands? B The extent of metastases can best be A Ulnar deviation of the wrist demonstrated on a bone scan. B Spontaneous ruptures of the flexor C All patients with suspected bone tumours tendons should have plain X-rays. C Ulnar deviation and subluxation of the D Pathological fractures through metastases metacarpophalangeal joints should be fixed but patients should not D W-deformity of the thumb be given radiotherapy as this will prevent E Carpal tunnel syndrome healing. F Hyperflexion of the proximal E Biopsy should be performed at the same interphalangeal joint with hyperextension time as staging to minimise delay. of the distal interphalangeal joint

257 ELECTIVE ORTHOPAEDICS ➜ ➜ Extended matchingquestions Extended Choose and match the correct treatment with each of the following scenarios: Chooseandmatchthecorrecttreatmentwitheachoffollowing 258

I H G F E D C B A 2. Diagnosisinarthritis 8 7 6 5 4 3 2 1 G F E D C B A 1. Treatment ofrheumatoidarthritis H G F Septic arthritis Avascular necrosis Yersinia arthritis Psoriatic arthropathy spondylitis Ankylosing Osteomyelitis Infected implant Juvenile rheumatoidarthritis Tuberculosis movement. inallrangesof presentswithamobilebutpainful elbow A patientwithrheumatoidarthritis stiffankle. presentswithapainfulandvery A patientwithrheumatoidarthritis pollicislongus. presentswitharuptureofextensor A patientwithrheumatoidarthritis localised overtheproximalradialhead. onpronation andsupination presentswithapainfulelbow A patientwithrheumatoidarthritis cause neurologicalcompromise. to spinewhichisstarting presentswithanunstablecervical A patientwithrheumatoidarthritis treatment. presentswithpainfultenosynovitisresistanttomedical A patientwithrheumatoidarthritis wrist onthemedialside.Whatcouldbeofferedsurgically? presentswithapainfulprominentlumponthebackofher A patientwithrheumatoidarthritis treatment forherdisease? fromsystemic function. Shedoesnotwishtoconsidersurgery. Whatcouldshebeofferedapart presentswithanunstablewristwhichcompromiseshand A patientwithrheumatoidarthritis Synovectomy Fusion Local boneexcision splint Supportive Tendon transfer Arthrodesis notoriously vascularandsoshouldbe Metastases fromtheprostateare amputation. ratethanprimary survival and soft-tissuetumourshasnoworsea bone ofprimary Local enblocexcision approach away fromanypossiblefuturesurgical The biopsytrackshouldbekeptwell I surgical approachisattempted. embolised ifatallpossiblebeforeany optimised isignored. that themedicalconditionshouldbe through metastases,thenormalrule In patientswithpathologicalfracture ➜ Choose and match the correct condition with each of the following scenarios: Chooseandmatchthecorrectconditionwitheachoffollowing Choose and match the correct diagnosis with each of the following scenarios: Chooseandmatchthecorrectdiagnosiswith eachofthefollowing

5 4 3 2 1 E D C B A 3. Bonetumours 9 8 7 6 5 4 3 2 1 osteoclasts. lesion inapatient’sfemurisbiopsied. Thecellsarefoundtobemultinucleate A singlelytic Her gamma-globulinsareraised. lymphoid-likecells. lesionsinherskeleton.Biopsyreveals A patientisfoundtohavemanylytic onlyasmallspikeofbone. X-rayshows growing. notchangingsizesinceshehasstopped butwhichisnow the femur,whichrestrictsflexion, A youngadultisfoundtohaveaprominenthardlumponthebackof herkneeattachedto likely source? An elderlypatientisfoundtohaveamalignantscleroticlesioninthelumbar spine.Whatisits aminortrip.Whatisthelikelycauseoflesion? following lesionin theproximalfemur An elderlypatientisfoundtohavefracturedthroughalytic Multiple myeloma Prostate Giant celltumour Metastasis Osteochondroma noticed increasingpainandfeltunwell. andthe nailwereremoved,thepatient afterthelockingscrews to holdthefracture.Shortly lockednailinserted whosufferedafractureofthetibiahadanintramedullary A motorcyclist afluid-filledlesionsurroundingtheL3vertebra. shows Magnetic resonanceimaging(MRI) A patientfromChinapresentswithbackpainwhichiswakinghimatnightandsweats. feelgenerallyunwell. andthey tohurt hip replacementstarts hourslaterthe A patientwhohadahipreplacement5daysagosuffersminorfall.Forty-eight painful tomove. andisvery swelling. Histemperatureisnotraised.Thekneeheldinthe‘positionofcomfort’ paininthekneewithsomerednessand A youngadultdrugaddictpresentswithsevere X-raysareunremarkable. course ofsteroidsforhissystemiclupuserythematosus. paininthehipaftera A patientwhohaspositiveanti-Smantibodiespresentswithsevere noticesthatshecanonlyopenhermouthalittle. The examiner smallforherage,presentswithpainandstiffnessinknees. A youngteenager,whoisvery formation. massiveosteophyte An X-rayofthespineshows backache. develops ofhiselbows surfaces A patientwhohashadscalyrashesontheextensor swollen knee.Healsohaspaininthesoleofhisfoot,whichmakeshimlimp. apainfuland developed mild diarrhoeabutnomalariaorothertropicalillnesses.Hehasnow A medicalstudentreturnsfromhiselectiveinthetropicswherehehadusualattackof which isatitsworstinthemornings. back(questionmarkposition).Hehashadsomeyearsofbackache, bent legsandacurved thatheiswalkinginastrangewaywith A middle-agedmanpresentswithhiswifewhosays 259

37: INFECTION AND TUMOURS ELECTIVE ORTHOPAEDICS ➜ ➜ ➜ 5.A,B,C,G 4.B,C,E,F,G 3.C,E,F,G 2.B,D 1. A,C,E Answers: Multiplechoicequestions 260

Bone tumours Gout andpseudogout Rheumatoid arthritis growth andreducing pain. growth tumour usefulinslowing healingand can bevery surprisingly, doesnotprevent Radiotherapy, quickly aspossibletoremovepain and toimprovemobilityinthelastprecioustimeleftoflife. way oflookingforothermetastases, especiallyremoteones.Metastaticfracturesneedfixingas (the initialinvestigationofchoicein allcases)androutinebloodtests.Bonescanisthebest high degreeofsuspicionthatthisis metastaticspread.TheleastthatisneededareplainX-rays mustbetreatedwitha ofmalignancy history witha previous Patients presentingwithbackache especially favourslargejointssuchasthehiporknee. ispseudogout, which canattackanyjointbut elderly thecommonestcauseofacutemonoarthritis inthe of thepresentingcasesgout,soitisbyfarcommonestjointtobe affected.However, percent the attackworse.Themetatarsophalangealjointisfirsttobeattacked inaround50 plasma uratewouldbethefirst-linetreatmentofgoutbutparadoxically thismayactuallymake of the levels stones.Itmightbethoughtthatlowering urine andtheformationofuratekidney inthe intheplasmagoutcanlead tohighlevels arenot.Theraiseduratelevels urate crystals arebirefringent,while can bedistinguishedmicroscopicallybythefactthatpyrophosphatecrystals Pseudogoutproduces pyrophosphate. Thecrystals Gout leadstothedepositionofuratecrystals. isnotassociatedwithrheumatoidarthritis. tendons,butDupuytren’s the flexor synoviumaround tunnelsyndromedoesoccurwithhypertrophied interphalangeal joint).Carpal ofthedistal oftheproximalinterphalangealjointwithhyperflexion deformity (hyperextension ofthedistalinterphalangealjoint)orswan-neck interphalangeal jointwithhyperextension oftheproximal of itsshape.Similarly,inthefingerstherecanbeBoutonnière(hyperflexion tendons.ThedeformityinthethumbisusuallydescribedasaZ-deformitybecause the flexor tendonswhichtendtorupturespontaneously,not metacarpophalangeal joints.Itistheextensor atthe andsubluxation atthewristwithulnardeviation There isradialdeviation and vascularneuritisarewelldescribed. compression either asthmaorearlymalignantchangearemorecommoninrheumatoidarthritis; that butamyloidis.Thereisnoevidence isnotassociatedwithrheumatoidarthritis Sarcoidosis andcanleadtorenalfailure. protectiveofatherosclerosis.Nephritisoccursinthekidneys anything, appearstobe,if myocarditisiswelldescribedbutrheumatoidarthritis Intheheart, osteoarthritis. Subcutaneous nodulesdooccurbutthesearenotHeberden’snodes,whichassociatedwith includescleritisandiritisbutnotretinaldetachment. problemsinrheumatoidarthritis The eye anderodingthesubchondralbone. cartilage destroying thearticular pannusspreadsacross, and focusesonthesmalljointsofhandsfeet.Aninflammatory Thediseaseisusuallysymmetrical andtumournecrosisfactor(TNF). andIL-6, including IL-1 diagnosis. ThediseaseisbasedonaT-cell autoimmuneresponsewithreleaseofinterleukins, the percentofcases,soanegativetestdoesnotexclude factor isonlypositiveinaround80 occursin3percentofwomenbutonly1men.Rheumatoid Rheumatoid arthritis ➜ ➜ 2G 1E 1D,2E,3G,4F,5E,6C,7B,8A matchingquestions Answers: Extended

inflammation is inversely proportional to the severity of the original bowel infection. ofthe originalbowel totheseverity inflammation isinversely proportional 2. Diagnosisinarthritis 1. Treatment ofrheumatoidarthritis by organismssuchas tract associated withinfectionofthegastrointestinalorurinary reactivearthritides There areseveral ‘question mark’posture. producinga its lordosis,becomingstiffandstraight, whilethehipsandkneesgointoflexion, todominate.Thelumbarspineloses butrapidlystiffnessstarts withbackache, condition starts B27-positive. The spondylitiscommonlyaffectsmiddle-agedmenwhoareSLA Ankylosing somemobility.relief whileatleastpreserving shoulder,hipandkneehavewell-provenjointreplacementswhichcan givegoodpain elbow, the operation asfusioniseasytoobtainandfunctionmaynotbebadlycompromised. However, ratherthanajointreplacement.Thewristandankleare especiallysuitableforthis an arthrodesis good functionalresult. because theirhealingpotentialiscompromisedandsoatendontransfermorelikelytogive dorupturethendirectrepairofthesetendonsisrarelysuccessful rupture ofthetendons.Ifthey toprevent If thisdoesnotrespondtomedicaltreatmentthensynovectomyshouldbeundertaken can alsoattacktendons,causingspontaneousrupture.Beforethisthereisobvioustenosynovitis. pannus neurologicaldeterioration.Theinflammatory anyfurther spineshouldprevent the cervical reduces painandsoimprovesmobility. oftheoffendingbone(thistimeproximalradialhead) supination. Onceagain,excision canselectivelyaffecttheproximalradialhead,producingpainonpronationand arthritis rheumatoid compromise stabilitybutdoesreducepainandsoimprovesgrip.Attheelbow, ofthedistalendulnadoesnot painful.Anexcision posteriorly andbecomesvery ofthewrist,distalulnasubluxes does notwanttoconsidersurgery. arthritis Inmoresevere A wristsplintcanimprovehandfunctioninapatientwithpainfulunstablejointwho made as good as possible. Surgery cannot lengthen life but may shorten it. lifebutmayshorten cannotlengthen made asgoodpossible.Surgery untiltheirconditionhasbeen mustnotbeundertaken dehydration andhypercalcaemia.Surgery result. uncontrollablehaemorrhagemay need tobeembolisedbeforesurgicalapproach,asotherwise inboth. the same is amputation,butoverallthesurvival a slightlyhigherincidenceoflocalrecurrencethanprimary any tumourseedingitwillberemovedatdefinitivesurgery. sothatifthereis completelyatthetimeofsurgery, zone,andmustbeexcised surgical excision sothatthecorrectareaisbiopsied.Thebiopsytrackmustbewithin biopsy isundertaken Patients withpathologicalfracturesthroughmetastasesareoftenmetabolicallyunwell, Prostatic metastasesarenotespeciallyvascularbutrenalare,anditisthesethat andreconstructionwithanendoprostheticreplacementisassociated Local enblocexcision When investigatingatumourstaging,imagingandbloodtestsmustbecompletedbefore If a joint is very stiff and painful as a result of rheumatoid arthritis, it maybebesttoconsider stiffandpainfulasaresultofrheumatoidarthritis, If ajointisvery spinebyerodingthefacetjoints.Fusion of cancreateanunstablecervical Rheumatoid arthritis Chlamydia, Shigella or Yersinia. In the latter case, the severity ofthejoint Inthelattercase,severity 261

37: INFECTION AND TUMOURS ELECTIVE ORTHOPAEDICS ➜ 3A 2D 1B 9D 8A 7C 6I 5H 4B 3F 262

3. Bonetumours idiopathic spinalhyperostosis)orForestier’s disease. (diffuse formationreminiscentofDISH the spineiscommonlyaffectedwithmassiveosteophyte andinthecaseofpsoriasis Psoriasis andotherautoimmuneconditionscanalsocausearthritis, causing problems. stalk. Thisisthecharacteristicpicture ofanosteochondroma,whichshouldberemovedifitis caponabone but onX-rayisalmostcompletelyinvisible. Thismeansthatitismainlyacartilage fromthebone, soislikelytobe benign.Itappearstobeanoutgrowth This lumpisnotgrowing ofBatson. from theprostateandgotospineviavenousplexus A malignantlesioninthespineisagainlikelytobeametastasis.Sclerotic lesionscommonlycome bonetumours. are farcommonerthanprimary lesionislikely tobeametastasis.These A pathologicalfractureinanelderlypatientthroughalytic osteomyelitis. intoafull-blown The situationmaythendevelop biofilm andliedormantuntilactivatedbysomemajorchange(suchasremovalofmetalware). bacteriamayforma contaminated withbacteria.Inthepresenceofanimplant,low-virulence The introductionofmetalworkintobonealwayscarriestheriskthatmaybecome thepossiblediagnosisoftuberculosis. location inthespineallsupport oforiginandthe suggestsinfectionandthecountry proven.ThefluidcollectiononMRI otherwise Pain whichwakesapatientatnightandsweatsarecausedbyeithertumourorinfectionuntil intravenous antibioticsifthejointistobesaved. onhigh-dose for culture.Thejointshouldthenbethoroughlywashedoutandthepatientstarted activate theinfection.Ifdiagnosisissuspected,jointmustbeopenedandbiopsiestaken andpaininthejoint.Aminorfallmaybearedherringor,somethink, fever with alow-grade andpresents periodaftersurgery jointusuallyoccurswithinashort inanartificial Septic arthritis painontheslightestmovement. characteristic featureisextreme Itismostcommonintheknee.Thepatientmaynotbesystemicallyunwell,but arthritis. Drug addictsandpatientswhoareimmunocompromisedespeciallysusceptibletoseptic nochangesonX-ray.cause avascularnecrosis,whichintheearlystagesshows canproducejointpainsbut thesteroidsusedtotreatitcanalso Systemic lupuserythematosus difficultanddangerous. their mouths.Thismaymakeintubationforsurgery havedifficultyopening affected,asisthetemporomandibularjointsothatthey commonly severely Thekneesare usuallyaffectsyounggirlsunder5andgrowth. Juvenile rheumatoidarthritis 5C 4E is calledmultiplemyelomawhentherearemanylesions.Theseconsistoflymphoidtissue. which (solitary), mostcommonlesionintheskeletonisplasmacytoma After metastases,thenext (it doesnotmetastasise)butislocallyquiteaggressiveandsoneedsearlytreatment. lesionfulloflargemultinucleateosteoclastsisagiantcelltumour.A solitary Itisabenigntumour 263

37: INFECTION AND TUMOURS 38 Paediatric orthopaedics

Multiple choice questions ➜ Paediatric orthopaedics – ➜ Perthes’ disease general 3. Which of the following statements 1. Which of the following statements are regarding Perthes’ disease are true? true? A Perthes’ disease is a spontaneous A The Heuter–Volkmann principle states avascular necrosis of the hip. that compressive forces stimulate growth. B It is most common in boys around B Wolff’s law states that bone is deposited puberty. and resorbed according to the loads C The condition frequently settles placed on it. spontaneously. C Limb bud embryogenesis starts at D A similar problem can occur in children month 4. with . D In-toeing can be caused by persistent E The prognosis is best in those children in femoral neck anteversion. whom the condition develops late. E All children start with knock-knees which ➜ then become bow legs by the time that Slipped upper femoral they are 2 or 3 years old. epiphysis F All children under the age of 3 years 4. Which of the following statements have flat feet. regarding slipped upper femoral G If toe-walking develops after the child has epiphysis are true? started walking normally, a neurological A It may present with pain in the knee. problem should be suspected. B It can best be diagnosed on an anteroposterior (AP) X-ray. ➜ Developmental dysplasia of C It occurs in boys around the age of the hip puberty. 2. Which of the following statements D Perfect reduction must be obtained. regarding developmental dysplasia of E The other hip is also commonly involved. the hip are true? ➜ A It is more common in boys than in Hand abnormalities girls. 5. Which of the following statements B It occurs in up to 20 per cent of breech regarding hand abnormalities are true? deliveries. A Surgery to correct hand abnormalities C It is more common in the first-born. should be deferred until it is clear how D It does not run in families. much disability the problem is causing. E It is not associated with spina bifida. B Radial club hand tends to occur in F It is best diagnosed by X-ray. isolation of other deformities. G It is best treated early by non-operative C Radioulnar synostosis requires surgery to means. restore pronation and supination. H The goal of surgery is to get the head D Congenital radial head dislocation is concentric within the acetabulum. anterior.

264 ➜ ➜ ➜ Extended matching questions Extended statements Whichofthefollowing 7. Whatisthelikelydiagnosis? (a) Ayoungchildpresentswithpainin 6. Choose and match the correct diagnosis with each of the following scenarios: Chooseandmatchthecorrectdiagnosiswitheachoffollowing Whattreatmentshouldbestarted? (c) (b) What investigationswouldyou (b)

1 E D C B A 1. Kneeproblems A Osteomyelitis F E D C B A G F E D C B A E D C B A Case study–shoulderpain femoral condylehas brokenoffandisjamminginthe joint. thatafragmentofthe shows A youngmancomplainsofclunking andlockingoftheknee.MRI Chondromalacia patellae Osteochondritis dissecans Osgood–Schlatter disease Discoid meniscus Blount’s disease spread. It isusuallycausedbyhaematogenous Steroids. Intravenous antibiotics Anti-inflammatories Analgesia Aspiration Splintage X-ray. Ultrasound Blood cultures Gamma-globulins C-reactive protein(CRP) and sedimentationrate(ESR) Erythrocyte Full bloodcount Serum urate A dislocation. A fracture Septic arthritis Irritable joint Gout regarding osteomyelitisaretrue? moved. Sheistoxicandpyrexial. trauma. Shescreamsifthearmis her shoulder. of Thereisnohistory organise? ➜ . A2-year-olddisabledchildisbrought 8.

E D C B A Case study–injuryinachild F E D C B bone following anattackofosteomyelitis. bone following An involucrumisadeadfragmentof necrosis willresult. any circumstancesbedrainedasbone Pus collectioninboneshouldnotunder osteomyelitis nearby. appears tobeasepticjoint,theremay If asterileaspirateisobtainedfromwhat centrally. boneandsotracks penetrate cortical bonecannot Pus forminginmedullary osteomyelitis. hip iseffectivelysynonymouswith ofthe In neonates,septicarthritis from anephroblastoma. Pathological fracturethroughametastasis Fracture throughabenignbonecyst olderbrothers Exuberant (NAI) Non-accidental injury Osteogenesis imperfecta be most carefully investigated? be mostcarefully ages. Whatisthediagnosisthatmust fair shareofbruisesonhim,varying a quietchildwhohasmorethanhis older brothers.Onexaminationheis the daybeforewhenplayingwithhis think hefellfromasofaontothefloor did notseeexactlywhathappenedbut a previoustibialfracture. Theparents distal femur. from Thereisalsocallus handle metaphysealfracture ofthe to theA&Edepartmentwithabucket- 265

38: PAEDIATRIC ORTHOPAEDICS ELECTIVE ORTHOPAEDICS ➜ ➜ Choose and match the aetiology or diagnosis with each of the following scenarios: ordiagnosiswitheachofthefollowing Chooseandmatchtheaetiology scenarios: Chooseandmatchthecorrecttreatmentwitheachoffollowing 266

6 5 4 3 2 1 K J I H G F E D C B A 3. Spineandneurological problems inchildren 5 4 3 2 1 E D C B A 2. Footproblems 5 4 3 2 Scheuermann’s disease Leg-length inequality A childisbornwith theheadfixedtooneside,due acontractureofthesternocleidomastoid. on theL5. 30percent bodydisplacedforward isfoundtohave theL4vertebral A patientwithbackache slopingshoulders andaprominentthoracickyphosis. todevelop A teenagerstarts sitdown. inthespinewhich seemstovanishwhenthey A childhasacurve totouchtheirtoes. forward tip ofthespinewhichproducesaribhump whenthey A teenagerpresentswithacurve ofthespineatT12.A youngchildpresentswithasharpcurve injury Brachial plexus Idiopathic scoliosis Cerebral palsy Muscular dystrophy Spondylolisthesis Spina bifida Congenital abnormalityofavertebra Torticollis Polio toappear.settling andthefirstsignsofdegenerativechangeincoalitionarestarting coalition.Itisnot confirmsatarsal A schoolchildpresentswithastiffandpainfulfoot.CT butlookunsightly. arecausingnopainordisability A childpresentswithcurlytoes.They talus. A childpresentswithcharacteristicrocker-bottomfootofthecongenitalvertical A childpresentsat2yearsofagewithastiffclubfootdeformity. corrected. withaclubfootwhichcanbepartially A childpresentsatbirth andstretching Regular massage Soft-tissue releases Soft-tissue releaseswithcorrectionofthebonealignmentsfoot Excision Serial plasters A West ofthetibia. Indianchildpresentswithvarusbowing A teenagegirlhaspainwhichappearstoarisefrompatellofemoralmaltracking. A childpresentswithapainfullumpoverthetibialtubercle. is asoliddiscratherthancrescent. anabnormallateralmeniscuswhich shows A childhasakneewhichclunksandlocks.AnMRI ➜ ➜ ➜ 3.A,C,D 2.B,C,G,H 1. B,D,F,G Answers: Multiplechoicequestions

Perthes’ disease Developmental dysplasiaofthehip Paediatric orthopaedics–general boys but develops long before puberty inchildrenaged around 6years.Providingthatthefemoral long beforepuberty boys butdevelops tobeaspontaneous avascularnecrosisofthe hip.Itiscommonestin diseaseisbelieved Perthes’ normallyandthehipwillbecome stable. will develop using doublenappiesoraharness. Providingthehipisheldconcentricinacetabulum,two where thehipcanbeheldproperlylocated,itismuchbettertoholdthis positionnon-operatively position canbefound areductiontobesustained.Ifsafe and, ifnot,thenwhatpositionallows ofchoice,asitispossibletoseeifthefemoralheadfitssnugly intheacetabulum examination havenotyetossified.Ultrasoundisthe asthey nor theacetabulumcanbevisualisedatbirth X-rayisofnouseindiagnosisearlyon,because neitherthefemoralhead and arthrogryposis. racialgroups.Itisassociatedwithothercongenitalabnormalities, especiallyspinabifida as certain 20 percentofbreechdeliveries.Itismorecommoninthefirst-bornand runsinfamiliesaswell dysplasiaofthehipismorecommoningirlsthanboysand occursin Developmental neurological sign. older,thisisanominous grow walkingplantigradeandthengoontotheirtoesasthey start they walkingontheirtoes,butif withtime.Manynormalchildrenalsostart of thefootonlydevelops offwithflatfeet.Thearch time,itistruethatallnormalchildrenstart 2 or3yearsold.Atthesame grows. by tibialtorsionandmetatarsusvarus.Mostoftheseconditionscorrectthemselvesasthechild time. atthatcritical insulttotheembryo abnormalities becausebotharetheresultofsame limbbudabnormalitiesareassociatedwithspecific cardiac are forming.Thismeansthatcertain timeassomeinternalorgans,suchtheheart, (all withinthefirsttrimester).Thisissame Wolff’s lawstatesthatboneisdepositedandresorbedaccordingtotheloadsplaceduponit. Volkmann whiletensileonesstimulateit. principlestatesthatcompressiveforcesinhibitgrowth accordingtotheHeuter–Volkmann inpart Bones grow principleandWolff’s law. TheHeuter– 11 11 10 9 8 7 Children actually start with bow legs. These then become knock-knees by the time that they are legs.Thesethenbecomeknock-kneesbythetimethatthey withbow Children actuallystart butcanalsobecaused In-toeing canindeedbecausedbypersistingfemoralneckanteversion, early(aroundweek4)andiscompleteby8 very starts Limb budembryogenesis A child is noticed to have a hairy tuft at the base of the spine, and lower motorneuronsigns tuftatthebaseofspine,andlower Achildisnoticedtohaveahairy in the lower legs. in thelower A childpresentswithprogressivemuscularweaknessinthelimbs. is normal. highmuscletone.Intelligence limbswithvery contracturesofbothlower A childhasflexion loss. legs butnosensory hasmarkedmotorweaknessandwastinginthelower A childwhohadafebrileillnessnow achildisnotedtohaveflaccidarm. After adifficultbreechbirth, 267

38: PAEDIATRIC ORTHOPAEDICS ELECTIVE ORTHOPAEDICS ➜ ➜ ➜ ➜ 7. A,B,D,F 6.(a)C;(b)B,C,E,F,G;(c)A,E 5. A 4.A,C,E 268

Osteomyelitis Case study–shoulderpain Hand abnormalities Slipped upperfemoralepiphysis infection,andtheuseofhigh-dosesteroids.Theprognosisisworselateritdevelops. following inmanyotherconditions,includingsicklecelldisease,skeletaldysplasia, the hipcanalsodevelop beforeitcollapses,theconditionsettlesspontaneously.head revascularises Avascular necrosisof that the metaphyses of bones which are intra-articular (suchastheproximalendof femur) that themetaphyses ofboneswhichareintra-articular bone.This means the pressurerises and thepusappearsabletopunch itswaythroughcortical toaccumulate, Osteomyelitis usuallyarisesfromhaematogenous spread.Whenthepusstarts rapid improvement,thejointshould beopenedandwashedout. oncethecultureandsensitivitiesareback.Ifthereisnot empirically thenchangedifnecessary atonce patient willneedstronganalgesia.High-doseintravenousantibioticsshould bestarted ajointeffusionwhichcanthenbeaspiratedunderultrasoundcontrol. will show abnormalities, suchasunderlyingosteomyelitis.Ultrasoundistheimaging modalityofchoiceasit other butmayshow changesinearlysepticarthritis andbloodcultures.X-rayswillnotshow CRP and goutinchildrendoesnotoccur. Thebloodinvestigations ofchoicearefullbloodcount,ESR, pain, trauma sofractureanddislocationareunlikely. Irritableshoulder doesnotproduceextreme of astimeisoftheessenceifpermanentdamagetobeavoided. Thereisnohistory arthritis, quicklyhere,isseptic The mostlikelyandseriousdiagnosis,whichneedstobeexcluded posterolateral. radial headisanterior,butincongenitaldislocationittheoppositedirection, cannot leadtotherecreationofthismovement.InMonteggiafracturesdislocation anus,cardiovascularsystem,trachea,renalandlimbbuds. involve thevertebrae, functionality. thecomplicationofreducing whichmaycarry embarking onwhatmayonlybecosmeticsurgery, candowithadeformedupperlimbbefore whatthey handdominanceandshows child develops towaituntilthe much betterthanauselessbutcosmeticallynormalone.Itisthereforeimportant Hand abnormalitiesareacosmeticandfunctionalproblem,butdisfiguredhandis prophylactically pintheotherhipbeforeithasachancetoslip. the slipgoinganyfurther. theotherhipiscommonlyinvolved,manysurgeonswillchooseto As most acuteslips,theheadisfixedinitsslippedpositionusingpinswhicharedesignedtoprevent diagnosisofavascularnecrosis.Therefore,inallbutthe femoral headandcausingthedisastrous an attemptismadetoreducetheslip,thereamajorriskofdamagingbloodsupply must alwaysberequested.Theconditioniscommonestinboysaroundtheageofpuberty. If formakingthediagnosisand isthemostimportant in retrospect),alateralor‘frog’slegview’ head tendstobemainlybackwardsandso,althoughtheslipcanseenonAP(usually inthehipsometimespresentingaspainknee.Theslipoffemoral results inpathology asthemusclesactingoverit.This supplytoajointisthesame Hilton’s lawstatesthatthenerve Aspiration is both diagnostic and therapeutic. The joint will need splinting for comfort andthe isbothdiagnosticandtherapeutic.Thejointwillneedsplintingfor comfort Aspiration surgery pronationandsupination,butunfortunately Congenital radioulnarsynostosisprevents These clubhandisclassicallyassociatedwithaseriesofotherabnormalities(VACTERL). Radial ➜ ➜ ➜ 1A/E,2D, 3C,4D,5B 5A 4E 3C 2B 1D matchingquestions Answers: Extended 8. B

2. Footproblems 1. Kneeproblems Case study–injuryinachild involucrum). outerlayerofabnormalboneundertheperiosteum(the (a sequestrum)andcreatinganew pus drainedout.Ifthisisnotdone,thewillliftperiosteumkillingbonebeneath soonasacollectionofpusissuspected,thebonemustbeopenedand aswell.As arthritis septic symptoms(withasterilehipeffusion)and/orfull-blown can producesepticarthritis-like tethering the deformity will be needed, followed byplasters designedtoholdthatposition. tethering thedeformity willbeneeded,followed rigid,then surgicalreleaseofthesofttissues whichare presentation islateortheclubfoot is very thiscorrectiontobe obtainedandsustained(thePonsetti technique), butif serial plasteringallows normally. cangrow the hindfootintocorrectrelationship witheachothersothatthey Careful totreatmentisgetthebonesof the posteromedialstructuresof footandthecalf.Thekey abnormalityof improvement. Amorerigidformof clubfootrelatestosomekindofdevelopmental andstretchingshouldleadtoarapid caused byabnormalpositioninthe uterus.Regularmassage There aretwotypesofclubfoot.Thefirstissuppleandcalledphysiological. Itisprobably oftheproximaltibialepiphysis. disease, anabnormalityofgrowth Varus inaWest bowing Indianchildmightbecaused by rickets,butismorelikelytobeBlount’s stairsandaftersittingstillforalongperiod. when goingupanddown Pain andpossiblemaltrackingofthepatellaiscalledchondromalaciapatellae. Itismostpainful inflammation oftheepiphysisundertibialtubercle.Itusuallysettlesspontaneously. A painfullumpoverthetibialtubercleinanadolescentisOsgood–Schlatterdisease, centre-like adisc.Thismeniscusissusceptibletotearsandthencauseslockingintheknee. isnotacrescentbuthassolid- meniscus calleddiscoidmeniscus.Inthesecasesthecartilage A secondpossibilityforaclunking,lockingkneeisthecongenitalabnormalityoflateral These loosefragmentscanjaminthejointandcauselocking. Fragments ofthefemoralcondylecanbelostinaconditioncalledosteochondritisdissecans. the appropriateauthorities. investigateby NAIandthisisfarmorethanthethresholdneededtoactivatefurther point towards healedfractures.Therearethereforeatleastninefeaturesinthiscasewhich there areprevious handle ofchipfracturethemetaphysis,arealsocommonlyfoundinNAI,asisfindingthat fractures,suchas thebucket- different ages,especiallypinches,isanotherfeatureofNAI.Certain ofNAI.Thepresencebruises wasnotwitnessed.Theseareallcharacteristicofahistory injury hereisimprobable,thereadelayinpresentation,andthe Themechanismofinjury are disabled. ismostcommoninchildrenundertheageof3yearsandespeciallyifthey Non-accidental injury 269

38: PAEDIATRIC ORTHOPAEDICS ELECTIVE ORTHOPAEDICS ➜ 1C,2H,3I,4J,5E,6B,7K,8D,9A,10G, 11F 270

3. Spineandneurological problems inchildren excised. itshouldbe tobecomearthritic, starts will leadtopathologicalfractures.Ifthispseudoarthrosis coalitionwillgiveapathologicalflatfoot.Thestressesontheabnormalbonebridge A tarsal thetoestostraightenout. insists onsomethingbeingdonethensoft-tissuereleasesshouldallow thecorrectalignmentofbonestobeachieved. allows only surgery talus(rocker-bottomfoot),as A similartreatmentregimewillbeneededforthecongenitalvertical neuron signs with spasticity, but once again there should be no sensory impairment. neuron signswithspasticity,butonceagainthereshouldbenosensory arenotprogressive.Bycontrast,cerebralpalsyproduces uppermotor notupperandthey lower changes. Themotorsignsare cells ofthespinalcord,causingmuscleweaknessbutnosensory legs. neuron signsinthelower motor Itisasignofspinabifida.Ifthisaffectingthespinalroots,therewillbelower for atbirth. areprogressivedisorders. characteristic isthatthey abreechdelivery.especially commonfollowing dysplasiaofthehip. abnormalities suchasdevelopmental afibrousmassinthesternocleidomastoidmuscle.Itisassociatedwithothercongenital and even repeated load,suchasweight-lifting. Thiscanbecongenital,traumaticorcommonlyastressfracture following between thevertebrae. producing astoopinggaitanddroopyshoulders. posteriorly), anteriorly(convex Inthiscase,thespinecurves spurt. during theadolescentgrowth creatingthecharacteristicribhump. the childbendsforward, spineandis,ineffect,abucklingofthespine.Inthiscondition,rotatesmoreas growing inthespine. therewillbeaprogressivesharpcurve grows, Eitherway,asthechild ortoafusionononesideoftwoadjacentvertebrae. does notdevelop) (oneside normallycanleadeithertoahemivertebra bodiestodevelop Failure ofthevertebral Curly toestendtobeabenignconditionand,ifpossible,shouldleftalone,butthepatient Polio presentsasafebrileillnesswhichthengoeson topermanentlydamagetheanteriorhorn tuftatthebaseofspineisanothercongenitalabnormalitywhichneedschecking A hairy There aremanytypesofmusculardystrophyaffectingdifferentmusclegroups,buttheir Thisis tothebrachialplexus. mayhavesufferedinjury A childwithaflaccidarmafterbirth andinthiscaseitiscongenitalcausedbycontracture A twistedneckiscalledtorticollis bodies(spondylolisthesis)iscausedbyafailureofthepedicles A slipbetweentwovertebral abnormalityofthespineisScheuermann’sdisease,whichalsodevelops A secondgrowth intherapidly deformityofthespine,whichdevelops Idiopathic scoliosisisprimarilyarotatory PART 6 Skin and subcutaneous tissue

39 Skin and subcutaneous tissue 273 This page intentionally left blank 39 Skin and subcutaneous tissue

Multiple choice questions ➜ Anatomy 4. Which of the following statements are 1. Which of the following statements are false? true? A The vast majority of malignant melanoma A The epidermis accounts for the major form in pre-existing naevi. part of the skin. B Change in colour, shape, size and surface B The majority of epidermal cells are in a naevus should be looked upon with keratinocytes. suspicion. C Melanocytes are cells found in the C Incision biopsy should be done in all dermis. suspicious lesions. D Sweat glands are of two types: eccrine D Superficial spreading melanoma is the and apocrine. most common type. E Adnexal structures span the epidermis E Exposure to ultraviolet rays is a major and dermis. cause of malignant melanoma. ➜ ➜ Ulcer edges Necrotising fasciitis 2. Which of the following descriptions of 5. Which of the following statements ulcer edges denote malignancy? are false with regard to necrotising A Sloping fasciitis? B Overhanging A This is a surgical emergency. C Everted B It is a polymicrobial synergistic infection. D Punched out C To confirm the diagnosis a plain X-ray is E Rolled. done to look for air. D 80 per cent of cases have a history of ➜ Malignant melanoma trauma or previous infection. A period of observation is advisable to 3. Which of the following are not types of E see if the condition spreads. malignant melanoma? It can rapidly progress to septic shock. A Superficial spreading F Urgent measures necessary are B Salmon patch G resuscitation, antibiotics and surgical C Nodular debridement. D Giant congenital pigmented naevus The mortality rate is 30–50 per cent E Lentigo maligna H F Acral lentiginous.

Extended matching questions ➜ 1. Definitions A Ulcer B Sinus C Fistula

273 SKIN AND SUBCUTANEOUS TISSUE ➜ ➜ Choose and match the correct term with each of the following descriptions: Chooseandmatchthecorrecttermwitheachoffollowing Choose and match the correct condition with each of the following descriptions: Chooseandmatchthecorrectconditionwith eachofthefollowing descriptions: Chooseandmatchthecorrectconditionwitheachoffollowing 274

5 4 3 2 1 E D C B A 3. Benignskintumours 5 4 3 2 1 E D C B A 2. Infections 3 2 1 is unclearbutmay becausedbyinfectionofahairfollicle byapapillomavirus.Although it measuredinweeks.Theaetiology history, filled withakeratinplug.Thelesion hasashort A lesion(Fig.39.1), usuallyfound onthehead,neckandface,shapedlikeacupwithcentre This isoneofthecommonestbenignskinlesionsinelderly. cells containingmelanocytes. layeroftheepidermal lesionwhichisoften pigmentedandarisesfromthebasal A soft,warty andcondylomataacuminata. warts which alsocausesplantar A benignskintumourcausedbythehumanpapillomavirus(HPV) Ithasnomalignantpotential. compound orintradermalnaevus. cellsandprogressesintheolder person toforma a dermo-epidermalproliferationofnaevus A deeplypigmentedmacularorpapularlesioncommonlyseeninchildhood. Itiscausedby cellswithnestsandcolumnsinthedermis. proliferation ofnaevus A maculopapularpigmentedlesionthatismostprominentatpuberty. Itisajunctional Keratoacanthoma Compound naevus Junctional naevus wart Papillary cell papilloma Basal This producesawell-demarcatedinfectionbystreptococci,usuallyontheface. by traumaorulceration. This isageneralisedbacterialinfectionoftheskinandsubcutaneoustissue,usuallypreceded This isahighlyinfectiousskinlesion,usuallyaffectingchildren,causedby In thislesionthereisintravascularthrombosisproducinghaemorrhagicskininfarction. Streptococcus A polymicrobialsynergisticinfectionmostoftencausedbygroupbeta-haemolytic Purpura fulminans Necrotising fasciitis Cellulitis/lymphangitis Erysipelas Impetigo A discontinuityofanepithelialsurface. with anepithelialsurface. A blind-endingtractthatconnectsacavitylinedwithgranulationtissue(usuallyanabscess) may belinedbygranulationtissuebutinchroniccasesitepithelialised. Thecommunicationortract An abnormalcommunicationbetweentwoepithelial-linedsurfaces. or Streptococcus . . Staphylococcus and/ ➜ ➜ 1. B,D,E Answers: Multiple choicequestions Choose and match the correct condition with each of the following descriptions: Chooseandmatchthecorrectconditionwitheachoffollowing

Anatom 5 4 3 2 1 E D C B A 4. Premalignant andmalignantskinlesions The epidermis is composed of keratinised, stratified, squamous epithelium. From superficial to The epidermisiscomposed ofkeratinised,stratified, squamous epithelium.From superficial The epidermisaccountsfor5percent ofthetotalskin,whiledermisaccountsfor95percent. elderly male. scalypatchonthedorsumof righthandofan enlargingerythematous There isaslowly around thelesion. smallblackspotsirregularly scattered tobleed.Thereareafew become itchyandstarted There isapigmentedskinlesiononthescalpthathasrecentlychanged incolourandhas from metastasis. indurated inflamedsurroundingarea.Aminoritymayhaveenlargedregional lymphadenopathy edgeandan months’duration,witharaised,everted This isanulceratedskinlesionofafew axilla. Itisaformofintraepidermaladenocarcinoma. This occursinthegenitalorperianalregionsskinrichapocrineglands,suchas typically raisedandrolledoccurmoreoftenontheface,headneck. years’duration.Theedgesare intoneighbouringtissues)ofafew like arodentburrowing locallyinvasivetumour(hencealsocalledarodentulcer,asitbehaves This isaslow-growing, Malignant melanoma(MM) Squamous cellcarcinoma(SCC) cell carcinoma(BCC) Basal Paget’sdisease Extramammary disease Bowen’s differentiate itfromsquamouscellcarcinoma. itforasuperiorcosmeticoutcomeandalsoto regresses spontaneously,itisbettertoexcise Figure 39.1 y Keratoacanthoma. 275

39: SKIN AND SUBCUTANEOUS TISSUE SKIN AND SUBCUTANEOUS TISSUE ➜ ➜ ➜ ➜ 3A 2B 1C matchingquestions Answers: Extended 5. 4.A,C 3.B,D 2.C,E 276

1. Definitions Necrotising fasciitis Malignant melanoma Ulcer edges occursfromthesestructures. dermis. Ininjurieswhereepidermisislost,re-epithelialisation hydradenitis suppurativa. andgroinareasbecomeactiveatpuberty. Persistentaxillary infectionoftheseglandscauses asympathectomy.condition canbecuredbyperforming Apocrineglandsarefoundinthe limbs),the sweating(commonlyseeninthepalms,axillaandlower where thereisexcessive sympathetic activitysuchasemotionandareresponsibleforthermoregulation.Inhyperhidrosis, secretesweatinresponseto forthelips.They except present throughouttheentirebodysurface, within thekeratinocytes. Differences inskincolouraredeterminedbyvariationstheamountanddistributionofmelanin inthestratagranulosumandspinosumcontainmelanin. radiation. Thekeratinocytes synthesisesthepigmentmelanin,whichprotectscellnucleifromultraviolet melanocyte areclassifiedaccordingtotheirdepthanddegreeofdifferentiation. Keratinocytes melanocytes. layer. layerandadeeperreticular papillary Thedermisconsistsofasuperficial and stratumbasale. deep ithasfivelayers:stratumcorneum,lucidum,granulosum,spinosum malignant. that maybenecrotic,granulatingor malignant.Thesecanbeclassifiedasspecific,non-specificand characterisedbygradualdestructionwithabase An ulcerisadiscontinuityofanepithelial surface malignancy. sinus. Acquiredcausesarearetained foreignbody,specificchronicinfection,inflammationand whichgetsinfectedandburstswhich,althoughcalled a‘fistula’,isstrictly becomes acyst tractthatan abscess.Itmaybeclassifiedascongenital,inaremnantofthyroglossal Thecavityisusually A sinusisablindtractconnectingcavitywithanepithelialsurface. as inenterocolic,colovesical,enteroentericfistulae. Thesemaybecongenital,asintracheo-oesophagealorrectourethral fistulae,oracquired, surfaces. A fistulaisanabnormalcommunicationlinedbygranulationtissuebetween twoepitheliallined E The majority of epidermal cells are keratinocytes. The basal epidermisalsocontains Thebasal The majorityofepidermalcellsarekeratinocytes. Adnexal structures,suchashairfollicles,sebaceousandsweatglands,spanepidermis Adnexal The sweatglands,eccrineandapocrine,openintoporesinthehairfollicles.Eccrineglandsare epidermis.Each originatefromtheneuralcrestandarefoundinbasal Melanocytes 5B 4C 3A 2E ➜ ➜ ➜ 1C 1D,2C,3B,4A,5E 1D

leucocytosis. Theappropriatebroad-spectrumantibioticisthetreatment ofchoice. leucocytosis. with with traumatotheskinofface.Theareaisredandoedematouspatienthasfever lymphatics,usuallyassociated isawell-localisedstreptococcalinfectionofthesuperficial Erysipelas totheaffectedpart. elevation Treatment isthepromptadministrationofappropriateintravenousantibioticswithrestand overlying reddishstreaksoflymphangitis; withared,swollen,tenderarea associated withtraumaorulceration.Thepatienthasfever Cellulitis/lymphangitis isageneralisedbacterialinfectionoftheskinandsubcutaneoustissue washing oftheareaandapplicationtopicalorbroad-spectrumoralantibiotics. crust.Treatmentproduces blistersthatruptureandjoinupiscoveredbyhoney-coloured is skinlesion,usuallyaffectingchildren.Theinfection infectioussuperficial Impetigo isavery and idiopathicpurpurafulminans. tissuelossrequiringlimbamputation;neonatalpurpurafulminans; which mayresultinextensive types: acuteinfectiouspurpurafulminanscausedbyeitherbacterialorviralinfectionand produces haemorrhagicskininfarction.Itcanrapidlyprogresstosepticshock.Therearethree Purpura fulminansisararecondition,usuallyoccurringinchildren,whereintravascularthrombosis shock. thepatientmaygointoseptic resuscitation, antibioticsandsurgicaldebridement;otherwise promptlywith pain.Thismustbetreatedvery muscle groups,soft-tissuecrepitusandsevere hard indurationofthesubcutaneoustissue,inabilitytodistinguishbetweenfascialplanesand woody theskinerythema, in asmokerordiabetic.Thetypicalsignsareoedemabeyond Proteus 4. Premalignant andmalignantskinlesions 3. Benignskintumours 2. Infections Necrotising fasciitisusedtobecalledsynergisticgangrene.Itresultsfrompolymicrobialinfection, nose. oftheearor damagetoneighbouringvitalstructures, suchasthelensandcartilages prevent treatment israrelyusedbecausethe locationofthetumourprecludesuseradiotherapyto surgeon comprisetheoptimumtreatment. Althoughtheconditionisradiosensitive,thisformof closure,if possible, orskingraftinguseofrotationalflapsbyaplastic andprimary excision spreading,multifocal orinfiltrative,sometimescalledgeographicaltype.Surgical are superficial type,whichislocalised.Thegeneralisedtypes the commonestisnodularandnodulocystic clinical types,byfar raised, rollededgewhichmaylooklikeapearl.Althoughthereareseveral to ultravioletlight.Itmostoftenoccursonthehead,neckandface(see Fig.39.2).Ithasa lesion(rodentulcer)isusuallyinyearsandcaused byexposure The durationofthisslow-growing most commonlygroupAbeta-haemolytic , Bacteroides or Clostridium . The majority occur following trauma or infection, particularly traumaorinfection,particularly . Themajorityoccurfollowing Streptococcus Streptococcus is the commonest causative organism. isthecommonestcausative alongwith E. coli , Pseudomonas , 277

39: SKIN AND SUBCUTANEOUS TISSUE SKIN AND SUBCUTANEOUS TISSUE 3D 2B 278 often duetoinfectionasonly2percentmetastasise. edgewiththesurroundingarearedandindurated.Regionallymphadenopathyismore everted and immunosuppression.Thedurationisusuallyinmonths.lesionanulcerwitharaised tosun,chronicinflammation,scarsofburns 39.3). Theusualcausesareprolongedexposure the keratinisingcellsofepidermisoritsappendagesandusuallyaffectselderly(seeFig. thesecondmostcommonformofskincancer,arisesfrom Squamous cellcarcinoma(SCC), isthetreatment. excision ofthemareassociatedwithanunderlyinginvasiveneoplasm.Surgical London). Aboutaquarter Paget’sdisease:SirJamesPaget1814–1899,(extramammary Hospital, surgeon, StBartholomew’s This isanintraepidermaladenocarcinomaoccurringinthegenitalorperianalregionsaxilla oncologist andthe pathologist. betweentheplasticsurgeon, the radiation teameffort management should beamultidisciplinary marginclearance. Dependinguponthesizeandlocation,surgical circumferential excision and the depthofinvasion,presence ofanyperineuralorvascularinvasionandthevertical Compton Broders,1885–1964, theBroders’grade(Albert morphology, American pathologist), The histology of the excised lesionshouldinclude thepathologicalpattern,cellular oftheexcised The histology Figure 39.3 Figure 39.2 Squamous cellcarcinoma. Basal cellcarcinoma. 4E of the eye, leptomeninges and the bowel mucosa. Although it accounts for less than 5 per cent of skin Althoughitaccountsforlessthan5percentofskin mucosa. leptomeningesandthebowel of theeye, arepresent,suchasthechoroid Thereforeitcanarisefromanyorganwheremelanocytes melanocytes. nodules(seeFig.39.4).Itisamalignanttumourarisingfromthe withsatellite This isaMM 3 percentofallmalignanciesworldwideandisthecommonestcancerin20–39year-olds. accountsfor itisresponsibleforover75percentofdeathscausedbyskinmalignancy. MM malignancy, lung orliver. with clinicallyinvolvednodes,70–85 percentwillhaveoccultdistantmetastases,suchasinthe percentwillhavenootherinvolvedregionalnodes.Inthose disease,70–80 with SLNB-positive shouldbeofferedandinnode-positive patientsblockdissectioniscarriedout.Ofthose (SLNB) thicknessismorethan1mm. Therefore,insuchpatients,sentinellymphnodebiopsy the Breslow regionallymphnodedissectionisdonewhentheremetastasis. Radical 1980, definitivetreatmentandindicatesprognosis. Americanpathologist).Thisisaguidetofurther 1928– Breslow, thickness(Alexander accordingtotheBreslow isreported subdermal fat.Histology biopsywitha2mm marginofskinand of distantspread.Confirmationisdonebyexcision gastrointestinal tract. type andconfirmationisbybiopsyofthenailmatrix.Amelanoticmelanoma usuallyoccursinthe spreading alsooccursunderneaththenailbed;theseareusually asuperficial discharge. MM itchiness andserosanguinous arechangeinshape,size,colour,surface, suggestingMM naevus (5–10 percent)andacrallentiginousmelanoma(2–8 percent).Thepresentingfeaturesina spreading(70 percent),nodular(15types are:superficial percent),lentigomalignamelanoma andimmunocompromised patients.Thevarious red hair,giantcongenitalpigmentednaevus dysplasticnaevi, melanoma,largenumberofnaevi, previous pigmentosum, familyhistory, Clinically, impalpable lymph nodes may have microscopic metastasis, particularly inthosewhere Clinically, impalpablelymphnodes may havemicroscopicmetastasis,particularly regionallymphnodemetastasisshouldbesought, besidesevidence In asuspectedMM toultravioletraysisthemajorcausewhilstriskfactorsareenhancedinxeroderma Exposure satellite lesionswithcervicall Figure 39.4 Malignant melanomaofscalpshowing y mphnodal metastasis. 279

39: SKIN AND SUBCUTANEOUS TISSUE SKIN AND SUBCUTANEOUS TISSUE 5A 280 spreads locally (see Fig. 39.5). A minority of these lesions may progress to SCC. Chronic exposure Chronicexposure spreads locally(seeFig.39.5).AminorityoftheselesionsmayprogresstoSCC. University. redandscaly atHarvard fromtheskinsurface, Itiselevated of dermatology This isasquamouscarcinomainsitufirstdescribed1912 byJohnTempleton professor Bowen, excision. yearsearlier).Treatment disease50 same isbytopicalapplicationof5-fluorouracilorsurgical describedtheconditionin1911,dermatologist AugusteQueyrat althoughPagetdescribedthe (Parisian erythroplasia When theconditionoccursonglanspenis,itiscalledQueyrat’s 16to sun,HPV andinorganicarseniccompoundshavebeenthoughttobepossiblecauses. Figure 39.5 Bowen’s disease:squamouscarcinoma insitu. PART 7 Head and neck

40 Elective neurosurgery 283 41 The eye and orbit 292 42 Cleft lip and palate: developmental abnormalities of face, mouth and jaws 302 43 The nose and sinuses 309 44 The ear 319 45 Pharynx, larynx and neck 330 46 Oropharyngeal cancer 341 47 Disorders of the salivary glands 351 This page intentionally left blank 40 Elective neurosurgery

Multiple choice questions ➜ Intracranial pressure C It is always present if ICP is greater than 1. Which of the following statements 25 mmHg. about intracranial pressure (ICP) in a D It can cause blindness if left untreated. normal adult are true? E It can be caused by optic nerve A ICP varies from 5 to 15 mmHg at rest. compression. B ICP decreases with coughing. ➜ C ICP decreases with normal inspiration. Hydrocephalus D ICP increases with an increase in arterial 5. Which of the following statements PCO . about hydrocephalus are true? 2 It is the diagnosis when the ventricles E ICP increases with an increase in PO . A 2 are enlarged on CT scan. ➜ Cerebral oedema B It may present with a 6th nerve palsy. It may present with sunsetting eyes and 2. Which of the following statements C a bulging fontanelle in infants. about cerebral oedema are true? D It can cause head enlargement in A It may be either vasogenic or cytotoxic. infants. B It causes increased ICP. E It can cause papilloedema. C It causes brain shift. D It can be treated with steroids. ➜ Cerebrospinal fluid (CSF) E It can be treated with phenytoin. physiology ➜ Brain metabolism 6. Which of the following statements are 3. Which of the following statements are true? true? A The total volume of CSF in adults is A The brain is dependent on glucose and 150 mL. oxygen for energy. B The total volume of CSF in the normal B The brain has considerable stores of ventricular system is 35 mL. glucose. C Normal production of CSF is 0.33 mL/ C The brain can switch easily to anaerobic min. metabolism. D CSF is produced by the choroid plexus D Brain blood flow is 55 mL/100g per min. alone E The brain can utilise proteins for energy E CSF is absorbed in the arachnoid villa production. and superior sagittal sinus. ➜ Papilloedema ➜ Obstructive hydrocephalus 4. Which of the following statements 7. What can cause obstructive about papilloedema are true? hydrocephalus? A It can be seen in raised ICP. A Meningitis B It can be associated with retinal B Tumours of the fourth ventricle haemorrhages. C Intraventricular haemorrhage

283 HEAD AND NECK ➜ ➜ ➜ ➜ 11. Which of the following statements Whichofthefollowing 11. statements Whichofthefollowing 10. are Whichofthefollowing 9. statements Whichofthefollowing 8. 284

Tuberculous meningitis Cerebral abscesses Ventriculoperitoneal shunt Lumbar puncture C B A E D C B A E D C B A E D C B A E D reaction. It canbedetected bypolymerasechain markedly reducedglucose. polymorphs, increasedproteinand increased classicallyshows CSF glucose. slightly low increasedprotein and lymphocytes, increased classicallyshows CSF are common. Mixed anaerobicandaerobicorganisms disease. congenitalheart with cyanotic There isanincreasedriskinpatients subacute bacterialendocarditis. arearecognisedcomplicationof They septicaemia). to can beblood-borne(secondary They from anairsinusinfection. can becausedbydirectspread They Seizures. Intraventricular haemorrhage ofthedistalcatheter Breakage oftheproximalcatheter Blockage Infection () anaesthetic. It isusuallydoneundergeneral hydrocephalus. It canbeusedtotreatcommunicating It canbedonetodiagnosemeningitis. It canbedonetomeasureICP. supratentorial masslesions. It iscontraindicatedincasesof Tumours ofthemidbrain. Encephalitis true? about tuberculous(TB)meningitisare about cerebral abscessesaretrue? (VP) shunt? ofventriculoperitoneal complications about lumbarpuncturearetrue? ➜ ➜ ➜ ➜ 14. What may be caused bydominant Whatmaybecaused 14. may Withwhichofthefollowing 13. 12. statements Which ofthefollowing 16. Which of the following statements Whichofthefollowing 16. 15. What may be caused bydominant Whatmaybecaused 15.

Brain lesions Brain tumours (CJD) Creutzfeldt–Jakob disease Pituitar C B A E D C B A E D C B A E D B A E D C B A E D about CJD aretrue? about CJD about pituitary adenomasaretrue? about pituitary parietal lobelesions? brain tumourspresent? definite casesmustbediscarded. Instruments usedwhenoperatingon since 1970. hasbeendiagnosedinUK Variant CJD prion protein. The transmissibleagentisanabnormal It cancausehydrocephalus. It isusuallytreatedwithantibiotics. They maypresentwith galactorrhoea. They superior quadrantanopia. maycausea In theirearlystages,they Blindness intheipsilateraleye. Dysphasia Ipsilateral hemiparesis Contralateral superiorquadrantanopia dysfunction Memory temporal lobelesions? side. Hand weaknessonthesame Acalculia andagraphia Gerstmann’s syndrome Foot dropontheoppositeside Left torightdisorientation Visual loss. Intracranial bleeds Endocrine dysfunction Focal neurologicaldeficits Seizures blood cellsandtonsiltissue. The infectiousagentispresentinwhite and brain. The infectiousagentispresentinCSF y adenomas ➜ ➜ ➜ 20. Which of the following statements Whichofthefollowing 20. statements Whichofthefollowing 19. statementsare Whichofthefollowing 17. 18. Which of the following statements Which ofthefollowing 18.

Cerebral metastases Meningiomas Astroc E D C B A E D C B A E D C B A E D C B A E D C common type. Breast canceristhe secondmost primary’. lesion –‘unknown 15 percenthavenoobvious source affect1in4cancersufferers. They renal tumoursarerare. Cerebral metastasesfromcolonand tumours. lung percentarefromprimary 40 areoftenslow-growing. They 10 percent. complete resectionisapproximately The 10-year recurrencerateafter brainoedema. cancausecytotoxic They percentaresupratentorial. 80 are usuallybenign. They are notusuallyfoundinadults. They are radiosensitivetumours. They have apoorprognosis. They grade 2tumours. are WHO They in children. fossa often presentintheposterior They true? are astrocytomas regarding pilocytic excised. canusuallybecompletely Astrocytomas tumours. areradiosensitive Astrocytomas The celloforiginistheoligodendrocyte. canpresentwithseizures. Astrocytomas aregliomas. Astrocytomas cancauseCushing’sdisease. They hemianopia. oftenpresentwithabitemporal They production. cancausereducedcortisol They true? regarding cerebral metastasesare regarding meningiomasaretrue? true? y tomas ➜ ➜ 21. Which of the following statements Whichofthefollowing 21. 24. Which of the following statements Whichofthefollowing 24. 23. Which of the following statements Whichofthefollowing 23. presenting What arethetypical 22.

haemorrhage Aneur malformations Arteriovenous D C B A E D C B A E D C B A D C B A E haemorrhage (SAH)aretrue? subarachnoid regarding aneurysmal malformations (AVMs) aretrue? regarding arteriovenous abuse andfibromusculardysplasia. cocaine Risk factorsarehypertension, SAH. of Patients usuallyhaveafamilyhistory headache andvomiting. It presentswithasuddensevere population inUK. The incidenceis150 per100 000ofthe hydrocephalus. areoftenassociated with They vessels. arepresentonthefeeding aneurysms The riskofrebleedingisgreaterif cancauseepilepsy.They year. The riskofrebleedis2–4percent per done within2hofthebleed. inmanycasesiftheLPis naked eye Xanthochromia maybedetectedbythe is diagnostic. A bilirubinpeakonspectrophotometry 12 hafterthebleed. LP canbenegativeifdonelessthan puncture (LP)istraumatic. False-positives canoccurifthelumbar of casesinthefirst24h. percent scanispositiveinover90 CT regarding diagnosisofSAHaretrue? Purpuric rash Photophobia palsy Third nerve Neck painandstiffness headache Gradual onsetofsevere features ofSAH? hypercholesterolaemia. Risk factorsarediabetes,obesityand y smal subarachnoid 285

40: ELECTIVE NEUROSURGERY HEAD AND NECK ➜ ➜ ➜ ➜ 2.A,B,C, D 1. A,C,D Answers: Multiplechoicequestions statements Whichofthefollowing 26. statements Whichofthefollowing 25. 286

Cerebral oedema Intracranial pressure Parkinson’s disease Epileps oedema. andhasnoeffectoncerebral isananticonvulsant Phenytoin steroids suchasdexamethasone. This increasesthemasseffectandsubsequent brainshift.Cerebraloedemacanbetreatedwith water frombloodvessels.Tumours suchasgliomasandmeningiomascausevasogenicoedema. oedema iscausedbyanincreasein intracellularwater. Vasogenic of oedema iscausedbyleakage orvasogenic.Cytotoxic Cerebral oedemaisanincreaseinbrain waterandcanbecytotoxic increase in falls.An Inspiration isaccompaniedbyareductioninintrathoracicpressureand thereforeICP torise. thatincreasesintrathoracicpressurewillcauseICP coughing andstraining.Anything Intracranial pressureinanormaladultvariesbetween5and15 mmHg atrest.Itrisesduring E D C B A E D C B A E Symptoms includetremoranddystonia. pallidus. Stimulation targetsincludetheglobus hypothalamus. Stimulation targetsincludethe stimulation. It canbetreatedwithdeepbrain It canbetreatedwithL-dopa. The amygdalaisinthefrontallobe. the cortex. in identifyingaseizureonsetzone computed tomography)canbehelpful (singlephotonemission Ictal SPECT hippocampus. Mesial temporalsclerosisaffectsthe tumours. dysplasias andlow-grade Good surgicaltargetsarecortical attacks. Corpus callosotomycancuredrop radiosurgery canbetreatedwithstereotactic They true? regarding Parkinson’s diseaseare aretrue? regarding epilepsysurgery y P surger CO 2 willcausecerebralbloodvesselstovasodilateandincreaseICP. y ➜ ➜ 28. Which of the following statements Whichofthefollowing 28. statements Whichofthefollowing 27.

Carpal tunnels Trigeminal neuralgia E D C B A E D C B A true? regarding trigeminalneuralgia are true? tunnelsyndromeare regarding carpal It is often associated with a red eye and It isoftenassociatedwitharedeye pain. facial It causeschronic,constantsevere It canbecausedbymultiplesclerosis. zone. near itsrootentry compression ofthe5thcranialnerve It canbecausedbyvascular arm. It isapainfulconditionaffectingthe helpful inmildcases. Wrist splintswornatnightcanbe middle fingers. and paraesthesiae inthethumb,index Patients oftenwakewithpainand pregnancy. Risk factorsincludehypothyroidismand asTinel’ssign. symptoms isknown ofthewristbringingon Flexion atthewrist. median nerve It iscausedbyentrapmentofthe runny noseontheaffectedside. y ndrome ➜ ➜ ➜ ➜ ➜ ➜ 8.A,B,C, D 7. B,C,E 6.A,B,C,E 5.B,C,D,E 4.A,B,D 3.A,D

stretch withbrainshiftsorhydrocephalus. due tohydrocephalusintheelderly. hasalongintracranialcourseandisproneto The6thnerve lossandincontinencecanbe palsy,papilloedema.Ataxia,memory double visiondueto6thnerve loss ofbrainsubstance. sunsetting eyes, irritability,vomitingandfailuretothrive. sunsetting eyes, Lumbar puncture Obstructive h Cerebrospinal fluid(CSF)ph H Papilloedema Brain metabolism The brain is dependent on glucose and oxygen for its energy anddoesnothavelargestoresof foritsenergy The brainisdependentonglucoseandoxygen of the conus (usually done at L4/5 or L5/S1) to obtain a specimen of CSF, e.g.forcultureand of theconus(usually doneatL4/5orL5/S1)toobtain aspecimenofCSF, thelevel of aneedleintothelumbarsubarachnoid space,below Lumbar puncture is theinsertion and isnotusuallyacauseofhydrocephalus. causes communicatinghydrocephalus. Encephalitisisaninfectionorinflammationofthebrain Intraventricular haemorrhagecanobstructtheaqueductorforamen ofMonro. ventricle.Midbraintumoursoftenobstruct thecerebralaqueduct. cerebral aqueductorfourth cisterns,subarachnoidspaceorarachnoidvilli. obstruction iswithinthebasal cisternorsubarachnoidspace,‘communicating’,communicate withthebasal of wherethelevel Hydrocephalus canbedividedintodifferenttypes.It‘obstructive’, i.e.doesnot subarachnoid spaceofthebrain. produce 475 isinthelumbarthecaand mL/24h.Theventricleshold35mL.restofthe CSF process.Therateofproductionis0.33mL/min.Anaverageadultwill is anenergy-dependent bythechoroidplexus fluidfromthebrain.TheproductionofCSF by diffusionofextracellular ventriclesand ofthelateral,thirdandfourth Cerebrospinal fluidismadebythechoroidplexus dilatation oftheventriclesasan Enlarged ventriclesarenotalwaysindicativeofhydrocephalus.Cerebralatrophycancause not causepapilloedema. always presentincasesofraisedICP. compressioncancauseopticatrophybutdoes Opticnerve andhaemorrhages.Untreatedpapilloedemawillcausevisualloss.Papilloedemaisnot exudates Papilloedema isgradedfromstage1to4.Initslaterstagesitcanbeassociatedwithretinal source. thebraincanswitchtousingketonebodiesasanenergy After 48hofstarvation, is55mL/100 20percentofnormal.Normalcerebralbloodflow fallsbelow blood flow ml either. Thebraincannotswitchtoanaerobicmetabolismandfunctionwillceaseif y In infants,thepresentingfeaturesmaybeincreasingheadcircumference,bulgingfontanelle, –headache,vomiting, Hydrocephalus canpresentwithsymptomsandsignsofraisedICP Meningitis often causes adhesions at the base of the brain in the basal cisternsandtherefore Meningitis oftencausesadhesionsat thebaseofbraininbasal tumoursblockingthe foramenofMonro, Obstructive hydrocephalusiscausedby,forexample, drocephalus y drocephalus ex vacuo ex effect.Theventriclestendtoincreasewithagedue y siolog y per min. per min. 287

40: ELECTIVE NEUROSURGERY HEAD AND NECK ➜ ➜ ➜ ➜ 12. A,C, D,E 11. A,C, D,E 10. A,B,C,D,E 9.A,B,C,D,E 288

Creutzfeldt–Jakob disease Tuberculous meningitis Cerebral abscesses • • • • • • • Ventriculoperitoneal shunt hydrocephalus. Thisisoftendoneinpatientswithsubarachnoidhaemorrhage. canbedrainedfromthelumbarthecatotreat of communicatinghydrocephalus,CSF fromthelumbarsubarachnoidspacecanprovoke‘coning’of CSF ofthebrainstem.Incases asICP.will bethesame Theprocedureisusuallydoneunderlocalanaesthetic. circulation,thispressure astheback,andthereisnoblocktoCSF level with theheadatsame pressurecanalsobemeasured.Ifthepatientislyingflat, microscopy incasesofmeningitis.CSF blood orbrainhavetobediscarded. entered thefoodchain.Itwasnotrecognised asanentityin1970. infected withbovinespongiformencephalitiswhichthen It appearstohavespreadfromcows since1986. has been diagnosedintheUK whitebloodcellsandtonsiltissue.Variant CJD CSF, diseaseistransmittedbyanabnormalprionprotein whichispresentinbrain, Creutzfeldt–Jakob cisterns.Itistreatedwithantibiotics. adhesions inthebasal isbecomingmorecommon.Itcancausehydrocephalus dueto chain reactionofbacterialDNA bacillimaynotbeseen.Detection bypolymerase glucose.TB increased proteinandaslightlylow Tuberculous increasedlymphocytes, classicallyshows meningitiscanbedifficulttodiagnose.CSF organisms arecommon. as bloodwillbypassthelungswhichmopupalotoforganisms.Mixedaerobicandanaerobic areatincreasedrisk, organisms inthebloodstream.Patientswithleft-to-rightshuntsheart toothdecayorgumdiseasecanalsobeasourceof e.g. subacutebacterialendocarditis.Severe or byblood-bornespread.Anyconditionwhichcausesasepticaemiacancausebrainabscess, Cerebral abscessescanbecausedbydirectspreadfromaninfectedsinus(frontalormastoid) Complications ofVPshuntincludethefollowing: Shunts can over-drain and cause low-pressure headachesorsubduralhaemorrhages. Shunts canover-drainandcauselow-pressure The shuntvalvecanstopworkingorbecomeblockedwithdebrisredbloodcells. isbreachedcancauseseizures. Any neurosurgicalprocedurewherethecortex There isa1percentriskofcausingintraventricularhaemorrhageduringshuntinsertion. The catheterscanbreak,especiallyinchildren.Acommonplaceforthisistheneck. often blockedbychoroidplexus. Itis The proximalcatheterwhichliesintheventricleiscommonestsiteofshuntblockage. with IVandintrathecalantibiotics. inflamed.Theshunthastoberemoved.infection istreated of theventriclesbecomevery Infection oftheshuntcatheterandintraventricularCSF. asventriculitis.Thewalls Thisisknown Lumbar punctureiscontraindicatedinthepresenceofintracranialmasslesions,asremoval Instruments used during surgery in known CJD cases that came into contact with CSF, tonsil, cases thatcameintocontactwithCSF, CJD inknown Instruments usedduringsurgery ➜ ➜ ➜ ➜ 18. A,D 17. A,B,D 16. A,B,C,D,E 15. A,B,D 14. A,C,D 13. A,B,C,D,E

Astroc Pituitar Brain lesions Brain tumours Brain tumours can present in many ways. They areoftenassociatedwithseizuresorfocal Brain tumourscanpresentinmanyways.They choice. Pilocytic astrocytomas canalsobefoundinadults. astrocytomas choice. Pilocytic andisthetreatmentof is oftenpossibleintheposteriorfossa tumours butcompleteexcision 10-year areradiosensitive inchildren.They oftenoccurintheposteriorfossa They survival). percent goodprognosis(90 withavery grade 1astrocytomas are WHO astrocytomas Pilocytic areradiosensitive. Many astrocytomas seizures. Gliomasaregraded1–4.Themostmalignanttype(grade4) is glioblastomamultiforme. canpresentwith them. They diffuse infiltrativetumoursanditisdifficulttocompletelyexcise andependymoma.Thesetumoursare oligodendroglioma (celloforiginistheoligodendrocyte) Othertypes ofgliomainclude aregliomas.Thecelloforiginistheastrocyte. Astrocytomas stages abitemporalhemianopiaisusual. from below. In theearlystagesthiswillcauseasuperiortemporalquadrantanopia.later cortisol. often havealow glandwillcauselossofnormalsecretionandhypopituitarism. Patients on thenormalpituitary secreting tumourstendtobelargeratpresentationandoftenpresentwithvisualfailure.Pressure hormone willcauseCushing’sdisease.Atumoursecretingprolactingalactorrhoea.Non- adenomascanbesecretingornon-secreting.Atumouradrenocorticotrophic Pituitary isintheposteriorfrontallobebutsuppliescontralaterallimbs. hemiparesis –themotorcortex temporal lobelesions.Acontralateralsuperiorquadrantanopiaisseeninthesecases. oftheopticradiationpassesaroundtemporallobeandcanbeaffectedby Part cerebral artery. lobe iscloselyappliedtotheposteriorfrontalatSylvian fissure,whichcarriesthemiddle speechandhearing.Thetemporal involvedinmemory, The temporallobestructuresarevery intheposteriorfrontallobe. down islower ofthemotorcortex tothefalx.Thehandpart next together. disorientation, fingeragnosia,acalculiaandagraphia.Gerstmann’ssyndromeiswhentheseoccur lobe.Dominantparietallobelesionscauseleft–right The parietallobeisessentiallyasensory papilloedema. toraisedpressureand Visual losscanbecausedbythetumouritselforpresentsecondary tumours haveapropensitytobleedandmaypresentaslife-threateningintracerebralbleed. andhypothalamictumourscancauseendocrinedysfunction.Some neurological deficits.Pituitary Temporal oranipsilateral lobelesionswouldnotcauseblindnessintheipsilateraleye ontheuppermedialside isintheposteriorfrontalcortex ofthemotorcortex The footpart A pituitary tumour will grow out of the pituitary fossa and compress the optic nerve orchiasm andcompresstheopticnerve fossa outofthepituitary tumourwillgrow A pituitary y y tomas adenomas 289

40: ELECTIVE NEUROSURGERY HEAD AND NECK ➜ ➜ ➜ ➜ 24. A,B,C,E 23. A,B,C,D 22. B,C,D 21. B,D 20. A,B,C,D,E 19. A,B,D,E 290

Arteriovenous malformations Aneur Cerebral metastases Meningiomas Meningiomas are usually benign tumours; 80 percentarefoundsupratentorially.Meningiomas areusuallybenigntumours;80 cancause They combination ofthese tofullytreatalargeAVM. endovasculartechniques orstereotacticradiotherapy.can betreatedby surgery, Itoftentakesa vessels. AVMs oftencause epilepsybutarenotusuallyassociatedwithhydrocephalus.They The riskofrebleedis2–4percent per year. areseenonthefeeding Thisisgreaterifaneurysms LP122 hafterthebleed.Itisusualtoperform haftertheonsetofheadache. Xanthochromiawillnotbedetectableto thenakedeye A bilirubinpeakisdiagnosticofSAH. helpful. isvery ofCSF Atraumatictapcancauseconfusion.Spectrophotometry develop. takestimeto tooearlyasxanthochromia LPshouldnotbeperformed LP mustbeperformed. scanisnegative, isgoodandtheCT within thefirst24hafterbleed.Ifhistory accuracy percent scanalone with>90 canbediagnosedonCT scanners,SAH With third-generationCT mayruptureatanytime. as theaneurysm emergency Itisaneurosurgical causingittopressonthethirdcranial nerve. oftheaneurysm, rapid expansion movements. Thisisdueto palsywiththepupilbeingaffectedbeforeeye a painfulthirdnerve maypresentwith oftenpresent).Aposteriorcommunicatinganeurysm withrashandfever slowly, consciousness. Thefeaturesneedtobedistinguishedfrommeningitis(headachecomesonmore vomiting, neckstiffnessandphotophobia.Neckpainmayalsobepresent.Somepatientslose sudden,headacheassociatedwithnauseaand The presentingfeaturesaretypicallysevere, formation. predisposes toaneurysm cocaine abuseandfibromusculardysplasia,whichleadstoweaknessofbloodvesselwalls depends ontheinitialgrade,amongotherthings. scoringsystem.Grade1isagoodgradeand5theworst.Prognosis using theWFNS ofthebleedisscored pain mayalsobepresent.Somepatientsloseconsciousness.Theseverity sudden, headacheassociatedwithnauseaandvomiting,neckstiffnessphotophobia.Neck usuallypresentswithasevere, malformation.SAH e.g. trauma,bleedfromanarteriovenous SAH, is15/100 SAH The incidenceofaneurysmal Thereareothercausesof 000populationintheUK. lesion isundiagnosed. tumour.The commonesttypeofbraintumourisasecondary In15 percentofcasestheprimary bybreastcancer.closely followed Brainmetastasesfromrenalandcoloncancersaremuchrarer. tumourthatgivesrisetocerebralmetastasesisbronchogeniccarcinoma, The commonestprimary a widemarginofassociatedduracanalsobeexcised. on microscopy. morerapidlyandhaveanincreasedriskofrecurrence. Thesemaygrow atypicalfeatures show butasmallproportion areoftenslow-growing meninges ofthebrain.They arisefromthe vasogenic brainoedemaandseizuresaswellfocalneurologicaldeficits.They If a meningioma is completely excised, therecurrencerateisapproximately10If ameningiomaiscompletelyexcised, percentunless Most patients with SAH do not have a family history of SAH. Risk factors are hypertension, Riskfactorsarehypertension, ofSAH. donothaveafamilyhistory Most patientswithSAH y smal subarachnoidhaemorrhage ➜ ➜ ➜ ➜ 28. A,C,D,E 27. B,C 26. A,B,D,E 25. A,B,C,D

Carpal tunnels Trigeminal neuralgia Parkinson’s disease Epileps Corpus callosotomy can often cure drop attacks. Focal abnormalities such as cortical dysplasiasand Corpus callosotomycanoftencuredropattacks.Focal abnormalitiessuchascortical nerve conduction tests. Sometimes surgery isrequiredtodecompressthecarpaltunnel. conductiontests.Sometimessurgery nerve over thecarpaltunnelandelicitingparaesthesia. bytappingthenerve asPhalen’stest.Tinel’stest is performed on thesymptoms.Thisisknown ofthewristmaywellbring inmanycases.Flexion provoked bydrivingandreadinganewspaper andmiddlefingers.Symptoms canbe at nightwithpainandparaesthesiainthethumb,index sheaths andtransversecarpalligament),hypothyroidismpregnancy. Patients typicallywake include age(dueto‘wearandtear’changesinthecarpalbonesthickeningoftendon atthewrist.Causes tunnelsyndromeiscausedbycompressionofthemediannerve Carpal zone. closetoitsrootentry a vascularloopcanbeseenincontactwiththe5thnerve, eating orbrushingteethbylighttouch.Itcanbecausedmultiplesclerosisbutinmanycases andepisodic.Thepaincanbeprecipitatedby Thepainissevere divisions ofthetrigeminalnerve. Trigeminal neuralgiaisapainfulconditionaffectingone-halfoftheface.Itcanaffectoneormore involved inParkinson’disease. for stimulationincludethesubthalamicnucleusandglobuspallidus.Thehypothalamusisnot Parkinson’s diseaseisL-dopa.Somepatientscanbetreatedwithdeepbrainstimulation.Targets Parkinson’s diseasepresentswithtremor(pill-rolling)anddystonia.Theusualtreatmentfor seizure foci.Theamygdalaisamedialtemporalstructure. isbeingincreasinglyusedtoidentify scan.IctalSPECT This canbeseentoabnormalonMRI tumourscanbegoodsurgicaltargets.Mesialtemporalgliosisaffectsthehippocampus. low-grade A red eye andrunnynoseisnottypicaloftrigeminalneuralgia. A redeye In mildercases,wristsplintswornatnightcanbehelpful.Thediagnosis canbeconfirmedwith y surger y y ndrome 291

40: ELECTIVE NEUROSURGERY 41 The eye and orbit

Multiple choice questions ➜ Tears ➜ Basal cell carcinoma 1. How do tears produced in the 5. How should a recent-onset suspected lacrimal gland enter the conjunctival basal cell carcinoma of the eyelid skin sac? be treated? A Through the nasolacrimal duct A Photograph and clinical follow-up B Through the lacrimal duct B Liquid nitrogen cryotherapy C Through the 10–15 lacrimal gland ducts C Curettage D Through the meibomian orifices D Excision biopsy E Through the ducts of Moll. E Radiotherapy. ➜ Spread of infection ➜ Infection of the tear sac 2. What is a potential route of spread of 6. What is the typical cause of acute infection from the eyelid skin to the infection of the tear sac (acute intracranial cavity? dacryocystitis)? A Orbital lymphatics A Spread of infection from the ethmoid B Ophthalmic veins sinus C Subperiosteal potential spaces B Nasolacrimal duct obstruction D Facial artery C Spread of infection from the maxillary E Perineura of branches of the trigeminal sinus nerve. D Spread of infection from the nasal mucosa ➜ Meibomian cysts E Viral infection of the lacrimal gland. 3. What does a meibomian cyst consist of? A A meibomian eyelash follicle infection ➜ Dysthyroid exophthalmos B A sebaceous (‘meibomian’) cyst of the 7. In acute dysthyroid exophthalmos, eyelid margin skin when should urgent orbital C A meibomian sweat gland inclusion cyst decompression surgery be D Chronic granulomatous inflammation of considered? a meibomian gland A Diplopia is present. E An epidermoid cyst of embryonic B Raised intraocular pressure is present. meibomian structures. C Fat prolapse into the eyelids is present. D Eyelid oedema is present. 4. How are persistent meibomian cysts E Compressive optic neuropathy is present. treated? A Incision and curettage from the ➜ Retinoblastoma conjunctival surface 8. How does retinoblastoma, a B Incision and curettage from the eyelid malignant tumour of the retina, margin surface commonly present in young children? C Incision and insertion of a small pack A Reduced vision D Long-term oral antibiotic therapy B Pain E Excision biopsy.

292 ➜ ➜ ➜ ➜ 2 Whatdoesthefindingofhyphaema 12. fracture ofthe isa‘blow-out’ How 11. isherpessimplexinfectionofthe How 10. Whatisthemostcommonocular 9.

H Fracture oftheorbit the cornea Herpes simplexinfectionof Ocular tumour B A E D C B A E D C B A E D C B A E D C y phaema the development ofglaucoma. the development given immediatelyinordertoprevent treatmentshould be That thrombolytic cornealstaining. prevent be washedoutpromptly,inorderto That theanteriorchambershould antrum. Soft-tissue prolapseinthemaxillary Air intheorbit antrum Blood inthemaxillary Blood intheethmoidsinus papyraceum Fracture oftheethmoidlamina sclera. lump atthejunctionofcorneaand The presenceofaninflamed‘phlycten’ staining. the cornealepitheliumonfluorescein The presenceofaherpeticvesicle epithelial fluoresceinstaining. The presenceofabranchingpattern skin. on theeyelid The presenceoftypicalherpeticvesicles Viral cultureofcasesconjunctivitis. Choroidal epithelioma. Retinal glioma Choroidal sarcoma Choroidal malignantmelanoma Lung carcinomametastasestotheretina Regional lymphadenopathy. A whitepupilreflex A swolleneye following blunttrauma indicate? following seenonradiography? orbit typically diagnosed? cornea typically tumour inadults? ➜ ➜ ➜ 4 Whenanintraocular foreignbody 14. Afront-seatpassengerinaroad 13. 15. Which of the following aremostlikely Whichofthefollowing 15.

Intraocular foreign bod Irregular pupil necrosis withischaemic burns Tissue C B A E D C B A E D C E D C B A E D traffic accidentpresentswithfacial ischaemic necrosis? severetissueburnswith to cause ordered? imaging techniquesshouldnotbe is suspected,whichofthefollowing due toahammerandchiselinjury of theirregularpupil? one eye.Whatisthemostlikelycause lacerations andanirregularpupilin especially oftheretina. forassociatedinjuries, examined shouldbecarefully That theeye scan shouldbeorderedimmediately. haemorrhage islikely,andacranialCT That associatedintracranial haemorrhage. secondary immediately inordertoprevent shouldbeinstituted the affectedeye, That strictbedrest,withbandagingof Ionising radiation. Thermal energy Ultraviolet radiation chemicals Alkaline Acid chemicals Optical tomographyoftheeye. andorbit eye scanofthe Computed tomography(CT) andorbit of theeye scan Magnetic resonanceimaging(MRI) oftheeye Ultrasound examination andorbit Plain radiographoftheeye due totraumaticcataractdevelopment. oftheanteriorlenscapsule Distortion Traumatic Horner’ssyndrome injury thirdcranialnerve A partial Blunt traumatothepupilsphincter some irisprolapse oftheglobe,with injury A perforating y 293

41: THE EYE AND ORBIT HEAD AND NECK ➜ ➜ ➜ ➜ ➜ Extended matching questions Extended referto? WhatdoesLASIK 19. suspected Whichofthefollowing 18. maycause Whichofthefollowing 17. topical Whichofthefollowing 16. 294

1. Initialinvestigations Surgical techniques Investigation oflossvision Painful, red e infection Herpes simplexcorneal G F E D C B A B A E D C B A E D C B A E D C B A Send bloodforconnective tissuediseasesscreen. Arrange urgentabdominalultrasound. andCRP.Send bloodforESR scanoftheorbits. Arrange urgentCT No investigations.Contacttheduty ophthalmologist. Measure bloodpressure,random glucoseandfullbloodcount. Arrange plainskullX-rays. Laser-assisted interstitialkeratectomy Laser insitukeratomileusis Macular degeneration. Cranial arteritis Posterior vitreousdetachment Central retinalveinocclusion Retinal detachment Keratitis. Episcleritis Acute glaucoma Iritis Conjunctivitis drops. eye Non-steroidal anti-inflammatory drops Chloramphenicol eye drops Steroid eye drops Atropine eye Aciclovir ointment C-reactive protein (CRP) level? C-reactive protein(CRP) and sedimentation rate (ESR) measurement oferythrocyte diagnoses requiresimmediate vomiting? a painful,redeyeaccompaniedby herpes simplexcornealinfection? for treatments arecontraindicated y e 22. What isusedinincisionandcurettage 22. Whatisremovedintheoperation of 21. might how surgery, cataract Following 20. C B A E D C B A E D C B A E D C E D capsulotomy laser Photodisruptive eximer Photoablative argonlasercapsulotomy garnet) lasercapsulotomy Photodisruptive YAG aluminium (yttrium Ruby lasercapsulotomy tissue becut? thickened posteriorlenscapsule Lateral segmentinterstitialkeratotomy. Laser epithelialkeratomileusis keratotomy Long-acting stromalinhibition An eyelid creaseskinincision. An eyelid lid margin conjunctivaincisionradialtothe A tarsal to thelidmargin conjunctivaincisiontangential A tarsal A skinincisionradialtothelidmargin margin A skinincisiontangentialtothelid of ameibomiancyst? within thesclera. The corneaandthecontentsofeye body oftheeye lensandthevitreous The crystalline The vitreousbodyoftheeye within theorbitalperiosteum The completecontentsoftheorbit, muscles oftheextraocular portions alongwithshort The intacteyeball, eyeball evisceration? Diode lasercapsulotomy. ➜ Choosethecorrectdiagnosisforeachof scenariosgivenbelow: scenarios: ChoosethecorrectactiontobetakenbyA&Edoctorwitheachoffollowing

5 4 3 2 1 2. E 5 4 3 2 1 J I H G F E D C B A J I H skin of the right lower eyelid. Thelumphasasmall,centralcrater thattendstobleedattimes. eyelid. skin oftherightlower A 70-year-old enlargingpearl-coloured,firm,smoothlumponthe malepresents withaslowly hasbeenintermittentlyredandirritable for6weeks. eye Theleft A 9-year-oldgirlpresentswithasmall, firmlumpatthemarginofleftuppereyelid. of thelefteye. A 72-year-oldfemalepresentswithatense,fluctuantswellingjustmedial tothemedialcanthus monthsandremainsunchangedinsize.Itis notpainful. Ithasbeenpresentforseveral eyelid. A 16-year-old malepresentswithaslightlyred,firmlump deeptotheskinofleftlower andorbitappearnormal. underlying eye The of thelefteye. A 2-year-oldchildpresentswithacuteswellingandrednessoftheeyelids What initialinvestigation(s)wouldyouarrange? redspotatthefovea.You occlusion. retina ispale,withacherry suspectacentralretinalartery A 70-year-old Onophthalmoscopythe malepresentswithacutelossofvisioninthelefteye. suspected. Whatinitialinvestigation(s)wouldyouarrange? andacentralretinalveinocclusionis there arenumerousretinalhaemorrhagesintheeye, A 70-year-old Onophthalmoscopy femalepresentswithacutelossofvisionintherighteye. feelshardtopalpation.Whatinitialinvestigation(s)wouldyouarrange? the eye present for18 thepupilislargeanddoesnotreact, isred,thecorneahazy, h.Theeye A 67-year-old andvomiting.Thesymptomshavebeen femalepresentswithapainfullefteye investigation(s) wouldyouarrange? and headache.You Whatinitial areconcernedaboutapossibleintracranialaneurysm. palsy A 63-year-oldmalepresentswithanacute-onset,completerightthirdcranialnerve You Whatinitialinvestigation(s)wouldyouarrange? suspectcranialarteritis. ofscalptenderness.Onophthalmoscopytherightopticdiscispaleandoedematous. history andgivesa3-week An 83-year-oldwomanpresentswithsuddenlossofvisionontherighteye, Basal cellcarcinoma Basal Cyst ofMoll Meibomian cyst Preseptal cellulitis Dermoid cyst Sebaceous cyst Lacrimal glandtumour Molluscum contagiosum mucocele Lacrimal sac Squamous cellcarcinoma ultrasound. Measure bloodpressure,randomglucoseandrequestcarotidduplex usingDopplerultrasound. perfusion Request urgentmeasurementofophthalmicartery angiogramwiththedutyneuroradiologist. Discuss urgentMR y elid lumps 295

41: THE EYE AND ORBIT HEAD AND NECK ➜ ➜ Choosethecorrectmanagement actionsforeachofthescenariosgivenbelow: Choosethecorrectdiagnosisforeachofscenariosgivenbelow: 296

3 2 1 4. Managementofoculartrauma 5 4 3 2 1 3. Causesofred e J I H G F E D C B A J I H G F E D C B A conjunctival laceration justlateraltothecornea,with associated subconjunctivalhaemorrhage. suddenlybecamepainful,withblurred vision.Thereisasmall protection. Hisright eye A 36-year-oldmalewashammering ametalblockwithchisel.Hewasnotwearingeye cornea. You instillocalanaesthetic, andremovetheforeignbodywithaneedle. A 48-year-oldmalewasgrindingmetal andpresentswithametalforeignbodyadherenttothe attempted upwardgaze. movementon hecomplainsofdoublevisionandhasrestrictedeye isintact.However, the eye but Thereissomebruisingof the eyelids, A 23-year-oldmalewaspunchedintherighteye. temporal andsuperiorscleraisadeepredcolour,withsomeoverlyingoedema. the Onexamination painintherighteye. A 55-year-oldfemalepresentswithmoderatelysevere crusted andstucktogether. Hervisionremainsnormal. A 12-year-old are Onawakeninginthemorning,eyelids eyes. girlpresentswithred,sticky Hervisionisnormalandshehasnodiscomfort. of therighteye. ofthesclera A 63-year-oldfemalepresentswithabrightreddiscolorationofthetemporalpart staining ofthecornealepithelium. isslightlyred.Whenfluoresceindyeinstilled,thereanareaofbranching-shaped the eye Thevisionisslightlyblurredand A 33-year-oldfemalepresentswithanirritablerighteye. is smallandirregular. vision isslightlyreduced,therepinkcolorationaroundtheedgeofcornea,andpupil the Onexamination A 23-year-oldmalepresentswithpainandphotophobiaoftherighteye. Irrigate thecorneaandconjunctivawithweakaceticacid,untilpH is neutral. Take acornealscrapeforbacterialculture. underanaestheticandsurgicalrepairasrequired. Arrange examination andorbits. scanoftheeyes Arrange anMRI scanofthefacialandorbitalbones,toincludecoronal views. Arrange aCT oftheorbits. Arrange ultrasoundexamination scanoftheeyes. Arrange plainX-rayorCT withsaline irrigationofthecorneaand conjunctivalsac Instil localanaestheticandperform tarsorrhaphy. atemporary Commence prophylacticintravenousantibioticsandperform padovertheeye. Instil antibioticointmentandplaceaneye keratitis. Herpes simplex Scleritis Bacterial keratitis Adenovirus conjunctivitis Bacterial conjunctivitis Scleritis Uveitis Subconjunctival haemorrhage Allergic conjunctivitis Acute angleclosureglaucoma y e ➜ ➜ ➜ ➜ ➜ ➜ 7. 7. E 6. B 5. D 4. A 3. D 2. B 1. C Answers: Multiplechoicequestions

D Infection ofthetearsac Basal cellcarcinoma Meibomian c Spread ofinfection Tears 5 4 pending surgery. control, therapymaybeusedtoprovidetemporary in reducedvision.Systemic glucocorticoid results may beneededwhencompressionoftheopticnerve Orbital decompressionsurgery bypassoperationshouldbeperformed. dacryocystorhinostomy initialantibiotictreatment,a occurs.Following If thisbecomesinfected,acutedacryocystitis Obstruction ofthenasolacrimalductmayleadtoformationamucocele withinthetearsac. micrographic surgicalexcision. lesionsmay requiretreatmentwithMohs’ hasbeencomplete.Moreextensive that theexcision biopsy,with histologicalconfirmation cellcarcinomamaybetreatedbyexcision Recent-onset basal shouldbebiopsied. or recurrentcysts Atypical maybetreatedbyinclusionandcurettagefromtheconjunctivalsurface. meibomian cysts persistent oftenresolvespontaneouslyoveraperiodofmonths.However, Meibomian cysts a meibomiangland. isachronicgranulomatousinflammationof around meibomianlipidsecretions.Acyst ofthemeibomianglandsresultinaccumulationgranulomatousreaction Retention cysts margin. plates,and openattheeyelid The meibomianglandsaresituatedwithinthetarsal and posteriorlywiththecranialcavity. Thesechannelscanprovidearouteforthespreadofsepsis. The orbitcontainsbranchesoftheophthalmicveins,whichanastomoseanteriorlywithface through 10–15 eye. ducts.Trauma, scarringinthisareacanresultadry orinflammatory surgery The lacrimalglandliesundertheupper,outerorbitalrimandopensintoupperconjunctivalfornix y is shallow andyouthinktheremaybesomeprolapsediristissue. is shallow theanteriorchamber is alargevolumeofsubconjunctivalhaemorrhage.Thepupildistorted, duringafight.Thevisionisreducedandthere A 55-year-oldmanwaspunchedintherighteye painful. very isnow andtheeye the liquidsplashedintohisrighteye liquid.Someof A 33-year-oldmalewaswashingoutaglasscontainerthatcontainedalkaline sth y roid exophthalmos y sts 297

41: THE EYE AND ORBIT HEAD AND NECK ➜ ➜ ➜ ➜ ➜ ➜ ➜ ➜ 15. B 14. C 13. A 12. E 11. E 10. C 9. B 8. D 298

Tissue burns withischaemicnecrosis burns Tissue Intraocular foreign bod Irregular pupil H Fracture oftheorbit Herpes simplexinfectionofthecornea Ocular tumour Retinoblastoma These tumoursareoftencalcified,andfrequentlypresentwiththeappearanceofawhitepupil Immediate irrigationisrequired,and shouldbecontinueduntilaneutralpHisobtained. ischaemiaandnecrosis. chemicalspenetratetissuesquickly, andcancauseextensive Alkaline withresultingiatrogenictrauma. will resultinmovementoftheforeign bodywithintheeye, Intraocular foreignbodiesareoftencomposedofferrousmetalinthis situation. Amagneticfield surgical repair. underanaestheticandprimary with ashield,pendingexamination butshould beprotected examined, shouldnotbeextensively displacement oftheiris.Theeye dueto cornea andadjacentscleraresultinareasofirisprolapse.Thepupilbecomes distorted, injuriesofthe injuriesarelesscommonwhenseatbelts areworn.Perforating eye Perforating especiallytheretina,maybepresent. injuries tootherstructuresoftheeye, associated haemorrhage,withrelatedraisedintraocularpressure.However, bysecondary followed blunttrauma.Hyphaemacanbe Hyphaema referstobloodintheanteriorchamberfollowing movementresults. Reduced upwardeye prolapseortheinferiororbitaltissues. fracture,withdownward trauma. Thisresultsina‘blow-out’ byblunt The flooroftheorbitisweakestpointwhenorbitalpressureacutelyelevated infectioncausesatypicalbranching(‘dendritic’)disturbance. Herpessimplex patternofstaining. inbluelight,demonstratesanyepithelial Fluorescein stainingofthecornealepithelium,viewed is theliver. butenucleationisneededinsomecases.Themostcommonsiteofmetastasis theeye, preserve for theiris.Treatment inorderto orprotonbeamirradiationmaybeperformed bybrachytherapy adults. Itmostoftenarisesinthechoroid,andprognosisislessgoodforchoroidlesionsthan body,choroid)isthemostcommontumourin Malignant melanomaoftheuvea(iris,ciliary performed. fundus examination mayalsopresentwithasquint,andallchildrensquintshouldhavecareful They reflex. y phaema y ➜ ➜ ➜ ➜ ➜ 2H 1E matchingquestions Answers: Extended 22. D 21. E 20. B 19. A 18. D 17. C 16. C

1. Initialinvestigations Surgical techniques Investigation oflossvision Painful, red e infection Herpes simplexcorneal Herpes simplex corneal infections show acharacteristicbranching(‘dendritic’) cornealinfectionsshow Herpes simplex patternwhen aneurysm. Prompt treatment may prevent catastrophicsubarachnoidhaemorrhage. Prompttreatmentmay prevent aneurysm. palsyandpainissuggestiveofanintracranial The combinationofacutethirdcranial nerve bilateralblindness. mayprevent Prompttreatmentwithglucocorticoids elevated. areusually andCRP TheESR ofsuspicionshould be maintainedforcranialarteritis. A highindex plate. ofthetarsal shortening is radialtothelidmargin,inlinewithaffectedmeibomiangland, ordertoavoidcicatricial inordertoavoida visiblescar. oftheeyelid, The incisionismadeontheinnersurface Theincision choroid) areremoved. bodyand intraocular tissuesareremovedwithacurette.Allfragmentsoftheuvealtract(iris,ciliary andall Evisceration referstoremovalofalltissueswithinthesclera.Thecorneaisexcised, the posteriorlenscapsule,irisandvitreousbandsmaybedivided. pulsesofYAGShort producephotodisruption,withnothermaleffect.Tissuessuchas laserenergy flap isthenrepositioned. laser. cornealflapiscut,andalayerofstromaablatedwithaneximer The A superficial optic neuropathy. anteriorischaemic blindnessduetobilateralarteritic canprevent therapy withglucocorticoids, Promptdiagnosis,leadingtoprompt incranialarteritis. elevated areoftenvery andCRP The ESR the conditionmaybemistakenforanacuteabdominaldiagnosis. visceralpainthataccompaniesacuteglaucomamaycausevomiting,and palpation. Thesevere, on ofintraocularpressureresultsinanoedematouscorneaandahardeye Acute elevation cornealscarring. in severe dropscausemarkedworseningoftheinfection,whichmayresult aciclovir ointment.Steroideye shouldbetreatedwithtopicalantiviralpreparationssuchas stained withfluoresceindye.They y e 299

41: THE EYE AND ORBIT HEAD AND NECK ➜ ➜ 3C 2J 1D 5J 4C 3B 2H 1G 5J 4B 3C 300

3. Causesofred e 2. E for anacuteabdominalpathology. pain,whichmayresultinvomitingandbemistaken Acute angleclosureglaucomacausessevere hypertension. However, typicallynounderlying causeisevident. However, hypertension. tobeassociatedwith aresaid straining,coughing,sneezing orminortrauma.They occur following may Subconjunctival haemorrhageslook dramaticbutareofnofunctionalsignificance.They worsening oftheconditionandpermanentscarring. be avoided,asthiswillcausesevere disturbance ofthecornealepithelium.Treatment iswithaciclovir ointment.Steroidtreatmentmust infectionofthecornealepitheliumtypicallycausesa branching-patterned Herpes simplex ofthecornea.Treatmentsurface drops. dropsand steroideye iswithpupil-dilatingeye cellsmaybeseenontheinner adhesions betweentheirisandlens.Depositsofinflammatory Anterior uveitis,inwhichtheirisisinflamed,resultsspasmofpupil sphinctermuscleand withagenerousmargin. must befullyexcised arelocallyinvasive,and skinneoplasia.They cell carcinomasareacommonformofeyelid Basal ortreatedwithcryotherapy. may becuretted,excised Viruses shedfrommolluscumcontagiosumlesionscausearecurringviralconjunctivitis.Thelesion surgery. Recurrent infectionmayoccur,andthedefinitivetreatmentconsistsofdacryocystorhinostomy areoccluded. fromthesac occurswhenproximalanddistaloutflow A mucoceleofthelacrimalsac curetted ifthisisdesired. and resolvespontaneously,butmaybeincisedontheinneraspectofeyelid, slowly They ofthemeibomianglands,andcontainsebaceousmaterial. areretentioncysts Meibomian cysts needed. isoftenduetounderlyingsinusdisease.Aggressiveantibiotictreatment Cellulitis oftheeyelids surgery. endarterectomy is basedonreductionofcardiovascularriskfactors,aspirinandassessmentforpossiblecarotid shouldbesought.Therapy In additiontoaskingifthepatientsmokes,causesofarteriosclerosis outpatientappointment. should bereferredfora‘soon’ophthalmology syndromes(andglaucoma).Theseshouldbetreated,ifdetected,andthepatient hyperviscosity diabetesand Underlying causesforcentralretinalveinocclusionsincludehypertension, y elid lumps y e ➜ 5H 4C 3D 2A 1F 5E 4F

4. Managementofoculartrauma condition isself-limiting. asthe thedurationofinfection,butareunnecessary, dropsslightlyshorten Antibiotic eye Bacterial conjunctivitisisacommon,self-limitingcondition.Typically thereisapurulentexudation. injury should be explored, and a primary repairperformed. andaprimary shouldbeexplored, injury underanaesthetic.The shouldbeperformed injury. Amorecompleteexamination result infurther detailmay infurther theeye hasbeenruptured.Attemptstoexamine likelythattheeye It isvery denature proteinsandpenetratedeeplyintothetissues. substancesaremuchmoredamaging thanotherchemicals,asthey have beenremoved.Alkaline chemical untilalltracesofalkaline shouldbeusedtoirrigatetheeye, A largevolumeofsaline injury. field willcausemovementofanintraocularferrousforeignbody,withfurther scanningisabsolutelycontraindicated,asamagnetic MRI bypressingontheeye. injury further gently,butthereisariskofcausing maybedonevery scan.Ultrasoundexamination X-ray orCT smallradio-opaqueforeignbodiesmaybedemonstratedwithaplain circumstance. Evenvery There isahighlikelihoodofthepresencemetallicintraocularforeignbodyinthis appropriate foracornealabrasion. padaregenerallyusedinthiscircumstance.Thistreatmentisalso ointment andaneye ispainful.Antibiotic After removalofacornealforeignbody,thereisriskinfectionandtheeye oftheorbitalfloor. scanviews antrum,willbedemonstratedwithcoronalCT maxillary soft tissuesattachedtotheinferiorrectusmuscle.Thefracture,andprolapsedinto fractureofthefloororbit,withentrapment The clinicalfeaturesaretypicalof‘blow-out’ areneeded. cases,glucocorticoids more severe drugs,butin Theinflammationmaybecontrolledwithnon-steroidalanti-inflammatory arthritis. Scleritis isaformofconnectivetissueinflammation,frequentlyassociatedwithrheumatoid 301

41: THE EYE AND ORBIT 42 Cleft lip and palate: developmental abnormalities of face, mouth and jaws

Multiple choice questions ➜ Cleft palate – general B Environmental maternal epilepsy is one 1. Which of the following statements factor not associated with clefts. about cleft palate are true? C Down’s syndrome may be associated A The incidence of cleft lip and palate is 1 with clefts. in 6000 live births. D Apert’s and Treacher–Collins syndromes B The incidence of cleft palate is 1 in are not associated with clefts. 1000 live births. E An isolated cleft palate is more C The typical distribution of cleft lip alone commonly associated with a syndrome is 35 per cent. than cleft lip alone. D The typical distribution of isolated cleft ➜ palate is 40 per cent. Pierre Robin syndrome E Cleft palate alone is more common in 4. Which of the following statements are males. true? A Pierre Robin syndrome is the ➜ Orofacial congenital commonest syndrome associated with abnormalities clefts. Pierre Robin syndrome includes 2. Which of the following statements B glossoptosis. regarding congenital abnormalities Retrognathia is not a feature of the are true? C Pierre Robin syndrome. A The most common congenital Pierre Robin syndrome is named after abnormalities of the orofacial structures D the first patient in whom the condition are cleft lips, alveolus and palate. was described in 1729. B They are also an associated feature in Pierre Robin syndrome is associated over 300 recognised syndromes. E with early respiratory and feeding C There is an increased incidence in the difficulties. black population. D Genetics and the environment both play ➜ Muscle and structural a part in causation. E Family history with a first-degree relative considerations affected increases the risk to 1 in 100 5. Which of the following statements are live births. true? A In cleft lip there is disruption of the ➜ Causes and syndromes 2 groups of muscles of the upper lip 3. Which of the following statements and nasolabial region. regarding causal factors of cleft lip B In bilateral cleft lip, the disruption is and palate are true? associated with a prolabium. A Environmental factors are less important C Prolabial tissue contains muscle tissue. for cleft palates than for cleft lip/palate.

302 ➜ ➜ ➜ 8. Which of the following statementsare Whichofthefollowing 8. statementsare Whichofthefollowing 7. statementsare Whichofthefollowing 6.

structure inclefts problems andmuscle Pierre Robins considerations respirator Use ofscansincleftsand consideration Further anatomical A E D C B A E D C B A E D in PierreRobinsyndrome problems. isnota procedureused Labioglossopexy inPierreRobinsyndrome. exclusively problemsoccur Major respiratory palate feedwellandthrive. Most babiesbornwithacleftlipand made before15 weeksofgestation. Diagnosis byscanofacleftcanbe and counselling. to-be shouldgetappropriatesupport aresultofscanresults,theparents- As diagnosis ofcleftpalates. Antenatal scansareusefulintheearly lahSh. onto thehardpalatearerepresentedby incomplete cleftofsoftpalateextending incomplete rightunilateralcleftlipand classification,the Using theLAHSHAL describes thefeaturesofacleft. systemofclassification The LAHSHAL isnotintact. mucosa In asubmucoussoft-palatecleft,the into theposterioredgeofhardpalate. fibres areorientatedwronglybutinsert acleftofthesoftpalate,muscle In separated fromthepalatineprocesses. septum andvomerarecompletely In acompletecleftpalate,thenasal the 2palatineshelves. Cleft palateresultsinfailureoffusion structures anteriortotheincisiveforamen. palateisdefinedasthe The secondary true? true? true? y andnutritional y ndrome ➜ ➜ ➜ 10. Which of the following statementsare Whichofthefollowing 10. statementsare Whichofthefollowing 9. 11. Which of the following statements Whichofthefollowing 11.

care oftheear Team approach incleftsand Repair ofcleftlipandpalate Speech problems C B A E D C B A E D C B B A E D true? patients aretrue? regarding speechproblemsincleft true? not encourage normal facial growth in not encouragenormalfacialgrowth Restoration ofnormalanatomydoes Tensor palatiisasoftpalatemuscle. palate. There are4musclesofthesoft the muscles. theemphasisisrepairof In cleftsurgery sleep. awake PierreRobinbabythanduring Hypoxia ismorelikelytooccurinthe months. at18 shouldbeperformed Assessment palate patients. Speech problemsarecommonincleft be doneduringchildhood. testsshouldalways Regular audiology effusion. temporarily eliminatemiddleear myringotomy andgrommetinsertion Early (6–12 months)prophylactic media. dysfunction isnotthecauseofotitis In cleftpalate,Eustachiantube isnotrequired. Long-term review teamapproach. multidisciplinary Management ofcleftsrequiresa cleft lipclosure. methodfor lip istheonlysatisfactory The Delairemethodofrepairacleft more tissuedamageoccurs. A two-stagerepairofcleftpalatemeans between19performed and24months. Cleft palaterepairisfrequently between 3and6monthsofage. Cleft liprepairisusuallyperformed cleft surgery. 303

42: CLEFT LIP AND PALATE HEAD AND NECK ➜ ➜ ➜ 14. Which of the following statementsare Whichofthefollowing 14. statementsare Whichofthefollowing 13. statements Whichofthefollowing 12. 304

Bone grafts Revisional surger Dental care A E D C B A E D C. B A E D C an osseointegrated dentalimplant. Alveolar bonegrafts areabletoreceive palate. closing residualfistulaoftheanterior Alveolar bonegraftsareusefulin considered. are longbeforeorthodontics performed Alveolar bonegraftsshouldbe cosmetic areaofthesoftpalate. isanimportant The Cupid’sbow reconstruction ofskindeformity. deformityconfirmsincomplete Nasal inadequate. muscle repairhasbeenjudged delayed for2yearsunlesstheoriginal repaired cleftlipsshouldusuallybe inpreviously lipsurgery Revisional done at14–18 years. archesis ofthemaxillary Expansion years. carried outat8–10 yearsand14–18 treatmentiscommonly Orthodontic patients. problems ofteethrarelyoccurincleft An abnormalnumberoreruption poorly maintained. in caseswheredentitionisdiseasedor careshouldonlybedone Orthodontic issue incleftcases. Tooth managementisnotusuallyan and speechtrainingdevices. speech andlanguagetherapy,surgery Speech problemsaremanagedby speech. associated withhyponasal Velopharyngeal incompetenceis with airflow. Speech problemsarenotassociated true? true? are true? incleftpatients regarding dentalcare y ➜ ➜ ➜ 16. Which of the following statementsare Whichofthefollowing 16. statementsare Whichofthefollowing 15. 17. Which of the following statements Whichofthefollowing 17.

audit Role ofnasalsurger Surgical repair patients Tooth structure incleftlip B A E D C B A E D C B A E D C true? regarding toothstructurearetrue? true? correct poormid-facegrowth. isdesignedto surgery Orthognathic in thelongterm. Failure ofDresultsinboneabsorption the alveolarbonegraft. It isusefultoensureatootheruptsinto the humerusandfemur. Alveolar bonegraftsareobtainedfrom liprevision. simultaneous secondary Alveolar bonegraftscannotbeusedwith patients. anodontia isnotfoundincleftlip Partial patients. essential intheoverallcareofcleft speech recordingsandaudits,are Meticulous record-keeping,including improved byapostauricularonlaygraft. Tip projectionofthenosecannotbe onthecleftside. cartilage lateral there iscollapseofthelower Open rhinoplastyisalsoindicatedwhen septum intothecleftnostril. there isdislocationofcartilaginous Open rhinoplastyisindicatedwhen surgery. be doneafterorthognathic Open rhinoplastyisnotaprocedureto designated centresmustbeinvolved. when acraniofacialteamworkingin andCrouzon’ssyndromes Apert’s Major osteotomiesarerequiredin commence untiltheageof6years. doesnot surgery Orthognathic Mandibular advancementmayalsohelp. face problem. method ofchoicetocorrectthemid- Elective setbackofthemaxillais y and ➜ ➜ ➜ ➜ 2.A,B,D 1. B,D Answers: Multiplechoicequestions statementsare Whichofthefollowing 19. statements Whichofthefollowing 18.

Orofacial congenitalabnormalities Cleft palate–general problems Mandible andmaxilla Tooth eruption causation. A family history ofacleftinfirst-degreerelative increasestheriskto1in25live births. Afamilyhistory causation. in Bothgenetic andenvironmentalfactorsplaya part the NativeAmerican tribesofMontana,USA. not haveanincreasedcleftincidence, butitisincreasedinethnicChineseandhighestamong at least300recognisedsyndromes that haveassociatedcleftproblems.Theblackpopulationdoes The commonestorofacialcongenital abnormalitiesarecleftlips,alveoliandpalates.There percent.Thelatterismorecommoninfemales. 40 1000 Cleftlipalonecomprises15 livebirths. percentofallcleftswhilecleftpalatealonecomprises andthat ofisolatedcleftpalateis1in livebirths The incidenceofcleftlipandpalateis1in600 A E D C B A E D C B between themaxillaandmandible. ingrowth when thereisdisproportion Dental occlusionproblemscanarise teethisinthemandible. supernumerary The mostcommonsitefor teeth. supernumerary causedby including overcrowding involves removalofanyobstruction, Management ofuneruptedteeth possible. anodontiaisnot Management ofpartial cleidocranial dysostosis. Eruption ofteethisnotaproblemin dentigerous cyst. Eruption ofteethmaybeimpairedbya structure ofteeth. Tetracycline cancausedefectsinthe structure ofteeth Measles doesnotcausedefectsinthe structure andattritionofteeth. Genetic disorderscancausechangesin dentition. encourages eruptionofsecondary teeth Removal ofsupernumerary true? regarding tootheruptionaretrue? ➜ 20. Which of the following statementsare Whichofthefollowing 20.

plans investigation andtreatment Deformities ofthejaw– E D C B A E D C B true? feature ofTreacher–Collins syndrome. isnota growth Facial disproportionate andhorizontalplanes. the vertical ofthejawinboth asymmetrical growth Condylar hyperplasiacauses years ofage. condition occurringbetween35and45 Condylar hyperplasiaisanidiopathic the mandible. of associated withoverdevelopment Class 11 occlusiondeformityis in theaboveplanning. Cephalometric studiesareoflittlevalue 17–18 years. treatmentattheageof orthodontic Treatment planningusuallybeginswith surgery. in orthognathic and maxillofacialsurgeonsisimportant A combinationbetweenorthodontic the jaw. to surgicalcorrectionofdeformities isthetermgiven surgery Orthognathic condylar hyperplasia. A bonescanisausefulinvestigationin 305

42: CLEFT LIP AND PALATE HEAD AND NECK ➜ ➜ ➜ ➜ ➜ ➜ ➜ 9. B 8.C,E 7. B,D,E 6.A,B,D 5.A,B,E 4.A,B,E 3.C,E 306

Repair ofcleftlipandpalate Pierre Robins considerations Use ofscansincleftsandrespirator Further anatomicalconsideration Muscle andstructuralconsiderations Pierre Robins Causes ands The environment is of greater importance incleftpalate thanincleftlipandpalate.Environmental The environmentisofgreaterimportance Cleft liprepairisdone attheageof3–6months,while cleftpalatesarerepairedat6months. by restoringnormalanatomy. toencouragenormalfacialgrowth incleftsurgery It isimportant which isthetensorpalatimuscle. the crucialthingincleftrepairs.There are5musclesthatcontrolactivityinthesoftpalate,oneof Inthatsyndrome,hypoxiaoccurswhileasleep,notawake.Repairofmuscleis labioglossopexy. complicationsin Pierre Robincasesis respiratory A techniquethatmaybeusefulinpreventing problems occurinthePierreRobinsyndrome. not before.Feeding andthrivingincleftlippalate babiesareusuallynormal.Majorrespiratory been made.Ultrasonicscansanddiagnosisofcleftlipscanbemadeafter 18 weeks’gestationbut palates. Appropriatecounsellingshouldbegiventoprospectiveparents whenthediagnosishas areofnovaluein thediagnosisofcleft Antenatal scansareofuseindiagnosingcleftlipsbutthey ontothehardpalate. partly cleft softpalateextending system—lahShisanincompleterightunilateralcleftlipandalveoluswithacomplete LAHSHAL occurs asaresultofmuscleabnormality. Classificationofcleftscanbesimplifiedusingthe isintactandagroove cleft ofthesoftpalate.Insubmucouscleftspalate,mucosa processes. Theattachmentofthemusclefibresisintoposterioredgehardpalateina septumandthevomerareseparatedfrompalatine In completecleftpalate,thenasal is theresultoffailureinfusion2palatineshelves. palateisfoundposteriortotheincisorforamenandacleft bilateral cleftlips.Thesecondary and symmetricalinbilateralclefts.Theprolabiumhasnomuscletissueisassociatedwith Cleft lipiscausedbydisruptionofthenasolabialandbilabialmuscleswiththismoreprofound commonest syndromeassociatedwithclefts. andfeedingproblems. Itisthe and retrognathiainthissyndromecontributetoearlyrespiratory whodescribedthesyndromein1929.Pierre Robinwasaprofessorofdentistry, Glossoptosis especially anisolatedcleftpalate. andTreacher–Collins Apert’s, Down’s, and phenytoin. syndromescanbeassociatedwithclefts, factors implicatedincleftsincludematernalepilepsyanddrugs,includingsteroids,diazepam y y y ndromes ndrome ndrome problems andmusclestructure inclefts y andnutritional ➜ ➜ ➜ ➜ ➜ ➜ 15. A,E 14. A,D, E 13. A,E 12. D 11. A,B,E 10. A,D,E

Surgical repair Bone grafts Revisional surger Dental care Speech problems Team approach incleftsandcare oftheear years. techniques availableforcleftrepairsandtheMillardmethodshavebecomepopularinrecent palatal cleftsissensibleaswellbeinglessdestructivethanasingle-stageone.Thereareseveral at15–18the repairofhardpalatewithorwithoutliprevision months.A2-stagerepairof When therearecombinedcleftlipandpalates,usually2operations,the2ndbeing craniofacial surgeons indesignatedcentres. 11 orCrouzon’ssyndromesareoperated onbyspecialised years.MajorosteotomiesinApert’s attheageof5yearsbut isbestdonebeforethecaninetootherupts–between8and performed isnot Thistypeofsurgery whenmid-face retrusionexists. back whilethemaxillaissetforward problems.Themandibleisset isusedto correctpoormid-facegrowth surgery Orthognathic absorption inthelongterm. is usefultogetanyteetheruptintothegraftandfailureforthishappen willresultinbone Boneforgraftingisobtainedfromtheiliac crest ortibia.It liprevision. timeassecondary same atthe Alveolar bonegraftcanreceiveanosseointegrateddentalimplantand can beperformed fistula oftheanteriorpalate. and canbeusefulinclosinga done asarule,thoughnotalways,afterperiodoforthodontics cosmeticfeatureoftheupperliprepair–andnormallip. Alveolarbonegraftsare important isan are theresultofincompletereconstructionnasolabialmusclering.TheCupid’sbow cleftdeformities repairedcleftlipsisconsideredafter2years.Nasal ofpreviously Any revision tocorrectamalpositionedorretrusivemaxillabyosteotomyisperformed. when surgery at 8–10 and14–18 ofthemaxillaisdoneearlierthan14–18 yearsofage.Expansion years;thisis and abnormaldentition,includingeruptionproblemsornumbersofteeth.Itisdone disease careisusedtoprevent Problems withteetharecommonincleftpatients–orthodontic training devices. therapists,andspeech speech.Themanagementofspeechproblemsinvolvessurgery, hypernasal are assessedattheageof18They months.Velopharyngeal incompetenceisassociatedwith problems. Speech problemsarecommonlyfoundincleftpatientsandassociatedwithairflow hearing problems. testsshouldbealwaysdonethroughoutchildhoodtocheckon potential effusion. Audiology ofgrommetsat6–12Elective myringotomyandinsertion monthscaneliminatemiddleear including auditofresults.OtitismediaisassociatedwithcleftpalateandEustachiantubeproblems. teamapproachisessentialinthemanagementofcleftsaslongtermreview A multidisciplinary y 307

42: CLEFT LIP AND PALATE HEAD AND NECK ➜ ➜ ➜ ➜ ➜ 20. A,B, C 19. A,D 18. A,D 17. B,C,E 16. C,E 308

Deformities ofthejaw–investigationandtreatment plans Mandible andmaxillaproblems Tooth eruption Tooth structure incleftlippatients Role ofnasalsurger Open rhinoplasty is usually performed after hascorrectedfacialstructureand surgery afterorthognathic Open rhinoplastyisusuallyperformed growth. of 17–18 inabnormalitiesoffacial years.Cephalometricinvestigations arehelpfulandimportant careshouldbedoneearlierthantheage surgery. Orthodontic inorthognathic isimportant surgery andmaxillofacial andthecombinationof orthodontics surgery of jawdeformitiesisorthognatics incasesofcondylarhyperplasia.Thecorrection A bonescanisausefulmethodofexamination horizontal planes.Treacher–Collins disparity. syndromeisassociatedwithfacialgrowth occurs inthe15–30 and ofthejawinbothvertical yearagegroupandcausesabnormalgrowth teeth.Condylar hyperplasia occlusion, themandibularteethareplacedposteriortomaxillary problemsforthemandibleandmaxilla.Inclass11Dental occlusiondisparitycausesgrowth dental teethisinthemaxillaandnotmandible. site ofsupernumerary helpful.Themostcommon teethisvery by removalofanyobstructionincludingsupernumerary anodontiaispossibleandmanagementofuneruptedteeth dysostosis. Themanagementofpartial cancausenon-eruptionofteethandthisisalsoafeaturecleidocranial Dentigerous cysts canalsocausedisordersofteeth. teeth. Diseases,suchasmeasles,anddrugs,tetracycline, dentition.Geneticdisorderscancausechangesinthestructureandattritionof eruption ofsecondary teethwillencouragethe anodontiacanbefoundinclefts,andremovalofsupernumerary Partial essential intheoverallmanagementofcleftpatients. or apreauricularapproach.Meticulousrecord-keepingandauditanalysisovermanyyearsare onlaygraftmaterial,whichcanbeobtainedfromtheear–byeitherapostauricular using cartilage onthecleftside.Tipprojectionisdoneby lateralcartilage and/or thereiscollapseofthelower deformities. Itisdonewhentheredislocationofthecentralseptumintonon-cleftnostril y andaudit 43 The nose and sinuses

Multiple choice questions In the following, choose the single best answer. ➜ Trauma to the nose and paranasal sinuses ➜ Anatomy of the nose and 5. Injury to which blood vessel is paranasal sinuses associated with severe post-traumatic 1. Which of the following sinuses do epistaxis? not open on to the lateral wall of the A External carotid artery nose? B Facial artery A Maxillary sinus C Sphenopalatine artery B Ethmoid sinus D Anterior ethmoid artery C Frontal sinus E Nasal artery. D Sphenoid sinus E Septal sinus. 6. How can clear nasal discharge be confirmed as cerebrospinal fluid 2. Via which route does the greater (CSF) rather than nasal mucus? palatine artery supply the nasal A High-resolution CT scanning septum? B Glucose stick testing A Foramen rotundum C T2-weighted MRI scanning B Vidian canal D Endoscopic examination C Incisive canal E Increased production with dependent D Sphenopalatine foramen head position. E Cribriform plate. 7. How should suspected nasal bone 3. Intracranial drainage of venous blood fracture be managed? from the nose and sinuses passes to A Facial X-rays the cavernous sinus via which route? B Immediate manipulation A Ophthalmic vein C Review in clinic at 4–5 days B Pharyngeal plexus D Arrangements for rhinoplasty C Pterygoid plexus E Examination under anaesthetic with or D Facial vein without manipulation. E Supratrochlear vein. 8. Which of the following is not ➜ Imaging of the paranasal a complication of nasal septal sinuses haematoma? A Meningitis 4. Which view(s) are necessary on B Septal abscess computed tomography (CT) for C Cosmetic deformity detailed assessment of the sinuses? D Cavernous sinus thrombosis A Coronal E Ischaemia of the soft palate. B Axial C Coronal and axial D Coronal and sagittal E Coronal, axial and sagittal.

309 HEAD AND NECK ➜ ➜ 12. Which of the following regarding Whichofthefollowing 12. procedures Whichofthefollowing 9. 4 Whatgender/agegroup doesjuvenile 14. isnotacause Whichofthefollowing 13. arenot Whichofthefollowing 11. isnotincluded Whichofthefollowing 10. 310

Epistaxis The nasalseptum E D C B A E D C B A E D C B A E D C B A E D C B A E D C B A Children aged10–14. Girls aged14–18 Boys aged14–18 Girls aged10–14 Boys aged10–14 angiofibroma tendtoaffect? Old age. Granulomatous disorders Infections Foreign bodies Trauma of epistaxis? septalcartilage. It tendstosparenasal It istreatedsurgically It canaffectthenose,lungsandkidneys scanning It isbestdiagnosedbyCT It affectsthenosealone Cosmetic deformity. polyps Nasal obstruction Nasal Whistling Epistaxis perforation? ofnasalseptal complications Toxic irritation. Malignancy Vasculitis Infection perforation? ofnasalseptal in theaetiology Septodermoplasty. Submucous diathermy Turbinectomy Septoplasty Submucous resection Wegner’s granulomatosis aretrue? ofcartilage? septum withpreservation involve correctionofadeviatednasal ➜ statements Whichofthefollowing 15. 18. Where do the majority ofnasal Wheredothemajority 18. 7 Whatshouldbeincludedin 17. Whatmaybeinvolvedinthe 16. 20. Which of the following arenot Whichofthefollowing 20. isassociated Whichofthefollowing 19.

Nasal pol B A E D C B A E D C B A E D C B A B A E D C B A E D C about juvenileangiofibromaaretrue? polyps arisefrom? Ligation ofthelingualartery Ligation ofthefacialartery epistaxis? management oflife-threatening It maybecuredbyembolisation. It isdiagnosedwithimaging. malformations ofthemeninges. It isassociatedwithvascular It shouldinitiallybetreatedmedically. It requiresbiopsyfordiagnosis. Nasal obstruction Nasal Epistaxis polyps? commonly associatedwithnasal thyroidcancer.Medullary polyps Gastric allergy Aspirin Chronic bronchitis Penicillin allergy with nasalpolyps? septum. Nasal Sphenoid sinuses Frontal sinuses Ethmoid sinuses sinuses Maxillary Platelet transfusion. Ligation oftheanteriorethmoidalartery Oestrogen creams cautery Nasal packing Nasal related epistaxis? haemorrhagic telangiectasia(HHT)- the managementofhereditary Ligation oftheinternalcarotidartery. theoriginoflingualartery below carotidartery Ligation oftheexternal above theoriginoflingualartery carotidartery Ligation oftheexternal y ps ➜ 23. What isthemostlikelyorganismin 23. 25. In acutesinusitis,pus maybeseen 25. Where isatrocharusuallyinsertedin 24. statements Which ofthefollowing 22. shouldbe Whichofthefollowing 21.

Sinusitis E D C B A E D C B A E D C B A E D C B A E D C B A E D C Nasolacrimal duct. Supreme meatus Superior meatus Middle meatus Inferior meatus draining fromwhich ofthefollowing? ostium. sinus Through thenaturalmaxillary Percutaneously Through themiddlemeatus Through theinferiormeatus Through thecaninefossa antral lavage? Staphylococcus epidermidis. Chlamydia trachomatis Streptococcus pneumoniae Pseudomonas aeruginosa Staphylococcus aureus margins. withclear excision incision toallow throughanexternal It isperformed of thesinuses. scanning It isrecommendedpriortoCT steroid treatments. It isrecommendedafterfailureof It isrecommendedasalong-termcure. surgery. It isrecommendedinallpatientsfitfor true? fornasalpolypsare regarding surgery Cocaine abuse. Fungal infection Allergy Malignancy Cystic fibrosis suspected inunilateral nasalpolyps? Sinus infection. Rhinorrhoea Ansomia maxillary sinusitis? maxillary ➜ isnota Which ofthefollowing 26. 27. Which of the following statements Whichofthefollowing 27. 28. Which of the following statements Which ofthefollowing 28. 29. Which of the following statements Which ofthefollowing 29.

sinuses Tumours ofthenoseand C B A E D C B A E D C B A E D C B A E D sinusitis? ofacutefrontoethmoidal complication about nasaltumoursistrue? Gingivitis. Intracerebral abscess abscess Extradural Meningitis Periorbital sinusitis disease. It tendstopresentintheearlystages of rhinorrhoea. It isassociatedwithwatery scanning. It canbediagnosedonCT septum. It tendstoarisefromthenasal wood dust tohard It isassociatedwithexposure aretrue? nasal cavity ofthe regarding adenocarcinoma donoterodebone. They polypectomy. routinenasal following aresometimesdiagnosed They polyps. aretreatedsurgicallyasforsimple They malignancy. have noassociationwith They cavity. only affectthenasal They are true? regarding transitional cellpapillomas obstruction. cause nasal sufficientlyto tumours rarelygrow Nasal bilateral symptoms. tumours tendtopresentwith Nasal symptoms. tumours giventhenon-specific nasal Missed diagnosisiscommonin tumours. diagnosis innasal forearly Early cosmeticdeformityallows diagnosis. tumoursfacilitatesearly from nasal The uncommonnatureofsymptoms 311

43: THE NOSE AND SINUSES HEAD AND NECK ➜ ➜ Extended matchingquestions Extended statementsis Whichofthefollowing 30. Choose the most appropriate action for each of the clinical scenariosbelow: Choosethemostappropriateactionforeachofclinical 312

J I H G F E D C B A 2. Anatom 5 4 3 2 1 J I H G F E D C B A 1. Imagingofthenoseandparanasalsinuses C B A Sphenoid sinus. Ethmoid sinus sinus Maxillary Nasolacrimal duct Superior meatus Middle meatus Inferior meatus Sphenopalatine artery Posterior ethmoidalartery Anterior ethmoidalartery mucosa. andgranulations onthenasal aseptalperforation reveals ofbreath.Examination and shortness obstruction,haematuria discharge,nasal femalepresentswithbloodstainednasal A 46-year-old sinusonX-ray,unresponsivetoantibiotics. systemic upsetwithopacificationofthemaxillary A 25-year-oldmalepresentswitha10-day ofunilateralfacialpain,rhinorrhoeaand history epistaxis. A 16-year-old obstructionandintermittentunilateral malepresentswithunilateralnasal to considersurgicalmanagement. polypshasfailedmaximalmedicaltherapyandwishes A 35-year-oldfemalewithsimplenasal fractureissuspected. clinical assessment,anisolatednasal An 18-year-old Following allegedassault. following department maleattendstheemergency in4–7days. Outpatient review fibrosis Sweat testingforcystic Plain sinusX-rays Biopsy underendoscopiccontrolinclinic Antral lavage andANCA biopsyandbloodtestforESR Mucosal scanofsinuses Contrast-enhanced CT scanofsinuses Non-contrast-enhanced CT Facial X-rays mucus Culture andsensitivityofnasal treated with surgery alone. treated withsurgery Malignant tumoursofthesinusesare require detailedcross-sectionalimaging. Malignant tumoursofthesinuses approach. require amultidisciplinary Malignant tumoursofthesinuses false? y ofthenoseandparanasalsinuses E D present withlocallyinvasivedisease. Malignant tumoursofthesinusesmay of bleeding. should notbebiopsiedduetotherisk Malignant tumoursofthesinuses ➜ ➜ ➜ 7. 7. C 6. B 5. D 4. E 3. 2. A 1. C D Answers: Multiplechoicequestions Choosethecorrectresponseforeachquestionbelow:

Trauma tothenoseandparanasalsinuses Imaging oftheparanasalsinuses Anatom 5 4 3 2 1 arrangements should be made for review after4–7dayswhenswelling hassubsided.X-ray arrangements should bemadeforreview forseptalhaematoma, whichrequiresurgenttreatment, for associatedinjuries, andparticularly afullassessment clinician toseepatientsintimetreat theseinjuriesatthefirstvisit.Following unusualforthe accurateassessment ofthebonyskeleton.Itisvery tissue swellingprevents fracturescan beassessedandmanagedimmediately,withinthefirsthour,soft- Although nasal retraction ofthebrain,butmaynotbesuitableinallcases. scarandprevent an external Transnasalmethod ofclosureisdependentonsite,sizeandsurgicalexpertise. approachesavoid identificationofthesiteleak, orintrathecalfluorescein.Following T2-weightedMRI CT, first instance.Thosethatfailtoceasespontaneouslycanbeinvestigated usinghigh-resolution the bedside. andtheaqueoushumour. at in CSF Thisrequiresspecialisedanalysis andcannotbeperformed tested forusingglucosesticks.Amoreaccuratemeasureisbeta-2-transferrin, whichispresentonly leak, whichcanbe headtraumashouldraisethesuspicionofCSF Clear rhinorrhoeafollowing midline ofthenose. forcontrol,throughanincisionplacedatthemidpointbetweenmedial canthusandthe allows approachtothisartery oedema subsides,leadingtoadelayedtorrentialhaemorrhage.Anexternal the lacrimal,ethmoidandfrontalbones.Soft-tissueswellingmaysplintvesselcloseduntil canbeinjuredintraumaandsplintedopenbybonyfragmentsof The anteriorethmoidartery inthisarea. surgery assessmentofthefrontalrecessandshouldbeinspectedpriorto allow views Sagittal nerve. assessmentoftheposteriorethmoidcellsandtheirrelationshiptooptic allow views Axial oftheuncinateandmedialorbitalwall(laminapapyracea). the cribriformplate,insertion accurateassessment of allow andmayrequiredigitalreconstruction.They are themostimportant arerequiredtoassessthesinusespriorsurgery.Both coronalandaxialviews Thecoronalviews Where dotheopeningsofposteriorethmoidsinuseslie? What structureliesintheinferiormeatus From whichsinusdoesperiorbitalcellulitisusuallyspread? sinusostiumlie? Where doesthemaxillary Which vesselismostlikelytocausesignificantpost-traumaticepistaxis? Depending on the nature of the trauma, CSF leaksmaybemanagedspontaneously inthe Depending onthenatureoftrauma,CSF y ofthenoseandparanasalsinuses 313

43: THE NOSE AND SINUSES HEAD AND NECK 8. E ➜ 11. 11. D 10. A 9. B 314

The nasalseptum packingasrequired. with nasal involves aspirationunderlocalanaesthetic,orincisionanddrainagegeneral injury. ofsuspicionseptalhaematomafollowing Treatmenthigh index ofseptalhaematoma sepsis, whichinturnmaybelife-threatening.Itisforthisreasonthattheclinicianmusthavea Sepsisfromthisareadrainstothecavernoussinusandcanresultinintracerebral pain andpyrexia. skeletonwithbothcosmeticandfunctionaleffects. canleadtocollapseofthenasal support Thislossof undergoesnecrosis,leadingtoseptalperforation. without thesenutrients,thecartilage surround theseptumonbothsides.Thederivesitsbloodsupplyfromtheseflaps,and septumandbloodcollectsbetween themucoperichondrialflapswhich from fractureofthenasal trauma,includingsurgery.Septal haematomaisgenerallyacomplicationofnasal Bleedingresults for formalrhinoplasty. withtheresults,andthosewhoarenotcanbeconsideredatthatpoint of patientswillbesatisfied torhinoplasty. forfracturereductionwithouttheneedtoresort 2–3 weeksallows Eightypercent identified, simplemanualmanipulationunderlocalorgeneralanaestheticarrangedinthefirst today. areuncommonlyperformed false-negatives arecommonduetovisiblebonysuturelines,sothey and assessment waspopularinthepast,butclinicalissufficienttodiagnoseinjury and canimprove crusting andthefeelingofobstruction. Patientswhocannottoleratesuch canbefittedwithasilasticobturatorwhichoccludestheperforation paraffin. Largerperforations no treatmentatall.Crustingandbleeding canbetreatedwithbarriercreamssuchassoftliquid of thenose.Thiscanleadtocosmetic andfunctionalproblems. nosedeformitywithcollapseofthemiddlethird dorsum,leadingto asaddle ofthenasal support cancausea whistling asairpassesby. canreducethe anterior perforations Largeperforations oftenbleeds repeatedly. rawmucosa crustsbreakoff,theexposed crusting iscommon.As Small obstruction and afeelingofnasal arecommonlyasymptomatic;however, Septal perforations forhistology. of theperforation aswellbiopsy andsyphilis serology, antibodies(ANCA) bloods forantineutrophilcytoplasmic conditions suchasWegener’s granulomatosisandmalignancy. andsyphilisinfectionshouldbeconsideredaswell granulomatous and cocaineabuse.TB nosepicking Commoncausesincludetrauma, fromsurgery, not arecognisedcauseofperforation. is however, Allergy, canbeappliedtoseptalperforation. withallconditions,thesurgicalsieve As andreplacedwithsplit-thicknessskingrafts. septumisexcised ofthenasal The mucosa patients. forpersistentepistaxisinHHT volume. Septodermoplastyisaprocedurerarelyperformed tissueoftheinferiorturbinatetoencouragescarringandareductionin the submucosal forenlargedturbinates,andsubmucousdiathermyinvolvesmonopolarto performed Turbinectomy cartilage. resection ofthedeviated involvesresectionoftheinferiorturbinate, Submucousresectionwasaprocedurewhichpredatedseptoplastyandinvolved cartilage. preserve septumwhilstaimingto nasal Septoplasty involvesstraighteningofthebonyandcartilaginous If a nasal injury is seen within a week and a fracture associated with a change in nasal shapeis isseenwithinaweekandfractureassociatedwithchangeinnasal injury If anasal The haematomaoftenbecomesinfectedtoformaseptalabscess,whichwillresultinincreasing Treatment willbetailoredtothecomplaintsandmanypatientsrequire forseptalperforation butmay includechestX-ray, willbeguidedbyhistory, Investigation ofseptalperforations ➜ 17. C 16. 15. C D 14. C 13. E 12. C

Epistaxis described, whichisanindicationthatnonehasproventobeuniformlysuccessful. Manyprocedureshavebeen treatment canbeconsideredforsurgicalclosureoftheperforation. needs toberepeated asthetelangiectasiareturnswith time.Septodermoplastyinvolves replacing protection forthefragilevessels.Laser treatmenttobleedingpointscanbesuccessful,althoughit the pseudostratifiedcolumnarepithelium toastratifiedsquamoustype,whichcanprovidemore removal ofpackscanleadtotraumatic haemorrhage.Oestrogencreamsresultinmetaplasiaof termbut packsmaybeusedtocontrolbleedingintheshort Nasal andseptalperforation. cautery term,butthe long-termnatureoftheconditionresultsinmultipleattemptsat effective intheshort tothebleedingvesselmaybe isadifficult conditiontotreat.Cautery Epistaxis relatedtoHHT ligated isnottheinternalcarotid,whichhasnobranchesinneck. carotid alsoensuresthatthevessel the ipsilateraltongue.Identificationofbranchesexternal canjeopardise thebloodsupplyto thislevel andligationbelow isanendartery, The lingualartery perioperative bloodloss,notasacurativetreatment. supply tothetumour. ofthesetumours has beenusedtominimise Preoperativeembolisation resuscitation. inatheatresettingwiththeappropriateequipmentforarrestofhaemorrhageand performed indicated asitcanbeassociatedwithsignificanthaemorrhage.Ifrequired,biopsyshould andimaging.Biopsyisnotroutinely and intermittentepistaxis.Stagingreliesonexamination obstruction angiofibromatendstoaffectadolescentboysandpresentswithnasal Juvenile nasal artery. ofthelingual carotidabovethelevel ligation; iflife-threatening,considerligationoftheexternal artery sphenopalatine andanteriorethmoidalligationand,ifunsuccessful,internalmaxillary setting.Surgicaltreatmentincludes These patientsmaybebestmanagedinahigh-dependency packs,whichcompromisetheairway.respond tosuchtreatmentcanbeconsideredforpostnasal packing.Thosewhodonot ordirectednasal Bleeding pointscanbetreatedwithlocalcautery the haemodynamicstressthatresults. tocopewith physiologicalreserve result inheavierbleeding.Suchpatientsoftenhavealower andanticoagulationareassociatedwithepistaxis idiopathic. Factors suchashypertension bodies andgranulomatousconditionscancausebleeding,itismorecommonthatthe Specific causesofepistaxisareoftenimpossibletoidentify. Althoughtrauma,infection,foreign unless thediseasehasbeeninlong-termremission. shouldnotbeconsidered septalperforation surgeon.Surgicalcorrectionofanasal than theENT complications.Treatmentand further ismedicalandoftencoordinatedbythephysicianrather obstruction.Granulomamayleadtoseptalperforation dischargeandnasal bloodstained nasal manifestationsinclude granulomata.Nasal byrenalbiopsyorofactivenasal achieved whichcanbe Diagnosisrequireshistology, commonly involvesthenose,lungsandkidneys. Wegener’s granulomatosisresultsinavasculitiswhichcanaffectanyorganthebody,but Treatment ofepistaxisshouldincludeidentificationandtreatmentanyunderlyingcauses. Management involves surgical excision following detailedimagingtodelineate thevascular following Management involvessurgicalexcision 315

43: THE NOSE AND SINUSES HEAD AND NECK ➜ ➜ 24. 23. B C 22. C 21. B 20. A 19. C 18. B 316

Sinusitis Nasal pol forresistantcases. andisreserved from turbulentairflow time. Young’s procedureinvolvessurgicallyclosingthenostril.Thisreducestraumatoseptum withsplit-skingrafts,althoughtheconditioncanalsoaffectskingraft mucosa the nasal through the middle meatus, function of the mucociliary transport mechanismisaided.This transport through themiddle meatus,functionofthemucociliary sinuses. Byopeningthedrainagepathways ofthesinuses natural functionof theparanasal the andpreserve which aimtosparemucosa modern techniquesoffunctionalsinus surgery, used regularlyinthepastfortreatment ofchronicsinusitisbuthasbeensupersededby isusedtowashoutthesinus andprovidepusforculture.Thisprocedurewas position. Saline direction oftheipsilateraltragus.The trocharcanthenbeusedtoaspirateairconfirmits under theinferiorturbinateandpassed throughthethinboneininferiormeatus sinus.Atrocharisinserted Antral lavageisusedinacutesinusitis towashpusfromthemaxillary required formalignantdiseaseofthenoseandsinuses. are incisions.Moreradical formsofsurgery Standard polypectomydoesnotinvolveexternal steroidspraysinanattempttoreduceordelaysymptom recurrence. asked tocontinueonnasal patientswillbe surgery, disease, improvepatients’symptomsandavoidcomplications.Following cavity.headlight toilluminatethenasal Thegoalofsurgicaltreatment istoreducethebulkof polypsiscommon,althoughsome practitionersusea An endoscopicapproachtosimplenasal leak. whichincludedamagetotheorbitalstructuresandskullbasewithCSF risks ofsurgery, treatment. steroids. Patientswhofailacourseofmaximalmedicaltherapyshouldbeconsideredforsurgical coursesoforal steroidsandshort causecanoftenbecontrolledwithtopicalnasal symptoms they Treatment polypsismedicalinthefirstinstance.Polyps cannotbecured,butthe ofnasal biopsy. and dysfunction.IdentificationofunilateralpolyposisrequiresimagingwithCT or cranialnerve Truly especiallyifassociatedwithbleeding,pain unilateralpolyposisissuggestiveofmalignancy, bilateraldiseasethatishiddenonanteriorrhinoscopy. ofthenosewhichmayreveal examination polypstendtobebilateral.Unilateralareanindicationforendoscopic Simple nasal with topicalsteroidsprays,antihistaminesandanticholinergics. cleft.Rhinorrhoeaistreated accesstotheolfactory successful inreducingpolypbulkandallowing Anosmia isoftenupsettingforpatientsanddifficulttotreat.Acourseoforalsteroidsmaybe (anosmia). Treatment obstruction. usefulformanagingnasal steroidsisparticularly withnasal Obstruction, rhinorrhoeaandsinusinfectionarecommonproblems,asislossofsensesmell epistaxis,itisnotacommoncomplaint. polypscandevelop Although patientswithnasal polyps. asthmaandnasal triaddescribesanassociationbetweenaspirinallergy, Sampter’s ofallsinuses. and mayinvolvethemucosa fillthemiddlemeatus they grow they polypsarisefromtheethmoidsinuses.As Most simplenasal Current surgical standards require preoperative CT scanninginanattempttominimisethe Current surgicalstandardsrequirepreoperativeCT y ps 26. E 25. B ➜ 29. A 28. D 27. C

Tumours ofthenoseandsinuses should beidentifiedandmanagedwithcooperationfromaneurosurgicalteam. abscess,subduralintracerebralmeningitisorvenoussinusthrombosis extradural drainage ofanyabscessshouldbeapriority. ofintracranialsepsisintheform Thedevelopment blindness, canoccur. For thisreasonearlyidentificationandtreatmentwithantibiotics damageand withirreversible the pressureinorbitincreases,ischaemiaofopticnerve, suggestingabscessformation.As progresses, proptosisandlimitationofgazemaydevelop, complications. intracranial have contrast-enhancedimagingofthesinuses,orbitandcranialcavitytoexclude of synchronouscomplications.Patientswithperiorbitalcellulitiswhorequireimagingshould thecliniciantopossibility is alsopossible.Thepresenceofonecomplicationshouldalert walls ofthesinustoorbitorcranialcavity. Haematogenousspreadfrominfectedsinuses Acute sinusitiscanspreaddirectlybyinfectedthrombophlebitisorthroughdehiscencesinthe meatus. All sinusesotherthantheposteriorethmoidsandsphenoidsinusdrainintomiddle sinuses. accessfortopicalmedicationtothe andallows promotes thenaturalfunctionofsinusmucosa Nasal obstruction and epistaxis are presenting features. It tends to arise from the nasal sinusesand obstructionandepistaxis arepresentingfeatures.Ittends toarisefromthenasal Nasal Adenocarcinoma hasanassociation withthehardwooddustinvolvedinfurnituremanufacture. of anysuspicioustissue. ofthepostoperative cavityisrequiredwithbiopsies withendoscopicexamination term follow-up Long- andmorecompleteexcision. orviaanopenapproachtoaffordwiderexposure endonasally separately.lesion develops 10 whetherthebenign lesionbecomesmalignantorasecond percent,althoughitisuncertain ofbetween5and Thereisaquotedassociationwithmalignancy excision. to recurlocallyfollowing Thediseaseisbenignbuttends fissureintotheinfratemporalfossa. through thepterygomaxillary out andgrows lateral wallofthenosetoinvolvemiddlemeatusandosteomeatal complex, Transitional fromthe ofthenoseandsinuses. Ittendstogrow cellpapillomainvolvesthemucosa repeated topicalchemotherapeuticapplications. radiotherapy. Someunitshavepublishedgoodresultsusinga debulkingsurgicalapproachand tendstobesurgicalwithpostoperative malignancy dural involvement.Managementofsinonasal orbitaland estimationofbonydestruction,aswellcranialnerve, the bestresultsallowing can bebilateralinadvanceddisease. orbitandcribriformplate.Symptoms tend beunilateralinthefirstinstancebut infraorbital nerve), (e.g.the of thebonyconfines,itinvolvessurroundingstructures,includingcranialnerves out inthesinusandextends thetumourgrows tumours areoftenmissedandpresentlate.As rarelyassociatedwithmalignantdisease.For thisreason,nasal in thegeneralpopulationandvery obstructionandepistaxis.Thesesymptomsarecommon tumours tendtopresentwithnasal Nasal Orbital cellulitis presents with chemosis and erythema of the preseptal tissues. As theinfection ofthepreseptaltissues.As Orbital cellulitispresentswithchemosisanderythema Staging with CT +/– MRI is required to plan treatment, which can be performed either isrequiredtoplantreatment,whichcan beperformed +/–MRI Staging withCT give andMRI management.CT iscrucialinmultidisciplinary malignancy Staging ofsinonasal 317

43: THE NOSE AND SINUSES HEAD AND NECK 30. C ➜ ➜ 1A,2E,3I,4G,5F 5E 4F 3D 2C 1J matchingquestions Answers: Extended 318

2. Anatom 1. Imagingofthenoseandparanasalsinuses the disease,andtreatmenttendstobesurgical. is diagnosedonbiopsy. instaging withothersinusdisease,cross-sectionalimagingisimportant As of thedisease.Managementisunlikelytoinvolvesurgery. These patientsshouldbemanagedinconjunctionwithaphysiciandue tothemultisystemnature mayalsobeusefultodiagnosethecondition. andANCA by biopsyofanactivegranulation;ESR The suspecteddiagnosishereisofWegener’s granulomatosis.Theconditionmaybeconfirmed obtained fromantrallavage. opacificationandpusmaybe ostium. Ifthisisunsuccessful,X-rayofthesinuseswillshow drainageofsinuscontentthroughthenatural of themiddlemeatusmaybesufficienttoallow withdecongestion sinusitis.Endoscopicexamination isoneofunilateralmaxillary The history loss. supply. toreduceperioperativeblood Imagingcanbecombinedwithpreoperativeembolisation forassessmentofthetumour,itsbonyconfinesandvascular juvenile angiofibromaallows cavity. amasslesionintheipsilateralnasal reveals Contrast-enhancedimagingof Examination scan. withoutpreoperativeCT shouldnotbeperformed a ‘roadmap’forsurgery. Sinussurgery assessmentofthebonyanatomysinusesandareusedas scanswithoutcontrastallow CT 2–3 weekspriortofracturehealing. oedemahassettledandleavestimeformanipulationwithin assessment oncepost-injury for inaround 1weekallows trauma.Review not requiredandaddlittletoassessmentofnasal are fractures,they boneshavebeenusedinassessmentofnasal Although X-raysofthenasal y ofthenoseandparanasalsinuses 44 The ear

Multiple choice questions In the following, choose the single best answer. C It contains both endolymph and perilymph, which communicate with ➜ Surgical anatomy of the ear CSF. 1. Which of the following statements is D It contains both endolymph true? and perilymph, which are not in A The external ear canal is approximately communication with CSF. 2 cm in length. E It contains endolymph and perilymph, B The outer third of the ear canal is which have a high potassium cartilaginous, the inner two-thirds bony. concentration. C The epithelium of the outer ear canal 5. Which of the following statements migrates outwards from the tympanic regarding the vestibular semicircular membrane. canals are true? D The pinna consists of fibrocartilage. A They contain otoconial membranes. 2. What does the tympanic membrane B They receive innervation from the consist of? greater auricular nerve. A Two layers: epithelial and mucosal. C They are embedded in the skull B An upper portion called the pars tensa positioned in a single plane. and a lower portion called the pars D They are supplied by the cochlear nerve. flaccid. E They react to angular acceleration. C A single layer of stratified squamous 6. What nerve does not supply sensation epithelium. to the external ear? D Three layers: epithelial, fibrous and A Cranial nerve V mucosal. B Cranial nerve VII E Two layers: mucosal and hyaline cartilage. C Cranial nerve VIII 3. Where does the facial nerve exit the D Cranial nerve IX temporal bone? E Cranial nerve X A Stylomastoid foramen B Internal acoustic meatus ➜ Radiological investigation of C Petrotympanic fissure the ear D Facioparotid foramen 7. Which of the following statements are E Foramen ovale. true? 4. Which of the following statements A Computed tomography (CT) is the regarding the cochlea are true? investigation of choice for identification A It contains endolymph, which of lesions of cranial nerve VIII. communicates with cerebrospinal fluid B Magnetic resonance imaging (MRI) is the (CSF). investigation of choice for identification B It contains perilymph, which of lesions of cranial nerve VIII. communicates with CSF. C MRI is the investigation of choice for imaging cholesteatoma.

319 HEAD AND NECK ➜ . Wheredoestheexternaleardevelop 8. 11. Which of the following statements Whichofthefollowing 11. statements Whichofthefollowing 10. statements Which ofthefollowing 9. 320

ear Conditions oftheexternal E D C B A E D C B A E D C B A E D C B A E D diabetics. It shouldbesuspected inelderly It iscausedbystreptococcalinfection. VII. It sparescranialnerve It tendstobebilateral. It tendstorunanindolentcourse. are true? regarding necrotisingotitisexterna sepsis. due totheriskofoverwhelming It shouldnotbetreatedwithsteroids, damage totheskin. microscopic auraltoilet,toavoid It shouldnotbetreatedwith infection. It iscommonlycausedbystreptococcal antibiotics. It isbesttreatedinitiallywithoral by painonmovementofthepinna. It canbedistinguishedfromotitismedia regarding otitisexternaaretrue? cosmetic deformity. It willresultinonlyshort-lasting drainage. It shouldbetreatedwithincisionand It shouldbetreatedwithaspiration. between theskinandperichondrium. It iscausedbyacollectionofblood It willresolvespontaneously. are true? regarding anauricularhaematoma The secondandthirdbranchialarches. The firstandsecondbranchialarches The thirdbranchialarch The secondbranchialarch The firstbranchialarch scanning. by preoperativeMRI Bony abnormalitiesshouldbeexcluded the middleear. bonyabnormalityin insufficient toshow scanningis The resolutionofCT from? ➜ statements Whichofthefollowing 12. 13. Which of the following statements Whichofthefollowing 13. 14. Which of the following statements Whichofthefollowing 14. 15. Which of the following statements Whichofthefollowing 15.

Conditions ofthemiddleear B A E D C B A E D C B A E D C B A E D C external eararetrue? about malignantlesionsofthe are true? regarding trauma ofthemiddleear surrounding structures. areunlikelytohavespreadfrom They chemotherapy. arebesttreatedwith They submandibular region. metastasisefirsttothe They aremostcommonlycarcinoma. They aremostcommonlymelanoma. They It presentswithotalgia. It isduetoEustachiantubedysfunction. It causessystemicupsetinthechild. membrane. ofthetympanic It resultsinperforation It isuncommoninchildren. are true? regarding otitismediawitheffusion effusion. It isassociatedwithasterilemiddleear It isnotassociatedwithmastoiditis. pneumonia. It iscommonlycausedbystreptococcus rupture ofthetympanicmembrane. It ismostpainfulimmediatelyfollowing It ismostcommoninadults. media aretrue? regarding acutesuppurative currently berepairedsurgically. Damage totheossicularchaincannot corrected surgically. identify abnormalitieswhichcanbe scanningofthemiddleearcannot CT affects thesmallestossicle,stapes. Dislocation oftheossicularchainusually the canal. inspectionof suctioned cleartoallow Blood intheearcanalshouldbe membrane usuallyhealsspontaneously. Traumatic ofthetympanic perforation 16. Which of the following regarding Whichofthefollowing 16. 20. Which of the following isnota Whichofthefollowing 20. Whatischolesteatoma? 19. presenting Whatarethetypical 18. statements Whichofthefollowing 17. E D C B A E D C B A E D C B A E D C B A E D C B A Subdural abscess. abscess Extradural Periorbital cellulitis Intracerebral abscess Meningitis otitis media? ofchronic recognised complication immunocompromised patient. An infectionmorecommoninthe A malignantconditionwithmetastasis metastasis A malignantearconditionwithout destructive A benignconditionwhichcanbelocally A benignconditionwhichisself-limiting Pain andtinnitus. Tinnitus anddizziness Dizziness andhearingloss Pain anddischarge Otorrhoea andhearingloss otitis media? symptoms associatedwithchronic It respondstoadenoidectomy. carcinoma. nasopharyngeal It maybethepresentingfeatureof medically. If persistent,itshouldbetreated It tendstopersist. It isacommonfinding. adults aretrue? regarding middleeareffusionin 6 weeks. percentover improvement in50 resultsin A watch-and-waitpolicy should notswimforaround1year. children grommetinsertion, Following grommet removalinmostchildren. requireslater Grommet insertion effusion hasbeenproveneffective. Medical treatmentofotitismediawith diagnosed improvesoutcome. Urgent treatmentofeffusiononce in childrenaretrue? treatment ofotitismediawitheffusion ➜ statements Whichofthefollowing 21. 23. Which of the following statements Which ofthefollowing 23. 22. Which of the following statements Which ofthefollowing 22. 24. Which of the following statements Which ofthefollowing 24.

Conditions oftheinnerear B A E D C B A E D C B A C B A E D C regarding otosclerosisaretrue? inner eararetrue? regarding congenitalanomaliesofthe The mostcommonmiddleeartumour about middleeartumoursaretrue? The diseaseisoftenbilateral. effective. Surgical treatmentisunlikelytobe commonly affected. The malleusistheossiclemost Men aremorecommonlyaffected. diagnosis. An abnormaleardrumsuggeststhe frequency hearing loss frequency A conditionwhich resultsinalow- elderly A conditionwhichtendstoaffectthe adolescents A conditionwhichtendstoaffect about presbycusisaretrue? will alsohaveanabnormalmiddleear. A childwithacongenitalhearingloss of thecochlearnerve. hearing lossareduetomaldevelopment Most casesofprofoundsensorineural implantation offerssignificantbenefits. In correctlyselectedcases,cochlear to thesocialstigmaofthesedevices. should notbeofferedhearingaidsdue Children withprofoundhearingloss be basedonclinicalsuspicion. with hearingtests,sotreatmentshould Children cannotbeformallyassessed with middleeartumours. foramen donotoccurinassociation inthejugular of thecranialnerves Because ofthebonyanatomy,palsies ear tendstosparethefacialnerve. Squamous cellcarcinomaofthemiddle present withaconstanttinnitus. Vascular middleeartumourstendto is aglomustumour. The mostcommonmiddleeartumour is squamouscellcarcinoma. 321

44: THE EAR HEAD AND NECK ➜ Extended matching questions Extended 27. Which of the following statements Whichofthefollowing 27. statementsis Which ofthefollowing 26. temporalFollowing bone fracture, 25. 322

1. Otologicalsepsis F E D C B A E D C B A E D C B A E D C B A E D Surgical repairof the eardrum Topical antibiotic dropswithaprolongedcourse ofintravenous(IV)antibiotics Topical antibioticandsteroid drops Prescribe analgesiawithsystemicantibiotics ifsymptomsdonotsettlewithin48h steroidsprays Prescribe nasal decongestantsprays Prescribe nasal manoeuvre. It maybehelpedwitharepositioning nystagmus. intheabsenceof It causesvertigo of recentheadinjury. It canonlybediagnosedintheabsence It isassociatedwithintermittenttinnitus. lastinghours. It causesvertigo positional vertigoaretrue? regarding benignparoxysmal risk ofototoxicityishigh. Loop diureticsshouldbeusedwhenthe factor inototoxicity. Renal functionmaybeanimportant treatment. Ototoxicity willmanifestduring function overvestibularfunction. Ototoxic drugsselectivelyimpaircochlear membranes. tympanic must beavoidedinperforated Aminoglycoside-containing eardrops true regardingototoxicdrugs? nothing totheexamination. oftheearcanalwilladd Examination assessment oftemporalboneanatomy. preoperative Plain X-raysallow recover giventime. Sensorineural hearinglosswilltendto uncommon. Hearing lossisvery function. facial nerve toassesanddocument It isimportant aretrue? which ofthefollowing helped byhearingaids. A conditionwhichisunlikelytobe ofthecochlea hair cellsattheapex A conditioncausedbylossofcochlear ➜ statements Which ofthefollowing 28. 30. Which of the following regardingthe Which ofthefollowing 30. 29. In patientspresentingwithfacial 29.

Neoplasms C B A E D C B A E D C B A E D Ménière’s diseasearetrue? regarding thevertigoassociatedwith true? management ofacousticneuromaare parotid andthecranialnerves. shouldincludetheear, Examination be done? should palsy, whichofthefollowing It iscausedbyrecurrentinfection. It isanisolatedsymptom. It lastsforsecondstominutes. It lastsforminutestohours. It lastsfordaystoweeks. mandatory. diagnosis,treatmentis Following choice. scanningistheinvestigationof CT and hearinglevels. involve factorsincludingtumoursize Treatment and optionsarecomplex appropriately timedtreatment. for allows Predictable tumourgrowth this condition. Hearing willbelostduringtreatmentfor deteriorationinvision. to prevent open advised tokeeptheaffectedeye During treatment,patientsshouldbe is unlikely. Patients shouldbewarnedthatrecovery ear isdischarging. Bell’s palsyshouldbediagnosedifthe Antibiotics shouldbeprescribed. ➜ ➜ 1. 1. C Answers: Multiple choicequestions Choosethemostappropriate responseforthequestionsbelow: scenariosgivenbelow: Choosethemostappropriateactionforeachofclinical

Surgical anatom 5 4 3 2 1 2. Appliedanatom 6 5 4 3 2 1 tympanic membrane outwardsalongtheearcanal,making theearself-cleaning. ofthepinnatobonymedial third.Epitheliummigratesfrom the from theelasticcartilage extending theoutertwo-thirdsarecartilage, earcanalisapproximately 3cminlength, The external J I H G F E D C B A J I H G structure? cancerduetopain referred throughwhich Otalgia maybeapresentingfeatureoflaryngeal Huntsyndrome? thepresenceofvesiclesinearcanalRamsay Which structureexplains structures? fromwhichembryological eardevelops The external Which areaismostcommonlyaffectedbyotosclerosis? enterthetemporalbone? Where doesthefacialnerve ComaScaleof9. andaGlasgow pyrexia byotalgiapresentstocasualtywitha ofrecentviralillnessfollowed A 28-year-oldwithahistory foul-smelling discharge. presentswithatticoantralchronicotitismedia(cholesteatoma)andpersistent A fit68-year-old episode ofotorrhoea. ofthetympanicmembraneandafirst A 52-year-oldpresentswithachronicperforation palsy.A 76-year-old otalgia,otorrhoeaandfacialnerve diabeticpresentswithsevere arecentholiday.discharge following presentswithaunilateralitchy,painfulearandassociated A 36-year-oldpatientwitheczema ear drum. systemicupsetandabulgingred A 4-year-oldwhoispullingattheirearpresentswithpyrexia, Jugular foramen Stapes Incus Internal acousticmeatus Stylomastoid foramen X Cranial nerve IIX Cranial nerve VII Cranial nerve Second andthirdbranchialarches First andsecondbranchialarches ofinternalacousticmeatus MRI Plain X-rayoftheaffectedtemporalbone scanoftemporalbonesandbrain Urgent contrast-enhancedCT thedisease toexteriorise Surgery y oftheear y oftheear 323

44: THE EAR HEAD AND NECK ➜ ➜ 10. A 9. 8. D D 7. B 6. C 5. E 4. B 3. A 2. D 324

Conditions of the external ear Conditions oftheexternal Radiological investigationoftheear malleus, is called the pars tensa, andthesectionsuperiortolateralprocessiscalledpars malleus, iscalledtheparstensa, squamous epitheliallayer. thelateralprocessof Theinferiorsectionofthemembrane,below middlefibrousandanouterstratified The tympanicmembraneconsistsofaninnermucosal, debris removalwith microscopicassistanceifthisfails. Systemic treatmentisrarely required. seen inotitismedia.Topical by antibiotics andsteroidsaretheinitialtreatmentofchoice,followed including pseudomonascanbeinvolved. Painonmovementofthepinnaistypical,whichnot earcanal.Pathogens conditionoftheexternal isageneralizedinflammatory Otitis externa the treatmentofchoice. the ruleandincisiondrainagewith antibioticcoverandpostoperativepressuredressingforms deformity. resolution,recurrenceis Althoughaspirationofahaematomawillresultintemporary necrosiscanoccur,resultinginalasting derives itsbloodsupplyfromthisperichondrium,cartilage cartilage As resultsinbleedingintothespacewhich develops. perichondrium fromthecartilage blunttraumatothepinna.Theshearingof Auricular haematomastendtofollow which canbeconfirmedduringsurgery. identificationofmost bony abnormalities, scanningallows The definitionofhigh-resolutionCT the bonyanatomyofmiddleearpreoperatively,althoughnotallsurgeonsuseitasroutine. isusefultoidentify suchasschwannomas.CT isusedtoidentifylesionsoftheacousticnerves MRI softtissuelesionsmostaccurately. shows bonylesionsandMRI Computed tomographyshows Huntsyndrome. –Ramsay zoster infectionofCN7 cancer. thepresenceofvesiclesinearcanal inassociationwithherpes laryngeal Italsoexplains thevariedcausesofreferredotalgia,classically supplyexplains of C2,3.Thisvariationsensory andX,aswellfrombranches IX V,VII, supplytotheearcomesfromcranialnerves Sensory activity carriedinthevestibularnerve. in the cochlea,shearingforcesproducehaircellmovementwhichistransducedintoelectrical As reacttolinearaccelerationandgravity.react toangularacceleration,whilsttheutricleandsaccule The threesemicircularcanalsliewithinthetemporalboneatrightanglestooneanother. They intoelectricalenergy. membrane withinthecochleavibrateandtransducemechanicalenergy whilsttheendolymphhasahighpotassiumconcentration.Haircellsonbasilar with theCSF, and endolymphareseparatedbyReissner’smembrane.Theperilymphisincommunication turns.Withinthisbonyspace,perilymph The cochleaisaspiralconsistingoftwoandthree-quarter stylomastoid foramen,whichisidentifiedduringparotidsurgery. theskullat labyrinthine, tympanicandmastoidsegmentsinthebonyfallopiancanal.Itexits entersthetemporalboneatinternalacousticmeatus,windingitswaythrough The facialnerve flaccid. 18. A ➜ 17. D 16. E 15. D 14. C 13. A 12. B 11. E

loss. Pain is unusual, and dizziness or vertigo isassociated withadvanceddiseaseaffecting the loss. Painisunusual, anddizzinessorvertigo Chronic otitismedia typicallypresentswithintermittent dischargeandamildconductive hearing which makesadenoidobstructionof theEustachiantubeuncommoninthisagegroup. spacelarge, Theadenoidissmallinadults,andthepostnasal if consideringgrommetinsertion. Conditions ofthemiddleear be suspected of the condition. As the condition spreads it can involve cranial nerves VII, IX andX. IX VII, theconditionspreadsitcaninvolvecranialnerves be suspectedofthecondition.As should Patientsareoftenimmunocompromised,andelderlydiabeticswithotitisexterna externa. otitis This condition,whichresultsinosteomyelitisoftheskullbase,presentswithunilateralsevere These patients should have an examination These patientsshouldhaveanexamination assuspiciousneoplasticobstruction oftheEustachianmaybecause. should beviewed Middle eareffusionisrareinadultsandwhenpresenttendstoresolve quickly. Persistent effusion 6–18 monthsandthereisnoneedtoavoidswimming duringthistime. outinaround andadenoidectomyaresurgicaloptions.Grommetsgrow Grommet insertion andaudiologicalassessment.Medicaltreatmentsare oflimitedvalue. by serialexaminations arethosewhohaveabilateralpersistenteffusion.These childrencanbeidentified opportunities Most middleeareffusionswillresolvenaturally. Childrenatrisk ofmissingoutoneducational problems. Painisrareinthiscondition. child tendstopresentwithfluctuatinghearinglossandpossibledelayed speechwithbehavioural ispresent,thefluidcannotcollect.The transudate fluidinthemiddleearspace.Ifaperforation to poorEustachiantubefunction,leadinganegativemiddleearpressureandcollectionof The majorityofchildrenwillhaveanepisodeotitismediawitheffusion.Itisthoughttobedue middle earandsomastoiditisisassociatedwithotitismedia. use onlyifsymptomsdonotsettlewithin48h.Themastoidaircellscommunicatefreelywiththe guidelines suggesttheprovisionofanantibioticprescriptiontoparentswithinstructions antibiotics iscontroversial.Almostallepisodeswillresolvewithoutantibiotics,andcurrentpractice most commonorganismsinvolved.Treatment willdependonthestateofchild,asrole oftheeardrum. perforation behind anintacttympanicmembrane.Theriseinpressureresultspain,whichsubsideson Suppurative otitismediaismostcommoninchildren,andresultsapurulentfluidbuildingup althoughitisnotmandatory. thepositionofossiclespriortosurgery, scanning canshow can bereconstructedbyreshapingthedamagedossicleorusingaprostheticimplant.CT and blastinjuriesdoworse.Inossiculartraumatheincusisusuallyinvolved,damage towelding Traumatic secondary tendtohealspontaneously,althoughperforations perforations be treatedprimarilywithsurgery. space.Mostresectabletumoursshould invaded bytumoursfromtheparotidglandorpostnasal crustinglesions.Theearcanal,whichissurroundedonallsides,maybe present asslow-growing, tendto cellorsquamouscarcinomas.They eararebasal Most malignanciesoftheexternal Treatment issystemicandprolongedwith6weekcoursesofantibioticscommon. Streptococcus pneumoniae ϩ / Ϫ biopsy of the nasopharynx asajoint procedure biopsyofthenasopharynx and Haemophilus influenzae arethe 325

44: THE EAR HEAD AND NECK 21. E 20. C 19. B 22. B ➜ 25. A 24. B 23. C 326

Conditions oftheinnerear discharge. Facial paralysisoftenoccurs. are notuncommon.Squamouscellcarcinomatendstopresentwithadeep-seatedpainandbloody andXII) X,XI IX, palsies(VII, appearanceandcranialnerve ear drummayhaveaclassiccherry-red tinnitusandhearingloss.The arehighlyvascularandtendtopresentwithapulsatile tissue. They The mostcommonmiddleeartumourisaglomustumour,fromthenon-chromaffinparaganglionic complicationsincludesensorineuralhearinglossandbalancedisturbance. percentofcases.Rare 90 bilateral. Treatment whichissuccessfulinover optionsincludereassurance,hearingaidandsurgery, leads toaconductivehearingloss.Women aremorecommonlyaffected,andthediseaseisoften the stapesinovalwindow. and Thisreducesthemobilityofstapesinovalwindow In otosclerosis,abnormalbonedepositioninthetemporaloccurs,mostcommonlyaffecting is acomplicationofsinusitisnototitismedia. complications ofotitismediaincludemeningitisandsigmoidsinusthrombosis.Periorbital cellulitis andsubduralabscess.Otherintracranial Chronic otitismediacanleadtointracerebral,extradural of hearingandbalancedysfunctionaswellintracranialsepsis. Thisputsthepatientatrisk betweentheearandmiddleposteriorcranialfossae. partitions disease canerodethroughlocalstructures,includingtheossicles,cochlea,vestibuleandbony andchronicfouldischargecanbebothseensmelt.The osteomyelitisdevelops, a low-grade thediseaseprogresses, accumulation ofcellsandgradualenlargementtheretractionpocket.As normalepithelialmigration,whichresultsin the tympanicmembrane.Thepocketprevents Cholesteatoma resultsfromacollectionofsquamousepitheliuminretractionpocket chronic otitismedia. vestibular labyrinth.Tinnituscanbeassociatedwithhearinglossbutisnotatypicalfeatureof trauma. Classically fracturesaredescribedinrelation to theearcanalaslongitudinalortransverse. head The temporalbone isthehardestboneinbody, but itcanbefracturedinsevere causedbythiscondition. disability bilateral andcanbeassociatedwith tinnitus.Hearingaidsareusedinthetreatmentof turnofthecochlea.Itisgenerally atthebasal elderly andistheresultofhaircell loss starting Presbycusis ischaracterisedbytheloss ofhearinginthehighfrequencies.Ittendstoaffect totheirnormalhearingpeers. atasimilarlevel schools andperform bepermanentlydeafcanattendmainstream significant benefits.Childrenwhowouldotherwise with aprofoundhearinglossmaybecandidatesforcochlearimplantation, whichcanoffer Patients with hearingaidsduringthisperiodofneuralplasticitytooptimisetheir development. thatchildrenwithahearing lossareprovided withthechildanditisimportant system develops whichhasimprovedthedetectionofchildrenwhoarehearing-impaired. Theauditory in theUK, routine inneonates and includesbothsubjectiveobjectivetesting.Objectivetestingis now concerns abouttheirchild’shearingshouldbetakenseriously. isaskilledfield Paediatricaudiology profound sensorineuralhearinglossareduetoofcochlearhaircells. Anyparentwhohas withorwithoutmiddleearabnormalities.Mostcasesof Congenital innereardisorderscanexist 27. E 26. D 29. A 28. B nerve palsies should prompt further investigation. palsiesshould promptfurther nerve prompt urgentreferral forintravenousantibiotics.The presenceofaparotidmassorother cranial Thepresenceofactiveearinfectionshould includingthe cranialnerve. areas shouldbeexamined, through thetemporalboneandout intotheparotidglandbeforereachingfacialmuscles.All thebrainstem,runs exits the paralysiscanbefound.Itshould berememberedthatthefacialnerve that steroidtreatmentimprovesoutcome. Bell’spalsycanonlybediagnosedifnoothercausefor suggesting withevidence dysfunction. Around85–95percent ofpatientsmakeagoodrecovery withinitsbony canalandsubsequent these isBell’spalsy,whichresultsinswellingofthenerve isacommoncauseofunilateralpalsy.Viral infectionofthefacialnerve Themostcommonof thatsupplyit. todestroythelabyrinthornerves interventions apharmaceuticallarbyrinthectomyormoredestructivesurgical to themiddleearperform Treatment theinnerearpressure, injectionofgentamicin includesdiureticswhichmayhelplower masslesionsatthecerebellopontineangle,which maymimicthecondition. toexclude MRI although itmayberelatedtopressurechangeswithintheinnerear. Investigationwillinclude associated withtinnitus,hearinglossandafeelingofauralfullness.The causeisunclear whichlastsforminutes tohoursandis Ménière’s diseasedescribesanintermittentvertigo hasten theresolutionofsymptoms. manoeuvrecanbeusedto repositioningmanoeuvressuchastheEpley 3 months,butparticle butmayalsobespontaneous.Theconditiontendstoself-limiting,lastingaround head injury andnystagmuswhichlastsforsecondsisprovokedbyheadmovement.Itmayfollow vertigo withtheirfunctionandproduces membrane floatingintothesemicircularcanals.Thisinterferes isprobablycausedbyfragmentsoftheotoconial (BPPV) positionalvertigo Benign paroxysmal hearing ear. theiruse,althoughmanywouldbecautious,especiallyinanonly supported experts from UK coursescausesignificantdamage.Arecentconsensusstatement during anactiveinfection,short that, thereislittleevidence however aminoglycoside dropsinthepresenceofaperforation, arecriticallyillduringthetimeoftreatment.Therehasbeencontroversyoveruse if they problems Ototoxicity maybeadelayedphenomenon,andsomepatientsunabletoreport risk,andloopdiureticspotentiatetheototoxiceffectsofaminoglycosides. function areatparticular hearingloss.Patientswithpoorrenal producesareversible hearingloss.Aspirin an irreversible system. Aminoglycosidesandcisplatinarecommonlyencounteredinclinicalpracticecause Many drugsarepotentiallyototoxic,withsomeaffectingthecochleaandvestibular scanningofthetemporalboneanatomy. shouldbeprecededbyhigh-resolutionCT intervention recorded. Ifthesepatientswakewithacompletepalsy,decision-makingismorecomplicated.Any functionwillnotbe Some patientswillbeunconsciousatthetimeofassessment,andfacialnerve tooedema,sowillnotbeimprovedwithsurgery. and anylaterlossoffunctionissecondary orcompletely,itsintegritycanbeassumed isfunctional,eitherpartially considered. Ifthenerve shouldbe islikelytohavebeendamaged andsurgicalintervention paralysis ispresent,thenerve isessential.Ifimmediatetotal ofthefacialnerve the cochleahasbeendisrupted.Assessment haemotympanum orossiculardisruption,fromsensorineuralloss,whichispermanentif the diagnosis.Hearinglossiscommon,eitherfromconductiverelatedtobloodincanal, afractureintheroofofcanal,confirming oftheearcanalmayreveal fractures. Examination cochlea. Transverse fractureshaveahigherrateofdamagetothesestructuresthandolongitudinal andthe Structures withinthetemporalbonewhichmaybedamagedincludefacialnerve 327

44: THE EAR HEAD AND NECK ➜ ➜ 6H 5G 4D 3E 2D 1C matchingquestions Answers: Extended 30. C 328

1. Otologicalsepsis Neoplasms Acoustic neuromaisthemostcommontumourofcerebellopontineangle.Itnormally thrombosis. An urgent contrast-enhanced CT scan will allow assessmentofintracranialdisease. scan will allow thrombosis. Anurgent contrast-enhancedCT abscess, intracerebralabscessandvenous sinus include meningitis, subduralabscess,extradural anintracranial complicationofacuteotitismedia.These This patientmayhavedeveloped disease. ofthe lead tointracranialsepsis.Appropriate treatmentissurgicalandinvolvesexteriorisation disease willincreaseinsizeandmay damagetheossicles,cochleaandvestibulecan tocontinue,the debris becomesinfectedandgivesrisetoafoul-smellingdischarge.If allowed In cholesteatoma,keratindebriscollectsinaretractionpocketofthetympanic membrane.This ofsymptoms. depend ontheseverity inthelongtermwill patients. Thedecisionastowhetheroperateclosetheperforation insuch coursesofaminoglycoside-containingdrops can beusedsafely and steroiddrop.Short andshouldbetreatedwithanantibiotic This patienthaschronicotitismediawithperforation weeks ofIVantibiotics. Treatmentavailable. Auraltoiletisalsoimportant. willbelong term andpatientsmayrequire6 involves parenteralandtopicalantibioticstargetedagainstPseudomonas,untilcultureresultsare whichtraversetheskullbasecanbeaffected.Treatmentto involvethebone,cranialnerves thediseaseprogresses immunocompromised andleadstoosteomyelitisoftheskullbase.As Thisconditionismorecommoninthe The diagnosisisofnecrotisingotitisexterna. resolution, auraltoiletmayberequired. treatment shouldincludetopicalantibioticsandsteroiddrops.Ifthisdoesnotleadtosymptom oftenprecipitatedbyswimmingwhilstonholiday. Theinitial The diagnosisisotitisexterna, helpful inacuteotitismedia. steroidsare decongestantsornasal thatnasal advised tousetheprescription.Thereisnoevidence symptoms normallyresolvespontaneously. arenotimprovingwithin48h,theparents Ifthey antibiotics andreassurance.Aprescriptionformaybeprovidedwithadvicethatthe Acute otitismediaisthelikelydiagnosis.Currenttreatmentguidelinessuggestprovisionof to undergosuchamajorprocedure. scans.Somepatientswilldecidenottohavetreatment,whileothersbefit lifelong interval ofthetumour,althoughpatientwillrequire gamma knifecanbeusedtoarrestthegrowth withthe Radiotherapy have beendescribed,eachofwhichhasadvantagesanddisadvantages. inparticular. Differentsurgicalapproachestothisarea totheirhearingandfacialnerve surgery istheinvestigationofchoice.Patientsshouldbemadeawarerisks radiotherapy. MRI or scanning,surgery Treatment withinterval andwillincludeobservation optionsarecomplex andunpredictable. ofthesetumoursisslow Growth a schwannomaofthevestibularnerve. ➜ 5E 4C 3A 2I 1G

2. Appliedanatom The seventh cranial nerve enters the temporal bone at the internal acoustic meatus and exits entersthetemporalboneatinternalacousticmeatusandexits cranialnerve The seventh present withipsilateralotalgia. tumoursmayalso involvementinoropharyngeal IX) (cranialnerve nerve Glossopharyngeal X)mayleadtootalgia. (cranialnerve cancer,involvementofthevagusnerve In laryngeal involvement. Otalgia isanominoussigninheadandneckcancer,asitsuggestscranialnerve aural toilet. whichoftenrequirestopicaltreatmentand develops apainfulotitisexterna form andbreakdown, whichsupplytheearcanal.Whenvesicles The infectionalsoaffectsfibresfromthefacialnerve whichleadstofacialpalsy. Huntsyndromeisavaricellazosterinfectionofthefacialnerve Ramsay withtherockingmovementofstapes,resultinginaconductivehearingloss. interferes withtheovalwindow. bone affected isthefootplateofstapes,whereitarticulates Theexcess Otosclerosis resultsinbonedepositionwithinthetemporalbone.Thesitemostcommonly labyrinthine, tympanicandmastoidparts. at thestylomastoidforamen.Duringitscoursewithintemporalbone,itcanbedividedinto y oftheear 329

44: THE EAR 45 Pharynx, larynx and neck

Multiple choice questions In the following, choose the single best answer. B A system for describing lymph nodes of the neck ➜ Clinical anatomy and C A system for describing the salivary physiology glands 1. The pharynx extends from the level of D A system for describing movement of the skull base to which vertebra? the larynx during swallowing A Fourth cervical vertebra E A system for describing venous drainage B Fifth cervical vertebra of the neck. C Sixth cervical vertebra ➜ D Seventh cervical vertebra Diseases of the pharynx E First thoracic vertebra. 6. Which of the following is an indication for adenoidectomy? 2. Infection of the parapharyngeal space A Hyponasal speech places the patient at risk of: B Velopharyngeal insufficiency A Otitis media C Recurrent epistaxis B Acute tonsillitis D Cleft palate C Dental infection E Obstructive sleep apnoea. D Mediastinitis E Epiglottitis. 7. In which patients presenting with unilateral nasal obstruction 3. The superior and recurrent laryngeal and recurrent epistaxis should nerves are branches of which cranial angiofibroma be suspected? nerve? A Elderly male smokers A Eighth cranial nerve B Adolescent patients of Chinese origin B Ninth cranial nerve C Middle-aged patients with atopy and C Tenth cranial nerve asthma D Eleventh cranial nerve D Adolescent males E Twelfth cranial nerve. E Immunocompromised patients. 4. Which of the following is not a 8. Which virus is associated with function of the larynx? nasopharyngeal carcinoma? A Preparation of fold bolus for swallowing A Epstein–Barr virus B Protection of the lower respiratory tract B Varicella zoster C Phonation C Human papillomavirus (HPV) 6 and 11 D Control of pressure during respiration D Herpes simplex E Fixation of the chest. E Rhinovirus. 5. What does the level system for 9. Treatment of a peritonsillar abscess describing the neck refer to? requires: A A system for describing skin lesions A Tonsillectomy involving the neck B Transoral incision and drainage

330 13. Which of the following statements Whichofthefollowing 13. (SCC) Squamouscellcarcinoma of 12. pouchdevelopsinan Apharyngeal 11. abscess shouldretropharyngeal How 10. E D C B A E D C B A E D C B A E D C B A E D C through cranial nerve XI. through cranialnerve It maypresentwithotalgiareferred spread. It isassociatedwithsubmucosal nodes. lymph It rarelyspreadstocervical outpatient clinic. It canbeadequatelyassessedinthe to earlypresentation. It carriesarelativelygoodprognosisdue are true? regarding SCC ofthehypopharynx Poorly fittingdentures. furniture industry tohardwooddustfromthe Exposure fish Consumption ofsalted Epstein–Barr virus Cigarette smoking isassociatedwith: the oropharnyx muscles. inferior constrictorandcricopharyngeus borderofthe Between thelower tonsillectomy Created following constrictors meetthemidlineraphe At theweaknesswherepharyngeal superior constrictorandskullbase At thepotentialspacebetween membrane penetratesthethyrohyoid nerve At thepointsuperiorlaryngeal area ofweakness–where? an outpatient. As With antibioticsalone patient headdown Under generalanaestheticwiththe supine Under localanaestheticwiththepatient position Under localanaestheticinthesitting be treated? Ultrasound/CT-guided aspiration. Endoscopic de-roofing Transcervical incisionanddrainage ➜ 4 Inpaediatricsuspectedacute 14. 17. Which of the following arenot Whichofthefollowing 17. are Whichofthefollowing 16. arenot Whichofthefollowing 15. 18. Which of the following istrueof Whichofthefollowing 18.

Diseases ofthelar D C B A E D C B A E D C B A E D C B A E D C B A E contraindicated? is epiglottitis, whichofthefollowing Acute upper airway obstruction Acute upperairway fortracheostomy? indications Tracheostomy. intubation Emergency undergeneralanaesthetic Examination headlight andspatula ofthemouthwith Examination Antibiotics tracheostomy. to a It shouldroutinelybeconverted cricoid cartilage oftheanterior It involvesexcision obstruction It isalong-termsolutiontoairway stenosis. It maybeassociatedwithsubglottic surgical expertise. of The procedurerequiresahighlevel cricothyroidotomy? A speakingvalveattachment. fornasogastricfeeding A port An outerflange An innertube cuff A low-pressure tube? features ofamoderntracheostomy Reduction ofanatomicaldeadspace. Reduced mucusproduction from theupperairway Increased rateofmoistureexchange An increaseintheworkofbreathing A decreaseinalveolarventilation advantages ofatracheostomy? ventilation. Prolonged artificial airway Protection ofthelower Tension pneumothorax obstruction Potential upperairway y nx 331

45: PHARYNX, LARYNX AND NECK HEAD AND NECK ➜ 23. Which of the following appliesto Whichofthefollowing 23. Whatisthepreferredtreatmentfor 19. 22. Which of the following statements Whichofthefollowing 22. presentswith: cancer Earlylaryngeal 21. Whichstatementistrueofunilateral 20. 332

The neck E D C B A E D C B A E D C B A E D C B A E D C B A They containclearfluid. They sternomastoid muscle. the upperandmiddlethirdof arefoundatthejunctionof They excision. increase insize,facilitatingsurgical leadstoan Infection ofthecyst neck. presentinthemidlineof They period. usuallypresentintheneonatal They cancer.early laryngeal It istherecommendedtreatmentfor patientsfromspeaking. It prevents subsides. closed oncepostoperativeswelling The surgicaltracheostomymaybe without affectingthevoice. The procedurecantreatmalignancy tract. fromtheGI the airway The procedureinvolvesseparationof istrue? regarding laryngectomy Weight loss. Dysphagia Hoarseness Lymph nodemetastasis Otalgia It leavesthevocalcordfullyadducted. It requirestracheostomy. voice. It maybeassociatedwithanormal larynx. It shouldbeinvestigatedbyCT It ismostcommonlyright-sided. cordpalsy? vocal Inhaled corticosteroid. Steroid injection Radiotherapy Surgical excision Speech therapy branchial cysts? foldnodules? vocal ➜ istrueofa Whichofthefollowing 24. 26. Following blunt laryngeal trauma, bluntlaryngeal Following 26. 25. A thyroglossal duct cyst isaremnant Athyroglossal ductcyst 25. 7 Ludwig’sanginadescribeswhich 27.

Trauma totheneck D C B A E D C B A E D C B A E D C B A E cystic hygroma? cystic which ofthesestatementsistrue? The cervical sinus The cervical trunk The thyrocervical of whichstructure? pattern. undergoapredictablegrowth They for treatment. Cosmetic concernistheonlyindication arefirmonpalpation. They arebrilliantlytranslucent. They presentduringpuberty.They the cervical dermatomes. the cervical tozosterlesionsin Cellulitis secondary swallowing Retrosternal chestpainrelatedto Gingivostomatitis related tonecrosis with darkdiscolorationinpatches neck Spreading cellulitisofthelower mouth oedemaofthefloor inflammatory Submandibular swellingwith condition? procedure isrequiredforremoval. stentisplaced,anopen If alaryngeal considered. tracheostomyshouldbe Low function. untouched tooptimiselaryngeal shouldbeleft Fractured subsides. oedema andmaintaineduntilswelling priortotheformationof performed Endotracheal intubationshouldbe function. significant swellingpriortolossof for allows The anatomyofthelarynx base totheneck. The trackofthethyroidfromtongue branchialpouch The fourth neck intothemediastinum The trackofthethymusthrough ➜ ➜ Extended matchingquestions Extended statements Whichofthefollowing 29. Acollar-studabscessreferstowhich 28. Choosethemostappropriate actionforeachofthescenarioslistedbelow:

5 4 3 2 1 J I H G F E D C B A 1. Investigationofthephar Primar A E D C B A A 66-year-old smoker presents with a new-onset leftvocal cordpalsy. smoker presentswithanew-onset A 66-year-old smokerpresentswith hoarsenessandanulceratedareaontheleftvocalcord. A 58-year-old up anddown. massin the leftneckwhichmovessidetobutnot presentswithapulsatile A 46-year-old abnormality onexamination. lymphnodeandnoother malesmokerpresents witha3cmpalpablecervical A 60-year-old which occurshoursaftermeals. A 72-year-oldpresentswithrecurrentaspirationpneumoniaandreflux ofundigestedfood Laryngectomy Referral forradiotherapy underanaesthetic Examination Barium swallow scan Positron emissiontomography(PET) scan MRI scanoftheneckandchest CT Ultrasound Open excision FNA history. percentofpatientshaveafamily 90 thyroidectomy. drains throughthecollarincisionof A postoperativewoundinfectionwhich space the parapharyngeal Tracking ofaperitonsillarabscessinto Tuberculous lymphadenitis cervical lymph node Infection ofanecroticmalignantcervical lymphadenitis Bacterial cervical about carotid bodytumoursistrue? about carotid disease process? y tumoursoftheneck y nx, lar 0 Aprocedurewhichremovesalllymph 30. y nx andneck E D C B A E D C B A radicalneckdissection called: vein andsternomastoidmuscleis jugular nerve, theaccessory preserves nodes ononesideoftheneckbut confirmsthediagnosis. (FNA) Preoperative fine-needleaspiration but notsidetoside. The masscanbemovedupanddown arehormonallyactive. They living athighaltitude. aremorecommoninpopulations They A modifiedradicalneckdissection. A subtotalneckdissection A selectiveneckdissection A classicalneckdissection 333

45: PHARYNX, LARYNX AND NECK HEAD AND NECK ➜ ➜ 3. C 2. D 1. C Answers: Multiplechoicequestions Choosethemostappropriatemanagementforeachofscenarioslistedbelow: 334

Clinical anatom 5 4 3 2 1 J I H G F E D C B A 2. Investigationandmanagementoftheacuteairwa nerve duringthyroidectomy willproduceacordparalysis andresultinghoarseness.Damage to nerve muscles. Damagetotherecurrentlaryngeal thevocalfoldsand allotherintrinsiclaryngeal below tothelarynx suppliessensation nerve supply tothecricothyroidmuscle.The recurrentlaryngeal abovethe truevocalfolds,andmotor tothelarynx suppliessensation nerve superior laryngeal The isderivedfrombranchesofthevagusnerve. ofthelarynx andmotorinnervation The sensory lymph nodesaredentalandtonsilinfections. and thesuperiormediastinum.Themostcommonsourcesofsuppuration intheparapharyngeal predictable patterns.Puscantrackthroughthesepotentialspacestothe para-oesophagealregion forthespread ofinfectionin The deepspacesoftheneckliebetweenfascialplaneswhichallow oesophagus. continuous withthecervical is Atthis point thepharynx ofthecricoidscartilage. level whichcorrespondstothelower vertebra, cervical ofthesixth fromskullbaseto thelevel isamusculartubewhichextends The pharynx larynx. A 4-year-oldwhopresentswithhoarsenessandstridorhaspapillomatouslesionsaffectingthe increasing stridor. A 28-year-oldwhocameoffhismotorbikeandstruckneckonalamppostpresentswith contralateral side. A 21-year-old ofthetonsilanduvulato andtrismushasdeviation withsorethroat,fever droolingandstridor. offever, history A 3-year-oldpresentswithashort with antibiotics,fluidsandanalgesia. 16-year-oldA stertorous failstoimprove withtonsillitisandapositivetestforglandularfever Soft tissueneckX-ray courseofsteroids A short oflesions Endoscopic excision Transfer anaesthetistandsurgeon totheatrewithexperienced ofanoralairway Insertion Cricothyroidotomy Intubation untilswellingsubsides Early tracheostomy Per-oral incisionanddrainage Ultrasound-guided aspiration y andph y siolog y y case 5. B 4. A ➜ 10. C 9. B 8. A 7. D 6. E

Diseases ofthephar withoutreducingdiseasecontrol. XI) related todivisionofcranialnerve These modifiedradicalneckdissectionsreducethemorbidityofprocedure(shoulderpain nerve. theaccessory whichsparethesestructures,particularly procedures havebeendeveloped ourunderstandingofpatternsmetastasishaveevolved, As nerve. vein andtheaccessory ofthesternocleidomastoidmuscle,internaljugular of alllymphaticstheneckwithsacrifice oftheheadandneckinvolvedradicaldissection,aprocedurewhichcomprisedremoval SCC aerodigestive tractmetastasisestopredictableareasoftheneck.Historictreatmentmetastatic systemdescribesthelymphnodesofneck.Squamouscellcarcinomaupper The level chest andaidsinlifting,climbingdefaecation. with vocalcordabductionduringinspiration.Closureagainstraisedsubglotticpressuresplintsthe cycle, forphonation,butalsohelpscontrolpressureduringtherespiratory iswelldeveloped larynx Thehuman andisinvolvedinthecoughreflex. ofrespirationduringtheswallow in thecessation inlet,thefalsecordsandglottis.Italsohelps atthelaryngeal which closesduringswallowing tract.Itactsasasphincter respiratory istoprotectthelower functionofthelarynx The primary cause achangeinvoice,whichmayonlybedetectablewhilstsinging. whichrunsclosetothesuperiorpoleofthyroid,will nerve, branches ofthesuperiorlaryngeal involves admission, with parenteral antibiotics and fluids, however incisionanddrainageare involves admission, withparenteralantibioticsandfluids, however Treatment compromise.Theabscessmay beseenonexamination. airway and mayhavesevere thesignsofinfection Patientswillshow ornasopharynx. with infectionofthetonsil,oropharynx abscessmostcommonly affectschildrenundertheageof1year.Retropharyngeal Itisassociated time. atthesame often performed drainage underlocalanaesthetic.In children,ageneralanaestheticisrequiredandtonsillectomy analgesia andantibioticsareoftenrequired,butdefinitivetreatmentis with transoralincisionand treatmentwithfluids, muscles.Supportive trismus duetoinflammationaroundthepterygoid in theperitonsillarspace.Thetonsilanduvulaarepushedmedially thereisoftensignificant Peritonsillar abscessorquinsydescribesaninfective processwhichresultsinacollectionofpus screening toolinsouthernChinawhenappliedtohigh-riskgroups. and astumourmarkerstodetectrecurrence.IgAantiviralcapsidantigen hasfounduseasa investigation forEpstein–Barrvirus-associatedantigenicmarkerscanbe usedindiseasedetection fish.Serological genetic susceptibilityandanassociationwithtraditionaldietscontaining salted carcinomaisassociatedwithEpstein–Barrvirus,aswelltherebeinga Nasopharyngeal tend tobebilateral. polyps ismorecommoninChinesepopulations.Nasal SCC result inblindness.Nasopharyngeal whichcan and,asitenlarges,causeslocaldamage,includingtotheopticnerve, nasopharynx, angiofibromatendstopresentinadolescentmales.Thetumourofteninvolvesthe Juvenile nasal clinically obviousorasubmucouscleft,areathigherrisk. Patientswithcleftpalate,either which resultsinrefluxofliquidsintothenosewhilstswallowing. combined withtonsillectomy. insufficiency, Therisksofadenoidectomyincludevelopharyngeal obstructionisoften Adenoidectomy forobstructivesleepapnoeainchildrenwithpostnasal y nx 335

45: PHARYNX, LARYNX AND NECK HEAD AND NECK 13. D 12. A 11. E ➜ 16. E 15. C 14. B 336

Diseases ofthelar metastasis common,althoughnotalwayspalpable. receivesarich,bilaterallymphaticsupply,makingcervical margins challenging.Thehypopharynx to detectassecondprimary. whichmakesdeterminingsurgical Lesionscanspreadsubmucosally, aswellvideofluoroscopyandchestX-ray anaesthetic andradiologicalinvestigations–CT/MRI undergeneral endoscopyintheclinic,laryngoscopy involvedflexible andX).Assessment IX nerve late, withvaguesymptomsincludingdysphagia,hoarsenessandotalgia(referredthroughcranial cancerisclassifiedaccordingtothesiteofprobableorigin.Itoftenpresents Hypopharyngeal tobesynergistic. cigarette smoke,theeffectshavebeenshown Alcoholconsumptionisalesssignificantriskfactor,althoughincombinationwith the oropharynx. of oftheheadandneck,includingSCC Cigarette smokingisthemostsignificantriskfactorforSCC contractionwave. of thepharyngeal oftheupperoesophagealsphincterorabnormalities but mayrelatetoincoordinationofrelaxation muscle.Thecauseofthisconditionisnotclear inferior constrictormuscleandthecricopharyngeus atKillian’sdehiscence,whichliesbetweentheinferiorfibresof pouchdevelops A pharyngeal approachescanbeused. the oralcavity,althoughtranscervical intubation doesnotresultinaspirationofpus.Theabscesscavitycanusuallybeaccessedthrough oftheabscessduring positionsothataccidentalperforation inthehead-down be performed indicated. Discussionwithasenioranaesthetistisadvisedandorotrachealintubationshould lead toaspiration during swallowing. can increase inmucus productionwhichcanleadtoobstruction ofthetube.Splinting larynx andthereisan functionsoftheupperairway, due tolossoftheheatandmoisture exchange ofthetrachealepitheliumoccurs of breathingdecreasesandalveolar ventilationincreases.Drying tracheostomy. percentfollowing The anatomicaldeadspaceisreduced byaround50 Thework forthepatient. medications andismorecomfortable areduction insedative for aprolongedperiodmaybesuitabletracheostomy,whichallows toilet.Patientswho areintubatedandventilated airway. accessforpulmonary Thisalsoallows risk ofaspirationandatracheostomycanbeusedtoplacecuffedtube andprotectthelower orcoma,patientsareat headinjury Following is whenyoufirstthinkitmightbenecessary! atracheostomy Thetimetoperform In suchcases,theindicationisusuallyurgentorlife-saving. ofthetracheostomyincasesepiglottitis. obstructionabovethelevel be doneincasesofairway Tracheostomy involvesproducinganentrancetothetracheathrough theskinofneck.Itcan with IVantibioticsuntiltheswellingsubsides. hasbeensecured,thechildwillbemanaged inanintensivecareunit airway required. Onceasafe general anaesthetic.Intubationshouldbeattempted,butifunsuccessful, atracheostomymaybe under should besummonedandthechildtransferredtooperatingtheatreforexamination surgeon anaesthetistandENT arrest.Anexpert be avoidedasthismayprecipitatearespiratory withaspatulaandintravenous(IV)cannulation,should may upsetthechild,includingexamination environment andwithaparentpresenttoencouragethemremaincalm.Anymeasureswhich shouldbenursedinaquiet present withstridorandrequireurgenttreatment.They They Children withepiglottitiswillbefebrileandoftendroolingduetothepainrelatedswallowing. y nx 22. A 21. C 20. C 19. A 18. B 17. D of aremovableinnertubeisthatremovingittocleanthedoesnotleavetracheostomy and outerdiameters,withasingletube,oraninnertubetube.Theadvantage Modern tracheostomytubesaremadeofplasticorsilver. areavailableinarangeofinner They is turned into speech by the patient’s remaining vocal tract, swallowed air can beregurgitated is turnedintospeech bythepatient’sremainingvocal tract,swallowed canbeusedtoproducesoundwhich surgery. patientstospeakfollowing allow Theelectrolarynx folds areremovedwhichclearlychanges thevoice,althoughtherearemultiplestrategieswhich tract.The vocal andtheGI closedtoseparatetheairway permanently, andtheremainingpharynx with thetracheabroughtoutonskinofneck involvesremovalof thelarynx, Laryngectomy function. as thediseaseprogressesitcanleadtosignificantchangesinpharyngeal but cancerdoesnotimpairswallowing, supraglottis, lymphnodemetastasisisrare.Earlylaryngeal glottis itselfhasnolymphaticsupply,sountilthediseaseenlargestoinvolve thesubglottisor The Otalgiaisasignofadvanceddiseaseaspainradiatesthrough thevagusnerve. of thelarynx. surgeonforvisualisation reason patientswithpersistenthoarsenessshouldbereferredtoanENT mostcommonlyaffectsthevocalcordandpresents withhoarseness.For this ofthelarynx Cancer the immediatepostoperativeperiod. in Thecordstendtolieinaparamedianpositionand compromisetheairway require intervention. thyroidectomy,are morelikelyto require atracheostomy,bilateralpalsies,whichcanfollow todocumentcordfunction.Althoughaunilateral cordpalsywillnot ofthelarynx visualisation shouldinclude For thisreason,post-thyroidectomyfollow-up contralateral cordcompensates. Although aunilateralpalsywillleadtoinitialvoicechange,symptomsmayresolveasthe pathtothelarynx. oftheneckandchesttoimagenerve’s Investigation shouldbewithCT routethrough thechest. nerve’s mustbesuspectedgiventherecurrentlaryngeal malignancy Vocal withaleftvocalcordpalsy,bronchial cordpalsycanbeunilateralorbilateral.Particularly must beawarethattheirvoicemaynotreturntoitsoriginalstate. not settlewithspeechtherapy,microsurgicaltechniquescanbeusedtoremovethem,butpatients the vocalfoldiscriticaltoitsfunction,andtreatmentsaimleavethisuntouched.Ifnodulesdo scarringofthevocalfold.Theunderlyinglamina propriaof radiotherapy wouldresultinfurther injectionor phonation. Speechtherapyisthepreferredtreatment,assurgicalexcision, during junction oftheanteriorandmiddlethirds.Thisispointmaximumconvexity Vocal arefibrousthickeningsofthevocalfoldat nodulesresultfromvocalabuse.They point islikelytoresultinpathologicalchangesthedelicatevocalfold. theglottis,andscarringatthis tothetrachealiesimmediatelybelow stenosis. Thepointofentry Its useiscontroversialanditmaybeassociatedwithlong-termvoicechangesubglottic accesstotheairway. isrequired.Itcanbeusedforemergency quickly andlittlesurgicalexpertise Itcanbeperformed cricothyroid membranefor1cmimmediatelyabovethecricoidcartilage. skinincisionandinvolvesdivision ofthe throughavertical Cricothyroidotomy isperformed feeding tubesmustnotbeplacedinthetrachea. phonation.Tracheostomy thusallowing the larynx withfeeding,however tubescaninterfere itupthrough airfromleavingthetube,diverting exhaled tube’s position.Speakingvalvesprevent or uncuffed,andhaveanouterflangetoconnecttapeswhichpassaroundthenecksecure empty. Thismakescleaningthetubeeasierfornursingstaffandpatient.Tubes canbecuffed 337

45: PHARYNX, LARYNX AND NECK HEAD AND NECK ➜ ➜ 28. C 27. A 26. D 25. E 24. B 23. D 338

Trauma totheneck The neck facilitating speechalso. airupthroughtheneopharynx pulmonary as speechormodernvalvescanbeusedtodivert abscesses will fail to resolve, at which point complete surgical excision isthetreatment ofchoice. abscesses willfail to resolve,atwhichpointcomplete surgicalexcision shouldbecommenced.Occasionally disease,ifcoexistent, management ofrenal orpulmonary drainage shouldbeavoided,asitwill alsoresultinadischargingsinus.Appropriateantibioticsand theclassicsignsofabacterialabscess (hencethetermcoldabscess)andideallyincision show collar-stud abscess.Eventuallytheabscess willburstandformadischargingsinus.Patientsnot fascia.Hereit spreadsoutwithinthisspaceandisreferredtoasa thesuperficial up towards inthesenodesand,withouttreatment, puscanerodethedeepfascialplanestopenetrate develop Tuberculous lymph nodes.Acollectioncan adenitistendstoaffecttheupperdeepcervical should notbeconfusedwithVincent’sangina,aninfectivegingivostomatitis. mylohyoid todecompressthefloorofmouth.Tracheostomy should beconsidered.Thiscondition surgical decompressionofthesubmandibulartrianglesmayberequired withdivisionofthe polymicrobial andrequiresbroad-spectrumantibiotics.Ifswellingcontinues despitetreatment, thetongue,whichcompromisesairway.floor ofthemouthelevates Theinfectionisoften Ludwig’s anginaisaninfectiveconditionaffectingthesubmandibularspace. Oedemaofthe stentsmaybeleftinplaceforaround5daysandremoved endoscopically.Laryngeal fractures. lacerationsandsuturerepairofdisplacedcartilage open repairofsignificantmucosal tracheostomyshouldbeconsideredwith tube whichcancausemassivepermanentfibrosis.Low willresultinaforeignbodyreactiontothe airway. suchaninjury Long-termintubationfollowing haematomaandswellingcanleadtoarapidlossofthe blunttraumatothelarynx, Following ofacufftonguebasetissue. removalofthehyoid,asdescribed,andexcision cyst, involvesamidlineneckdissectiontoincludeallbranchesofthe ofthesecysts recurrence. Excision ofthemiddlebodyhyoidtoprevent bone, sosurgicaltreatmentrequiresexcision isformed.Thepathofthethyroidhooksaroundhyoid ductcyst to involute,athyroglossal tractfails of thetonguetoitspositioninfronttrachea.Ifaremnantthisthyroglossal The thyroiddescendsfromtheforamencaecumatjunctionofposteriorandmiddlethirds diseasepresentsachallengeirrespectiveofthetreatmentmodality. multicystic extensive, however, orsclerotherapy; which isanindicationfortreatment.Definitivetreatmentinvolvesexcision withtheairway, translucent. Cystichygromasoftheneckbehaveunpredictablyandcaninterfere with labour. Theneckisacommonsiteandthesesoftcompressiblemassesare brilliantly interfere diagnosed intheprenatalperiodonultrasoundscanning–andwhenmassivecaneven Cystic hygroma,amalformationoflymphatics,presentsinearlyinfancy. Itcansometimesbe postponed untiltheinfectionresolves. ismademorechallengingandtheprocedureshouldbe becomeinfected,surgicalexcision If they containthickturbidfluidcontainingcholesterolcrystals. thirds ofthesternomastoidmuscle.They vary. presentinyoungadults,withaneckmassatthejunctionofupperandmiddle They arethoughttorepresentaremnantofthesecondbranchialcleft,althoughopinions Branchial cysts ➜ ➜ ➜ 1I 5D 4H 3E 2A 1G matchingquestions Answers: Extended 30. E 29. B

2. Investigationandmanagement oftheacuteairwa 1. Investigationofthephar Primar Carotid bodytumoursorchemodectomasaremorecommonataltitudeasthechronichypoxia Carotid ideally nursed in a high-dependency area. If they fail to respond to antibiotics, a short course of failtorespond toantibiotics,ashort area.Ifthey ideally nursedina high-dependency be enlargedsufficientlytothreaten the airway. Thesepatientsshouldbemonitoredcarefully,and filmoverthetonsils.Thetonsilscan virus cancausetonsillitis,oftenpresenting withagrey/white Epstein–Barr abovethelarynx. obstruction atalevel isthesoundmadebyupper airway Stertor including theoesophagusandthyroid,shouldalsobeconsidered. at anypointalongitsroute.Bronchialcarcinomashouldberuledout, but malignancieselsewhere, onthelefthasalongroutethroughchestandcanbedamaged nerve The recurrentlaryngeal staging ofmalignancy. forahistologicaldiagnosisandaccurate underanaestheticwithbiopsiesallows Examination Persistent hoarsenessandvocalcordchangesinasmoker aresuspiciousofmalignancy. disease anditsrelationshipwiththecarotidvessels. scansorangiograms willdemonstratethe bodytumoursshouldnotbebiopsied.MRI Carotid origin. fromanunknown scanning intheinvestigationofpatientswithmetastaticSCC roleforPET underanaesthetic.Thereisagrowing andexamination be scanningwithMRI/CT steps would andcancompromiseprognosis.Havingmadeadiagnosis,thenext malignancy Openbiopsyofthenoderisksseeding ThefirstinvestigationisFNA. This issuspiciousofSCC. Management optionsincludeendoscopicoropenprocedures. andtherefluxofpouchcontents. dysphagia, aspiration,gurglingnoisesonswallowing pouch,whichcanbeseenonbariumswallow.The diagnosisispharyngeal Symptoms include vein andmuscle.Subtotalisnotatermappliedtoneckdissection. fromthenerve, dissections canbeclassed1–3dependingonthenumberofstructurespreserved accessory. Modifiedneck Selectiveneckdissectionsremoveonlylymphatics,butnotalllevels. The classical,radicalneckdissectioninvolvedremovalofallnodes,thesternomastoid,jugularand surgical removalshouldbecarefullyconsidered. andbenignsotheneedfor compromise.Thesetumoursareslow-growing lead tofatalairway space,intraoralbiopsyisalsocontraindicatedasahaematomacan presents intheparapharyngeal whichiscontraindicated.Ifthemass notFNA, Investigationiswithangiogram/MRI, up anddown. ofthecarotidbifurcationcanbemovedsidetobutnot massatthelevel The classicpulsatile arenothormonallyactivethemselves. they tumours areassociatedwithphaeochromocytomas, leads tocarotidbodyhyperplasia.Only10 andalthoughthese percenthaveafamilyhistory, y tumoursoftheneck y nx, lar y nx andneck y case 339

45: PHARYNX, LARYNX AND NECK HEAD AND NECK 4C 3B 2G 5H 340 the aim is to maintain the normal function of the larynx asfarpossible. the aimistomaintainnormalfunctionoflarynx and The underlyingtissuesmustnotbedamagedasthisconditionoftenimprovesafterpuberty endoscopicallyusingcoldsteelorlaser.histological confirmation,thelesionsshouldbeexcised abiopsyfor papillomata atsitesofepithelialtransitionsuchasthevocalfolds.Following 6and11. papillomatosisisaconditioncausedbyHPV Recurrent respiratory Itresultsin function. injuriesinanattempttominimiselong-termlossof andcartilage andrepairofmucosal the larynx damage.Anearlytracheostomyisappropriate,withinspection of result inpermanentlaryngeal required. Endotrachealintubationshouldbeavoidedastheforeignbodyreactiontotubecan maybe andpromptintervention traumacanresultinlossoftheairway, Blunt laryngeal until theiroralintakeissufficient. incise theabscesswall.Thesepatientswillalsobemanagedwithantibiotics,fluidsandanalgesia ispreparedwithalocalanaestheticsprayandscalpelbladeusedto the mouth.Themucosa This patienthasaperitonsillarabscesswhichcanbetreatedwithincisionanddrainagethrough paediatric tracheostomyifrequired. anaesthetistandasurgeoncapableofperforming under anaestheticwithanexperienced arrest.Thepatientshouldbetransferredtotheatreandexamined which canprecipitaterespiratory thethroat noattemptstoexamine environment, withnothingdonetoupsetthem,particularly This childshouldbeconsideredtohaveepiglottitis.Thesepatientsmanagedinacalm urgent tonsillectomy. continuestodeteriorateshouldbeconsideredfor steroids shouldbegiven.Patientswhoseairway 46 Oropharyngeal cancer

Multiple choice questions In the following, choose the single best answer. 5. The term ‘field change’ in relation to oral cancer refers to what? ➜ Anatomy and pathology A The wide field radiotherapy techniques 1. The posterior border of the oral cavity which must be applied to the oral is: mucosa in head and neck cancer patients. A The base of the vallecula B The demarcated mucosal changes B The posterior tonsillar pillar seen in patients with oral cancer C The level of the circumvallate papillae of which resembles the appearance of the tongue agricultural fields. D The posterior pharyngeal wall C The effect upon the surrounding oral E The lingual surface of the epiglottis. mucosa conferred by long-standing dental caries. 2. Which of the following structures is D The widespread damage of epithelium not part of the oropharynx? leading to mucosal changes and a high A The hard palate incidence of separate tumours. B The tonsillar fossa E The requirement to rotate the C The superior surface of the hyoid radiotherapy field during a course of bone treatment to maximise tissue-sparing D The base of tongue effects. E The posterior pharyngeal wall. 6. What does the term ‘skip metastasis’ 3. What is the most common refer to? malignancy encountered in the A The tendency of oral cancers to oropharynx? metastasise to the liver without cervical A Adenocarcinoma nodal metastasis. B Squamous cell carcinoma (SCC ) B The tendency of oral cancers to C Adenoid cystic carcinoma metastasise to bone without cervical D Non-Hodgkin’s lymphoma nodal metastasis. E Salivary gland tumours. C The tendency of oral cancers to 4. Which of the following conditions metastasise to lower-echelon cervical is not associated with malignant lymph nodes without involving the transformation? higher echelons. A Erythroplakia D The tendency of oral cancers to B Chronic hyperplastic candidiasis metastasise to the higher-echelon C Sideropenic dysphagia (Paterson–Kelly cervical lymph nodes then on to the syndrome) lower level nodes. D Angular stomatitis E The tendency of oral cancers to E Oral lichen planus. metastasise the contralateral cervical nodes.

341 HEAD AND NECK ➜ 7. Which of the following statementsis Whichofthefollowing 7. 10. Which of the following istruewith Whichofthefollowing 10. istrueof Which ofthefollowing 9. isnota Whichofthefollowing 8. 342

orophar investigation andstagingof Clinical features, D C B A E D C B A E D C B A E D C B A CT, ittendstobe usedforconvenience. CT, ismorewidelyavailablethan MRI As soft-tissue infiltrationthanCT. of forbettervisualisation allows MRI of CT. that nodal metastasisgreatlyexceeds fordetecting The sensitivityofMRI usefulness ofMRI. fromdentalamalgamlimitsthe Artefact cancer? oropharyngeal regard totheinvestigationoforal/ seldom required. is biopsy forhistology malignancy, isthemostcommon SCC As treatment. for conservative Early presentationofmostcasesallows biopsy withoutanaesthetic. and forexamination cancer allows The painlessnatureoforopharyngeal disease occult. and tonguebasecanrenderearly natureofthetonsil The foldedmucosal required. underanaestheticisrarely examination means Ease ofofficeexamination cancers? oropharyngeal Unilateral tonsillarenlargement. Trismus toothmobility Unexplained Painless neckmass Mouth ulceration>4weeks feature oforal cancer? direction. Disease tendstospreadinavertical Lymph nodemetastasisoccursearly. Vfirst. level Lymph nodemetastasistendstoinvolve tendtoaffecttheupperlip. They tendtopresentearly.They true with regard to cancers ofthelip? true withregardtocancers y ngeal cancer ➜ 12. Which of the following factors Whichofthefollowing 12. 3 Managementofpremalignant 13. statements Whichofthefollowing 11. 14. Which of the following istrueof Whichofthefollowing 14.

cancer Treatment oforophar B A E D C B A E D C B A E D C B A E D C neck cancer? treatment forapatientwithheadand should notbeconsideredintailoring It isusedinassessmentofenlarged disease. It isusedfordiagnosisofprimary of theoperator. It providesreliableresultsindependent It causesseedingofdiseasetotheskin. istrue? cancer inoropharyngeal (FNAC) cytology regarding fine-needleaspiration the appropriatechoice. is scanning duetoclaustrophobia,MRI In patientsunabletotolerateCT preserve bloodsupplytothetissues. preserve should notbeentered,soasto In totallipreconstruction,theneck with distantflaps. liprequirereconstruction of thelower Defects ofbetweenoneandtwo-thirds cure rates. andradiotherapy havesimilar Surgery shave excision. Small tumourscanbemanagedwith ofthelip? treatments forcancer Radiotherapy. Surgical excision Biopsy frommorethanonesite Photographic recording Smoking cessation conditions shouldnotinclude: Social factors. The histology The stageofdisease The siteofdisease Chronological age interpretation. for It requiresadequatefixation lymphnodes. cervical y ngeal ➜ 18. Which of the following statements Whichofthefollowing 18. 7 Inthemanagementoffieldchange, 17. What effectdoesthedegreeof 16. Whateffectdoesthepresenceof 15.

Reconstructive techniques A E D C B A E D C B A E D C B A E nerve leadstoa compromisedswallow.nerve Unopposed action ofonehypoglossal Treatment iscontraindicated. to radiotherapy. arepreferred Multiple surgicalexcisions neck dissection. surgically inacombinedprocedurewith Lesions inthemouthshouldbetreated recognised treatment. Platinum-based chemotherapyisa side-effects. treatmentwithoutsevere superficial forwidespread allows Radiotherapy true? statementsis which ofthefollowing directly effectmanagement. The degreeofdifferentiationdoesnot used. thedoseofradiotherapy the lower The higherthedegreeofdifferentiation, used. the higherdoseofradiotherapy The higherthedegreeofdifferentiation, the higherstageofdisease. The higherthedegreeofdifferentiation, thestageofdisease. the lower The higherthedegreeofdifferentiation, management? haveonoverall and neckcancer differentiation ofSCC inhead Improved overallsurvival. No effectonsurvival percentdecreaseinsurvival 50 30 percentdecreaseinsurvival 10 percentdecreaseinsurvival in headandneckcancer? metastasis haveonprognosis cervical layer.important isthemost surface of themucosal In closureofliplesions,theapposition reconstruction isfalse? about theprinciplesoforopharyngeal 20. Which of the following techniquesis Which ofthefollowing 20. statements Which ofthefollowing 19. 22. The compositeforearm freeflapis 22. Whicharterialpedicleisharvested 21. E D C B A E D C B A E D C B E D C B A E D C B A Due to the excellent bloodsupplyof Due totheexcellent of mouthdefectsistrue? about reconstructionofanteriorfloor reconstruction flapisbeneficial. totalglossectomy,athick Following provide athinflapwithgoodresults. forearmfreeflapscanbeusedto Radial intention. may belefttohealbysecondary Small defectsofthelateraltongue be closedprimarily. Small defectsofthelateraltonguemay The profundabrachii artery. The brachialartery The lateraldecubitusartery The ulnarartery The radialartery based uponwhicharterialpedicle? artery. The internalmammary iliacartery The deepcircumflex femoralartery The circumflex The deepinferiorepigastricartery The inguinalartery flap? to supplyarectusabdominusfree Free flapreconstruction. Local flapreconstruction intention Closure bytertiary intention Closure bysecondary intention Closure byprimary tissue reconstruction? not partofastepwiseapproachto required. Treatment oftheneckisunlikelytobe delayed bonyreconstruction. anterior mandibleshouldbetreatedby ofthe Defects whichrequireexcision the onlyoptionavailable. Microvascular freeflapreconstructionis mucosa. tothelabial ventral tongueforward It isunacceptabletoadvancethe reconstruction. this area,defectsrarelyrequireformal 343

46: OROPHARYNGEAL CANCER HEAD AND NECK ➜ 23. Which of the following statements Which ofthefollowing 23. 25. Which of the following statementsis Whichofthefollowing 25. statementsis Which ofthefollowing 24. 344

disease Management ofneck D C B A E D C B A E D C B A metastasis. ifthere isariskofoccult performed neckdissectionshouldbe Radical effective assurgery. Treatment withradiotherapy isequally has notmetastasised. metastases confirmsthatthedisease radiologically obviouslymphnode The absenceofbothpalpableand metastasised. nodes confirmsthatthediseasehasnot lymph The absenceofpalpablecervical neck? node-negative true intheclinically volume disease. Chemoradiotherapy iseffectiveforlarge radiotherapy. the additionofchemotherapyto Frail patientsmaynottolerate oforganfunction. preservation Using chemoradiotherapyensures survival. toimprove base hasnotbeenshown radiotherapy forlesionsofthetongue The additionofchemotherapyto morbidity versusradiotherapyalone. radiotherapy causesnoincreasein The additionofchemotherapyto true? appropriate. andneck,maybe to treattheprimary bychemoradiotherapy followed surgery, Treatment oftheneckalonewith preferred treatment aloneisthe disease surgicalexcision In patientswithlargevolumeneck for tumoursinthisregion. Chemoradiotherapy iscontraindicated metastasesarerare. Cervical treatment. lesions inthisareaamenabletosurgical Transoral accessmakesmostprimary cancer istrue? cancer about managementoforopharyngeal ➜ 27. Which of the following isnota Whichofthefollowing 27. 28. Which of the following isnota Which ofthefollowing 28. statements Whichofthefollowing 26. 29. Which of the following statements Which ofthefollowing 29.

outcomes oftreatment Complications and E D C B A E D C B A E D C B A E E D C B A resection? oforopharyngeal complication All oftheabove. N4 diseaseinvolvescriticalstructures. disease. N3 diseasemustincludebilateral N2 diseasecanbeunilateralorbilateral. thesize. node whatever N1 diseasedescribesonepalpable neck? node-positive is trueintheclinically treatment. influence further planned neckdissectionwillnot The histologicalfindingsfollowing lived. treatmentisshort- Morbidity following carephysician. primary isbestcoordinated bythe Follow-up years post-treatment. Most recurrencesoccurataround5 care. likely tobemanagedbestwithpalliative is majorsurgery Recurrence following uncommon. psychologicaldisturbancesare severe, Although physicalimpairmentmaybe cancer? treatment fororopharyngeal is trueforpatientswhocomplete Renal toxicity. Osteoradionecrosis ofthemandible Visual impairment Trismus Atherosclerosis ofthecarotidartery and neckcancer? ofradiotherapycomplication forhead palsy. nerve Accessory Rupture ofcarotidartery Soft-tissue oedema Thoracic ductinjury Xerostomia ➜ ➜ Extended matchingquestions Extended cancer? factorindeterminingprognosisoropharyngeal What isthemostsignificant 30. Choose the mostappropriateresponseforquestionsbelow: termwiththedescriptionsgivenbelow: Matchthemostappropriateanatomical

5 4 3 2 1 2. Treatment andoutcomesinorophar 5 4 3 2 1 1. Anatom J I H G F E D C B A J I H G F E D C B A E D C B A Which factor will require planned surgery andpostoperativeradiotherapy? Which factorwillrequire plannedsurgery overradiotherapy? Which factorshouldencouragesurgery Which factorhasthebiggestimpact onprognosis? theadditionof chemotherapytotreatment? Which factormayprevent Which factorwillnecessitatepostoperative radiotherapyintheclinicallynode-negativeneck? Line ofthecircumvallatepapillae The posteriorborderoftheoropharynx The inferiorborderoftheoropharynx The posterosuperiorborderoftheoralcavity The anteriorborderoftheoralcavity Palsy of accessory nerve Palsy ofaccessory neckdisease Bulky nodalmetastasis Presence ofcervical spread Presence ofextracapsular Degree ofhistologicaldifferentiation fieldchange Presence ofextensive lesion Size ofprimary Smoking statusofpatient Social classofthepatient Physiological ageofthepatient Posterior tonsillarpillar Anterior tonsillarpillar ofthehyoidbone Upper surface wall Posterior pharyngeal oftheepiglottis Lingual surface Junction ofthehardandsoftpalate Junction ofanterior2/3andposterior1/3 ofthetongue Ventral ofthetongue surface ofthelip Buccal surface Vermillion border Smoking statusofthepatient. Concomitant medicalconditions Presence ofmetastasis Patient age Tumour size y y ngeal cancer 345

46: OROPHARYNGEAL CANCER HEAD AND NECK ➜ ➜ 8. E 7. A 6. C 5. D 4. D 3. B 2. A 1. C Answers: Multiplechoicequestions 346

orophar Clinical features, investigation andstagingof Anatom patients and the fact that denture-wearers may be used to discomfort andulcerationwithin the patients andthefact thatdenture-wearersmaybeused todiscomfort metastasis halvesapatient’sprognosis. Delaysmaybeduetotheelderlyandfrailnatureofthese Late presentationofheadandneck cancerresultsinpooroutcome.Thepresenceofacervical surface. tends tospreadlaterallyoverthemucosal Thedisease IandII. be moreaggressive.Metastasisoccurs lateandtendstobenodesinlevels lipisaffectedmorecommonlythantheupperlip.Diseaseon upperliptendsto The lower ofthelipisunusualincomparisonwithotheroralcancersas ittendstopresentearly.Cancer uncommon. andIVaswellthecontralateralneck.Distant metastasesare III II, metastasise tolevels nodescommonly Oropharyngeal andIV)withoutinvolvingtheupperechelons(III). III (levels lymph nodegroups tongue canpresentwithskipmetastasisanddiseaseinthelower-echelon oftheoral SCC Head andneckcancerstendtometastasiseinapredictablemanner,however metachronous lesion.Therateofsynchronoustumoursis4percent. mostcommonlya asecondprimary, field.Fifteenpercentofpatientswilldevelop the mucosal and alcohol.Thisso-called‘cancerisation’ effectresultsinahighrateofseparatetumourswithin totobacco causedbychronicexposure Field changereferstothewidespreaddamagemucosa Oral lichenplanushasaweakassociationwithmalignanttransformation. result inmalignanttransformation,andthesepatientsanimmunologicaldefectmaycoexist. describesalesionthatisbrightredplaque.Chronichyperplasticcandidiasiscan Erythroplakia inanareaofclinicallyobviouspremalignantdisease. Most oralcarcinomasdonotdevelop cancer.oropharyngeal tobaccoontheIndiansubcontinenthasledtohighratesoforalcavityand practice ofchewing methods oftobaccoconsumptionaccountforgeographicdifferencesindiseasedistribution.The a strongassociationwithtobaccoconsumptionandsynergisticeffectalcohol.Different Thereis inheadandneckmucosa. Squamous cellcarcinomaisthemostcommonmalignancy ofthehyoidisincluded. ofthehardpalatetolevel the level wallfrom to thesuperiorborderofhyoidinvallecula.Posteriorly, theposteriorpharyngeal toitsattachment laterallyandthebaseoftonguedown includesthetonsillarfossae oropharynx liesposteriortotheoralcavity,sohardpalateisoutsideitsboundaries.The The oropharynx anterior two-thirdsandtheposteriorthirdoftongue. superiorly andthecircumvallatepapillaeontongue.Thislineliesatjunctionbetween ofthesoftpalate The oralcavityrunsfromtheskin-vermillionborderonlipstolevel y y ngeal cancer andpatholog y 11. 11. D 10. C 9. B ➜ 15. C 14. B 13. E 12. A

Treatment oforophar can significantlyimproveyield. Recentworksuggeststhatultrasoundguidance required. SeedingisnotacomplicationofFNA. is isavailableimmediately,nofixative Results areoperator-dependent,andifthecytologist whereopenbiopsythroughthemouthispreferred. and neck.Itisnotusedfortheprimary, diseaseinthehead hasbecomearoutineinvestigationofcervical Fine-needle aspirationcytology metastases arecomparable. fordetectingcervical andCT CurrentlythesensitivityandspecificityofMRI by dentalartefact. available andlessclaustrophobicbutitprovidessoft-tissuedefinitioncanbeaffected andoralcavity. ismorewidely assessment oftissueinvasionfordiseasestheoropharynx CT superior soft-tissuedefinition,whichallows Magnetic resonanceimaginggivesexcellent cases. shouldbesoughtinall diseaseoninspection.Definitivehistology oftheprimary without evidence metastasis canceroftenpresentslate,sometimesasacervical almost allpatients.Oropharyngeal tenderforthepatientandrequiresgeneralanaestheticin exquisitely andeven uncomfortable mustincludepalpation.Thisisoften fool theexaminer. For thisreasoncompleteexamination foldswithintheseregionsand Diseasecanhideinthemucosal to seeonclinicalexamination. maybeimpossible diseaseinvolvingthetonsil,tonguebaseandnasopharynx Early mucosal nottheoralcavity. whichiswithintheoropharynx tonsillar fossa, mouth. Trismus muscles.Thepalatinetonsilslieinthe frominvolvementofthepterygoid develops Treatment of theneckshouldbeconsideredboth whenthereisclinicallypalpable metastatic rates. percentreductioninsurvival metastasis inheadandneckcancercarries a50 Cervical apoorcosmeticresult. metastasis. Inclosingthelip,vermillion borderisvitaltoprevent carotid. Neckdissectionislikelytobe requiredforsuchpatientswhoareathighriskofcervical are usedwhichrequireaccesstothe necktoanastomosevesselsthebranchesofexternal lipcanbeclosedwith localflaps.Intotalreconstructionofthelip,distantflaps thirds ofthelower closedprimarily. ofonetotwo- lesions canbemanagedsurgicallywithV-wedgeexcisions Excision percent forcancerofthelip.Small andradiotherapyoffercureratesofaround90 Both surgery malignancy. fortrue or vaporisedwithlaserbutnottreatedradiotherapy,whichshould bereserved site, amoreaccuratepicturecanbebuiltupofdiseaseprogression.Lesions canbeexcised are valuableandmoreaccuratethanclinicaldiagrams.Bytakingbiopsies frommorethanone premalignant conditions.Smokingisthemostcommonaetiologicalfactor. Photographicrecords isthebasisofmanaging Removing theaetiologicalfactorsassociatedwithmalignancy and chemotherapy. radiotherapy thanchronologicalage.Thetreatmentoptionsincludesurgery, age ismoreimportant smokersandmaybealcoholicshencephysiological and neckcancerpatientswillbeheavy onpatientswithsignificantcomorbidities.Head tobeperformed majorsurgery techniques allow Many factorsmustbeconsideredbeforedesigningatreatmentplan.Modernanaesthetic y ngeal cancer 347

46: OROPHARYNGEAL CANCER HEAD AND NECK 17. D 16. E ➜ ➜ 23. E 22. 21.A B 20. C 19. B 18. A 348

Management ofneckdisease Reconstructive techniques neck treated,ifindicated. andthe isdiscovered,thestagingprocessshouldbeundertaken amalignancy formal histology, the not bewarranted,asthediseasewouldhavespreadbydefinition.If,havingreviewed neckdissection would recognised treatmentfornon-malignantdisease.Incasesofpremalignancy, and makeinterpretationoffuturebiopsiesfromthatsitelessreliable.Chemotherapyisnota morbidity wouldrequiretreatmentofawidefield,whichresultinsevere Radiotherapy offieldchangeormultipleprecancerouslesionsshouldbetreatedwithsurgicalexcision. Cases differentiation increases,suchindicatorsmaybeincorporatedintostagingsystems. our understandingoftheimpactperivascularandperineuralinvasionaswellhistological the histologicaldifferentiationandhavelittleimpactontreatmentatpresent.Itislikelythat,as donotconsider andUSA of manycancergroups.ThecurrentstagingsystemsusedintheUK oftheminimumdataset Differentiation isoftencommenteduponbypathologists,andformspart and liplesions. nodesshouldbetreatedinallcasesotherthanearlyglottis In practicethismeansthatthecervical thantheimpactofpotentialdisease. themorbidityoftreatmentislower 20 percent.Atthislevel disease andwhenthechanceofmicrometastases(whichareimpalpable)isgreaterthan procedures and resections. Cervical metastasesare common.Chemoradiotherapyisa recognised procedures andresections. Cervical access isadifficultsitetoaccesssurgically.The oropharynx Smalllesionscannecessitate extensive fitness ofthepatient. technique willdependonthetissueremoved,tissuesremaining, functionalaimsandthe reliable freeflaps.Thechoiceof usedpriortotheadventofproceduresharvest extensively intention,somemaybeclosed primarily.lesions maybelefttohealbysecondary Localflapswere shouldbe approachedlogically.Reconstruction ofadefectintheoralcavityororopharynx Some be required. There isarich,oftenbilateral,lymphaticsupplytothisareaandtreatment oftheneckislikelyto functionalandcosmetic defects. severe resected, immediatereconstructionisrequiredtoprevent unacceptable. Optionsincludelocalandfreeflapreconstruction.Ifthe anteriormandibleis morbidity,whichisconsidered leadstosignificantspeechandswallowing the labialmucosa Most floor-of-mouthtumoursrequireformalreconstruction.Advancingtheventraltongueto contours ofthedefect. mouldtothe forearmfreeflapscanbeleftthinandpliablesothey to replacethetissue.Radial flapsuchasarectusabdominusshouldbechosen created. Ifthereismuchtissueloss,abulky intention.Free flapsshouldbechosendependingonthedefect or lefttohealbysecondary rehabilitation.Lateraltonguelesionsmaybeclosedprimarily tohelpswallow should bepreserved lead toalossoffunctionthetongueandsubsequentmorbidity. Ifpossibleatleastonenerve will nerves bothhypoglossal Sacrificing resectionhasanimpactonswallowing. Oropharyngeal 26. B 25. C 24. D ➜ 29. B 28. E 27. A

Complications andoutcomesoftreatment • • • • considered. orperineuralinvasion,postoperativeradiotherapyshouldbe withextracapsular malignancy shows advantageovermodifiedradicalneckdissection.Ifthehistology neck dissectionconfersasurvival thatradical areequallyeffective.Thereisnoevidence negative neck,radiotherapyandsurgery chance ofametastasisis20percentorgreater,treatmentshouldbeadvised.Intheclinically reason thattreatmentoftheneckshouldbeconsidered.Currentguidelinessuggestif Becauseofthis,micrometastaseswillalwaysbemissed.Itisforthis percentforCT). and 80–90 CT. Bothofthesemodalitieshaveasensitivityandspecificity(around70 percentforpalpation radiologicalassessment,oftenby A neckcanbedescribedasnegativeonpalpation,orfollowing chemoradiotherapy islesseffectiveinthesecases. can leadtolossoffunction.Treatment iffeasible,as oflarge-volumediseasestillrequiressurgery, there arepost-treatmentchanges,includingfibrosisandreducedvascularsupply. Thesechanges frail patientsmaynottoleratetheregime.Althoughchemoradiotherapyavoidssurgicaltreatment, whichiswhy chemotherapy toradiotherapysignificantlyincreasesbothmorbidityandmortality, forlesionsofthetonguebase.Theaddition for chemoradiotherapyoverradicalsurgery benefitsbeingdemonstrated The roleofchemotherapyisbecomingclearer,withsurvival radiotherapy shouldbeused. local recurrenceishighinpatientswithlargevolumeneckdisease(N2/N3),andpostoperative This approachcanbeusedwithsurgicaltreatmentoftheneckinselectedcases.Thechance theneedforglossectomyorlaryngectomy. designedtoprevent strategy, an organpreservation treatment fordiseaseinthisarea,asitisotherheadandneckcancers.Itmaybeused Pre- andpost-treatment counsellingmayminimisethe effects,butmanyofthesechanges will psychologicaldisturbance. Coupled withcosmetic deformity,thesechangescan resultinsevere andspeaknormallyhasa tremendouseffectonpatients'lifestyle. The lossoftheability toswallow Treatment forheadandneck cancercanbebothphysicallydisfiguringandfunctionallydisabling. oftreatment. part removal ofinfectedteethinthefield, andprovidingawell-vascularisedcovertothemandibleas Osteoradionecrosisisadifficultproblemwhichbestavoidedby inthecarotidartery. develop canaffectvision, andatherosclerosiscan results intrismus,damagetothelensoropticnerve structures withinaradiotherapyfieldareaffectedbyit.Fibrosisofthemuscles ofmastication Renal toxicityisacomplicationofchemotherapywhichmaybeadded to radiotherapy. All withasignificantriseinintracranialpressure. severe swellingis bilateralneckdissection.Ifbothinternaljugularveinsmustbesacrificed, following bad and is particularly surgery glands.Soft-tissueoedemaoccursfollowing involved thesalivary ofthemouthassociatedwithradiotherapyto headandneckthat Xerostomia isdryness N3 –anynode>6cm. nodes <6 cm N2 –singleipsilateralnode3–6cm/ipsilateralmultiplenodes<6 cm/bilateral orcontralateral N1 –singlelymphnodemetastasis<3cm N0 –noregionalnodalmetastasis 349

46: OROPHARYNGEAL CANCER HEAD AND NECK 30. C ➜ ➜ 5I 4E 3H 2A 1G 1A,2E,3H,4G,5D matchingquestions Answers: Extended 350

2. Treatment andoutcomesinorophar 1. Anatom have animpactonthepatient,theirresponsetotreatmentandsubsequentoutcome. The presenceofnodalmetastasisisthemostsignificantprognosticfactor,althoughallotherswill fromothermembersoftheteam. support butmayrequireongoing years manypatientscanbedischargedfromoncologicalsurveillance, suitable forcurativetreatmentandwilloftenbebestmanagedinapalliativecaresetting.At5 amajorresectionareunlikelytobe 2 years.Patientswhopresentwithrecurrencefollowing Most recurrencespresentwithinthefirst12 months(70 percentwithinthefirst percent)and90 oftheserareconditions. carephysiciansareunlikelytohavegreatexperience treatment, asprimary isgenerallybestprovidedbytheteamwhocoordinated be long-termorpermanent.Support operative radiotherapyshouldbeconsidered. neckdiseaserespondspoorlytosinglemodalitytreatmentanddissection withpost Bulky withinterpretationoffuturebiopsiesifrequired. changes canalsointerfere morbidity.radiotherapy involvesirradiatingalargeareawithconsequentsevere Theradiotherapy ifpossible,as Patients withfieldchangeormultipletumoursshouldbetreatedsurgery metastaseshalvestheprognosis. The presenceofcervical contraindicated infrailpatients. oftreatmentandsoitmaybe Chemotherapy addssignificantlytothemorbidityandmortality node-negative neck,postoperativeradiotherapyisusedtoreducethechanceoflocalrecurrence. astagingneckdissectionfortheclinically spreadfollowing demonstratesextracapsular If histology y y ngeal cancer 47 Disorders of the salivary glands

Multiple choice questions In the following, choose the single best answer. B A benign salivary mass involving the parotid and submandibular glands ➜ Anatomy, physiology and C A mucous retention cyst originating pathology of the salivary from the sublingual glands, limited by glands the mylohyoid muscle A mucous retention cyst originating 1. The minor salivary glands contribute D from the submandibular and sublingual what percentage of the total salivary glands which perforates the mylohyoid volume? muscle to enter the neck A 10 per cent A midline neck mass which moves on B 20 per cent E tongue protrusion. C 30 per cent D 40 per cent ➜ E 50 per cent. The submandibular glands 5. Which of the following structures 2. What percentage of minor salivary is not an anatomical relation to the gland tumours are malignant? submandibular salivary gland? A 50 per cent A The anterior facial vein B 60 per cent B The facial artery C 70 per cent C The inferior alveolar nerve D 80 per cent D The lingual nerve E 90 per cent. E The hypoglossal nerve. 3. Which statement most accurately 6. Which structure marks the posterior describes the anatomy of the boundary of the submandibular duct sublingual glands? which can safely be accessed via an A They drain through the sublingual duct intraoral approach? which opens into the floor of mouth. A The third molar tooth B They drain either directly on to the floor B The body of the submandibular duct of mouth or into the submandibular duct. C The lingual nerve C They consist of two lobes separated by D The posterior edge of the mylohyoid the mylohyoid muscle. E The marginal mandibular nerve D They are embedded in the intrinsic muscles of the ventral surface of the 7. Which statement best describes tongue. placement of the incision used for E They lie in the space between the submandibular gland excision? mandible and the two bellies of digastric. A Directly over the palpable position of the gland with the neck extended 4. The term plunging ranula refers to B Between the superior limit of the gland which clinical entity? and the mandible A A malignant congenital salivary mass C At the lower limit of the gland arising from the submandibular gland D 4 cm below the mandible

351 HEAD AND NECK ➜ 13. Which of the following structures Whichofthefollowing 13. 2 Whichisthemostappropriateform 12. isnota Whichofthefollowing 11. Whatpercentageofsubmandibular 10. isnota Whichofthefollowing 9. Whichstructureattachesthedeep 8. 352

The parotid gland C B A E D C B A E D C B A E D C B A E D C B A E D C B A E The glossopharyngeal nerve The glossopharyngeal Terminal carotid branches oftheexternal The facialnerve Biopsy iscontraindicated. (FNAC) Fine-needle aspirationcytology Frozen sectionduringformalexcision washings Salivary histology Open surgicalbiopsytoallow tumour? gland of biopsyforamajorsalivary consistency. Rubbery nodeenlargement Cervical Induration oftheoverlyingskin enlargement Rapid weakness Facial nerve malignancy? feature ofsalivary percent. 60 percent 50 percent 40 30 percent 20 percent tumours aremalignant? Paralysis oftheipsilateraltongue. Anaesthesia ofsubmentalskin Weakness ofthecornermouth Anaesthesia oftheipsilateraltongue syndrome Frey’s excision? ofsubmandibulargland complication The mylohyoid. The tendonofdigastric fascia The deepcervical The submandibularganglion nerve The hypoglossal the lingualnerve? lobe ofthesubmandibularglandto ofthegland. the level Parallel tothesternomastoidmuscleat does notlieintheparotidgland? 17. In a 4-year-old with recurrent bilateral a4-year-oldwithrecurrentbilateral In 17. bacteriais Whichofthefollowing 16. conditionsis Whichofthefollowing 15. complications Whichofthefollowing 14. 19. What percentage of salivary stones Whatpercentageofsalivary 19. Chronicparotitisinchildrenis 18. D C B A E D C B A E D C B A E D C B A E D E D C B A E D C B A E Dental abscess sialadenitis? associated withascendingbacterial arthritis. Inflammatory Uveitis Balanitis Pancreatitis otitismedia Secretory is associatedwithmumpsinfection? Lymph nodes. The retromandibularvein 50 percent. 50 percent 40 30 percent 20 percent 10 percent occur intheparotidgland? Wegener’s granulomatosis. Tuberculous parotitis Bacterial parotitis Sialolithiasis HIV pathognomonic ofwhatdisease? parotitis. HIV-associated Recurrent parotitisofchildhood Sjögren’s syndrome ductstricture Salivary Sialolithiasis what isthemostlikelydiagnosis? parotid swellingmadeworseoneating, Atypical mycobacteria. Pseudomonas aeruginosa Streptococcus pyogenes Staphylococcus epidermidis Staphylococcus aureus sialadenitis? ofbacterial the mostcommoncause Hyperglycaemia. Otitis media Dehydration Oral thrush ➜ 22. Which nerve mustbetransacted as Which nerve 22. What isthemostcommonsitefora 20. 25. Which of the following branches Which ofthefollowing 25. isnotabranch Which ofthefollowing 24. landmarksis Which ofthefollowing 23. shouldabenigntumour How 21.

Parotid surger E D C B A E D C B A E D C B A E D C B A E D C B A E D C B A Cervical. Buccal Zygomatic Oribtal Temporal graft repair? without theneedforimmediatecable bedivided can of thefacialnerve Cervical. Buccal Zygomatic Oribtal Temporal of thefacialnerve? ofmasseter.The insertion ofdigastric The insertion earcanal cartilaginous ofthe The superior-mostportion The greaterhornofthehyoid ofsternomastoid The insertion trunk? thefacialnerve used tolocate The auriculotemporalnerve. nerve The accessory The greaterauricularnerve nerve The hypoglossal The facialnerve parotidectomy.Superficial Total parotidectomy Radiotherapy Open biopsypriortoformalexcision Enucleation managed? involving thetailofparotidbe Behind theangleofmandible. Anterior totheear mass aparapharyngeal As Inferior totheangleofmandible At theanteriorborderofmasseter part ofasuperficialparotidectomy? parotid tumour? y ➜ What isFrey’s syndromefollowing 26. 27. Which of the following statements Whichofthefollowing 27. 29. Which of the following isnot Which ofthefollowing 29. isSjögren’s With whichmalignancy 28. 30. Management strategies fordrooling 30.

Degenerative conditions E D C B A E D C B A E D C B A E D C B A E D C B A parotidectomy? syndrome? Sjögren’s is trueofsecondary gland. Hyperplasia ofthecontralateralparotid parotid bulk Cosmetic deformityduetolossof parotid bed ofasialoceleoverthe Development flow mouthduetoreductioninsalivary Dry sweating Gustatory repositioning ofparotidducts. ofsubmandibularglandsand Excision Bilateral submandibularglandexcision Submandibular ductrepositioning glands tothesalivary Radiotherapy submandibular andparotidglands Botulinium toxininjectioninto include: Cholinergic medications. totheheadandneck Radiotherapy Sjögren’s syndrome Dehydration Depression associated withxerostomia? Lymphoma. sarcoma Salivary pleomorphicadenoma ex Carcinoma carcinoma Adenoid cystic Acinic cellcarcinoma syndrome associated? Sjögren’s. is greaterthaninprimary The chanceofmalignanttransformation commonly affected. The submandibularglandsaremore disorders. It isassociatedwithconnectivetissue It ismorecommonintheelderly. It ismorecommoninmales. 353

47: DISORDERS OF THE SALIVARY GLANDS HEAD AND NECK ➜ ➜ Extended matchingquestions Extended StudyFig.47.1 andchoosethemostappropriateresponseforquestionsbelow: 354

disease 2. Investigationandmanagement ofsubmandibulargland 5 4 3 2 1 1. Investigationandmanagementofaparotid mass C B A N M L K J I H G F E D C B A Which structure will be sacrificed duringthe procedure? Which structurewillbesacrificed Which formoftreatmentwouldyourecommendtothispatient? totestpriorsurgery? important isitparticularly Which nerve What formofbiopsywouldbemostappropriate? What isthemostlikelydiagnosis? Orthopantogram Floor ofmouthX-ray Sialogram Radiotherapy Enucleation oftheparotidmass parotidectomy Superficial Marginal mandibularnerve nerve Hypoglossal Facial nerve Greater auricularnerve Lymphoma carcinoma Adenoid cystic tumour Warthin’s adenoma Pleomorphic salivary Frozen sectionbiopsy FNA Open biopsy Figure 47.1 Pleomorphic adenomaoftheleftparotid. StudyFig.47.2 andchoosethemostappropriateresponsefromchoicesbelow: 5 4 3 2 1 N M L K J I H G F E D How isductalstenosisprevented? How isatrisk? What nerve What treatmentwouldyousuggest? What symptomsisthepatientlikely topresentwith? here? What typeofX-rayisshown Stenting oftheduct closure Primary Marsupialisation Submandibular nerve Lingual nerve nerve Hypoglossal Transoral stoneexcision Submandibular glandexcision Dental infection Weight lossandotalgia Intermittent painandswelling Figure 47.2 Floor ofmouthX-ra y . 355

47: DISORDERS OF THE SALIVARY GLANDS HEAD AND NECK ➜ ➜ 5. C 4. 3. D B 2. E 1. A Answers: Multiplechoicequestions 356

Figure 47.3 branch offacialn Marginal mandibular The submandibularglands Anatom during surgery onthesubmandibulargland(seeFig.47.3).during surgery entersthemandibleandisprotected in relationtotheskinincision.Theinferioralveolarnerve onthegland,dueto its position isalsoatriskduringsurgery The marginalmandibularnerve areincloseproximity. nerves thelingualandhypoglossal the gland.Onglands’deepsurface, gravity uptothesublingualpapillainfloorofmouth.Thefacialvesselsarecloselyrelated border ofmylohyoid.Theglandisdrainedbythesubmandibularduct,whichdrainsagainst The submandibularglandconsistsoftwolobeswhichcommunicatearoundtheposterior glands. withtheipsilateralsalivary treatment consistsofexcision and seen intheneckandfloorofmouth.ThediagnosiscanbeconfirmedwithMRI intotheneck.Themasscanbe down themylohyoidmuscleandextend toperforate expands arisesfromsubmandibularandsublingualglands.It This rareformofmucousretentioncyst ortotalmaxillectomy. withpartial hard palate,treatmentinvolvesexcision areoftendiscolouredwithapink,blueorblackappearance.Iflesionsinvolvethe malignant. They glandsarehighlylikelytobe glands,tumoursoftheminorsalivary In contrasttothemajorsalivary contributearound10 sinuses.Intotalthey trachea andparanasal flow. percentofthetotalsalivary larynx, glandsdistributedthroughtheoralcavity,oropharynx, minorsalivary There arearound450 gland Submandibular salivary y , ph Anatom y siolog y ofthesubmandibular gland. y andpatholog Mandible submandibular salivarygland Incision forexcisionof Damage tothenervesdepicted y ofthesalivar Hypoglossal nerve Lingual nerve y glands ➜ 14. B 13. C 12. D 11. E 10. D 9. A 8. B 7. D 6. C

The parotid gland explored behind this level, wherecalculusdiseaseshouldbemanagedwithsubmandibulectomy behindthislevel, explored ofthesecondmolar. markedbythelevel isatriskiftheduct to thelingualnerve, Thenerve lieanterior viaanintraoralapproachifthey Stones inthesubmandibularductmaybeexcised common inadults thaninchildren. pancreatitis, sensorineuraldeafness andmeningoencephalitis.Theseconditionsareraremore infectionareorchitis,oophritis, The classicconditionsassociatedwith mumps(paramyxovirus) liesmedialtoit. nerve space,andsothe glossopharyngeal parapharyngeal fascia,whichsplitsaroundit.Itlieslateraltothe The parotidglandisencasedinthe deepcervical sections aredifficulttointerpretandnotroutinelyused. whichcanhelpinobtaininginformedconsentandplanning procedures.Frozen prior tosurgery, canbeusedtoidentifymalignantdisease thatusethetechnique.FNA audited withindepartments surgeons, indicatingthattheresultsrarelyaltermanagement.Resultsare variableandshouldbe iscontroversialwithsome Theuseof FNA andprovidesmaterialforcytology. needle issafe withasize18Gleads toseedingoftumourcells,whichnegativelyimpactsonprognosis. FNA Open biopsyofneckmassesiscontraindicatedunlesstherenoother option.Theprocedure isalymphnodeinlymphoma. The neckmasstypicallydescribedasrubbery preoperativework-up. diagnosisisoftenunclearfollowing tumours. Theexact percentofparotid Fifty percentofsubmandibulartumoursarebenign,incontrastwith80 leadstoparalysisofthetongue. injury nerve during thedissection.Hypoglossal tothesubmentalskin,whichcanbedamaged tomylohyoidsuppliessensation mouth. Thenerve suppliesthecorner ofthe tongue aswelltastedisturbance.Themarginalmandibularnerve parotidectomy. fibresfollowing causesanaesthesiaofthe nerve Damagetothelingualnerve ofsweatglandsbycutparasympathetic sweating)isduetoinnervation syndrome(gustatory Frey’s the vesselwithit,whichcanleadtotroublesomebleeding. retractsupintothefloorofmouth, carrying structures shouldbeclampedandtied.Thenerve ofthegland.Thereisoftenabloodvesselassociatedwithganglion, andboth deep surface the towards canbeseendippingdown parasympathetic supplytothegland.Thelingualnerve oftheglandandistetheredtoitbysubmandibularganglion.Thisstructurerelays deep surface ofthesubmandibulargland.Itliesin mustbeidentifiedduringexcision The lingualnerve lies inasubplatysmalplane. which fasciawhichprotectsthemarginalmandibularnerve, layerofthedeepcervical superficial Theincisionshouldbecarriedthroughplatysmaandonthe no morethan6cminlength. Itshouldbe themandible.Thisavoidsdamagetomarginalmandibularnerve. breadths) below shouldbeplacedinaskincrease4cm(twofingers’ The incisionforsubmandibularglandexcision approach. viaatranscervical and excision 357

47: DISORDERS OF THE SALIVARY GLANDS HEAD AND NECK ➜ 24. B(seeFig.47.4) 23. D 22. C 21. E 20. E 19. B 18. A 17. D 16. A 15. C 358

Parotid surger The pain is exacerbated byeatingordrinking.Pusmaybemilkedfromtheparotidduct. The painisexacerbated overtheparotidgland. stasisandpresentswithpain,swellingerythema results fromsalivary bacterialsialadenitisclassicallyaffectsthedehydratedelderlypatient.Thecondition Ascending enclosed space developed hampersthesurgical view.enclosed spacedeveloped earcanal.Controlof bleedingatthetimeofidentificationisvital,asanybloodin cartilaginous ofthe ofthedigastricmuscleandinferiorportion The mainlandmarksusedaretheinsertion patients, asitcanmakeshavinginthis areamorechallenging. be warnedoftheanaesthesiaearlobulewhichresults.Thisismore ofaprobleminmale Allpatientsshould border ofthesternomastoidmuscle,greaterauricularmustbesacrificed. betweenitsposteriorborderand theanterior theglandismobilisedandaplanedeveloped As athighriskduetothescarrednatureofparotidbed. puts thenerve isoften required,which surgery ofrecurrencemeanrevision attractive proposition,thehighlevels Althoughenucleationisan to thepositionofresectedspecimeninrelationfacialnerve. andtotalparotidectomyrefer withacuffofnormaltissue.Thetermssuperficial excision perform shouldbeto There isnoplaceforenucleationinmodernpractice.Theaimofparotid surgery riskandthedifferentialdiagnosis,whichincludesvasculartumours. due totheairway ofthetonsil.Theseshouldnotbebiopsiedthroughatransoralapproach space withmedialisation mandible, inferiortothelobuleofear. Deeplobetumourscanpresentin theparapharyngeal areoftenbehindtheangleof they Parotid tumourscanpresentthroughoutthegland;however, topromotesialolithiasis. produced bythesubmandibularglandisalsosaid stasisandstoneformation.Themucinousformofsaliva against gravity,sopromotingsalivary tooccurastheglanddrains The majorityofstonesoccurinthesubmandibularduct.Thisissaid lesionsdemonstratedonMRI. only beconsideredforcosmesisifglandsareenlargedduetocystic should although withanegativeautoantibodyscreen.Theglandstendtobepainlessandsurgery similartoSjögren’ssyndrome, Thepresentationisvery canalsopresentwithparotidcysts. HIV incaseswhichfailtorespond. prophylactic antibioticsshouldbeconsidered,withsurgery coursesofantibiotics.Ifsymptomspersist,alongercourse measures, whichincludeshort aged 3–6yearsbutcanaffectolderchildrenwhoaremorelikelytorespondconservative aetiology.Recurrent parotitisofchildhoodisaconditionunknown Ittendstoaffectchildren should beplacedforupto72h. dissectionshouldbebluntandadrain toavoidthefacialnerve, andlow incision shouldbeshort fluctuant, aspirationispreferabletoincisionanddraining.Ifdrainagearerequired,the sialadenitis. Treatment shouldbewithfluids,analgesiaandantibiotics.Iftheglandbecomes Staphylococcus aureus y and Streptococcus viridians arethemostcommonorganismsinbacterial ➜ 29. E 28. E 27. C 26. A 25. D

Figure 4 To ZanzibarB Degenerative conditions using forms of artificial saliva. using formsofartificial troublesome.Treatmentparticularly abottleofwaterand involvesincreasedfluidintake,carrying radiotherapytotheheadandneckit can be condition dependsontheunderlying cause.Following areassociatedwithxerostomia. Managementofthe antidepressants, Anticholinergics, inparticular within theblood. There isanassociationwithlymphoma(B-celltype)whichheralded byimmunologicalchanges the disease. formof submandibular glands.Thechanceofmalignanttransformationishigher intheprimary theparotidismorecommonlyaffected thanthe glands rarelygetswollenorinfected,however diseasethereisnoassociationwithconnective tissue disorders.Thesalivary sicca). Intheprimary (keratoconjunctivitis eyes mouth(xerostomia)anddry Sjögren’s syndromepresentswithdry parotidectomy, butitisuncommonforpatientstocomplainaboutit. commonfollowing sweating.Itisvery the sweatglandsinskinflap.Thisleadstogustatory towards grow thesefibresregenerate,they theglandulartissueofparotid.As innervate The procedureinvolvesdivisionofthepostganglionicparasympatheticfibreswhichwould branches inthisregionresultssomepostoperativefunction. asithasalessvitalroleandthepresenceofmultiplecross The buccalbranchisanexception, isdamagedaccidentally,repairmustbeundertaken. circumstances, orifasectionofnerve in diseaseandthefunctioniscompromised,itmayrequireelectivedivision.Underthese ifitisclearlyinvolved atallcosts;however, shouldbepreserved The integrityofthefacialnerve Stylomastoid foramen Trunk ofFacialn. 7.4

VII nerve y MotorCar. Branches ofthefacial(seventhcranial)nerve.Theaide-memoire is Mandibulo-cervical Temporo-zygomatic Facial (7 branch branch th cranial) nerve adblrMotor By Mandibular Buccal Zanzibar Zygomatic To Temporal evclCar Cervical 359

47: DISORDERS OF THE SALIVARY GLANDS HEAD AND NECK ➜ ➜ 5L 4J 3H 2D 1B 5H 4L 3I 2B 1D matchingquestions Answers: Extended 30. B 360

disease 2. Investigationandmanagementofsubmandibulargland 1. Investigationandmanagementofaparotid mass It tends to give a short-term response. Surgery canbeusedtorepositionthesubmandibularducts response.Surgery It tendstogiveashort-term speech therapyandthoughtgiventoposture.Iftheseareunsuccessful,botoxcanbeconsidered. Most droolersarechildrenwithcerebralpalsy. measuresshouldbeemployedwith Conservative stricture occurrence. prevents bystricture formation.Marsupialisation closure oftheductislikelytobefollowed Primary gland. Anincisionisthenmadeover thestonewhichcanbedeliveredintofloorofmouth. dislodgingthestonebackintobodyof under theduct,posteriortostone toprevent isrecommended.Underlocalanaestheticasuture ispassed For thatreasontransoral excision marking forthelingualnerve. This stoneiswellanteriortothesecondmolarwhichsurface byeating. exacerbated the mostcommonsymptomswillbeintermittentunilateralpainandswelling. Thiswillbe infection,orhavingpalpatedthestonethemselves, Although thepatientmaypresentwithovert parotid stones. This isafloorofmouthX-ray. percent),unlike Submandibularstonesareoften radio-opaque(>80 ofrecurrence. Enucleation leadstounacceptablyhighlevels oftheretromandibularveinwhichcanbeseenonscan. estimated tobeatthelevel is Thepositionofthenerve confirmationcouldbeobtainedbypreoperativeMRI. Further parotidectomy. tothefacialnerve. The treatmentwillbesuperficial Mostlesionsliesuperficial procedure. todocumentitsfunction priortothe anditisimportant isatriskduringsurgery The facialnerve tumour whichcanaidinsurgicalplanningandconsent. identificationofamalignant itmayallow argueitdoesnotaltermanagement;however, as they abiopsypriortosurgery, Fine-needle aspirationisthebiopsyofchoice.Notallsurgeonsperform tumourisanotherbenignmasswhichlesscommonandcanbebilateral. Warthin’s adenoma. The mostcommondiagnosisinaparotidmassthisagegroupispleomorphicsalivary in therestingstate,somostattentionispaidtothese. danger ofturningadroolerintoanaspirator. Thesubmandibularglandsproducethemostsaliva priortorepositioningprocedures,asthereisa ofswallow mustbetakentoensuresafety Care theglandcompletely. ortoexcise to thetonsillarfossa, Theparotidductscanalsoberepositioned. PART 8 Breast and endocrine

48 Thyroid and the parathyroid gland 363 49 Adrenal glands and other endocrine disorders 375 50 The breast 385 This page intentionally left blank 48 Thyroid and the parathyroid gland

Multiple choice questions ➜ Non-thyroidal illness ➜ Medical therapy for 1. Which of the following are true with hyperthyroidism regard to sick euthyroid syndrome? 5. Which of the following are true A It can present as high T3 and T4. regarding medical therapy for B It is often seen in the ITU setting. thyrotoxicosis? C Thyroid-stimulating hormone (TSH) A Propranolol and nadolol reduce free T3 may be high. (fT3) and free T4 (fT4) levels. D TSH is usually suppressed. B Antithyroid drugs most often cure E Thyroid replacement is favoured. thyrotoxicosis due to a toxic nodule. ➜ C Carbimazole can be safely given in Hyperthyroidism pregnancy and lactation. 2. How does hyperthyroidism usually D Agranulocytosis is an uncommon present? problem with antithyroid drugs. A Graves’ disease E Patients with ophthalmopathy respond B Toxic nodule best to medical management. C Thyroid malignancy D De Quervain’s thyroiditis ➜ Surgery for thyrotoxicosis E Toxic multinodular goitre. 6. Surgery is the preferred option of ➜ treatment in which of the following Thyrotoxicosis cases of thyrotoxicosis? 3. What are the manifestations of A A diffuse toxic nodule thyrotoxicosis? B Severe manifestations of Graves’ A Irritability ophthalmopathy B Hair loss C Pregnant mothers not adequately C Muscle weakness and wasting controlled with medications D Hyperkinesias D Relapse of Graves’ E Heart failure. E Presence of local compressive symptoms. ➜ Clinical features of hyperthyroidism ➜ Radioiodine treatment 4. Which of the following are associated 7. Which of the following are indications with Graves’ disease? for radioiodine treatment? A Pretibial myxoedema A Relapsed Graves’ disease B Exposure keratitis B Thyrotoxicosis in young children C Optic neuropathy C Multinodular goitre D Chemosis D Severe ophthalmopathy E Lymphoid hyperplasia. E Pregnancy and lactation.

363 BREAST AND ENDOCRINE ➜ ➜ ➜ ➜ ➜ 12. Which of the following statements statements Whichofthefollowing 12. Hypothyroidismcouldbeapresenting 11. be hypothyroidismcan Biochemically 10. presentas: can Myxoedema 9. aretruein Whichofthefollowing 8. 364

Th h Classification of h Biochemical features of M Neonatal h A E D C B A E D C B A E D C B A E D C B A y y y poth poth y common. carcinoma isthemost Papillary Iodine deficiency. Thyroiditis Benzodiazepine treatment lesions Pituitary Sarcoidosis fT3andfT4. Low TSH Raised autoantibodies Thyroid peroxidase(TPO) Hyperlipidaemia Hyponatraemia Sepsis. ofbreath Shortness Mania Ventricular tachycardia block Complete heart toahypothyroidbaby.birth Women onantithyroiddrugsmaygive after thefirsttrimesterofpregnancy. iodinetreatmentissafe Radioactive neonates intheUK. hypothyroidism iscarriedoutonall Biochemical screeningfor Macroglossia isaclinicalfeature. deficiency. Endemic cretinismisduetoiodine regarding thyroidneoplasmsaretrue? conditions? feature inwhichofthefollowing associated with: neonatal hypothyroidism? xoedema roid cancer y y roidism roidism y poth y roidism ➜ ➜ ➜ 14. Which of the following statementsare Whichofthefollowing 14. statements Whichofthefollowing 13. 15. Which of the following statementsare Whichofthefollowing 15.

Papillar Th cancers Differentiated th cancers E D C B A E D C B A E D C B D C B A y regarding thyroglobulinaretrue? made? is of adifferentiatedthyroidcancer true whenapreoperative diagnosis true? Autoantibodies may interfere withits Autoantibodies mayinterfere It issecretedonlybycancerouscells. ofnormalthyroidfunctiontests. It ispart common. Anaplastic carcinomaistheleast the C-cells. carcinomaoriginatesfrom Medullary age of70 years. Thyroid canceriscommonestafterthe Men andwomenareequallyaffected. tumours. clearance isnotessential forsmall node Routine centralcompartment performed. Selective nodaldissectionmaybe tumours lessthan2cm. Lobectomy isrecommendedfor essential. imagingis orCT Whole-body MRI encapsulated andslow-growing. carcinoma isusuallynon- Papillary follicular cancer. cellcarcinomaisatypeof Hurthle follicular variety. Distant metastasisiscommonerinthe increased incidenceofnodalmetastasis. cancerisassociatedwith Papillary cancer.follicular cancerthaninpapillary Lymph nodemetastasisiscommonerin concentration. treatmentaffectsits Radioiodine thyroid cancer. It isasensitivemarkerofrecurrence levels. roglobulin y andfollicular y roid ➜ ➜ ➜ ➜ 19. Which of the following areassociated Whichofthefollowing 19. suggesta Whichofthefollowing 18. statementsare Whichofthefollowing 17. statements Whichofthefollowing 16.

H Calcium Primar th Medullar B A E D C B A E D C B A E D C B A E y y percalcaemia Chronic renalfailure (CRF) Thyrotoxicosis levels. PTH ionisedcalciumandelevated Raised urinecalciumlevel Low serumphosphatelevels Low levels PTH totalcalciumandelevated Raised levels PTH ionisedcalciumandsuppressed Raised adenoma. with apituitary Hyperparathyroidism canbeassociated sporadic. Familial hyperparathyroidismismostly presents asanadenoma. Familial hyperparathyroidismcommonly parathyroid hormone(PTH). Hypercalcaemia triggersthereleaseof sporadic. hyperparathyroidismisusually Primary typical are antigen(CEA) carcinoembryonic ofcalcitoninand High levels Diarrhoea isacommonfeature It isassociatedwithapoorprognosis It isnotTSH-dependent neoplasia. It canpresentasmultipleendocrine indication forcompletionthyroidectomy. arean Rising thyroglobulinlevels with hypercalcaemia? hyperparathyroidism? diagnosisofprimary biochemical true? are true? thyroidcancers regarding medullary roid y h y y carcinoma ofthe perparath y roidism ➜ ➜ ➜ ➜ 22. A 60-year-old retirednursepresents 22. Regardingoperation forprimary 21. are Which ofthefollowing 20. 23. Which of the following statements Which ofthefollowing 23.

Multiple endocrineneoplasia Primar Parath Imaging andtheparath E D C B A E D C B A E D C A E D C B A hyperparathyroidism? forprimary operative indications gland aretrue? regarding imagingoftheparathyroid appropriate tests? insulinoma issuspected.Whatarethe performed was2.5mmol/L.An and tiredness.Abloodglucose with weightloss,faintingepisodes statementsarecorrect? following hyperparathyroidism, whichofthe Serum calciumof2.75 Renal impairment bone density Low Renal stones syndrome. Milk alkali Sarcoidosis (FHH) Familial hypocalciurichypocalcaemia nearly 75percentofenlargedglands. ultrasoundcanidentify High-frequency All oftheabove. A 24hfast C-peptide levels Insulin levels synacthentest A short purposes. exploratory probescanbeusedfor Gamma favoured operativemode. Endoscopic techniqueisthemost occur inabout5–7percentofcases. damagecan nerve Recurrent laryngeal canbemeasuredintraoperatively.PTH Permanent hypoparathyroidismislikely. Young age. y y h roidectom y perparath y y roidism y roid 365

48: THYROID AND THE PARATHYROID GLAND BREAST AND ENDOCRINE ➜ ➜ Extended matching questions Extended statementsare Which ofthefollowing 24. Choose and matchthecorrectdiagnosiswitheachofscenarios givenbelow: 366

3 2 1 1. Causesofh parath Operative strategiesforthe E D C B A E D C B A E D C B creatinine 84 μmol/L, calcium3.20mmol/L. creatinine 84 An elderlymanis recovering fromaherniarepair. urea10.7 Hisbloodsareasfollows: mmol/L, low. are9pmol/L. Theurinecalciumcreatinineratioisvery levels phosphate is0.8mmol/L.HisPTH bankerattendsawell-man clinic.Hisserumcalciumis2.9mmol/Landhis A 40-year-old and hisphosphateis0.5mmol/L. A 70-year-old manpresents withconstipationandconfusion.Hisserumcalciumis3.7mmol/L Vitamin Ddeficiency Hypoparathyroidism FHH hyperparathyroidism Primary Dehydration adenoma. 10 percentofpatientswithmediastinal Preoperative imagingcanidentifynearly reduces theriskofrecurrence. totalparathyroidectomy 1 (MEN-1), In multipleendocrineneoplasiatype recommended. thymectomyisnot transcervical In patientswithfour-glanddisease, adenoma. incision ismadeoverthesiteof In atargetedapproach,2–3cm collar incisionismade. In aconventionalapproach,transverse imaging. reduced concordancewith show mg Adenomas weighinglessthan500 to first-timeneckexploration. prior mustbeundertaken andMRI CT surgical approach. scancaninfluence tomography (SPECT) Single photonemissioncomputed per centofenlargedglands. scanidentifiesaround90 isotope (MIBI) Technetium-99m-labelled sestamibi the parathyroid? true regardingoperative strategies for y roid y percalcaemia andh ➜ ➜ 26. Which of the following statementsare Which ofthefollowing 26. statementsare Which ofthefollowing 25.

y h Management of H E D C B A E D C B A pocalcaemia y y percalcaemia poparath correct? of primary hyperparathyroidism? of primary management true regardingmedical an episodeofhypocalcaemia. canoccurduring arrhythmias Cardiac of hypoparathyroidism. DiGeorge syndromeisamedicalcause hypercalcaemia. Chvostek’s signissuggestiveof can poseamedicalemergency. 1.90 below Serum calciumlevels mmol/L 24 hoftotalthyroidectomy. Serum calciummustbecheckedwithin medical emergency. Serum calciumover3.5mmol/Lisa management inhypercalcaemia. isthefirstlineof Intravenous saline to reducecalciumlevels. Palmidronate isabisphosphonateused reduce calciumlevels. Cinacalcet isabisphosphonateusedto viathekidneys. excretion byincreasingtheir calcium levels Use ofdiureticscouldreduceserum y roidism ➜ ➜ ➜ Chooseandmatchthecorrectdiagnosiswith eachofthescenariosgivenbelow: Chooseandmatchthecorrectdiagnosiswitheachofscenariosgivenbelow: Referencevaluesforthyroidfunctiontests(TFTs) areasfollows:

4. Goitre 5 4 3 2 1 3. H 4 3 2 1 2. H 5 4 B A E D C B A E D C B A A 40-year-old woman presents with hypothyroidism and elevated TPO antibodies. TPO womanpresentswith hypothyroidismandelevated A 40-year-old elevated. fT3andfT4all hasdiscordantTFTs,withTSH, A youngwomanwithrheumatoidarthritis of200μg,herthyroidfunctionsaresuggestivehypothyroidism. thyroxine dosage totreata62-year-oldwomanforhypothyroidism. Despite ishavingdifficultytrying A GP hypothyroid. TFTs werenormal.Heisnow chemicalcardioversionforatrialfibrillation (AF).Hisbaseline A 44-year-oldmanunderwent reflexes. and slow skin hasdry block.She is overweight, An elderlywomanpresetstoA&Ewiththird-degreeheart 0.01mIU/ TSH, are asfollows: L;T3,8.0pmol/T4,41 pmol/L. A 22-year-oldwomanwithasmoothgoitrepresentssweatingandweightloss.HerTFTs <0.01 TSH as follows: L;T3,2.0pmol/T4,0.5pml/L. mIU/ laparotomyforableedingulcer. emergency following A middle-agedmanisinITU HisTFTsare 5.2 pmol/L;T4,11 pmol/L. A 10-week L;T3, 0.02mIU/ TSH, vomiting.HerTFTsareasfollows: pregnantwomanhassevere <0.1TSH L;fT3,15.8 mIU/ pmol/L;fT4,1.0 pmol/L. isasfollows: An elderlywomanpresentswithsweatingandconfusion.Herbiochemistry calcium is1.8 IU/L. phosphataseis640 mmol/L,phosphateis0.7mmol/Landserumalkaline ahipfracture.Herserum An elderlywomanwhoismostlyhouse-boundwasadmittedfollowing tinglingandnumbnessaroundhermouth. the operation,shedeveloped athyroidectomyforlargemultinodulargoitre.Eighthoursafter A youngwomanunderwent fT4 10 –23pmol/L fT3 3.5–6.5pmol/L L 0.5–4mIU/ TSH Multinodular goitre Toxic nodule Heterophil antibodiesinterference Myxoedema Amiodarone-induced thyroiddisease adenomas(TSHoma) TSH-secreting Autoimmune thyroiditis Toxic nodule Sick euthyroiddisease Thyrotoxicosis effect Human chorionicgonadotrophin(HCG) Graves’ disease y y poth perth y y roidism roidism 367

48: THYROID AND THE PARATHYROID GLAND BREAST AND ENDOCRINE ➜ ➜ ➜ 2.A,B,D, E 1. B,D Answers: Multiplechoicequestions Chooseandmatchthecorrectdiagnosiswitheachofscenariosgivenbelow: Chooseandmatchthecorrectdiagnosiswitheachofscenariosgivenbelow: 368

H Non-th 4 3 2 1 5. Th 4 3 2 1 postpartum, can causehyperthyroidism. postpartum, Thyroidinflammation,especially rarelypresentsashyperthyroidism. Thyroid cancervery receptorantibodies. isnotassociatedwithTSH often autonomousanditshypertrophy/hyperplasia toxicnoduleis signs.Asolitary in middleageortheelderlyand is rarelyassociatedwitheye disease, type1diabetes,pernicious anaemiaandcoeliacdisease.Toxic nodulargoitreisoftenseen prolonged effect.Itcanbeassociated withotherautoimmuneconditionssuchasAddison’s and receptor antibodies produceadisproportionate signs.AbnormalTSH is associatedwitheye Graves’ diseaseisacommonautoimmuneconditionthatusuallyoccurs inyoungwomenand clear ifreplacementbenefitssuchcases. sofaris not Theevidence T3andT4aninappropriatelysuppressedTSH. characterised bylow infections,renalfailure,liverfailureandmalignancies. Itis illness,suchassevere of severe Non-thyroidal illnessissynonymoustosickeuthyroidsyndrome.Itappears inthecontext D C B A D C y A 28-year-old has a parathyroid adenoma and elevated levels ofcalcitonin. levels A 28-year-oldhasaparathyroidadenomaandelevated A 12-year-old boypresentswithanoduleandregionallymphnodeenlargement. hard noduleinherrightthyroidlobe. ofHashimoto’sthyroiditispresentswithanirregular, history An elderlywomanwithprevious hypercalcaemia. and An 82-year-oldwomanpresentswithathyroidnodule,weightloss,backache corneal reflexes. A 55-year-oldpresentswiththyroidnoduleandleft-sidedtotalophthalmoplegiaabsent nodule. increaseduptakebythe presentswithafirmthyroidnodule.Isotopescanshows A 60-year-old the eye. A youngwomanpresentswithafirmenlargedgoitrebruit.Shecomplainsofgrittinessin than theleft.Sheisclinicallyeuthyroid. A middle-agedwomanpresentswithalong-standinggoitre.Herrightlobeismoreenlarged perth Medullary carcinoma Medullary Lymphoma Anaplastic carcinoma thyroidcancer Papillary Graves’ disease. Follicular carcinomaofthethyroid y roid cancers y y roidal illness roidism ➜ ➜ ➜ ➜ ➜ ➜ 8.A,B,C, E 7. A,C 6.A,B,C,D,E 5. D 4.A,B,C,D,E 3.A,B,C,D,E

Neonatal h Radioiodine treatment Surger Medical therap Clinical features ofh Th Thyrotoxicosis ischaracterisedbytheclinical,physiologicalandbiochemicalchangesthat are due to either inborn error of thyroid metabolism or complete or partial agenesisof the are duetoeitherinborn errorofthyroidmetabolism or completepartial whereassporadic cases deficiency, early neonatalperiods. Endemiccretinismisduetodietary Cretinism isaconsequenceofinadequate thyroidhormoneproductionduringthefoetaland relapsed Graves’diseaseandinmultinodular goitreandadenomas. children, pregnantandlactatingmothers. Itisthedefinitivechoiceoftreatmentincases thyroid status.Hypothyroidismoccursin15–20 percentat2years. Itiscontraindicatedinyoung normal treatmentrecommendstheadministrationofenoughradioiodine toachieve Radioiodine resections. risk ofpermanenthypothyroidismintotalthyroidectomyandrecurrence oftoxicityinsubtotal thereisa reducingthemassofoveractivetissue.However, Cure canbeofferedbysurgery, reduce theadrenergiceffectsofthyroxineinperipheraltissues occursinonly0.1–0.5Agranulocytosis percentofpatientsonantithyroiddrugs.Beta-blockers management. Atoxicnoduleisautonomousinnatureandwillrelapseifmedicationsarestopped. fT3andfT4rarelybenefitfrommedical elevated Large goitres,youngerpatientsandvery 10 yearsafterdiscontinuationoftherapy. rarelyrelapsesarecorrectedwithmedication. However, percentofpatientsremaineuthyroid are thecommonantithyroiddrugsthatused;30–40 andlactation) (firstdrugofchoice)andpropylthiouracil(givenduringpregnancy Carbimazole in thedermisandcutis. radiotherapy issometimesneeded.Thyroiddermopathycausedbydepositionofhyaluronidase ulceration, congestiveophthalmopathyoropticneuropathy. Treatment withsteroidsandorbital of ophthalmicveins.Urgentaction(orbitaldecompression)isneededinthecasecorneal andconjunctivalinjectionmaybeworsenedbycompression round cells.Oedemaoftheeyelids diseaseisduetoretrobulbarinfiltrationoftissueswithfluidand inthyroideye The exophthalmos with hypokalaemia. failureand,rarely,periodicparalysiswhichisassociated onycholysis, hairloss,highoutputheart and fatigue.Thesignsoftenarefinetremor,warmmoistskin,proptosispalmarerythema, hyperactivity, insomniaandheatintolerance,weightlosswithincreasedappetite,palpitations thyroidhormone.Itcanpresentwithsymptomsof toexcess result whentissuesareexposed low radioiodineuptake. low pregnant womeninadequatelycontrolledonmedications;andlargethyroid glandswithrelatively presenceofsuspiciousthyroidnodulebyFNAC; arealsoindicationsforsurgery: The following occursin<5percentofcasesundergoingsurgery. Parathyroidinsufficiency been performed. y rotoxicosis y

Recurrence canalsooccurin5percentof forth y poth y rotoxicosis y forh y roidism y y perth perth y y roidism roidism

cases iflessthanatotalthyroidectomyhas 369

48: THYROID AND THE PARATHYROID GLAND BREAST AND ENDOCRINE ➜ ➜ ➜ ➜ ➜ ➜ 14. B,C,D,E 13. C,D 12. A,D 11. A,B,D,E 10. A,B,C,D,E 9.A,B,C,D,E 370

years. 100 ratio is3:1. 000populationandthefemale:malesex Itiscommonestinadultsaged40–50 and malignantlymphoma,5percenteach.Theannualincidenceofthyroid canceris3.7casesper percent;follicularcarcinoma,20anaplasticcancer, 10cancer, 60 percent;medullary papillary malignanttumourofthethyroidcancer isasfollows: The relativeincidenceofprimary Papillar Th • • • Th Classification ofh Biochemical features ofh M limitabilitytoinducehypothyroidisminthefoetus. preferred asthey delay.developmental pregnantwomen,drugssuchaspropylthiouracilare Inhyperthyroid long-termphysicalandmental gland. Immediatediagnosisandtreatmentareessentialtoprevent confined totheneck in>95percentofcases.Follicular cancerarises fromtheepithelium.Itis are andspreadtoadjacentstructures (nodes).They cancers areencapsulated, slow-growing Papillary Compared withmostcancers,theprognosis indifferentiatedthyroidcancersisexcellent. <2 ng/L.Itsmeasurementcanbemade difficultbythepresenceofantithyroglobulinantibodies. thyroid cancer. mustbeundetectableat andradioiodine treatment,levels surgery Following Thyroglobulin issecretedbynormalthyroidtissueandasensitivemarker ofrecurrence Depending onthecellsoforigin,thyroidcancerscanbeclassifiedasoriginatingfrom: infective (rare). or (panhypopituitarism);neoplastic;infiltrative(sarcoidosis); post-radiation); pituitary-related acid);non-goitrous(atrophicthyroiditis, such aslithium,amiodarone,iodidesandaminosalicylic Hashimoto'sthyroiditis ordrugs Hypothyroidism canbeclassifiedas:goitrous(iodinedeficiency, cholesterol. hyponatraemia andraisedtotalcholesterolLDL Itcanpresentas whichiselevated. secretesTSH, attempt,thepituitary a compensatory freet3andt4hormones.As Biochemically hypothyroidismischaracterizedbylow associated withincreasedmortality. failureandinfectionsare Heart hypothermia withitsassociatedtachy-andbradyarrhythmias. tingetotheskinandalteredmentalstate.Itcanpresentasbradycardia a malarflush,yellow isassociatedwithaccentuatedsignsandsymptomsofhypothyroidism.Thereoften Myxoedema C-cells – medullary carcinoma. C-cells –medullary –lymphoma lymphocytes – undifferentiatedanaplasticcarcinomas andfollicularcarcinoma – differentiatedsuchaspapillary cells papillary y y y xoedema roglobulin roid cancer y andfollicularcancers y poth y roidism y poth y roidism ➜ ➜ ➜ ➜ ➜ ➜ 20. A,B,C,E 19. A,B, C,D,E 18. B,C, E 17. A,E 16. A,B,D,E 15. B,C,D,E

Parath H Calcium Primar Medullar Differentiated th cellcarcinomaisanaggressivetypeoffollicularcancerwithapoorprognosis. Hurtle capsular andismorelikelytospreadviathehaematogenousrouteratherthanlocalinvasion. agonists andbisphosphonate therapyarehelpfulalong witheradicationofdrugsthatinduce istheonly curativeoptionforthiscondition. Medicaltherapysuchascalciumreceptor Surgery cause hypercalcaemia. such astuberculosisandsarcoidosis hyperparathyroidism.Granulomatousconditions bythegland,causingtertiary secretion ofPTH long-standingstimulationoftheparathyroidresults inautonomous hyperparathyroidism. However, causingsecondary these resultinstimulationoftheparathyroidglandandreleasePTH, andhypocalcaemia initially; Chronic renalfailureisassociatedwithvitaminDdeficiency phosphate andhypercalciuria. serum calcium isjustasgood.Hyperparathyroidismassociatedwithraised serum calcium,low is biologicallyactive.Measurementofionisedcalciumtediousandalbumin-adjusted (corrected) itistheionisedorfree calciumthat circulates intheplasmamostlyboundtoalbumin;however, bytheparathyroidgland. Calcium Hypercalcaemia normallysuppressesthereleaseofPTH ashyperplasia. exists asisolatedhyperparathyroidismand Thefamilialvarietycanalsoexist phaeochromocytoma). carcinomaofthyroidand hyperparathyroidism,medullary 2(primary andMEN and VIPoma) adenomaandpancreatictumourssuchasinsulinoma,gastrinoma hyperparathyroidism, pituitary (primary Familial hyperparathyroidismisgeneticallydeterminedandassociatedwithMEN-1 havemultipleadenomasorcancer. smallproportion hyperplasia oftheglands;andavery are sporadic.Nearly85percentofthesepatientshaveanadenoma;13 percenthave hyperparathyroidismincreaseswithadvancingage.Mostcases ofprimary The prevalence 2A). syndrome calledmultipleendocrineneoplasiatype2A(MEN andhyperparathyroidismina of diseasemaybeslow. Itcanoccurwithphaechromocytoma producedbytumourcells.Likemanyendocrineneoplasms,theprogression 5-hydroxytryptamine Diarrhoeaispresentin30percentofcasesandmaybedueto of serumcalcitoninandCEA. carcinomareferstotumoursoftheparafollicularcells.Itischaracterisedbyhighlevels Medullary ifrequired. isperformed, node dissectionofinvolvedlevels involvement ormetastasis.Lobectomyisrecommendedfortheremainder. Functional selective Total thyroidectomyisrecommendedfortumoursgreaterthan2cmandthosewithnodal issuggested. orCT differentiatedthyroidcancer,imagingoftheneckwithMRI thatshows a FNAC following controversiesregardingtreatmentfordifferentiatedcancers.However, There areseveral y percalcaemia y y h roidectom y y carcinoma oftheth perparath y y roid cancers y roidism y roid 371

48: THYROID AND THE PARATHYROID GLAND BREAST AND ENDOCRINE ➜ ➜ ➜ ➜ ➜ 25. A,B,D,E 24. A,B, D 23. A,C,E 22. B,C,D 21. A,B,E 372

H Operative strategiesfortheparath Imaging andtheparath Multiple endocrineneoplasia Primar bone densityorthepresenceofosteoporosisandrenalstonediseaseimpairment. serum calciumover2.9mmol/L(especiallyinthepresenceofsymptoms),falling isindicatedinthepresenceofhypercalcaemiawith hypercalcaemia (diuretics,lithium).Surgery given 3or4times daily. 10 percentmagnesiuminfusionis alsoconsidered.Inmildercases,1g oforalcalciumcanbe and10treated asanemergency mLor10 slowly; percentcalciumgluconatemustbegiven IVI 1.90 andseizures.Serumcalciumbelow cardiac arrhythmias mustbe neuromuscular excitability, It couldrangefrommildcircumoral anddigitalnumbnesstotetanywithcarpopedalspasm, ofhypocalcaemia. Signs andsymptomsofhypocalcaemia arerelatedtothedurationandlevel identifies around1percentofpatientswithamediastinaladenoma. hyperparathyroidism.Preoperativeimaging to reducetheriskofrecurrenceorrecurrentprimary thymectomyisconsidered fasciaisincised.Infour-glanddisease,atranscervical deep cervical superiorly and,inferiorly,the a transverseincisionismade,thesubplatysmalplanedeveloped Trendelenburgis suboptimal.Inaconventionalapproach,thepatientpositionedin reverse and incisioniftheimaging toaformalbilateralexploration adenoma. Thisisplacedtopermitextension permitsa2–3cmincisionlocatedoverthe siteofthe Confident preoperativelocalisation dimensional approachandcaninfluencesurgicalapproach. scangivesathree- area scannedmustincludethemediastinumtodetectectopicglands.SPECT Technetium-99m-labelled sestamibiisotopescanscanidentify75percentofglandsandthe despitegoodresolutionithaspoorpenetranceandcannotvisualisethemediastinum. However, ultrasoundisnon-invasiveandcanidentify75percentofenlargedglands. High-frequency required. andgenetictestingareothertests levels ofboneprofile,PTH carcinoma (type2).Assessment adenoma(type1)andmedullary associatedwithhyperparathyroidismandpituitary ofMEN part 24hfastmaybenecessary. Insulinomacanbea up thesesmalllesions(<4mm).Asupervised maynot alwayspick difficulttodiagnose,asimagingwithMRI and C-peptide.Itisoftenvery ofproinsulin levels by demonstratinghyperinsulinaemiadespitehypoglycaemiawithelevated An insulinomaisatumourwhichoriginatesfromthepancreas;itbiochemicallyconfirmed preoperativeinjectionofMIBI. following guide exploration A conventionalthyroidectomyincisionismostfrequentlyused.gammaprobecanbeusedto measured inanoperativesettingtoguidethesurgeonifglandhasbeencompletelyremoved. are levels suggestresidualgland.PTH despiteexcision, ofthegland.Highlevels, of excision minutes withinafew half-lifeandwilldisappear short hasavery hypoparathyroidism. PTH (1percent),postoperativehaemorrhagerecurrentandpermanent injury nerve Preoperative discussionforaparathyroidectomymustincludethepossibilityofrecurrentlaryngeal y poparath y h y perparath y roidism y roidism y roid y roid ➜ ➜ ➜ ➜ 1D 4A 3D 2B 1C 5E 4D 3A 2C 1B matchingquestions Answers: Extended 26. C,D,E

3. H 2. H 1. Causesofh Management ofh Medical management of primary hyperparathyroidism involves a low calciumdietandwithdrawal hyperparathyroidisminvolvesalow Medical managementofprimary Myxoedema can present with heart block,hypothermia andsometimescoma. can presentwithheart Myxoedema andafemalepreponderance. history Graves’ diseaseisanautoimmunecondition characterisedbyasmoothgoitre,strongfamily andfreethyroidhormones. euthyroid diseasecausessuppressionofbothTSH Thyroid functionsarenothelpfulintheimmediateperiodafteranacute illnessorstress.Sick well. arealteredas pregnancy. Thethyroid-bindingglobulinlevels ThisitselfcausessuppressionofTSH. inearly increasesexponentially Beta-HCG can beassociatedwithhyperthyroidism. Pregnancy areoftenlow.levels Duetorapid conversionofT4toT3, T3 toxicosiscanpresentwithallfeaturesofthyroidexcess. recentfracturecouldalsoraisetheALPlevels. osteomalacia. However, in phosphataseiselevated cases.Alkaline hypocalcaemia andhypophosphataemiainsevere iscommoninhouse-boundandinstitutionalisedindividuals.Itcauses Vitamin Ddeficiency mustalwaysbecheckedwithin24hofanythyroidsurgery.Levels removed. whentheparathyroidglandsareinadvertently cancomedown Serum calciumlevels commoncauseformildhypercalcaemia. Dehydration isavery receptors intheparathyroidgland. Mostpatientsareasymptomaticandtheproblemisincalcium-sensing calcium excretion. urinary fromlow hyperparathyroidismapart biochemicalfeaturesasprimary causes thesame hyperparathyroidism.It Familial hypocalciurichypocalcaemiaisanotherdifferentialforprimary are notsuppresseddespitethehypercalcaemia. levels The PTH phosphatelevels. hyperparathyroidismisassociatedwithraisedserumcalciumandlow Primary bisphosphonate suchaspalmidronateisessential. tocorrectdehydrationanda Intravenoussaline receptor agonistandreducescalciumlevels. of drugsthataggravatehypercalcaemia,suchasdiureticsandlithium.Cinacalcetisacalcium y y perth poth y y roidism roidism y percalcaemia andh y percalcaemia y pocalcaemia 373

48: THYROID AND THE PARATHYROID GLAND BREAST AND ENDOCRINE ➜ ➜ 4D 3A 2C 1B 4C 3A 2D 1B 5A 4E 3B 2C 374

5. Th 4. Goitre cause bothover-activationandunder-stimulationofthethyroidgland. andcan highiodineload,however, Itcontainsavery Amiodarone isapopularantiarrhythmic. and carriesagoodprognosis.These carcinomassecretecalcitoninfromtheparafollicularcells. syndrome.Itoccursinyoungadults oftheMEN carcinomaofthethyroid canformpart Medullary metastasis totheregionallymphnodes. arewell differentiatedandshow thyroidcancersoccurinchildhoodandmiddleage.They Papillary Lymphoma canariseinaglandaffectedbyHashimotoísthyroiditis. Anaplastic carcinomatendstooccurintheelderlyandismalignant poorlydifferentiated. signs. eye the relevant later inlifeandispronetoblood-bornemetastasis.Secondariesthecavernoussinusresult carcinoma.Itarises Follicular carcinomaofthethyroidissecondcommonestafterpapillary features). hyperthyroid normalordecreaseduptake)atoxicadenoma(increaseduptakewith adenoma (scanshows toxicnodulecouldrepresentasingleactiveinmultimodalgoitre,thyroid Solitary characteristic. is diffusegoitrewithabruitclinicalandbiochemicalhyperthyroidism, Exophthalmos, receptorantibodies. Graves’ diseaseisanautoimmuneduetothepresenceofTSH ifthereisadoubtaboutthediagnosis. undertaken shouldbe Simple multinodulargoitreisduetorelativeiodinedeficiency. Itislong-standing. FNAC areassociatedwithautoimmunehypothyroidism. Thyroid peroxidaseantibodies(TPO) heterophil-blocking antibodies. testsonanotheranalyserortheuse canbecorrectedbyperforming discordant TFTs.They andcause inthefT4levels caninterfere Autoantibodies, especiallyfoundinrheumatoidarthritis, despiteadequatethyroxinereplacement. levels TSH another causeofelevated Thyroidhormoneresistanceis secretingTSH. is a raretumourfromtheanteriorpituitary TSHoma y roid cancers 49 Adrenal glands and other endocrine disorders

Multiple choice questions ➜ Anatomy of the adrenal ➜ Adrenal masses gland 4. Which of the following statements is 1. Which of the following statements are false? correct? A All adrenal masses need to be biopsied A The right adrenal is located on top of the B The likelihood of a mass being malignant upper pole of the right kidney. increases with increasing size. B The left adrenal is covered by the C Nearly 80 per cent of adrenal masses are pancreatic tail. non-functioning. C The right adrenal vein drains into the D Conn’s disease accounts for 1 per cent of renal vein. adrenal adenoma. D It is difficult to cannulate the left adrenal E The non-functional adenoma should not vein. undergo surgical resection. E The inner layer of the adrenals is called ➜ the zona reticularis. Primary hyperaldosteronism 5. Which of the following are true in ➜ Hormones and the adrenals primary hyperaldosteronism? 2. Which of the following statements A Hypokalaemia is always present. regarding the adrenal gland are true? B Plasma renin activity is increased. A The zona glomerulosa secretes C Plasma aldosterone is increased. aldosterone. D Drugs such as beta-blockers have to be B The zona reticularis consists of discontinued before the test. chromaffin cells. E If a biochemical diagnosis is reached, no C Cortisol is produced in the adrenal imaging is required. medulla. ➜ D Aldosterone is a mineralocorticoid. Adrenal vein catheterisation E Catecholamines are only formed in para- 6. Which of the following statements adrenal glands. regarding adrenal vein catheterisation (AVC) are true? ➜ Adrenal hormones A Samples for catecholamines are sampled 3. Which of the following tests are from the inferior vena cava (IVC). indicated in the hormonal evaluation B Samples for aldosterone are sampled of an adrenal mass? from the IVC. A Catecholamine estimation in blood C Samples for aldosterone are sampled B Measurement of aldosterone in blood from the adrenal veins. C Urinary electrolyte analysis D AVC must only be considered if surgery D 24 h urinary cortisol is planned. E 24 h urinary catecholamines. E It is technically difficult to cannulate the left adrenal vein.

375 ➜ Cushing’s syndrome The results are as follows: 0 min, 7. How is Cushing’s syndrome clinically 300 mmol/L; 30 min, 450 mmol/L; characterised? 60 min, 390 mmol/L. What is this A Central obesity suggestive of? B Osteoporosis A Adrenal failure C Hirsutism B Pituitary failure D Hypokalaemia C Sepsis E Hyperkalaemia. D Cushing’s syndrome E None of the above.

BREAST AND ENDOCRINE AND BREAST 8. Which tests are indicated in the diagnosis of Cushing’s syndrome? ➜ Multiple endocrine neoplasia A 24 h urinary cortisol 12. Which of the following statements B 9am cortisol are true regarding multiple endocrine C Midnight cortisol tumours (MEN)? D Serum adrenocorticotrophic hormone A They are always benign in nature. (ACTH) B They are mostly inherited. E Dexamethasone suppression test. C The mode of inheritance is autosomal ➜ Surgery in Cushing’s disease dominant. D MEN-1 could be caused by mutations in 9. Which of the following statements are the menin gene. true? E MEN 2 is caused by mutations in the RET A Patients with Cushing’s are at increased proto-oncogene. risk of hospital-acquired infections. B Patients do not require prophylactic ➜ Pancreaticoduodenal anticoagulation. C Following surgical removal of endocrine tumours unilateral adrenal adenomata, cortisol 13. Which of the following statements are supplementation is not necessary. true? D Nelson’s syndrome is a cause of A Insulinomas are the commonest form of Cushing’s disease. pancreaticoduodenal endocrine tumours E Cushing’s-associated medical conditions (PETs). need not be treated medically B Diarrhoea is a common presentation in preoperatively. gastrinomas. C Peptic ulcers in gastrinomas are solitary. ➜ Congenital adrenal D VIPomas present with diarrhoea. hyperplasia E PETs occur in 50–60 per cent of MEN-1 patients. 10. Which of the following regarding congenital adrenal hyperplasia (CAH) ➜ are true? MEN-1 A Virilisation is characteristic. 14. Which of the following statements B Girls are affected more than boys. regarding operative management are C Some children are affected in puberty. correct? D 17-hydroxylase is the commonest A All MEN-1 parathyroid adenomas must enzyme deficiency. be operated on. E Cortisol excess is a common feature. B MEN-1 gastrinomas must be operated on. C MEN-1 insulinoma must be operated on. ➜ Short synacthen test D Distal pancreatectomy is the procedure 11. A short synacthen test was performed of choice in most gastrinomas. in a 26-year-old male admitted to the E Non-functioning PETs need not be ITU with multi-organ failure. operated on.

376 ➜ ➜ ➜ Extended matchingquestions Extended aretrue Whichofthefollowing 15. Choose and matchthecorrectdiagnosiswitheachofscenarios givenbelow: Chooseandmatchthecorrectdiagnosiswitheachofscenariosgivenbelow:

3 2 1 E D C B A 2. H 5 4 3 2 1 E D C B A 1. H E D C B A Neuroendocrine tumours A 48-year-oldman presentswithvisualdisturbance,headache andhypertension. truncal obesityand bruising. hirsutism, hasgraduallydeveloped A long-standingasthmaticwithfrequent exacerbations alkalosis. chronicsmokerpresents withweightloss,ankleoedemaandhypokalaemic A 40-year-old Conn’s syndrome Non-endocrine tumour Multiple endocrineneoplasia Cushing’s syndrome Cushing’s disease blanchable rashonherlegs. A universitystudentisbroughttoA&Eresuscitationareamoribund.She hasapurpuricnon- 6.1 mmol/L. Naof124 seizures.Bloodtestsshow A 4-day-oldneonatedevelops mmol/L andKof of4.2. ratio (INR) therapypresentswithhypotensionandaninternationalnormalised An elderlymanonwarfarin of bilateraladrenalectomy. malehasmarkedpigmentationofskinandscartissues.Heabdominalscars An 80-year-old A 17-year-old girlwithtype1diabetespresentsweaknessandgeneralisedpigmentation. Adrenal haemorrhage Congenital adrenalhyperplasia Addison’s disease Nelson’s syndrome Meningococcal septicaemia the appendix. arelocatedin percentofNETs Nearly 40 metabolites ofserotonin. arethemajor Catecholamines Chromogranin Aisatumourmarker. gastrin arestoredinitsgranules. Peptide hormonessuchasserotoninand ofbronchi. mucosa arefoundassinglecellsinthe They (NETs)? regarding neuroendocrinetumours y y percortisolism poadrenalism 6 statements Which ofthefollowing 16. E D C B A syndrome despitemetastasis. Patients donotsufferfromcarcinoid cases. inthese elevated isoftenvery 5-HIAA treated aslungcancer. mustbe A highlyundifferentiatedNET obstruction. to symptomsofcoughandairway The diagnosisismostlymadedue 1 percentofallNETs. The bronchiandlungconstitute true? regarding NETs ofthebronchiare 377

49: ADRENAL GLANDS AND OTHER ENDOCRINE DISORDERS BREAST AND ENDOCRINE ➜ ➜ ➜ Choose and matchthecorrectmanagementwitheachof scenariosgiven below: Chooseandmatchthecorrect diagnosiswitheachofthedescriptionsgivenbelow: Chooseandmatchthecorrectdiagnosiswitheachofscenariosgivenbelow: 378

1 E D C B A 5. H 5 4 3 2 1 E D C B A 4. Carcinoid tumours 5 4 3 2 1 E D C B A 3. Sexhormones 5 4 oligomenorrhoea. Herserumtestosteroneismildlyelevated. weakness. diabetes,weightgainandmuscle hypertension, A youngmanpresents withrecent-onsetsevere Reassurance scan MRI Plasma reninactivity cortisol 24 hurinary Urine K These tumourscanpresentwithhypoglycaemia. anddiabetes Ectopic hormonesecretioncausingweightgain,hypertension fibroticlesions Associated Liver metastasisoftumoursthatarisefromtheenterochromaffincells Tumours thatarisefromtheenterochromaffincells tumour Carcinoid Cushing’s syndrome syndrome Carcinoid Pancreatic carcinoid stenosis Pulmonary presentswithpoorlibidoandlossofbodyhair.A 50-year-old test. the pregnancy kitandhisurinetestedrepeatedlypositivefor pregnancy A youngmanwastestinghispartner’s featuresofambiguousgenitalia. A 3-day-oldfemalechildshows aremarkedlyelevated. voice. Herserumtestosteronelevels ofhirsutism,clitoromegalyanddeephoarse history A 42-year-oldwomanpresentswithashort A 21-year-old 37 kg/m BMI, womanwhoisobese(bodymassindex, ovariansyndrome Polycystic Arrhenoblastoma 21-hydroxylase deficiency Teratoma Hypogonadotrophic hypogonadism A 36-year-oldmanisbeinginvestigatedforresistanthypertension. occasions intherecentpast. glucose wasfoundtobe2.3mmol/L.Herfamilymentionthatshehascollapsedonafew Paramedics arecalledtothehomeofayoungwomanwhosufferedfromseizure.Herblood y pertension andh + , laxative anddiureticscreen , laxative y pokalaemia 2 ) develops hirsutismand ) develops ➜ ➜ ➜ ➜ ➜ 5.B,C,D 4.A,E 3.B,C,D,E 2.A,B,D 1. B,C,D Answers: Multiplechoicequestions

Primar Adrenal masses Adrenal hormones Hormones andtheadrenals Anatom 5 4 3 2 (PHA). Hypokalaemia is not always present. In normokalaemic PHA, 70 percenthave hyperplasiaof PHA, isnotalwayspresent.Innormokalaemic Hypokalaemia (PHA). hyperaldosteronism A unilateraladenoma (Conn’ssyndrome)isthecommonest causeofprimary mass. Non-functioningadenomasgreater than4cmmustbeconsideredforsurgicalresection. adrenal tumours.Theprobabilityofan adrenalmassbeingacarcinomaincreaseswiththesizeof often theindicationsforanadrenalbiopsyaretoconfirmmetastasis,which accountsfor2percentof itcanresultinfatalactivationoftargetorganssuchasthecardiovascular system.Most otherwise biochemically; has beenexcluded An adrenalmassmustnotbebiopsieduntilaphaeochromocytoma areessential.UrineK dehydroepiandrosterone (DHEA) steroids suchastestosteroneand plasmareninactivity,aldosterone,serumsex Serum electrolytes, are diagnosticofhypercortisolism. (thusamorningandmidnightcortisol) loss ofcircadianrhythm suppressiontestand catecholaminesarethefirst-lineinvestigations.Adexamethasone 24h urinary and cortisol A24hurinary istodemonstratehormonalexcess. The purposeofhormonalevaluation catecholamine granules. steroids.Themedullaconsistsoflargechromaffincellsthatstore reticularis, whichsecretesthesex and zona aglucocorticoid; thezonafasciculata,whichsecretescortisol, mineralocorticoid; whichsecretesaldosterone,a itselfhasthreezones:thezonaglomerulosa, The cortex andmedulla. ismadeupofthecortex andhistologically,theadrenalcortex Embryologically hyperaldosteronismandbilateralhyperplasia. to differentiatebiochemicallybetweenprimary Thisprocedureisperformed adrenal veinduetoitanatomicpositionduringvenoussampling. fromtheleft the leftadrenalveindrainsintorenalonleft.Henceitisdifficulttosample by thepancreatictailandspleen.AlargeadrenalveindrainsintoIVConrightside, andiscovered inferior venacava(IVC).Theleftadrenalliesontheupperpoleofkidney andthediaphragm,closeto The rightadrenalglandliesbetweenthelobeofkidney towards primary hyperaldosteronism where there is excessive lossofK hyperaldosteronismwherethereisexcessive primary towards A 23-year-old fitness enthusiast gives a history oftiredness,weightlossandhypokalaemia. A 23-year-oldfitnessenthusiastgivesahistory A 32-year-oldwomanpresentswithresistanthypertension. arenormal. electrolytes Hehasbeencommencedonadiuretic.HisserumKis3.2mmol/L.Other hypertension. CityofLondonworkerisfoundtohaveborderline sedentary A middle-aged,overweight, excess. suppressiontestsuggestscortisol dexamethasone abdominalpainandtachycardia.His A 52-year-oldmanpresentswithhypertension, y h y oftheadrenal gland y peraldosteronism + is an easy and informative test in the work-up isaneasyandinformativetestinthework-up + inurinedespitehypokalaemia. 379

49: ADRENAL GLANDS AND OTHER ENDOCRINE DISORDERS BREAST AND ENDOCRINE ➜ ➜ ➜ 9. A 8.A,B,C,D,E 7. A,B,C,D 6.B,C,E 380

Surger Cushing’s s Adrenal veincatheterisation todistinguishunilateralfrombilateraldisease. performed such asalpha-blockershavetobeconsidered.Despitebiochemicaldiagnosis,imaginghas significantlysuppressreninactivity. Beta-blockers, however, Alternativeagentstocontrolhypertension drugsaffecttherenin–angiotensin–aldosteronepathway. thisdiagnosis.Several pointstowards levels Demonstrationoflacksuppressionreninactivitydespitehighaldosterone the adrenalcortex. there is suppression of the contralateral gland which takes several monthstorecover.there issuppression ofthecontralateralglandwhich takes several Steroidcover toimproveoutcome.Incase ofunilateraladenomata, must bewellcontrolled priortosurgery diabetesandhypertension arealsoatincreasedriskofthromboembolic disease.Associated They canbe immunocompromisedandareatincreasedriskofinfection. Patients withsteroidexcess be imagedasitislikelytothesourceofhormonalexcess. issuppressed,theadrenalsmust IftheACTH to lookfortumoursthatmaysecreteectopicACTH. Thechestandabdomenareimaged sinusmaybesampled. isnegative,aninferiorpetrosal MRI If isimagedwithMRI. thepituitary areelevated, levels causes.IfACTH from ACTH-independent causes (seeFig.49.1). willdiscriminateACTH-dependent circadian rhythm levels SerumACTH isdemonstrationoflacksuppressionand/orloss todiagnosehormonalexcess The key and hypokalaemia. diabetes,skinchangesandhirsutism.Patientsoftenpresentwithmuscleweakness hypertension, suchasobesity, excess, Clinically Cushing’ssyndromeischaracterisedbyfeaturesofcortical right adrenalveindrainsdirectlyintotheIVCandiseasiertocannulate. The leftadrenalveindrainsfirstintotherenalvein,whichsubsequentlyIVC; Asignificantincreaseintheratioononesideindicatesunilateraldisease. each ofthesesamples. ratioisdetermined in from theIVCandrightleftadrenalveins.Thealdosterone:cortisol andaldosteroneareobtained forcortisol of bilateralhyperplasia.Duringthisprocedure,samples management isconsidered.Anapparentunilateralmasscouldbenon-functioninginthepresence isconsideredbeforeadecisionregardingsurgicalornon-surgical Adrenal veincatheterisation y inCushing’s disease Figure 4 y ndrome feedback Negative 9.1 H y pothalamic-Pituitar CRH (hypothalamus) ACTH (pituitary) y (adrenals) Cortisol -Adrenal axis. ➜ ➜ ➜ ➜ ➜ 14. B,C 13. B,D, E 12. B,C,D,E 11. A 10. A,C

MEN-1 Pancreaticoduodenal endocrinetumours Multiple endocrineneoplasia Short s Congenital adrenal h in ACTH. tumour, producingNelson’ssyndrome,characterisedbyhyperpigmentationandmarkedelevation 10 anaggressivepituitary percentofpatientswhoundergobilateraladrenalectomydevelop monthsafteraswell.Nearly is requiredintheimmediatepostoperativeandforseveral pancreatectomy isconsidered. pancreaticoduodenostomy isrecommended. Iflocatedonthebodyortailofpancreas,distal partial located intheduodenumorpancreatic head(gastrinomatriangle);pylorus-preserving distantmetastasis.Mostgastrinomasare and insulinomashavetobeoperated ontoprevent gastrinomas sporadic disease,i.e.degreeofhypercalcaemia, renalstonepresenceetc.MEN-1 criteriaasin thesame hyperparathyroidismfollow primary inMEN-1 The indicationsforsurgery innature. diarrhoea whichissecretory present withproducewatery VIPomas damageoftheuppersmallbowel. andmucosal enzymes Malabsorption anddiarrhoeaarecommonpresentationsduetoacid-related inactivationof totreatment. oftenpresentasmultiplepepticulcersthatarerefractory insulinoma. Gastrinomas by often multipleandrecuraftersurgery. followed isthemostcommon ofthePETs, Gastrinoma are patients.They percent of MEN-1 Pancreaticoduodenal endocrinetumoursoccurin50–60 habitus). neuromas accompaniedbyamarfanoid andcharacteristicfacial andmucoid thyroidcarcinoma,phaechromocytoma and 2b(medullary hyperparathyroidismandphaeochromocytoma) carcinomaofthyroid,primary into 2a(medullary germ-line mutationsinthemeningenelocatedonchromosome11, 2issubdivided whileMEN adenomas), hyperplasiaoftheparathyroidandpancreaticoduodenaltumours.Itiscausedby (presentingasprolactinoma,non-functioningpituitary involvetheanteriorpituitary glands. MEN-1 areinheritedsyndromespresentingasbenignandmalignanttumoursinvariousendocrine MEN function. A synacthentestdoesnotpituitary issuspected. value,adrenalinsufficiency nmol/Landlessthantwicethebasal is<500 cortisol administration.Iftherisein aftersynacthen(syntheticACTH) an insufficientincreaseincortisol administration.Incaseofadrenalfailurethereis minafterACTH takenagain30and60 samples and isperformed level cortisol Abasal toassessadrenalreserve. A synacthentestisperformed deficiencies presentlaterinlifeorduringperiodsofstresssuchaspuberty. genitalia inafemaleinfant.Maleinfantsarediagnosedlaterwithvirilisingfeatures.Milderenzyme which canleadtoincreasedproductionofandrogenicprecursors.Itpresentasambiguous secretion, ACTH andexcessive 95 percentofcases.Theconditionismarkedbyreducedcortisol 21-hydroxylase livebirths. in isthedeficientenzyme affected withanincidenceof1in5000 Congenital adrenalhyperplasiaisanautosomalrecessivecondition.Malesandfemalesareequally y nacthen test y perplasia 381

49: ADRENAL GLANDS AND OTHER ENDOCRINE DISORDERS BREAST AND ENDOCRINE ➜ ➜ ➜ 3A 2B 1D 5A 4D 3E 2B 1C matchingquestions Answers: Extended 16. C,E 15. B,C,E 382

2. H 1. H Neuroendocrine tumours Neuroendocrine tumoursarisefromthediffuseneuroendocrinecellsthatcanbefoundin Cushing’s disease is ACTH-dependent. It is caused by pituitary orhypothalamiclesions. Itiscausedbypituitary Cushing’s disease isACTH-dependent. frequent associations. are obesity, hirsutism,easybruisingand proximalmuscleweakness.Diabetesandhypertension Ittypicallycausesamoonface,truncal cortisol. Cushing’s syndromeisassociatedwith excess production. incasesofectopicACTH severe isparticularly Thedegreeofhypokalaemia Oat cellcarcinomaofthebronchusproducesACTH. Friderichen syndrome. Meningococcal sepsiscanresultinadrenalinfarctionandhaemorrhage andiscalledWaterhouse– biosynthesis ofcortisol. (21-hydroxylase) ofenzymes Itoccursduetodeficiency hyperkalaemia. requiredinthe resembleanadrenalcrisiswithhypotension,hyponatraemia and cases. Exacerbations insevere Itpresentsassalt-wasting livebirths. Congenital adrenalhyperplasiaisseenin1:2500 isprolonged. Adrenal haemorrhagecanoccurincaseswhentheINR hormone(MSH). melanocyte-stimulating duetoitssimilarity bilateral adrenalectomy. hasadirectactiononmelanocytes ACTH stimulation(lackofnegativefeedback),following ACTH Nelson’s diseaseoccursduetoexcessive such astype1diabetes,perniciousanaemia,prematureovarianfailureandautoimmunethyroiditis. Addison’s diseaseisautoimmuneadrenalfailure.Itassociatedwithotherconditions , wheezingetc.despitethepresenceoflivermetastasis. don'thavetypicalfeaturesof Thisiswhythey isnotelevated. not secreteserotoninand5-HIAA remain indolentforalongtime.Diagnosisisoftenmadebyradiography. Foregut tumoursdo tumours (whichpresentwithcoughandobstruction)thaninperipherallylocatedtumours,which Ten occursinthelung.Thediagnosisiseasiercentrallylocatedlung percentoftheNET and15nearly 25percentinthesmallbowel percentintherectum. arelocatedintheappendix, percentofNETs aserotoninmetabolite.Forty acid (5-HIAA), 5-hydroxyindoleacetic ofthejejunumandileumexcrete neuron-specific enolase.NET cells secretedifferentneuroendocrinemarkerssuchassynaptophysin,chromograninand tree,urogenitalsystemandthepancreas.The ofthebronchi,stomach,gut,biliary mucosa y y poadrenalism percortisolism ➜ ➜ ➜ 1B 5B 4D 3A 2C 1E 5A 4B 3C 2D 1E 5E 4C

5. H 4. Carcinoid tumours 3. Sexhormones elevated insulin levels (despite hypoglycaemia) and C-peptide levels isdiagnostic. (despitehypoglycaemia)andC-peptidelevels insulinlevels elevated Demonstrationof An isletcelltumoursecretinginsulin(insulinoma)isassociatedwithMEN. be performed. If increased cortisol is excreted in the urine, an overnight dexamethasone testcan in theurine,anovernightdexamethasone isexcreted If increasedcortisol be performed. must cortisol Inthefirstinstancea24hurinary excess. The symptomsaresuggestiveofcortisol Hypoglycaemia occursininsulin-producing pancreaticcarcinoid. bronchialcarcinoids. Ectopic humoralsyndromesoccurinACTH-producing tricuspid regurgitation. stenosisand lesionssuch as pulmonary failure.Therearepredominantright-sidedheart heart results inendocardialfibrosisand Chronic increaseinmetabolitessuchas5-hydroxytryptamine cutaneous flushinganddiarrhoea. impede hepaticclearanceofmetabolitessuchasserotoninandbradykinins. Thiscanresultin tumoursfromtheileum,stomachandduodenumcanmetastasise totheliverand Carcinoid appendix, ileum,rectum,lungs,gonadsandduodenum. tumoursarisefromtheenterochromaffincellsandaremostcommonlyfoundin Carcinoid testicularhormones(testosterone). luteinising hormone(LH))andlow (follicle-stimulatinghormone, pituitary Hypogonadotrophic hypogonadismisassociatedwithlow andalpha-fetoprotein(AFP). Germ celltumourssuchasteratomaproduceHCG steroidpathway,resultinginvirilisingeffectstheneonate. sex the precursorsareshunteddown Thecortisol 21-Hydroxylase causeshypocortisolism. deficiency such asandrostenedione. Arrhenoblastomaisanovariantumourwhichalsoproducesotherandrogens and masculinisation. oftestosterone Malignant tumoursoftheadrenalsandovariescanresultinmarkedelevation changes intheovaries. Ultrasound demonstratesthecystic the testosterone,prolactinandLHcouldbemildlyelevated. ovariansyndromepresentswitholigomenorrhoea,weightgainandhirsutism.Biochemically Polycystic hyperaldosteronismisacauseinapproximately1percentofallhypertensives. Primary y pertension andh y pokalaemia 383

49: ADRENAL GLANDS AND OTHER ENDOCRINE DISORDERS BREAST AND ENDOCRINE 5A 4C 3E 2D 384 syndrome isadifferentialdiagnosisinthiscase. resistanttosupplementation.Bartter abusecanpresentashypokalaemia Diuretics andlaxative oftherenin–angiotensin–aldosteroneaxis. with evaluation control)mustbeinvestigated, (requiringoverfourmedicationsachieving Resistant hypertension in thetreatmentandcancausemildhypokalaemia. Diureticssuchasthiazidesareoftenused This isnotanunusualfindinginessentialhypertension. imagingmustbeconsidered. biochemicaldiagnosisofhypercortisolism, Following testconfirmsthesourceofloss. of steroid).Ahigh-dosedexamethasone administration despiteexogenous (failuretosuppresscortisol toconfirmtheexcess be performed 50 The breast

Multiple choice questions ➜ Investigations in breast 4. In a patient with nipple discharge cancer which of the following statements are true? 1. Which of the following statements are A Clear, serous discharge may be true? physiological. A Five per cent of breast cancers B Bloodstained discharge occurs in are missed by population-based carcinoma, duct ectasia and duct mammographic screening. papilloma. B Ultrasound can also be used as a C Mammography is an important screening tool. investigation. C Magnetic resonance imaging (MRI) can D Microdochectomy is the treatment once be a useful imaging tool. cancer has been excluded. D Fine-needle aspiration cytology (FNAC) E Paget’s disease causes discharge from and core biopsy are equally useful the surface. diagnostically. 5. In breast carcinoma, which one of the ➜ Benign breast disease following statements is false? 2. Which of the following statements are A Ductal carcinoma is the most common false? variant. A Benign breast disease is the most B Lobular carcinoma occurs in 15 per cent. common cause of breast problems. C There may be a combination of lobular B Lipoma is a common condition of the and ductal features. breast. D Colloid, medullary and tubular C Traumatic fat necrosis can be mistaken carcinomas carry a poor prognosis. for a carcinoma. E Paget’s disease is a superficial D 30 per cent of breast cysts recur after manifestation of an underlying breast aspiration. carcinoma. E Non-cyclical mastalgia is more common 6. In the treatment of breast cancer, in postmenopausal women. which of the following statements are ➜ Breast cancer false? A There is a higher rate of local recurrence 3. Which of the following conditions after conservative surgery and have an increased risk of breast radiotherapy. carcinoma? B After mastectomy, radiotherapy to the A Breast cyst chest wall is not indicated. B Duct ectasia C Sentinel lymph node biopsy should be C Florid hyperplasia done in clinically node-negative disease. D Atypical ductal or lobular hyperplasia D Besides treating the patient, the role of E Fibroadenoma. axillary surgery is to stage the patient accurately.

385 BREAST AND ENDOCRINE ➜ Extended matchingquestions Extended Chooseandmatchthecorrectdiagnosiswitheachofscenariosgivenbelow: 386

10 10 9 8 7 6 5 4 3 2 1 J I H G F E D C B A Benign breast disease E Phyllodes tumour Fibroadenoma Galactocele Breast cyst andinvolution(ANDI) Aberrations ofnormaldevelopment Duct ectasia Mondor’s disease Tuberculosis Breast abscess Fat necrosis adjuvant chemotherapy. risk node-negativewomenshouldhave Lymph node-positivewomenandhigher- A 42-year-oldwomanpresentswithalargeirregularlump,about12 cmindiameter,her with herperiods,whichareregular. a lumpinherrightbreast6weeks ago andfeelsthatthesizeoflumpwaxeswanes A 38-year-oldwomancomplainsof painfulbreastsofsome6months’duration.Shenoticed shehasanirregular,firmlump deeptotheareolawhichlooksindrawn. examination A 44-year-oldwomancomplainsof greenishnippledischargeonandofffor4months.On diffuse lumpunderlyingtheareola. shefeelshotwithared,tender, induratedand her thirdchild2weeksago.Onexamination A 30-year-oldwomancomplainsofatender,painfulrightbreastwithfever. to Shegavebirth time. on herbreastbygrandchildataboutthesame tender lumpwhichisnotmobile.Onquestioningabouttrauma,sherecalls havingbeenhit womannoticedalumpinherleftbreast4weeksago. She hasafirm,slightly A 50-year-old 3 weeksago.ThishasnotchangedsinceherperiodaweekItismobile andtender. womancomplainsofalumpinherrightbreastthatshe noticedaccidentally A 40-year-old A 28-year-oldlactatingwomannoticedatenderlumpdeeptoherareola. Thelumpiscystic. shehasatenderstring-likebandwithoverlyingprominentveins. examination A 45-year-oldwomancomplainsofpaininherleftbreast2weeks’duration.On discharging abscess,whichtendstoclearupandrecuragain. months’duration.Thereisnopain.Inheripsilateralaxillashealsohasa breast ofseveral complainsofalumpinherright woman,recentlyarrivedintheUK, Asian A 40-year-old 3 weeksago.Shecanmovethelumpaboutwithinbreasttissue. A 25-year-oldwomanpresentswithadiscretelumpintheleftbreast.Shefounditaccidentally about toulcerate. mobileandisstretchingtheskin right breast,of4months’duration.Thelumpisvery F chemotherapy. There isnoroleforpreoperative ➜ ➜ ➜ ➜ 2I 1J matchingquestions Answers: Extended 6.B,F 4.A,B,D,E 5. 3.C,D D 2.B,E 1. A,C Answers: Multiplechoicequestions

1. Benignbreast disease Breast cancer Benign breast disease Investigations inbreast cancer wishes removal. or ifthepatient triple assessment, itneedstoberemovedonlyincase ofinconclusivecytology This 25-year-oldlady hasafibroadenoma–anincidental, mobilelump.Afterreassurance with orsimplemastectomydepending uponthesize. excision 1840 diseaseofbreast.Aftertripleassessment,itistreatedbyenucleation, widelocal assero-cystic SirBenjaminBrodie(1783–1862,sacrcoma. Surgeon,StGeorge’s Hospital,London)describeditin nora phyllodesalthoughitisneithercystic stretched. Thetumourisalsocalled cystosarcoma withtheoverlyingskin in appearance.Thesizecanvary. Ithasanirregularbosselatedsurface This ladyhasaPhylloidestumour,socalledbecauseitisleaf-like(Phylloides inGreek=leaf-like) percentofcases. up to80 tobecarriedout.Thisapproachissuccessfulin surgery the tumourtoenablebreast-conserving lymphovascularinvasion. nodes andextensive recurrence arehigh,suchasthosewithlargetumours,patientsnumbersofpositive After mastectomy,radiotherapytothechestwallisindicatedinpatientswhomrisksoflocal receptor statustobedeterminedhelpinneoadjuvantchemotherapy. the diagnosis, differentiatesbetweenductalcarcinomainsituandinvasivecancer,allows false-negatives dooccur. Acorebiopsygivesfarmoreinformation:itadefinitivepreoperative However, oftheoperatorandcytologist. increaseswiththeexperience diagnosis; theaccuracy carriedout.Itisthebestimagingtechniqueforbreastswhichhavehadimplantsinserted. conservation useful imagingmodalitytodistinguishscartissuefromrecurrenceinapatientwhohashadbreast isanextremely lymphnodes.MRI is alsobeingusedtolookforandbiopsyimpalpableaxillary andasolidlesion.Thistechnique in youngwomenwithdensebreaststodistinguishbetweenacyst mammography arebeingusedasmoreadvancedtechniques. in theolderagegroupasbreastbecomeslessdense.Digitalmammographyandtomo- high-amperage X-rays.Theradiationdoseisabout0.1 cGy. Thisinvestigationismoresensitive ittolow-voltage, by placingthebreastindirectcontactwithultrasensitivefilmandexposing cancers aremissedonpopulationscreening.Inmammography,soft-tissueradiographstaken thepresenceofacarcinomaas5percentbreast A normalmammogramdoesnotexclude Fine-needle aspiration cytology is the least invasive technique of obtaining a cytological istheleastinvasivetechniqueofobtainingacytological Fine-needle aspirationcytology useful Ultrasound isnotusefulasascreeningtoolandhighlyoperator-dependent.Itparticularly Primary neoadjuvantchemotherapyisusedforlargeoperabletumours, theaimbeingtoshrink Primary 387

50: THE BREAST BREAST AND ENDOCRINE 10F 9E 8B 7A 6G 5H 4D 3C 388 The discharging sinus in her axilla is from TB lymphadenitis.Thediagnosisisconfirmedby The dischargingsinusinheraxillaisfromTB countries. intheindigenousBritishpopulationbutstillcommondeveloping is almostunknown .It Tuberculosis cervical orTB TB ofthebreastisusuallyassociatedwithpulmonary (TB) conservative measures istheoverallmanagement. conservative by reassurancesupplementedby followed atadifferenttime ofthemenstrualcycle Examination fibrosis, hyperplasiaandpapillomatosis. Triple carcinoma. assessmentisdonetoexclude formation, from thatreferredthechestwall. Theunderlyingpathologicalfeaturesarecyst andthepainshouldbedistinguished orafibroadenoma.Mastalgia maybequitedisabling a cyst with theperiods;andsometimesadiscretelump,whichmaybe quadrant; changesbeingcyclical Thesymptomsaregeneralisedlumpiness ofbreasts,lessoftenconfinedtoone involution (ANDI). and BreastClinicdescribed aconditioncalledaberrationsofnormaldevelopment The Cardiff terminal ducts. bytripleassessment.TreatmentA carcinomamustbeexcluded isbysurgicalremovalofall The conditioniscausedbydilatationandinflammationoftheducts,leading toperiductalmastitis. deformed nipplewithretraction,andasubareolarinduratedmasswhich maymimiccarcinoma. This patienthasthetypicalfeaturesofductectasia–nippledischarge(not bloodstained), ‘antibioma’ –asterile,brawny,oedematousswellingmayformwhich cantakeweekstoresolve. a radialincisionanddrainagearecarriedout.Ifantibioticisusedwithout drainageofthepus,an iftheskinisthinnedand abscessisabouttoburst, comprise thetreatmentofchoice.Rarely, tothebreastandanalgesia cover, repeatedaspirationswithultrasoundguidance,goodsupport stage, cellulitisispresent.Anappropriateantibioticusedafteraspirationofpus.Under painful,tenderandindurated.Intheearly aureus. Asegmentofthebreastisinflamed,red,very A breastabscessismostoftenassociatedwithlactationandusuallycausedbyStaphylococcus carcinoma. Thefulltripleassessmentshouldbecarriedouttosureofthediagnosis. the traumamighthavedrawnpatient’sattentiontopresenceoflumpwhichisa oftrauma,thediagnosismaynotbefatnecrosis,because retraction. Justbecausethereisahistory beskin-tetheringandnipple lump appearswhichmayfeellikeacarcinoma.Thereeven Traumatic fatnecrosisusuallyoccursinmiddle-agedwomenafterblunttrauma.Apainless and psychologicalrelief. willgivebothphysical whichitcompletelydisappears following ofthecyst assessment. Aspiration bytriple malignancy Itisessentialtoexclude This womantypicallysuffersfromabreastcyst. ofherlactation.Confirmationisbyaspirationmilk. start lumpwhichshedatesfromthe This ladyhasagalactocele–rarecondition;sub-areolarcystic anunderlyingoccultcarcinomacausinglymphaticpermeation. mustbemadetoexclude efforts subcutaneous bandmadeclinicallyobviousbystretchingtheskinonraisingarm.Alldiagnostic Mondor’s disease(HenriMondor,1885–1962, aParisiansurgeon)isassociatedwithcord-like veinsofthechest.Thecondition,called This patientsuffersfromthrombophlebitisofthesuperficial persistent residualinfection. triple assessment.Treatment for isantituberculouschemotherapy,whilemastectomyreserved PART 9 Cardiothoracic

51 Cardiac surgery 391 52 The thorax 397 This page intentionally left blank 51 Cardiac surgery

Multiple choice questions ➜ Cardiopulmonary bypass coronary characteristics helpful in 1. Which of the following statements is planning surgery. false? D An estimate of the operative mortality A Cardiopulmonary bypass has brought can be calculated using risk scoring about remarkable progress in cardiac systems. surgery. E The left internal mammary (or thoracic) B The cardiopulmonary bypass circuit artery has a better 10-year patency rate consists of a venous reservoir, than long saphenous vein grafts. oxygenator, heat exchanger, filter and ➜ roller pump. Valvular heart disease C Cardiopulmonary bypass is not used 4. Which of the following statements are outside cardiac surgery. false? D Patients require full-dose heparin with A The indication for surgery in mitral valve the use of cardiopulmonary bypass. disease (regurgitation and stenosis) is E Cardiopulmonary bypass can cause severe symptoms, as assessed by the serious systemic complications. New York Heart Association (NYHA) functional classification. ➜ Ischaemic heart disease B Acute and chronic mitral regurgitation 2. Which of these is not a risk factor for have similar pathophysiology and clinical ischaemic heart disease? presentation. A Smoking C Mitral stenosis causes left atrial B Obesity enlargement, pulmonary hypertension, C Advancing age atrial fibrillation and haemoptysis. D Reduced physical activity D Mitral valve repair is superior to mitral E Female gender. valve replacement but more technically demanding. ➜ Coronary artery bypass E All of the above. surgery 5. Which of the following statements is 3. Which of the following statements are false? true? A Aortic stenosis is associated with a risk of A Coronary artery bypass surgery affords sudden death related to the severity of symptomatic and prognostic benefit for stenosis. subsets of patients with ischaemic heart B Rheumatic heart disease can cause both disease. aortic stenosis and aortic regurgitation. B Myocardial infarction is an indication C Distinguishing between aortic stenosis for emergency coronary artery bypass and chronic aortic regurgitation is usually surgery. difficult on clinical grounds alone. C Selective coronary angiography is D Aortic valve repair is not a common the ‘gold standard’ investigation and practice like mitral valve repair. demonstrates coronary stenosis and E None of the above.

391 CARDIOTHORACIC ➜ ➜ ➜ Extended matching questions Extended statementsis Whichofthefollowing 7. Whichofthesestatementsaretrue? 6. Choose and match the most appropriate diagnosis or investigation for the clinical scenariosbelow: Choose and matchthemostappropriatediagnosisorinvestigation fortheclinical 392

1 D C B A 1. Cardiac disease D C B A Congenital heartdisease E D C B A Prosthetic heartvalves recent myocardial infarction.Whatisthelikelydiagnosis? congestion,andelectrocardiographyconfirms cardiomegaly andpulmonary radiograph shows loudapicalpansystolicmurmur. crepitations andanew bilateralbasal Chest mild exertion, dyspnoeaon A 72-year-oldfemalewithchronicstable anginapresentswithnew-onset Mitral regurgitation disease Ischaemic heart scanofthechest Computed tomography(CT) Echocardiography atrioventricular andsubarterial. are perimembranous,muscular, Four typesofatrialseptaldefects is referredtoasFallot’s tetralogy. stenosis andrightventricularhypertrophy pulmonary defect, overridingaorta, ofventricularseptal The coexistence failureininfancy. and usuallycauseheart diseases, heart common thatcyanotic diseasesaremore heart Acyanotic the heart. abnormal connectionofbloodvesselsto runs inparalleltosystemiccirculation,or circulationthat shunt orapulmonary diseases, andresultfromaright-to-left comparedwithacyanotic complex diseasesareoftenmore Cyanotic heart prosthetic valveendocarditis. Biological valvesarenotatriskof of prostheticvalve. Age istheonlydeterminantofchoice prosthetic valves. Lifelong anticoagulationisrequiredforall biological valves. Mechanical valvesaremoredurablethan patient (autograft). humans (allograftorhomograft)andthe animals (xenograftorheterograft),dead Biological valvesareobtainedfrom false? ➜ . Whichoneofthesestatementsisfalse? 9. heart Whichofthesearenotacyanotic 8.

E D C B A pericardial disease The thoracicaortaand E D C B A E Patent foramenovale Transposition ofthegreatvessels drainage Total venous anomalouspulmonary Patent ductusarteriosus diseases? leading toEisenmenger’ssyndrome. occur ifthedefectsarenotclosed, vascular resistance.Progressivechanges andpulmonary bloodflow pulmonary left-to-right shuntcausesanincreasein In septaldefects(atrialandventricular)a and bloodpressure(cardiactamponade). decreases venousreturn,cardiacoutput theatrialpressure.This pressure exceeds compression oftheatriawhenthis in intrapericardialpressureand Pericardial effusioncausesanincrease repair.require emergency dissection Stanford typesAandBaortic repair.descending aneurysm dysfunction aresomecomplicationsof Paraplegia, renalfailureandventricular involved. thoracic aorta ofthe dependsonthepart aneurysm inthoracic Indication forsurgery connective tissuedisorders. areatherosclerosisand aneurysm Common causesofthoracicaortic Coarctation oftheaorta. ➜ ➜ ➜ 3.A,C,D,E 2. E 1. C Answers: Multiplechoicequestions

common arterial conduit, and the graft of choice for the left anterior descending artery. Venous conduit,andthegraftofchoicefor theleftanteriordescendingartery. common arterial isthemost artery vein isthemostcommon veinconduitusedandthe left internalmammary informed consent. ofapatient’s formspart the riskofoperativemortality. Thispredictedoperativemortality whichallocate scorestoriskfactorsinapatient,isroutinelyusedestimate (EuroSCORE), forsurgery. astrategy with otherimagingtechniques,thatisusefulindeveloping angiographyprovidescomprehensiveinformation, notpossible Selectivecoronary also revealed. andfistula,are aneurysm artery suchascoronary lesionsthatcanaffectoutcomeofsurgery, Rare tohelpplansurgery. anestimation ofleftventricularfunction. Italsoallows of thedistalcoronary ofstenosis,andassessment andcomplexity ofthesite,extent size,evaluation artery of coronary • • • • Coronar Ischaemic heartdisease Cardiopulmonar 4 3 2 Survival benefit of surgery overangioplastyandstentinghasbeendemonstratedinpatientswith: benefitofsurgery Survival bypasssurgery. artery gender isassociatedwithaworseoutcomeofcoronary thatfemale studieshaveshown disease.However, Male genderisariskfactorforischaemicheart malformations. arteriovenous vascular tumoursandextensive it facilitatestheresectionofrenalandhepatictumoursinvadinginferiorvenacava,highly domain, itcanbeusedinthemanagementofpatientsoutsidecardiacsurgery. For example, remainsits principal and lungsneedstobetemporarilyinterrupted.Whereascardiacsurgery whenthefunctionofheart support bypassprovidescardiacandrespiratory Cardiopulmonary coronary artery stenosiswithpoorleftventricularfunction. artery coronary descending andcircumflex) leftanterior (rightcoronary, arteries >70 percentstenosisofallthreemajorcoronary >70 percentstenosisoftheproximalleftanteriordescending artery percentstenosisoftheleftmainstem >50 Venous and arterial conduits are used for coronary artery bypasssurgery. The longsaphenous artery Venous conduits areusedforcoronary andarterial OperativeRiskEvaluation Risk scoringsystemsliketheEuropean System forCardiac anatomyand enablesthevisualisation angiographydemonstratescoronary Selective coronary descending thoracic aorta. Whatistheappropriateinvestigation? descending thoracicaorta. ofthe On routinechestradiography,a78-year-old maleissuspectedofhavingananeurysm bleeding upto5percent.Whatconditionisthisprocedurefor? upto30percentandsignificant dysfunction oflessthan2percent,cardiacarrhythmia ofabout2percent,thereisariskneurological addition toaprocedure-relatedmortality maleisinformedduringconsentforaprocedurecardiacdiseasethat,in A 66-year-old patient? mid-diastolic murmur. Whatprocedureisappropriateatthisstageinthemanagementof widepulsepressure,water-hammeranda reveals dilatation oftheascendingaorta syndromewithaneurysmal ofa35-year-oldmanwhohasMarfan’s Clinical examination y arter y b y b y pass surger y pass y 393

51: CARDIAC SURGERY CARDIOTHORACIC 5. C ➜ 4.A,B 394

sternal border. Theelectrocardiographicand radiologicalappearancesarenotusually distinctive. regurgitationishigh-pitched, mid-diastolicandbestheardattheleft characteristic murmurofaortic include: stenosis. areaandradiatingtothecarotids,istypicalofaortic in theaortic stenosis. dyspnoeaandangina.Syncope isalsocommonwithaortic exertional regurgitation,patientsdevelop andacuteaortic regurgitation. Withcardiacdecompensation, ventricular dilatationtoaccommodatetheincreasedleftvolume loadduetoaortic stenosis,andleft obstructionofaortic in wallthickness)toovercometheleftventricularoutflow (increase mechanismsincludeventricularhypertrophy occurs.Compensatory decompensation regurgitationareusuallyasymptomaticuntilcardiac stenosisandchronicaortic Aortic murmur isaudibleinbothacuteandchronicmitralregurgitation. occur, andleftventricularenlargementbecomesapparentradiologically. Aloudapicalpansystolic andatrialfibrillation(duetoleftenlargement) orthopnoea fatigue, dyspnoeaonexertion, Thensymptomslike congestionandleftventricularfailuredevelop. asymptomatic untilpulmonary oedema.Chronicmitralregurgitationisusually ofpulmonary clinical andradiologicalevidence mechanismsareoverwhelmed. congestive cardiacfailurewhenthecompensatory ultimatelycauses congestionandoedema.Chronicmitralregurgitation,however, to pulmonary venouscirculation,leading atrial pressureandabackincreaseaffectingthepulmonary intoasmallleftatriumcausessuddensurgein changes. High-volumeretrogradeflow timeforthesecompensatory increase inpressure.Acutemitralregurgitationdoesnotallow asudden circulationisprotectedfromdeveloping aresultthepulmonary pressure increase.As withoutsubstantial andleftatrialdilatation,todevelop ventricular dilatationandhypertrophy, adaptivechangeslikeprogressiveleft inchronicmitralregurgitation,allowing increases slowly ejection leadstoanincreaseinleftventricularvolumeload.Theamountofretrograde forsurgicalcorrection.Inmitralregurgitation,retrograde function, andconsequentlythestrategy • • • • • • Valvular heartdisease hasa10-year percent. artery rateof90 The leftinternalmammary patency graftsarebetterprotectedagainstatherosclerosis. vein.Bycomparison,arterial the longsaphenous grafts aresusceptibletoatherosclerosis,resultingina10-year percentfor rateof50–60 patency stenosis (moderate and severe stenosiswithvalveareaof stenosis (moderateandsevere of regurgitation aretheotherindications.For mitralstenosis,theotherindicationsareseverity onsetof regurgitation, progressiveleftventriculardilatationand/ordysfunctionandsevere symptoms,therefore,arenottheonlyindicationforsurgery.(prognostic). Severe For mitral symptomsandtoimprovesurvival inmitralvalvediseasetorelieve isperformed Surgery uvular pulsation (Müller’ssign). uvular pulsation (Traube’s‘pistol shot’soundonauscultationof thefemoralartery sign) (Corrigan’ssign) ofneckarteries visible pulsation headbobbing(deMusset’ssign) pulsatile inthenailbed(Quincke’ssign) pulsation visible capillary collapsing pulse(water-hammerpulse) In aortic regurgitation also, the apex beatisoften visibleanddisplacedlaterally. regurgitationalso,theapex The In aortic The distinctive features of aortic regurgitationareduelargelytoawide pulsepressureand The distinctivefeaturesofaortic withclinicalfindings.Aharshsystolicejection murmur,heardloudest Marked differencesexist Acute mitralregurgitationpresentswithsuddenonsetandrapidlyprogressivedyspnoea and The onsetofmitralregurgitationhasimplicationsforchangesincardiacmorphology ≤ 1.5 cm 2 ) andsystemicembolisation. ➜ ➜ ➜ 9. D 8.B,C 7. D 6.A,B

The thoracicaortaandpericardial disease Congenital heartdisease Prosthetic heartvalves Biological valvesaremadeorobtainedfromanimalhumantissue.While include: dissection initially. intypeBaortic is bestmanagedconservatively Theindicationsforsurgery according totheStanfordclassification. Type buttype B surgicalintervention Arequiresemergency istypeB, theoriginofleftsubclavianartery) (beyond that originatesinthedescendingaorta istypeA,andone dissectionoriginating(defect orintimalflap)intheascendingaorta Aortic ofFallot. diseaseaftertetralogy congenital heart period,and,ingeneral,thesecondmost common cyanotic inthenewborn causing cyanosis mixing ofblood.Transposition disease, ofthegreatvesselsismostcommon congenitalheart defectslikeforamenovaleandventricularseptaldefect provide channelsfor with life.Associated bloodtothesystemiccirculation,thisconditionisnotcompatible circulation anddeoxygenated bloodisconfinedto thepulmonary oxygenated to runinparallelratherthanseries.As andsystemiccirculation thepulmonary transposition ofthegreatvessels.Thisarrangementallows flow.high pulmonary with isminimaleven atrialseptaldefect,cyanosis after thefirstweekoflife.Ifthereisacoexistent sinusorrightatrium,insteadoftheleftatrium.Thetypicalpresentationiscyanosis cava, coronary venous circulationusuallythroughacommunicationwiththeinferiorvenacava,superior venousreturnisintothesystemic venousdrainage,pulmonary In totalanomalouspulmonary ofshunt. pressureandreversal highpulmonary bloodflow, pulmonary right shuntoccursandinthelongtermcancauseEisenmenger’ssyndromefromhigh-volume four typesofventricularseptaldefects.Inbothatrialanddefects,aleft-to- described are:ostiumsecundum,primumandsinusvenosus. Three maintypesofatrialseptaldefect(defectintheseptumbetweenrightandleftatria) infection andthrombosis/thromboembolism. risksofparavalvularleak, infection. Mechanicalvalvesandtheotherbiologicalhavelow autografts (entirelyhumantissues)arelesssusceptibletothrombosis/thromboembolismand related tothevalveandinfectionof(prostheticendocarditis).Homografts anaemia,thrombosisofthevalve,thromboembolism haemodynamic instabilityorhaemolytic vulnerable. Theneedtoreplaceaprostheticvalvealsoarisesifthereisparavalvularleakcausing to structuralfailure,andhencetheirdurabilityislimited.Younger patientsareprobablymore implanted. Biologicalvalvesarenon-thrombogenicsodonotrequirelifelonganticoagulation. heterografts aremostcommonlyused;specifically,stentedthefrequently valves andaremountedonaframe(stented)orframeless(stentless).Amongbiologicalvalves, configuration, valvesmadefromanimaltissue(heterograft)areconstructedtoresemblehuman andimplantedwithoutmajoralterationofthe humans (homograftandautograft)areharvested The defects listed in D (perimembranous, muscular, atrioventricular and subarterial) arethe The defectslistedinD(perimembranous,muscular,atrioventricularandsubarterial) Biological valves,unlikemechanicalaresubjecttodegenerativechangeswhichlead The aorta arises from the right ventricle and the pulmonary artery fromtheleftventriclein artery arisesfromtherightventricleandpulmonary The aorta 395

51: CARDIAC SURGERY CARDIOTHORACIC ➜ 4B 3C 2A 1D matchingquestions Answers: Extended 396

1. Cardiac disease • • • indicated, duetoacuteenlargementorprogressiveincreaseinsize over 6cm. is todeterminewhensurgery isusefulforserialassessmentoftheaneurysm CT conservatively, is oftenmanaged descendingthoracicaneurysm presence ofthrombicanbeassessed.As ofthe chest.Thesite,sizeand canbeconfirmedbyCT The diagnosisofthoracicaneurysm approximately5–6percent. mortality, for Surgery systems liketheEuroSCORE. canbecalculatedusingriskstratification complications. Anestimateoftheoperativemortality grafting, oneofthemostinvestigatedsurgicalprocedureswithwellandconsistentlydocumented bypass artery bycoronary diseaseisachieved forischaemicheart Surgical revascularisation echocardiography isvitalindecidingtheoptimaltypeofsurgicalrepair. andleftventriculardimensionsfunction.Theinformationprovided by valve morphology root,aortic ofregurgitation,thediameteraortic Echocardiography assessestheseverity investigation. warrantsfurther aneurysm regurgitationinthesettingofascendingaortic aortic rootdilatationiscommon.Thepresenceofclinicalfeatures regurgitation duetoaortic arecommonlyaffected.Aortic rootandascendingaorta and dilatationofthevessel.Theaortic medialnecrosisofthevesselwall,causingweakness syndromeisassociatedwithcystic Marfan’s valve leaflets. sothatfunctionalmitralregurgitationoccursinthepresenceofgrosslynormal leaflets apart myocardial ischaemiacausesleftventriculardysfunctionanddilatation,whichpullsthemitral muscledysfunctionorrupture.Global ischaemia andmyocardialinfarctioncausepapillary (ischaemic mitralregurgitation)asaresultofitslocalorglobaleffects.Regionalmyocardial still causemorphologicalcomplicationsandremainsacommonofmitralregurgitation diseasecan Even withstablesymptomsduetoadequatemedicaltherapy,ischaemicheart neurological complications. syndromeresultinginorgandysfunction,suchasrenalfailure,orlimband malperfusion scan) oflesiononserialchestimaging(radiographorCT progressive expansion worsening chestpainindicatingimminentrupture mitral valvediseaseisassociatedwithhigheroperative 52 The thorax

Multiple choice questions ➜ Anatomy and physiology B Compared with non-small-cell cancer, 1. Which of the following statements is small-cell lung cancer, formerly known false? as oat cell cancer, is less common, A The lungs are derivatives of the primitive metastasises early and is less amenable foregut. to surgery. B The left lung has more lobes and C Finger clubbing and hypertrophic segments than the right lung. pulmonary osteoarthropathy, sometimes C Bronchial arteries arise directly from the described as clinical features of lung thoracic aorta to provide systemic blood cancer, are usually incidental findings supply to the trachea and bronchi. and not due to primary lung cancer. D Anatomical differences between the D The appropriate treatment strategy is right and left main bronchi favour the dependent on tumour type, tumour inhalation of foreign bodies into the right. stage, and the general fitness and lung E Pulmonary function tests assess the function of the patient. functional capacity, the severity of E Late survival has a direct relationship pulmonary disease and help to predict with the tumour stage at the time of response to treatment. treatment. ➜ Pleural space disease ➜ Investigating bronchial 2. Which of the following statements are carcinoma true? 4. Which one of these statements is true? A Tension pneumothorax can cause A Chest radiograph yields very useful haemodynamic compromise. information about primary lung cancer. B Pleural effusions due to cardiac B Computed tomography (CT) is only failure, renal failure, hepatic disease, useful for guiding fine-needle aspiration. inflammatory disease and malignancy C Positron emission tomography (PET) have different protein content. has high specificity for bronchial C Infection of the pleural space (empyema) carcinoma. results from iatrogenic and non- D Sputum cytology has a high sensitivity. iatrogenic causes. E Invasive procedures such as D Video-assisted thoracoscopic surgery mediastinoscopy, mediastinotomy and (VATS) plays a major role in the thoracoscopy are not staging procedures. management of pleural space diseases. E Chest drains are no longer critical to the ➜ Non-malignant conditions management of chest disease. 5. Which of the following statements is false? ➜ Primary lung cancer A Bronchopulmonary carcinoid tumours 3. Which one of these statements is false? usually arise from neuroendocrine cells A Lifetime cigarette smoking, quantified in major bronchi, are very vascular and as ‘pack-years’, is a major risk factor for slow-growing, and, although benign, bronchial carcinoma. sometimes metastasise.

397 CARDIOTHORACIC ➜ ➜ 1. 1. B Answers: Multiplechoicequestions matchingquestions Extended Choose and match the appropriate answer for the following questions: Chooseandmatchtheappropriateanswerforfollowing 398

Anatom 5 4 3 2 1 E D C B A 1. Managementofthoracicdisease D C B five in the lower lobe. five inthelower right lungalsohas 10 threeintheupper,two inthemiddleand segmentsdistributedasfollows: separated fromamiddlelobebyhorizontalfissure.The lobes.Theupperlobeisfurther lower veintributary. branchandpulmonary artery unit withnamedbronchi,pulmonary lobe.Eachsegmentisananatomicallydefined segments intheupperlobeandfive inthelower lobebytheobliquefissure.Therearefive The leftlungisdividedintoanupper lobeandalower left upperlobelungcancerandemphysema? Which procedureisrequiredtoassessthefitnessforlungresectionof a 73-year-oldmalewith would yourecommendtothispatientattime? healthy,withnosignificantpastmedical history. Whichoption non-smoker whoisotherwise A mediastinalmassisfoundincidentallyonthechestradiographofa65-year-old female positioning thepatienttolieonoperatedside,whatprocedureisurgentlyrequired? onchestradiograph.Inadditionto amounts ofpurulentsputumandahighfluidlevel oflarge expectoration is complicatedbybronchopleuralfistula,whichpresentswithpyrexia, ofa52-year-oldmalepatient arightpneumonectomy,thepostoperativerecovery Following have? complications ofpainandpossibleribfractures.Whatprocedureisthispatientplannedto female patientbeingconsentedforaprocedureisworriedaboutthepost-procedural After readingtheinformationbookletabouthisproposedprocedure,ananxious45-year-old procedure ofchoice? noabnormalfindings.Herchestradiographisnormal.Whatthe reveals examination A middle-agedmalepatientpresentswithspontaneousrecurrenthaemoptysis,andclinical None oftheabove scanofthechest CT Bronchoscopy Chest draininsertion Thoracotomy foramen ofMorgagniisusuallyposterior, Diaphragmatic herniathroughthe andtamponade. hypoxaemia can causedeathfromhypovolaemia, Blunt andpenetratingchesttrauma tuberculosis. treatment ofbronchiectasisand hasalimitedroleinthe Surgery The rightlung,ontheotherhand,has threelobes.Anobliquefissureseparatestheupperand y andph y siolog y E surgery mainlyforcosmeticreasons. surgery are chestwalldeformitiesthatrequire Pectus carinatumandpectusexcavatum Bochdalek liesmoreanteriorly. while herniationthroughtheforamenof ➜ ➜ ➜ 4. A 3. C 2.A,B,C,D

Investigating bronchial carcinoma Primar Pleural spacedisease In tensionpneumothorax,positivepressurebuildsupinthehemithoraxasairaccumulates lymph nodestatus(Nstage).Italso facilitatespercutaneousbiopsy. to site,tumoursize( T stage),proximity tochestwallandmediastinalstructures, on chestradiograph. effectsoflungcancerseen aresomesecondary hemidiaphragm duetoinvasionof phrenic nerve effusion, distallungcollapseorconsolidationduetobronchialobstruction, andraised effectsaredetectedonchestradiograph.Pleural Most lungcancerlesionsortheirsecondary regression ofthesemuscularandskeletalabnormalitieswhenthecancer isresected. lungcancer.patients withprimary Thedirectassociationwith lungcancerisdemonstratedby are clinicalfindingsseeninsome osteoarthropathy pulmonary Finger clubbingandhypertrophic adjunctive tootherthoracicprocedures. or areusedaseithersoletherapeuticinterventions andthey of chestdrainscanbelife-saving, empyema. Itplaysamajorroleinthemanagementofthoracicdiseases. thoracic ,includingpleuraldiseasessuchaspneumothorax,effusionand from pneumonia,bronchiectasis,tuberculosis,fungalinfectionsandlungabscess. haemothorax providesafavourableculturemediumforinfection.Endogenousspreadcanoccur ofchestdrains,thoracoscopyandthoracotomy.during aspirationofeffusions,insertion Traumatic within thepleuralspace).Iatrogenicintroductionofbacteriaintospacecanoccur ontheother,increasesriskofempyema(infection environment(exogenous) the external the pleuraandbronchialsystemononehand(endogenous),between effusion. obstructsthelymphaticsystemandcausesaprotein-rich pleural of fluidandprotein.Malignancy permeabilitytocausethe accumulation diseasesincreasepleuralcapillary content. Inflammatory inprotein pressure. Thepleuraleffusionthatresultsfromreducedfluidabsorptionislow proteincontent. of pleuraleffusionwithlow (30g exudates /L ormore). on theproteinconcentration,pleuraleffusionsareclassifiedastransudates(lessthan30g/L)or by capillariesofparietalpleuraorabsorptionthevisceralpleura.Depending andhenceareductionofcardiacoutput. to theheart andshift,impairmentofvenousreturn flattening ofthehemidiaphragm,mediastinaldistortion air outofthelung.Thehighintrapleuralpressureresultsincompressionipsilaterallung, of aunidirectionalflow through abreachinthevisceralpleura,whichactslikevalveallowing The pleuralcavityissterilebutthebronchialtreenot.Abreechofbarrierbetween plasmaproteinandintravascularoncotic Renal andhepaticfailureareassociatedwithlow leadingtoincreasedproduction pressureiselevated, capillary In cardiacfailure,thepulmonary witheitherthemechanismsofpleuralfluidproduction Pleural effusionresultsfrominterference Computed tomography is central to further characterisation of primary lungcancerwithregard ofprimary characterisation Computed tomographyiscentralto further Intercostal chestdrainsarefundamentaltothemanagementofdisease.Theinsertion isalessinvasivemethodforinvestigatingand treating Video-assisted thoracoscopicsurgery y lungcancer 399

52: THE THORAX CARDIOTHORACIC ➜ ➜ 3B 2A 1C matchingquestions Answers: Extended 5. D 400

1. Managementofthoracicdisease Non-malignant conditions specialised centres. in managementofbronchopleuralfistulaisundertaken to controlthesourceofsepsis.Further ofinfectedcollectioninthe chestwarrantimmediatedraininsertion evidence fluid level) purulentsputum)andradiological(high withclinical (expectorating systemic infection(pyrexia) Invariably, thepneumonectomyspace andfluidgetinfectedinbronchopleuralfistula.Signsof communication betweencolonised bronchial treeandthesterilepneumonectomyspace. tissue fluid.Dehiscenceofthebronchialstumpoccursinbronchopleural fistulatoestablish of theribsoccurafterpneumonectomytocontract space,whichfillswith ofthehemidiaphragm andcrowding Morphological changessuchasmediastinalshift,elevation space. orextrapleural paravertebral orlocalanaesthesiadeliveredvia cathetersintothe patient-controlled analgesia(PCA) early post-thoracotomypainincludeoralanalgesia,andeitherintravenous opiatesusedas Thestrategiescommonly usedtomanage normalbreathingandgasexchange. of mobilisation, functional consequenceofpost-thoracotomypainleadstoothercomplications suchasimpairment anddifficult tocontrol.The postoperative period,therefore,thoracotomypaincanbesevere arebruisedduringribspreading. Intheearly rib fracturesoccurandtheintercostalnerves Thoracotomy involvesmusclecutting,ribspreadingandparietalpleurabreeching.Sometimes anaesthesia. Rigidbronchoscopyovercomesthesesetbacksbutrequiresgeneral clots obscurevisualisation. optimal diagnosticandtherapeuticyieldsoonafteranepisodeofhaemoptysisbecauseblood bronchoscopesmaynothave thecalibreissmallandsuctionlimited,flexible tissue biopsies.As bronchoscope canbeadvancedintosegmentalbronchiandisusefulforobtainingsputum treatment.Theflexible lesion, thepotentialtoobtainbiopsiesortreat,insightplanfurther indirect communicationwiththebronchialtree.Bronchoscopyaffordsabilitytovisualise system.Theunderlyingpathologiesareusuallyindirector and malignant)lesionsofpulmonary trauma,infectionorneoplastic (benign Haemoptysis iscommonlycausedbybronchopulmonary dome ofthediaphragm. costal attachmentsofthediaphragm,andforamenBochdalek,whichliesposteriorlyin congenital sites,namelytheforamenofMorgagni,whichisananteriordefectbetweensternaland Non-traumatic herniationofabdominalviscerathroughthediaphragmcommonlyoccursattwo tumour spread(staging)andsometimesforestablishinghistologicaldiagnosis. forperipherallungcancers. tumourcellswhichcanbelow withexfoliated a sample update ofFDG. avid lesionsshow high metabolism,soinadditiontocancer,infectionsandotherinflammatory Mediastinoscopy, mediastinotomy and thoracoscopy are frequently used to assess the extent of Mediastinoscopy, mediastinotomyandthoracoscopyarefrequentlyusedtoassesstheextent hasahighfalse-negativeratebecauseitreliesontheprobabilityofobtaining Sputum cytology bylesionswith (FDG) highuptakeoffluorodeoxyglucose Positron emissiontomographyshows 4D 5E information. Ingeneral,apatientisfitforlobectomyiftheFEV • • • • >2.0 L. where possible,toobtainaguidedpercutaneousbiopsyforhistologicaldiagnosis.Onthebasis to definethesite,size,natureandattachmentsofmasssurroundingstructures,and, ofthechest investigationwithCT further A mediastinalmassonchestradiographdeserves mediastinal masses in the different parts ofthemediastinumare: mediastinal massesinthedifferentparts of thelocationinmediastinum,possiblecausemasscanbesuspected.Common volume in1sec(FEV rate,forcedexpiratory flow functiontestisnecessary. Peak expiratory can tolerate,pulmonary ofresectionthey In ordertodeterminethefitnessofpatientsforlungresection,andextent posterior mediastinum – neurogenic tumours, cystic lesionsandmesenchymaltumours. posterior mediastinum–neurogenictumours,cystic lesions,lymphomaandmesenchymaltumours middle mediastinum–cystic anterior mediastinum–thymoma,lymphomaandgermcelltumour superior mediastinum–lymphoma,thyroidandparathyroid 1 ), forced vital capacity (FVC) and arterial bloodgasesprovideuseful andarterial ), forcedvitalcapacity(FVC) 1 is>1.5 Landpneumonectomyif 401

52: THE THORAX This page intentionally left blank PART 10 Vascular

53 Arterial disorders 405 54 Venous disorders 411 55 Lymphatic disorders 416 This page intentionally left blank 53 Arterial disorders

Multiple choice questions ➜ Claudication E Bruits are commonly found in the lower 1. Which of the following statements are abdomen. true? ➜ A Intermittent claudication may be present Investigations at rest. 4. Which of the following statements are B Intermittent claudication is commonly true? relieved by getting out of bed. A Doppler ultrasound works on the basis C Intermittent claudication is most of a frequency shift when sound waves commonly felt in the calf. hit moving red blood cells. D Intermittent claudication distance is B ABPI means ‘ankle brachial pulsatility usually inconsistent on a day-to-day index’. basis for a given patient. C An ABPI of greater than 0.9 is probably E Intermittent claudication is thought to normal. be due to nerve compression in the leg D Duplex scans are a combination muscle compartments. of waveform analysis and B-mode ultrasound. ➜ Critical ischaemia E Carotid surgery is often performed 2. Which of the following statements are on the basis of duplex scans without true? further imaging. A Rest pain is commonly found in the calf 5. Which of the following statements in critical limb ischaemia. regarding angiography are true? B Rest pain is often relieved by getting out A It is only done once it is considered that of bed and taking a few steps. intervention may be appropriate. C Hypoaesthesia is a feature of B Compared with conventional pregangrene. angiography, the digital subtraction D Absence of foot pulses implies critical angiograms require an increased ischaemia. contrast load for reliable images. E Paralysis of the leg may suggest an C Magnetic resonance angiography acutely ischaemic leg. (MRA) is largely replacing conventional angiography for routine imaging. ➜ Aorto-iliac disease D Neurological dysfunction is a potential 3. Which of the following are true in complication of arteriography. aorto-iliac disease? E Routine aortography cannot be A Impotence is common. performed via the femoral artery. B It may be the cause of thigh and calf claudication. ➜ Drugs C It is very unlikely in the presence of 6. Which of the following drugs have femoral pulse. been shown to improve claudication? D Surgical intervention has the poorest A Beta-blockers results in aorto-iliac disease when B Aspirin compared with infra-inguinal disease. C Oxpentifylline

405 VASCULAR ➜ ➜ 0 Acutearterialocclusion maybe 10. statementsare Whichofthefollowing 7. . Rupturedabdominalaortic 9. statements Which ofthefollowing 8. 406

Acute arterialocclusion Abdominal aorticaneur B A E D C B A E D C B A E D C B A E D external trauma external Damage totheintima onlyfollowing swelling Compartmental of lessthan5cm. Ruptures arerarelyseeninaneurysms to confirmthediagnosis. scanisalwaysneededpreoperatively CT resuscitated priortosurgery. be immediatelyandaggressively Patients witharuptureshould peritoneal cavity. Most rupturesoccuranteriorlyintothe AAA. ruptured Most patientsdiefollowing aneurysms cent ofelectivecases. Graft infectionisseeninlessthan1per open repair. equivalentto perioperative mortality repairhasa Endovascular aneurysm failure. respiratory Most electivedeathsarefrom repair. elective AAA Renal failureisuncommonfollowing 10 percentofpostoperativepatients. Colonic ischaemiamayoccurinupto aretrue? surgery ofAAA regarding complications resulting inthepatient’sdeath. rupture, will eventually Most AAAs assessed byangiography. isbest The sizeoftheaneurysm sensitivity andspecificityforAAAs. Ultrasound hasahighdegreeof surgery. Older chronologicalageisnotabarto usually symptomatic. are (AAAs) aneurysms Abdominal aortic Prostacyclin. Simvastatin caused by: caused true? y sms ➜ ➜ ➜ 12. Which of the following are indications areindications Whichofthefollowing 12. isnotacause Whichofthefollowing 11. 14. Which of the following statementsare Whichofthefollowing 14. 13. Which of the following regarding Whichofthefollowing 13.

Amputation Gangrene Ra E D C B A E D C B A E D C B A E D C E D C B A y of gangrene? true? for majoramputation? Deep veinthrombosisinsufficiency. Frostbite druginjection Intra-arterial Infection Buerger’s disease spasm. Arterial Intramural haemorrhage Intra-abdominal swelling treatment of Raynaud’s phenomenon. treatment ofRaynaud’s Sympathectomy isaneffective Raynaud’s. common underlyingcauseofsecondary isa (SLE) Systemic lupuserythematosus with vibratingtools. maybeassociated Raynaud’s Primary seen inyounggirls. ismostcommonly Raynaud’s Primary results indigitalgangrene. phenomenonoften Raynaud’s Primary weight-bearing’ amputations. amputationsarenot‘end-Below-knee at theanklelevel. A Gritti–Stokesamputationisperformed vessel occlusivedisease. inlarge- better choicethanbelow-knee Above-knee amputationisgenerallya rest pain. Transmetatarsal amputationcanrelieve diabetics. Toe amputationsusuallyhealwellin amputations aretrue? restpainwithoutgangrene. Severe contracture Knee flexion Neurofibroma trauma Severe leg. Clostridium infectionofthelower naud’s phenomenon ➜ ➜ ➜ 2.B,C,E 1. C Answers: Multiplechoicequestions matchingquestions Extended Chooseandmatchthecorrectdiagnosiswitheachofscenariosgivenbelow:

Critical ischaemia Claudication 5 4 3 2 1 G F E D C B A 1. Emboli commonly present inpatientswithvasculardiseaseand doesnotinitselfimplycriticalischaemia. be duetoanacutelyischaemicleg or anacute-en-chronicischaemia.Theabsenceoffootpulsesis Hypoaesthesia isoftenpresentinthreatened limbs,butwhereparalysisispresentthislikelyto bygetting outofbedandwalkingaround. whichisoftenachieved bydependency, be relieved ischaemia.Itcan Rest painisfoundinthetoes or forefootofthefootbecausesevere etc. cytokines withstimulation ofpainpathwaysvia is duetorelativeischaemiaofthemusclesduringexercise consistentonaday-to-daybasis.Intermittent claudication the claudicationdistanceisremarkably bygettingoutofbed.Thecommonestsiteforintermittentclaudication isthecalf,and relieved Itisrestpainthat exercise. Intermittent claudicationispainthatonlypresentfollowing and thigh. He has a grade V aortic murmur.and thigh.HehasagradeVaortic markedmottlingovertherightglutealarea reveals His temperatureis38ºCandexamination breathlessnessandpaininhisrightbuttock. IVdrugabuserpresentswithfevers, A 40-year-old of poorfillingintheretinalarteries. areas several blindness whichresolvedoneachoccasionafterabout5min.Retinoscopyshows lefteye recentepisodesoftemporary withseveral A 65-year-oldwomanpresentstoherGP abdomen. therearenofemoralpulsesandhehasmottlingoftheskinlower On examination limbs. malepresentstoA&Ewithparalysisandparaesthesiaofhislower A 59-year-old normal. thegroinbutherleftlegpulsesarepresentand leg. Shehasnopalpablepulsesbelow lower shehasanirregulartachycardiaof120examination beats/minandawhite,paraestheticright A 75-year-oldfemalepresentswithasuddenepisodeofacutepainintherightleg.On haemorrhage. areasofpetechial isnotedtohavebecomedisorientatedwithseveral fixation, and, following accidentpresentstoA&Ewithafracturedfemur A 22-year-oldmaleinvolvedinamotorcycle thrombosis Acute aortic embolism Popliteal arterial Cholesterol embolism Foreign bodyembolism Mycotic embolism Fat embolism Air embolism 407

53: ARTERIAL DISORDERS VASCULAR ➜ ➜ ➜ ➜ 8.A,B,E 7. B,C 6. C 5.A,C,D 4.A,C,D,E 3.A,B,E 408

Abdominal aorticaneur Drugs Investigations Aorto-iliac disease Impotence is commonly found in patients with aorto-iliac disease,althoughitisnotalwaysdueto Impotence iscommonlyfoundinpatientswithaorto-iliac the graft is confined to the abdominal cavity and does not extend tothefemoralarteries. the graftisconfined totheabdominalcavityanddoes notextend where repair,particularly rarelyseeninAAA that usinganopentechnique.Graft infectionisvery to myocardialischaemiaandthedeath ratefromendovascularrepairisprobablyonlyone-thirdof rupturedrepair.repair althoughitisquitecommon following Mostelectivedeathsaresecondary elective uncommonfollowing it rarelypresentswithamajorproblem. Renalfailureisextremely Transient repair. electiveaneurysm colonicischaemiais surprisinglycommonfollowing Fortunately rateisonlyapproximately 0.2cm/year.relatively smallwhenfirstfoundandthegrowth are intactasthemajority ofaneurysms will actuallydiefromothercauseswiththeiraneurysm contains clot.Mostpatients poormethodofassessingsizeasmuchtheaneurysm is avery highdegreeofsensitivityandspecificity,aswellreproducibility.and hasavery Angiography aneurysm which initselfisnotabartosurgery. Ultrasoundisthestandard investigationforaortic aremorecommonwithincreasing age, areusuallyasymptomatic.They The majorityofAAAs distance butisnotoftenprescribednowadays. tohave amarginalbenefitonwalking claudication distance.Oxpentifyllinehasbeenshown isoccasionallyusedforcriticallimbischaemiabuthasnoeffecton distance. Prostacyclin Simvastatin areroutinelyprescribedforcardiovascularriskfactorsbutdonotaffectclaudication and andcanworsenclaudication.Aspirin Beta-blockers actuallyreducemusclebloodflow andcanroutinelyimagetheaorta. the Seldingertechniqueviafemoralartery doneusing arenow recognised complicationofarteriography. Thevastmajorityofarteriograms suchasangioplasty.for subsequentintervention, Neurologicalcomplications,thoughrare,area initial imagingmodalityofchoice,althoughconventionalangiographyisstillgenerallyneeded angiographyisincreasinglytakingoverasthe significantcontrastload.MR indeed requireavery althoughitshouldbenotedthatintravenousdigitalsubtractionangiogramsdo arteriography, requireareducedcontrastloadcomparedwithconventional Digital subtractionarteriograms isseriouslybeingconsidered. unlessintervention Angiography shouldnotbeperformed combination ofwaveformanalysisandB-modeultrasound. andarea flow thegoldstandardassessmentofcarotidarterial scansarenow normal. Duplex >0.9wouldberegardedas andgenerallyanABPI meansanklebrachialpressureindex, ABPI shiftofreflectedultrasoundwaveshittingredcells. The Dopplerprincipledependsonfrequency occlusivedisease. abdomen asaconsequenceofaorto-iliac as inthefemoropoplitealsegment,andbruitsarecommonlyfoundwhenlistenedforlower muchbetterthanfoundinsmallervesseldisease, such reconstructionarevery aorto-iliac following pathology. Theresults presence ofafemoralpulsedoesnoteliminatethepossibilityiliacarterial diseaseandthe insufficiency. Boththighandcalfclaudicationcanbecausedbyaorto-iliac arterial y sms ➜ ➜ ➜ ➜ 14. B,C,D 13. A,E 12. A,B,D,E 11. E 10. A,B,D 9.A,E

Ra Amputation Gangrene Acute arterialocclusion time for intervention to be performed. Intraperitonealruptureisoftenfairlyrapidlyfatal.Patients tobeperformed. time forintervention percentmaystilldie.Most rupturesoccurposteriorly,givingsome who doreachhospital,30–50 reachhospital.Ofthose ever beforethey The majorityofpatientswilldiefromrupturedaneurysm of Raynaud’s phenomenon. of Raynaud’s phenomenon. Althoughusedhistorically, asympathectomyhaslittleroletoplayinthetreatment Raynaud’s andotherconnectivetissue disorderscommonlyhavesecondary syndrome. SLE ofthisbeinghand-armvibration example be associatedwithvibrationtools,the moreextreme resultsinulcerationordigitalgangrene.Itmay occursinyoung girlsandnever Raynaud’s Primary tuberosity andpatellatendonprimarily. knee amputationstransfertheweightviaasuctionsockettakingpressure ontheanteriortibial andbelow– should bethepreferredchoice.AGritti–Stokesamputationisatknee jointlevel forprostheticfittingthanabove-kneeamputationsand much betterqualityoflifeandopportunity amputations providea thesymptoms.Below-knee knee amputationisusuallyrequiredtorelieve arenotprimarilyclosedatthetimeofsurgery.that they Inthepresence ofrestpain,abelow- wellindiabeticsprovided In contrasttolarge-vesseldisease,toeamputationsoftenhealvery majoramputation. following restpainandnon-reconstructablediseasewillonlyobtainrelief Some patientswithsevere inthepresenceofakneecontractureispointless. vasculardisease.Revascularisation coexistent wherethereis contracturesarebetteroffwithmajoramputation,particularly with kneeflexion traumamay,onoccasions,requiremajoramputation.Somepatients severe amputation. Likewise, oftenrequiring guillotine for amputation.Clostridiuminfection(gasgangrene)isanemergency and,inthemselves,wouldnotbeanindication Neurofibromas arebenigntumoursofthenerves deepveinthrombosisthrombosis. extensive occasionally occurbutthisisduetovery willnotcausegangrene.Venous onitsown Deep veinthrombosisinsufficiency gangrenemay thecarotidbifurcation. atheroma andisfrequentlyseenin,forexample, arelativelycommoncomplicationof Intramural haemorrhagewithintheplaqueis,however, intheabdomen. betenseenoughtooccludethemajorarteries abdominal swellingcannever intra- occlusion.Likewise itdoesnotinitselfresultarterial ofbloodflow, cessation temporary spasm cangiveriseto Althougharterial totherestofarteries. tears withoutsignificantinjury blunttrauma,itispossibletogetintimal majortraumaandcrushinjuries.Following following occlusionandiscommonlyseen syndromeisawell-recognisedcauseofarterial Compartment oflessthan5 cm islessthan1percentoverthefirstyear.of rupturesinpatientswithaneurysms are significantlyhypotensive,andthesepatientsshouldbetakenstraighttotheatre.Theincidence bedoneinpatientswho scanshouldnever hypotension shouldbeinitiated.Althoughuseful,CT with rupturesshouldnotbeaggressivelyresuscitated,butratheraprogrammeofcontrolled y naud’s phenomenon 409

53: ARTERIAL DISORDERS VASCULAR ➜ 5C 4E 3G 2F 1B matchingquestions Answers: Extended 410

1. Emboli the buttocksandthigh. to valvevegetationsandinfectionmycoticembolisation aortic that thepatienthasdeveloped complications.Inthiscase,itislikely Increasingly, intravenousdrugaddictspresentwitharterial presentation. Thecholesterolembolicanoftenbeseenonretinoscopy. toeithersmallclotsorcholesterolemboliisquiteacommon Amaurosis fugaxsecondary Classically, mottlingisseenuptotheumbilicusinthesepatients. serious,life-threateningconditionwithapoorprognosis. thrombosisisavery Acute aortic byanticoagulation. followed femoralorpoplitealembolectomyisabsolutelyindicated, gotatrialfibrillation.Emergency certainly inapatientwhohasalmost embolismtothepoplitealartery ofarterial This isaclassicalexample +/–ventilationmaybeessentialhere. cerebral symptoms,highdoseoxygen Giventhe fractureandlongbonefixation. This isaclassicalpresentationoffatembolifollowing 54 Venous disorders

Multiple choice questions ➜ Venous leg ulcers E Computed tomography (CT) pulmonary 1. Which of the following statements angiography is the definitive test for regarding venous leg ulcers are true? pulmonary embolism. A Less than 10 per cent of patients will get 4. Which of the following regarding a recurrence within 5 years after healing. treatment of DVT are true? B Superficial venous surgery reduces the A Warfarin helps remove clot from the risk of subsequent ulceration. veins. C Venous ulcers are best managed by ‘two B Low-molecular-weight heparin is now the layer’ bandaging. standard initial treatment for DVT. D Greater than 60 per cent of all leg ulcers C INR stands for ‘intrinsic notional ratio’. are venous in origin. D Warfarin is generally continued for E A class 2 stocking provides about 3–6 months following DVT. 50 mmHg pressure at the ankle. E Most patients with an iliofemoral DVT will need a caval filter. ➜ Venous thrombosis 2. Which of the following statements ➜ Varicose veins regarding venous thrombosis are true? 5. Which of the following regarding A Between 5 and 10 per cent of the varicose veins are true? population may have a thrombophilia. A Varicose veins may cause venous B Virchow’s triad describes extra luminal, claudication. intramural and intraluminal obstructive B Varicose veins may be associated with causes. the fOXC2 gene. C A family history of venous thrombosis is C The prevalence of varicose veins is twice a risk factor for a patient. as common in women than in men. D Risk can be abolished by effective D Right leg varicose veins are more prophylaxis. common than left leg varicose veins. E Homan’s sign is pathognomonic in this E Handheld Doppler provides an accurate condition. assessment of saphenopopliteal 3. Which of the following regarding competence. diagnosis of deep vein thrombosis 6. Which of the following are indications (DVT) are true? for surgery in varicose veins? A Clinical findings are a reliable way of A Recurrent haemorrhage establishing the diagnosis. B Skin changes such as B A raised D-dimer level is high specific for lipodermatosclerosis or ulceration venous thrombosis. C Itching C Duplex scanning is the investigation of D Cosmesis choice for DVT. E Deep vein thrombosis. D Radioiodine-labelled fibrinogen is commonly used in the diagnosis of DVT.

411 VASCULAR ➜ ➜ Extended matchingquestions Extended statements Whichofthefollowing 7. Chooseandmatchthemost appropriatemanagementwitheachofthescenariosbelow: Chooseandmatchthecorrectdiagnosiswitheachofscenariosbelow: 412

H G 3 2 1 I F E D C B A 2. Varicose veins 5 4 3 2 1 G F E D C B A 1. Ulcers 2 1 Palma procedure stockingsandprophylacticFragmin Thromboembolic deterrent(TED) saphenous veinreflux. saphenous junctionincompetence andlong anisolated saphenofemoral scanshows phobia. Duplex symptomswithaneedle varicoseveinsandsevere A 42-year-oldwomanpresentswith primary warm leg;allperipheralpulsesare palpable. womanpresentswith arecentlyfracturedneckoffemurandswollenpainless An 80-year-old ulcer. scandemonstrates isolatedpoplitealvenousreflux. Duplex ofDVT5yearsagoanda recently healedvenous A 75-year-oldmanpresentswithahistory Thrombolysis Anticoagulation stockings Class 2support ligation,strippingandstabavulsions Saphenofemoral Foam sclerotherapy Valve surgery Endovascular lasertreatment(EVLT) A patientpresentswithsmallpunched-out,painfululcersoverthedorsumofherfoot. his calfandwhichhasrecentlybecomepainfulenlarged. yearslyingoverthemedialaspectof A 65-year-oldmanpresentswithachroniculcerofseven whereheisnotedtohaveamildhalluxvalgus. metatarsal A 70-year-old lifelongmalesmokerpresentswithapainfululcerovertheheadofhisfirst head. back operationsandanulcerovertheplantaraspectofhissecondmetatarsal multiple ofpinsandneedlesinhisleftfootfollowing manpresentswithahistory A 60-year-old 3-cm-diameter ulcerlyingjustabovethemedialmalleolus. discolorationandindurationaroundtheanklea A 55-year-oldwomanpresentswithbrown disease Arterial Neuropathy Diabetes (SLE) Systemic lupuserythematosus Marjolin’s ulcer Deep veinthrombosisinsufficiency Diabetes DVT occursinabout5percentofcases. Drop footmayoccur. treatment ofvaricoseveinsaretrue? ofsurgical regarding complications 5 4 3 tibial nerve neuropraxia. tibial nerve Numbness oftheheelmaybecausedby paraesthesia oftheforefoot. mayleadto injury nerve Saphenous isaninfrequentcomplication. Causalgia ➜ ➜ ➜ 5. B 4.B,D 3.C,E 2.A,C 1. B,D Answers: Multiplechoicequestions

Varicose veins Venous thrombosis Venous legulcers 5 4 veins are not a cause, although they are very common,affecting some5percentormoreof the arevery veins arenotacause, althoughthey Venous claudication isuncommonandcaused bydeepveinthrombosisobstruction. Varicose adequate anticoagulation.Aniliofemoral DVTaloneisnotsufficientjustification. embolidespite caval filterareacontraindicationto anticoagulation oroccurrenceofpulmonary activity. iscontinuedfor3–6monthsinDVT. Generallyspeaking, warfarin The indicationsfora standsforinternationalnormalisedratioand isameasureofwarfarin heparin inthisarea.INR regardedasastandardinitialtreatmentforDVT.nowadays Ithaslargelyreplacedunfractionated heparinis then tissueplasminogenactivatororstreptokinaseisneeded.Low-molecular-weight doesnotremoveclotfromveins(thrombolysis).Ifthis isbeingconsidered onitsown Warfarin embolism. choice inapatientsuspectedofhavingpulmonary angiogramistheinvestigationof labelled fibrinogenandascendingvenography. pulmonary CT thegoldstandardinvestigationforDVT.nowadays Ithasalmost completelyreplacedradioiodine- scanningis Duplex andcaremustbetakenintheinterpretationoflevels. suchassurgery, events willberaised following such,they for venousthrombosis,beinganacute-phaseprotein.As arenotspecific with asuspectedclinicalDVThaveanothercauseoftheirsymptoms.D-dimers signsofDVTonpresentation.Atleasthalfpatients Many patientshavenosymptomsandfew generally unhelpfulasitiscommonlyfoundinotherconditionswell. is it cannotbeeliminated.Homan’ssign,whichisresistanceofthecalfmusclestodorsiflexion, andalthoughriskcanbereducedbyeffectiveprophylaxis, of venousthrombosisisimportant, increased coagulabilityofbloodasthethreeelementspredisposingtothrombosis.Afamilyhistory and changes inthevesselwall(endothelialdamage),stasisasaresultofdiminishedbloodflow described antiphospholipid antibodies,allofwhichcanpredisposetothrombosis.Virchow the population.Inaddition,therearedeficienciesofAT3 andproteinSaswelllupus associatedwithfactorVLeiden,occursinabout7percentof Activated proteinCdeficiency, provide upto30mmHgpressureattheankle. stockingscanofferanequivalentcompressionregime.Class2may class 2support subsequent ulceration.Three-orfour-layerbandagingisregardedasthegoldstandard,although doesreduce,butnoteliminate,theriskof venoussurgery first 5yearsafterhealing.Superficial After successfulmanagementthereisa20–30percentincidenceofre-ulcerationwithinthe swelling which interferes withhisworkinglife. swelling whichinterferes iliacveinocclusion.Hehassignificantvenousclaudicationandleg completeexternal shows idiopathiciliofemoralDVT. of30andaprevious manhasaBMI scan A 40-year-old Duplex reflux. saphenous junctionincompetenceandshort saphenopopliteal scandemonstratesanisolated symptomatic varicoseveinswithskinchangesandduplex smoker. of37 andisaheavy (BMI) Hehasprimary A 48-year-oldmanhasabodymassindex 413

54: VENOUS DISORDERS VASCULAR 7. A,C,E 6.A,B ➜ ➜ 1E 5D 4C 3G 2F 1D matchingquestions Answers: Extended 414

2. Varicose veins 1. Ulcers debilitatingforpatients. whichcanbeextremely causalgia, injuries cangiverisetosevere generally causesnumbnessofthemedialaspectcalfandankle.Occasionallythesenerve injury nerve inthisprocedurecangiverisetoheelnumbnesswhilesaphenous the tibialnerve Bruisingordamageto of bruisingordamagetothecommonperoneal(lateralpopliteal)nerve. andisaconsequence exploration A dropfootisaninfrequentcomplicationofsaphenopopliteal israrelyanindicationforsurgery.common inpatientsand,onitsown, andcosmesisisnotanappropriateindication.Itchingvery is oftenacontraindicationtosurgery Bleeding andskinchangesareregardedasgenuineindicationsforsurgery. Deepveinthrombosis incompetence. demonstrate saphenopopliteal scanningisrequiredto regionandduplex in thegroin,isinaccurateoversaphenopopliteal useful in therightleg,thoughitisnotclearwhythiscase.HandheldDoppler,whilevery roughly equalbetweenmenandwomen.Leftlegvaricoseveinsareactuallymorecommonthan fOXC2 theremaybeanabnormalityinthe adult population.Inpatientswithadefinitefamilyhistory, risk offutureulceration. and reducesthe stockings.This improvessymptomatology management iscompression support ofDVTanddeep vein thrombosisreflux,themainstay of In anolderpatient withahistory disease havebeenruledout. wherediabetesandmajorarterial patients, anantibodyscreenshould beobtained,particularly Insuch Small punched-outulcersareoftensuspiciousofvasculitis,suchasisfound inSLE. carcinoma). Biopsies(traditionallyfourquadrants)areindicatedinsuch ulcers. cell Long-standing ulcerationmaygiverisetoaMarjolin’stypeofulcer(squamous orbasal disease. pressure indicesshouldbeobtainedtoconfirmorruleoutarterial diseaseshouldbesuspectedandpulsespalpated. Inaddition,anklebrachial of smoking,arterial In apatientwithulcersoverbonyprominencewherethereisnoneuropathy andaclearhistory isoftenhighlysuccessful. support fittingandarch head.Protectionoftheulcerbyorthotic occur overthesecondorthirdmetatarsal with dysfunctionalfootarchitecture,suchaslossofthetransversearch,theseulcerscancommonly Peripheral neuropathymaycommonlypresentwithulcerationoverbonyprominences.Inpatients venousdisease. tosuperficial some ulcerscanbesecondary scanbut isoftenpresentonduplex haemosiderin deposition.Deepveinthrombosisinsufficiency to is thickeningpigmentationandindurationoftheskin.Thissecondary or theuppercalf.Venous ulcersgenerallyhavesurroundinglipodermatosclerosiswherethere intothefoot Classical venousulcersoccuronthemedialaspectofcalf,rarelyextending gene,andalthoughwomenpresentmorecommonlywithvaricoseveins,thedistributionis 5H 4A 3D 2G with thromboembolicstockingsandprophylacticFragmin. This womanwitharecentfracturedneckoffemurisathighriskDVTandshouldbetreated choice. Unfortunately thisoperationhasmodestresults. choice. Unfortunately veinwouldtheoptimum then thePalmacross-femoralbypassusingoppositelongsaphenous This manhassymptomaticdeepveinthrombosisobstructionandifheistohaveanytreatment ofhisBMI. better optioninview This mancouldeitherhaveEVLT ligation.EVLT orconventionalsaphenopopliteal mightbethe avulsions becauseofherneedlephobiaandsymptomaticvaricoseveins. ligation,strippingandstab withconventionalsaphenofemoral This patientwouldbebestserved 415

54: VENOUS DISORDERS 55 Lymphatic disorders

Multiple choice questions ➜ Lymphatics 4. Causes of lymphoedema include: 1. Which of the following statements are A Metastatic carcinoma in lymph nodes true? B Arterial insufficiency A Lymphatics contain endothelium. C Deep vein thrombosis (DVT) B Lymphatics may be surrounded by D Radiotherapy smooth muscle. E Sedentary occupation. C Lymphatics contain valves. 5. Which of the following is not a risk D Flow is from superficial to deep in the factor for lymphoedema? limbs. A Limb surgery (e.g. varicose vein E Lymphatics communicate with arterials in operation) the capillary bed. B Obesity ➜ C Family history Lymphangitis D A Baker’s cyst 2. Which of the following statements are E Air travel. true? A Lymphangitis is usually caused by 6. Which of the following statements Pseudomonas. regarding filariasis are true? B Lymphangitis is indicated by red streaking A Filariasis is the commonest cause of in the limb. secondary lymphoedema. C Lymphangitis may predispose to further B It may lead to elephantiasis. attacks. C It may be eradicated by antibiotics. D Lymphangitis is less common in patients D Once treated, the lymphoedema with lymphoedema. resolves. E Lymphangitis may often result in E The mosquito is heavily implicated in this bacteraemia. disorder. 7. Which of the following are ➜ Lymphoedema true regarding the treatment of 3. Which of the following statements lymphoedema? regarding lymphoedema are true? A Surgery will be required in about A Primary lymphoedema occurs in more 10 per cent of patients. than 5 per cent or the population. B The mainstay of management is bedrest. B Early treatment is usually successful. C Compression stockings should provide a C Early treatment includes surgical pressure of 40–60 mmHg at the ankle. drainage. D Early treatment of infected episodes with D Fluid is relatively low in protein in penicillin is important. lymphoedema. E Exercise is contraindicated because of E Lymphoedema often involves the muscle increased swelling. compartments.

416 ➜ ➜ Extended matchingquestions Extended Choose andmatchtheinvestigationthatcorrespondsbestwitheachoffollowing scenarios: Chooseandmatchthecorrectdiagnosiswitheachoffollowing

6 5 4 3 2 1 F E D C B A 2. Investigationsinl 7 6 5 4 3 2 1 G F E D C B A 1. Lowerlimbswelling statements: This canprovideuseful informationonvenousabnormalities. This isnolongerusedasafirst-line testforlymphoedema. This providesthebestreflectionof lymphatic function. apelvicor intra-abdominalmasscausinglegswelling. This isbesttoexclude inlymphnodes. This isbestfordiagnosingmalignancy This isbestfordistinguishingbetweenvenousandlymphaticcausesof aswollenlimb. Biopsy ofnodes Ultrasound Magnetic resonanceimaging(MRI) Computed tomography(CT) Lymphoscintigraphy Lymphangiography hemispheric strokewhichhasleftherbed-bound womanpresentswithapainfulswollenrightleg6daysafteradmissionforleft An 80-year-old legishotandswollen. tender overthepast48h.Thelower toenailwhichhasbeenexquisitely ofaningrown manpresentswithahistory A 50-year-old ofDVTorinjury. leg. Thereisnohistory ofprogressivepainlessswellinginherleft A 30-year-oldwomanpresentswitha1-yearhistory critical ischaemia.Pulsesarepresentbutthecalfistenseandtender. manhasmarkedlegswelling1dayafterundergoingafemoraldistalbypassfor A 60-year-old afootballmatch. A 25-year-oldfitmalepresentswithpainandswellingofthelegfollowing withbreathlessness. presents toherGP A 45-year-oldwomanwithapansystolicmurmurandbilateralpainlesslimblegswelling swelling. thepainbysleepinginasittingpositionandpresentswithleg unable tosleep.Herelieves A 70-year-old painatnightinhisrightlegforthepast5weeksandis manhashadsevere Critical ischaemia syndrome Compartment muscletear Calf Cellulitis DVT Lymphoedema Congestive cardiacfailure y mphatic disorders 417

55: LYMPHATIC DISORDERS VASCULAR ➜ ➜ ➜ 7. C,D 6.A,B,E 5. D 4.A,C,D 3. B 2.B,C,E 1. A,B,C Answers: Multiplechoicequestions 418

Untreated lymphangitis may often result in bacteraemia and even septicaemia. Untreated lymphangitismayoftenresultinbacteraemiaandeven todamageofthelymphchannelsandnodes. attacksaremorelikely,owing attack, further is muchmorelikelytorecurinpatientswithunderlyinglymphoedema.Onceapatienthasone redstreaksalongthelinesoflymphaticchannels and It classicallypresentswithsuperficial lymph channels. is positivelyencouragedincasesof lymphoedema,asitaidsthereturnoflymphupremaining L L L need tobeaddedthisincaseswhich arethoughttobedue episode ofinfectionorcellulitis,early treatmentwithpenicillinismandatory. may Flucloxacillin an mmHgattheankle.Inpatientswhoexperience and60 provide apressureofbetween40 themainstayofmanagement iscompressionstockings,whichshould exacerbations, severe rarelyrequiredinthetreatmentoflymphoedema.Whilst bedresthasarolein isvery Surgery lymphatic changesandthepatientwillstillrequiretreatmentfortheirlymphoedema. the thisdoesnotreverse the treatmentisnotantibioticsbutdiethylcarbamazine.Unfortunately bancrofti Filariasis affectsabout100 millionpeopleworldwide and iscausedbyanematode, including, interestingly,airtravel. isthecorrectanswer.Baker’s cyst Alltheothershavebeenassociated directlywithlymphoedema, well-documented causesoflymphoedema. DVTandmetastaticcarcinomaareall tobeassociatedwithlymphoedema.Radiotherapy, known isnotacauseoflymphoedema insufficiency Arterial muscle, theoedemararelyinvolvesmusclecompartments. percentoflymphisformedwithinskeletal is classicallyhighinproteincontentandalthough40 ofcases.Lymphoedema fluid forthemostextreme andisreserved drainage) israrelyperformed well.Surgicaltreatment(whichdoesnotinclude treated appropriately,patientsgenerallydovery lymphoedemaoccursinlessthan1percentofthepopulation.Ifidentifiedearlyand Primary Lymphangitis iscommonlycausedbystreptococcalinfectionor,lesscommonly, directly. bed anddonotcommunicatewiththecapillary veins, lymphchannelsareblind-endingsacks system,althoughlymphchannelsrunalongsideveinscommonly.deep tothesuperficial Unlike isactuallyfromthe lymph channelscontainvalvesthataugmentthisprocess.Unlikeveins,flow elementin themovementoflymphlimbs.These by smoothmuscle,whichisanimportant Lymphatics areindeedlinedbyendothelialcellsandthelargerlymphchannelssurrounded y y y mphoedema mphangitis mphatics . Thisisspreadbythecommonmosquito.Itmayindeedleadtoelephantiasis and per se . Likewise, sedentary occupationisnot sedentary . Likewise, Staphylococcus Staphylococcus . Gentle exercise . Gentleexercise Wuchereria . ➜ ➜ 6E 5A 4B 3C 2F 1D 7C 6D 5B 4F 3E 2A 1G matchingquestions Answers: Extended

2. Investigationsinl 1. Lowerlimbswelling circulation, particularly in the lower limb. inthelower circulation, particularly investigationinassessment ofthevenous important isavery Ultrasonography, specifically duplex, painfulforthepatient andtechnicallydifficulttoperform. abnormalities. Itis,however, structural of thelymphaticchannelsandassuch isstillseenasthegoldstandardforshowing forananatomicalpicture allow Lymphangiography Itdoes,however, rarely performed. isnow investigating thelymphaticsystem.Itprovidesausefulmeasureof function. Lymphoscintigraphy diagnostictechniquefor hasreplacedlymphangiographyastheprimary whichmightberesponsibleforlimbswelling orlymphoedema. abdominal malignancy, Computed tomographyscanistheinvestigationofchoicewhenlooking forpelvicorintra- aspiration orsurgicalbiopsy. issuspectedinalymphnode,thisshouldbeconfirmed eitherbyfine-needle If malignancy between venousandlymphaticcausesoflimbswelling. anditishelpfulindistinguishing lymphatichyperplasiawithMRI channels. Itispossibletoshow clearimagesoflymphatic usefulandcanshow Magnetic resonanceimagingisparticularly daysoftheirstrokeshouldbeconsideredtohavehadaDVTuntilprovenotherwise. within afew highriskofDVTandanypatientwithapainfulswollenlegpresenting Stroke patientsareatvery andantibiotics. elevation toadistalinfectivecauseandrequiresbedrest, ofcellulitissecondary This isaclassicalexample good. early, theoutcomeislikelytobevery This islikelytobelymphoedematardaand,provideditrecognisedandtreatmentinstituted leg swellinginvascularpatientsandsuchmayneedurgentfasciotomy. seriouscauseofacute syndromecanbeavery resultinginacompartment Ischaemia perfusion The treatmentisicecompressionandelevation. fityoungindividuals. foracutelegswellinginotherwise Simple traumaisacommonexplanation this case,possiblyduetomitralortricuspidincompetence. Bilateral legswellingandbreathlessnessintheabsenceofpainwouldsuggestacardiaccause confusing totheuninformed. over theedgeofbed.Thiseffectivelyresultsinagravitationoedemalimbwhichcanbe Patients withcriticalischaemiaandrestpainoftensleepeithersittinguporonelegdangling y mphatic disorders 419

55: LYMPHATIC DISORDERS This page intentionally left blank PART 11 Abdominal

56 History and examination of the abdomen 423 57 Hernia, umbilicus and abdominal wall 425 58 The peritoneum, omentum, mesentery and retroperitoneal space 432 59 The oesophagus 437 60 Stomach and duodenum 441 61 The liver 451 62 The spleen 458 63 The gall bladder and bile ducts 461 64 The pancreas 468 65 The small and large intestines 474 66 Intestinal obstruction 488 67 The vermiform appendix 495 68 The rectum 499 69 The anus and anal canal 504 This page intentionally left blank 56 History and examination of the abdomen

Multiple choice questions ➜ Conditions causing ➜ Parietal peritoneum abdominal pain 2. Which of the following statements are 1. Which of the following medical true? conditions is not a cause of abdominal A The parietal peritoneum is innervated by pain? somatic nerves. A Diabetic ketoacidosis B The parietal peritoneum is innervated by B Porphyria autonomic nerves. C Angina C Pain originating in the parietal D Pneumonia peritoneum is poorly localised. E Coeliac disease D Pain from the parietal peritoneum may F Herpes zoster. radiate. Extended matching questions ➜ 1. Sources of abdominal pain A Splenic rupture B Testicular pain C Ureteric colic D Appendicitis E Cholecystitis F Pancreas Match the following sources of pain to the site of radiation: 1 Back 2 Umbilicus 3 Lower angle of scapula 4 Loin 5 Groin 6 Left shoulder tip Answers: Multiple choice questions

➜ Conditions causing abdominal pain 1. E An insulin-dependent diabetic should always raise the suspicion of ketosis causing abdominal pain. An urgent blood sugar and urine examination should give the diagnosis. Violent intestinal

423 ABDOMINAL ➜ ➜ 1F,2D,3E,4B,5C,6A matchingquestions Answers: Extended 2.A,D 424

1. Sources ofabdominalpain Parietal peritoneum dermatome; painprecedesvesicularskineruption. abdominal painisnotafeature. with signsinthechestarediagnosticpointers.Coeliacdiseasecausesmalnutritionand pneumonia.Pyrexia diagnosis. Painintheupperquadrantmaybepresentationoflower-lobe present withanaemia,photosensitivity,hypersplenismandconstipation. colic mayoccurinporphyria,acongenitalabnormalityofhaemoglobinmetabolism.Patients nerve, as in acute cholecystitis whenthepainradiatestoback. asinacutecholecystitis nerve, Pain fromparietalperitonealinflammationmayradiatealongthelinesuppliedbysomatic in thedistributionofoverlyingdermatomes.Painiswelllocalisedtositeinflammation. throughthespinalnerves somaticnerves bytherelevant The parietalperitoneumisinnervated Epigastric pain can mimic angina. A careful history followed by an ECG shouldgivethe byanECG followed Epigastric paincanmimicangina.Acarefulhistory Herpes causespainacrosstheabdomenwithoutguardingorreboundtendernessalonga 57 Hernia, umbilicus and abdominal wall

Multiple choice questions ➜ Predisposing factors for ➜ Anatomy of the inguinal hernia canal 1. Which of the following is not a 5. Regarding the anatomy of the inguinal predisposing factor for a hernia? canal, which of the following are true? A Chronic obstructive pulmonary disease A In infants the internal and external rings B Obesity are almost superimposed. C Urinary stones B The inferior epigastric vessels lie posterior D Pregnancy and lateral to the internal ring. E Peritoneal dialysis. C The inguinal canal is about 10 cm long and is directed downwards, medially and ➜ Causes of hernia forwards. 2. Which of the following are more D In males, the normal constituents of the common in multiparous women? inguinal canal are the spermatic cord, the A Indirect inguinal hernia inguinal nerve and the genital branch of B Lumbar hernia the genitofemoral nerve. C Umbilical E The internal ring is a U-shaped opening D Direct inguinal hernia in the external oblique aponeurosis 1.25 E Femoral hernia. cm above the mid-inguinal point. ➜ Complications of hernia ➜ Sliding inguinal hernia 3. Which of the following is not a 6. Which of the following are true about complication of an inguinal hernia? a sliding inguinal hernia? A Irreducibility A It is far more common in men. B Inflammation B It should be suspected clinically in C Strangulation small hernias confined to the inguinal D Obstruction canal. E Bleeding. C It is more common in the young patient. ➜ Strangulated hernia D It is impossible to control with a truss, 4. Which of the following is not true in and hence an operation is indicated. relation to strangulated hernias? E It is unnecessary to excise the sac and A They present with local and then attempts to dissect the bowel wall can be generalised abdominal pain and dangerous. vomiting. B A normal hernia can strangulate at any ➜ Femoral hernia time. 7. Which of the following statements C This is more common in femoral hernia. regarding a femoral hernia are true? D They can be reliably excluded in A It is more common in women. irreducible hernias on clinical examination. B The femoral canal occupies the most E They require urgent surgery. lateral component of the femoral sheath.

425 ABDOMINAL ➜ Extended matchingquestions Extended Chooseandmatchthecorrect diagnosiswitheachofthescenariosgivenbelow: ➜ 8. Which of the following statementsare Whichofthefollowing 8. 426

4 3 2 1 I H G F E D C B A 1. Diagnosisofvariousabdominalwallhernias E D C B A Umbilical hernia E D C aspect oftheleftgroin. Thereisnocoughimpulse. a2cm hardandirreduciblelumpoverthemedial reveals left groinlymphnode.Examination ofsuspectedenlarged A 78-year-old womanis referred toyourclinicwitha6-weekhistory anirreduciblelumpjustabove theumbilicus. reveals Examination ofpainfullumparoundherumbilicus. obesewomanisreferred withahistory A 60-year-old tothescrotum.Internalringocclusiontestispositive. extending coughimpulseandareducibleherniaovertheleftgroin reveals last 3years.Examination A 26-year-oldbuilderpresentswithalumpoverhisleftgroinwhichhas beenthereforthe which increasesinsizewhenthebabyiscrying. oflump overthebelly-button, A 6-month-oldinfantisbroughtbyhismotherwithahistory Spigelian hernia Divarication ofrecti Epigastric hernia Paraumbilical hernia Indirect inguinalhernia Incisional hernia Umbilical hernia Femoral hernia Direct inguinalhernia true regarding umbilical hernia? true regardingumbilical An operationisonlyoccasionally percentofpatients. 40 Strangulation istheinitialpresentationin It canbeeasilycontrolledbytruss. immediate surgery. Infantile umbilicalherniasneed the hernia. Mayo’s operationinvolvesmeshrepairof adhesions. Irreducibility iscommonduetoomental women. Men areaffectedmorefrequentlythan usually paraumbilicalinadults. through theumbilicalcicatrixbutis The umbilicalherniainaninfantis required. 10. Which of the following aretrue Whichofthefollowing 10. of arecauses Whichofthefollowing 9. ➜ ➜

Burst abdomen Umbilical discharge E D C B A E D C B A regarding ‘burstabdomens’? discharge? umbilical The incidenceisaround10–15 percent. Endometrioma. Omphalitis Pilonidal sinus Persistent urachus Persistent vitellointestinalduct Most casesaremanagedconservatively. cases. forerunner ofdisruptioninhalfthese dischargeisthe A serosanguinous burst abdomen. riskof isassociatedwithalower Catgut likely toburstthantransverseincisions. incisionsaremore Midline andvertical ➜ ➜ Chooseandmatchthecorrectdiagnosiswitheachofscenariosgivenbelow:

E D C B A 3. Treatment ofabdominalwallhernias 5 4 3 2 1 E D C B A 2. Complicationsofhernia 9 8 7 6 5 Non-surgical Open/laparoscopic meshrepair Open meshrepair Transabdominal preperitoneal(TAPP) repair Wait andwatch ulcerated skinovertheincarceratedlong-standinghernia. and excoriated reveals with faeculantdischargefromtheherniasite.Examination emergency ofincisionalhernia presentstothe womanwithalong-standinghistory A 90-year-old Theabdomenissoft,non-tenderandnotdistended. examination. havebeennormal.Anirreducibleherniaisconfirmed on over thepast3h.Hisbowels over hisrightinguinalhernia maleisreferredwithirreducibilityanddiscomfort A 50-year-old sounds. soft abdomenwithincreasedbowel a distended but reveals a rightgroinlumpwhichisnotreducibleandabdominalexamination reveals the past2days.Shehasbeenvomitingrepeatedlyoverthisperiod.Clinical examination ofincreasingabdominal painanddistensionover femaleisreferredwithahistory A 60-year-old tenderwithsignsofperitonitis. theherniaisextremely over thepast8h.Onexamination maleisreferredwithirreducibilityandincreasingpainoverhisrightinguinalhernia A 40-year-old you seeagiantinguinoscrotalherniawhichisnotcompletelyreducible. havebeennormal.Onexamination noticed anyrecentchangesinhisherniaandbowels A 70-year-old maleisreferredtoyourclinicwithalong-standingleftinguinalhernia.Hehasnot Irreducibility Perforation Incarceration Obstruction Strangulation umbilicus. Thereisnocoughimpulseandthelumpnotreducible. abutton-likesoftswellingequidistantbetweenthexiphisternumand reveals examination A 30-year-oldfemalepresentswithalocalisedsharppainintheupperabdomen.Clinical gutter intheupperabdomen. amidline reveals asymptomatic.Clinicalexamination Itisotherwise inthegym. exercising A 22-year-oldfitmalepresentswithanepigastricbulge.Thisbecomesprominentwhile impulse isfeltovertheareaandlumpreduceswithdifficulty. theumbilicusandlateraltorectus.Acough manpresentswithalumpbelow A 50-year-old both hisgroins.Internalringocclusiontestisnegative. confirmsreduciblehernias over Clinicalexamination are onlycausinghimminimaldiscomfort. oflumpsoverbothhisgroins.These A 76-year-old manisreferredtoyouwitha2-yearhistory totherightgroin. aspect ofthisscarextending aPfannenstielscar.reveals Thereiscoughimpulseandareduciblelumpovertherightlateral sectionsinthepast.Clinicalexamination abdomen.Shehashadthreecaesarean over thelower ofagraduallyincreasinglump womanisreferredtoyourclinicwithahistory A 40-year-old 427

57: HERNIA, UMBILICUS AND ABDOMINAL WALL ABDOMINAL ➜ ➜ ➜ ➜ 4. D 3. E 2.C,E 1. C Answers: Multiplechoicequestions Chooseandmatchthemostappropriateoperation witheachoftheconditionsgivenbelow: 428

Strangulated hernia Complications ofhernia Causes ofhernia Predisposing factorsforhernia 9 8 7 6 5 4 3 2 1 H G F constipation may be absent. butvomitingand symptoms inastrangulated omentocelearesimilarto astrangulatedbowel, may mimicgastroenteritisinastrangulated Richter’shernia,whichmaydelaydiagnosis.Theinitial andrequirespromptsurgery. Theclinicalfeatures Strangulation isagenuinesurgicalemergency inthedependentareasofalargehernia. causessuchasa trophic ulcerdeveloping from external or e.g.acuteappendicitisand acutesalpingitis, occur frominflammationofthecontentssac, without delay. strangulationandhencetheseshouldbeexplored Inflammationcan reliably exclude withsuspicionandtreatedasanemergency.of irreducibilityshouldbeviewed Itisnotpossibleto history Any herniawhichischronicallyirreducibleprobablyincarcerated.A withashort increased intra-abdominalpressure. itselfpredisposes toherniasdue who havehadchildrenthaninnulliparousones.Pregnancy Femoral herniasaremorecommoninwomen.Even amongstwomenitismorecommoninthose which maybeasaresultofanacquiredcollagendeficiency. Whooping coughisapredisposingcauseinchildhood.Herniasaremorecommonsmokers, do not.Othercausesofstrainingincludechronicconstipationandwork-relatedphysicalexertion. stones symptomscanpredisposetoherniabutuncomplicatedurinary with obstructiveurinary Any causeofincreasedabdominalpressurewillpredisposetoanhernia.Prostatism Paraumbilical hernia Epigastric hernia Incisional hernia Divarication ofrecti Recurrent inguinalhernia–post-laparoscopicrepair Infantile umbilicalhernia Recurrent inguinalhernia–post-openrepair bilateralinguinalhernia Primary unilateralinguinalhernia Primary Open suturerepair Mayo repair Total (TEP)/TAPP repair extraperitoneal ➜ ➜ ➜ ➜ ➜ 9.A,B,C,D,E 8.A,C 7. A,D 6.A,D,E 5.A,D

Umbilical discharge Umbilical hernia Femoral hernia Sliding inguinalhernia Anatom The inferiorepigastricvesselslieposteriorandmedialtotheinternalring.Identifyingthese may open. fluid. Henceitisaptlyremarkedthat theumbilicusisacreekintowhichmanyfistulousstreams oftheductispatent,leading torecurrentinfectionsanddischargeofserousorpurulent a part discharge.Occasionally, fistulaattheumbilicus,whileapersistent urachusresultsinurinary bowel Persistent vitellointestinalductcauses asmall- to antibioticsleavingbehindagranulating surface. oftheumbilical cord.Thisiscalledomphalitisandusuallyresponds practising asepticseverance dayafterdelivery. Thiscangetclinicallyinfected,especiallyincommunitiesnot third orfourth per centofthebabiesby The stumpoftheumbilicalcordiscolonisedbystaphylococciin50 recurrence. hernia islarge,recurrentorthepatienthaspredisposingfactorswhich might increasetheriskof herniorrhaphywithoverlappingofthemargins.A mesh repairisrequiredifthe repair isaprimary treatmentisindicatedunderthe ageof2years.Mayo spontaneously andhenceconservative repeated pregnanciesarepredisposingfactors.Ninety-fivepercentof infantile herniasresolve theumbilicus.Itisfivetimesmorecommoninwomen, andobesity just aboveorbelow The umbilicalherniainadultsisreallyaparaumbilicalhernia,asthedefect isinthelineaalba operation isusuallyadvised. togetstrangulated.Hencean cannot becontrolledbyatrussandhasanincreasedtendency vein laterallyandthelacunarligamentmedially. isthatthishernia Theoverridingimportance ofthefemoralsheath.Itisrelatedto the femoral which occupiesthemedialmostcompartment 20 percentofherniasinwomenand5men.Thisoccursthroughthefemoralcanal hernia,accountingforabout The femoralherniaisthethirdmostcommontypeofprimary largeglobularherniadescendingintothescrotum. suspected inavery but anyopeningmadeshouldbeclosedcarefullyandreducedback.Aslidinghernia inaslidinghernia shouldnotbeexcised it.Thesac orexcising injured whileopeningthesac ofitswall.Hence,utmostcareshouldbetaken,asthesestructurescan actually formingpart butare can alsobeinvolved.Itistorememberedthatthesestructuresarenotwithinthesac bladder most commonstructureinvolvedisthesigmoidcolonbutcaecumandurinary yearsofage.Itisusuallytheposteriorwallthatinvolved.The is nearlyalwaysmorethan40 inmenandthepatient and occursapproximatelyoncein2000cases.Italmostexclusively alongwiththeposteriorparietalperitoneum.Itisrare abdominal viscerawhichhasslippeddown ofthewallisformedbysome isoneinwhichpart A slidinghernia(synonym:hernia-en-glissade) obliqueaponeurosis. openingisatriangularintheexternal the external fasciaand ofthetransversalis is about3.75cmlong.TheinternalringaU-shapedcondensation uptointernalring.Inadults,theinguinalring at operationconfirmscompletedissectionofthesac y oftheinguinalcanal 429

57: HERNIA, UMBILICUS AND ABDOMINAL WALL ABDOMINAL ➜ ➜ 9G 8H 7I 6A 5D 4B 3F 2E 1C matchingquestions Answers: Extended 10. B,D 430

1. Diagnosisofvariousabdominalwallhernias Burst abdomen Burst abdomen (also known asabdominaldehiscence)occursin1–2percentofcasesaround Burst abdomen(alsoknown sort outthepain. sort strangulationbutanoperationisadvisedto fat.There isnoriskofbowel contains extraperitoneal Pain isthepredominant(andsometimes theonly)symptom.Thelumpisusuallysmalland associated withadiscreteherniawhich mayneedtoberepaired. Henceitcannotstrangulateandanoperation isnotneeded.Thedivaricationsometimes sac. The patientpresentsbecauseoftheobviousbulge.Thisiswide-based anddoesnothavea scanissometimeshelpfulindiagnosis.Operationadviseddue to theriskofstrangulation. CT oblique muscles.Anultrasoundor spreads outlikeamushroombetweentheinternalandexternal ofthearcuate line.Thehernia This isararevarietyofinterparietalherniaoccurringatthelevel elderly. A directherniaislesslikelytostrangulateandmoreoftenbilateral.The patientisusually Pfannenstiel incisionisseen. An incisionalherniahastobeconsideredwhenamedialgroinoverlyingprevious hernia hastoberuledout. ofthepalpablemedialgroin‘lymphnode’–afemoral confused withalymphnode.Beware The clinicalappearancecanbedeceptive.Itiseasytomissasmalllumpclinicallyorit andhencestrangulationcanbeaproblem. larger onesmayinvolvebowel Pain andirreducibilityaretheusualsymptoms.Thesmalleronescontainomentumwhile incidence ofirreducibilityinindirectinguinalhernias. An earlyoperationwouldberequiredasitislikelytoaffecthisworkandbecauseofthehigher treatmentisindicatedundertheageof2years. spontaneously andhenceconservative The incidenceinblackinfantsiseighttimeshigher. Ninety-fivepercentofinfantileherniasresolve andtheanxiouspatientisgivenanalgesicsreassured. covered withasteriletowel anyobstructionandresuturethewound.Whileawaitingoperation,woundis relieve bowel, operationisrequiredtoreplacethe abdominal distensionandpoormetabolicstate.Anemergency closure technique,deepwoundinfection,andpatientfactorssuchaspersistentcough,vomiting, causesarepoor discharge.Themostimportant earlier andisassociatedwithaserosanguinous days the 6thand8thdaysafteroperation.Thedisruptionofsuturesusuallyoccursafew ➜ ➜ 9G 8H 7D 6E 5B 4A 3F 2F 1C 5D 4E 3B 2A 1C

3. Treatment ofabdominalwallhernias 2. Complicationsofhernia Long-standing andlargeinguinoscrotalherniasaredifficulttoreducecompletely. Thisisasaresult factors. A meshrepairisconsideredinbigdefects, recurrentcasesandinthepresenceofpredisposing The defectisusuallysmallandonly requiresacoupleofsutures.Meshisrarelyrequired. oftheincreasedincidence ofrecurrence. repairisrecommendedinview Use ofmeshinprimary repaired. A repairisunnecessary. Anassociateddiscretehernia,ifdefinitely confirmed,willneedtobe repairisdifficultastheplanesareobliterated. A TEP to 2yearsofage. approachisadvisedup Ninety-five percentofthesespontaneouslyresolve.Henceaconservative through avirginterritory. openoperationmakethisasuitablecaseforlaparoscopicrepair Adhesions duetoprevious should bethetechniqueofchoice. The morbidityandtimeoffworkarelesscomparedwithabilateralopenrepairhencethis stillbeappropriateinselectivepatients. however, and hencethepresentguidelinesstillrecommendanopenoperation.Alaparoscopicrepairmay, There arenosignificantadvantagesindoingalaparoscopicrepairforanunilateralinguinalhernia well. cavity isusuallynotaffectedandthepatientcanstillremainremarkably anddischargesasafaecalfistula.Theperitoneal iscontainedwithinthesac Theleakage perforate. closed-loopobstructionand Long-standing (usuallyincisionalhernia)herniascandevelop strangulationinanirreduciblehernia. It isnotpossibletoreliablyexclude obstruction. treatmentforsmall-bowel usual trialofconservative It isimpossibletoruleoutstrangulationandhenceitshouldbeoperatedonurgentlywithoutthe andrequiresurgentexploration. This isasurgicalemergency anditscontentsaswellthesurroundingstructures. betweenthesac of adhesionsdeveloping 431

57: HERNIA, UMBILICUS AND ABDOMINAL WALL The peritoneum, omentum, 58 mesentery and retroperitoneal space Multiple choice questions ➜ Peritoneum E Children can localise infection 1. Which of the following statements are effectively. true? 4. Which of the following statements are A The surface area of the peritoneal false? membrane is nearly equal to that of A Perforation proximal to an obstruction the skin. is associated with severe generalised B The parietal peritoneum is poorly peritonitis. innervated. B Stimulation of peristalsis helps in C The peritoneum has the capacity to localisation of peritonitis. absorb large volumes of fluid. C The greater the virulence of the D The peritoneum has the ability to organism, the lesser the chance of produce fibrinolytic activity. localisation. E When injured, the peritoneum produces D The patient with diffuse peritonitis an inflammatory exudate. writhes around in pain unable to assume ➜ a comfortable position. Peritonitis E Systemic inflammatory response 2. Which of the following organisms syndrome (SIRS) is a late manifestation is not a gastrointestinal source of of peritonitis. peritonitis? A Escherichia coli ➜ Diagnostic aids B Streptococcus 5. Which of the following statements are C Bacteroides true? D Chlamydia A Raised serum amylase is always E Clostridium diagnostic of acute pancreatitis. F Klebsiella pneumoniae B There is always gas under the right dome G Staphylococcus of the diaphragm in perforated duodenal 3. Which of the following statements are ulcer. true? C Ultrasound (US) and computed A Peritonitis in perforated duodenal ulcer is tomography (CT) scan are often initially sterile. diagnostic. B Immunocompromised patients may D On diagnostic peritoneal lavage (DPL), present with opportunistic peritoneal bile aspiration indicates perforated infection. duodenal ulcer or perforated gall C Bacteroides are sensitive to penicillin. bladder. D In perforated duodenal ulcer there may be signs of peritonitis in the right iliac fossa.

432 ➜ ➜ Extended matchingquestions Extended Chooseandmatchthecorrectdiagnosiswith eachofthescenariosgivenbelow: Chooseandmatchthecorrectdiagnosiswitheachofscenariosgivenbelow:

4 3 2 1 D C B A 2. Ascites 5 4 3 2 1 E D C B A 1. Intra-abdominalinfection ovarian masses. solidbilateral shows massiveascites. US shows for almost4months. Abdominalexamination A 70-year-old femalepatienthashadweight loss,abdominaldistension,malaise andanorexia has recentlyreturnedfromthesubcontinent afterlivingtherefor2years. period hehasfeltgradualdistension ofhisabdomen,weightlossandgeneralisedmalaise.He malepatientcomplains offeelinggenerallyunwellfor3–4months.Duringthis A 60-year-old jaundiced. Heisslightly andspider naevi. palmarerythema veins aroundhisumbilicus(caputmedusa), patientwhoisanalcoholiccomplainsofabdominaldistension. Hehasdistended A 50-year-old ofbreathandabdominaldistension.Shehasaraisedjugular venouspressure. shortness A 70-year-old tohavehadamyocardialinfarct inthepast,complainsof woman,whoisknown Congestive cardiacfailure hypertension Portal Peritoneal malignancy Tuberculous peritonitis rebound tenderness. tenderintherightupperquadrantwithguardingand a tingeofjaundiceandwasextremely shehad rightupperabdominalpainandintheshouldertip.Onexamination severe on thefirstpostoperativeday. suddenonsetof shedeveloped evening Whileathomethesame a35-year-oldfemalepatientwassenthome laparoscopiccholecystectomy, After anuneventful chestwhichisdullonpercussion. right lower inthe tachypnoeicandtenderintherightupperquadrant.Thereisreducedairentry pyrexial, A 14-year-old boyhasbeenbroughtwithrightupperquadrantpainof3days’duration.Heis complained oftenesmusandfoul-smellingvaginaldischarge. and female patientprogressedwellforabout6days.Afterthatshefeltunwell,waspyrexial appendicitis,a30-year-old appendicectomyforacuteperforated emergency Following shoulder tipandwastenderrigidoverhisrightupperquadrant. lookedtoxic,complainedofpaininhisright pyrexia, postoperativeday,hedeveloped fourth duodenalulcer. alaparoscopicclosureofperforated Onthe patientunderwent A 50-year-old generalised abdominaltendernessandrigiditywithreboundtenderness. pyrexia, untilthefifthpostoperativedaywhenhedeveloped progressed satisfactorily anastomosis forcarcinomaofthecaecumpresentingasacuteintestinalobstruction.He righthemicolectomyandileotransverse anemergency A 72-year-oldpatientunderwent pneumonia Basal Bile peritonitis Postoperative peritonitis Subphrenic abscess Pelvic abscess 433

58: THE PERITONEUM, OMENTUM, MESENTERY AND RETROPERITONEAL SPACE ABDOMINAL ➜ ➜ ➜ 4.B,D 3.A,B,D 2. D 1. A,C,D,E Answers: Multiplechoicequestions Chooseandmatchthecorrectdiagnosiswitheachofscenariosgivenbelow 434

Peritonitis Peritoneum 4 3 2 1 D C B A 3. Themesenter exacerbation ofthepain. exacerbation spreads theinfection.Thepatientwith diffuseperitonitislikestoliestillasanymovementcauses and Stimulation ofperistalsisbyfood,water oradministrationofanenemahinderslocalisation localise peritonitis. ofthegreater omentum,donot oftheirpoordevelopment cephalosporins. Children,byvirtue Bacteroides mimickingacute appendicitis. paracolic guttergivingsignsofperitonitisintherightiliacfossa, theleakingduodenalcontentsmay trackalongtheright hoursafterperforation, peritonitis. Afew ofaduodenal ulcer,thereissterilechemical hoursimmediatelyafterperforation For several andlocalisedtothearea. sothatpainarisingfromitissevere richly innervated which canabsorblargevolumesoffluidisusedforperitonealdialysis.Theparietalperitoneum area equal tothatoftheskin,almost2squaremetresinadult.Thisaspectalargesurface areaofthemembranebeing The peritonealcavityisthelargestinbody,surface tender intheumbilicalregionwithrigidity. heiswrithingaroundinpainandvery periumbilicalabdominalpain.Onexamination of severe complainingofsuddenonset obesemanhasbeenbroughtinasanemergency A 50-year-old tenderness, rigidityandreboundtenderness. forabdominalwallbruisingfromtheseatbelt.Hehasmarked no injuriesexcept onassessment,hehad sudden decelerationandahead-oncollision.IntheA&Edepartment, abdominal pain.Hewasafront-seatpassengerwearingseatbelt.Intheaccidentthere A 30-year-oldmalepatienthasbeenbroughtinafteraroadtrafficaccidentcomplainingof region whichismobileinanobliqueplane. a5–6cmfluctuantmassintheumbilical reveals intermittent vomiting.Abdominalexamination A 20-year-oldmalepatientpresentswithrecurrentattacksofperiumbilicalabdominalpainand tenderandbimanuallypalpable. left loin,smoothinconsistency, thereisalargemassinthe feels herabdomenisenlargingandconstipated.Onexamination A 55-year-oldfemalepatientpresentswithvagueabdominalandbackpainfor3months.She Retroperitoneal sarcoma Torsion ofomentum Mesenteric tear Mesenteric cyst areresistanttopenicillinandsensitivemetronidazole,clindamycin, lincomycin and y ➜ ➜ ➜ 3A 2C 1D 5D 4E 3A 2B 1C matchingquestions Answers: Extended 5.C,D

2. Ascites 1. Intra-abdominalinfection Diagnostic aids The serum amylase may be raised in perforated pepticulcerandthereforenotdiagnosticof The serumamylasemayberaisedinperforated ascitic fluidandchest X-ray. analysisof C-reactiveprotein(CRP), sedimentation rate(ESR), loculated ascites,raised erythrocyte ofabdomenmayshow suspected ofhavingtuberculousperitonitis andinvestigatedassuch:US A patientwhohasrecentlyreturned fromthesubcontinentafterastayof2yearsshouldbe clinical features.Thepatientshould bereferredtoaphysician. withascitesaretheclassical andspidernaevi palmar erythema featuresofcaputmedusa, external The hypertension. ofalcoholabuseshouldgivethesuspicionportal The patient’shistory physician. This womanhasthehallmarksofascitesfromcardiacfailureandshould bereferredtoa bydrainageeitherlaparoscopicallyor openmethod. followed orCT imaging withUS resuscitation, ductresultinginbileperitonitis.Thisrequires emergency of thecliponcystic thispatient aftergoinghomehadslippage laparoscopiccholecystectomy, anuneventful Following intravenous antibiotics. presentation ofsuchacondition.AchestX-raywillconfirmthediagnosisandistreatedwith lobarpneumonia.Paininthe right upperquadrantisaclassical The youngboyhasrightbasal not resolvenaturally. drainageiftheabscessdoes ThismayrequireUS-guided progress canbeassessedbyserialUS. and posterior fornixofthevagina.Thediagnosiscanbeconfirmedbyapelvi-orintravaginalUS This youngfemalehasthefeaturesofapelvicabscessthatisabouttodrainnaturallythrough because oftheabscessbeingmultilocular. guidancebytheradiologist.Thismayrequiremorethanoneaspiration and drainedunderUS/CT orCT right upperquadrantandtoxicafteranintra-abdominaloperation.ThisisconfirmedbyUS The patienthasthehallmarksofaclassicalsubphrenicabscess–paininshouldertipand enema anddealtwithaccordingly. oragastrograffin peritonitis fromanastomoticleak.ThisneedstobeassessedbycontrastCT postoperative developed righthemicolectomyhasnow anemergency This patientwhounderwent reliablediagnosticaids,asisDPL. scanarevery andCT stage. US issmallorifitpluggedbyomentumatanearly the diaphragminearlycases,ifperforation duodenalulcerthere maynotbegasundertherightdomeof acute pancreatitis.Inperforated 435

58: THE PERITONEUM, OMENTUM, MESENTERY AND RETROPERITONEAL SPACE ABDOMINAL ➜ 4C 3B 2A 1D 4B 436

3. Themesenter She shouldbereferredtoagynaecologist. (carcinomaperitonei)fromovariancarcinoma. This 70-year-old womanhasperitonealmalignancy omentum ismadeatoperation. Most oftensuchapatientisoperatedonfor‘acuteappendicitis’andthediagnosisoftorsion Such anobesemalepatientcouldeasilybemistakenlydiagnosedashavingacuteappendicitis. mesenteric lacerationisassociatedwithatearintheintestine. percentofcasesthe scanoradiagnosticperitoneallavagemayhelpinthediagnosis.In60 CT be suspectedofhavingamesenterictear,conditionreferredtoas‘seatbeltsyndrome’.Acontrast forbruisingovertheabdominalwall,should except injury This patient,whohasnootherexternal orinfection. haemorrhage intothecyst duetotorsion, confirm thediagnosis.Sometimessuchapatientmaypresentasanemergency will –atrightanglestotheattachmentofmesentery.upper quadranttotheleftiliacfossa US swelling neartheumbilicuswhichisnon-tenderandmovesfreelyalongalinefromright occursintheyoungeragegroup.Thecharacteristicphysicalfindingisafluctuant A mesentericcyst byguidedcorebiopsygivestheanswer.followed Itisthenstagedandmanagedaccordingly. cancercentre.AnMRI andneedtobemanagedinatertiary are retroperitonealliposarcoma This womanwithvagueabdominalpainhasthefeaturesofaretroperitonealmass.Usuallythey y 59 The oesophagus

Multiple choice questions ➜ Anatomy D Oesophageal manometry should be 1. Which of the following statements are done when motility disorder is suspected. true? E 24 h pH recording is an accurate method A The oesophagus is 30 cm long. of evaluating GORD. B The oesophagus has three natural ➜ constrictions. Oesophageal emergencies C It is lined throughout by columnar 4. Which of the following statements are epithelium. true? D The lower oesophageal sphincter (LOS) A In a suspected foreign body (FB) in is a zone of high pressure. the oesophagus, water-soluble contrast E The pressure at the LOS is 50 mmHg. examination should be carried out. B When a food bolus is stuck in the ➜ Dysphagia oesophagus, always suspect an underlying disease. 2. Which of the following statements are Most iatrogenic perforations of false? C the oesophagus can be treated A Difficulty on swallowing (dysphagia) conservatively. is a cardinal symptom of oesophageal carcinoma. D Most spontaneous perforations of the oesophagus (Boerhaave’s syndrome) B Retrosternal pain on swallowing require an operation. (odynophagia) is always of cardiac origin. C Heartburn is a common symptom of E In Mallory–Weiss syndrome the tear gastro-oesophageal reflux disease (GORD). is usually in the lower end of the oesophagus D Dysphagia in the oral or pharyngeal (voluntary) phase, when patients say ➜ Cancer of the oesophagus they cannot swallow, is usually from 5. Which of the following conditions are neurological or muscular diseases. precancerous? E Regurgitation and reflux are the same A Pharyngeal (Zenker’s) diverticulum and are caused by obstruction to the B GORD oesophagus. C Achalasia 3. Which of the following statements D Corrosive stricture regarding investigations in dysphagia E Barrett’s oesophagus are false? F Schatzki’s ring A Barium swallow is the investigation of G Oesophageal candidiasis choice in GORD. 6. One of the curative operations for B Flexible oesophagogastroduodenoscopy carcinoma of the oesophagus is called (OGD) is the initial investigation of A Whipple’s operation choice in suspected carcinoma. B Anderson–Hynes operation C Endosonography (EUS) should be carried C Heller’s operation out when a carcinoma is seen in the D Ivor–Lewis operation oesophagus. E Hartmann’s operation

437 ABDOMINAL ➜ ➜ ➜ 1. B,D Answers: Multiplechoicequestions matchingquestions Extended arenot Whichofthefollowing 7. Chooseandmatchthecorrectdiagnosiswitheachofscenariosgivenbelow: 438

Anatom 6 5 4 3 2 1 F E D C B A 1. D C B A Complications ofGORD and bronchialconstriction,diaphragmatic andsphincterconstriction.Theseare15, 25and junction,aortic The oesophagusis25cmlong.Ithas threenaturalconstrictions:cricopharyngeal for chestinfections. is embarrassedbybadbreath.Healsohassomedysphagia.Hisdoctor hasbeentreatinghim coughwhenfoodmaterialseemstoprojectout.He times heiswokenupbecauseofincessant coughwhichoccurswhen he liesinbed.At malecomplainsofquiteasevere A 60-year-old his mouthwithwaterbrash.Healsohascoughonoccasions.not lostanyweight. allhislifeandoccasionallyhasasourbittertastein region. Hehassufferedfromheartburn retrosternal A 65-year-oldmalepatientcomplainsofdysphagia,withfoodsticking in thelower suffered formanyyears.Hehaslostabout10 kginweightsincetheonsetofthesesymptoms. anyfoodorfluids.Hefeelsthisisdifferenttohis‘smoker’scough’whichhehas to swallow coughwhenhetries duration. Atpresenthecanonlytakefluids.Hehasquitesevere smoker,complainsofdysphagiatosolidfood2months’ A 65-year-oldmale,aheavy formanyyears. retrosternal region.Shehashadoccasionalheartburn A 35-year-oldfemalecomplainsofoccasionaldysphagiawithfoodstickinginthelower woken upbycoughing.Hehaslostabout7–8kginweight6months. liquidsthansolids.Recentlyhehasnoticedthatatnightis greater difficultyswallowing A 45-year-oldmalepatientcomplainsofdysphagia2years’duration.Heseemstohave havebecomemorefrequentandsevere. had thesesymptomsformanyyearsbutrecentlythey Thisisusuallyepisodic.Hehas patientcomplainsofchestpainonswallowing. A 50-year-old (Zenker’s)diverticulum Pharyngeal ring Schatzki’s Diffuse oesophagealspasm Achalasia Carcinoma Peptic stricture Oesophageal shortening Barrett’s oesophagus Stricture complications ofGORD? complications y sphagia y H G F E D Iron deficiency anaemia Iron deficiency Carcinoma Plummer–Vinson syndrome CREST Diffuse oesophagealspasm syndrome ➜ ➜ ➜ ➜ 7. D,E,F 6. D 5.A,B,C,D,E 4.B,C,D 3. A 2.B,E

Complications ofGORD Cancer oftheoesophagus Oesophageal emergencies D LOS isazone3–4cmlongofrelativelyhighpressure10–25 mmHg. cm,respectively,fromtheincisorteeth.Itislinedthroughoutbysquamousepithelium.The 40 of carcinoma. oesophagitis duetoretentionandfermentationoffoodaccountingfor theincreasedincidence carcinomainthedamagedsegment.Inachalasia,largeoesophagus develops developing general population.Incorrosivestricturethereisalifetimeriskofless than 5percentof which isprecancerous.Theincreasedriskofadenocarcinomaabout 25 timesthatofthe Barrett’soesophagus, maygoontodevelop resulting insquamouscarcinomasitu.GORD mayhavedysplastic changes diverticulum inalong-standingpharyngeal The mucosa the stomach. split,occursin percentthetear,informofavertical syndromein90 scan. InMallory–Weiss withacontrastCT Boerhaave’s syndromeusuallyrequiresanoperationafterthoroughevaluation arealmostalwaystreatedconservatively. Ontheotherhand, oesophagealperforations Cervical islikelytobelocalised. inacleanoesophaguswithoutobstructionandleakage small perforations stricture orcarcinomaasanunderlyingcause. treatment. Whenafoodbolusisstuck,onemustsuspectmechanicalobstructionsuchaspeptic intheoesophagusacontraststudyisnotindicated;itmayactuallyhinder In suspectedFB images. theUS distort soasnotto the stateofmediastinallymphnodes.ThebiopsyshouldbedoneafterUS incarcinoma.Thiswouldassessthelocalspreadand should alwaysbecarriedoutduringOGD andenableabiopsytobetaken.Endoluminalultrasound(EUS) presence ofahiatushernia(HH) oftheoesophagus,seedegreeoesophagitis,qualityrefluxingfluid, shortening ItwillindicatethesiteofLOSis OGD. fromtheincisors,therebyassessingifthereisany idealinvestigation Thepreliminary GORD. isnotagoodinvestigationtoevaluate Barium swallow GORD. indicating indicates gastriccontentsenteringtheoesophagusormouthandcausingheartburn, intothetracheobronchialtree.Refluxclassically overflow and thecontents(foodorsaliva) neurological aetiology. Regurgitationactuallyoccurswhenthereisobstructionoftheoesophagus phaseisofmuscularor (oralorpharyngeal) inthevoluntary Difficultyinswallowing excluded. a symptomofinflammationandulcerationalthoughcardiacoriginthepainshouldbe sticking’ isasinistersymptomandcarcinomamustberuledoutinitially. Odynophagiaisprimarily of‘food withahistory phaseofswallowing) Oesophageal dysphagia(duringtheinvoluntary y Most iatrogenic perforations of the oesophagus can be managed conservatively as they are as they oftheoesophaguscanbemanagedconservatively Most iatrogenicperforations sphagia 439

59: THE OESOPHAGUS ABDOMINAL ➜ 6F 5A 4B 3E 2C 1D matchingquestions Answers: Extended 440

1. D should alert onetothediagnosis,whichisconfirmedbyabariumswallow.should alert This resultsinrepeatedattacksofchestinfectionandistreatedassuch.Dysphagiawithhalitosis pouch. aretypicalofapharyngeal Classical featuresoffoodregurgitationwhenapersonliesdown timehecanbetreatedbyballoondilatation. carcinoma.Atthesame to exclude andbiopsy and nastytasteinthemouth.Hehasnotlostanyweight.requiresanurgentOGD waterbrash causingapepticstricture–heartburn, This patient’ssymptomsaretypicalofGORD oftheimpendingtracheo-oesophagealfistula). contemplated, althoughunlikelyinview scanofchestandabdomenlaparoscopicstaging(ifresectionis CT withEUS, urgent OGD recent onset,weightlossandtypicalsymptomofatracheo-oesophagealfistula.Heneedsan This patient,achronicsmoker,hasthehallmarksofanoesophagealcarcinoma–dysphagia time. confirm thediagnosisandtreatmentbydilatationiscarriedoutatsame will ring.OGD shouldarousethesuspicionofaSchatzki’s dysphagia whichisnotundulydisabling onandoffforyearscomplainsofoccasional A patientwhohasoccasionalheartburn andbiopsy.diagnosis isconfirmedbyOGD of achalasia.Thepatientalsohastypicalsymptomsregurgitationatnightwheninbed. forliquidswithsomeweightloss,isindicative A patientwithlong-standingdysphagia,particularly anycardiacpathology. shouldbedonetoexclude diagnosis. AnECG willconfirmthe byoesophagealmanometry followed oesophageal spasm.Abariumswallow onetothemotilitydisorderofdiffuse anddysphagiashouldalert Chest painonswallowing y sphagia 60 Stomach and duodenum

Multiple choice questions ➜ Anatomy of the stomach ➜ Helicobacter pylori and duodenum 4. Which of the following statements 1. Which of the following statements are regarding H. pylori are true? true? A The organism is round in shape. A The right gastric artery is a branch of B It has the ability to hydrolyse urea with the coeliac artery. the production of ammonia. B Vagal fibres to the stomach are afferent. C Some strains produce the cytokinins C The parietal cells are in the body of the cagA and vacA, which are associated with stomach and are the acid-secreting cells. the ability to cause gastritis, ulceration D The venous drainage of the stomach and cancer. ends in the inferior vena cava (IVC). D The incidence of infection in a general E The chief cells secrete pepsinogen. population appears to be high. E Treatment is recommended for infected ➜ Physiology of the stomach patients who are asymptomatic. and duodenum ➜ Gastritis 2. Which of the following statements are true? 5. Which of the following statements are A The ‘gastric phase’ of gastric acid true? secretion is mediated by the vagus A Type B gastritis is an autoimmune nerve. condition. B Hydrogen ions are exported from the B Type A gastritis affects the antrum. parietal cells via the proton pump. C Both types A and B gastritis predispose to C Cholecystokinin is secreted by the malignancy. duodenal endocrine cells. D Erosive gastritis due to NSAIDs is D The gastric motor activity is propagated mediated via inhibition of Cox1 enzyme. from the fundus and moves caudally at E Reflux gastritis is commonly seen after a rate of three per minute. gastric surgery. E Secretin increases gastric acid secretion. ➜ Peptic ulcers ➜ Gastric mucosal barrier 6. Which of the following regarding 3. Which of the following can damage the peptic ulcers are true? gastric mucosal barrier? A The term ‘peptic ulcer’ is a misnomer. A Non-steroidal anti-inflammatory drugs B Patients with gastric ulceration (NSAIDs) have increased levels of gastric acid B Alcohol production. C Trauma C ‘Kissing ulcers’ refers to adjacent gastric D Bile and duodenal ulcers. E Shock.

441 ABDOMINAL 10. Which of the following statements Whichofthefollowing 10. aretruewith Whichofthefollowing 9. are Whichofthefollowing 8. Commonsitesforpepticulcers 7. 442 E D C B A E D C B A E D C B A E D C B A E D therapy. All pepticulcerscanbehealedbyPPI duodenal ulcer. whendealingwitha mandatory Biopsy oftheulcermarginsis bowel. increased osmoticloadinthesmall isduetoasudden after gastricsurgery Late dumpingsyndromeoccurring all patientswithpepticulceration. Eradication therapyisgivenroutinelyto treatment. remainsthemainstay Surgery ulceration aretrue? regarding treatmentofpeptic Weight lossisatypicalsymptom. obstruction. may causegastricoutlet They Bleeding israre. Vomiting isanotablefeature. colic. this differentiatesfrombiliary radiatestothebackand The painnever peptic ulcers? featuresof regard totheclinical High socioeconomicgroups. Smoking NSAIDs H. pylori acid Gastric predisposing factorsforgastriculcer? Meckel’s diverticulum. gastric surgery Jejunum, suchasastomalulcerafter Oesophagus Fundus ofstomach ofduodenum First part include: proton pumpinhibitors(PPIs). All pepticulcerscanbehealedbyusing duodenal ulcers. than gastric ulcersandmalignancy There isagreaterassociationbetween infection ➜ ➜ ➜ 12. Which of the following aretruewith Whichofthefollowing 12. aretruewith Whichofthefollowing 11. 13. Which of the following aretruein Whichofthefollowing 13.

Haematemesis gastric operations Sequelae/complications after Gastric outletobstruction E D C B A E D C B A E D C B A after gastricoperations? regard tosequelae/complications (GOO)? respect ofgastricoutletobstruction regard tohaematemesis? Late dumpingsyndromeisdueto bowel. sudden highosmoticloadtothesmall Early dumpingsyndromeisduetoa absence ofgastricresection. inthe canoccureven Early satiety forpepticulcer.surgery Recurrent ulcerationisrareaftergastric paradoxical renalaciduria. mayoccurdueto Hypokalaemia isthemainstayoftreatment. Surgery Endoscopy hasnoroleinmanagement. hyperchloraemic alkalosis. The usualmetabolicabnormalityis gastric cancer. long-standing pepticulcerdiseaseand It ismostcommonlyassociatedwith malformation. arteriovenous Dieulafoy’s lesionisagastric junction. thegastro-oesophageal tear below tearisalongitudinal Mallory–Weiss patients. inelderly It isbettertoavoidsurgery common cause. Oesophageal varicesarethemost cent. isabout5per The in-patientmortality isusually10malignancy years. of operation anddevelopment The lag-phasebetweenthegastric reactive hypoglycaemia. ➜ 14. Which of the following areimportant Whichofthefollowing 14. 18. Which of the following are Whichofthefollowing 18. aretruewith Whichofthefollowing 17. areassociated Whichofthefollowing 16. statements Whichofthefollowing 15.

Gastric cancer A E D C B A E D C B A E D C B A E D C B A Haematogenous metastases in gastriccancer? evidenceofincurability unequivocal oesophageal lymphatics. the cardiahavenorelationto The lymphaticvesselsrelatedto gastric cancer. Sister Joseph’snoduleisdiagnosticof transcoelomic spread. associated withotherareasof Krukenberg’s tumoursarealways spread. occur intheabsenceoflymphnode Blood-borne metastasescommonly indicates incurability. Tumour usually reachingtheserosa regard tospreadofgastriccancer? instability.Microsatellite Inactivation ofp53 Mutations ingenecodingforE-cadherin Mutation ingenecodingforbeta-catenin APC gene in Mutation orlossofheterozygosity gastric cancer? of with themolecularpathology Trousseau’s signisdiagnostic. This isararecauseofGOO. supraclavicular fossa. node’intheright ‘Virchow’s Troiser’s signreferstoapalpable Anaemia canbeapresentingsymptom. stages. may benon-specificintheearly They aretrue? cancer featuresofgastric regarding clinical Smoking. gastricsurgery Previous Pernicious anaemia atrophy Gastric H. pylori in the causation ofgastriccancer? in thecausation infection ➜ 20. Which of the following statementsare Whichofthefollowing 20. 19. Which of the following aretrue Whichofthefollowing 19. 21. Which of the following statements Whichofthefollowing 21.

Gastric conditions E D C B A E D C B A E D C B E D C B A true? chemotherapy. There isadefiniteroleforneoadjuvant and75percent. 50 isbetween intheUK The 5-yearsurvival resection. D1 resectionissuperiortoaD2 cancer? regarding thetreatmentofgastric outletobstruction. Gastric Involvement ofadjacentorgans N3 nodaldisease Involvement ofdistantperitoneum symptomatic. Management isessentiallymedical and The conditionisusuallychronic. a diaphragmaticdefect. The conditionisusuallyassociatedwith common thanorganoaxialrotation. Mesenteroaxial rotationismore occurs inahorizontaldirection. In organoaxialvolvulustherotation true? regarding gastricvolvulusare female patients. Trichobezoars aremorecommonin (MALT). lymphoidtissue mucosa-associated T-cell-derived andarisefrom gastriclymphomasare Primary young people. lymphomaismorecommonin Gastric imatinib. apoorresponseto tumoursshow GIST oncogene mutation. are associatedwithtyrosinekinasec-kit stromaltumours(GIST) Gastrointestinal useful adjunct. tothegastricbedisa Radiotherapy after totalgastrectomybyaRouxloop. continuityisestablished Gastrointestinal 443

60: STOMACH AND DUODENUM ABDOMINAL ➜ ➜ Extended matchingquestions Extended Chooseandmatchthecorrect diagnosiswitheachofthescenariosgivenbelow: Chooseandmatchthecorrectdiagnosiswitheachofscenariosgivenbelow: 444

3 2 1 F E D C B A 2. Sequelaeofgastricsurger 6 5 4 3 2 1 F E D C B A 1. Differential diagnosisofupperGIpresentations tingling in his arms and legs. He is found to have macrocytic anaemiaonbloodtests. tingling inhisarms andlegs.Heisfoundtohavemacrocytic totalgastrectomypresentswithanaemiaand malewhohashad a previous A 54-year-old abdominal painwhichisburningin nature.Thisisepisodicandaggravatedbyalcohol. presentswith similarsymptomsasbefore.Hehasupper yearsbutnow symptoms forafew ofhis operation foraduodenalulcerwasfullyrelieved malewithaprevious A 50-year-old and aggravatedbyexercise. tremor, faintnessandprostration1.5 byfood haftereating.Thislastsfor 30minandisrelieved gastricresectionforhispepticulcer presentswith A 48-year-oldmalewhohashadprevious Vitamin B12 deficiency Postvagotomy diarrhoea Late dumpingsyndrome Malignancy Early dumpingsyndrome Recurrent ulceration pale andcachectic. A 63-year-oldmalepresentswithrapidlyprogressivedysphagiaandprofoundweightloss.Heis forherarthritis. NSAIDs A 78-year-old bleedandmelaena.Shehasbeenonlong-term femalepresentswithupperGI Sheisalsoconcernedaboutherpersistenthalitosis. sensation. hours.Shehasoccasionallywokenupwithachoking regurgitate ingestedfoodafterafew progressivedysphagia.Shetendsto ofslowly A 52-year-oldfemalepresentswithahistory black. food.Hementionsthathisstoolsaresometimes burning innatureandaggravatedbyspicy malepresentswithrightupperquadrantpainradiatingtotheback.Itisepisodic, A 40-year-old normal. otherwise but himtobeoverweight reveals and stressseemtomakeitworse.Clinicalexamination Smoking andheartburn. presentswithupperabdominaldiscomfort A 42-year-oldexecutive cachectic. He hasupperabdominalfullnessandlost3stonesinweight.isfoundtobepale A 63-year-oldmalepresentswithrepeatednon-biliousvomitingcontainingingestedfood. Achalasia cardia outletobstruction Gastric Gastritis Duodenal ulcer refluxdisease(GORD) Gastro-oesophageal oftheoesophagus Carcinoma y ➜ ➜ Chooseandmatchthecorrectdiagnosiswitheachofdescriptionsgivenbelow:

B A 4. Gastricoperations 9 8 7 6 5 4 3 2 1 I H G F E D C B A 3. Gastricconditions 6 5 4 Total gastrectomy Lap bandprocedure types: organoaxialandmesenteroaxial. Itiscommonlyassociatedwithadiaphragmaticdefect. This isonecauseofupperabdominal pain,vomitinganddifficultyineating.Thisisoftwo bleedingandobstruction. lead toulceration,perforation, foundinyoungfemalepsychiatric patients.Thiscan These areunusualandalmostexclusively pancreas. type1syndrome.Theseareusuallyfoundinthe duodenal looporthe neoplasia (MEN) ofmultipleendocrine This causesZollinger–Ellisonsyndrome.Thesemaybesporadicorapart are twocommoncomplicationsafterchemotherapy. arisefromtheMALTThey decade. Bleedingandperforation inthesixth andaremoreprevalent occur inthestomachandduodenum.Theirbiologicalbehaviourisdifficult topredict. These areassociatedwithamutationinthetyrosinekinasec-kitoncogene andcommonly anaemia andgastricatrophyaresomeofthepredisposingfactors. These areprincipallyoftwotypes–intestinalanddiffuse. adenoma typehasamalignantpotential. H. pylori The mostcommontypeofthisconditionisthemetaplastictype,whichassociatedwith strenuous vomiting. thegastro-oesophagealjunctioninducedbyrepetitiveand This isalongitudinaltearbelow which iscoveredbynormalmucosa. malformation bleedandisessentiallyagastricarteriovenous It isararecauseofupperGI polyps Gastric cancer Gastric volvulus Gastric Gastrinoma lymphoma Gastric Dieulafoy’s lesion tear Mallory–Weiss GIST Trichobezoar onset abdominalpain,vomitingandweightloss. gastrectomy30yearsagopresentswithrecent- apartial A 70-year-old malewhounderwent medications. andnotrespondingtousualantidiarrhoeal diarrhoea.Thisisassociatedwithurgency explosive and operationforpepticulcerpresentswithsevere malewhohashadaprevious A 46-year-old andaggravatedbymorefood. bylyingdown The attacklastforhalfanhourandisrelieved epigastric fullness,sweating,light-headednessanddiarrhoeaalmostimmediatelyaftereating. gastricresectionforhispepticulcerpresentswith malewhohashadprevious A 58-year-old infection. The other types include inflammatory andfundicglandtype.Thetrue infection.Theothertypesincludeinflammatory H. pylori infection,pernicious 445

60: STOMACH AND DUODENUM ABDOMINAL ➜ 1. C,E Answers: Multiplechoicequestions Chooseandmatchthecorrectoperations witheachofthedescriptionsgivenbelow: 446

Anatom 9 8 7 6 5 4 3 2 1 I H G F E D C producing histamineandDcells somatostatin. cells also hasnumerousendocrinalcells suchasGcellsproducinggastrin,enterochromaffin(ECF) fromparietalandchiefcells,thestomach isintothe splenicvein.Apart that ofthegreatercurve veinand isintotheportal Thevenousdrainageofthelessercurve branches ofthesplenicartery. whichareboth gastricarteries, andthe leftgastroepiploicandshort the gastroduodenalartery branchof the rightgastroepiploicartery, respectively. arethreearteries, Onthegreatercurvature branchesofcoeliacaxis andcommonhepaticartery, andtherightgastricartery gastric artery theleft –on thelessercurve, blood supplyofthestomachisderivedmainlyfromfivearteries The anastomoticarcades amongallarteries. The stomachhasarichbloodsupplywithexcellent deficiency. Patientsareadvisedtoeatsmallamountsatatimeafterthissurgery. Roux-en-Y loopofjejunum.VitaminB12 afterthisoperationtoprevent injectionsarenecessary majoroperationusuallydoneformalignancies.Continuationisrestoredbya This isacomplex direct communicationbetweentheposteriorwallofstomachandcyst. andinvolvescreatingawide This isanoperationtotreatchronicpancreaticpseudocysts Slippage anddysphagiaaretwocommoncomplications. rateoflessthan1percent. This isapopularprocedureformorbidobesitywithmortality effects andacceptablerateofrecurrence. incidenceofside- theparietalcellmassandhasalow This procedureselectivelydenervates oftheduodenum.Thishasahigherrateanastomoticleak. the firstpart halfofthestomachisresectedandgastricremnantdirectlyanastomosedto The lower isoperistaltic orantiperistaltic.Diarrhoeaisacommonpostoperativesequela. more oftenusedasapalliativeprocedureinmalignancies.Itcanbeanteriororposteriorand andispresently The wasacommongastricdrainageprocedureusedinpepticulcersurgery postoperative problemandhencethiswascombinedwithadrainageprocedure. stasisisacommon forpepticulcers.Gastric This isanon-selectiveoperationusedpreviously continuity isrestoredbydoingananastomosiswiththejejunum. byclosureoftheduodenalstump.The ofthestomachfollowed This involvesresectionofpart ing a This hasthebestlong-termresultsformorbidobesity. Itcanbedonelaparoscopicallyandisbecom- Billroth 2procedure Highly selectivevagotomy Truncal vagotomy Billroth 1procedure bypass Gastric Cystogastrostomy Gastrojejunostomy standard procedure for the morbidly obese. The operative mortality isaround2percent. standard procedureforthemorbidlyobese.Theoperativemortality y ofthestomachandduodenum ➜ ➜ ➜ ➜ ➜ 6.A,D,E 5.C,D,E 4.B,C,D 3.A,B,C,D,E 2.B,C,D

adjacent ulcerson theanteriorandposteriorwallsof theduodenum. ofgastricacidsecretion. Kissingulcersrefertotwo gastric ulcershave relativelynormallevels Patientswith ofpepticulceration.Theother majorfactoristheingestionofNSAIDs. development Peptic ulcers Gastritis Helicobacter pylori Gastric mucosalbarrier Ph Classically threephasesofgastricsecretionaredescribed:thecephalicphase,mediatedby Meckel’s diverticulum. It is now widely accepted that Itisnow Meckel’s diverticulum. and gastricsurgery mayoccurintheoesophagus, thestomafollowing stomach, butthey of ofduodenumandthelessercurve The commonsitesforpepticulcers arethefirstpart A gastritisarepronetogastriccancer. hypertrophy. Patientswithtype increased gastrinproductionandenterochromaffin-like(ECL) pernicious anaemia.IntypeAgastritis,theantrumisnotaffectedandhypochlorhydrialeadsto achlorhydria. ThisalsoaffectstheintrinsicfactorwhichcanleadtovitaminB12 and deficiency byhypochlorhydriaandfinally cell. Thisresultsintheatrophyofparietalcellmassfollowed Type Agastritisisanautoimmuneconditioninwhichtherearecirculatingantibodiestotheparietal who areinfected. patients withduodenalulcerdisease,butnotfornon-ulcerdyspepsiaorinasymptomatic is inverselyrelatedtosocioeconomicgroup.Atpresent,eradicationtherapyrecommendedfor unusual. Itappearsthatmostinfectionisacquiredinchildhoodandtheprobabilityof H. pylori resultsinincreasedgastrinreleaseduetoanegativefeedback. to produceammonia,analkali, Helicobacter pylori recover. thehighincidenceofstressulceration. Thisexplains to gastrointestinal tractthestomachismostsensitivetoischaemiaandalsoslowest ofthisbarrier.can leadtothebreakdown Tonometry thatintheentire studieshaveshown producedbythemucus-producingcellsofstomach.Manyfactors of mucopolysaccharides Itisaviscidlayer The gastricmucouslayerisessentialtotheintegrityofmucosa. by secretin,somatostatinandthepresenceofchymeinduodenum. thegastricphase,mediatedprincipallybygastrin;andintestinal vagus nerve; is characterised by reduction in blood supply to the superficial mucosa ofthestomach. mucosa is characterisedbyreductioninbloodsupplytothesuperficial and alcohol arecommoncauses.Stressgastritisisasequelafterserious illnessorinjury chelating orprokineticagentsmaybehelpfulinmanagement. Patients withpangastritisandthosedysplasticchangeshavemalignant potential. the patientispronetopepticulcerdisease.Intestinalmetaplasiaseen ingastritiswithatrophy. Type Bgastritisisassociatedwith Erosive gastritis is caused by all agents that disrupt the gastric mucosal barrier. NSAIDs and barrier.Erosive gastritisiscausedbyallagentsthatdisruptthegastricmucosal NSAIDs Reflux gastritisiscausedbyenterogastricrefluxandcommonaftergastric surgery. Bile y siolog infection is very common and population infection rates of 80–90 percentarenot commonandpopulationinfectionratesof80–90 infection isvery y ofthestomachandduodenum H. pylori is spiralshapedandresidesinthemucouslayerofstomach.Itsability is now classified by the WHO asaclass1carcinogen. classifiedbytheWHO is now H. pylori infection. Thiscommonlyaffectstheantrumand H. pylori is the most important factorinthe isthemostimportant 447

60: STOMACH AND DUODENUM ABDOMINAL ➜ ➜ ➜ 13. A,D, E 12. A,D,E 11. B,C,D,E 10. B,E 9. D 8.B,C,D 7. A,C,D,E 448

Gastric outletobstruction Haematemesis Sequelae/complications aftergastricoperations occurring intheepitheliumleastresistanttoaciddamage. The pepticulcerstendtooccuratajunctionbetweentwodifferenttypesofepithelium,theulcer management. remainsthemainstayof balloon dilatationhasbeentriedin benign pyloricstenosisbutsurgery metabolic abnormalitiesanddealing withthemechanicalproblem.Endoscopictreatment reduced circulatingionisedcalcium, whichmaycausetetany. Treatment involvescorrecting the ensuesalongwith dehydrationprogresses, hypokalaemia As in hypochloraemicalkalosis. loses weight,andappearsunwell dehydrated.Thevomitingofhydrochloricacidresults isgastriccancer.In recentyears,themostcommoncauseofGOO Thepatient commonly 6 unitsofbloodneedssurgery. younger patientsandhenceshouldhaveearlysurgery. Ingeneral,anyoneneedingmorethan to requiresurgicaltreatment.Elderlyandunfitpatientsaremorelikely to diefromthebleedthan vesseloranulcerwithaclotonthebase –arestatisticallylikely vessel atulcerbase,aspurting diathermy. –patientscontinuing tobleed,rebleed,visible Thecriteriaforsurgicalintervention numerous endoscopicmeasurestocontrolpepticulcerbleeding,such asinjectionsandargon by urgentinvestigationstodeterminethecauseofbleedinganddefinitive therapy. Thereare tear(4percent)andoesophagealvaricescent). (26 percent),Mallory–Weiss percent),erosions The mostcommoncausesofhaematemesisarebleedingpepticulcer(60 andinabout5percentofthesepatients,thesymptomsareintractable. ulcer surgery, peptic tosufferadegreeofdysfunctionfollowing Approximately 30percentofpatientscanexpect bilious vomiting,earlyandlatedumpingsyndrome,diarrhoeamalignanttransformation. arerecurrentulceration,smallstomach syndrome, sequelaeaftergastricsurgery The important unlikegastriculcers. of malignancy, duodenalulcerisnotrequiredastherenorisk load. Biopsyoftheulcermarginsaperforated due toreactivehypoglycaemiaandearlydumpingsyndromeissuddenincreasedosmotic The vastmajorityofuncomplicatedpepticulcersaretreatedmedically. Latedumpingsyndromeis bleed. Some maypresentwithchronicanaemiawhileothershaveacuteGI Allpepticulcersmaybleed. seen, althoughpatientswithgastriculcerstendtobeunderweight. Vomiting isnotanotablefeatureunlessstenosishasoccurred.Weight gainmaysometimesbe The painisepigastricandmayradiatetotheback.Itintermittentdemonstratesperiodicity. gastric ulcerstendtobeolder. andpatientswith world.Theincidenceisequalbetweenthesexes common inthedeveloping socioeconomicgroupsandareconsiderablymore inlower ulcersaremoreprevalent Gastric Whatever the cause, the principles of management are identical – initial resuscitation followed thecause,principlesofmanagementareidentical–initialresuscitationfollowed Whatever ➜ ➜ 20. A,E 19. C,D 18. A,B 17. A 16. A,B,C,D,E 15. A,B 14. A,B,C,D,E

Gastric conditions Gastric cancer The aetiology ofgastriccancerismultifactorial.Theotherpredisposingfactorsarepolyps, The aetiology This hasdramatically improvedtheprognosisofadvanced metastaticGIST. gastric Primary tumours. afterImatinibtherapyofGIST percentobjectiveresponse ratecanbeobserved An 80 the gastricbed,whichlimitsdose ofradiotherapythatcanbegiven. and75percent.Thereareanumber ofradiosensitivetissuesin between50 with a5-yearsurvival percent.InJapan,approximately75cent ofpatientshaveacurativeresection West is25–50 inthe ofmorbidityandmortality. beassociatedwithahigherlevel The5-year survival however, trunks)andissuperiortoD1resections(removaloftheperigastric nodesonly).Thismay, arterial D2 gastrectomyinvolvesen-blocresectionofthesecondtiernodes (clearance ofthemajor There aredebatesonthequestionofoperabilityinN3involvement. Involvement ofanotherorgan tostructuresthatcannot beremoved. the distantperitoneum,N4nodaldiseaseandfixation ofincurabilityishaematogenousmetastases, involvementof help them.Unequivocalevidence thatpatientswithincurablediseasearenotsubjectedtoradicaltherapycannot It isimportant with thelymphaticsofoesophagus. also seeninmanyotherabdominalmalignancies.Thelymphaticsatthecardiacommunicatefreely Tumour mayspreadviathefalciformligamenttoumbilicus(SisterJoseph’snodule),whichis metastases. Krukenberg’stumoursintheovariesmaybesolesiteoftranscoelomicspread. disease spreadslocallyandblood-bornemetastasesareuncommonintheabsenceoflymphnode tonotethatdistantspreadisunusualbeforethe blood andtransperitonealroutes.Itisimportant ofthestomachspreadsbyallmodes,includingdirectspread,lymphatic Carcinoma oftheirspectrum. havegastriccanceraspart polyposis colorectalcancer(HNPCC), k-Sam, c-ErbB2,TGF-alpha, EGFadVEGF. Well non- knownsyndromes,suchashereditary includingc-Met, factorsarealsoover-expressed, growth havebeenestablished.Several events genetic ofgastriccancerisstilltobecompletelyworkedoutbutseveral The molecularpathology (Trousseau’s sign)anddeepveinthrombosismaybeseen,butarenotdiagnostic. signifyingmetastases.Non-metastaticeffectssuchasthrombophlebitis supraclavicular fossa, lymphnodemaybepalpableintheleft to dysphagia,epigastricfullnessorvomiting.Virchow’s may occur. anaemia.Obstructionleads Thetumourfrequentlybleedsleadingtoirondeficiency bloating,distensionandvomiting high incidence(e.g.Japan).Inadvancedcancer,earlysatiety, ofsuspicionandusescreeningprogrammesinplaces and itthusrequiresahighindex toimprovingoutcomeisearlydiagnosis.Thesymptomsarenon-specificincases The key elucidated. butimperfectly important toN-nitrosocompoundsarealsoimplicated.Geneticfactors antioxidants andexposure of intake,deficiency salt andexcessive of dustfromindustrialsources.Dietisalsoimportant duodenogastric refluxandgastritis.Itisalsoassociatedwithcigarettesmokingingestion per se doesnotimplyincurabilityprovidedthatitcanberemoved. 449

60: STOMACH AND DUODENUM ABDOMINAL 21. A,C,D ➜ ➜ ➜ ➜ 1E,2I,3G,4C,5F,6H,7A,8D,9B 1D,2C,3I,4H,5B,6E,7F,8A,9G 1D,2A,3F,4B,5E,6C 1E,2B,3C,4F,5D,6A matchingquestions Answers: Extended 450

4. Gastricoperations 3. Gastricconditions 2. Sequelaeofgastricsurger 1. Differential diagnosisofupperGIpresentations approach. surgical treatmentistheonlysatisfactory the pylorus.Organoaxialvolvulusismostcommontype.Ifproblemcausingsymptoms, Rotation ofthestomachusuallyoccursaroundanaxisbetweentwofixedpoints–cardiaand aloneseemsappropriateforlocaliseddiseaseprocess. controversialbutsurgery is somewhat This remainsinthestomachforaprolongedperiodbeforeinvolvinglymphnodes.Thetreatment gastriclymphomasareB-cellderived,thetumourarisingfromMALT.decade oflife.Primary inthe6th lymphomas accountfor5percentofallgastricneoplasmsandismostprevalent Trichobezoars foundinyoungfemales,oftenwithpsychiatricproblems. arealmostexclusively y 61 The liver

Multiple choice questions ➜ Liver anatomy D Alanine transaminase (ALT) 1. Which of the following statements are 135–240 IU/L false? E Gamma-glutamyl transpeptidase A The liver is fixed in its place by peritoneal (GGT) 110–148 IU/L reflections called ligaments. F Albumin 35–50 g/L B The major part of the blood supply of G Prothrombin time (PT) 12–16 s the liver is derived from the hepatic artery. ➜ Clinical features of chronic C The portal vein lies posterior to the liver disease hepatic vein and common bile duct. 4. Which of these clinical features are D The bile duct, portal vein and hepatic not primarily a feature of chronic liver artery are contained in the lesser disease? omentum. A Jaundice E The portal vein is formed by the union of B Muscle wasting the superior mesenteric and left gastric C Bruising veins. D Oliguria 2. Which of the following statements are E Ascites and true? F Thirst A The left hepatic duct has a longer G Confusion. extrahepatic course than the right. ➜ Liver imaging B The major venous drainage is by three veins – right, middle and left hepatic 5. Which of the following statements are veins draining directly into the inferior true for imaging of the liver? vena cava (IVC). A Ultrasound (US) is the first-line C All the three major veins join the IVC investigation. within the liver parenchyma. B Radioisotope liver scan gives an idea D The functional lobes of the liver are of the extent of anatomical divided by the falciform ligament. derangement. E The liver is divided into eight functional C The ‘gold standard’ is contrast- segments – I to IV in the left hemi-liver enhanced computed tomographic scan and V–VIII in the right hemi-liver. (CECT). D Magnetic resonance imaging (MRI) has ➜ Liver function advantages over CECT. 3. Which of the following liver function E Angiography is best reserved when tests (LFTs) are abnormal? embolisation is contemplated. A Bilirubin 5–17 μmol/L ➜ B Alkaline phosphatase (ALP) Liver trauma 35–130 IU/L 6. Which of the following statements are C Aspartate transaminase (AST) false? 55–80 IU/L A Liver injuries are common.

451 ABDOMINAL ➜ Extended matchingquestions Extended Chooseandmatchthecorrectdiagnosiswith eachofthescenariosgivenbelow: Whichofthesestatementsaretrue? 7. 452

5 4 3 2 1 E D C B A 1. Chronic liverconditions F E D C B A E D C B pain of 3–4 months’ duration. Examination shows noabnormality. shows Liverfunctiontestsare pain of3–4months’ duration.Examination malecomplainsofrecentonset ofdull,aching,rightupperquadrant A fit-looking50-year-old congested liver. alarge scanofthelivershows hepatomegaly andascites.AlltheLFTs arederanged.CT shehas 3 episodesofsmallhaematemesis in thepast6months.Onexamination anddistension.Shehashad A 30-year-oldfemalecomplainsof abdominal discomfort prothrombin time.Shehashadrecurrent smallhaematemesisandhasascites. and being presentoverthelast3months.TheLFTsariseinbilirubin, thetransaminases show pruritus andjaundice,thelatter femalecomplainsofgeneralmalaise,lethargy, A 50-year-old bileducts. intra- andextrahepatic irregular narrowed shows attacks. Endoscopicretrogradecholangiopancreatography(ERCP) forthese apanproctocolectomyforulcerativecolitisandhasbeen well since,except underwent anobstructivepatternofthejaundice.Fiveyearsagoshe shows jaundice. Biochemistry femalepresentswithrecurrentepisodesofrightupper quadrantpainwith A 40-year-old intrahepaticductaldilatationwithstones. shows scan ajaundiceofobstructivenature.CT shows pain andjaundicewithitching.Biochemistry withrigors, right upperquadrant ofrecurrentattacksfever A 30-year-oldpatienthasahistory disease Simple cystic disease Caroli’s cirrhosis biliary Primary sclerosingcholangitis Primary Budd–Chiari syndrome requires urgentlaparotomy. A subcapsularorintrahepatichaematoma considered inmajorlivervascularinjury. Venovenous bypassshouldbe packing. isideallytreatedby crushinjury Severe by alongmidlineincision. isbestdone foraliverinjury Exploration conservatively. Blunt injuriesareusuallytreated Penetrating injuriesshouldbeexplored. role. Laparoscopy asaninvestigationhasa case oflivertrauma. scanmustbecarriedoutinevery CECT with pericardialorchestinjuries. Penetrating traumaisoftenassociated splenic, mesentericandrenalinjuries. Blunt traumaisoftenassociatedwith ➜ 8. Which of the following statementsare Whichofthefollowing 8.

E D C B A Portal h with sclerotherapyregimeorendoscopic Long-term beta-blockertherapycoupled ulceration. has lesschanceofoesophageal as opposedtoinjectionsclerotherapy oesophageal variceswithbanding Initial endoscopictreatmentof from gastricvarices. occursmostcommonly hypertension Acute haematemesisfromportal false? peritoneovenous shunt. incirrhosiscanbetreatedbya Ascites anastomosis. portocaval the idealchoiceissurgicalshuntof In faileddrugorendoscopictreatment, hypertension. portal banding isthemainstayoftreatment y pertension ➜ ➜ Choose and matchthecorrectdiagnosiswitheachofscenarios givenbelow: Chooseandmatchthecorrectdiagnosiswitheachofscenariosgivenbelow:

2 1 E D C B A 3. Livertumours 5 4 3 2 1 E D C B A 2. Liverinfections elevation of the transaminases andmuchraisedalpha-fetoprotein (AFP). ofthetransaminases elevation anenlarged liverwithamassintherightlobe. Liverfunctiontestsshow shows examination weight lossandoccasional fever. Hehadoneepisode ofhaematemesis.Abdominal malepatientpresents withdull,persistentupperabdominalpain,weakness, A 40-year-old well-circumscribed vascularsolidtumour. scandemonstratesa asinglewell-demarcated hyperechoicmassandCT ofthelivershows US aching pain.Sheisfitwithoutanyphysical findings.HerLFTs andotherbloodtestsarenormal. A 35-year-oldwomanwhoisonthe contraceptivepillpresentswithrightupperquadrant Hepatocellular carcinoma(HCC) liver metastasis Secondary Focal nodularhyperplasia Hepatic adenoma Haemangioma space-occupying lesionwithmultipleseptawithinit. right lobeoftheliver. asmooth scanshows raisedeosinophils.The CT Thebloodcountshows quadrant. Thepainisacontinuousdullacheandhasthefeaturesofmassarisingfrom malepatient,anativeofCyprus,presentswithpainfulmassintherightupper A 50-year-old scan. mass,afindingconfirmedonCT a multiloculatedcystic thereisweightlossandtenderhepatomegaly. shows Onexamination US quadrant discomfort. malaiseandrightupper fever, A 75-year-oldmalediabeticpatientcomplainsofanorexia, transaminases. atenderhepatomegaly. shows raisedbilirubinand examination Liverfunctiontestsshow jaundiceforthelast2weeks;hehasnopruritus.Abdominal weeks.Hehasdeveloped several andmalaisefor A 35-year-oldmalepatientpresentswithgeneralill-health,weightloss,anorexia is 1100 adilatedcommonbileductwithstones. shows US IU/L. scratch marksalloverherbodyandhepatomegaly. ShehasraisedserumbilirubinandtheALP sheisjaundiced with laparoscopiccholecystectomy. anuneventful Onexamination underwent epigastric painwithjaundice,hightemperaturerigorsanditching.Ten monthsagoshe rightupperquadrantand femalepatienthashadrecurrentattacksofcolicky A 50-year-old borders. quadrant withhepatomegaly. ahypoechoiccavityintherightlobewithill-defined shows US hehasatenderrightupper subcontinent wherehewasworkingfor6months.Onexamination has hadbloodstainedmotionsonandoffforthelast6weeksafterhereturnedfrom andweightloss.He A 25-year-oldmalecomplainsofgenerallyfeelingunwellwithfever Hydatid liverdisease Amoebic liverabscess Pyogenic liverabscess cholangitis Ascending Viral hepatitis lesion. cystic solitary normal, asisanuppergastrointestinalendoscopy. a6cm Anultrasoundofthelivershows 453

61: THE LIVER ABDOMINAL ➜ ➜ ➜ ➜ 5.A,C,D,E 4.D,F 3.C,D,E 2.A,B,E 1. B,E Answers: Multiplechoicequestions 454

Liver imaging Clinical features of chronic liverdisease Liver function Liver anatom 5 4 3 tumours and biliary pathology. It is operator-dependent. Radioisotope liverscanningdoes not pathology. Itisoperator-dependent.Radioisotope tumours andbiliary available.Itdetectsliver anduniversally Ultrasound isthefirst-lineinvestigation asitissafe indicatehepatocellulardamage. Derangement oftheseenzymes (ALTThe normaltransaminase ThenormalGGT is10–48 IU/L. andAST) are5–40 levels IU/L. intheright. V–VIII anatomical lobes.Theliverisdividedintoeightfunctionalsegments:segments I–IVintheleftand line)andnotthefalciformligament,which dividestheliverinto middle hepaticvein(Cantlie’s liver substance. right hepaticveinjoinstheIVCoutsideliverwhileothertwoemptyintowithin jejunum. Themajorvenousdrainageisbythreehepaticveins–right,middleandleft. a side-to-sideanastomosisiscarriedoutbetweentheleftintrahepaticductandRouxloopof This anatomicalsituationismadeuseofintherepairhighcommonbileductstrictureswhere coursewhereastherightoneisentirelyintrahepatic. The lefthepaticducthasalongerextrahepatic neck ofthepancreas. veinisformedbytheunionofsuperiormesentericandsplenic behindthe portal vein.The andbothlieanteriortotheportal ligament). Thebileductlieslateraltothehepaticartery arecontainedwithinthelesseromentum(hepatoduodenal veinandthehepaticartery duct, portal Thebile veinandtherestfromhepaticartery. blood supplyofthelivercomesfromportal percent) reflections. Thesearefalciformligament,rightandlefttriangularligaments.Themajor(80 The liverisfixedinsidetheperitonealcavitybyitsligamentswhicharevarious showed avascularlesionsurroundingsolidmasswithcentralscarring. showed scanwhich asolidlesionintheliver.tract wasnormalbutshowed ThereforeshehadaCECT ofthebiliary pain.TheUS forsuspectedbiliary anUS A 45-year-oldfemalepatientunderwent lesionswhichareblueincolour. oftheliverhasseveral the under-surface thesurgeonnoticedthat In a30-year-oldfemalepatient,duringlaparoscopiccholecystectomy, the masswithlackofenhancement. confirms asolidmassintherightlobeandCECT shows ofalltheparameters.US elevation arighthemicolectomyforcancerofthecaecum.LFTs show Three yearsagoheunderwent heisslightlyjaundiced.Hehasanenlargedliver.3 monthsandweightloss.Onexamination malepatientpresentswithdraggingpainintherightupperquadrantfor A 60-year-old The functional lobes of the liver are divided by a line between the gall bladder fossa andthe The functionallobesoftheliveraredividedbyalinebetweengall bladderfossa y ➜ ➜ 8.A,D 7. A,B,D,E 6.A,D

Portal h Liver trauma iscontemplated,asintraumaortumour.embolisation whentherapeutic hepaticangiographyismostlyreserved andMRI, can beobtainedfromCECT vascularinformation cannulation.As veinwithoutrecoursetoarterial andportal hepatic artery giveshigh-definitionimagesof tract;andmagneticresonanceangiography(MRA) of thebiliary images providesexcellent is noradiation;magneticresonancecholangiopancreatography(MRCP) noriskofallergicreactionasiodine-containingcontrastisnotused;there advantages overCECT: hasthefollowing andvenousphases.MRI the liverhilum.Intravenouscontrastgivesarterial uptheanatomicalrelationshipofstomachandduodenumto lesions. Oralcontrastshows infections andinflammations.Triple-phase, providesfinedetailofliver multi-slice,spiralCECT used, anadenomaorahaemangiomacanbediagnosed,asthesedonottakeupsulphurcolloid. obstruction.Whenaradioactivesulphurcolloidis screening testinsuspectedbileleakorbiliary upfunctionalderangementandisausefulnon-invasive anatomicalderangementbutshows show debris blockingtheshunt. peritoneovenous shunt–eitheraLe Veen any oraDenvershunt.Thelatterhelpstoevacuate ofa canbetreatedbyinsertion anastomosis.Ascites preference totheoperationofportocaval isthetreatmentofchoicein stentshunt(TIPSS) fails, transjugularintrahepaticportosystemic Whenthis hypertension. endoscopic sclerotherapyorbanding isthemaintreatmentforportal having alesserincidenceofoesophageal ulceration.Long-termbeta-blockertherapywith varices. Theinitialdefinitivetreatmentisendoscopicsclerotherapyorbanding, thelatter oesophageal mostoftenoccursfromlower hypertension Acute haematemesisfromportal resolves spontaneously. vein repair. andusually Asubcapsularorintrahepatichaematomadoesnot needanyintervention clampedtofacilitate cavalorhepatic theIVCtobesafely vein tothesuperiorvenacava;thisallows damage, thepatientisputonavenovenousbypass.Inthiscannula is passedfromthefemoral upwardsforamediansternotomy. which canbeextended isideallycarriedoutbyarooftopincision forliverinjury continuetobestable.Exploration as they aslong scanandbetreatedconservatively conservatively. StablepatientsneedtoundergoaCECT to theoperatingtheatrewhileactiveresuscitationisunderway. Bluntinjuriesaremostlytreated Thepatientshouldbetransferred amounts ofbloodreplacementwillneedurgentexploration. rightchestandabdominalstabwound,requiringlarge suchasalower A penetratinginjury, suspected, inthestablepatient,alaparoscopyiscarriedouttolookfordiaphragmaticrupture. scan.Whenhaemoperitoneumis suspected ofhavingliverdamageshouldundergoaCECT withoutwastingtimeonascan.Ontheotherhand,stablepatients whoare to theatreforthwith trauma. and mesentery. Stabandgunshotwoundscausingpenetratinginjuriesareassociatedwithchest trauma isoftenassociatedwithdamagetoneighbouringstructures,suchasthespleen,kidneys isserious.Blunt thoraciccage.Thereforewhentheliverisdamaged,overallinjury by thelower Liver injuriesarerarebecauseofitsanatomicalpositionunderthediaphragmwhereitisprotected CECT isthe‘goldstandard’forliverimaginginmostclinicalsituations–trauma,tumour, CECT CECT scan is not done in every caseoflivertrauma.Ifunstable,thepatientneedstobetaken scanisnotdoneinevery CECT Severe crush injury is treated by packing and re-exploration after48h.In majorlivervascular istreatedbypackingandre-exploration crushinjury Severe y pertension 455

61: THE LIVER ABDOMINAL ➜ ➜ 4C 3A 2B 1D 5E 4A 3C 2B 1D matchingquestions Answers: Extended 456

2. Liverinfections 1. Chronic liverconditions aspiration. scanfindingsarediagnostic. Thepatientistreatedwithantibiotics and andCT this patient.TheUS Pyogenic liverabscessusuallyoccurs intheelderlyandinfirmwhoareimmunocompromised,asis bleeding andascites.Diagnosisisconfirmed bytheantibodytitretoinfectiveagent. patients maypresentacutelywithfulminating liverfailureoratalatestagewithcirrhosis,variceal of liverdiseaseworldwide.Hepatitis Bisthemostseriousandlatercancauseahepatoma.Some Viral hepatitiscanbeduetoA,BandC.HepatitisCisoneof the mostcommoncauses andendoscopicpapillotomy.antibiotics, vitaminKandanERCP TheLFTsanobstructivepicture.She needstobetreatedwith willshow stone asseenontheUS. fromaretained This patienthasclassicalCharcot’sintermittenthepatictriadalmostcertainly coloured. diagnostic –thecontentsareusuallysterilebutaspirateischaracteristically chocolate- bydiagnosticaspirationofthecontentsareboththerapeutic and oftheliverfollowed US andisolation oftheparasite. amoebic abscess.Thediagnosisisconfirmedbystoolexamination withan Having returnedfromastayinthesubcontinent,thispatienthasamoebicdysentery symptoms fromit,laparoscopicderoofingshouldbeconsidered. or variationindensitywithinthecavity. areleftalone.Butasthispatienthas cysts Asymptomatic regular, thin-walledunilocularspace-occupyinglesionwithoutanysurroundingtissueresponse confirmedonultrasoundwithtypicalfindings:a hepaticcyst This patienthasasimplesolitary abiopsy.hepatic venographyviathetransjugularroutewhichmayallow scanisdiagnostic.Confirmationby ascites andoesophagealvarices.CT hypertension, portal deficiency. obstructioncauses acongestedliver,impairedliverfunction, Thehepaticvenousoutflow myeloproliferative disorderoraprocoagulantstateduetoantithrombin3,proteinCS This isBudd–Chiarisyndromewheretherehepaticveinthrombosiscausedbyanunderlying hepatocellulardysfunction.Confirmationisbyliverbiopsy.function testsshow cirrhosis,aconditionofgradualonset.Theliver biliary This femalepatientsuffersfromprimary acholangiocarcinomainfuture. out becauseshemaydevelop shouldbecarried isdiagnostic.Aclosesurveillance cholangitis hascontinuedunabated.TheERCP sclerosing herprimary Although herulcerativecolitishasbeentreatedsuccessfullybysurgery, sclerosingcholangitis. This patientwhohassufferedfromulcerativecolitissuffersprimary scanconfirmsthediagnosisofthiscongenitalcondition. duct stones.TheCT This patienthasrecurrentattacksofobstructivejaundicewithfeaturessepsis–typicalbile ➜ 5C 4A 3D 2E 1B 5E

3. Livertumours antibodies to hydatid antigen in the enzyme-linked immunosorbent assay (ELISA) and typical CT andtypicalCT (ELISA) immunosorbentassay antibodies tohydatidantigenintheenzyme-linked serologicaltestfor The diagnosisisconfirmedbyeosinophiliaonhaematologicalexamination, oftherightlobeliver. thispatienthasahydatidcyst anativeofMediterraneancountry, As tumours. ThusasulphurcolloidscanwhichistakenupbyKupffercellswouldbediagnostic. in few andKupffercellswhicharevery Focal nodularhyperplasiacontainsbothhepatocytes imagesaretypical. andCT imaging.US andCT increasingly diagnosedwithimprovementsinUS Focal nodularhyperplasiaisthesecondcommonestbenignliverlesion,anincidentalfinding, best leftaloneinthevastmajority. are theclassical‘lightbulb’sign.They willshow MRI reporting. inUS the availabilityofexpertise This isaliverhaemangioma,thecommonestbenignlesion,increasinglybeingdiagnosedwith thorough reassessmentsothathecanbeconsideredforhepaticresection. needsa secondaries intheliver:jaundice,hepatomegalyandalivermassonimaging.Henow hastheclinicalfeaturesof This patientwhohadacoloniccancerremoved3yearsagonow Therefore, oneofthechoicesfornon-surgicaltreatmentisconsidered. scanofchest.Mostpatientsarenotamenableforresection. bonescanandCT CECT, contrast MRI, Heneedstobethoroughlystagedwith This malepatientclinicallyhasthefeaturesofaHCC. potential andthereforelargesymptomatictumoursarebestresected. arethoughttohaveamalignant tohaveproducedtumourregression.They the pillisknown vascularityofthesetumours.Stopping findings. Biopsyiscontraindicatedbecauseoftheextreme andCT This womanonthecontraceptivepillhasahepaticadenomaconfirmedUS andmultiplesepta. scan findingsofafloatingmembranewithinthecyst 457

61: THE LIVER 62 The spleen

Multiple choice questions ➜ Anatomy B During splenectomy the tail of the 1. Which of the following ligaments are pancreas can be damaged. connected to the spleen and keep it in C A gastric or pancreatic fistula can occur as its place? a post-splenectomy complication. A Gastrocolic omentum D In a left hemicolectomy the spleen can B Lienorenal ligament be in danger. C Phrenicocolic ligament E The risk of opportunist post-splenectomy D Gastrophrenic ligament infection (OPSI) is greatest after the first E Gastrosplenic ligament. 6 months of splenectomy. 2. Which of the following statements are ➜ Splenic abnormalities false? 4. Which one of the following statements A The spleen lies in front of the left 10th, is false? 11th and12th ribs. A Splenunculi are present in approximately B The splenic artery arises from the coeliac 10–30 per cent of the population. axis. B Splenic artery aneurysm can occur as a C The inferior mesenteric vein empties into complication of acute pancreatitis. the splenic vein. C Plain abdominal X-ray is the ideal D The tail of the pancreas lies in the imaging modality. lienorenal ligament. D A massively enlarged spleen is prone to E The inner surface of the spleen has two infarction. impressions – gastric and colic. E The splenic hilum is a common site for ➜ the development of a pseudocyst of the Splenectomy pancreas. 3. Which of the following statements are true? A Partial splenectomy can result in splenic regeneration. Extended matching questions ➜ 1. Splenic pathology A Idiopathic thrombocytopenic purpura (ITP) B Hereditary spherocytosis C Splenic infarction D Splenic artery aneurysm E Splenic rupture Choose and match the correct diagnosis with each of the scenarios given below 1 A male patient of 55 years, known to suffer from myelofibrosis, presents with onset of severe left upper quadrant and shoulder tip pain of 8 h duration. On examination he has a

458 ➜ ➜ ➜ 4. C 3.A,B,C,D 2.A,E 1. B,C,E Answers: Multiplechoicequestions

Splenic abnormalities Splenectom Anatom 5 4 3 2 destruction ofred bloodcells. sitefor scanning isusefultofindoutifthespleenanimportant similarly useful.Radioisotope is isCECT.The idealimagingmodalityforsplenic pathology Magneticresonanceimaging(MRI) 2–3 yearsaftersplenectomy. left hemicolectomy. isduringthefirst TheperiodduringwhichapersonismostatriskforOPSI tothespleenisa as apostoperativecomplication.Thecommonestcauseofiatrogenicinjury the pancreatictailandstomachisnotrecognised,thenagastricor pancreaticfistulacanoccur Ifdamageto gastricarteries. stomach canbedamagedduringligationanddivisionoftheshort ofthe andveinatthehilum. Thegreatercurve be damagedwhileligatingthesplenicartery regeneration oflosttissuewithnoreductioninsplenicfunction.Thetail ofthepancreascan splenectomyintraumashouldalwaysbeattemptedbecausethereisrapid If possible,partial renal andpancreaticimpressions. ofthespleenhasgastric,colic, shouldbesuspected.Theinner(visceral)surface splenic injury The spleenliesinfrontoftheleft9th,10th and11th ribs.Thereforeinafractureoftheseribs, gastricvessels. ligament; and,anteriorly,itisthegastrosplenicligamentthatcontainsshort andvein;totheanterolateralabdominalwallbyphrenicocolic contains thesplenicartery The spleenisconnectedtotheposteriorabdominalwallbylienorenalligament,which in theepigastrium.Abruitwasheardepigastricregion.Shedoesnothaveanysymptoms. wasfoundtohaveacalcified1cmmass A 30-year-oldwomanduringherultrasoundforpregnancy whenhehadsufferedfrommalaria. tropical country and reboundtendernessinhislefthypochondrium.Untiltheageof18 yearshelivedina left upperabdomen.Inhospitalhecomplainedofpaininhisshouldertipandhadrigidity withintensepaininhis minutes,butthenhefeltdizzy and continuedtoplayforanotherfew A 25-year-oldmalewaskickedinhisabdomenaccidentallywhileplayingfootball.Hegotup megakaryocytes. largenumberofplatelet-producing reveals plateletcount.Bonemarrow anaemia andlow of skinecchymoses.Thecoagulationandbleedingtimesarenormal.bloodtestsshow noorganomegalybutmanyareas shows suffers frommenorrhagia.Abdominalexamination A 30-year-oldfemalepatientpresentswithpetechialhaemorrhagicspotsinherskin.She multiplesmallgallstonesandconfirmssplenomegaly.shows hehaspallor,jaundiceandsplenomegaly. Onexamination Anultrasoundofhisabdomen anorexia. An 18-year-old malepresentswithintermittentgeneralisedabdominalpain,mildjaundiceand defectinanenlargedspleen. aperfusion shows tomography scan(CECT) temperature andmarkedtendernessoveranenlargedspleen.Acontrast-enhancedcomputed y y 459

62: THE SPLEEN ABDOMINAL ➜ 5D 4E 3A 2B 1C matchingquestions Answers: Extended 460

1. Splenicpatholog aneurysm by interventional radiology. byinterventional aneurysm Hence aseriousconsiderationshouldbegiventoprophylacticallytreat thissplenicartery years and25percentoccurinpregnantwomenusuallythethirdtrimester orduringlabour. rupture.Fiftypercentofcasesruptureoccurinpatientsyoungerthan45 silent unlessthey Theseareusually aneurysm. This pregnantwomanhastheincidentalfindingofasplenicartery byoperation,isthedefinitivemanagement. (if timepermits),followed orCECT byanUS causedruptureofhisspleen.Resuscitationfollowed during agameofcontactsport wherehesufferedfrommalaria,whichwouldhavecausedanenlargedspleen.Trivialcountry trauma ofhislifeinatropical This youngmanhasthehallmarksofarupturedspleen.Hespentearlypart remainder theoperationwillmakenodifference. 15of patientswillbecuredbysplenectomy;afurther percentwillbeimproved,whilstinthe is considered.Responsetosteroidsindicatesagoodresponsesplenectomy. Uptotwo-thirds andthepatienthastworelapsesonsteroidtherapy,splenectomy If theplateletcountremainslow plateletcountpointstothediagnosisofITP.Her low Thespleenispalpableinaminorityofcases. This 30-year-oldfemalehassignsofpetechialhaemorrhagesandgeneralisedbleedingtendency. shouldbecarriedout. sequestration. Splenectomyandcholecystectomy thespleentobesiteofredcell redcellswillshow chromium labellingofthepatient’sown Radioactive anaemia,apositivefragilitytestandlargenumberofreticulocytes. picture willshow pigmentstones).Theblood hyperbilirubinaemia, splenomegalyandgallstones(almostcertainly anautosomaldominantdisorder. Hehasunconjugated spherocytosis, This patienthashereditary denoting asplenicabscess,splenectomyiscarriedout. managementistriedbutifthepatientcontinuestobeseptic, confirms thediagnosis.Conservative orsplenicveinthrombosis.CECT hypertension such asinmyeloproliferativedisorders,portal This patienthassplenicinfarction,aconditionthatoccursinthosewithmassivesplenomegaly, y 63 The gall bladder and bile ducts

Multiple choice questions ➜ Anatomy of the gall bladder C An ‘end-viewing’ endoscope is and bile ducts used during endoscopic retrograde cholangiopancreatography (ERCP) to 1. Which of the following statements are cannulate the ampulla. true? D Biliary scintigraphy can be helpful in the A The normal capacity of the gall bladder is diagnosis of cholecystitis, bile leaks and 250 mL. iatrogenic obstruction. B The cystic duct always joins the E Magnetic resonance common hepatic duct (CHD) above the cholangiopancreatography (MRCP) has duodenum. excellent diagnostic and therapeutic C The cystic artery is a branch of the right applications in bile duct disorders. hepatic artery. D Moynihan’s hump refers to the neck of ➜ Congenital abnormalities the gall bladder. E The lymphatics from the gall bladder of the gall bladder and drain into the cystic lymph node of Lund. bile ducts 4. Which of the following statements are ➜ Physiology of the gall true? bladder A The gall bladder is never absent. 2. Which of the following statements are B The ‘Phrygian cap’ refers to the true? septum present in the gall bladder in A The approximate water content of bile approximately 2–6 per cent. when it leaves the liver is 50 per cent. C An accessory cholecystohepatic duct may B The liver excretes bile at an approximate be present, passing directly into the gall rate of 40 mL/h. bladder from the liver. C The rate of bile secretion is primarily D Caroli’s disease refers to the multiple controlled by secretin. saccular dilatations of the extrahepatic D The sole function of the gall bladder is to ducts. act as a reservoir for bile. E Choledochal cyst increases the risk of E The gall bladder secretes about 20 mL of cholangiocarcinoma. mucus daily. ➜ Gallstones ➜ Investigations on the biliary 5. Which of the following statements tract regarding gallstones are true? 3. Which of the following statements are A Cholesterol stones are more common true? than pigment stones in the UK. A CT scan is more sensitive than ultrasound B Biliary colic is typically present in for gallstones. 10–25 per cent of patients. B A plain radiograph can show radio- C The risk of having acute cholecystitis in a opaque gallstones in 10 per cent of person with asymptomatic gallstones is patients. 10 per cent/year.

461 ABDOMINAL ➜ ➜ 8. Which of the following statements Whichofthefollowing 8. statementsare Whichofthefollowing 7. 6. Which of the following statements Whichofthefollowing 6. 462

B A Cholec E D C B A Gall bladderconditions E D C B A E D difficult openoperations. A fundus-firstapproachishelpfulin 0.05 percent. is The incidenceofbile-ductinjury demonstrated bycholecystography. ofthegallbladdercanbe Diverticulosis >1 cmareidentified. isadvisedifgallbladderpolyps Surgery bacteria intheirbile. Typhoid typhoid carrierscanexcrete andesthers. crystals submucous aggregationsofcholesterol gallbladder’isduetothe ‘Strawberry significance. A ‘porcelaingallbladder’hasnoclinical course. hasamildclinical Acalculous cholecystitis surgery. conversion ratecomparedwithelective associated withafivetimesgreater is in apatientwithacutecholecystitis Urgent laparoscopiccholecystectomy absence ofjaundice. inthe management ofacutecholecystitis Antibiotics arenotrequiredinthe treatment. respondtoconservative cholecystitis Ninety percentofpatientswithacute cholecystectomy. should routinelybeadvisedtohavea Patients withasymptomaticgallstones are true? regarding thetreatmentofgallstones acute inflammation. Murphy’s signsuggeststhepresenceof with gallstones. lawisusuallyassociated Courvoisier’s regarding cholecystectomy aretrue? regarding cholecystectomy true? y stectom y ➜ ➜ ➜ 10. Which of the following predisposeto Whichofthefollowing 10. statements Whichofthefollowing 9. 11. Which of the following regardinggall Whichofthefollowing 11.

Gall bladdercarcinoma Bile ductcarcinoma Obstructive jaundice E D C B A E D C B A E D C E D C B A true? regarding obstructivejaundiceare bladder carcinoma aretrue? bladder carcinoma bile ductcarcinoma? Charcot’s triadconsistsofpain,stones stones variesfrom5to8percent. The incidenceofsymptomaticbileduct dissected inMirizzi’ssyndrome. ductisfreeandeasily The cystic in 15 percentofthepatients. syndromeoccurs Post-cholecystectomy mandatory. Cholangiogramis Routine Per-Operative Prognosis isgenerallygood. A palpablemassisanearlysign. of thecases. The CA19–9 percent in80 iselevated The majorityareadenocarcinomas. It isararedisease. Colorectal carcinoma. Choledochal cyst Sclerosing cholangitis bladder stones Gall Ulcerative colitis stricture. strictureisahilar Bismuth type3biliary duct carcinoma. Clonorchiasis canpredisposetobile smoothmuscleantibodies. elevated hypergammaglobulinaemia and isassociatedwith (PSC) sclerosingcholangitis Primary and jaundice. ➜ ➜ Extended matchingquestions Extended Choose and match the correct gallstone type witheachofthedescriptions given below: Chooseandmatchthecorrectgallstonetype Chooseandmatchthecorrectdiagnosiswitheachofscenariosgivenbelow:

2 1 D C B A 2. T 8 7 6 5 4 3 2 1 H G F E D C B A 1. Clinicalconditionscausedb presence offoreign bodiessuchasendoprosthesisor parasites. deconjugation of bilirubin diglucuronidebybacteria. These arealsoassociatedwiththe Bile stasisandinfectedbilepredispose tothesestones.Thisstoneformationisrelated medicationsincreasetherisk ofthesestones. certain andpaleovoid inappearance.Obesity,high-caloriedietsand areusuallysolitary They pigmentstones Brown Black pigmentstones Cholesterol stones Mixed stones chills. with ofepisodicabdominalpain, jaundiceandfever malepresentswithahistory A 60-year-old upperabdominaltendernesswithsomeguarding. reveals Clinical examination andinshock. uncomfortable This isassociatedwithrepeatedvomitingandretching.Hevery epigastricpainradiating totheback. A 42-year-oldmalepresentswithasuddenonsetofsevere sounds. adistendedabdomenwithincreasedbowel reveals examination days.Clinical abdominal painandvomiting.Shehasalsobeenconstipatedforthepastfew A 78-year-oldyearspresentswithcentralcolicky gallstonesforseveral femalewithknown palpable gallbladder. Sheisafebrileandsystemicallywell. tohaveanon-tender isobserved femalebeingtreatedforacutecholecystitis A 40-year-old radiates tothebackandrightshoulder. sheisafebrilewithasoftabdomen. Onexamination This abdominalpainovertheRUQ. ofcolicky A 38-year-oldfemalepresentswitha6hhistory generalised guardingandreboundtenderness. unwellandinshock.Theabdomenisdistendedwith heisvery pain. Onexamination andconstantgeneralisedabdominal ofsevere malepresentswitha6hhistory A 60-year-old anincreasedwhitecellcount. underlying, vaguelypalpable,tenderlump.Bloodtestsreveal unwellwithspikingfever.is very Hehassignsofrightupperquadrantperitonismwithan upperabdominalpain.He ofsevere malepresentswithaweek-longhistory A 56-year-old associatedwithvomiting.SheisfebrileapositiveMurphy’ssign. quadrant (RUQ) ofconstantpainovertherightupper A 30-year-oldfemalepresentswitha2-dayhistory peritonitis bladder perforation/biliary Gall bladder mucocele Gall Acute cholecystitis Empyema ofthegallbladder ileus Gallstone Pancreatitis Cholangitis colic Biliary y pes ofbiliar y stones y gallstones 463

63: THE GALL BLADDER AND BILE DUCTS ABDOMINAL ➜ Chooseandmatchthecorrectmanagementwitheachofscenariosgivenbelow: 464

8 7 6 5 4 3 2 1 K J I H G F E D C B A 3. Managementofbiliar 4 3 empyema ofthegall bladder. Percutaneous drainageisunsuccessful andhenceitisdecidedto A 53-year-oldmanhaspresentedwith anacuteabdomenandhasbeendiagnosedtohave unsuccessful instentingthestricture. are the hilum.Acontractedgallbladder andnormalbileductareseen.AttemptsatERCP tender liver. dilatedintrahepaticbileductswithamassat scansshow Ultrasoundand CT apalpable reveals A 76-year-old malepresents withprogressivepainlessjaundice.Examination functions. abnormalliver confirms gallbladderstonesanddilatedbileductswithstones.Blood tests show The painradiatestoherbackandisassociatedwithdyspepticsymptoms. Ultrasoundscan painforthepast 3months. RUQ femalehasbeenhavingepisodesofcolicky A 40-year-old ofspread. is consideredtoberesectablewithnoevidence andthetumour small, malignant-lookingtumourattheampulla.Biopsiesconfirmmalignancy palpable non-tendergallbladder. adilatedbileductwith scans show UltrasoundandCT a reveals femalepresentswithprogressivepainlessjaundice.Examination A 59-year-old head tumourwhichisnotresectable. non-tender gallbladder. adilatedbileductwithpancreatic scansshow UltrasoundandCT apalpable reveals malepresentswithprogressivepainlessjaundice.Examination A 60-year-old normalliverfunctions. gall bladderstoneswithnormalbileducts.Bloodtestsshow pain radiatestoherbackandisassociatedwithdyspepticsymptoms.Ultrasoundscanconfirms painforthepast3months.The RUQ femalehasbeenhavingepisodesofcolicky A 40-year-old abdominal ultrasound. A 92-year-oldfemalewithmultipleco-morbiditiesisfoundtohaveincidentalgallstoneson obstructionwithpneumobilia. of small-bowel sounds.AbdominalX-rayconfirmsfeatures adistendedabdomenwithincreasedbowel reveals days.Clinicalexamination pain andvomiting.Shehasalsobeenconstipatedforthepastfew abdominal ofgallstonespresentswithcentralcolicky A 79-year-old femalewithalonghistory Cholecystostomy Subtotal cholecystectomy Whipple’s operation No activetreatment andcholedocholithotomy Cholecystectomy Laparotomy –enterotomyandremovalofstone –palliativebypass Surgery bycholecystectomy followed –stoneextraction ERCP Percutaneous transhepaticcholangiography(PTC) andantegradestenting –stenting ERCP Laparoscopic cholecystectomy spherocytosis. conditionssuchassicklecellanaemiaandhereditary with haemolytic These areusuallyamorphousandcontainaninsolublebilirubinpolymer. areassociated They structure oncross-section. haveacrystalline These arethecommonestvarietyandusuallymultiplefaceted.They y problems ➜ ➜ ➜ 3.B,D 2.B,E 1. C,E Answers: Multiplechoicequestions

10 10 Investigations onthebiliar Ph Anatom 11 9 has notherapeutic application. disordersbut anddiagnosisofbiliary ishelpful inevaluation cholangiopancreatography (MRCP) tohelplocatethepositionof ampullaandhelpincannulation.Magneticresonance an ERCP endoscopeisusedduring occasionally, thepresenceofstones inthebileduct.Aside-viewing and, presence ofinflammationaroundthe gallbladder,sizeofthecommonbileduct(CBD) also giveinformationonthesizeof thegallbladder,thicknessofbladderwall, Itcan non-invasive,involvesnoradiationandisquick toperform. available, inexpensive, Abdominal ultrasoundistheinitialimagingmodalityofchoiceasitaccurate, readily concentrating thebileandsecretionofmucus. forthebile, cholesterol andcalcium.Thefunctionsofthegallbladderincludeacting asareservoir bile getsconcentrated5to10 bilepigments, times,withacorresponding increaseinbilesalts, Thehepatic whichisreleasedfromtheduodenal mucosa. (CCK), controlled bycholecystokinin and1percentpigments,cholesterolfattyacids.Therate of bilesecretionis cent bilesalts The compositionofthebileasitleavesliverisapproximately97 percentwater,1–2 of Luschka. multipleindentationscalledthecrypts bladder mayshow ofthegall duct.Themucosa whichmayruninfrontofthecystic course oftherighthepaticartery the bileductbeforejoiningit.Theterm‘Moynihan’shump’refersto‘caterpillar turn’–atortuous of retropancreaticpart intotheretroduodenaloreven down per centofcasesbutmayextend sphincteric structurecalledthesphincterofLutkens.Itjoinssupraduodenalsegmentin80 asthevalvesofHeisterandissurroundedbya ductisarrangedinspiralfoldsknown cystic ofthe canbevariablebutisaround3cmlong.Themucosa ductlength mL.Thecystic 35–50 The gallbladderisapear-shapedstructure,7.5–12 cmlong,withanormalcapacityofabout head tumour which is not resectable. ERCP isunsuccessful. head tumourwhichisnotresectable.ERCP adilatedbileductwithpancreatic scansshow of gastricoutletobstruction.UltrasoundandCT apalpablenon-tender gallbladderandfeatures reveals unable totolerateorally. Examination A 62-year-oldmalepresentswithprogressivepainlessjaundice.Healsohasvomitingandis toremovethebileductstonesareunsuccessful. functions. AttemptsatERCP abnormalliver confirms gallbladderstonesanddilatedbileductswithstones.Bloodtestsshow The painradiatestohisbackandisassociatedwithdyspepticsymptoms.Ultrasoundscan painforthepast8months. RUQ malehasbeenhavingepisodesofcolicky A 46-year-old triangle andreliablyidentifythestructures. open operation.Afundus-firsttechniqueisusedbutitstilldifficulttodissecttheCalot’s toan triangleandhencehadtobeconverted dense adhesionswerepresentintheCalot’s malewasadmittedforanelectivelaparoscopiccholecystectomy. Atoperation, A 58-year-old difficult. There aredensevascularadhesionsaroundthegallbladderwhichmakesdissectionvery frankpus. reveals operate onhim.Atoperationthegallbladderistenseanddistended.Aspiration y siolog y ofthegallbladderandbileducts y ofthegallbladder y tract 465

63: THE GALL BLADDER AND BILE DUCTS ABDOMINAL ➜ ➜ ➜ ➜ 8.A,B,D 7. B,C,D,E 6.B,D 5.A,B,E 4.B,C,E 466

Cholec Gall bladderconditions Gallstones Congenital abnormalitiesofthegallbladderandbileducts Extrahepatic biliary atresia is present in approximately 1 in 12 000 births. The extrahepatic bile atresiaispresentinapproximately112 biliary Theextrahepatic 000births. Extrahepatic bladder shouldbe opened,thestoneremovedand infundibulumoversewn. (stone ulceratingthroughintothebile duct)and,ifthisissuspected,theinfundibulumofgall patients andtoclarifyanatomy. triangledissectionmaybehazardousinMirizzi’ssyndrome Calot’s butmaybehelpfulin selected isnotperformed Cholangiogram(POC) Routine Per-Operative therefore anindicationforcholecystectomy. inupto25percentofpatients.Itis The porcelaingallbladderisassociatedwithmalignancy canbehigh. Themortality multiple organfailure(MOF). trauma,burnsand inpatientsrecoveringfrommajorsurgery, isseenparticularly cholecystitis Acuteacalculous spectrum antibioticisadvisedduringthemanagementofacutecholecystitis. formorbidobesity. anaemiaandthoseduetoundergosurgery A broad- congenital haemolytic shouldbeconsideredindiabeticpatients, thosewith however, Prophylactic cholecystectomy, patientswithasymptomaticgallstones. toobserve Most authorswouldsuggestthatitissafe than gallstones. other colicis5percent/year. lawisusuallyassociatedwithapathology with biliary Courvoisier’s are 1–2and0.2percent/year,respectively. inapatient acutecholecystitis Theriskofdeveloping presence offoreignbodiesinthebileduct. gall bladderbutareformedinthebileductandrelatedtostasis,infected pigmentstonesarerareinthe palmitate andcalciumstearate,aswellcholesterol.Brown pigmentstonescontaincalciumbilirubinate, andsicklecelldisease.Brown spherocytosis are blackandthisincidenceriseswithage.Blackstonesaccompanyhaemolysis,usuallyhereditary 20–30percentofstones polymer mixedwithcalciumphosphateandbicarbonate.Overall Theblackstonesarelargelycomposedofaninsolublebilirubinpigment types: blackandbrown. cent arepigmentstones.Pigmentstonescontainlessthan30percholesterol.Theretwo per 80 ofAsia percentofthestonesarecholesterolormixed,whileinsomeparts 80 In theUK, and alump. diagnosed beforetheageof10 abdominalpain years.Usualpresentationsarejaundice,fever, percentofcasesare system.Sixty biliary congenital dilatationsoftheintra-orextrahepatic antibiotics forinfections,removalofstonesandoccasionallylobectomy. are Choledochalcysts stasis,stonesandcholangiocarcinoma.Treatmentconditions includebiliary usuallyinvolves butisthoughttobehereditary. isunknown Associated system.Theaetiology biliary extrahepatic dilatationsoftheintrahepaticductswithanormal characterised bymultipleirregularsaccular diseaseisararecondition procedure)isthepreferredoperationinmostcases.Caroli’s (Kasai ortherightandlefthepaticducts.Portoenterostomy theCHD This caninvolvetheCBD, ducts areprogressivelydestroyedbyinflammationand,ifuntreated,liverfailureensues3years. The risks of having biliary colic and acute cholecystitis inapersonwithasymptomaticgallstones colicandacutecholecystitis The risksofhavingbiliary y stectom y ➜ ➜ ➜ ➜ ➜ ➜ 1F,2H,3A,4B,5I,6D,7C,8K,9J,10G, 11E 1B,2D,3A,4C 1F,2E,3H,4A,5G,6D,7C,8B matchingquestions Answers: Extended 11. A,B,C 10. A,C,D 9.A,C,D,E

3. Managementofbiliar 2. T 1. Clinicalconditionscausedb Gall bladdercarcinoma Bile ductcarcinoma Obstructive jaundice Charcot’s triad consists of intermittent jaundice, abdominal pain and fever with chills. PSC isan withchills.PSC Charcot’s triadconsistsofintermittentjaundice,abdominalpainandfever survival of less than 6 months and a 5-year survival of5percent. oflessthan6monthsanda5-yearsurvival survival poorprognosiswithamedian bladdercarcinomahasavery A palpablemassisalatesign.Gall is18cent aresuitableforsurgicalresectionandthemediansurvival months. only10–15The otherassociationsincludehepatolithiasisandliverflukeinfestations.Overall, per years. 60 ducts.Itismorecommoninmalesandusuallyoccursbetweentheagesof30 extrahepatic treethataffectsboththeintra-and conditionofthebiliary idiopathic fibrosinginflammatory y pes ofbiliar y stones y problems y gallstones 467

63: THE GALL BLADDER AND BILE DUCTS 64 The pancreas

Multiple choice questions ➜ Anatomy B Pancreatic injury is often accompanied 1. Which of the following statements are by damage to the liver, spleen and true? duodenum. A The pancreas weighs 200 g. C The serum amylase is raised in most B The uncinate process lies behind the cases of pancreatic injury. superior mesenteric vessels. D A CECT scan will delineate the damage. C The vast majority of pancreatic tissue is E In doubtful cases, urgent ERCP is composed of exocrine acinar tissue. helpful. D Of the endocrine cells, 75 per cent are 4. Which of the following statements is B cells, 20 per cent are A cells and the false? remainder are D cells A All patients with pancreatic trauma E The accessory pancreatic duct drains the should undergo an exploratory head and uncinate process. laparotomy. ➜ B Pancreatic duct disruption requires Investigations surgical exploration. 2. Which of the following statements are C Severe injury to the duodenum and false? the head of the pancreas requires a A Ultrasonography (US) is the initial pancreatoduodenectomy. investigation of choice in the jaundiced D After conservative management for patient. pancreatic injury, duct stricture and B When doing a computed tomography pseudocyst may occur as (CT) scan, initially an unenhanced scan complications. must be done followed by a scan after E During splenectomy, iatrogenic injury to intravenous contrast injection (CECT). the pancreatic tail can occur. C While doing a magnetic resonance cholangiopancreatography (MRCP), ➜ Acute pancreatitis intravenous secretin injection helps 5. Which of the following statements is to determine any obstruction to the false? pancreatic duct. A Acute pancreatitis accounts for 3 per cent D An increase in serum amylase is of hospital admissions in the UK for diagnostic of acute pancreatitis. abdominal pain. E Endoscopic retrograde cholangiopan- B Acute pancreatitis is classified into mild creatography (ERCP) should always be and severe. preceded by a plain radiograph. C 80 per cent of cases are mild acute pancreatitis, with a mortality rate of ➜ Pancreatic injury 1 per cent. 3. Which of the following statements are D 20 per cent are severe acute pancreatitis, true? with a mortality of 20–50 per cent. A Pancreatic injury is common following E In all cases of acute pancreatitis, there is blunt abdominal trauma. a marked rise in serum amylase.

468 ➜ Extended matchingquestions Extended causes Whichoneofthefollowing 6. 9. Which of the following signshavebeen Whichofthefollowing 9. arenot Whichofthefollowing 8. statements Whichofthefollowing 7.

C B A 1. Pancreatic patholog E D C B A F E D C B A E D C B A E D C B A Carcinoma ofthe headofthepancreas Carcinoma Chronic pancreatitis Acute pancreatitis Cullen’s sign. Grey–Turner’s sign Boas’ sign sign Courvoisier’s Trousseau’s sign tooccurinacutepancreatitis? known (AST). transaminase aspartate and Lactate dehydrogenase(LDH) Blood urea Serum calcium Serum amylase White cellcount Age score? Glasgow acute pancreatitisineitherRansonor of parameters toassesstheseverity stone. pancreatitis maybeduetoaretained patient,acute In apost-cholecystectomy acutepancreatitis. develop The post-gastrectomypatientmay acutepancreatitis. develop may Patients aftercardiothoracicsurgery a higherincidence. has duringERCP Therapeutic intervention pancreatitis is10 percent. theincidenceofacute ERCP Following postoperative acutepancreatitis? of are trueregardingtheaetiology Hyperparathyroidism. Autoimmune pancreatitis Pancreatic divisum pancreatitis Hereditary Gallstones congenital anatomical variation? congenital anatomical of acutepancreatitisisduetoa y ➜ 12. The radical curative Theradical operation 12. 10. Which of the following statementsare Whichofthefollowing 10. 11. Which of the following statementsare Which ofthefollowing 11.

Surgical treatment E D C B A E D C B A E D C B A name? goesbywhich endofCBD the lower or carcinoma pancreas, periampullary ofthehead carcinoma of pancreatoduodenectomyfor true withregardtopseudocysts? upper abdomen. can ariseafterblunttraumatothe They complication ofapseudocyst. bleedingmaybea Gastrointestinal intervention. The majorityofthemrequire neoplasms. canbeconfusedwithcystic They of onsetacutepancreatitis. occurwithinthefirstweek Pseudocysts Ombrédanne. Wertheim Whipple Billroth Millin per centofpatients. Pleural effusionisseenin10–20 conservatively. peripancreatic sepsiscanbetreated The vastmajorityofpatientswith canoccur. artery ofthesuperiormesenteric Aneurysm pancreatic necrosis. laparotomy mustbedoneinallcasesof acutepancreatitis,a In severe necrosis. scantodetectpancreatic require aCECT acutepancreatitis Patients withsevere acute pancreatitis? in true withregardtocomplications 469

64: THE PANCREAS ABDOMINAL ➜ ➜ 2. D 1. B,C,D, E Answers: Multiplechoicequestions Chooseandmatchthecorrectdiagnosiswitheachofscenariosgivenbelow: 470

Investigations Anatom 5 4 3 2 1 E D abdomen should precede an ERCP tolookforcalcification. abdomen should precede anERCP theabsence orpresenceofobstruction. A plainX-rayofthe the pancreaticduct, therebyshowing emptyingof secretininjectionwillshow magnetic resonancecholangiopancreatography (MRCP), andvenousphases delineateaccuratelyspace-occupyinglesions.Duringan injection, arterial calcification.Aftercontrast tolookforpancreaticorbiliary alwaysprecedes a CECT unenhanced CT andaretroperitonealhaematoma, canproducearaisedamylase.An ectopic pregnancy pepticulcer,mesentericvascularocclusion,ruptured as otherconditions,suchperforated A markedlyincreasedserumamylaseishighlysuggestiveofacutepancreatitis, butnotdiagnostic, consumption. Clinical examination shows no abnormality except forgeneralised tenderness. noabnormalityexcept shows consumption. Clinicalexamination with type2diabetesandisonoralmedication.Headmitstomorethanaveragealcohol most days.Hehaslostsomeweightoverthisperiod.Two monthsagohewasdiagnosed lasts foradayortwo.Thisisassociatedwithnauseaandoccasionalvomitingdiarrhoea ofthispainwhich radiating tothebackforlast6months.Hehasepisodesofexacerbation malepatientpresentswithdullachingpaininhisepigastriumandumbilicalareas A 50-year-old just palpable. anaemic, hasscratchmarksoverhisbody,isslightlyjaundicedandagallbladderthat thepatientis Onexamination weightlossandupperabdominaldiscomfort. with anorexia, A 55-year-oldmancomplainsofintermittentjaundiceassociatedwithitching.Thisis rebound andrigidity. tendernessallovertheabdomenwith aCullen’ssign,extreme reveals Abdominal examination is tachypnoeic,hastachycardiaandabloodpressureof110/60 mmHg.Sheisslightlyicteric. she timesandhasretching.Onexamination 3 hduration.Shehasnausea,vomitedafew epigastricpainradiatingtothebackandrestofabdomen withsevere an emergency presentsas A 45-year-oldwoman,whoisonthewaitinglistforalaparoscopiccholecystectomy, result ofhisfallingoffbikeandthehandlebarstickingintotummy. itwasthe bruising overtheskinofepigastriumand,whenquestionedaboutit,hesays smooth massinhisepigastriumwhichistenseanddoesnotmovewithrespiration.There helooksunwell,witha tohislossofappetite.Onabdominalexamination he putsdown with upperabdominaldistension,nausea,intermittentvomitingandsomeweightloss,which A 10-year-old boypresentswithupperabdominalpainof2weeks’duration.Thisisassociated right upperquadrant. aglobulardiscretemassinthe reveals scratch marksalloverhisbody;abdominalexamination heisdeeplyjaundicedwith and hisstoolsarepale.Hehassomeweightloss.Onexamination incolour andhasnoticedoflatethathisurineisdeepyellow upper abdominaldiscomfort A 65-year-oldmancomplainsofintenseitchingandjaundice6weeks’duration.Hehas carcinoma Periampullary ofthepancreas Pseudocyst y ➜ ➜ 10. B,D,E 9.D,E 8. C 7. B,C,D, E 6. C 5. E 4. A 3.B,C,D,E

Acute pancreatitis Pancreatic injur Pancreatic injury israreinbluntupperabdominaltraumabecauseoftheretroperitonealposition Pancreatic injury Occasio neoplasm.Endoscopic canbeconfusedwithacystic nally achronicpseudocyst usuallytakes upto4weeksormoredevelop. After acutepancreatitis, apseudocyst having trackedtherethroughthefalciform ligament. is haemorrhagearoundtheumbilicus,blood sign (describedinrupturedectopic pregnancy) is abluishdiscolorationoftheflanks asaresultofbloodtrackingalongthefascialplanes.Cullen’s haemorrhagic pancreatitis.Grey–Turner’s aneurysm) sign(describedinleakingabdominalaortic suggestiveofacute The abovesignsarenotpathognomonicalthough,ifpresent,very factors. are the otherimportant saturation oxygen mentioned inthelist,serumalbuminandarterial Serum amylaseisnotoneofthebloodresultsforassessingseverity. Besidestheothertests suchassphincterotomyorballoondilatation iscarriedout. therapeutic intervention is1–3percent.Thisincidencerisesif ERCP The incidenceofacutepancreatitisfollowing which theventralductdrainsuncinateprocess. not fuse.Insuchasituation,separatedrainageoccursintotheduodenum–pancreaticdivisum,in percentofadultsisitpatent.In10 ducts,whileinonly40 patent accessory percenttheductmay orinthepostnatalperiod,ductsfuse.Eighty-fivepercentofinfantshave in thedevelopment pancreaticduct.Late ductpersistsasaccessory ofthedorsal the ventralduct.Theproximalpart of ductdrainsintotheproximalpart ofthepancreas,mostdorsal During thedevelopment acute attack,whichhasbeentransient. may alsobebecausethebloodhasbeentakentoolateandpatientrecoveredfrom Anormalamylaselevel and thereisnotenoughpancreatictissuelefttoelaboratetheenzyme. thattheentirepancreashasbeendestroyed pancreatitis mayoccurwhenthediseaseissosevere All casesofacutepancreatitisdonotcauseariseinserumamylase.Anormalamylase tail duringsplenectomymaycauseapancreaticfistula. intheaftermathofinjury. maydevelop Unrecogniseddamagetothepancreatic A pseudocyst surgeon. hepatobiliary procedure tobecarriedoutbytheexpert pancreatoduodenectomy –a the headofpancreasandduodenumrequireanemergency injuriesto Severe inanunstablepatientneedsurgentsurgicalexploration. Penetrating injury be thoroughlyassessed.Disruptionofthemainpancreaticductisanindicationforoperation. All pancreaticinjuriesdonotneedalaparotomy. should abluntinjury Astablepatientfollowing should bedone. scanwilldelineatethedamage,failingwhichanurgentERCP damage tothepancreas.ACECT Thus theliver,duodenumandspleenarefrequentlydamaged.Araisedserumamylaseindicates of theorgan.Therefore,forpancreastobeinjured,forcetraumahassevere. Stricture ofthepancreaticductmayoccurlater,resultinginrecurrentacutepancreatitis. y 471

64: THE PANCREAS ABDOMINAL 11. A,C,D,E ➜ ➜ 3A 2D 1C matchingquestions Answers: Extended 12. C 472

1. Pancreatic patholog Surgical treatment andthrombosisofthevein. oftheartery (sometimesreferredtoasapseudoaneurysm) aneurysm the pancreasmakesthesevesselsvulnerabletocompressionandinflammation,resultinginan mesenteric vesselsbehindtheneckandbetweeninferiorborderuncinateprocessof encountered. –achallengingprocedurewhichisnotoften pancreatic necrosectomyshouldbeundertaken regular flushingandrepeatedreplacement.Ifthesepsisworsensdespitevigorousmeasures,a ofthewidestpossibletubedrains.Thefluidcanbeviscousanddrainmayrequire insertion Ifthefluidispurulentandobviouslyinfected,patienttreatedbyantibiotics cytology. that thenecrosisisinfected.Confirmationofinfectioncarriedoutbyfine-needleaspiration reduced enhancementandperipancreaticfluidcollectionwithpocketsofgaswithin,itmeans areasof shows otherdaytolookforpancreaticnecrosis.IftheCECT is carriedoutatleastevery arethere,aCECT and,whilethey acutepancreatitisaretreatedintheITU Patients withsevere rupturesintothestomachorduodenum. pseudocyst bleedingcanoccurifthe a communicationwiththemainpancreaticduct.Gastrointestinal helpsinarrivingatadiagnosis. andcytology, amylaselevels (CEA), antigen fluid,whichissentforcarcinoembryonic aspirationofthecyst ultrasound (EUS)-guided are carried out with laparoscopic cholecystectomy at the same admission a few dayslater. admissionafew at thesame are carriedoutwith laparoscopiccholecystectomy andendoscopicpapillotomy anERCP Ifthereisastoneinthe CBD, in thecommonbile duct(CBD). isdonetolookforastone repeated oranmagneticresonance cholangiopancreatography (MRCP) tractis acutepancreatitis andmanagedaccordingly.stratified asmildorsevere ofthebiliary AnUS scan.Thepatientisthen the diagnosisofacutepancreatitisshould thenbeconfirmedbyaCECT to wellover1000serum amylase.Thiswouldbeelevated Iftheserumamylaseisnotelevated, IU. analgesia andintravenousfluids, bloodinvestigationsneedtobecarriedout,inparticular with origin.Sheneedstoberesuscitatedforthwith This patienthasacutepancreatitisof biliary endoscopic drainage,ortheopenoperationofcystogastrostomy. bya decisionastothebestmethodoftreatinghim,whetherbypercutaneousor is tobefollowed scanforconfirmation.This and CT HeneedsanUS pancreas whichlaterresultedinapseudocyst. enoughforhimtoseekhelp,there was acontused symptoms atthattimewerenotsevere producedbluntupperabdominaltraumawithtransientacutepancreatitis. Althoughhis injury ofthepancreas.About4weeks agohisbicycle apseudocyst This youngboyhasdeveloped hisintense pruritus. toalleviate ofastentintheCBD are, heshouldbetreatedbyinsertion ofsecondariesintheabdomen,asthere usually Ifthereisevidence endoftheCBD. the lower scantoseethesolidmassinheadofpancreasobstructing byaCT followed needs anUS classical featuresofacarcinomatheheadpancreas.Besidesallusualbloodtests,he This patienthaspainlessobstructivejaundicewithadistendedgallbladderandweightloss– Spontaneous resolution of a pseudocyst occursinmostcasesbecausethemajorityhave Spontaneous resolutionofapseudocyst A superior mesenteric artery aneurysm canoccur. aneurysm Theanatomicalpositionofthesuperior A superiormesentericartery y 5B 4E malignant lesion in the region of the lower end of the CBD. As thejaundiceisintermittent, As endoftheCBD. malignant lesionintheregionoflower bladder thatisminimallypalpable.Hisanaemiaandweightlossshouldpointtoanunderlying This patienthasintermittentobstructivejaundice(wheretheicteruswaxesandwanes)withagall insufficiency andthepainclinicforanalgesia. insufficiency the anatomyofpancreaticduct.Heshouldbemanagedbyphysiciansforhis tovisualise andERCP andCECT including estimationof24-hoursfaecalfat.ConfirmationisbyUS and diabetes).Heshouldhavealltheusualhaematologicalbiochemicalinvestigations andendocrinedysfunction(diarrhoea This patienthasalcoholicchronicpancreatitiswithexocrine pancreatoduodenectomy. secondaries. Iftherearenosecondaries,thepatientshouldthenbeconsideredforradical tolookforsmallperitonealorliver staging procedureoflaparoscopyandlaparoscopicUS undergothefinal nodistantspread,thepatientshouldnext of thelesion.Ifstagingshows andbiopsy EUS CECT, ofthejaundice.HeneedsanUS, of thecancer,resultinginalleviation itsbloodsupply,thereisnecrosis patient’s jaundicegetsdeeper;asthecarcinomaoutgrows the carcinoma.Inthiscondition,asthetumourgrows, diagnosis isobviouslyaperiampullary 473

64: THE PANCREAS 65 The small and large intestines

Multiple choice questions ➜ Anatomy D Those with perforation have a 10 times 1. Which of the following statements higher mortality than those with an are false? inflammatory mass. A The small bowel is approximately E Sepsis is the principal cause of morbidity. 10 metres long. 4. Which of the following is not a B The colon is approximately 3 metres complication of diverticular disease of long. the colon? C The most fixed part of the small bowel is A Paracolic abscess the duodenum. B Fistulae D The jejunum is wider, thicker and more C Lower gastrointestinal haemorrhage vascular than the ileum. D Carcinoma E Peyer’s patches are contained in the E Stricture. ileum. 5. Which of the following are not true in ➜ Diverticula of small intestine complicated diverticular disease? 2. Which of the following statements A Urinary symptoms may be the are true? predominant presentation at times. A Duodenal diverticulum may result from a B Profuse colonic haemorrhage may occur long-standing duodenal ulcer. in 17 per cent. B Jejunal diverticula may give rise to C Fistulae occur in 5 per cent of cases. malabsorption problems. D The commonest fistula is coloenteric. C A Meckel’s diverticulum can cause severe E In acute diverticulitis, CT scan is the ‘gold lower gastrointestinal haemorrhage. standard’ for imaging. D A suspected Meckel’s diverticulum is best 6. Which of the following is not a cause imaged by a barium meal and follow of vesicocolic fistula? through. A Carcinoma of rectosigmoid E Pain originating in a Meckel’s B Radiation enteritis diverticulum is located around the C Crohn’s colitis umbilicus D Diverticular disease E Amoebic colitis. ➜ Diverticular disease of colon 3. Which of the following statements 7. In the surgical treatment of diverticular are false? disease, which of the following A In the Western world, 60 per cent of statements are true? the population over the age of 60 have A Colonoscopy must be carried out in all diverticular disease. elective cases. B A low-fibre diet causes the disease. B Barium enema is essential prior to C These diverticula consist of mucosa, elective operation. muscle and serosa. C Primary resection and end-to-end anastomosis mustt be carried out in all cases.

474 ➜ 8. Which of the following statementsare Whichofthefollowing 8. 12. Which of the following criteriadonot Whichofthefollowing 12. Whatiscolonoscopyroutinelyused 11. isnota Whichofthefollowing 10. isnota Whichofthefollowing 9.

Ulcerative colitis E D C B A E D C B A E D C B A E D C B A E D E D C B A true inulcerative colitis(UC)? operation canusuallybedone. In vesicocolicfistula,aone-stage diverticulitis. of choiceinperforated operationistheprocedure Hartmann’s Hypoalbuminaemia <30g/L. Tachypnoea >20/min Tachycardia >90/min ofover37.5°CPyrexia More thanfourmotionsaday severediseaseinUC? indicate To inacutecases. assessseverity ofcancer development To forthe outsurveillance carry To monitortheresponsetotreatment To distinguishitfromCrohn’sdisease To ofthedisease assesstheextent for inUC? Backwash ileitis. Cobblestone appearance space Increase inthepresacral contractedcolon Narrow Loss ofhaustrations barium enemafindinginUC? Perforation. spondylitis Ankylosing Internal fistulae sclerosingcholangitis Primary Carcinoma ofUC? complication with fulminatingcolitisin5–10 percent. Patients maypresentasanemergency megacolon. The transversecolonisaffectedintoxic feature. Granulomas areatypicalmicroscopic layers ofthelargebowel. It isadiffusediseaseaffectingallthe proximally. intherectumandspreads starts In 95percentofcases,thedisease 3 WhichUCpatientshaveanenhanced 13. 16. What are the indications for surgery forsurgery Whataretheindications 16. Inthemanagementof asevereattack 15. drugsarenot Whichofthefollowing 14. 17. Which of the following statements Whichofthefollowing 17. A E D C B A E D C B A E D C B A B A E D C B risk ofdevelopingcancer? used in the medical treatmentofUC? used inthemedical the familywithdisease. Those whohaveanothermemberof Those whohavetheentirecoloninvolved. attack. first severe Those whohavehadavery over 10 years. Those whohavehadthediseasefor childhood. Those whohavehadthediseasesince rate. complication associated withthe lowest Proctocolectomy andileostomyare should betheprocedureofchoice. abdominal colectomyandileostomy situation,total In theemergency inUCisfalse? regarding surgery disease. Extraintestinal requiring largedoses. Chronic steroid-dependentdisease Non-compliance ofmedicaltreatment. biopsy. dysplasticchangeorcanceron Severe treatment. responding tovigorousmedical fulminatingdiseasenot Severe in UC? Aisgiven. orcyclosporin Azathioprine isgiven. Intravenous hydrocortisone instituted. Parenteral high-caloriealimentationis assess transversecolondilatation. Daily plainabdominalX-rayistakento after resuscitation. patientneedsaproctocolectomy Every true? isnot of UC,whichthefollowing Isoniazid. Azathioprine Predsol enema compounds 5-ASA Prednisolone 475

65: THE SMALL AND LARGE INTESTINES ABDOMINAL ➜ 22. Which of the following is not a clinical isnotaclinical Which ofthefollowing 22. aretrueof Whichofthefollowing 21. cause can Which ofthefollowing 20. pathological Whichofthefollowing 19. 18. Which of the following statementsis Whichofthefollowing 18. 476

Crohn’s disease A E D C B A E D C B A E D C B A E D C B A E D C B A perianalabscessorafissuremay be can occur. astricturedarea,malignancy In most commoninperianaldisease. Non-caseating giant-cellgranulomasare percentwillhaveananallesion. 50–70 CD? large-bowel All oftheabove. Toxic megacolon Intestinal obstruction Perforation Mimicking acuteappendicitis acute presentationofCD? Cobblestone mucosa. Pseudopolypi wall of thebowel Chronic inflammationinvolvesalllayers Serpiginous andaphthousulcers Internal fistulae features isnotfoundinCD? isremoved. diseased smallorlargebowel A patientcanbecuredofCDoncethe relative withthedisease. One in10 patientshaveafirst-degree percentofpatients. only 60 Non-caseating granulomasarefoundin It affectstheentirethicknessofbowel. cases. percentof The ileumisaffectedin60 abdominal pouchisnotoftendone. Ileostomy withacontinentintra- the mostfavouredprocedure. Colectomy withileorectalanastomosisis all patients. ileoanal pouchshouldbeconsideredin Restorative proctocolectomywithan Intermittent abdominal pain Bloodstained diarrhoea presentation ofCD? isusuallyindicated. Surgery the firstpresentingfeature. not trueofCrohn’sdisease(CD)? 25. Which of the following operations is Which ofthefollowing 25. drugsareused Which ofthefollowing 24. statements Which ofthefollowing 23. 26. Which of the following statements Which ofthefollowing 26. E D C B A F E D C B A E D C B A E D C E D C B A CT scanisusedforsuspectedintra- CT fistulae. isthe‘goldstandard’forperianal MRI disease. be doneforlarge-bowel Barium enemaandcolonoscopyshould disease. choice insmall-bowel enemaistheimagingof Small-bowel about imaginginCDaretrue? infections. tract Pneumaturia andurinary Typical riseoftemperature evening Mass intherightiliacfossa postoperatively. to operationandmanaged intheITU must bevigorouslyresuscitatedprior presentation,patients In emergency thaninCD. inUC surgery There ismorechanceofacureafter surgery. optimum medicaltreatmentpriorto Patients mustbegivenagoodtrialof radical aspossible. whenindicated,mustbeas Surgery, physician andsurgeon. Patients mustbemanagedjointlybythe (UC andCD)isnottrue? disease bowel about inflammatory ileoanal pouch. Restorative proctocolectomywith Colectomy andileorectalanastomosis Proctocolectomy andileostomy Strictureplasty Segmental resections not doneinCD? All oftheabove. Metronidazole Infliximab Azathioprine compounds 5-ASA Steroids in thetreatmentofCD? All oftheabove. fistulae. abdominal abscessandinternal ➜ ➜ 29. Which of the following aretruein Which ofthefollowing 29. Infamilialadenomatouspolyposis 27. 28. In FAP, whichofthesestatements 28. 30. Which of the following statementsare Which ofthefollowing 30.

Carcinoma ofthecolon intestine Tumours ofthelarge E D C B A E D C B A F E D C B A A F cancer (HNPCC)? cancer non-polyposis colorectal hereditary statements isfalse? (FAP), whichofthefollowing It isamuchmoremalignanttypeof It canbediagnosedbygenetictesting. cancer. endometrial per centriskofdeveloping havea30–50 Females withHNPCC The meanageofdiagnosisis44years. percent. cancer is80 colorectal The lifetimeriskofdeveloping may resultinrectalcancerlater. Colectomy withileorectalanastomosis other thanthecolon. else anywhere Polyps donotdevelop by theageof30years,FAP isunlikely. iftherearenopolyps On surveillance, after theonsetofdisease. canceroccurs15–20Large-bowel years polyposis coli(APC)gene. mutations intheadenomatous new 20 percentofFAP ariseasaresultof are true? fromtheageof12surveillance years. be offeredannualcolonoscopic At-risk familymembersshould genetic testingintheirearlyteens. Family membersshouldbeoffered age of15 years. The conditionusuallymanifestsbythe The majoritywillbecomemalignant. with apositivefamilyhistory. percentoccurinpatients More than80 condition. It isinheritedasaMendeliandominant rectosigmoid region. percent occurinthe Almost 60 true withregardtocolorectalcancer? The majorityoccurintheproximalcolon. cancer. 33. Which of the following isfalsewith Which ofthefollowing 33. ➜ 32. In large bowel cancer, whichofthe cancer, In largebowel 32. statements Whichofthefollowing 31.

fistula Enterocutaneous or faecal E D C B A E D C B A E D C B E D C B A fistula? regard toanenterocutaneous rectal bleedingandobstructive Left coloniccancerspresentwith features ofanaemia. Right coloniccancerspresentwith are false? All oftheabove. with intestinalobstruction. 20 percentpresentasanemergency type. cauliflower The leastmalignantformisthe an increasedrisk. fibreisassociatedwith Reduced dietary accompany thecondition. Hypoproteinaemia andsepsisoften the problem. alwaysneedanoperationtocure They scans. with bariumstudiesandCT shouldbethoroughlyassessed They where thereis>1L/day. A high-outputfistulaisdefinedasone The commonestcauseispostoperative. can beresected. 95percentofcoloniccarcinomas Over procedure. should beconsideredasastaged Hepatic resectionformetastases found, biopsyshouldbedone. If, atoperation,hepaticmetastasesare liver metastases. Resection isnotdoneifthepatienthas andspiralCT.colonoscopy, US and stagingshouldbedonewith Thorough preoperativeassessment statementsaretrue? following Synchronous cancersoccurin5 routinely done. shouldbe Intravenous urography(IVU) is 10 percent. the failureratetovisualisecaecum colonoscopist, Even foranexperienced symptoms. per cent. 477

65: THE SMALL AND LARGE INTESTINES ABDOMINAL ➜ ➜ ➜ Choose andmatchthecorrectdiagnoseswitheachofscenariosgivenbelow: Extended matchingquestions Extended With regardtoastoma,whichofthe 34. 478

G F E D C B A 2. Imagingofsmalland largeintestinepathology 5 4 3 2 1 E D C B A 1. Clinicalpresentations ofsmallandlargeintestinepathology Stoma B A requiring an extra notchinhisbelt. requiring anextra ofweightloss,asseenbyhis loosetrousers someevidence there isnothingtofindexcept Onstrainingatstools,hepassesbloodfromhisanus. Onexamination insufficient evacuation. action,heisleftwith afeelingof abowel action.Following tohaveabowel of laxatives A 70-year-old mancomplainsofconstipationfor4months. He hastotakeincreasingamounts hehasanacuteperianalabscessandamassintherightiliac fossa. examination On abdominalpain.Hehasafeelingofheavinessinhisright iliacfossa. suffers fromcolicky frequent loosemotionsoccasionallymixedwithbloodonandoffformany months.Healso withanacuteperianal abscess.Hehashad A 55-year-oldmalehaspresentedasanemergency of growth heavy shows of hisabdomenthereisnothingtofind.Urineexamination admits topassingairbubblesinhisurineandrecentlyhasbeenconstipated.Onexamination material.Onquestioninghe foulsmellingandsometimescontainsbrown urine whichisvery whilepassing ofmicturition,discomfort malepatientcomplainsoffrequency A 60-year-old whichbleedseasily. hyperaemicmucosa, blood andmucus,sigmoidoscopyshows shows somevaguetenderness.Rectalexamination reveals in weight.Abdominalexamination had thesesymptomsforthepast4monthsduringwhichtimehehaslostabout10 pounds abdomen.Hehas four suchmotionsaday,whichareassociatedwithdullachinginhislower A 45-year-oldmalepatientcomplainsofdiarrhoeawithmucusandblood.Hehasthreeto amobilemassintherightiliacfossa. reveals haemoglobin is8.5g/L.Abdominalexamination ofbreathclimbingoneflightstairsathome.Her the last3months.Shehasbeenshort womancomplainsoffeelingtiredwhiledoingherusualhouseholdworkover A 60-year-old Intestinal obstruction Ulcerative colitis Intussusception Crohn’s disease oftheascendingcolon Carcinoma stricture Diverticular Foreign bodyin rectum ofthedescendingcolon Carcinoma ofthecaecum Carcinoma Ulcerative colitis Crohn’s disease disease Diverticular A stomaisalwaysspouted. by atransverseloopcolostomy. de-functioned byaloopileostomythan An anteriorresectionisbetter following statementsaretrue? following E D C gutter to prevent internalherniation. gutter toprevent essential toclosethelateralparacolic When fashioningacolostomy,itis to poortechnique. Most colostomycomplicationsaredue often encounteredinileostomypatients. problemsaremore Fluid andelectrolyte E. coli . Choose andmatchthecorrectdiagnoseswitheachofFigs65.1–65.7 below: F F i i gure 65 gure 65 . . 1b 1a 479

65: THE SMALL AND LARGE INTESTINES ABDOMINAL 480 F F i i gure 65 gure 65 . . 2 3a F F i i gure 65 gure 65 . . 3b 4 481

65: THE SMALL AND LARGE INTESTINES ABDOMINAL 482 F (a) i gure 65 . 6 F i gure 65 . 5 (b) ➜ ➜ ➜ 3. C 2.A,B,C,E 1. A,B Answers: Multiple choicequestions

Diverticular diseaseofcolon Diverticula ofsmallintestine Anatomy weakness. protrude throughthecircularmuscle wherethebloodvesselsentercolonicwall,apointof They only,coveredbyserosa. consistofmucosa areacquired.Therefore they Colonic diverticula midgut, painoriginatingfromitwouldbefeltinitiallyaroundtheumbilicus. ofthe ispart thediverticulum an accuratemethodofidentifyingsuchasourcebleeding.As is redbloodcellsarelabelledwithtechnetium-99 enema.Scanningafterapatient’sown bowel Theidealimagingmethod would beasmall- peptic ulcerarisingfromectopicgastricmucosa. malabsorption problems:anaemia,steatorrhoea,hypoproteinaemiaand vitaminB12 deficiency. maybeasymptomatic, cancause althoughthey causing duodenalstenosis.Jejunaldiverticula, isalwaystheoutcomeofalong-standing duodenalulceration An acquiredduodenaldiverticulum bluntabdominaltrauma. fromsevere makes itvulnerabletoinjury Thisanatomicalfact ofthesmallbowel. andthereforethemostfixedpart ofmesentery is devoid isapproximately7metreslongandthecolon1.5The smallbowel metreslong.Theduodenum A Meckel’s diverticulum can be the source of a major lower gastrointestinal bleedfroma canbethesourceofamajorlower A Meckel’sdiverticulum F i gure 65 . 7 483

65: THE SMALL AND LARGE INTESTINES ABDOMINAL ➜ 16. A,B,D,E 17. D 15. 13. A,B,C,D 14. A E 12. D 11. A,B,C,D 10. 9. D C 8.A,D,E 7. A,B,D,E 6. E 5. D 4. D 484

Ulcerative colitis inflammatory bowel disease that affects the mucosa and superficial submucosa only; in severe only;insevere submucosa andsuperficial diseasethataffectsthemucosa bowel inflammatory In 95percentofcases,thediseaseaffectsrectumandspreadsproximally. Itisadiffuse done. a vesicocolicfistula,afterthoroughinvestigation,one-stageresectionandanastomosiscanbe alternative.In operationisthesafest afterintraoperativecolonicirrigation.Hartmann’s emergency onlyinselectedcasescanitbeattemptedan diverticulitis; the patientwithperforated full colonoscopy. resectionandanastomosiscanbedoneintheelective patientbutnotin Primary itmaynotalwaysbepossibletodoa ofthebowel, coincidental carcinoma.Becauseofnarrowing a In theelectivesituation,acolonoscopyandbariumenemamustbedonetoexclude guidancecanalsobeusedtodrainanabscesspercutaneously.disease. CT wallthickening,abscessformationandanyothercoincidental of choiceasitdemonstratesbowel scanistheimaging aCT management.Inacutediverticulitis and usuallysettleswithconservative fistula, whichisthecommonesttypeoffistulathatoccurs.Profusecolonichaemorrhagemayoccur disease,itindicatesavesicocolic symptomsindiverticular When apatientpresentswithurinary in12 disease,itcoexists Whilst carcinomaisnotacomplicationofdiverticular percent. restorative proctocolectomy. dysfunction,ithaslargelybeenreplacedby avoids astomaandhasminimalrisk ofsexual vast majority. formalignancy. Although theoperation Thepatientneeds regularrectalsurveillance Colectomy withileorectalanastomosis isarareprocedurebecausetherectumdiseasedin canoccur.is dangerousbecausesilentperforation Prolonged high-dosesteroidtreatment shouldbeundertaken. not occurwithin3–5days,surgery Vigorous medicaltreatmentisinstitutedaccordingtotheaboveregimen andifimprovementdoes patientdoesnotrequireaproctocolectomy. butevery isamedicalemergency caseofUC A severe ofthedisease. AplainabdominalX-raycanhelptoassesstheseverity fear ofcausingperforation. Itshouldnotbeusedintheacutecasefor Colonoscopy hasavitalroleinthemanagementofUC. 10 withacutefulminantUC. percentofpatientsmaypresentasanemergency typical microscopicfeature.Inacutetoxicmegacolon,thetransversecolonisaffected.Upto abscessesarethe crypt disease thedeeperlayersareinvolved.GranulomasdonotoccurinUC; ➜ ➜ ➜ 29. A,B,C,D,F 30. E 28. A,B, C,E 27. C 26. B 25. E 24. 23. F E 21. A,B,C,D 22. D 20. E 19. D 18. E

Carcinoma ofthecolon Tumours ofthelargeintestine Crohn’s disease Crohn’s disease is not a curable disease. It can recur in other parts ofthegastrointestinaltract Crohn’s diseaseisnotacurabledisease.Itcanrecurinotherparts 30 years. intherectalstump is10have gastroscopiescarriedout.Theriskofcancerdeveloping percentover unlikely. inthestomachand duodenum; hence,postoperatively,patientsshould Polyps dodevelop bytheage of30years,FAP polypshavenotdeveloped If, onannualcolonoscopicsurveillance, is APC gene.Onsetofthediseaseisaroundage15 15–20 yearsand cancerdevelops yearslater. FAPIn afifthofpatientswherethereisnofamilyhistory, mutationsinthe occurs asaresultofnew In all(100 willresult. percent)ofuntreatedpatients,carcinoma ofthelargebowel skin. pelvic dissection.Therectalstumpisbroughtoutasamucousfistulaorclosedjustbeneaththe avoidinga atotalabdominalcolectomyandileostomyisperformed, megacolon orperforation, situationssuchastoxic inemergency aspossibleinCD.InUC, mustbeasconservative Surgery thisoperationbeingcarriedout. disease. Perianal disease,whichiscommonincolonicCD,prevents incidence ofrecurrence.Moreover,theileumtobeusedforapouchmayinvolvedwith In provenCD,restorativeproctocolectomywithileoanalpouchisnotdonebecauseofthehigh tocontroldiseaseactivity inileocolicandcolonicdisease. perianal ones.Metronidazoleisknown their steroid-sparingeffect.Infliximabisamonoclonalantibodythatusefulinfistulae,particularly usefulincolonicdisease.Immunosuppressiveagentsare usedfor compounds areparticularly percentofcases.5-ASA Steroids arethemainstayoftreatmentandhelpinremission70–80 Crohn’s diseaseismainlytreatedmedically,withalltheabovedrugsusedfromtimetotime. isnotaclinicalfeatureofCD.Itsuggestiveabdominaltuberculosis. Evening pyrexia polyps. alsocalledinflammatory of normalmucosa, duetoregeneration Pseudopolypi donotoccurinCD.ItisafeatureseenadvancedchronicUC aspossibleshouldbesacrificed. andaslittlebowel conservative, HenceresectionforCDmustalwaysbe afterremovalofadiseasedsectionthebowel. even 485

65: THE SMALL AND LARGE INTESTINES ABDOMINAL ➜ ➜ ➜ 4B 3A 2C 1D matchingquestions Answers: Extended 34. A,C,D 33. D 32. A,D,E 31. D 486

1. Clinicalpresentations ofsmallandlargeintestinepathology Stoma Enterocutaneous orfaecalfistula the extent ofinvolvementtheleftureter. the extent and lefthydronephrosistoseethefunctionofrightkidney shows orCT cancers whenanUS Intravenous urographyisnotaroutineinvestigationforcolorectalcancer. Itisdoneinleftcolonic anaesthesia (EUA) andinvestigationsforthorough assessment –colonoscopyandbiopsy, barium anaesthesia (EUA) under byan examination drainageoftheabscessfollowed abscess. Heneeds anemergency withanacuteperianal whichissuggestiveofCrohn’sdisease. Hepresentsasanemergency fossa diseasehasamassintherightiliac bowel This patientwhohashadsymptoms ofinflammatory and aone-stageresectionend-to-end anastomosisdone. bladder,theholeinbladderclosed isdissected offtheurinary preparation, theaffectedbowel disease,afterthoroughbowel scanofthepelvisandacystoscopy. Indiverticular the cause,CT sigmoidoscopy andbiopsytofind disease, causingstricture.Heneedsabariumenema,flexible E. coli tractinfectionsfromavesicocolic fistula–gross symptomsofurinary This patienthassevere enema andcolonoscopywithmedicalmanagementthereafter. sigmoidoscopic findings.Abiopsywillgivethefinaldiagnosis.Heneeds assessmentbybarium This patient’ssymptomsaresuggestiveofulcerativecolitis,whichisclinically confirmedon right hemicolectomy. scanoftheabdomen.Thetreatmentis barium enemaiffullcolonoscopyisunsuccessful,andCT Sheneedsconfirmationbycolonoscopyandbiopsy,a anaemia andamassintherightiliacfossa. This womanhastheclassicalpresentationofacarcinomacaecum–symptomsfrom that itiseffective. end colostomy,mostsurgeonsstillclosethelateralparacolicgutter,butthereisnoevidence fossa An ileostomyisalwaysspoutedbutacolostomyflushwiththeskin.Whilemakingleftiliac abscess. complex is epithelialcontinuitybetweenthegutandskin,ifthereactivediseaseoranassociated spontaneously ifthereisnodistalobstruction.Afistulawillfailtoheal fistulaeheal All enterocutaneousfistulaedonotalwaysneedanoperation.Low-output cause tumourdissemination. than toremovetheoriginaltumour. bedoneasthismay shouldnever Biopsyofaliversecondary liver secondaries,theappropriatehemicolectomyiscarriedoutastherenobetterpalliation Thorough preoperativeassessmentwithconfirmationbybiopsyandstagingisessential.Even infection, pneumaturia and faecaluria. He has suffered from constipation due to diverticular infection,pneumaturiaandfaecaluria.Hehassufferedfromconstipation duetodiverticular 5E ➜ Fig.65.6,E Fig.65.5,A Fig.65.4,G Fig.65.3aandb,B Fig.65.2,F Fig.65.1a andb,D Fig.65.7,C

2. Imagingofsmallandlargeintestinepathology A radicallefthemicolectomyisthetreatment. scan. byaCT asynchronouscancer,followed toexclude lesion. Abariumenemaisthennecessary biopsy. Afullcolonoscopymaynotbepossible,ashissymptomssuggestanannularortubular action, shouldbesuspectedofhavingaleftcoloniccarcinoma.Heneedscolonoscopyand This patient,whohasincreasingconstipationandafeelingofincompleteemptyingafterbowel Heismanagedmedically. which willrequireMRI. leadtoafistula-in-ano aftermath ofthedrainageperianalabscesswouldalmostcertainly anyareasofsepsisinsidetheabdomen.The scantoexclude enemaandaCT enema, small-bowel clinical featuresofanaemiaandamass intherightupperquadrantofabdomen. – typical ofapolypoidcarcinoma.Thepatientpresentedwith proximal tothehepaticflexure anirregular fillingdefectintheterminalascendingcolonjust This isabariumenemashowing intussusception. withacrab-clawdeformity.of bariumjustdistaltothehepaticflexure Thisisanileocaecocolic completearrest This isabariumenemainchild,asobviousbytheskeletalfeatures. Itshows laparotomy, atwhichtheforeignbodywasmilkedthroughrectum outoftheanus. aforeignbody(vibrator)intherectum.Thepatient neededa This aplainX-rayshowing obstruction. distal small-bowel withacute The patientturnedouttohaveacarcinomaofthecaecumpresenting as anemergency whichisthehallmarkofilealobstruction. thereischaracterlessdistendedbowel, part In thelower therearevalvulaeconniventesfromjejunal distension. centre oftheabdomen.Inupperpart, inthe grossdistensionoftheentiresmallbowel This isaplainX-rayoftheabdomenshowing obstruction. and featuresofintermittentlarge-bowel diseasemimickingacarcinoma.Thepatientpresentedwithconstipation stricture fromdiverticular ofa stricture.Thisisaclassicalexample wasseen.Thediagnosisdiverticular malignancy sigmoidoscopyandbiopsyweredoneatwhichno is suggestiveofacarcinoma.Aflexible anirregularstrictureinthesigmoidcolon.This These twobariumenemapicturesshow ulcerative colitis. ofthecolonandbackwashileitis,featurestypicaltotallong-standing appearance, shortening completelossofhaustrationstheentirecolonwithahosepipe This isabariumenemashowing an enteroentericfistula. radiatingspicules,cobblestoneappearanceand interspersedwithdiseasedbowel, normal bowel oftheterminalileum(stringsignKantor),skiplesionswith typical Crohn’sdisease–narrowing allthefeaturesof Itshows These twopicturesareaseriesofbariummealandfollow-through. 487

65: THE SMALL AND LARGE INTESTINES 66 Intestinal obstruction

Multiple choice questions ➜ Adynamic intestinal ➜ Internal herniation obstruction 4. Which of the following are potential 1. Which of the following are causes of sites for internal herniation? adynamic intestinal obstruction? A A hole in the transverse mesocolon A Paralytic ileus B Defects in the broad ligament B Hernia C A hole in small bowel mesentery C Mesenteric vascular obstruction D Foramen of Winslow D Pseudo-obstruction E Paracaecal fossae. E Adhesions. ➜ Bolus obstruction ➜ Fluid and electrolyte 5. Which of the following are examples of changes in intestinal bolus obstruction? A Gallstones obstruction B Phytobezoar 2. Which of the following cause C Trichobezoar dehydration and electrolyte loss in D Stercoliths intestinal obstruction? E Worms. A Reduced oral intake B Defective intestinal absorption ➜ Postoperative adhesions C Vomiting 6. Which of the following are associated D Diarrhoea with increased postoperative E Sequestration in the bowel lumen. adhesions? ➜ A Ischaemic areas Strangulation in intestinal B Good surgical technique obstruction C Foreign material 3. Which of the following statements D Covering anastomosis and raw areas regarding strangulation in intestinal E Crohn’s disease. obstruction are true? A The common causes are hernial orifices, ➜ Intussusception adhesions, volvulus and closed-loop 7. Which of the following statements obstruction. regarding intussusception are true? B The arterial supply is compromised A It is most common in children with before the venous return. a peak incidence between 5 and 10 C Marked translocation and systemic months of age. exposure to anaerobic organisms occur. B About 10 per cent of infantile D The morbidity of strangulation due intussusceptions are idiopathic. to an external hernia is greater than C It causes obstruction but not intraperitoneal strangulation. strangulation. E The morbidity and mortality in strangu- D Meckel’s diverticulum can be a cause in lation are independent of patient age. older children.

488 ➜ ➜ ➜ ➜ Extended matching questions Extended statements Whichofthefollowing 10. statements Whichofthefollowing 9. statementsare Whichofthefollowing 8.

1. Causesofintestinalobstruction obstruction Imaging inintestinal E D C B A obstruction Clinical features ofintestinal E D C B A Sigmoid volvulus H G F E D C B A E Richter’s hernia. Constipation isapredominantfeatureof after theonsetofobstruction. even Some patientsmaypassflatusorfaeces obstruction. large-bowel Distension isalatefeaturein indicative ofstrangulation. painis ofsevere The development obstruction. small-bowel Vomiting occursearlyinhigh surgery. There isnoroleforemergency rateofrecurrence. with alow Flatus tubedecompressionisassociated direction. The rotationisusuallyinaclockwise attachmentofthemesocolon. narrow constipation, longpelvicmesocolonand The predisposingfactorsinclude in adults. This isthemostcommonsiteofvolvulus intestinal obstructionaretrue? featuresof regarding clinical true aboutsigmoidvolvulus? Congenital malrotation Pseudo obstruction Sigmoid volvulus Faecal impaction colonictumour Primary hernia Strangulated external Recurrence ofmalignancy Adhesions common siteinadults. The ileocolicregionisthemost obstruction aretrue? regarding imaginginintestinal ➜ 11. Which of the following statements Whichofthefollowing 11.

obstruction Treatment ofintestinal E D C B A E D C B A obstruction aretrue? regarding treatmentofintestinal from pseudo-obstruction. differentiating mechanicalobstruction A water-solubleenemaishelpfulin obstruction. diagnosis ofsmall-bowel ishelpfulinthe Barium follow-through colonic obstruction. loopscanbedilatedin Small-bowel valvulae conniventes. The ilealloopsarecharacterisedby be obtained. Erect abdominalfilmsshouldroutinely 70 percentofcases. reduced non-operativelyinmorethan Infantile intussusceptioncanbe initially non-surgical. The treatmentofpseudo-obstructionis obstruction. foradhesive duringsurgery mandatory Division ofalltheadhesionsis suspected internalstrangulation. herniaand obstructed external include Indications forearlysurgery treatment. beforeattemptingsurgical necessary replacementarealways electrolyte drainageandfluid Gastrointestinal / 489

66: INTESTINAL OBSTRUCTION ABDOMINAL ➜ Chooseandmatchthecorrectdiagnosiswitheachofscenariosgivenbelow: Choose and match the correct radiological signwitheachofthediagnosesbelow: Choose and matchthecorrectradiological 490

5 4 3 2 1 2. Radiologicalsignsinintestinalobstruction 8 7 6 5 4 3 2 1 E D C B A Gallstone ileus Gallstone Sigmoid volvulus Intussusception perforation Bowel obstruction Small-bowel conservatively. ‘coffee AbdominalX-rayreveals bean’sign. similarepisodeswhich havebeentreated the abdomenandconstipation.Shehashadprevious ofrapiddistension femalewithchronicconstipationisbroughtahistory A 68-year-old with nocut-off. uptotherectum massivelydilatedlargebowel sounds areabsent.AbdominalX-rayreveals distendedandtympanic.Bowel massive abdominaldistension.Clinicallytheabdomenisvery 2 weekspreviously. formorethanaweekandhasdeveloped Hehasnotopenedhisbowels bypassgraft(CABG) You artery coronary arecalledtoseea70-year-old manwhounderwent dextrocardia. abdominal distension.ChestX-rayreveals andassociatedwithvomiting since hewasyoung.Thisepisodehasbeenmoresevere A 10-year-old boyisbroughttothehospitalwithrecurrentepisodesofabdominalpainever thatherrectumisfullofsoftstools. reveals in hercolon.Per rectalexamination adistendedabdomenwithmultiplepalpablemasses reveals days.Clinicalexamination several Shehasbeenhavingdiarrhoeaandincontinence for she seemstohaveabdominaldiscomfort. that dementedfemalehasbeenbroughtfromhernursinghome.Hercarersays A 90-year-old irreducible lumpispalpableoverthemedialaspectofherrightgroin. obstruction.A2cmtenderand signsofsmall-bowel reveals 3 days.Abdominalexamination femalepresentswithincreasingabdominaldistensionandvomitingforthepast An 86-year-old appendix 5yearsago. admitted tothehospitalfourtimeswithsimilarsymptomsafterhislaparotomyforaperforated week. Hehasbeenmanagingtopasssmallamountofstoolsoccasionally. He hasbeen A 30-year-oldmalepresentswithon-and-offabdominalpainandvomitingovertheprevious chemotherapy. 2yearsagoandhasrecentlyreceived She hadanoperationforovarianmalignancy ofabdominalpain,distensionandvomiting. A 42-year-oldfemalepresentswitha4-dayhistory adilatedcaecumandproximalcolonuptothesplenicflexure. X-ray shows atenseandtympanicabdomen.Abdominal reveals lost astoneinweight.Clinicalexamination absolute constipation.Hehadnoticedincreasingconstipationoverthelast3monthsand ofabdominalpain,distensionand A 70-year-old maleisbroughttotheA&Ewithahistory Rigler’s sign Pneumobilia Step-ladder pattern Coffee beansign Claw sign ➜ ➜ ➜ ➜ 3.A,C 2.A,B,C, E 1. A,C,D Answers: Multiplechoicequestions Choose and match the correct intervention witheachofthediagnosesbelow: Chooseandmatchthecorrectintervention

Strangulation inintestinal obstruction Fluid andelectrol Ad 6 5 4 3 2 1 3. Managementofvarioust blood supply. supply. Thevenousreturniscompromised beforethearterial theviabilityof As tocompromised isthreatened secondary When strangulation occurs,theviabilityofbowel supplementation.Aclosemonitoring of thefluidbalanceisvital. electrolyte toresuscitatethempromptlywithIVfluidsandappropriate abnormalities. Itisthereforeimportant All patientswithintestinalobstructionwillhavesomedegreeofdehydration andelectrolyte needed forcomplications. maybe thinginmanagementiscorrectionofthepredisposing cause.Surgery The mostimportant of myopathyorneuropathy. Thisoccursinbothacute(Ogilvie’s syndrome)andchronicforms. myocardial infarction,shock,burns,drugsandvariousmetabolicconditions thathaveanelement haemorrhage) andmetabolicconditionssuchasuraemia. (afterspinaloperations,retroperitoneal (commonly potassium)abnormalities,infections,reflex neuromuscular failure.Thishasavarietyofcauses,includingabdominaloperations,electrolyte quiet. The abdomenistypicallydistended,softortenseandvery problemorasystemiccondition. bowel system.Thiscanbeaprimary of theentericnervous asaresultofparalysis obstruction isduetotheabsenceofpropulsivemovementsinbowel sounds.The abdominalpainandhyperactivebowel mechanical obstructionsuchascolicky This typeofintestinalobstructionischaracterisedbytheabsencetypicalfeatures F E D C B A Signs ofperitonitis Faecal impaction obstruction Adhesive small-bowel Obstructing leftcoloniccarcinoma Irreducible inguinalhernia Sigmoid volvulus Hernia and adhesions are examples ofdynamicintestinalobstruction. Hernia andadhesionsareexamples causes. Mesenteric ischaemiacanbeocclusiveornon-occlusiveandhave venous orarterial Pseudo-obstruction usuallyaffectsthecolon.Itisassociatedwithvarious conditions,suchas ileusischaracterisedbyafailureoftransmissionperistalticwavesasresult Paralytic Urgent exploration Colonic stenting Manual disimpaction Flatus tubeinsertion Trial management withconservative laparotomy Emergency y namic intestinalobstruction y te changesinintestinalobstruction y pes ofobstruction 491

66: INTESTINAL OBSTRUCTION ABDOMINAL ➜ ➜ ➜ ➜ 7. A,D 6.A,C,E 5.A,B,C,D,E 4.A,BC,D,E 492

Intussusception Postoperative adhesions Bolus obstruction herniation Internal failure. of bloodandfluidwillcauseperipheralcirculatory andtheloss involvementcanbeextensive loss issmall.Insomeinternalstrangulation,thebowel andtheresultantbloodfluid hernia,thesegmentinvolvedisshort In strangulatedexternal organisms withtheirassociatedtoxins. toanaerobic iscompromised,theremarkedtranslocationandsystemicexposure the bowel years, apathological pointisfoundinatleastone-third ofcasesandthelikelycausesare Meckel’s and commonviralpathogenshave all beenimplicatedinthepathogenesis.Afterageof2 Weaning,in theterminalileummaybeinitiating event. lossofpassivelyacquiredimmunity patches thathyperplasiaofPeyer’s percentofcases areidiopathic.Itisbelieved of agewhen90 immediately adjacentsegment.Itis mostcommonlyseeninchildrenbetween5and10 months becomesinvaginatedwithinan ofthebowel Intussusception occurswhenoneportion for adhesions. proceduresare commonprecursors Operationsforappendicitisandgynaecological small bowel. effective. Postoperative adhesiongivingrisetointestinal obstructionusuallyinvolvesthelower tobeparticularly Currentlynosingleagenthasbeen shown andpolyvinylpropylene(PVP). dextran silicone, adhesionformation,includinghyaluronidase,hydrocortisone, peritoneal cavitytoprevent gauze andcoveringanastomosisrawareas.Numeroussubstances havebeeninstilledinthe toremoveclotsandthelike,minimising contactwith washing oftheperitonealcavitywithsaline anastomoses, rawareas,Crohn’sdisease,talc,starch,gauze,silkandanyothercauseoftrauma. conditions,radiationenteritis,sitesof ischaemic areas,foreignmaterial,infection,inflammatory Thecommoncauseofintra-abdominaladhesionsare adhesions betweenapposedsurfaces. world. Anysourceofperitonealirritationresultsinlocalfibrinproduction,whichproduces obstructionintheWesternAdhesions andbandsarethemostcommoncauseofsmall-bowel inchildren. obstruction,particularly small-bowel low orilealstricture. inassociationwithajejunaldiverticulum bowel undigested hairandfruit/vegetablefibre,respectively. Stercolithsareusuallyfoundinthesmall through thegallbladderintoduodenum.Trichobezoars arefirmmassesof andphytobezoars toerosionofalargegallstone obstructiontendstooccurintheelderlysecondary Gallstone liabletocauseobstruction. Fruit andvegetablesareparticularly directly intothesmallbowel. canpass articles ortotalgastrectomywhenunchewed Bolus obstructionmayoccurafterpartial andintersigmoidfossae. retroperitoneal fossae Internal herniationcanalsobecausedbycongenitaloracquireddiaphragmatichernias,duodenal operations. orcongenitalmesentericdefect,defectscreatedafterprevious retroperitoneal fossae becomesentrappedinoneofthe ofthesmallbowel Internal herniationoccurswhenaportion The morbidity and mortality associated with strangulation are dependent on age and extent. associatedwithstrangulationaredependentonageandextent. The morbidityandmortality Several factors have been shown toreduceadhesions,includinggoodsurgicaltechnique, factorshavebeenshown Several Ascaris lumbricoides can cause ➜ ➜ ➜ ➜ 11. A,B,D,E 10. C,E 9.A,B,C,D 8.A,B

Treatment ofintestinalobstruction Imaging inintestinalobstruction Clinical features ofintestinalobstruction Sigmoid volvulus not reduced. common inadults.Theintussusceptioncancauseobstruction,whichleadtostrangulationif invariably associatedwithaleadpointandtumourhastoberuledout. polyp,duplicationandHenoch–Schönleinpurpura.Inadults,anintussusceptionis diverticulum, of obstruction,thecauseobstruction andtheviabilityofgut. angulationispresent. and theremainingadhesionsleftin situunlesssevere Thisshouldbedivided Although multipleadhesionsareusually found,onlyonemaybecausative. another alternativeinthissituation. scanis obstruction. CT worsen matters.Awater-solubleenemaisusefulinconfirminglarge-bowel is contraindicatedifobstructionsuspected,asthiscanprecipitatecomplete obstructionand obstructionwhenassociatedwithanincompetentileocaecalvalve.A barium follow-through bowel loopscanbedilatedinlarge- Small-bowel intherightiliacfossa. byaroundedgasshadow shown The distalileumdoesnothavetheselinesandisdescribedasfeatureless. Adistendedcaecumis andareregularlyplaced,giving a‘concertina’completely passacrossthewidthofbowel effect. on asupineabdominalfilm.Thejejunumischaracterisedbyitsvalvulae conniventes,which Erect abdominalfilmsarenolongerroutinelyobtainedandtheradiologicaldiagnosisisbased diarrhoea). obstruction duetofaecalimpaction(theremaybeoverflow obstruction, mesentericvascularobstructionassociatedwithpelvicabscessandpartial that constipationispresentinintestinalobstructiondoesnotapplyRichter’shernia,gallstone aftertheonsetofsymptoms.Therule even obstructiontocontinueopeningtheirbowels bowel in thepresenceofmesentericvascularocclusion.Itisnotuncommonforpatientswithsmall- distal thelesion.Distensionisdelayedincolonicobstructionandmaybeminimalorabsent The degreeofdistensionisdependentonthesiteobstructionandgreatermore onetothepossibilityofstrangulation. painshouldalert ofconstantandsevere The development abdominalpain. appearance ofnauseaandvomiting.Intestinalobstructiontypicallycausescolicky betweentheonsetofsymptomsand The moredistaltheobstruction,longerinterval intervention. orstrangulation.Failure ofnon-surgicalmeasuresmayalsoneedsurgical such asperforation settingtodealwithcomplications intheemergency maybecomenecessary of recurrence.Surgery reliefbuthasahighincidence Flatus tubeorcolonoscopicdecompressionmayprovidetemporary Sigmoid volvulusisthemostcommonvarietyandtwistoccursinananticlockwisedirection. tovascularocclusion. it formsaclosedloopofobstructionwithresultantischaemiasecondary aboutitsmesentery. ofbowel Whencomplete A volvulusisatwistingoraxialrotationofportion In mostchildren,theintussusceptionisileocolic,whereascolocolicmore During operative intervention for bowel obstruction,threethingsneedtobeassessed–thesite for bowel During operativeintervention 493

66: INTESTINAL OBSTRUCTION ABDOMINAL ➜ ➜ ➜ 1C,2F,3E,4B,5D,6A 5D 4B 3A 2E 1C 1D,2B,3A,4C,5E,6H,7G,8F matchingquestions Answers: Extended 494

3. Managementofvarioust 2. Radiologicalsignsinintestinalobstruction 1. Causesofintestinalobstruction into thebowel. fistulawhichhelpedthegallstonestopasson The pneumobiliaisduetothecholecystoduodenal as‘bentinnertubesign’. This wasalsoknown the clawshape. This characteristicappearanceoccursassomeofthecontrastentersoutertube,givingriseto wall. and outermarginsofthebowel ofairintotheperitonealcavityoutlinesbothinner withresultantleakage perforation Bowel central abdomen. loopsoneuponanotherinthe This appearanceoccursasaresultofstackingsmall-bowel y pes ofobstruction 67 The vermiform appendix

Multiple choice questions ➜ Anatomy B Pointing sign 1. Which of the following statements is C Obturator sign false? D Psoas sign A The appendicular artery arises from the E Murphy’s sign. right colic artery. 5. Which of the following statements are B The commonest position of the appendix true in acute appendicitis? is retrocaecal. A Perforation is more common in acute C The position of the base of the appendix obstructive appendicitis than in acute is constant. catarrhal appendicitis. D The submucosa is rich in lymphoid B Acute appendicitis in pregnancy will aggregates. result in fetal loss in 3–5 per cent. E Argentaffin cells are found in the base of C An Alvarado score of 7 or more is the crypts. suggestive of the diagnosis. ➜ D Routine use of contrast-enhanced CT Clinical aspects scan will reduce the incidence of removal 2. Which of the following statements are of a normal appendix. true? E Cancer of the caecum may masquerade A The peak incidence of acute appendicitis as acute appendicitis. is in the teens and early 20s. B The incidence of acute appendicitis is 6. In children, which of the following is lowest in those who have a high intake not part of the of of dietary fibre. acute appendicitis? C Obstruction of the appendix lumen by a A Gastroenteritis caecal carcinoma may give rise to acute B Mesenteric adenitis appendicitis. C Meckel’s diverticulitis D Aerobic and anaerobic organisms are D Intussusception responsible for acute appendicitis. E Urinary tract infection E A mucocele of the appendix is a clinical F Lobar pneumonia. variation of acute appendicitis. 7. In the adult male, which of the 3. Which of the following types of following is not part of the differential patients do not have an increased risk diagnosis of acute appendicitis? of perforation? A Regional ileitis A Extremes of age B Ureteric colic B Immunosuppressed C Perforated peptic ulcer C Diabetes mellitus D Acute pancreatitis D Pelvic position of appendix E Torsion of testis E Obese patient. F Mesenteric cyst. 4. Which of the following is not a sign of 8. In the adult female, which of the acute appendicitis? following is not part of the differential A Rovsing’s sign diagnosis of acute appendicitis?

495 ABDOMINAL ➜ Extended matchingquestions Extended Chooseandmatchthecorrectdiagnoseswitheachofscenariosgivenbelow: is Intheelderly,whichoffollowing 9. 496

5 4 3 2 1 G F E D C B A 1. Differential diagnosisofacuteappendicitis F E D C B A tympanitic note. over therightside ofhisabdomenwithsomerigidity butnorebound.Onpercussionthere isa heistenderall which togetanyrelieffromhispain. Hehassomestrangury. Onexamination positionin is unabletolocalise.Heinagony, writhingaroundandcannotfindacomfortable right-sidedabdominalpainwhichhe A 28-year-oldfemalecomplainsof sudden onsetofsevere and hasreboundtenderness. withrigidity of38°C, istenderovertherightiliacfossa shehaspyrexia Onexamination fossa. Thepainhas shifted totherightiliac unwell, hasvomitedacoupleoftimesandisanorexic. An 18-year-old abdominal painforabout6h.Shefeels femalecomplainsofgeneralisedcolicky tenderandrigidovertheright iliacfossa. (39°C)andvery heispyrexial examination antacids whichhehasbeentakingonandofffor‘indigestion’almost 18 months.On hours.Thisinitialpainsubsided withsome quadrant pain3daysagothatlastedforafew epigastricandrightupper wasprecededbyaboutofsudden-onsetsevere the rightiliacfossa overthelast2 days. Hispainin malecomplainsofpaininhisrightiliacfossa A 40-year-old rebound tendernessintherightiliacfossa. fromherpainandhastenderness, rigidityandminimal temperature of38°C,isindiscomfort In thepastshehashaddiarrhoeaonandoffforalmostayear. shehasa Onexamination timesandfeelsfeverish. between theattackssheisleftwithadullache.Shehasvomitedfew forthelast6h.In painintherightiliacfossa A 35-year-oldfemalecomplainsofcolicky abdomen. Shehassomediscolorationaroundherumbilicus. andhasmarkedtenderness,rigidityreboundtendernessoverherentirelower apyrexial sheisinagony,lookspaleandcold,clammysweaty. Onexamination (OCP). Sheis thirsty. Sheisnotsureaboutherlastmenstrualperiodandontheoralcontraceptivepill aroundhersuprapubicarea.Shefeelsfaintandisvery withsomeinitialdiscomfort It started abdomenof4hduration. paininhercentrallower A 25-year-oldfemalecomplainsofsevere Mittelschmerz pepticulcer Perforated Crohn’s disease Ruptured ectopicpregnancy ofcaecum Carcinoma Right uretericcolic Acute appendicitis acute appendicitis? not partofthedifferentialdiagnosis Torsion orrupturedovariancyst. Ruptured ectopicpregnancy colic/acutecholecystitis Biliary Pyelonephritis disease Pelvic inflammatory Mittelschmerz F E D C B A Bladder calculus. ofthecaecum Carcinoma aneurysm Leaking abdominalaortic Mesenteric infarction Intestinal obstruction Diverticulitis ➜ ➜ ➜ 2E 1D matchingquestions Answers: Extended 9. 8. F 7. D F 6. E 5.A,B,C,E 4. E 3. E 2.A,B,C,D 1. A Answers: Multiplechoicequestions

1. Differential diagnosisofacuteappendicitis Clinical aspects Anatom 7 6 obstructive feature. Her past history of diarrhoea should alert one to the possibility of primary onetothepossibilityof primary ofdiarrhoeashould alert obstructive feature. Herpasthistory denoting an right-sidedabdominal painfromthestart, This femalepatient complainsofcolicky (bloodstained discolorationaround theumbilicus),ofarupturedectopicpregnancy. withCullen’ssign theclinical pictureistypical,particularly shock. AlthoughsheisontheOCP, abdominalpainand haspresentedwithfeaturesofhypovolaemic lower severe onset ofvery This isayoungfemalewhounsure abouthermenstrualperiods.Shehashadasudden appendicectomy. anunnecessary acuteappendicitisandthusprevents Alvarado scoreof5–6,helpstoexclude scanisnotusedroutinely. inanequivocal Contrast-enhanced CT Selectiveuse,particularly Murphy’s signisfoundinacutecholecystitis appendix andreducetheriskofperforation. The obesepatientwillhaveamplegreateromentum.Thishelplocalisetheacutelyinflamed appendix resolves.Thiscausesthetodistendwithmucus,resultinginamucocele. A mucoceleisnotavariationofacuteappendicitis.Itoccurswheninflammationthe branchoftheileocolicartery. arisesfromthelower The appendicularartery rigidity andreboundtendernessintherightiliacfossa. helookspaleandistenderwith ofbreathduringhisnormalactivities.Onexamination short months,beingunduly Hehasfeltunwellforafew has beenconstantlyintherightiliacfossa. of24hduration.Thepain malepatientcomplainsofpaininhisrightiliacfossa A 60-year-old tenderness intherightiliacfossa. andistenderwithrigidityrebound shelooksslightlypale,apyrexial ago. Onexamination abdomen.Herlastmenstrualperiodwas2weeks suddenly andhasspreadalloverthelower Thepainstarted A 22-year-oldfemalepatientcomplainsofpainintherightiliacfossa. y 497

67: THE VERMIFORM APPENDIX ABDOMINAL 5B 4A 3F 7C 6G 498 not causeanaemia. torememberthattrueacuteappendicitiswill masquerading asacuteappendicitis.Itisimportant typicalofacarcinomathecaecum Thescenarioisvery He hassignsintherightiliacfossa. ofbreathinhisdailyroutineandlookspale–featuresanaemichypoxia. He hasfeltshort patienthaspainandsignsofanacuteabdominalepisodeintherightiliacfossa. This 60-year-old is Mittelschmerz. Thediagnosis abdomenofbleedingfromarupturedluteincyst. period. Shehassignsinherlower halfwaythroughhermenstrual painintherightiliacfossa This youngwomanhasdeveloped accompaniment ofuretericcolic. atympaniticabdomenwhichisduetoileus,common shows Abdominal examination to localise.Heisinagonyandwrithingaroundpain,unablefindapositionofcomfort. right-sidedabdominalpainwhichheisunable This youngmanhassuddenonsetofsevere it subsequentlymoved(rightiliacfossa). (periumbilical)andwhere pointtowherethepainstarted of herpain,shewouldalmostcertainly ofvisceral-somaticsequencepain.Whenaskedtopointthesite in keepingwithherhistory This 18-year-old is femalehastheclassicalfeaturesofacuteappendicitis.Theclinicalexamination right paracolicgutter,mimickingacuteappendicitis. alongthe closed offbyomentumandtheleakedcontentshavegravitatedtorightiliacfossa Thisscenarioistypicalofaleakingpepticulcerthathas been of peritonitisintherightiliacfossa. hedidnotgetbetterandhascomeinwithsigns relief bytakingsomemoreantacids.However, pain wasthoughtbythepatienttobeanacuteepisodeofhisindigestionforwhichhetriedget had asuddenattackofepigastricandrighthypochondrialpainfromleakingpepticulcer. This This patientwhohassufferedfromindigestioninthepast,forwhichhetakenantacids, features shouldpointtoadiagnosisofacuteterminalileitisfromCrohn’sdisease. pathology.bowel These Sheisgenerallyfeelingunwellwithacutesignsintherightiliacfossa. 68 The rectum

Multiple choice questions ➜ Anatomy B A cancer in the upper third of the rectum 1. Which of the following statements are can be felt by the index finger. false? C A proctoscope visualises the lower 10 cm A The adult rectum is approximately 15 cm of the rectum. long. D A rigid sigmoidoscope is 25–30 cm in B The anorectal angle is 120°. length. C The fascia in front of the rectum is E A flexible sigmoidoscope visualises Waldeyer’s fascia. 45–50 cm of the terminal large bowel. D The fascia at the back of the rectum is Denonvilliers’ fascia. ➜ Rectal injury E Waldeyer’s and Denonvilliers’ fascias 5. Which of the following is not a prevent the spread of cancer. mechanism of rectal injury? A Falling over a pointed object, causing 2. Which of the following statements are impalement true? B Sexual assault A The part of the rectum below the middle C Crush injury during prolonged childbirth valve of Houston is called the ampulla. from the fetal head B The superior rectal artery arises from the D Penetrating injuries, including gunshot internal iliac artery. trauma C The middle rectal artery is contained in E Road traffic accidents. the lateral ligaments. D The superior rectal veins ultimately drain 6. Which of the following statements into the portal system. regarding the treatment of rectal E The usual drainage flow of the injuries are false? lymphatics is upwards. A A computed tomography (CT) scan ➜ with rectal contrast is useful for Clinical aspects assessment. 3. What are the main symptoms of rectal B Intraperitoneal tear is treated by closure disease? and defunctioning left iliac proximal A Rectal bleeding colostomy. B Tenesmus C In extraperitoneal injury, debride ment C Altered bowel habit of the external wound with left iliac D Mucous discharge defunctioning colostomy is carried out. E Anorexia D In a large defect, resection of the F Urinary frequency damaged bowel with end-to-end G Prolapse. anastomosis is done. Broad-spectrum antibiotic cover is 4. In clinical examination, which of the E mandatory. following statements are true? A Visual examination of the perianal region is essential to exclude fissures, fistulae or other painful conditions.

499 ABDOMINAL ➜ ➜ ➜ 10. In the pathology of rectal cancer, ofrectalcancer, Inthepathology 10. Inproctitis,whichofthefollowing 9. Inrectalprolapsewhichofthe 7. 8. Which of the following procedures Whichofthefollowing 8. 500

Rectal cancer Proctitis Rectal prolapse E D C B A E D C B A E D C B A E D C B A and inguinalnodes. iliac blood vesselstothepara-aortic, Lymphatic spreadoccursalong the cent. Synchronous cancersareseen in5per important. Circumferential localspreadisthemost prognosis. Mucoid adenocarcinomashaveapoor percent. account foralmost50 well-differentiatedcancers Low-grade Treatment isusuallyconservative. mainstay inthediagnosis. Proctoscopy andbiopsyarethe It canbeduetospecificinfections. tenesmus. It causesbleeding,diarrhoeaand rectum The conditionisalwayslimitedtothe Milligan–Morgan operation. Ripstein’s operation Delorme’s operation operation Hartmann’s Thiersch operation prolapse? are notdoneforcompleterectal All oftheabove. Children aretreatedconservatively. common inwomen. Full-thickness prolapseismuchmore an intussusception. Full-thickness prolapsecommencesas mucosal. or In childrentheprolapseispartial false? statementsare which ofthefollowing statements aretrue? statementsaretrue? following 11. Assessment in rectal cancer should inrectalcancer Assessment 11. 14. In the staging for rectal cancer, which Inthestagingforrectalcancer, 14. whichof forrectalcancer, Insurgery 13. Inallpatientswithrectalcancer, 12. E D C B A E D C B A E D C B A E D C B A include: routine preoperative preparation? isnotpartof which ofthefollowing All oftheabove. pelvis andendoluminalultrasound of Magnetic resonanceimaging(MRI) scanofliverandchest CT cancer synchronous barium enematoexclude colonographyor Colonoscopy orCT Sigmoidoscopy andbiopsy nodes. involvement ofonetothreelymph staging,N1indicatesthe In theTNM ormesorectalfascia. the serosa staging, T3indicatesinvasionthrough (tumour,node,metastasis) In theTNM metastasis, usuallytotheliver. Dukes’ stageDindicatesdistant per cent. The incidenceinDukes’stageCis50 per cent. The incidenceofDukes’stageAis15 aretrue? of thefollowing for cureinaselectedgroup. subsequent operationoffersachance Liver resectionforsecondariesata chemotherapy shouldbegiven. In node-positivedisease,adjuvant ispossibleinaselectedfew.excision Anterior resectionwithtotalmesorectal preoperatively bychemoradiotherapy. Locally advancedtumoursaretreated abdominoperineal resection. The vastmajorityaretreatedby statementsarefalse? the following Prophylaxis fordeepveinthrombosis Prophylactic antibiotics Radiotherapy Counselling andsitingofstoma preparation Mechanical bowel ➜ ➜ 1. C,D Answers: Multiplechoicequestions matchingquestions Extended Choose and match the following diagnosiswitheachofthescenariosgivenbelow Chooseandmatchthefollowing

Anatom 6 5 4 3 2 1 F E D C B A 1. Clinicalconditionsaffecting therectum rectum fromtheprostateandseminal vesiclesorthevagina. Thefasciainthe front iscalledDenonvilliers’fasciaandseparatesthe vertebrae. twosacral lower andthe fasciaandseparatestherectumfromcoccyx The fasciaatthebackiscalledWaldeyer’s sigmoidoscopy. Abiopsyhasbeentaken. thereisahardinduratedmasswhichcanbeseenat8cmon Onrectalexamination iliac fossa. hehasindentable massesinhisleft blood andmucusinhisstools.Onabdominalexamination ofhisrectumwithtenesmus. Hepasses of defaecationandafeelinginsufficientevacuation A 65-year-oldmalecomplainsofearlymorningdiarrhoeaforthelast4 months.Hehasurgency gurgle. ofrectumoutsidetheanus,andwhenitispushed backitreduceswitha has a5cmlength she has topushthelumpbackbutitreappearswhenshepassesamotion. Onexamination A 55-year-oldfemalecomplainsofalumpprotrudingfromheranusforalmost10 months.She been taken. prolapse. Onsigmoidoscopyatabout10 cm,thereisanulcerontheanteriorwall.Abiopsyhas rectal shehasapartial protruding fromheranuswhichshecanreduceherself.Onexamination with occasionalincontinence.Duringthistimeshehasbeenconstipatedandfeelsalump months ofmucusforseveral femalecomplainsofrectalbleedingandpassage A 40-year-old about 15 cmwithoutanyulceration. upto inflamedrectalmucosa thereisbloodonthefinger;sigmoidoscopyshows examination littlefaecalmatter.During amotionshepassesflatus,mucusandbloodvery Onrectal femalecomplainsofbleedingfromtherectumassociatedwithdefaecation. A 50-year-old years. ago hereturnedtoBritainafterlivingintheFar Eastforafew Abiopsyhasbeentaken.year On sigmoidoscopythereisamasswithanulceratedsurface. afleshymassisfeltinthemid-rectum. nothing tofindintheabdomen.Onrectalexamination, thereis A 35-year-oldmalecomplainsofbleedingperrectumandtenesmus.Onexamination He hasaserumpotassiumof2.2mmol/L. thatispalpable. thereisafrondedlargegrowth lost someweightandonrectalexamination hehas weak.Onexamination Recently thishasbecomeprofuseandmadehimfeelvery months. A 45-year-oldmalecomplainsofmucousdischargefromtherectumforseveral Amoebic granuloma Villous adenoma ofrectum Carcinoma Rectal prolapse rectalulcer Solitary Proctitis y 501

68: THE RECTUM ABDOMINAL ➜ ➜ ➜ ➜ ➜ 12. B,C 11. E 10. A,E 9.B,C,E 8.B,E 7. E 6. 5. D E 4.A,B,C,D 3.A,B,C,D,G 2.A,C,D,E 502

Rectal cancer Proctitis Rectal prolapse Rectal injur Clinical aspects artery arisesfromtheinternaliliacartery. artery Themiddlerectal isacontinuationoftheinferiormesentericartery. The superiorrectalartery rectal spread. Pre-operative radiotherapy isnotroutinelyusedinall patients.Itisusedincasesoflocal extra- contemplated thenthepatientshould becounselledaboutthepossibilityofaloopileostomy. anteriorresectionis ifalow because inananteriorresectionastoma isnotrequired.However, do not requireroutinecounsellingforstoma While allpatientsrequirethorough counselling, they nodes. principally upwardsviathesuperiorrectalvesselstopara-aortic 25 percent.Althoughthelymphaticsdrainalongbloodvessels,spread ofcanceroccurs percent;andhigh-gradeanaplastic cancersoccurin (moderately differentiated)occurin64 (well-differentiated)cancersaccountforonly11Low-grade percent;averagegradecancers ofthedisease. multiple biopsiestodeterminetheextent byafullcolonoscopy and andstoolculture.Thisshouldbefollowed bacteriological examination proctoscopy isnotsufficient.Sigmoidoscopyandbiopsyaredone,supplemented by toinvolvetheentirecolon–totalulcerativecolitis. Tocondition extends obtainadiagnosis, Although thediseaseislimitedto5–15 cmfromtheanusinmajority,10 percentthe done forprolapse.MilliganandMorgandescribedoneoftheoriginaloperationshaemorrhoids. andtherectumisclosed,notanoperation operationthesigmoidcolonisexcised In Hartmann’s is restored. continuity months),bowel procedure. Afterallthewoundshavehealed(andthismaytakeseveral the proximalendisbroughtoutasancolostomyanddistalclosedaHartmann’s and isexcised injuries.Theinjuredbowel bedoneinextensive anastomosisshouldnever Primary ideal circumstances. in cmlongandcanvisualiseuptothesplenicflexure sigmoidoscopeis60 an enema).Aflexible sigmoidoscopy(aftergivingthepatient byarigidsigmoidoscopyor,betterstill,flexible followed Thisshould be ofclinicalexamination. of therectumcanbefelt.Proctoscopyisanintegralpart ifapatientisaskedtostrain,cancerintheupperthird When doingadigitalrectalexamination, y ➜ 6D 5C 4B 3A 2F 1E matchingquestions Answers: Extended 14. A,B,C,E 13. A,C

1. Clinicalconditionsaffecting therectum anterior resection, a temporary defunctioningileostomyissometimescarriedout.Thisclosed anterior resection,atemporary Inalow vast majorityaresuitableforrestorativeanteriorresectionwithtotalmesorectalexcision. inonlyaminority ofpatientswithrectalcancer.Abdominoperineal resectionisperformed The resection. ofpelvis. Mostpatientsaresuitableforrestorative anterior ofabdomenandMRI and stagingbyCT asynchronouscancer(5percent) colonographyor bariumenematoexclude colonoscopy, CT an obstructingrectalcancer. Confirmationisbybiopsyatsigmoidoscopy. byfull Thisisfollowed proper motionbutonlywindandwater. faecesproximalto Theabdominallumpsareinspissated are typicalofacarcinomatherectum. Itisreferredtoasspuriousdiarrhoeathereno This patient’ssymptomsofearlymorning diarrhoeaalongwithafeelingofinsufficientevacuation during manualreductionoftheprolapse. whichcause thegurglingsound anterior walloftheprolapse.Thiscontainsloopssmallbowel, associated withuterineprolapse.Thereisaperitonealpouch,likehernia, accompanyingthe This isfull-thicknessrectalprolapse,whichsixtimesmorecommon in womenandmaybe Confirmation isbyhistology. verge. Thereisaninternalrectalintussusceptioncausedbychronicstraining fromconstipation. is associatedwithrectalprolapseandanulcerontheanteriorwallat7–10 cmfromtheanal decades.Thecondition rectalulcer(syndrome)whichpeaksinthethirdtofourth This isasolitary ofscrapingsorswabsfromulcers,andserologicaltests. examination andcultureofthestools, bybacteriologicalexamination venereum, Aids,bilharziasis)isexcluded ( cause. Afullcolonoscopywithmultiplebiopsiesisthencarriedout.Diagnosisofspecificinfection This patientissufferingfromproctitis.Abiopsydoneonsigmoidoscopytodeterminethe where itisendemic.Thetreatmentmedical. establish thediagnosis.Theconditioncanrarelybeseeninapatientwhohasnotvisitedcountry ofthescrapingswithaspoonhelps A biopsyconfirmsthediagnosis.Microscopicexamination Amoebic granuloma(sometimescalledamoeboma)canbeeasilymistakenforacarcinoma. resection. resected endoscopicallybysubmucosal thesetumourscanbe hasbeenexcluded, anotherlesionhigherup.Ifmalignancy out toexclude tobecomemalignant.Acolonoscopyiscarried Thesehaveatendency hence thehypokalaemia. This isavillousadenomawhichcausesprofusemucousdischargethatrichinpotassium– ormesorectalfascia,itisT4. hasinvadedtheserosa growth ormesorectalfascia.If the invasion throughthemuscularispropriabutnotserosa staging,whichistheinternationallyrecognisedstagingsystem,T3indicatestumour In TNM after about6weeks,havingconfirmedtheintegrityofanastomosisbyabariumenema. Clostridium difficile , bacillary or amoebic dysentery, tuberculosis, oramoebicdysentery, , bacillary Gonococcus , lymphogranuloma 503

68: THE RECTUM 69 The anus and anal canal

Multiple choice questions ➜ Anal canal anatomy level of rectal termination in relation to 1. Which of the following are true the urinary bladder. regarding anal canal anatomy? C Low defects are easier to correct but A The anorectal ring marks the junction more prone to constipation. between the rectum and the anal canal. D Postanal dermoid usually remains B The puborectalis muscle is not concerned asymptomatic until adult life. with the continence mechanism. E A postanal dimple is of no clinical C The intersphincteric plane is not of much significance. significance. ➜ D The internal sphincter is the thickened Pilonidal sinus distal continuation of the circular muscle 4. Which of the following statements layer of the rectum. regarding pilonidal sinus are true? E The dentate line is an important A It is usually congenital. landmark representing the site of fusion B The primary sinuses are always in the of the proctodaeum and postallantoic midline between the sacrococcygeal joint gut. and the tip of coccyx. C Bascom’s procedure involves a midline ➜ Anal glands incision directly over the sinus cavity. The Karydakis operation avoids a midline 2. Which of the following are true D scar. regarding anal glands? It is more common in women. A These are found in the anal mucosa. E B They normally number between 20 and ➜ Anal incontinence 25 in an individual. C They drain via ducts into anal sinuses at 5. Which of the following statements the level of the dentate line. about anal incontinence are true? D They are widely considered as the A The most common cause of anal potential source of anal sepsis. sphincter disruption is obstetric E They secrete mucus, which helps to ease damage. defaecation. B Anal incontinence cannot occur in the absence of sphincter disruption. ➜ Congenital anomalies of C Double overlap repair is the standard technique to repair discrete muscle anal canal disruption. 3. Which of the following statements D Gracilis muscle transposition can be regarding the congenital anomalies of effective in approximately 20 per cent of the anal canal are true? patients in the long term. A The dorsal part of the cloacal membrane E Sacral nerve stimulation works by is called the anal membrane. neurophysiological modulation of the B Imperforate anus is divided into two hindgut by stimulation of the sacral nerve types – high and low – depending on the roots.

504 ➜ ➜ 8. Which of the following statements Whichofthefollowing 8. 7. Which of the following statements Whichofthefollowing 7. statements Whichofthefollowing 6.

Haemorrhoids Anal fissure E D C B A E D C B A E D C B A submucosa. and astripofmucosa is usedtoexcise method whereaspecialstaplinggun Stapled haemorrhoidectomyisa closed. completely andthemucosa are excised is aclosedtechniquewherethepiles Milligan–Morgan haemorrhoidectomy are alsomethodsoftreatingpiles. andinfraredcoagulation Cryotherapy of thepiles. pedicle whichcauseischaemicnecrosis tight elasticbandsontothebaseof Banding ofpilesinvolvesapplication grade 1piles. phenol isaneffectivetreatmentfor percent injection of50 Submucosal true? regarding managementofpilesare seen. Profuse haemorrhageisnever complication ofhaemorrhoids. pyaemiaisararebutknown Portal 3 haemorrhoids. prolapsed arecalledgrade Piles whichremainpermanently 1 haemorrhoids. Pain israreinuncomplicatedgrade Most haemorrhoidsarecongenital. 30 percentofpatients. incontinence, whichmayaffectupto after lateralinternalsphincterotomyis complication The mostimportant Diltiazem. trinitrate(GTN) or agents likeglyceryl per centofpatientsbytheuselocal Operation canbeavoidedinabout50 anal sphincter. The commonfindingisahypotonic It ismostcommonintheposteriorwall. rectal mucosa. It isalongitudinalsplitinthelower about haemorrhoidsaretrue? regarding analfissurearetrue? ➜ ➜ ➜ ➜ 11. Which of the following statements Whichofthefollowing 11. statements Whichofthefollowing 10. statements Whichofthefollowing 9. 12. Which of the following statements Whichofthefollowing 12.

Anal fistula Perianal abscess Pruritus ani Anal warts A E D C B A E D C B A A E D C B regarding pruritusaniaretrue? regarding analwarts aretrue? about analfistulaaretrue? regarding perianalabscessesaretrue? helpful. Washing theareawithsoapcanbe needed toconfirmdiagnosis. Skin biopsiesmaysometimesbe especially inyoungpatients. Threadworms needtobeexcluded, The perianalskinisusuallynormal. It isacommonproblem. They are associated with HPV infection. areassociatedwith HPV They Setons caneitherbe‘loose’or‘tight’. sphincter. more than30percentoftheexternal in trans-sphinctericfistulainvolving Fistulotomy isthetreatmentofchoice communication. trackandahorseshoe have acurved Posterior fistulaearemorelikelyto verge. openingclosetotheanal an external Trans-sphincteric fistulaeusuallyhave ulcerative colitis. It maybefoundinassociationwith underlying analfistula. on puscultureisassociatedwithan Finding Gram-positiveorganisms curettage. point, deroofingthecavityandfinger cruciate incisionoverthemostfluctuant Treatment ofabscessinvolves a at thetimeofdrainingabscess. Fistulotomy isadvisedifafistulafound underlying analfistula. are alwaysassociatedwithan They swelling intheperianalarea. present asapainful,throbbing They 505

69: THE ANUS AND ANAL CANAL ABDOMINAL ➜ ➜ Extended matchingquestions Extended Theriskfactorsforanalsquamous 13. Chooseandmatchthecorrectdiagnosiswitheachofscenariosgivenbelow: 506

8 7 6 5 4 3 2 1 H G F E D C B A 1. Diagnosisofrectal bleeding Anal cancer B A E D C B rectum. He is known to beHIV-positive. rectum. Heisknown tenesmusandfreshbloodper of severe A 30-year-oldmale presentswitha2-weekhistory problems andpresentlyhasastoma. thathermotherhasalsohadchronicbowel mucus. Shehaslostastoneinweight. Shesays ofdiarrhoeaalongwithbloodand A 24-year-oldfemalepresentswith a 6-monthhistory abdominal pain. bloody diarrhoeaandcolicky An 18-year-old student whohasjustreturnedfromtravellinginhisgapyearpresentswith abdominal painonandoffpainlessdarkbloodperrectum. ofconstipationisreferred tothehospitalwithlower femalewithalonghistory A 90-year-old bleeding. repaircomplainsofabdominalpainand darkishrectal (AAA) aneurysm abdominal aortic myocardialinfarctionsand arecent An 82-year-oldmalewhohashadthreeprevious ofhard stoolsandstraining. also hasstreaksoffreshbloodperrectum.Healonghistory analpainassociated withdefaecation.He ofsevere A 28-year-oldmaleisreferredwithahistory constipation. towards recent pregnancy. Shementionsatendency a following A 32-year-oldfemalecomplainsoffreshbloodperrectumandsomediscomfort ofincompletedefaecation. andasensation recent changeinbowels of malepresentswithbrightredpainlessbleeding.Hehasanassociatedhistory A 68-year-old Diverticulitis Ischaemic colitis Infective colitis Ulcerative colitis Rectal cancer Proctitis Anal fissure Haemorrhoids HIV infection HIV infection Human papillomavirus(HPV) podophyllin andsurgicalexcision. Treatment optionsinclude25percent Acetic acidishelpfulindiagnosis. last threedecades. The incidencehasdecreasedoverthe malignancy. with greaterriskofprogressionto subtypes4and8areassociated HPV cell carcinoma (SCC)cell carcinoma include: 14. Which of the following statements Whichofthefollowing 14. E D C B A E D C common symptoms. Pain andbleedingarethemost Treatment isusuallysurgical. the iliacnodesinpelvis. groupoflymphnodesare The primary areusuallySCCs. They cancers. all bowel constitutelessthan2percentof They aretrue? regarding analcancers Rectal cancer. Other genitalcancers Renal transplant ➜ ➜ ➜ Chooseandmatchthecorrectoperation witheachoftheconditionsgivenbelow: Chooseandmatchthecorrectinvestigationswitheachofscenariosgivenbelow:

D C B A 4. Aetiologyofanorectal disorders 6 5 4 3 2 1 F E D C B A 3. Treatment ofanorectal disorders 7 6 5 4 3 2 1 G F E D C B A 2. Investigationsinanorectal symptoms Urge incontinence Anal fistula Haemorrhoids Anal fissure Grade 3piles High analfistula Anal fissureresistanttomedicaltherapy Anal polyp treatmentofanalfissure Primary Rectal prolapse insertion open /seton Lay Altemeier’s procedure trinitrate Glyceryl Lateral internalsphincterotomy biopsy Excision Stapled anopexy diarrhoea andleft-sidedabdominalpain. ofdark-colouredrectalbleedingassociatedwith A 30-year-oldmalepresentswithhistory hard lumpattheanalverge. a1.5 reveals ofalumpintheanus.Examination cm A 65-year-oldmaleisreferredwithhistory yearsago. several delivery 2 years.Shehashadaperinealtearduringprevious femalepresentswithurgeincontinenceprogressivelygettingworseoverthelast A 58-year-old isnotpossible. examination analsphincterspasmandhencefurther reveals examination analpainpersistingdespitelocalmedications.Clinical A 28-year-oldmalepresentswithsevere arenormalandshehasnoabdominalsymptoms. bowels arecentpregnancy.A 26-year-oldfemaleisreferredwithfreshbloodperrectumfollowing Her operations. ofrecurrentanalfistulaeneedingseveral A 36-year-oldmaleisreferredwithahistory andweightloss. ofrecentchangeinbowels A 70-year-old maleisreferredwithahistory Anorectal physiology Biopsy underanaesthesia(EUA) Examination scan Magnetic resonanceimaging(MRI) sigmoidoscopy Flexible Colonoscopy Rigid sigmoidoscopy 507

69: THE ANUS AND ANAL CANAL ABDOMINAL ➜ ➜ ➜ 3.A,C,D, E 2.C,D,E 1. A,D,E Answers: Multiplechoicequestions Chooseandmatchthecorrectdiagnosiswitheachofdescriptionsgivenbelow: 508

Congenital anomaliesofanalcanal Anal glands Anal canalanatomy 8 7 6 5 4 3 2 1 H G F E prone X-ray at 24 h after birth indicates the distance between the rectum and the perineum. Low indicatesthedistance betweentherectumandperineum. Low prone X-rayat24 h afterbirth abnormalities.Alateral tract– andisusuallyassociatedwithcomplex rectum, vaginaandurinary The findingofasingleperinealorifice indicatesapersistentcloaca–commonopeningforthe defect. ishelpful.Presence ofmeconiumontheperineumindicatesalow examination Careful systems. andthegenitourinary by thetypeandassociatedabnormalities ofthesacrum termination oftherectuminrelation tothepelvicfloor. Treatment andprognosisareinfluenced of typesdependingonthelevel anushashistorically been dividedintohighandlow Imperforate number between0and10 inanindividual. normally andtheintersphinctericspace.They The analglandsarefoundinthesubmucosa for operationsonthesphinctermuscles. This istherouteforspreadofpus.planecanbeopenedupsurgically toprovideaccess potentialspaceandcontainsintersphincteric analglands. intersphincteric planeisanimportant ofthisfacilitatesdefaecation,andfailuretorelaxcausesanismus.The anorectal angle.Relaxation componentofthecontinencemechanism.Itmaintains The puborectalisisanimportant oftheinternalsphincter. sphincter,conjoinedlongitudinalmuscleandthehighestpart external ofthe the fingerasathickenedridge.Itisformedbyjoiningofpuborectalis,deeppart ends attheanalverge.Thisismarkedbyanorectalbundle(orring),whichcanbefeltwith wheretherectumpassesthroughpelvicfloorand The analcanalcommencesatthelevel glands. oftheanal andassociatedwithsepsisintheterminalpart The causeisusuallycryptogenic straining. This isassociatedwithdilatationandprolapseoftheanalvascularcushionsusuallydueto ischaemia. analsphincterwithresultantmucosal This iscausedbyahypertonic ofstrainingduringdefaecationwithaninternalintussusceptioninmostcases. This isevidence ofthepuborectalismuscleduringdefaecation. This isusuallycausedbythefailureofrelaxation infectionviruscanpredisposetothis. andHPV AIN The mostcommoncauseisanoccultobstetricsphincterinjury. oftherectum usuallycompoundedbystraining. This isessentiallyduetofailureofsupports rectalulcersyndrome(SRUS) Solitary Anismus Rectal prolapse Anal SCC ➜ ➜ ➜ ➜ 7. B,D 6.B,D,E 5.A,C,E 4.B,D

• • • • Haemorrhoids Anal fissure Anal incontinence Pilonidal sinus lateron. anorectoplasty–PSARP) (posteriorsagittal bydefinitivesurgery colostomy followed abnormalitiesrequireadefunctioning anomalies canbetreatedbyanoplasty,whilecomplex haemorrhoids areasfollows: deepintheskinsurroundinganalverge.The fourdegreesof haemorrhoidal plexus 7 and11 haemorrhoidsrelate tovenouschannelsoftheinferior o’clockpositions.External Internal haemorrhoidsaresymptomaticanalvascularcushionswhichtypically lieinthe3, hence beentriedwithequallygoodresults. has potentialrisksofanalincontinence.Alternativemethodssuchasadvancementflapshave analsphincter.finding isahypertonic Lateralinternalsphincterotomyisthe‘goldstandard’but defaecation, lesselasticanodermandrelativelysmallbloodsupply. Themostcommonassociated shearingforcesactingatthatsite the posteriormidlineperhapsrelatestoexaggerated An analfissureisalongitudinalsplitintheanodermofdistalcanal.Thelocation percentofpatientsinthelongterm. about 60 term. Ifdirectrepairisnotpossiblegracilismuscletranspositionhasbeenfoundtobeeffectivein percentinthelong percentofpatientswhichmayreduceto50 repair issuccessfulinabout80 due toneuropathyandurgeincontinencevariousreasons.Doubleoverlapsphincter andtrauma.Faecal incontinencecanoccurwithoutsphincterdisruption byanalsurgery followed Obstetric damageisthemostcommoncauseofanalincontinenceduetosphincterdisruption the gapwithawoundoffmidline. tissuestoclose and aimstoremovetheaffectedtissues,flattendeepnatalcleftbringnew procedureinvolvesalateraladvancementflap andclosureofthemidlinepits.Karydakis’s excision procedure involvesanincisionlateraltothemidlinegainaccesssinuscavityalongwith prolonged hoursinfrontofacomputer. Bascom’s Culturalfactorsmayalsobeimportant. factors whichinvolveprolongedsitting,e.g.long-distancedrivers,cabdriversandworkinvolving area,deepnatalcleftandoccupational in menandthepredisposingfactorsincludeahairy lined bygranulationtissueandcontaininghairlyinglooselywithinthelumen.Itismorecommon consisting ofoneormoremidlineopenings.Theseopeningscommunicatewithafibroustrack The termpilonidalsinusdescribesaconditionfoundinthenatalcleftoverlyingcoccyx, common symptom and can be profuse or sustained, leading to anaemia. Portal pyaemiais common symptom andcanbeprofuseorsustained, leadingtoanaemia.Portal trauma.Bleedingisthemost lead tocaudaldisplacement oftheanalcushionsand mucosal ageing, maybecontributory. thatshearingforcesactingontheanus Itiscurrently believed ofcollagenthanmuscledueto repeated infectionoftheanallining andahigherproportion first degree–bleedonly,noprolapse degree–permanentlyprolapsed. fourth third degree–prolapseandhavetobemanuallyreduced second degree–prolapse butreducespontaneously Haemorrhoids are acquired, and numerous predisposing factors, such as anal hypertonia, Haemorrhoids areacquired,andnumerous predisposingfactors,suchasanalhypertonia, 509

69: THE ANUS AND ANAL CANAL ABDOMINAL 8.B,C,E ➜ ➜ ➜ ➜ 12. A,D,E 11. C,E 10. A,D 9.A,C,D 510

Anal warts Anal fistula Perianal abscess Pruritus ani more popularintheUSA. involves dissectionandligationofthepilesleavingwoundopen.Theclosedtechniqueis ofeachpile.Milligan–Morganhaemorrhoidectomy(opentechnique) attheapex submucosally Injection sclerotherapyinvolvesinjecting5–10 mLof5percentphenolinalmond/arachisoil haemorrhoidectomy. sclerotherapy orbanding,butgrade3and4varietiesaremorelikelytoneedopen/stapled a rarecomplicationafterstrangulation.Thegrade1and2pilescanbemanagedbyinjection colorectal surgeons. There has been an increase in this condition over the last three decades. They colorectal surgeons. Therehasbeenanincreaseinthis conditionoverthelastthreedecades. They transmitteddiseaseencounteredby Condyloma accuminataisthemost commonsexually ‘tight’, forcutting.Analflapsandplugs canalsobeusefulinselectedpatients. Thesetoncanbe‘loose’–fordrainage/markeror lay-openwithaseton insertion. do apartial sphincter. optionwhenthetrack involves thesphinctermusclesisto Thesafer cent oftheexternal fistula) isdoneinintersphinctericandsometrans-sphinctericfistulaeinvolving lessthan30per trackwithaninternalopeninginthemidlineposteriorly.curved Afistulotomy(layingopenofthe opening ismorelikelytohavea likely tohaveastraighttrackwhileposteriorlyplacedexternal openingismore predicts thetypeoftrackandlocationinternalopening–ananterior external rule openingclosetothe analverge.Goodsall’s Intersphincteric fistulaeusuallyhaveanexternal oftheinternalopeninginrelationto dentate line. dependingonthelevel as ‘high’or‘low’ areclassified varieties. They sphincteric, intersphincteric,suprasphinctericandextrasphincteric trackintotrans- aredividedintovarioustypesdependingontherelationofprimary They perianal skin.Thismaybefoundinassociationwithspecificconditions suchasCrohn’sdisease. An analfistulaisanabnormalchroniccommunicationbetweenthe canalandthesurrounding hands. inthemostexperienced anaesthetic. Itisnotrecommendedtodoimmediatefistulotomyexcept skinsource.Treatmentpoint toasuperficial involvesdrainingtheabscessundergeneral suggestsanunderlyinganalfistula,andGram-positive organisms organisms onpusswabfurther byapersistentanalfistula.FindingGram-negative andmaybefollowed space (cryptoglandular) itisasaresultofsepsisintheintersphincteric infection. Inabouthalfofthecases,however, associated withanunderlyinganalfistula.Thiscanarisefromasimpleboilorskinappendage Acute sepsisintheanalregioniscommonandmenaremoreaffected.Notallcases ismadetofindandtreatthecause. effort need toberuledoutbybiopsy. Symptomatic treatment remainsthemainstay,althoughevery toilet paper. may Soapisavoidedandreplacedbywateralone.Analintraepithelialneoplasia(AIN) for thecausesis‘pus,polyps,parasites,pilesandpsyche’.Cottonwoolshouldbesubstituted usually reddenedandhyperkeratotic.Itcanalsobecomecrackedmoist.Ausefulmnemonic This isacommonandembarrassingconditionwhichcanbedifficulttotreat.Theperianalskin ➜ ➜ 6F 5H 4G 3B 2A 1D matchingquestions Answers: Extended 14. A,B,E 13. A,B,C,D 7E

1. Diagnosisofrectal bleeding Anal cancer warts. isdoneinextensive Staged excision tumour). mayleadtogiantcondylomata(Buschke–Lowenstein carpet theskin.Relentlessgrowth areinitiallydiscretebutlateroncoalesceto bleeding andpainaretheusualsymptoms.Thewarts are16,a greaterriskofmalignancy 18, 31 and33.Manyareasymptomaticbutpruritus,discharge, subtypesofthisvirusbuttheassociatedwith Therearemorethan80 are causedbyHPV. Recent internationaltravelmightsuggest aninfectivecause–amoebiasisand arenotoftenassociatedwithacuteinflammation. painless asthey surgery. Thismaybe rarelyrequireemergency bleedsareself-limitingandvery Most diverticular thesurgeon. lactate shouldalert bleed.Thepresence ofperitonitisorraised The mostcommondifferentialdiagnosisisdiverticular gastrointestinal bleed. presentingwithlower arteriopath Consider ischaemiccolitisinaknown analsphincterspasmandasentinel skintag. ispossible,whichusuallyreveals examination analpainaggravatedbydefaecation. Onlylimited The typicalsymptomofanalfissureissevere duringpregnancy. Most pilesinfemalesstart withmalignancy. frequentlycoexist symptoms.Pilesanddiverticuloses bowel Rectal cancershouldberuledoutinanypatientthisagegrouppresentingwithnew-onset ifthatfails. chemoradiotherapy inthefirstinstance(Nigroregimen),andmajorablativesurgery inguinal group,althoughsometimesretrogradeandalternativespreadsmayoccur. Treatment isby groupoflymphnodesarethe patients. Thismayaffectthevergeoranalcanal.Theprimary andmorecommoninHIV-positive Anal cancerisanuncommonassociatedwithHPV confusion. prevent disease,thoughoftenused,isbestavoidedto ThetermBowen’s carcinoma hasbeenobserved. butprogressiontoinvasive isuncertain 3).Thenaturalhistory 1)tothefullthickness(AIN (AIN third oftheproliferativezonefromlower maturationandtheextension lack ofkeratocyte 1, 3accordingtothe according tothedegreeofdysplasiaintothreetypes–AIN 2andAIN AIN perianal orintra-analdermis.Thiscanbeasymptomaticandconfirmedonbiopsy. Itisclassified isavirallyinduceddysplasiaofthe AIN Rectal adenocarcinomaisnotariskfactorforanalSCC. bleeding and change in bowel habits. bleeding andchange inbowel diseasewillneedto beruledoutintheyoungpatientpresentingwith bowel Inflammatory common causesofbloodydysentery. Shigella are 511

69: THE ANUS AND ANAL CANAL ABDOMINAL ➜ ➜ ➜ 1F,2D,3E,4G,5H,6A,7B,8C 1E,2D,3B,4C,5F,6A 1B,2D,3A,4E,5G,6F,7C 8C 512

4. Aetiologyofanorectal disorders 3. Treatment ofanorectal disorders 2. Investigationsinanorectal symptoms Tenesmus mayberequiredfordetailedevaluation. isusuallyassociatedwithproctitis.AnEUA PART 12 Genitourinary

70 Urinary symptoms and investigations 515 71 The kidneys and ureters 521 72 The urinary bladder 533 73 The prostate and seminal vesicles 541 74 The urethra and penis 546 75 The testis and scrotum 549 76 Gynaecology 554 This page intentionally left blank 70 Urinary symptoms and investigations

Multiple choice questions ➜ Haematuria C In glomerulonephritis or acute cortical 1. Which of the following statements are necrosis, loss of glomeruli causes renal false? failure. A Microscopic haematuria is not always D In pyelonephritis, tubular function is abnormal. impaired, causing renal failure. B Haematuria at the start of urinary stream E Cytological examination of urine indicates a cause in the lower urinary tract. is more likely to be abnormal in C Haematuria where the urine is uniformly poorly differentiated rather than well- mixed with the urine points to a cause in differentiated transitional cell cancer of the upper urinary tract. bladder. D Terminal haematuria is caused by ➜ bladder irritation or infection. Imaging in urological E Painful haematuria indicates malignant conditions pathology. 4. Which of the following statements are true? ➜ Pain of urological origin A Most urinary calculi are radiodense. 2. Which of the following statements are B Uric acid calculi can be seen on a plain true? X-ray. A Pain of renal origin is a deep-seated, C Ultrasound scanning (US) provides sickening ache. broadly similar anatomical information as B Pain from a ureteric stone is colicky and an intravenous urogram (IVU). the patient rolls around in agony. D IVU has no advantage over US. C Pain from the urinary bladder is a E IVU can be dangerous. suprapubic discomfort. D Perineal pain is a penetrating ache and ➜ Renal failure can occur in both sexes 5. Which of the following statements are E All of the above. false? A Anuria is defined as complete absence of ➜ Preliminary investigations urine production. and renal failure B Oliguria is defined as an urinary output 3. Which of the following statements are of less than 300 mL in 24 h. false? C Certain drugs can cause renal failure. A If 30 per cent of kidney function is lost, D All patients with renal failure will require renal failure becomes evident by blood renal replacement therapy. results. E Indwelling stents can be used to relieve B In hypertension and renal artery stenosis, ureteric obstruction. the plasma flow is impaired, causing renal failure.

515 GENITOURINARY MatchtheinvestigationswithFigs70.1–70.5 below: ➜ Extended matchingquestions Extended 516

E D C B A 1. Imaging Retrograde ureterogram Intravenous urogram Antegrade pyelogram Renal angiogram bladder scanofurinary CT Figure 70.1a Figure 70.1b Figure 70.3 Figure 70.2 517

70: URINARY SYMPTOMS AND INVESTIGATIONS GENITOURINARY 518 Figure 70.4 Figure 70.5 ➜ ➜ ➜ ➜ ➜ ➜ Fig.70.3, B Fig.70.2, E Fig.70.1a andb,A questions matching Extended Answers: 5. D 4.A,C,E 3. A 2. E 1. A,E Answers: Multiplechoice questions

1. Imaging Renal failure Imaging inurological conditions Preliminary investigationsandrenal failure Pain ofurological origin Haematuria to theright. largeandvoscular leftrenalcellcarcinomapushingtheaorta avery Left renalangiogramshowing carcinoma. irregularfillingdefect inleftrenalpelvisfromtransitionalcell Left retrogradeureterogramshowing bladderfrom vesicocolicfistula. airintheurinary scanshowing Computed tomography(CT) someformofdialysisisindicated. life-threateninghyperkalaemia, severe measureshelpedbyanephrologisthave failedandthepatienthas cause. Whenallconservative should beestablished–prerenal,renalorpostrenal.Thepatientistreated dependingonthe All patientswithrenalfailurewillnotrequirereplacementtherapy. Thecauseofrenalfailure hypersensitivityreactions. a minorityofpatients,theintravenouscontrastcancausesevere canbedangerousas,in IVU doesnot.However, renalfunction,whichUS shows inthatanIVU US Uric acidcalculiareradiolucentandhencenotvisibleonaplainX-ray. hastheadvantageover IVU havealargefunctionalreserve. because thekidneys More than70 functionmustbelostbeforerenalfailurebecomesevident percentofkidney tooccurinwomen. of prostaticoriginitisknown strangury. Painintheperineumisapenetratingacheradiatingtorectum. Althoughitisusually attheendofmicturitionreferred toas Bladder painmaytaketheformofwrenchingdiscomfort position,andthepainradiatestogroingenitalia. around unabletofindacomfortable withthepatientwrithing painsever, of thehip.Uretericcalculuscausesonemostsevere theremaybedeeplocaltendernesswithpsoasspasmcausingflexion cause isinflammatory, Renal painisadeepacheintheloinandcausedbystretchingofrenalcapsule.When malignant causeisusuallypainlessalthoughrenalcellcarcinomapresentswithloinpain. Haematuria, whethermacroscopicormicroscopic,isalwaysabnormalHaematuriafroma 519

70: URINARY SYMPTOMS AND INVESTIGATIONS GENITOURINARY Fig.70.5, D Fig.70.4, C 520 pelvis. multiplefillingdefectsfromtransitionalcellcarcinomaofrenal Left antegradepyelogramshowing carcinoma. irregularfillingdefectinleftrenalpelvisfromtransitionalcell Intravenous urogramshowing 71 The kidneys and ureters

Multiple choice questions ➜ Congenital abnormalities of D Intravenous urography or contrast- the kidney, renal pelvis and enhanced computed tomography (CECT) ureters scan should be performed urgently in suspected renal injury. 1. Which of the following statements are E Hypertension may be a long-term true? complication of renal injury. A The incidence of absence of one kidney is 1 in 5000. 4. Which of the following statements are B The incidence of ectopic kidney is 1 in false? 1000. A Most ureteric injuries are due to surgical C Horseshoe kidney occurs in 1 in 1000. trauma during pelvic surgery. D Polycystic kidneys can be transmitted by B Preoperative ureteric catheterisation either parent as an autosomal dominant helps to protect them from injury during trait. operation. E Polycystic kidneys present in childhood C When recognised during operation, as renal failure. an injured ureter should be repaired immediately. 2. Which of the following statements are D When ureteric injury is diagnosed false? postoperatively, delayed repair is A Duplication of the renal pelvis is found in undertaken. 4 per cent of patients. E Urinary fistula through an abdominal B Duplication of the ureter is found in or vaginal wound indicates a damaged 3 per cent of patients. ureter. C Ureterocele is a cystic enlargement of the intramural ureter. ➜ Clinical conditions D All ureteroceles must be treated by endoscopic diathermy incision. 5. In hydronephrosis from pelviureteric E Retrocaval ureter can cause obstructive junction (PUJ) obstruction (idiopathic symptoms. or congenital hydronephrosis), which of the following statements are false? ➜ Renal and ureteric injury A The condition may be asymptomatic. It can be diagnosed in utero. 3. Which of the following statements are B Ultrasound scanning is the least invasive true? C method of imaging. A Haematuria following trivial injury of the kidney indicates a previously pathological D An intravenous urogram (IVU) is the kidney. ideal imaging. B Closed renal injury is always E If there is more than 5 per cent of extraperitoneal. function in the obstructed kidney, a procedure to preserve the kidney is C In closed renal injury, surgical exploration carried out. is necessary in the majority.

521 GENITOURINARY . Withregardtorenalstones,whichof 6. 9. Which of the following statements Whichofthefollowing 9. imaging Whichofthefollowing 8. statementsare Whichofthefollowing 7. 522 D C B A E D C B A E D C B A E D C B A poorer functionis treatedfirst. with In bilateralrenalstones,thekidney minimal-access techniques. calculicanbetreatedby Most urinary requires surgicalintervention. tractobstructionduetoa stone urinary Infection inthepresenceofupper pass spontaneously. Ureteric calculismallerthan1cmwill arefalse? calculi regarding themanagementofurinary Retrograde pyelography. Ultrasound (US) IVU CECT X-ray ureterand bladder(KUB) Plain kidney, disease? calculus techniques areusedforurinary physicalsigns. few There arevery Haematuria iscommon. common. with apalpableloinswellingis Hydronephrosis and/orpyonephrosis related tothesizeofstone. ofpainuretericcolicis The severity X-ray. Renal stonesareusuallyvisibleonaplain false? phosphate. of calcium-ammonium-magnesium A staghorncalculusiscomposed Pure uricacidstonesareradio-opaque. calculus. be removedbeforetreatmentofthe be thecauseofrenalcalculi,itshould If aparathyroidadenomaisfoundto with radio-opaquecalculi. 5 percentorlessofthosepresenting Hyperparathyroidism isfoundin E. coli. stonesarestaphylococciand for urinary The commonbacteriafoundasanidus the following statementsaretrue? the following ➜ 11. Which of the following statementsare Whichofthefollowing 11. statementsare Whichofthefollowing 10. 12. Which of the following statementsare Whichofthefollowing 12.

Infections E E D C B A E D C B A G F E D C B A loin approach. throughanextraperitoneal performed Open operationsforrenalstonesare true inuretericstones? false? fragmentation. ureteroscopic electrohydraulic Most stonesaretreatedby two sitesofuretericnarrowing. Stones arecommonlyarrestedatthe stone shouldberemoved. If obstructionpersistsafter6weeks,the The majoritypassspontaneously. None oftheabove. All oftheabove. abscess. ofaretrocaecalappendix extension Perinephric abscess mayoccurfrom in theimmunocompromised. Renal carbuncleismostcommonly seen due toastonecausingobstruction. hydronephrosis andismostcommonly Pyonephrosis isaninfected associated withvesicouretericreflux. Chronic pyelonephritisisoften possibility ofrenaltuberculosis. onetothe Sterile pyuriashouldalert true? ofmicturition. dysuria andfrequency All patientspresentelectivelywith abnormality. infectionhaveanunderlying urinary percentofchildrenwith Up to50 acidic urine. Proteus responsible. organisms aremostcommonly E. coli is themostcommonroute. tract infectionintheurinary Ascending passed. day ortwosuggeststhatthestonehas renalpainsubsidingaftera Severe andotherGram-negative andstaphylococcithrivein ➜ ➜ ➜ Extended matchingquestions Extended statements Whichofthefollowing 13. Choose and match the following diagnoseswitheachofthescenariosgivenbelow: Chooseandmatchthefollowing

E D C B A 2. Renalpatholog 5 4 3 2 1 E D C B A 1. Renalpatholog Renal neoplasms E D C B A Hydronephrosis Horseshoe kidney Hypernephroma Staghorn calculus Renal trauma tendernessinherright loin. extreme in herrightloin.Shehadabloodpressureof160/70 and mmHg,boundingpulseof90/min withhigh temperature, rigorsandpain femalepatientwasseenasanemergency A 60-year-old his leftloin. Hisbloodpressurewas110/60A&E department. mmHg,pulse110/min andhehadfullnessin haematuria becameworseandhenoticedsomefullnessinhisleftloin. Hethencametothe inhisleftloin. Ongoinghome,his his urineinthechangingroom.Hefeltsomediscomfort whichhepassed frankbloodin A 20-year-oldfityoungmanhadagameofrugby,following of redbloodcellsandagrowth shows examination shehastendernessinherloinoverthekidney. months.Onexamination Urine for several frequency A 45-year-oldfemalepatientcomplainsofnaggingpaininherrightloin andurinary avaricocelewhichhefindsuncomfortable. reveals andscrotalexamination kidney, hehasanenlargedleft abdomen. Hepassesclotswhichare‘worm-like’.Onexamination lower mancomplainsofhaematuriaassociatedwithpaininhisleftloinradiatingtothe A 60-year-old position. heiswrithingaroundinpainandcannotfindacomfortable examination frequency. On groin andthelefttestis.Hehasvomitedacoupleoftimesurinary paininhisleftloinof4hduration,radiatingtothefrontlumbarareaand severe complainingofasuddenonsetvery manhascometotheA&Edepartment A 40-year-old Ureteric calculus Pyonephrosis Congenital (idiopathic)hydronephrosis Hypernephroma Renal stone and radiotherapy. Treatment isbychemotherapy,surgery andMRI. CT Imaging modalitiesareUS, Lymphatic spreadisrare. commonest presentations. arethe Haematuria andfever age of5years. the nephrogenic tissueoccurringbelow It isatumourofembryonic about Wilms’tumourisfalse? y y imaging 14. Which of the following statementsare Whichofthefollowing 14. Proteus F E D C B A true abouthypernephroma? and radiotherapyshouldbegiven. nephrectomy,chemotherapy Following methods ofchoice. aretheimaging andCECT US IVU, commonest presentations. Haematuria andclotcolicarethe presenting featureinamale. A left-sidedvaricocelemaybea local symptoms. In 25percentofpatientsthereareno It occursmorecommonlyinwomen. andstaphylococci. 523

71: THE KIDNEYS AND URETERS GENITOURINARY Chooseandmatchthediagnoseswithimages(Figs71.1–71.5) below: shown 524 Figure 71.1 Figure 71.2 Figure 71.4 Figure 71.3 Figure 71.3 525

71: THE KIDNEYS AND URETERS GENITOURINARY ➜ 526

E D C B A 3. Ureteric patholog Antegrade pyelogram showing transitionalcellcarcinomaoftherenalpelvis Antegrade pyelogramshowing transitionalcellcarcinomaoftheleftrenalpelvis showing IVU Left uretericcalculus transitionalcellcarcinomaoftherenalpelvis Retrograde pyelogramshowing oftheureter Carcinoma Figure 71.5 y imaging Chooseandmatchthediagnoseswithimages(Figs71.6–7 Figure 71.7 Figure 71.6 1.10) below: shown 527

71: THE KIDNEYS AND URETERS GENITOURINARY 528 Figure 71.9 Figure 71.8 ➜ ➜ 3.A,D,E 2. D 1. B,C,D Answers: Multiplechoicequestions

Renal andureteric injur Congenital abnormalitiesofthekidne 10 cases. percentofclosed renalinjury inlessthan isnecessary Surgicalexploration is conservative. of managementrenalinjury tear withtherenalcapsulecausing urine andbloodtoleakintotheperitoneum.Themainstay can fat,theperitoneum,whichiscloselyadherentofkidney, littleextraperitoneal have very inchildren,becausethey inadults.However, isusuallyextraperitoneal Closed renalinjury result inpostoperativevesicouretericreflux. is recurrentinfectionandstoneformation,ittreatedbyendoscopicdiathermy incision.Thiscan shouldbe leftalone.Ifthere Ureteroceles donotalwaysproducesymptoms,inwhichcasethey clinically beforetheageof30years. on standardimaginginthesecondandthirddecades.Theconditiondoes notmanifestitself areusuallydetected kidneys pole.Polycystic areusuallyfusedbyitsisthmusatthelower kidneys The incidenceofrenalagenesisis1in1400 andis mostoftendiagnosedincidentally. Horseshoe Figure 71.10 y y , renal pelvisandureters 529

71: THE KIDNEYS AND URETERS GENITOURINARY ➜ ➜ 12. F 11. C,E 10. A,D 9.A,D 8.A,B,C,D 7. B,C 6.A,B,C,E 5.D,E 4. D 530

Infections Clinical conditions of injury andrepairundertaken. of injury thenature toevaluate oraCECT general conditionisgood.ThepatientshouldundergoanIVU providedthepatient’s isdiagnosedpostoperatively,earlyrepairsafe When uretericinjury features ofshock. withsepticaemia presentingwithhightemperature,rigors,tachycardiaand seen asanemergency are predisposingtostoneformation. Whilepatientsmaypresentelectively,oftenthey alkaline, Proteus ureteric obstruction. renalpainsubsidingafteradayorsoissinistersymptom:it denotescomplete orifice. Severe vessels, proximityofvasdeferensorbroadligament,entrancetothebladder wallandureteric pelviuretericjunction, crossingoftheiliac arrested atoneofthe5anatomicalsitesnarrowing: itmaycauserenalatrophyandlossoffunction.Intheureter, stonesarecommonly otherwise If obstructionfromauretericstonepersistsafter1or2weeks,the shouldberemoved; withpainorpyonephrosisistreatedfirstbydecompression bynephrostomy.is thatthekidney withbetterfunctionistreated first.Theexception Inbilateralrenalstones,thekidney intervention. Ureteric calculismallerthan0.5cmwillpassspontaneously,largerstonesrequiringsurgical shockwavelithotripsy(ESWL). with extracorporeal uptheanatomicalsiteofstone.Ultrasoundisessentialfortreatment available andalsoshows isnot hasaroleifCECT isthegoldstandardofinvestigationinacuteuretericcolic.IVU CECT gallstones, foreignbodies,phlebolithsandcalcifiedadrenalglandmaycauseconfusion.Spiral Plain radiographyhasalimitedplaceasopacitiesfromcalcifiedmesentericlymphnodes, position. Hydronephrosisorpyonephrosisisrare.Moststonespassspontaneously. passing fromthelointogroinwithpatientwrithingroundunablefindacomfortable ofuretericcolicisnotrelatedtothesizestone.Thepainagonising,typically The severity onplainX-ray.therefore castafaintshadow scan. ConfirmationisdonebyaCT transitional cellcarcinomaoftherenalpelvis.Mosturicacidstonescontainsomecalciumand mimickinga upasafillingdefectonanIVU, Pure uricacidstonesareradiolucentandshow pyelotomy andballoondilatationarealsobeingdone. being carriedoutlaparoscopically.now Minimal-accesssurgicaltechniquessuchasendoscopic of renalfunction.TheoperationchoiceisAnderson–Hynespyeloplasty,aprocedurewhich thatthereismorethan20percent iscarriedoutwhenarenogramshows thekidney preserve is usefulonlywhentheresignificantrenalfunctionoftheobstructedkidney. Aprocedure to also determinethepercentagefunctionofkidney. isnottheidealimagingmethod.It IVU Isotope renographyisthebesttesttoestablishthatdilatationcausedbyobstruction.Itwould and staphylococciareurea-splitting organismsandformammonia,whichmakestheurine ➜ ➜ ➜ Fig. 7 5D 4C 3A 2B 1E matchingquestions Answers: Extended 14. B,C,D,E 13. B

obstruction –idiopathic orcongenitalhydronephrosis. dilatation withclubbing ofthecalyces.Theupperend oftheureterisnormal,indicating aPUJ fromtheright kidney. normalexcretion shows Ontheleftsidethereisgrosspelvicalyceal This IVU 2. Renalpatholog 1. Renalpatholog Renal neoplasms The commonestpresentationisanabdominalmassnoticedbythemotherwhenbathing the kidney andappropriatedefinitivetreatment. the kidney bynephrostomy.followed Indue course,sheshouldhaveisotoperenogramtoassessfunctionof andCECT. US Thisshouldbe urgent resuscitation,bloodcultures,appropriateantibiotics,IVU, pressure) andboundingpulse.Lointendernesspointstoadiagnosisof pyonephrosis.Sheneeds This womanhasallthefeaturesofsepticaemia:shockandhyperdynamic circulation(largepulse Anderson–Hynes operation. andpossible byurgentexploration followed andCECT IVU HeneedsanUS, extravasation. urinary obstruction whichhasrupturedwith a fityoungmanthiswouldbehydronephrosisfromPUJ minortraumaoccurstypicallyinapathologicalkidney. Haematuriafollowing loin discomfort. In completely unawareofthetraumaandsurprisedtohavehaematuria. He didnoticesomeleft minortraumawhileplaying rugby. haematuriafollowing This 20-year-oldmandeveloped Hewas instituted. oracombinationofboth shockwavelithotripsy(ESWL) extracorporeal nephrolithotomy (PCNL), scan shouldbedonetoconfirmthediagnosis,andappropriatetreatmentofpercutaneous andCT US urinewhichisconducivetotheformationofphosphatecalculi.AnIVU, alkaline ofurea-splittingorganisms stone.Thegrowth of akidney hassymptomssuggestive frequency This patientwithnaggingpaininherrightloinandurinary andachestX-rayradicalnephrectomy. CECT IVU, oflefttesticularvein.Heneedsan intotheleftrenalvein,obstructingentry extending growth haematuria, loinpainandclotcolicanenlargedkidney. Healsohasaleftvaricocelecausedby This malepatienthastheclassicalpresentationofalefthypernephroma(adenocarcinoma): treatment shouldbeinstituted. andappropriatedefinitive andIVU, indomethacin. ConfirmationofthediagnosisismadebyCECT This patienthastypicaluretericcolic.Heneedsimmediateanalgesiawithdiclofenacand interleukin-2hashadsomeencouragingoutcomes. no roleinadjuvanttreatment.Thecytokine Hypernephroma istwiceascommoninmenwomen.Chemotherapyandradiotherapyhave thus apoorprognosis. ofthetumourintorenalpelvisand child. Haematuriaisalatesymptomanddenotesextension 1.1, E 1.1, y y imaging Proteus andstaphylococciindicate 531

71: THE KIDNEYS AND URETERS GENITOURINARY ➜ Fig. 7 Fig.71.9, E Fig.71.8, B Fig.71.7, D Fig.71.6, C Fig.71.5, A Fig.71.4, C Fig.71.3, D Fig.71.2, B 532

ureter. Somephlebolithsarealsoseen. endoftheleft area offillingdefect.Thisistypicalatransitionalcellcarcinomathe lower anirregularfillingdefectdistaltothedilatationwithminimalcontrast goingthroughthe showing endoftheleft ureter phase.Thereisdilatationofthelower thecystographic shows This IVU 3. Ureteric patholog This is a plain X-ray showing atypicalstaghorncalculusintherightkidney.This isaplainX-rayshowing irregular fillingdefects–typicalofatransitionalcellcarcinomatherenal pelvis. ahugelydilatedrenalpelviscontaininglargenumberof This isanantegradepyelogramshowing transitional cellcarcinoma. into theleftureterandrenalpelvis.Thereisanirregularfillingdefectinpelvisdenotinga bladderanddyeinjected intheurinary thecystoscope This isaleftretrogradepyelogramshowing carcinoma oftheleftrenalpelvis. anirregularfillingdefectintheleftrenalpelvisclassicalofatransitionalcell shows This IVU nodes onbothsides. ureter. andthebladderisfilled.Therearecalcifiedlymph hascompletelyexcreted Therightkidney lefthydronephrosisandhydroureterproximaltoastoneinthemiddlethirdof shows This IVU suggestive ofaleftperirenalhaematomafromrenalinjury. contrastintheleftrenalarea.Thesefeaturesare andextravasated fromrightkidney excretion obliterated,good diaphragm, thestomachandcolonicgasbubblespushedup,leftpsoasshadow leftdomeof markedscoliosiswithconcavitytotheleft,elevated inachildshows This IVU normally. Thediagnosisislefthypernephroma. bladderisfilled ofcontrast.Theurinary withirregularexcretion there isahugesoft-tissueshadow withabifidrenalpelvis.Ontheleftside fromtherightkidney normalexcretion shows This IVU anabnormalityfoundin11000.horseshoe kidney, Thisismostoftendiscoveredincidentally. vase’appearanceofatypical therenalpelvisandcalycesfacingmedially,a‘flower shows This IVU 1.10, A y imaging 72 The urinary bladder

Multiple choice questions ➜ Anatomy ➜ Bladder stones 1. Which of the following anatomical 4. With regard to bladder stones which of statements are true? the following statements are false? A The bladder is lined by transitional A Men are more often affected than epithelium. women. B Hypertrophy of the detrusor muscle B An oxalate calculus develops in sterile results in bladder trabeculation. urine. C The epithelium of the trigone extends C A cystine calculus is not radio-opaque. into the lower ends of ureters and D Treatment is crushing with an optical proximal urethra. lithotrite in all cases. D The internal sphincter prevents urinary E In men with stones and outflow incontinence. obstruction from enlarged prostate, both E The distal urethral sphincter is supplied can be dealt with at the same time. by S2–S4 fibres via the pudendal nerves. ➜ Bladder diverticula ➜ Bladder injury 5. Which of the following are true about 2. In bladder trauma, which of the bladder diverticula? following statements are false? A They are diagnosed when they produce A Extraperitoneal rupture is more common double micturition and repeated urinary than intraperitoneal rupture. tract infections. B Extraperitoneal rupture mimics rupture of B All bladder diverticula must be excised. membranous urethra. C Trabeculation and sacculation precede C CT scan is the investigation of choice. diverticula formation. D Retrograde cystography is useful in D The normal intravesical pressure during

making a diagnosis. voiding is 35–50 cmH2O. E Laparotomy is required in all cases of E The majority occur in males. bladder rupture. F Haematuria is a symptom in 30 per cent. ➜ Detrusor instability ➜ Urinary fistulae 3. Which of the following statements 6. Which of the following statements about detrusor instability is false? about urinary fistulae is false? A 50 per cent of men with bladder outflow A Most urinary fistulae are vesicovaginal. obstruction have detrusor instability. B A combined ureterovaginal fistula may B In 50 per cent the detrusor instability be present in 10 per cent of patients. resolves after prostatectomy. C Examination under anaesthesia, C Patients with neurogenic bladder may intravenous urogram (IVU) and have detrusor instability. cystoscopy are necessary to evaluate D Genuine stress incontinence (GSI) is them. indistinguishable from detrusor instability. D When there are multiple tracts, the E Urodynamic studies are essential for causes may be radiation, malignancy or evaluation of detrusor instability. sepsis.

533 GENITOURINARY ➜ ➜ ➜ Extended matching questions Extended aretrueof Whichofthefollowing 8. tract infection(UTI)whichof Inurinary 7. Choose and match the following diagnoseswitheachofthe scenariosgivenbelow: Choose and matchthefollowing 534

1 E D C B A 1. Bladderpatholog D C B A Bladder cancer E D C B A Urinar E patches resemblingsand. he wasfoundtohavescatteredtuberclesandislandsofpale cystoscopy clinic. Onflexible noabnormality. revealed Clinical examination Hewasreferredtotheone-stophaematuria similarepisodeswhileathomeinEgypt. haematuria attheendofmicturition. Hehadafew withpainless attendedtheA&Edepartment A 35-year-oldmale,arecentvisitor from Egypt, Schistosomiasis ofbladder Tuberculosis ofthebladder Bladder carcinoma Bladder diverticulum Bladder stone is invaded. patients,themuscle ofnew In aquarter factors inplanningtreatment. areimportant III) classification I,II, classification andgrade(WHO Depth ofinvasion(T)inTNM symptom. Painful haematuriaisthemostsignificant are transitionalcellcarcinomas. bladdercancers 95 percentofprimary recurrent abacterialcystitis. insitumaypresentas Carcinoma lymphogenous. infection isusuallyhaematogenousor therouteof In tuberculouscystitis, Sterile pyuriaisasinisterfinding. essential. andimagingare rigors, cystoscopy In recurrentinfections,haematuriaand women. The conditionismorecommonin successful. drainage ofthebladderisusually managementbycatheter Conservative bladder cancer? statementsarefalse? the following y tractinfection y 10. In the treatment of bladder cancers, Inthetreatmentofbladdercancers, 10. Whataretheessentialinvestigationsin 9. E D C B A E E D C B A Urine cytology. forstaging –mandatory examination Cystourethroscopy andbimanual secondaries bone toexclude A bonescan–necessary for suspectedmuscle-invasivetumours –thisisnecessary tomography (CECT) Contrast-enhanced computed andUS IVU bladder cancer? do notinvademuscle. diagnosedpatients,70In newly percent true? statementsare which ofthefollowing muscle-invasive tumours. in chemotherapy improvessurvival Neoadjuvant cisplatin-based choice. treatmentof should betheprimary andlymphadenectomy cystectomy In muscle-invasivedisease,radical invasive disease. be thefirst-linetreatmentinmuscle- beamradiotherapyshould External managed byBCGimmunotherapy. diseaseisbest Grade 3superficial of mitomycininstillation. endoscopic resectionandasingledose tumoursaretreatedby Superficial ➜ Chooseandmatchthediagnoseswithimages(Figs72.1–72.5) below: shown

E D C B A 2. Bladderimaging 5 4 3 2 Bladder diverticulum Bladder stone ofthebladder Carcinoma Rupture ofthebladder (BPH) Benign prostatichypertrophy pyuria. noabnormality.sterile reveals carriedoutbyherdoctorshows examination Urine generalmalaiseandweightloss.Clinical pyrexia, Shehasevening and bilateralloindiscomfort. ofmicturition A 35-year-oldfemalepatient,arecentvisitorfromIndia,complainsoffrequency noabnormality. reveals Clinical examination hisstreambychangingposition. micturition streamsuddenlystops.Hehaslearnttore-start mancomplainsofhaematuria,painfulmicturitionandoccasionallyfindsthathis A 60-year-old noabnormality.reveals aftermicturitionheagainpassesalargeamountofurine.Clinicalexamination that shortly feeling ofinsufficientemptyinghisbladder. Hehassomedysuria.Onoccasionshefound A 70-year-old malecomplainsofpoormicturitionstream,post-micturitiondribblinganda abnormality. no reveals Clinicalexamination ofmicturitionandsomeretropubicdiscomfort. frequency periodic haematuriafor6weeksorso.Thebloodisuniformlymixedwiththeurine.Hehas years,complainsofpainless,profuseand smokerforover50 A 70-year-old male,aheavy Figure 72.1 535

72: THE URINARY BLADDER GENITOURINARY 536 Figure 72.3 Figure 72.2 Figure 72.4a Figure 72.4b 537

72: THE URINARY BLADDER GENITOURINARY ➜ ➜ ➜ ➜ 4. D 3. D 2. E 1. A,B,C,E Answers: Multiplechoicequestions 538

Bladder stones Detrusor instabilit Bladder injur Anatom optical lithotrite, the procedure is contraindicated in patients below 10optical lithotrite,the procedureiscontraindicatedinpatients below years. usingan bylitholapaxy While mostbladderstonescanbetreated byminimal-accesssurgery correctly diagnosedpatientwithGSI. can beproperlydiagnosedbyurodynamicstudies.Surgicaltreatmentwill onlybesuccessfulinthe aredifferentconditions that Butthey andmaycoexist. Idiopathic detrusorinstabilitycanmimicGSI rupture, catheterdrainageofthebladderfor10 days isthemanagement. Inextraperitoneal treatmentislaparotomyandclosureoftheperforation. In intraperitonealinjury, continence. retrograde ejaculationandhasnoroleinurinary The internalsphincteristhesmoothmusclearoundmalebladderneckwhichprevents y Figure 72.5 y y ➜ ➜ ➜ ➜ ➜ 1E matchingquestions Answers: Extended 10. A,B,D,E 9.A,B,D 8.A,C,D,E 7. D 6. E 5.C,D,E,F 3B 2C

1. Bladderpatholog Bladder cancer Urinar Urinar Bladder diverticula Bladder diverticula are most often detected incidentally on imaging (US or IVU) oroncystoscopy. orIVU) aremostoftendetectedincidentallyonimaging(US Bladder diverticula using (ELISA) immunosorbent assay of thelivershouldbedone.Antibodydetectionbyenzyme-linked bladder.This patienthasschistosomiasisoftheurinary Investigations todetermineinvolvement orthosewhodecline surgery. as anoptionshouldbeconsideredinthoseunfitformajorsurgery beamradiotherapy systemic chemotherapy. byabout5–7percent.External Thisimprovessurvival withlymphadenectomy.of radicalcystectomy Thisshouldbecombinedwithcisplatin-based intheform treatmentofchoiceshouldberadicalsurgery In muscle-invasivedisease,theprimary ishelpfulonlyifpositivebutanegativeresulthasnovalue. Urine cytology A bonescanisnotessential.Itonlydonewhenskeletalspreadstronglysuspectedclinically. spread. bladder cancer,itdenotesextravesical Painless, periodic,progressivegrosshaematuriaistypicalofbladdercancer. Ifpainoccursin oedema.Tubercles andsubmucosal the formofpallormucosa appearlater. lymphogenous spread.Therefore,changescommencearoundtheuretericorificesandtrigonein Tuberculous torenaltuberculosisandnotduehaematogenousor issecondary cystitis of diseasedtissueandtension-freevascularisedrepairinanatomicallayers. excision managementisrarelysuccessful.Theprinciplesofrepairaregoodexposure, Conservative required inthevastmajority. isallthat infections, stoneortumour. obstruction,TURP arecausedbybladderoutflow they As unlessthereisacomplicationsuchasrecurrent Most ofthemdonotneedtobeexcised micturition is typical of a bladder diverticulum which is the direct result of BOO. An US isdone AnUS which isthedirectresultofBOO. micturition istypical ofabladderdiverticulum ofdouble Thehistory obstruction(BOO). This manhasalltheclinicalfeatures ofbladderoutflow bytheappropriatetreatment. bimanual palpationforaccuratestaging followed and bycystourethroscopy followed CECT US, haematuria. Heneedsurgentassessment byIVU, This patienthasthetypicalsymptoms ofabladdercarcinoma–painless,profuseandperiodic by praziquanteliseffective. Schistosoma mansoni y y tractinfection fistulae

adult microsomalantibody(MAMA)iscarriedout.Medicaltreatment y 539

72: THE URINARY BLADDER GENITOURINARY 5D 4A ➜ Fig.72.5, A Fig.72.4a andb,C Fig.72.3, B Fig.72.2, E Fig.72.1, D 540

2. Bladderimaging team. Sheshouldbemanagedbyamultidisciplinary and tuberclesstuddedinthemucosa. X-ray. acontractedbladderwith‘golfhole’ureters whichmayshow bycystoscopy, Thisisfollowed andchest IVU The patientrequiresurineculture,fullhaematologicalandbiochemicalexamination, andthebladder.Bilateral loinpainandsterilepyuriawouldsuggesttuberculosisofthekidneys malaiseandweightloss. pyrexia, typicaloftuberculosis–evening This womanhasahistory andtreatmentbyopticallitholapaxy. cystoscopy by followed ofmicturitionstreamisatypicalfeature.ImagingshouldbedonebyUS cessation ofsudden This man’spainfulmicturitionandhaematuriaarefromabladderstone.Thehistory isdone,asmostdonotrequireremoval. diverticulum assessmentofthe byUS transurethral resectionofhisprostateiscarriedout.Follow-up noabnormality, Ifthisshows toseetheinsideofdiverticulum. should be,firstly,cystoscopy Thetreatment byurodynamicstudiestoassesshisBOO. to confirmthediagnosis,followed ureters show aclassical‘fishhook’deformity.ureters show Thepictureistypical of benignprostatichypertrophy. aregular,smoothfillingdefectinthebaseofbladder. shows This IVU endsofthe Thelower the leftureter,causinghydronephrosisandhydroureter. anirregularfillingdefectontheleftsideofbladderobstructing picturesshow These twoIVU rupture ofthebladder. catheterastreatment. PatienthasanindwellingFoley ofcontrastatthebladderneck–typicalextraperitoneal extravasation shows This cystogram ureteric orifice. ontheleft,closeto abladderdiverticulum showing phaseofanIVU This isacystographic alarge,dumbbell-shapedbladderstonelyingatthebase. This isaplainX-rayshowing 73 The prostate and seminal vesicles

Multiple choice questions ➜ Anatomy and physiology C The condition affects the transitional 1. Which of the following statements is zone and the central zone. false? D All lower urinary tract symptoms (LUTS) A The prostate is anatomically divided into in men >70 years are due to BPH. a peripheral zone, a central zone and a E The prostatic urethra is elongated. transitional zone. ➜ B The glands of the peripheral zone are Investigations lined by transitional epithelium. 4. In urodynamic studies, which of the C Benign prostatic hypertrophy occurs in following statements are true? the transitional zone. A A voided volume >200 mL with a peak D Most carcinomas arise in the peripheral flow rate >15 mL /s is normal. zone. B A voiding pressure of <60 cmH2O is E Denonvilliers’ fascia separates the normal. prostate from the rectum. C Increase in voiding pressures with a low flow rate is diagnostic of BOO. 2. Which of the following statements D In patients with a residual volume regarding androgenic hormones are of 250 mL or more, bilateral true? hydronephrosis will result. A Luteinising hormone (LH) from the E Urodynamically proven BOO anterior pituitary controls the secretion only occurs from benign prostatic of testosterone. hypertrophy. B 90 per cent of testosterone is secreted by Leydig cells from the testes. 5. In assessing LUTS in men, which of C The enzyme 5␣-reductase is found in the following statements are false? high concentration in the prostate. A Charting patient’s symptoms according D Adrenal androgens have a major effect to the International Prostate Symptom in the normal male. Score (IPSS) is routinely used. E 5␣-reductase converts testosterone B Flow rate measurement studies and into1,5-dihydrotestosterone (DHT). pressure-flow urodynamic studies are done. ➜ Benign prostatic hypertrophy C Thorough nervous system examination 3. In benign prostatic hypertrophy (BPH) is mandatory. which of the following statements are D The upper urinary tract should always false? be imaged. A It is the commonest cause of bladder E Transrectal ultrasound (US) is carried outflow obstruction (BOO) in men >70 out. years of age. F Cystourethroscopy is a good guide as B Decrease in serum testosterone levels an indication for surgery. and therefore relative increase in serum oestrogens cause BPH.

541 GENITOURINARY ➜ ➜ 8. Which of the following statementsare Whichofthefollowing 8. statements Whichofthefollowing 6. . Intheprinciplesoftreatment 7. 542

Prostate cancer causing BOO Management ofBPH E D C B A E D C B A E D C B A extremely reliable methodofscreening. extremely isan Prostate-specific antigen(PSA) peripheral zoneoftheprostate mostlyoriginatesinthe Carcinoma prostatecancer.develop can for benignprostatichypertrophy A patientwhohashadaprostatectomy positive familyhistory. prostate cancer,10–15 percenthavea In theyoungeragegroupwhodevelop over theageof65years. It isthemostcommoncancerinmen patients afterTURP. Retrograde ejaculationoccursinall improvement inthesymptomscores. resultsin75percent prostate (TURP) Transurethral resectionofthe improvement insymptomscores. Drug therapyresultsin20percent 5 muscle. the contractionofprostaticsmooth Alpha-adrenergic blockingagentsinhibit statements aretrue? BOO,whichofthefollowing causing of benignprostatichypertrophy are indicationsforsurgery. and aresidualvolumeof100–250 mL rateof<10 maximum flow A low mL /s 5 and /or Drug therapyiswithalpha-blockers cent ofprostatectomies. per symptomsaccountfor60 Severe cent ofprostatectomies. Chronic retentionaccountsfor15 per ultimately requireaprostatectomy. All patientswithacuteretention true in carcinoma oftheprostate? true incarcinoma BOOaretrue? causing regarding themanagementofBPH conversion oftestosteronetoDHT. ␣ -reductase inhibitorsinhibitthe ␣ -reductase inhibitor. 9. In carcinoma oftheprostate,which Incarcinoma 9. 12. Which of the following statements Whichofthefollowing 12. insuspectedprostate Assessment 11. Onrectal examination,whichofthe 10. E D C B A E D C B A E D C B A E D C B A the following statementsisfalse? the following cancer arealwaysosteolytic. Skeletal metastasesfromprostate for skeletalmetastasis. Prostate canceristhecommonestsite spread andprognosis. The Gleasonscorecorrelateswellwith degree ofglandulardifferentiation. Gleason scoringsystemisbasedonthe adenocarcinoma. The histologicaltypeisan is about2percent. incontinence afterradicalprostatectomy stress The incidenceofsevere by radicalprostatectomy. Incidentally diagnoseddiseaseistreated radiotherapy and /or androgenablation. T3 patientsaretreatedbyradical an alternativetoradicalprostatectomy. beamradiotherapyis external Radical for T1andT2disease(earlycancer). prostatectomyisonlysuitable Radical isfalse? cancer regarding treatmentofprostate IVU. function tests andliver General bloodtests,PSA Bone scan imaging Cross-sectional MRI (TRUS) Transrectal ultrasound-guidedbiopsy shouldinclude: cancer bladder base. thecapsuleinto beyond Extension overtheprostate Mobile rectalmucosa Irregular stonyhardinduration Obliteration ofthemediansulcus Nodules withintheprostate carcinoma? featuresdonotsuggest following ➜ ➜ 2.A,B,C, E 1. B Answers: Multiplechoicequestions matchingquestions Extended Chooseandmatchthediagnoseswithscenariosbelow:

Anatom 5 4 3 2 1 1. Clinicalconditionsoftheprostate play aminorrole, accountingfor5–10 percentoftestosterone. phenomenon utilised forandrogendeprivationtherapy inprostatecancer. Adrenal androgens manner,a half-lifeandisreleasedinapulsatile hasashort the outputoftestosterone.LHRH whichinturncontrols the hypothalamuscontrolssecretion ofLHfromtheanteriorpituitary, secretedby more potentthantestosterone.Luteinising hormone-releasinghormone(LHRH) Testosterone 5 bytheprostaticenzyme into DHT isconverted from thereisanadenocarcinoma. The glandsoftheperipheralzonearelinedbycolumnarepitheliumand henceacancerarising E D C B A type 2diabeticandisonmedication. dribbling.Themassisdull onpercussion.Heisa from thepelvisandhascontinuousurinary hehasalargepainlessmassinhissubumbilical regionarising incontinence. Onexamination abdominal distensionandurinary A 75-year-oldmalecomplainsofgeneralmalaise,lethargy, grooveiseasilyfelt. andthemedianvertical prostate withoverlyingmobilerectalmucosa noabnormality. reveals hehasanenlargedsmooth Clinical examination Onrectal dribbling. Thesesymptomshavebeengoingonfor2yearsandaregraduallygettingworse. intermittentstreamandpost-micturition of incompleteemptying.Healsohaspoorflow, ofmicturitionwithafeeling andhesitancy frequency A 70-year-old mancomplainsofurgency, pelvis. Themassisdulltopercussion. painfromalargemassinhissubumbilicalregionarising heisinsevere examination he boughtacrossthecounterforcoldandflu.Thishasalsomadehimconstipated.On abdomen nothavingpassedurineforalmost10 h.Hehasbeenonsomemedicineswhich acutepaininhislower withsevere A 72-year-oldmanhascometotheA&Edepartment hehasageneralisedhard,nodularprostatewithoverlyingfixedrectalmucosa. noabnormalityisfound.On localisedtothesmallofhisback.Onclinicalexamination, backache havebeenthereforabout4months.Morerecentlyhehashad These symptomsofBOO intermittentstream,afeelingofincompleteemptyingandterminal haematuria. night, hesitancy, ofmicturitionbothinthedayandat A 70-year-old malecomplainsofpoorstream,frequency atender,swollenprostate. reveals examination Hehaspainonmicturitionandpassesthreadsinhisurine. Rectal and perinealdiscomfort. all overandhasflu-likesymptomsingeneral.Herectalirritation,painondefaecation of39°C.Hehasrigorswithaches malecomplainsoffeelingunwellwithpyrexia A 40-year-old Acute prostatitis Acute retentionofurine ofprostate Carcinoma retentionwithoverflow Chronic urinary causingBOO Benign prostatichypertrophy y andph y siolog y ␣ -reductase. DHT isfivetimes -reductase. DHT 543

73: THE PROSTATE AND SEMINAL VESICLES GENITOURINARY ➜ ➜ ➜ ➜ ➜ 10. D 9. E 8.A,B,C,D 7. A,B,C,D 6.B,C,D,E 5.D,E,F 4.A,B,C,D 3. D 544

have fewer side-effects. After TURP, retrogradeejaculationoccursin65percent ofpatients. side-effects.AfterTURP, have fewer g,needtobetakenforatleast6monthsand better inpatientswithadenomaslargerthan50 Prostate cancer Management ofBOO Management ofBPHcausingBOO Investigations Benign prostatic h Lower urinary tract symptoms increase in both sexes withage.Therefore,inoldermen,such tractsymptomsincreaseinbothsexes urinary Lower mucosa willbeadherenttotheprostate. mucosa overtheprostateis anormalfinding.Inprostatecarcinomatherectal Mobile rectalmucosa lumbarvertebrae. being thepelvisandlower Skeletal metastasesfromprostatecancer aremostlyosteosclerotic,themostfrequentlyinvolved screening programmesshouldbeestablished. for prostatecanceriscarriedoutwithinclinicaltrialsanditremainsunclear whethernational Population-based screening per centofpatientswithprovenprostatecancerwillhavenormalPSA. willhaveproven prostatecancer. percentofmenwitharaisedPSA Twentyis 2–4percent.Thirty asascreeningprocedure.Detection ofcancerusingPSA There isnoconsensusaboutuseofPSA oftheprostategland.Alpha-blockersactmorequickly.cent shrinkage 5 residualvolumedonotrequireanoperation.Drugtherapyoverayearproduces25per with low rates istheassessmentofsymptoms.Moreover,goodflow thekey In counsellingmenforTURP, provided renalfunctionhasbeenstabilisedbycatheterisation. istrueofpatientswithchronicretentionurine, management issometimessuccessful.Thesame bytrialwithoutcatheter. drugtherapyisusedforawhilefollowed after initialcatheterisation, Such been found,e.g.constipation,drugs,recentoperation,needaprostatectomy. Insomepatients Only 25percentoffit,malepatientswithacuteretention,wherenocausefortheretentionhas andtheneedforsurgery.Cystourethroscopy doesnotgiveanyguideastothedegreeofBOO Transrectal acarcinomaissuspected. scanningisnotroutineunless,onrectalexamination, US ofhaematuriaorifinfectionispresent. routinely carriedout.Itisonlydoneifthereahistory isnot orIVU uppertractimagingbyUS asaresultofBOO, In theassessmentofLUTS urethral strictureandfunctionalobstructionfromneuropathicconditionsareothercauses. prostate cancer, bladderneckstenosis,hypertrophy, benign prostatichypertrophy, provenbyurodynamicstudies.Besides isnottheonlycauseofBOO Benign prostatichypertrophy overactivity orneuropathicbladderdysfunction. symptoms arenotalwaysfrombenignprostatichypertrophy. Itcanbeduetoidiopathicdetrusor y pertroph y ␣ -reductase inhibitorswork ➜ 5B 4A 3D 2C 1E questions matching Extended Answers: 12. D 11. A,B,C,D

1. Clinicalconditionsoftheprostate particularly useful when the PSA is>10 usefulwhenthePSA particularly nmol high-gradecancer./mL orthebiopsyshows is IVU isthemostaccuratemethodoflocalstaging.Bonescan biopsy cores.Cross-sectionalMRI Transrectal ultrasound-guidedbiopsyisusedtoconfirmthediagnosisbytaking10 systematic assessed. Heprobablyhasdiabetic neuropathy causingthechronicretention. urodynamicstudies shouldbecarriedoutandhisdiabetes adequate resuscitation,pressure-flow failure. Hewouldhavepost-obstructive diuresiswhichwouldrequireintravenoushydration.After backpressureandtreathisrenal torelieve would beinfected.Heneedsimmediate catheterisation andhisurine metabolicacidosiswithhyperkalaemia wouldreveal anaemia, hisbiochemistry with overflow. Heisunawareofhishugelydistendedbladder. wouldshow Hishaematology This diabeticelderlymalehasfeaturesofchronicrenalfailurewith retentionofurine urodynamic studies.Heshouldthenbeconsideredfordrugtherapyinitially.and pressure-flow ratemeasurement byflow haematologicalandbiochemicaltests,followed HeneedsPSA, the IPSS. benignprostatichypertrophy. shows Hissymptomsneedtobe assessedaccordingto Examination Hehasbothirritativeand obstructivesymptoms. fromBOO. This patienthasfeaturesofLUTS residualvolumeonUS. tohavealow ifheisshown regime willbesuccessful,particularly a trialwithoutcatheter. Aperiodofdrugtreatmentmayprecederemovalthecatheter. This hewillbenefitfrom islow, IfhisIPSS isdonebytheIPSS. ofhisLUTS be treated.Assessment Hisconstipationshould resultant constipation.Heneedsanalgesiawithimmediatecatheterisation. broughtonbythedrugsand the This patienthasacuteretentionofurinealmostcertainly thorough assessment. bybonescan.Appropriatetreatmentisinstitutedafter followed orMRI local stagingbyCT generalbloodtests,confirmationofthediagnosisbyTRUS, prostate carcinoma.HeneedsPSA, atypical shows Rectalexamination durationfromBOO. ofrelativelyshort This patienthasLUTS the abscesscavity. fluctuant prostate.Thetreatmentofaprostaticabscessisperiurethralresectionandderoofing anenlarged,hot,tender perineal andrectalpainwithtenesmusand,onexamination, byhightemperaturewithrigors,severe aprostaticabscessasshown these patientsmaydevelop Ifnotpromptlytreated, withtrimethoprimorciprofloxacin. is culturedandtreatmentstarted This youngmalepatienthasthetypicalfeaturesofacuteprostatitis.Heneedsanalgesia;urine For metastaticdisease,androgenablationiseffectiveinthemajorityofpatients. estimation. andPSA (DRE) withregulardigitalrectal examination managed byactivesurveillance is forclinicallydiagnosedbenignprostatichypertrophy Incidentally discovereddiseaseatTURP not necessary. 545

73: THE PROSTATE AND SEMINAL VESICLES 74 The urethra and penis

Multiple choice questions ➜ Congenital abnormalities D The prostate may be high-riding and out 1. Which of the following statements is of reach on rectal examination. false? E A urethral catheter is inserted as a part of A The verumontanum marks the proximal initial resuscitation. end of the external urethral sphincter. 4. In superficial extravasation of urine, B Posterior urethral valves causing which of the following statements are obstruction can be diagnosed antenatally. true? C The incidence of hypospadias is 1 in A It occurs in ruptured bulbous urethra. 200–300. B Extravasated urine is confined by the D Hypospadias is the commonest Denonvilliers’ fascia. congenital urethral anomaly. C Extravasated urine collects around the E Glandular hypospadias should be scrotum and penis. surgically treated early in life. D A urethrogram is carried out to confirm ➜ the diagnosis. Urethral rupture E Suprapubic cystostomy is the immediate 2. In bulbous urethral rupture, which of treatment. the following statements are true? A It is caused by direct blow to perineum. ➜ Urethral stricture B Acute retention occurs. 5. Causes of urethral stricture include: C There is perineal haematoma with blood A Inflammation and infection at the urethral meatus. B Trauma D In a full bladder, suprapubic C Neoplasm catheterisation is carried out. D Postoperative E Urethral catheterisation may be E Congenital. attempted as an alternative. 6. Which of the following statements 3. In rupture of the membranous urethra, about urethral stricture is false? which of the following statements is A The symptoms are those of bladder false? outflow obstruction (BOO). A It is almost always associated with a B Ascending urethrography is necessary. pelvic fracture. C Urethroscopy is mandatory to evaluate. B 10–15 per cent of fractured pelvis cases D Periurethral abscess is a complication. will have associated urethral injury. E Urethral dilatation is the ideal treatment. C It is usually a part of multiple trauma. Extended matching questions ➜ 1. Conditions of the penis A Balanoposthitis B Peyronie’s disease

546 ➜ ➜ 4.A,C,E 3. E 2.A,B,C,D 1. E questions choice Multiple Answers: Chooseandmatchthecorrectdiagnoseswithscenariosbelow:

Urethral rupture Congenital abnormalities 5 4 3 2 1 E D C and thighstodrain theurine. andincisionsoverthescrotum necessary. Thepatientistreated byurgentsuprapubiccystostomy fasciaoftheabdominal wall.Thediagnosisissoobviousthaturethrogramnot the superficial the inguinalligament.Thusurinecollects inthescrotumandpenisbeneathdeeplayerof Colles’ fasciatothetriangularligament andbytheattachmentofScarpa’sfasciajustbelow urineisconfinedin frontofthemidperinealpointbyattachment The extravasated contrast maybecarriedout. is theinitialtreatmentofchoice.Incasedoubt,anascendingurethrogram withwater-soluble Asuprapubiccystostomy ofaurethralinjury. Urethralcathetershouldnot beinserted. otherwise The plainX-rayofthepelvisinacasemultipletraumawilloftengive anideaofthepresenceor tear intoacompletetear. iscarried out withfullassessmentlater. Suprapubiccystostomy apartial rarelyadvocatedbysome,isbestavoidedforfearof converting Urethral catheterisation, meatusandperinealhaematoma. retention,bloodattheexternal The clinicaltriadisacuteurinary totheperineum. accidents,aretheusualcausesofdirect blow by afallacrossbeamorcycling accidentscaused A fallastrideaprojectingobject,suchasmayoccurinscaffolders,gymnasium case ameatotomyisperformed. Glandular hypospadiasdoesnotneedsurgicaltreatmentunlessthemeatusisstenosed,inwhich inflamed. HeisatypeIdiabeticoninsulin. purulentdischargefromthesubprepucialarea.Hisglansandforeskinlookred developed A 22-year-oldmalecomplainsofitchingaroundhisglanspenisandprepuce.Recentlyhehas distressinghim. erection withoutanyreason.Thisisprolongedandpainfulnow penile developing recentlystarted malewhoisundertreatmentforleukaemia A 60-year-old on bothhands. can bepalpatedaroundthepenileshaft.Onquestioninghehasthickeningofhispalmarfascia induratedplaques during erection;thishasbeengoingonforthepast2years.Onexamination, pronounced A 45-year-oldmalecomplainsofprogressivedeformityhispeniswhichisvery ulcerated lesionunderhisprepuce.Healsocomplainsoflumpsingroin. A 38-year-oldmalecomplainsofbloody,foul-smellingdischargefromhispenis.Hehasahard antibiotics. intercourse.Inthepasthehadurethritis,whichwastreatedsuccessfullywith difficulty insexual A 35-year-oldmalecomplainsofdeformityhispenisduringerection.Hehasincreasing Chordee ofpenis Carcinoma Persistent priapism 547

74: THE URETHRA AND PENIS GENITOURINARY ➜ ➜ 5A 4C 3B 2D 1E questions matching Extended Answers: 6. E 5.A,B,D,E 548

1. Conditionsofthepenis Urethral stricture Gonococcal urethritisandnon-specificcausessuchaschlamydialinfectioncanresultin to localhygiene. should makesurethathisdiabetesisundergoodcontrol.Hebe givenadvicewithregard This youngmanhasbalanoposthitisandneedstobetreatedwiththe appropriate antibiotics.One injection ofadrenalinesolution. or cell disease.Hecanbetreatedbyaspirationofthesludgedbloodin corporacavernosa haspersistentpriapism.Itcanalso occurduetosickle This patient,whosuffersfromleukaemia, function. withsexual maybeindicatedwhentheconditioninterferes surgery upcalcification.Theconditionisself-limitingbut in thiscase.AplainX-rayofthepenismayshow contracture,as disease,whichisoftenaccompaniedbyDupuytren’s This patienthasPeyronie’s bytheappropriatedefinitivetreatment. followed nodes, liverultrasoundandchestX-ray. meetingis Thereafter,discussioninamultidisciplinary enlargement isfromsepsis.Heneedsabiopsyofthelesion,fine-needleaspirationhisgroin percentofpatientstheinguinallymphnode staging ofthediseaseanddefinitivetreatment.In50 confirmationofdiagnosis, The usualprinciplesofcancermanagementshouldbefollowed: This patienthasacarcinomaofthepeniswithprobablesecondariesinhisinguinallymphnodes. hypospadias, inthispatientitistheaftermathofchronicurethritis. ofhispenis.Whileitcanbetheresult This patientsuffersfromchordee,whichisafixedbowing complicated post-traumaticstricturesafterpelvictrauma,urethroplastyisthetreatmentofchoice. percentofsimplestrictures.In treatment isinternalvisualurethrotomy,whichcures50 inelderlymen.Theideal Urethral dilatationasaformoftreatmentisnotoftenused,except urethral duplication;symptomsoccurinadolescence. rareandoccursfrom Congenitalstrictureisvery of thepenisandprolongedcatheterisation. in strictureduecourse.Postoperative causesareafterprostatectomy,urethroscopy,amputation result totheperineumcausingbulbousurethralrupture,inevitably membranous urethraorblow trauma,suchasfractureofthepelvisproducingrupture stricture.External inflammatory 75 The testis and scrotum

Multiple choice questions ➜ Incompletely descended ➜ Hydrocele testis 4. The following statements are true 1. In incompletely descended testis, about a hydrocele except which of the following statements is A A hydrocele is a collection of fluid within false? the tunica vaginalis. A The incidence is 4 per cent. B A congenital hydrocele causes an B A testis absent from the scrotum after intermittent swelling. 3 months is unlikely to descend. C An acute hydrocele in a young man may C An incompletely descended testis tends be a sinister finding. to atrophy as puberty approaches. D Testicular pathology may cause a D Early orchidopexy can preserve function. hydrocele. E Orchidopexy reduces the chances of E Drainage is an effective treatment. developing a testicular tumour. ➜ Epididymo-orchitis 2. The hazards of incomplete descent of 5. In epididymo-orchitis, which of the testis include: following are true? A Sterility in bilateral cases A The causes are chlamydia and gonococci B Trauma as sexually transmitted infections. C Torsion B The initial symptoms are those of urinary D Increased liability to malignant change in tract infection (UTI). later life C In mumps 18 per cent of men may E Epididymo-orchitis. develop the condition. ➜ D It is a common postoperative Testicular torsion complication after prostatectomy. 3. Which of the following statements E Treatment is doxycycline for chlamydia or are true with regard to testicular a broad-spectrum antibiotic. torsion? A The typical symptom is sudden agonising ➜ Testicular tumour pain in the scrotum. 6. In testicular tumours, which of the B It is most common between 10 and 25 following statements are false? years of age. A A scrotal lump that is inseparable from C Inversion of the testis and a transverse lie the testis is likely to be a tumour. are the commonest cause. B Lymphatic spread is usually to the D Doppler ultrasound (US) scan should be inguinal lymph nodes. done to confirm the diagnosis. C Teratomas occur in the third decade and E The emergency operation of fixation of seminomas in the fourth decade. the testis to the tunica albuginea with D Seminomas usually spread via the non-absorbable sutures should be done lymphatics. on both sides.

549 E Diagnosis is confirmed by US-guided B Initial surgical treatment is orchidectomy fine-needle aspiration cytology (FNAC). through the groin. C Staging is done by CT scan and MRI. 7. Which of the following statements are D Seminomas are radiosensitive. true with regard to the management of E Secondaries from teratoma are always testicular tumours? treated by surgery. GENITOURINARY A Tumour markers are measured and a chest X-ray carried out. Extended matching questions ➜ 1. Scrotal and testicular conditions A Vaginal hydrocele B Epididymal cyst C Spermatocele D Testicular tumour E Acute epididymo-orchitis F Torsion of the testis G Torsion of the hydatid of Morgagni H Varicocele I Idiopathic oedema of the scrotum J Encysted hydrocele of the cord Choose and match the correct diagnoses with the clinical scenarios below: 1 A 22-year-old male complains of acute pain in the top of his left testis for 16 h. On examination the scrotum looks normal and the testis can be felt with care without undue tenderness. There is a small blue pea-sized tender mass on top of the testis. 2 A 30-year-old male complains of a swollen scrotum for almost a year. This is not painful but recently has caused discomfort. On examination he has a tense fluctuant lump in his right scrotum where the testis cannot be felt separately. The swelling transilluminates. 3 A 45-year-old man complains of a painful left testis of 4 days’ duration. This is associated with fever and rigors. He has painful micturition and considerable frequency. On examination he looks unwell, has pyrexia of 39°C and has a swollen, red, oedematous and shiny left hemiscrotum; the testis and epididymis feel indurated and tender. 4 An 18-year-old male complains of sudden onset of agonising pain in his right groin and suprapubic area of 6 h duration. He has no urinary symptoms. On examination he is in agony, the right testis is drawn up and the scrotum is red. The testis is impossible to feel because of pain. 5 A 25-year-old male complains of feeling heavy in his left scrotum where he noticed a lump while in the shower. A few days before he felt a lump in the periumbilical region of his abdomen. At the same time he noticed another lump on the left side of his neck. The abdominal and neck lumps do not give him any symptoms. On questioning he admits to coughing up some blood in his sputum a month ago. On examination he has a 2 cm irregular lump over his left supraclavicular area and a 5 cm irregular firm lump in his umbilical region. Examination of the left testis reveals a large lump which is hard and heavy. 6 A 50-year-old man complains of a lump in his right scrotum of a year’s duration. This is gradually growing larger in size and gives him some discomfort. Examination of his scrotum

550 ➜ ➜ ➜ 4. E 3.B,C,E 2.A,B,C,D 1. E Answers: multiplechoicequestions

10 10 H Testicular torsion Incompletel 9 8 7 condition inevitably recursandmaycause infection. condition inevitably man mayheraldthe onsetofatesticulartumour. Drainageisnotaneffectivetreatment asthe intotheabdomencausing swellingtodisappear.disappears Anacutehydroceleinayoung the peritonealcavity. thefluid Hence itcansometimesbeemptiedandwhenthechildliesdown A congenitalhydroceleisduetoapatent processusvaginalisandisthereforecontinuouswith may belost.Incaseofdoubt,thescrotumshouldexplored. lackofbloodsupplybutitshouldnotbedoneroutinely asprecioustime mayshow Doppler US tunica vaginalisandseparationoftheepididymisfrombody testisaretheusualcauses. testis. Anatomicalabnormalitiessuchasinversionandtransverselie,high investmentofthe The typicalsymptomissuddenagonisingpaininthegroinandsuprapubic areaandnotthe descended testisontherightside,itcanbeconfusedwithacuteappendicitis. shoulditoccurinanincompletely There isnoincreasedriskofepididymo-orchitis.However, size, weightandfeelofatestismuchmoreeasilywhenitisinthescrotum. enhances thechancesofearlydetection.Thisisbecauseyoungmenwillnoticechangesin doesnotreducetheincidenceofatumour butgreatly Correcting theabnormalitybyorchidopexy atumourinlaterlife. An incompletelydescendedtestishasamuchgreaterchanceofdeveloping y Abdominal examination in the left renal area does not show anyabnormality. intheleftrenalareadoesnotshow Abdominal examination theveinsdisappear.bunch ofveinsinthescrotumandtestisfeelsnormal.Onlyingdown hehasa inthelump.Onexamination for manyyearsbutonlyrecentlynoticeddiscomfort noticed anirregularlumpinhisscrotumseparatefromthetestis.Hethinkshehashadthis intheleftsideofhisscrotum.He A 45-year-oldmalecomplainsofadraggingdiscomfort testis. ontractiontothe 2 cmindiameterandabovethetestis.Ittransilluminatescomesdown thetestisfeelseparatefromswellingwhichisabout chance inthebath.Onexamination A 30-year-oldmanpresentswithaswellinginhislefthemiscrotumwhichhediscoveredby and doesnottransilluminatewell. swellingontopofhisrighttestis.Itisnottender hehasalax,cystic symptoms. Onexamination andisworriedthathehasgotcancer.testis’. Hehasjustnoticeditintheshower Hehasno feelslike‘athird malepresentswithaswellinginhisrightscrotumwhichhesays A 40-year-old the penisandperineum. thechildisnotinanypainandthererednessoedemaaround scrotum. Onexamination withared,swollen A 4-year-oldboyhasbeenbroughtbyhisparentstotheA&Edepartment Chinese lantern. minimally tender. Transillumination septawithinthelumplookinglikea ispositiveandshows lump,about7cmindiameter,aboveandbehindthelefttestis,whichis acystic reveals drocele y descendedtestis 551

75: THE TESTIS AND SCROTUM GENITOURINARY ➜ ➜ ➜ 5D 4F 3E 2A 1G questions matching Extended Answers: 7. A,B,C,D 6.B,E 5.A,B,C,E 552

following chemotherapy.following lymphnodalmassesarestill present lymph nodedissectionisonlycarriedoutifsecondary monitored whilststages2–4aretreatedbychemotherapy. intheform ofretroperitoneal Surgery treated bychemotherapy. Secondariesfromteratomaarenotalwaystreatedbysurgery. Stage1is radiotherapy;laterstagesare resultsfollowing of seminoma(stages1and2)haveexcellent Treatmentdehydrogenase (LDH). isaccordingtothestage,ofwhichtherearefour. Earlystages 1. Scrotal andtesticularconditions Testicular tumour Epidid Initial symptoms are dysuria and frequency followed bytender,red,swollenepididymisandtestis. followed Initial symptomsaredysuriaandfrequency abdominal andsupraclavicular lumpsfrommetastatic lymphnodesandhaemoptysisfrom This 25-year-oldman hasthehallmarksofametastatic testicularteratoma–hardenlarged testis, absorbable sutures. a scrotalincisionandthetestisshould befixedonbothsidestothetunicaalbugineawithnon- through abdomenandthetestis ispulledup.Thepatientshouldbeexplored the groinandlower This 18-year-old typicallyin itstarts male hastesticulartorsion.Painissuddenandexcruciating, treated withtheappropriateantibiotics.Itmaytake6–8weekstoresolve. arecarried out.Thepatientis epididymo-orchitis. BloodandurineculturestesticularUS tractinfectionwithanenlarged,tenderlefttestissuggestiveofacute This patienthasurinary treatment isbyoperation. and isbyUS not separatelypalpableandtheswellingtransilluminates.Confirmation, ifnecessary, swellinginhisscrotum wherethetestisis This patienthasavaginalhydrocele–tensecystic hydatidofMorgagni.Removalthelumpcurescondition. torted tender butheisoverasmallswellingatthetopoftestis.Thebluelump thetestisisnot This 22-year-oldmancomplainsofacutepaininhisscrotum.Onexamination markers –beta-humanchorionicgonadotrophin( tomeasuretumour Once testiculartumourisdiagnosedonclinicalgrounds,itmandatory orchidectomy shouldbearranged. along theneedletrack.Incaseofanydoubtaboutdiagnosis,frozensectionduringinguinal bedoneasitwoulddisseminatetumourcells shouldnever multiple tumourreflections.FNAC wherethehomogenoustumourtissueproduces which israre.DiagnosisconfirmedbyUS blood vessels.Metastasistoinguinallymphnodesdonotoccurunlessthescrotalskinisinvolved, groupoflymphnodesalongsidethe The lymphaticdrainageofthetestisistopara-aortic rarebecauseofclosedcatheterdrainageandprophylacticantibiotics. epididymitis isvery Post-prostatectomy urethritisorUTI. Infection spreadstotheepididymisviavasfromprimary y mo-orchitis ␤ -HCG), alpha-fetoprotein( -HCG), ␣ -FP) andlactate -FP) 10H 9J 8C 7I 6B testicular veincausingthevaricocele. atumourthrombusmayspreadalongtheleftrenalvein,blocking carcinoma. Rarely aleftrenal toexclude itisontheleftside,important This patienthasaleftvaricocele.As above thetestis.Traction onthetestisbringsswellingdown. swellingseparatefromandsituated hydroceleofthecord–acystic This manhasanencysted presence ofspermatozoa. behind thetestiswhichdoesnottransilluminatewellbecausefluidisopalescentdueto swellingaboveand This patientwhocomplainsofa‘thirdtesticle’hasspermatocele–laxcystic not requireanyactivetreatment. This 4-year-oldboyhasidiopathicoedemaofthescrotum.isaself-limitingdiseaseanddoes excision. and behindthetestis.Transillumination aChineselanternappearance.Treatment shows isby swellingseparatefromthetestiswhichliesabove –acystic This patienthasanepididymalcyst meetingandappropriatetreatment. bydiscussioninamultidisciplinary followed ofchest CT andMRI, chestX-ray,highinguinalorchidectomy,stagingbyabdominalCT US, secondaries.Heneedsbloodstobesentfortumourmarkers,testicularandabdominal pulmonary 553

75: THE TESTIS AND SCROTUM 76 Gynaecology

Multiple choice questions ➜ Anatomy and physiology B In the non-pregnant state it can occur 1. Which of the following are true from an intrauterine contraceptive device anatomical facts? (IUCD). A The cervix divides the vault of the vagina C All non-pregnant women >40 years of into anterior, posterior and lateral fornices. age should have an ultrasound (US) and B The pouch of Douglas is the area endometrial biopsy. between the uterus anteriorly and the D Women on tamoxifen have a higher rectum posteriorly. incidence of vaginal bleeding as they C The uterus is normally anteverted. have a greater risk of developing D The fallopian tube is 10 cm long and has endometrial cancer. four parts. E Women with hereditary non-polyposis E The uterine arteries arise from the colorectal cancer (HNPCC) have a higher abdominal aorta. incidence of endometrial cancer and vaginal bleeding. 2. Which of the following statements F All women should undergo endometrial regarding reproductive physiology is biopsy. false? A The hypothalamus produces ➜ Ectopic pregnancy gonadotrophin-releasing hormone 4. Which of the following statements (GRH). about ectopic pregnancy is false? B GRH stimulates the pituitary to release A Lower abdominal pain with vaginal follicle-stimulating hormone (FSH) and bleeding in early pregnancy should luteinising hormone (LH). alert one to ectopic pregnancy unless C FSH and LH control the production of otherwise proven. oestrogen and progesterone. B Transvaginal US showing absence of D The proliferative or follicular phase (first intrauterine gestational sac and a positive half) of the menstrual cycle is controlled urinary pregnancy test points to ectopic by oestrogen and the second half is pregnancy. controlled by progesterone. C Levels of beta-human chorionic E Gestational age is best calculated by the gonadotrophin (␤-HCG) are a useful date of the last menstrual period (LMP). guide. ➜ D Laparoscopy is the best diagnostic test. Abnormal /excessive vaginal E Salpingectomy is the treatment of choice. bleeding 5. Which of the following are risk factors 3. In vaginal bleeding, which of the for ectopic pregnancy? following statements are true? A Past pelvic inflammatory disease (PID) A Painless vaginal bleeding in pregnancy B Past history of spontaneous denotes a threatened miscarriage. miscarriage

554 ➜ ➜ Extended matchingquestions Extended statements Whichofthefollowing 6. Chooseandmatchthecorrectdiagnoseswitheachofscenariosbelow:

4 3 2 1 F E D C B A 1. G C B A Pelvic inflammator E D C abdomen. Vaginal examination shows foul-smelling cervical discharge. foul-smelling cervical shows abdomen. Vaginal examination andistenderoverherlower shelooksindiscomfort atpresent.Onexamination inserted IUCD abdominalpainandfever. ofmicturition and intermittentlower She hasfrequency Shehasan months’duration. A 27-year-oldwomancomplainsof foul-smellingvaginaldischargeofseveral Douglas. tendernessinthefornices andirregularnodulesarefeltinthepouchof show examinations abdomen,andvaginal andrectal sheistenderinherlower chronic fatigue.Onexamination from mid-menstrualpain.Shehasnotconceivedasyet,althoughshe would liketo.Shehas abdomen.She alsosuffers chronicpaindeepinthelower months. Recentlyshehasdeveloped dysmenorrhoea forseveral In theoutpatientclinic,a38-year-oldwomancomplainsofsevere andtheos isclosed. excitation thereiscervical abdomen. Onvaginalexamination mmHg,andpulseis120/min; tenderovertheentirelower pressure is80/50 sheisextremely looks pale,herblood sheisapyrexial, toconceiveforthefirsttime.Onexamination trying abdominal painof6hduration.Sheisunsureaboutherlastmenstrual periodandhasbeen right-sided lower A 35-year-oldwomanhasbeenadmittedwithsuddenonsetofsevere abdomen. rebound tendernessovertheentirelower 110/60 mmHgandapulseof120/min. tenderness,rigidityand reveals Abdominalexamination sweaty,withabloodpressureof settled toacontinuousagonisingpain.Sheisapyrexial, now withbuthas tostart 4 hduration.Thepaingraduallyspreadtotheleftside.Itwascolicky abdominalpainof right-sidedlower A 25-year-oldwomanhasbeenadmittedwithsevere disease(PID) Pelvic inflammatory ofovary Carcinoma cyst Ovarian Endometriosis Uterine fibroids Ectopic pregnancy should beadopted. threshold forempiricaltreatment A low organism. Streptococcus transmitted ascendinginfection. The majorityarecausedbysexually Past abdominaloperation. ofinfertility History medicaltermination Previous (PID) isfalse? (PID) disease about pelvicinflammatory y naecological patholog isthemostcommon y disease y 7. Which of the following patientswith Whichofthefollowing 7. E D C B A E D PID inpregnancy PID Tubo-ovarian abscess symptoms diseasewithconstitutional Severe appendicitis, cannotbeexcluded e.g.acute A surgicalemergency, needtobeadmitted? suspected PID for treatment. Some patientsmayneedtobeadmitted suspected. soonafterthediagnosisis be started Outpatient antibiotictreatmentshould Fitz-Hugh–Curtis syndrome. Fitz-Hugh–Curtis 555

76: GYNAECOLOGY GENITOURINARY ➜ ➜ ➜ ➜ 6. B 5.A,B,C,D 4. E 3.A,B,C,D,E 2. 1. A,B,C,D E Answers: Multiplechoicequestions 7. A,B,C,D,E 556

Pelvic inflammator Ectopic pregnanc Vaginal bleeding Anatom 6 5 common organismis Chlamydia trachomatis abdominaloperationdoesnotincreasethechanceofectopicpregnancy.A previous surgery. Thiscanbecarriedoutbythelaparoscopicroute. conservative following ratesarehigherandrecurrentectopiclower subsequent intrauterinepregnancy as itisthoughtthat maybeperformed Salpingostomy isnotalwaysperformed. Salpingectomy tomeasureendometrialthicknessandbiopsywhenappropriate. transvaginal US shouldbescreenedannuallyfromtheageof35yearsby who havebeentreatedforHNPCC israrelyfoundandthereforemaybetreatedsymptomatically.any riskfactors,pathology Women andendometrialbiopsy. yearswithout patient shouldundergoUS theageof40 Inwomenbelow ofvaginalbleedingasitmayheraldtheonsetendometrialcancer.make theclinicianwary The oftheconditionshould orafamilyhistory ofbeingontamoxifenorhavinghadHNPCC History Vaginal bleedingcanbeasinistersymptomandthereforeneedstothoroughlyinvestigated. use ultrasoundmeasurementsoffetalsize. maybewrong.Therefore,thealternativepracticeisto The woman’sdateofrecollectionLMP arisefromtheanteriorbranchofinternaliliacarteries. The uterinearteries US ofthegallbladder willhelpinmakingthediagnosis. US asaresultof inflammationofthelivercapsuleand diaphragm. manifestation ofPID extrapelvic also complainofpainintherightupper quadrantwherethereistenderness.Theconditionan Thesepatients,besideshavingtheusualclinicalfeaturesofPID, confused withacutecholecystitis. syndrome isinterestingasitcanbe Whilst alltheaboveneedadmission, Fitz-Hugh–Curtis long and 5 cm wide. The lower limitofthemasscannotbefelt. long and5cmwide.Thelower afirmmassinthesuprapubicareaabout8cm reveals she lookspale;abdominalexamination Onexamination ofmicturition,constipationandbackache. periods,frequency She hasheavy months’duration. ofseveral abdominaldiscomfort womancomplainsoflower A 40-year-old has generalisedabdominaldistensionandasciteswithshiftingdullnessfluidthrill. she ofmicturition.Onexamination breath. Recentlyshehasbeenconstipatedwithfrequency of twosizeslarger. andshortness skirts Shehassomegeneralisedabdominaldiscomfort the last6months.Recentlyshefoundthatherclothesaregettingtightandhastowear over gaininweightandincreasehergirth womancomplainsofunexplained A 60-year-old y andph Neisseria gonorrhoeae is the commonest organism responsible for PID. Probably the next Probablythenext is thecommonestorganismresponsibleforPID. y y siolog y disease y . ➜ 6B 5E 4F 3C 2A 1D questions matching Extended Answers:

1. G treatment istheninstituted. wouldconfirmthe diagnosisandappropriate arising fromthepelviswhichisfibroid.AnUS ofmicturitionandconstipation.Shehasamidlinemass bladder andrectumcausingfrequency periodsandsymptomsfrompressuretoneighbouringstructures–urinary This womanhasheavy done tolookforpleuraleffusion.Afterstaging,sheistreatedappropriately. scan.AchestX-rayis andCT proven.BloodshouldbesentforCA-125 byUS followed otherwise ofbreath,probablyfromherascites.Shehasthefeaturesanovariancarcinomaunless is short gradualabdominaldistensionduetoascites.She yearsold,hasdeveloped This woman,whois60 condition confirmedbylaparoscopy. onempiricalgrounds,avaginalswabsent,generalbloodtestsdone andthe should bestarted Treatment initselfcancausePID. andabdominaltenderness.PresenceofanIUCD pain, pyrexia abdominal This womanhasfeaturesofpelvicinfection–foul-smellingvaginaldischarge,lower laparoscopy. Diagnosisisconfirmedby syndromecouldbeentertained. diagnosis ofirritablebowel pouch ofDouglas.Shehasthefeaturesendometriosis,althoughsometimesadifferential tendernessinthevaginalfornices.Therearenodulesfelt showing internal examination abdominalsymptomsofdeeppainand This womanwithdysmenorrhoeahaslower permits. definitive procedurecanbecarriedoutbythelaparoscopicrouteifexpertise prior tolaparotomyorthe and operateduponasanemergency. Alaparoscopymaybeperformed andat35yearsofage,itisherfirstpregnancy. Sheshouldbevigorouslyresuscitated her LMP abdominalpainandsignsofperitonisminthepelvis.Sheisunsureabout sudden onsetoflower –featuresofhypovolaemicshock, This womanhasthehallmarksofarupturedectopicpregnancy and/orlaparoscopy. laparotomy.US byanemergency Thisshouldbefollowed Confirmationisby atwistedovariancyst. and hasnoconstitutionalsymptoms.Shedeveloped with clinicalfeaturesofperitonism–tenderness,rigidityandreboundtenderness.Sheisapyrexial abdominalpain lower withsuddenonsetofsevere This youngwomanisasurgicalemergency y naecological patholog y 557

76: GYNAECOLOGY This page intentionally left blank PART 13 Transplantation

77 Transplantation 561 This page intentionally left blank 77 Transplantation

Multiple choice questions ➜ Transplantation terms 4. Which of the following statements 1. Which of the following definitions are regarding hyperacute rejection are correct? true? A Allograft means an organ or tissue A It is due to preformed anti-HLA transplanted from one individual to antibodies. another. B It occurs 2–4 weeks after transplant. B Xenograft refers to a cadaveric donor. C It can also occur due to ABO blood C Autograft refers to transplants within the group graft incompatibility. same species. D The liver is resistant to hyperacute D Orthotopic graft refers to a bone graft. rejection. E A heterotopic graft is placed in a site E The antibodies are mainly towards HLA different from where the organ is class 2 antigens. normally located. 5. Which of the following statements ➜ regarding acute rejection are true? Human leucocyte antigen A It occurs within a month after transplant. 2. Which of the following statements B It is usually reversible. regarding human leucocyte antigen C It is predominantly mediated by T cells. (HLA) are true? D A characteristic finding is mononuclear A They are highly monomorphic. infiltration of the graft. B HLA class 2 antigens are present in all E Antibodies have no role. nucleated cells. C HLA antigens present on graft cells 6. Which of the following statements activate T-cells. regarding chronic rejection are true? D HLA-A and B (class 1) and HLA-DR A It usually occurs several years after the (class 2) are the most important in transplant. transplantation. B It is a rare cause of rejection. E Anti-HLA antibodies may cause C It is characterised by myointimal hyperacute rejection. proliferation in graft arteries. D Immunosuppressive therapy is not ➜ Graft rejection helpful. The liver is more resistant to this type of 3. Which of the following statements E rejection. regarding graft rejection are true? A Hyperacute rejection is characterised by ➜ Graft-versus-host disease intravascular thrombosis. B Acute rejection is T-cell-dependent. 7. Which of the following statements C Chronic rejection usually happens after regarding graft-versus-host disease a period of 6 months post-transplant. (GVHD) are true? D Acute rejection is irreversible. A It is due to the donor lymphocytes E Acute rejection is the most common reacting against the host antigens. cause of graft failure.

561 TRANSPLANTATION ➜ ➜ 10. Which of the following statements Whichofthefollowing 10. statements Whichofthefollowing 8. 9. Which of the following statements Which ofthefollowing 9. 562

Brainstem death Immunosuppressive therap E D C B A E D C B A E D C B A E D C B preclude brainstemdeath. doesnot The presenceofspinalreflexes in theUK. Electrophysiological testsaremandatory two consultantgradedoctors. The diagnosishastobeconfirmedby Hypothermia shouldberuledout. present. Traumatic shouldnotbe headinjury isarecognisedentity.disorder (PTLD) Post-transplant lymphoproliferative squamous canceroftheskin. There isincreasedincidenceof Chemoprophylaxis hasnorole. problem. isamajor (CMV) Cytomegalovirus first monthaftertransplantation. Risk ofviralinfectionishighestinthe immunosuppression aretrue? of regarding complications ofT-cells.depletion andblockade monoclonalantibodycauses OKT3 transcription. Tacrolimus gene blocksIL-2 transduction. receptorsignal blocksIL-2 Azathioprine effects. inflammatory causewidespreadanti- Corticosteroids transcription. gene Cyclosporin blocksIL-2 transplant. It commonlyoccursafterrenal gastrointestinal tract(GIT). It canalsoinvolvetheliverand isaminorcondition. GVHD rash overthepalmsandsoles. It frequentlyinvolvestheskinwitha regarding brainstem deatharetrue? immunosuppressive therapy aretrue? regarding modeofaction y ➜ ➜ ➜ 12. Which of the following statements Whichofthefollowing 12. statements Whichofthefollowing 11. 13. Which of the following statementsare Whichofthefollowing 13.

Transplantation ingeneral Renal transplantation Organ donation E D C B A E D C B A E D C B A regarding organdonationaretrue? true? true? regarding renaltransplantation are The optimalcoldstoragetimeforthe is8h. for thekidneys maximumcoldstoragetime The safe between 0and4ºC. organsarepreserved The retrieved donationis65years. heart The usualacceptableupperagelimitfor contraindication fordonation. areanabsolute system(CNS) nervous tumoursofthecentral Primary pancreas transplantationalone. hasbetterresultsthan transplant (SPKT) Simultaneous pancreasandkidney pancreas isthepreferredtechnique. Bladder drainageoftheexocrine indication forlivertransplant. isnotan hepaticmalignancy Primary transplant. small-bowel problemafter isaparticular GVHD is 70 percentat5years. transplant afterheart The graftsurvival is about75percentat5years. afteracadaverictransplant Graft survival cadaveric transplants. after livingdonortransplantthan Delayed graftfunctionismorecommon asymptomatic. Peritransplant lymphocelesareusually for transplantationis65years. The upperagelimittobeconsidered transplants inUK. about 25–30percentofallrenal Living donortransplantsaccountfor islessthan3h. heart ➜ ➜ ➜ Extended matchingquestions Extended Chooseandmatchthecorrectdiagnosiswitheachofscenariosgivenbelow: Chooseandmatchthecorrectdrugwiththeirmechanismofactionbelow: Chooseandmatchthecorrectdiagnosiswitheachofdescriptionsgivenbelow:

1 E D C B A 3. Post-transplantationcomplications 6 5 4 3 2 1 2. Mechanismofimmunosuppressive agents 4 3 2 1 1. T F E D C B A D C B A painful. Blood culture and sputum culture show organisms withcharacteristiccolonies. painful. Bloodcultureandsputum show responding toantibacterialmedications. Hehasalsobeenhavingcoughandfindingswallowing not fever malewhohadaliver transplant3monthsagopresentswithsevere A 50-year-old Squamous cellcarcinoma PTLD Fungal infection carinii Pneumocystis infection CMV receptorsignaltransduction Blocks IL-2 genetranscription Blocks IL-2 oflymphocytes Depletion andblockade effects Widespread anti-inflammatory proliferation lymphocyte Prevents ofT-cellsDepletion andblockade characteristic rashonpalmsandsoles.Thisisseriousitcanbefatal. transplants.Itfrequentlyinvolvestheskin,causinga This isseenafterliverandsmall-bowel leading toischaemiaandfibrosis. This isthemostcommoncauseofgraftfailureandcharacterisedbymyointimalinfiltration, This isT-cell-dependent andischaracterisedbymononuclearcellinfiltration.Itreversible. thrombosis. antibodies.Itischaracterisedbyintravascular orpreformedanti-HLA This isduetoABO Azathioprine globulin(ALG) Antilymphocytic serum(ALS) /antilymphocytic OKT3 Sirolimus /everolimus Cyclosporin / tacrolimus Corticosteroids Acute rejection Graft-versus-host response Hyperacute rejection Chronic rejection y pes ofallograftreactions infection 563

77: TRANSPLANTATION TRANSPLANTATION ➜ ➜ ➜ 6.C,D, E 5.B,C,D 4.A,C,D 3.A,B,C 2.C,D,E 1. A,E Answers: Multiplechoicequestions 564

Graft rejection Human leucoc Transplantation terms 5 4 3 2 liver appearstobe moreresistantthanothersolidorgans tothedestructiveeffectsofchronic This isthemostcommon causeofgraftfailureandusually occursafterthefirst6months. The byadditionalimmunosuppressivetherapy.Most episodesofacuterejectioncan bereversed role. butalloantibodiesmayalsoplayanimportant predominantly mediatedbyT-lymphocytes This usuallyoccurswithinthefirst6 monthsoftransplantationbutmayoccurlater. Itis antigens. Livertransplantsrarelyundergohyperacuterejection. a cross-matchtestonrecipientserumtoensurethatthereareantibodies againstthedonorHLA groupcompatibility and byperforming minutes andhours.ItcanbeavoidedbyensuringABO class 1antigens.Thisischaracterisedbyintravascularthrombosisand graft destructionwithin orpreformedantibodies againstHLA Hyperacute occursimmediatelyandismediatedbyABO playanessential roleinmediatingrejection. immunosuppressive therapyisgiven.Tlymphocytes immuneresponsethatresultsinrapidgraftrejection unless Allografts provokeapowerful as dendriticcells,macrophagesandB-lymphocytes. morestronglyonantigen-presentingcells,such cells, whereastheclass2antigensareexpressed molecules:class1and2.Theantigensarepresentonallnucleated two typesofHLA molecules.Thereare arehighlypolymorphiccellsurface role asantigenrecognitionunits.They oftheirspecialphysiological antigensarestrongtransplantbyvirtue Human leucocyte graftisaplacedinitsnormalanatomicalsite. individual.Orthotopic recipient arethesame betweendifferentspecies.Inautografts,thedonorand Xenograft referstoagraftperformed enlarged tonsilsandatenderspleen. an ‘infectiousmononucleosis’typeillness.Sheisfoundtohavemultiplelymphadenopathy, A 10-year-old yearsago,presentswith ofrenaltransplant,afew history girlwithaprevious to haveleucopenia. and lethargy. Hehascough,rightupperquadrantpainandsomeocularsymptoms. Heisfound A 38-year-oldmalewhohadarenaltransplant4monthsagopresentswithhighswingingfever dressings. ulcerated withsomebleedinganddischarge.Ithasnotrespondedtotheusualwoundcare lump onthedorsumofhishandforpast6months.Itispainlessbuthasrecentlybecome farmerwhohadarenaltransplant15A 60-year-old yearsagopresentswitharapidlygrowing ontrimethoprimwithgoodresponse. She isstarted and persistentcough.Thediagnosisisconfirmedonbronchoalveolarlavagelungbiopsies. chestpain femalewhohadarenaltransplant4monthsagopresentswithfever, A 40-year-old y te antigen ➜ ➜ ➜ 10. B,E 9.B,D,E 8.A,B,D,E 7. A,B,D

Brainstem death Immunosuppressive therap Graft-versus-host disease raised bloodlipidsandinadequateimmunosuppression. infection, (CMV) match,longcoldischaemiatime,cytomegalovirus of acuterejection,poorHLA episodes involved. Theriskfactorsforchronicrejectionafterrenaltransplantationareprevious mechanisms areimmunological,andbothalloantibodiescellulareffector ofchronicrejectionisnotcompletelyunderstood.Theunderlying rejection. Thepathophysiology electrophysiological or brain perfusion studies. electrophysiological orbrainperfusion thereisnoneedto perform motor responseand absenceofspontaneousrespiration. IntheUK absence of reflexes, determine brainstem deathaimtoconfirmtheabsence ofcranialnerve should beaconsultantandneither should beconnectedtothetransplantteam.Thetests inthisarea.Atleastoneofthem on twoseparateoccasionsbyclinicians experienced guidelinesstate that theteststoconfirmdiagnosisshouldbeperformed is made.TheUK beforethediagnosisofbrainstemdeath hormonal conditions,anddrugsshould beexcluded hypothermia,profoundhypotension,metabolicand to exclude brainstem death.Itisimportant Traumatic andsuddenintracranialhaemorrhageare themostcommoncausesof headinjury andin issues oforgantransplantation. withnoprospectofrecovery brain damageonlifesupport inmanagement ofpatientswithirreversible of thepatient.Theconceptbraindeathisimportant accepted thattheconditionofbraindeathequatesinmedical,legaland religioustermswithdeath capacityforbreathing.Inmost countriesitis consciousness combinedwiththeirreversible lossofthe capacity for causesirreversible braininjury Brainstem deathoccurswhensevere percent. 50 mononu rate ofupto isaseriousconditionwithmortality cleosis typeofillness.PTLD usually inresponsetoEpstein–Barrvirusinfection.Thiscanpresentasaninfectious infections. to andherpeszoster. herpessimplex Protozoalinfectionsdue viralinfectionsincludeCMV, important Pre-transplant vaccinationagainstcommunity-acquiredinfectionshouldbeconsidered.The the first6monthsoftransplantationandriskbacterialinfectionishighestinmonth. inhigh-riskpatients.Theriskofviralinfectionishighestduring Chemoprophylaxis isimportant Transplant infections,especiallybyviruses. recipientsareathighriskofopportunistic Sirolimus oflymphocytes. and blockade receptorsignaltransduction. blocksIL-2 /everolmus during theearlypost-transplantperiod.Polyclonal antibodypreparationsALG /ALS causedepletion (CD25)areusedtoaugmenttheeffectsofcalcineurinblockade receptorsonT-lymphocytes IL-2 proliferation.Monoclonal antibodiesdirectedagainst lymphocyte prevents of agents.Azathioprine act atdifferentsitesduringT-cell activation.Mostimmunosuppressiveprotocolsuseacombination rejectionactpredominantlyonT-cells,The agentsusedtoprevent anddifferentclassesofagents serious andsometimesfatalcomplication. antigens.Thisfrequentlyinvolvestheskin,liverandGIT. isa which reactagainsthostHLA GVHD bothcontainlargenumbersofimmunocompetentlymphocytes The donorliverandsmallbowel This isthereciprocalproblemofanimmunologicalreactionmountedbygraftagainsthost. Pneumocystis carinii Pneumocystis Post-transplant lymphoproliferative disorder is an abnormal proliferation of B-lymphocytes, Post-transplant lymphoproliferativedisorderisanabnormalproliferationofB-lymphocytes, and fungalinfectionsdueto y Candida and Aspergillus are other important areotherimportant 565

77: TRANSPLANTATION TRANSPLANTATION ➜ ➜ ➜ ➜ 4C 3A 2D 1B matchingquestions Answers: Extended 13. A,B,E 12. A,C,E 11. B,C,E 566

1. T Transplantation ingeneral Renal transplantation Organ donation Most of the organs used for transplantation are obtained from brainstem-dead, heart-beating Most oftheorgansusedfortransplantationareobtainedfrombrainstem-dead,heart-beating This isthereciprocal problemofanimmunologicalreaction mountedbythegraftagainst thehost. inadequate immunosuppression. infection,raisedbloodlipidsand match,long coldischaemiatime,CMV rejection, poorHLA episodesofacute The riskfactorsforchronicrejection afterrenaltransplantationareprevious byadditionalimmunosuppressivetherapy.Most episodesofacuterejectioncan bereversed hours. antigens. Thisischaracterisedbyintravascularthrombosisandgraftdestruction withinminutesand class1 orpreformedantibodiesagainst HLA This occursimmediatelyandismediatedbyABO urethritis leaks, cystitis, andrefluxpancreatitis. /urethral stricture,UTI complicationssuchasanastomotic preferred becausebladderdrainageisassociatedwithseveral hepaticmalignancy. Entericdrainageofthepancreasis metabolic liverdiseaseandprimary respectively. Theindicationsforlivertransplantationarecirrhosis,acutefulminantdisease, transplantationarearound85and70 percent, ratesafterheart The 1-and5-yeargraftsurvival livingdonortransplantation. uncommon (<5percent)following deceaseddonorsbutis of acutetubularnecrosisandoccursinupto30percentheart-beating obstruction andoedemaoftheipsilateralleg. of cases.Peritransplant lymphocelesareusuallyasymptomaticbutmayoccasionallycauseureteric which usuallyoccursyearsafterthetransplant.Urologicalcomplicationsoccurinabout5percent stenosis(upto10(1 percent),renalveinthrombosis(upto5cent)andartery percent) transplants, superiorresultsandthelegislation.Thereisnoupperagelimitforarenaldonor. ofdeceaseddonor India. Thejustificationforlivingdonorrenaltransplantisbasedontheshortage The livingdonorrenaltransplantactivityismuchhigherinsomecountries,suchasScandinaviaand around80–100In theUK, end-stagerenaldisease. peoplepermillionofthepopulationdevelop 48, 24,6and8hrespectively. isusuallyhalfofthis. Theoptimalcoldstoragetime,however, andlungare liver,heart maximumstoragetimesforkidney, (UW) solution,areavailable.Thesafe solutions,suchasUniversityofWisconsin yearsforpancreas.Various organpreservation it is60 andlungdonorsis65years andliverdonors.Theupperagelimitforheart limit forkidney Thereisnoupperage non-melanotic skintumoursandcarcinomainsituoftheuterinecervix. tumoursoftheCNS, ofprimary the past5yearsisanabsolutecontraindication,withexception in donors, andinthemajorityofcasesmultipleorgansareprocured.Thepresencemalignancy Delayed graftfunctionisdefinedastheneedfordialysispost-transplantation.Thisaresult thrombosis Vascular complicationsaftertransplantareuncommonandincluderenalartery y pes ofallograftreactions ➜ ➜ 5D 4A 3E 2B 1C 1D,2F,3A,4E,5B,6C

3. Post-transplantationcomplications 2. Mechanismofimmunosuppressive agents children. andlivertransplantsisconsiderablyhigherin This happensinaround1–3percentofkidney disease, hepatitis,retinitisandencephalitis. picture dependsontheorgansystemaffectedandmaypresentaspneumonia,gastrointestinal andleucopenia.Theclinical lethargy typicallypresentswithahighswingingfever, Cytomegalovirus within20yearsaftertransplant. askinmalignancy transplant patientswilldevelop percentofthe tosunlight.Ithasbeenpredictedthat50 increases withageandexposure cellcarcinomaandmelanoma.Theriskofskincanceraftertransplantation frequently arebasal This isthemostcommonskincanceraftertransplantation.Theothercancersseenless 6 monthsaftertransplant. iseffectiveandcontinuedforupto months afterthetransplant.Prophylaxiswithco-trimoxazole protozoalinfectionaftertransplantation.Itoccursinthefirstfew This isthemostimportant aggressive treatmentareessentialtoavoidfatalinfection. Fungal infectionusuallyoccursinthefirst3monthsaftertransplantation.Earlydiagnosisand 567

77: TRANSPLANTATION